Sei sulla pagina 1di 2356

Statics Worked Examples With Integrated Text and Graphics

Mark P. Rossow
Southern Illinois University Edwardsville
2009 Mark P. Rossow

Why Integrated Text and Graphics?


The traditional way to learn in a problem-solving course such
as statics is to solve a large number of homework problems.
This procedure is often inefficient, time-consuming and
frustrating because so much time is spent searching for the
solution that little time is left for learning the principles that
will enable a student to solve other related problems. Recent
educational research has suggested that a way to learn that is
superior to simply solving problems is to study worked
examples. However, the research has also shown that the
superiority of studying worked examples is maintained only
if the text and graphics appearing in the examples are
"integrated," by which is meant that the text on the page is
positioned immediately adjacent to the figure to which the
text refers. Additionally, liberal use is made of arrows
relating specific words or labels to appropriate features of the
figure, and the sequence of steps in the solution procedure is
identified by circled numbers.

The worked examples then follow the problem statements.


The solutions, as has already been mentioned, contain
integrated text and graphics, but in addition provide much
more detail and explanation than are found in examples in
most conventional textbooks.
The purpose of providing problem statements separated from
the solutions is to allow students, after they have studied
some of the examples in a particular topic section and wish
to test their understanding, to attempt to work other problems
without inadvertently seeing part of the solution. This
approach to learning is discussed further in the next section,
"How to Study Worked Examples."
A final note: although considerable effort has been expended
in checking and proofreading the examples presented herein,
in a work of this size undoubtedly some errors remain.
Readers who find errors are encouraged to report them to the
author at markrossow@gmail.com.

The present book, which is intended as a supplement to a


course in statics, contains 445 worked examples in which
text and graphics are integrated. The range of topics covered
by the examples is sufficiently broad to encompass most
statics courses. Each topic section, except the introductory
section, begins with a Procedures and Strategies discussion
that outlines techniques for problem solving. Next are given
problem statements but not the solutions for all the
examples for that topic.

ii

A Note to Students: How to Study Worked Examples


You might think of the present book as an unusually detailed
and easy-to-read solution manual designed for students, who
are learning the subject matter, rather than for faculty
members, who already know the subject matter and are only
looking for numerical details of particular solutions. The
book does indeed provide numerical details of solutions, but
more importantly it also provides the rationale behind the
solution steps so that you can understand the principles and
solution techniques in the field of statics.

that the examples embody. If you read but do not study, you
may fall into the trap that educational psychologists refer to
as the "illusion of understanding": you think that you
understand but in fact you do not. Worked examples have
the considerable advantage over the traditional approach of
working homework problems in that they can save you time
and frustration, but working homework problems has the
advantage that you get immediate feedback on whether or
not you are learning. That is, when you cannot solve a
problem, you know immediately that you may have failed to
learn something important (or you may have made a simple
calculation error). It is obvious that you have to study more
or seek assistance. To use worked examples as a substitute
for working homework problems, you must continually
challenge yourself (by, for example, working some problems
without looking at the solutions in advance) so that you get
feedback on whether or not you are learning.

Here is how to make the best use of the book: When you
begin a topic, for example, "equilibrium of a particle," read
the first several examples. Then, when you think that you
understand the solution procedure, go on to the later
examples and try to work them. Look at the book's solution
only when you encounter a difficulty. In this way, your
mind will be focused on a specific question that you want
answered, and you will be motivated to study the example
solution much more closely than if you were simply to read
the example without first having attempted to work it. Once
you understand the solution thoroughly, summarize to
yourself the general principle that the example illustrates.

By the way, to save time and avoid frustration when you are
trying to find your own solution to the worked examples, it is
essential that you use a scientific graphing calculator or,
better, a calculation program on a computer. Having such a
computational tool will allow you to concentrate on learning
principles of statics rather than spending time on mere
numerical manipulation. See the next section, Using
Scientific Graphing Calculators Effectively, for ideas on how
best to do your numerical work with such tools.

The way not to study worked examples is simply to read


them straight through without ever challenging yourself to
work an example on your own or without ever pausing to
think and crystallize in your mind the general principles

iii

A Second Note: Using Scientific Graphing Calculators Effectively


The solutions given in the present book contain detailed
numerical calculations that make it clear how the final
answers are obtained. However, when you are working an
example by yourself without looking at the solution, as
explained previously in the Section "How to Study Worked
Examples" having to deal with numerical details is a
distraction that may prevent you from seeing and learning the
underlying principle that the example is meant to illustrate.
To avoid this danger, you are strongly encouraged to use a
scientific graphing calculator (or a calculation program such
as Mathcad, Matlab, Maple, Mathematica, TK Solver, etc.).
But you are not only encouraged to use one of these
calculation tools, but also to use it effectively that is, in a
manner specifically designed to minimize errors, maximize
speed, and provide confidence in the final answer. Because
the author has encountered few students even very able
ones who have taught themselves to operate a calculator in
the manner just described, guidelines are given below for
using calculators and calculation programs effectively, so
that you can concentrate on learning concepts rather than
spending time puzzling over how a particular number
appearing in an example is calculated.

For example, if you have an equation such as the following


to solve,
14.78 + 5X + 6(3.42) + 2.23(X

4) = 0,

do not rearrange this equation on paper and solve for X the


way you learned in algebra class. Instead, enter the equation
directly in your calculator's solver, and let the calculator do
the algebra.

Guideline 2. Check for errors by proofreading rather than


by repeating the calculation.
If Guideline 1 has been followed, then the equation on paper
can be compared character by character with the equation
appearing in the calculator display or on the computer screen.
Error checking by means of proofreading is preferable to
error checking by repeating the calculation because repeating
the calculation may introduce new errors, but proofreading
never does.

Guideline 3. Data that appear in several equations should be


entered only once.

Guideline 1. Enter information in the order in which it


appears in the original problem formulation.

For example, if the value of a force, 3.491 N, appears more


than once in equations or expressions to be entered into the

iv

Using Scientific Graphing Calculators Effectively (Continued)


(81*37.2/16)/(( /2)*(20^4+(9.87/12)^4)).

calculator, then the number should first be stored under the


name of a variable, and that name should be used in all terms
subsequently entered into the calculator.

This is a very error-prone operation, since many parentheses


must be inserted that are not present or are not easily
identified with their position in the original expression. It is
much better to introduce intermediate variables. For
example, define X and Y as follows:

Guideline 4. Store intermediate results in the calculator


rather than writing them on paper.
Just as in Guideline 3, storing intermediate results under the
name of a variable in the calculator decreases the number of
possibilities for errors.

X
(81)(37.2)
16
9.87 4
4
2 [20 + ( 12 ) ]

Guideline 5. Use cut-and-paste rather than re-type.

That is,

Again, the number of possibilities for errors is reduced.

Guideline 6. Introduce intermediate variables to simplify


writing multilevel mathematical expressions.

X = 81*37.2/16

(1)

Y = ( /2)*(20^4+(9.87/12)^4).

(2)

and
Consider this multilevel mathematical expression:
(81)(37.2)
16
9.87 4
4
2 [20 + ( 12 ) ]

Eqs. 1 and 2 can be entered in the calculator, and then the


original multilevel expression can be entered in the form
X/Y,

Entering this expression as a single line of characters would


produce

which is much less prone to error.

Using Scientific Graphing Calculators Effectively (Continued)


Keystrokes
3 <Enter>
2 <Enter>
+ <Enter>
6 <Enter>
* <Enter>

When should intermediate variables be used? A rule of


thumb is to introduce intermediate variables if parentheses
have to be nested three (or more) deep when the expression is
written on one line

Guideline 7. Know thoroughly how to use the solver.

Calculator Display
3
2
5
6
30

Note that the parentheses do not have to be entered, and thus


the user saves two keystrokes, compared to what would have
been required had RPN not been used. RPN is a standard
feature on some types of calculators and has gained wide
acceptance. However, because the complete arithmetic
expression never is displayed on the screen when RPN is
used, the calculator user cannot easily check for errors by
proofreading. Instead the user must check the calculation by
re-entering the data, thus doubling the number of keystrokes
required to obtain a verified answer, and RPN ends up taking
more keystrokes than a conventional left-to-right data-entry
scheme. Furthermore, if the user makes an error in entering
data during the repeat calculation, then yet another repeat
calculation must be performed and the number of keystrokes
is tripled. The more keystrokes, the more opportunity for
making an error.

As was pointed out in the discussion of Guideline 1, the


solver is the key to avoiding errors in algebraic manipulation.

Guideline 8. Use built-in functions whenever possible.


For example, definite integrals should always be evaluated
with the calculator's built-in integration function rather than
by finding the antiderivative and substituting the values of
the limits. Vector operations such as dot product, cross
product, magnitude (norm), and finding a unit vector should
all be done using built-in functions.

Guideline 9. Do not use Reverse Polish Notation.


Reverse Polish Notation (RPN) is an ingenious technique for
reducing the number of keystrokes needed to perform certain
arithmetic operations. As an example, consider the
evaluation of the expression 6(3 + 2) using RPN:

vi

Using Scientific Graphing Calculators Effectively (Continued)


Guideline 10. Violate any of the other guidelines when
common sense says to do so.
Guidelines 1 to 9 should be followed in the large majority of
cases, but special situations will arise in which the sensible
thing to do is to violate the guidelines. That is why the word
"guideline" has been used rather than "law.

vii

Acknowledgments
The examples and drawings in this text were developed with
the support of Southern Illinois University Edwardsville
Excellence in Undergraduate Education Grants Nos. 05-25
and 09-29 (Mark P. Rossow, Principal Investigator) and
through the hard work and dedication of the following
people:
Jason Anderson
Paul Cayo
Madan Gyanwali
Tyler Hermann
Vishnu Kesaraju
Jennie Moidel
Binod Neupane
Sanjib Neupane
Ramesh Regmi
Laxman Shrestha
Shikhar Shrestha
Sagun Thapa

viii

Contents
1. Preliminaries: Units and Rounding ............................................................................... 1
2. Force and Position Vectors .......................................................................................... 22
2.1 Adding Forces by the Parallelogram Law ......................................................... 23
2.2 Rectangular Components in Two-Dimensional Force Systems ........................ 60
2.3 Rectangular Components in Three-Dimensional Force Systems .................... 113
2.4 Position Vectors. Use in Defining Force Vectors. ........................................ 146
2.5 Applications of Dot Products .......................................................................... 200
3. Equilibrium of a Particle ............................................................................................ 239
3.1 Particles and Two-Dimensional Force Systems ............................................... 240
3.2 Particles and Three-Dimensional Force Systems ............................................. 261
4. Moments and Resultants of Forces Systems .............................................................. 306
4.1 Moments in Two-Dimensional Force Systems ................................................ 307
4.2 Moments in Three-Dimensional Force Systems .............................................. 340
4.3 Moment of a Couple......................................................................................... 401
4.4 Moment of a Force About a Line. .................................................................. 444
4.5 Equivalent Force-Couple Systems ................................................................... 472
4.6 Distributed Loads on Beams ............................................................................ 556
5. Equilibrium of a Rigid Body ...................................................................................... 605
5.1 Constraints and Static Determinacy ................................................................. 606
5.2 Rigid Bodies and Two-Dimensional Force Systems ....................................... 688
5.3 Rigid Bodies and Three-Dimensional Force Systems ..................................... 731
6. Structural Applications .............................................................................................. 790
6.1 Frames and Machines ....................................................................................... 791
6.2 Trusses: Method of Joints and Zero-Force Members ...................................... 903
6.3 Trusses: Method of Sections ............................................................................ 960
6.4 Space Trusses. .............................................................................................. 1000
6.5 Cables: Concentrated Loads ........................................................................... 1035
6.6 Cables: Uniform Loads .................................................................................. 1082
6.7 Cables: Catenaries .......................................................................................... 1133

ix

7. Friction ..................................................................................................................... 1191


7.1 Basic Applications ......................................................................................... 1192
7.2 Wedges ........................................................................................................... 1250
7.3 Square-Threaded Screws ................................................................................ 1286
7.4 Flat Belts ........................................................................................................ 1347
7.5 Thrust Bearings and Disks ............................................................................. 1380
7.6 Journal Bearings ............................................................................................. 1402
7.7 Rolling Resistance.......................................................................................... 1450
8. Internal Forces .......................................................................................................... 1481
8.1 Internal Forces in Structural Members ........................................................... 1482
8.2 Shear and Bending-Moment Diagrams: Equation Form ................................ 1536
8.3 Shear and Bending-Moment Diagrams Constructed by Areas ...................... 1597
9. Centroids and Mass Centers ..................................................................................... 1670
9.1 Centroids by Integration ................................................................................. 1671
9.2 Centroids: Method of Composite Parts .......................................................... 1749
9.3 Theorems of Pappus and Guldinus ................................................................ 1845
9.4 Hydrostatic Pressure on Submerged Surfaces. ............................................. 1924
10. Inertia Properties of Plane Areas............................................................................ 1981
10.1 Moments of Inertia by Integration ............................................................... 1982
10.2 Method of Composite Areas ........................................................................ 2027
10.3 Products of Inertia ........................................................................................ 2067
10.4 Moments of Inertia About Inclined Axes; Principal Moments .................... 2115
11. Energy Methods ..................................................................................................... 2165
11.1 Virtual Work ................................................................................................ 2166
11.2 Potential Energy ........................................................................................... 2270
Appendix: Geometric Properties of Lines, Areas, and Solid Shapes ............................ 2338
Index .............................................................................................................................. 2343

1. Preliminaries: Units and Rounding

1. Preliminaries: Units and Rounding Problem Statement for Example 1


1. Round off the following numbers
to three significant figures.
a) 54.27 m
b) 2 927 124 m
c) 3.6143

10-3 in.

d) 6.875 km
e) 6.885 km

1. Preliminaries: Units and Rounding Problem Statement for Example 2


2. Express the following quantities in proper SI units.
a) 2491 N
b) 0.02491 N
c) 2 491 000 N
d) 0.0987

10-5 m

e) 0.0987

108 km

f) 0.000 987

10-2 mm

1. Preliminaries: Units and Rounding Problem Statement for Example 3


3. Evaluate the arithmetic expressions and express the
results to three significant figures and in proper SI
form.
a)

13 N
17 mm

b) 13 N
17 kg
c) 934.2 mm2

2m

d) 99.7 mN2

3.6 mm

e) 8 kN

f)

12.1 mm

43 ms
(82.1 ms)2

1. Preliminaries: Units and Rounding Problem Statement for Example 4


4. Express the following forces in pounds (lb).
a) 1,230 kip
b) 0.0230 kip
c) 23.0 oz

1. Preliminaries: Units and Rounding Problem Statement for Example 5


5. a) A 20-kg box is set upon a scale that displays
weight in SI force units. What weight will the scale
display?
b) A 20-lb box is set upon a scale that displays
weight in U.S. customary units. What weight will
the scale display?

1. Preliminaries: Units and Rounding Example 1, page 1 of 5


1. Round off the following numbers
to three significant figures.
a) 54.27 m
b) 2 927 124 m
10-3 in.

c) 3.6143

d) 6.875 km
e) 6.885 km
1 a) 54.27 lies between 54.20 and 54.30.

54.27
(midpoint
of interval)

54.20

54.30

3 significant figures

3 significant figures

Because 54.27 lies closer to 54.3, round to that value:


54.27 m

54.3 m

Ans.

1. Preliminaries: Units and Rounding Example 1, page 2 of 5


b) 2 927 124 lies between 2 920 000 and 2 930 000.

2 927 124
(midpoint
of interval)

2 920 000

2 930 000

3 significant figures

3 significant figures

Because 2 927 124 lies closer to 2 930 000, round to that


value:
2 927 124 m

2 930 000 m
= 2.93

Use engineering
notation in which the
significant figures are
followed by a factor of
10 raised to a power.

106 m

Ans.

6 To make it easy to select the


correct SI prefix, use an
exponent that is a multiple of 3.

1. Preliminaries: Units and Rounding Example 1, page 3 of 5


7

c) 3.6143

10-3 lies between 3.610

10-3 and 3.620

3.6143
3.610

10-3

10-3.

10-3
(midpoint of
interval)

3 significant figures

10-3 in.

10-3

3 significant figures

Round to the closer value:


3.6143

3.62

3.61

10-3 in.

Ans.

1. Preliminaries: Units and Rounding Example 1, page 4 of 5


9

d) 6.875 lies precisely halfway between 6.870 and 6.880.

6.875
(midpoint
of interval)

6.870
3 significant figures

10 The rule is "If the number lies at the midpoint of the interval,
then round off so that the last digit is even":
6.875 km

6.88 km

Ans.

11 Even digit (8)


12 The rationale for the rule is that over a long chain of
calculations, the rule will lead to rounding down approximately
the same number of times as rounding up, and, on balance, these
rounding errors will cancel.

10

6.880
3 significant figures

1. Preliminaries: Units and Rounding Example 1, page 5 of 5


13 e) 6.885 lies precisely halfway between 6.880 and 6.890.

6.885
(midpoint
of interval)

6.880
3 significant figures

6.890
3 significant figures

14 Round off so that the last digit is even:


6.885 km

6.88 km

Ans.

15 Even digit (8)

16 In this case (6.885), we rounded down to get an even digit; in


the previous case (6.875), we rounded up to get an even digit.
Round up
6.870

6.875
Odd digit (7)

Round down
6.880

6.885
Even digit (8)

11

6.890
Odd digit (9)

1. Preliminaries: Units and Rounding Example 2, page 1 of 4


2. Express the following quantities in proper SI units.
a) 2491 N
b) 0.02491 N
c) 2 491 000 N
d) 0.0987

10-5 m

e) 0.0987

108 km

f) 0.000 987

Try to keep the value of the


coefficient between 0.1 and 1000.
Here it is possible. In other cases,
such as 2222 mm4, it is not.

Choose the exponent of 10 to be a


multiple of 3, because SI prefixes
are defined for 103, 106, 109, ...,
and 10-3, 10-6, 10-9, ... .

-2

10 mm

103 N

a) 2491 N = 2.491

= 2.491 kN
4

Ans.
Prefix "k" = "kilo" = 103.

12

1. Preliminaries: Units and Rounding Example 2, page 2 of 4


6

Coefficient lies
between 0.1 and 1000.

7
5

Exponent of 10 is a multiple of 3.

-3

b) 0.024 91 N = 24.91

10 N

= 24.91 mN

Ans.
Prefix "m" = "milli" = 10-3.

10 Coefficient lies
between 0.1 and 1000.
11 Exponent of 10 is a multiple of 3.
9

c) 2 491 000 N = 2.491

= 2.491 MN

10 N

Ans.

12 Prefix "M" = "mega" = 106.

13

1. Preliminaries: Units and Rounding Example 2, page 3 of 4


13 d) 0.0987

10-5 m = 0.987

10-6 m

= 0.987 m

Ans.

14 Prefix " " = "micro" = 10-6

16 Convert to base unit (from km to m).

15 e) 0.0987

108 km = 0.0987

1011 m

= 0.0987
= 9.87
17 Coefficient lies
between 0.1 and 1000.

108(103) m
18 Exponent of 10 is a multiple of 3.

109 m

= 9.87 Gm

Ans.

19 Prefix "G" = "giga" = 109.

14

1. Preliminaries: Units and Rounding Example 2, page 4 of 4

21 Convert to base unit (from mm to m).

20 f) 0.000 987

10-2 mm = 0.000 987


= 9.87

10-6 (10-3) m

= 9.87

10-9 m

= 9.87 nm

10-2 (10-3) m
22 Exponent of 10 is a multiple of 3.

Ans.

23 Prefix "n" = "nano" = 10-9.

15

1. Preliminaries: Units and Rounding Example 3, page 1 of 4


3. Evaluate the arithmetic expressions and express the
results to three significant figures and in proper SI
form.
a)

13 N
17 mm

b) 13 N
17 kg
c) 934.2 mm2
2

d) 99.7 mN
e) 8 kN

f)

2m
3.6 mm

12.1 mm

43 ms
(82.1 ms)2

2 Always express the denominator in base units


(In statics, the base units are m, s, and kg).
1

a)

13 N
=
17 mm

13 N
17(10-3) m

3
= 13(10 ) N
17 m
= (13/17)(103) N/m

= 0.765 (103) N/m


= 0.765 kN/m

16

Ans.

1. Preliminaries: Units and Rounding Example 3, page 2 of 4


4 "kg" is a base unit and so may appear in the
denominator.

b) 13 N
17 kg

= (13/17) N/kg
= 0.765 N/kg

Ans.

c) 934.2 mm2

2 m = 934.2 (10-3 m)2


= (934.2

The exponent applies to the


prefix "milli" as well as to
the base unit "meters".

2m

2)(10-6 m2) m

= (1868.4)(10-6 m3)
= (1.87
= 1.87
7

In this example, it is not possible to make


the coefficient lie between 0.1 and 1000
by choosing an SI prefix.

17

103)(10-6 m3)
10-3 m3

Ans.

1. Preliminaries: Units and Rounding Example 3, page 3 of 4


9

d) 99.7 mN2

3.6 mm = 99.7(10-3 N)2


= (99.7

The exponent applies to the


prefix "milli" as well as to
the base unit "newtons".
3.6(10-3 m)

3.6)(10-6 N2)(10-3 m)

= (358.92)(10-6 N2)(10-3 m)
= (359)(N210-9 m)
= 359 N2nm

10 The raised dot indicates


the product of two units.

11 Prefix "n" = "nano" = 10-9

12 e) 8 kN

12.1 mm = 8(103 N)
= (8

Ans.

12.1(10-3 m)

12.1)(103-3Nm)

= 96.8 Nm

Ans.

18

1. Preliminaries: Units and Rounding Example 3, page 4 of 4

15 Raised dot means


"product of meters and
seconds."
13 f)

14 No raised dot so "ms"


means "milli-seconds."

43 ms
=
ms)2
=

43 ms
10-3 s)2
ms
(10-6 s2

43

= 6.38

10-3(106) m/s

= 6.38 km/s

Ans.

19

1. Preliminaries: Units and Rounding Example 4, page 1 of 1


4. Express the following forces in pounds (lb).
a) 1,230 kip
b) 0.0230 kip
c) 23.0 oz

2 "kip" = "kilo-pound" = 1,000 lb


1

a) 1,230 kip = 1,230


= 1.23

1,000 lb
106 lb

b) 0.0230 kip = 0.0230

1,000 lb

= 23.0 lb

c) 23.0 oz = 23.0 oz

Ans.

Ans.

1 lb/16 oz)

= 1.4375 lb
= 1.44 lb

Ans.

20

1. Preliminaries: Units and Rounding Example 5, page 1 of 1


5. a) A 20-kg box is set upon a scale that displays
weight in SI force units. What weight will the scale
display?
b) A 20-lb box is set upon a scale that displays
weight in U.S. customary units. What weight will
the scale display?

a) Weight = mass

acceleration due to gravity

= (20 kg)

(9.81 m/s2)

= 196.2 kgm/s2
2
= 196.2 N

b) Weight = 20 lb

1 kgm/s2

1N

Ans.

Ans.

("lb" is a force unit; no factor corresponding


to the acceleration of gravity is needed.)

21

2. Force and Position Vectors

22

2.1 Adding Forces by the Parallelogram Law

23

2.1 Adding Forces by the Parallelogram Law: Procedures and Strategies, page 1 of 2
Procedures and Strategies for Solving Problems Involving
Addition of Forces by the Parallelogram Law

To add two force vectors,


1. make a sketch showing the vectors placed tail-to-tail;

2. construct a parallelogram, using the vectors as two of the


sides;

3. draw the diagonal that goes from the tail-tail vertex to the
opposite vertex (This is the resultant the vector sum of
the two forces); and

A
B

4. use the sine and cosine laws and geometrical relations


between angles to calculate the magnitude and direction of
the resultant.

A
A+B
B
Law of sines
a

C
b

c
A

24

sin c
sin b
sin a
=
=
C
B
A
Law of cosines
C2 = A2 + B2 - 2AB cos c

2.1 Adding Forces by the Parallelogram Law: Procedures and Strategies, page 2 of 2
To resolve a given force into components in two given
directions,

1. make a sketch showing the tail of the force vector at the


intersection of two lines in the given directions;

2. construct a parallelogram with the force vector as a


diagonal and the sides parallel to the given directions;
and

3. use the sine and cosine laws and geometrical relations


between angles to calculate the lengths of the sides of the
parallelogram. The sides are the components of the
force vector.

Fu
v

25

Fv

2.1 Adding Forces by the Parallelogram Law Problem Statement for Example 1
1. Determine the magnitude and direction
of the resultant of the forces shown.
150 N

30
200 N

26

2.1 Adding Forces by the Parallelogram Law Problem Statement for Example 2
2. Determine the magnitude and direction
of the resultant of the forces shown.
y

x
10
3 kN

2 kN
20

27

2.1 Adding Forces by the Parallelogram Law Problem Statement for Example 3
3. Determine the magnitude and direction of the resultant force.
100 N

110

40

80 N

28

2.1 Adding Forces by the Parallelogram Law Problem Statement for Example 4
4. The resultant of the two forces acting on the screw eye is known to
be vertical. Determine the angle and the magnitude of the resultant.
y

x
30
40 lb

60 lb

29

2.1 Adding Forces by the Parallelogram Law Problem Statement for Example 5
5. Determine the magnitude F and the angle , if the
resultant of the two forces acting on the block is to be a
horizontal 80-N force directed to the right.
50 N

F
25

30

2.1 Adding Forces by the Parallelogram Law Problem Statement for Example 6
6. To support the 2-kg flower pot shown, the resultant of the two
wires must point upwards and be equal in magnitude to the
weight of the flower pot. Determine the angles and , if the
forces in the wires are known to be 25 N and 30 N.

A
C
25 N

30 N
B

31

2.1 Adding Forces by the Parallelogram Law Problem Statement for Example 7
7. Resolve the 120-lb force into components
acting in the u and v directions.
120 lb

25
u
40

32

2.1 Adding Forces by the Parallelogram Law Problem Statement for Example 8
8. Resolve the 4-kN
horizontal force into
components along truss
members AB and AC.

4 kN

12
5
35
C

33

2.1 Adding Forces by the Parallelogram Law Problem Statement for Example 9
9. Find two forces, one acting along rod AB and one
along rod CB, which when added, are equivalent to the
200-N vertical force.
200 N

30

40
C

34

2.1 Adding Forces by the Parallelogram Law Example 1, page 1 of 4


1. Determine the magnitude and direction
of the resultant of the forces shown.
150 N

30
200 N

1 Construct a parallelogram by drawing two


lines. Each line starts at the tip of one vector
and is parallel to the other vector.

Tip

150 N
Parallel
30

30
200 N

35

Tip

2.1 Adding Forces by the Parallelogram Law Example 1, page 2 of 4


2

Since opposite sides of a parallelogram are


equal in length, the length of each line
represents the magnitude of the vector opposite.

200 N
150 N

150 N
30

30
200 N

200 N
150 N

150 N

30
200 N
Tails
3 The resultant R is drawn from the tails of the
vectors to the opposite vertex of the parallelogram.

36

2.1 Adding Forces by the Parallelogram Law Example 1, page 3 of 4


4 This is not the resultant because it is not
drawn from the intersection of the tails.

200 N

Heads

150 N
150 N

200 N
Tails

Head

200 N
150 N

150 N

150 N

30

30
200 N

200 N

5 To calculate the magnitude and direction of R, consider


the triangle formed by one half of the parallelogram.

37

2.1 Adding Forces by the Parallelogram Law Example 1, page 4 of 4


Trigonometric formulas for a
general triangle are given below.

150 N

a
30
200 N
C

6 Use trigonometry to calculate the magnitude and direction


of the resultant.
R2 = (200 N)2 + (150 N)2

2(200 N)(150 N) cos 30


b

The result is
A
R = 102.66 N

Law of cosines

Ans.

C2 = A2 + B2

sin
= sin 30
R
150 N
= 102.66 N
Solving gives
= 46.9

2AB cos c

Law of sines
sin a
sin b
sin c
=
=
A
B
C

Ans.

38

2.1 Adding Forces by the Parallelogram Law Example 2, page 1 of 3


2. Determine the magnitude and direction
of the resultant of the forces shown.
y

x
10
3 kN

2 kN
20

1 Construct a parallelogram by drawing


two lines parallel to the forces.

y
x

10
3 kN

2 kN

2 kN

3 kN

39

20

2.1 Adding Forces by the Parallelogram Law Example 2, page 2 of 3


2

Draw the resultant R from the tails of the vectors to the


opposite vertex of the parallelogram.

y
Tails
x

10
3 kN
20
2 kN

2 kN

3 kN
3 To calculate the magnitude and direction of R, consider
the triangle formed by one half of the parallelogram.

y
x

10

2 kN

3 kN
20

2 kN

20 20

10
3 kN

2 kN

10
3 kN

40

2.1 Adding Forces by the Parallelogram Law Example 2, page 3 of 3


4

Use trigonometry to calculate the magnitude


and direction of the resultant.

5 Law of cosines
R2 = (3 kN)2 + (2 kN)2

10
3 kN

Law of sines
sin
sin 120
=
R
3 kN
= 4.359 kN

20
20
2 kN

2(3 kN)(2 kN) cos 120

R = 4.359 kN

Ans.

total angle
= 10 + 90 + 20 = 120

Angle measured with


respect to the vertical axis

y
x

Solving gives
= 36.6

= 36.6
R = 4.36 kN

20

36.6 + 20 = 56.6

41

Ans.

2.1 Adding Forces by the Parallelogram Law Example 3, page 1 of 3


3. Determine the magnitude and direction of the resultant force.
100 N

110
1
40

Construct a parallelogram
100 N

80 N
y

110

40
80 N

42

2.1 Adding Forces by the Parallelogram Law Example 3, page 2 of 3


2 Draw the resultant R from the tails of the vectors to
the opposite vertex of the parallelogram.
100 N

3 To calculate R, consider the triangle formed


by the lower half of the parallelogram.
100 N

80 N

180

40

100 N
80 N

40
80 N
6 Calculate angle
30 + 40 = 70

7 Law of cosines

R = 104.54 N

40 = 30

5 Parallel lines make 30 angle


with vertical direction

110

110

R2 = (80 N)2 + (100 N)2

110

40

Calculate angle

2(80 N)(100 N) cos 70


Ans.

43

30

2.1 Adding Forces by the Parallelogram Law Example 3, page 3 of 3


8 Calculate the angle that the
resultant makes with the vertical.

y
R = 104.54 N

Law of sines
sin
sin 70
=
100 N
R 104.54 N
Solving gives

40

70

= 64.0

9 Angle measured from the vertical


y
R = 104.54 N
64
40

40 + 64.0 = 104.0

Ans.

44

100 N

2.1 Adding Forces by the Parallelogram Law Example 4, page 1 of 4


4. The resultant of the two forces acting on the screw eye is known to
be vertical. Determine the angle and the magnitude of the resultant.
y

x
30
40 lb

60 lb

45

2.1 Adding Forces by the Parallelogram Law Example 4, page 2 of 4


1 To determine what needs to be calculated, make some
sketches of several possible parallelograms.
y

y
x

30

40 lb

y
x

30
40 lb

x
30
40 lb

Each parallelogram is based on two facts that are given:


1) One side of the parallelogram is known (40 lb at 30), and
2) The resultant R lies on the y axis.

46

2.1 Adding Forces by the Parallelogram Law Example 4, page 3 of 4


3

How do we determine the actual parallelogram? We have


to use the additional fact that one of the forces is 60 lb.
y
x
30
40 lb

Radius of circular
arc = 60 lb

4 The point of the intersection of the arc and the


vertical axis must be the vertex of the
parallelogram since it lies on the vertical axis and
also lies a "distance" of 60 lb from the tip of the
40-lb vector.
5 Now the parallelogram is completely defined.
y
x
30
40 lb
60 lb

60 lb

47

40 lb

2.1 Adding Forces by the Parallelogram Law Example 4, page 4 of 4


6 To calculate the resultant R and and the
angle (see below), analyze the triangle
formed by the left half of the parallelogram.
30
40 lb

9 Law of sines
sin
sin 60
=
40 lb
60 lb

Angle = 90 30
= 60

40 lb
60

Solving gives

+ 30
R

= 35.26

30

60 lb
R
Parallel
10 The sum of the angles of the triangle is 180:
+ ( + 30) + 60 = 180
60 lb
8

35.26
Solving gives

Corresponding angles are equal

= 54.74

Ans.

11 Law of sines

54.74
sin 60= sin( + 30)
R
60 lb

Solving gives
R = 69.0 lb

48

Ans.

2.1 Adding Forces by the Parallelogram Law Example 5, page 1 of 2


5. Determine the magnitude F and the angle , if the
resultant of the two forces acting on the block is to be a
horizontal 80-N force directed to the right.
50 N

F
25

Draw the parts of parallelogram that are known:


50 N

25

3 Two sides are of length 50 N and make


an angle of 25 with the horizontal axis.

80 N
50 N
25

2 The diagonal of the parallelogram (the


resultant) is 80 N long and horizontal.

49

2.1 Adding Forces by the Parallelogram Law Example 5, page 2 of 2


4

Complete the parallelogram.


50 N

25

F
80 N
25
50 N

25

5 Analyze the triangle forming the lower half of the parallelogram.


80 N
25
F

50 N

Calculate F from the law of cosines.


F2 = (50 N)2 + (80 N)2

7 Calculate

The result is

from the law of sines.

sin
sin 25
=
F
50 N

F = 40.61 N

40.61 N
Solving gives

= 31.4

2(50 N)(80 N) cos 25

Ans.

50

Ans.

2.1 Adding Forces by the Parallelogram Law Example 6, page 1 of 2


6. To support the 2-kg flower pot shown, the resultant of the two
wires must point upwards and be equal in magnitude to the
weight of the flower pot. Determine the angles and , if the
forces in the wires are known to be 25 N and 30 N.

A
C
25 N

30 N
B

2 Resultant, R, of forces in wires


balances the weight.
1 Weight of flower pot
mg = (2 kg)(9.81 m/s2)

R = 19.62 N

= 19.62 N

51

19.62 N

19.62 N

2.1 Adding Forces by the Parallelogram Law Example 6, page 2 of 2


3 The resultant R = 19.62 N must be the diagonal of a
parallelogram with sides 25 N and 30 N long.

4 Analyze the triangle forming the left-hand


half of the parallelogram.

25 N
30 N

30 N

19.62 N

19.62 N
30 N
25 N

25 N

B
5 Law of cosines to calculate
(25 N)2 = (30 N)2 + (19.62 N)2

2(30 N)(19.62 N) cos

Solving gives
= 55.90

Ans.

6 Law of sines to calculate


sin
sin
55.90
=
25 N
30 N
Solving gives
= 83.6

52

Ans.

2.1 Adding Forces by the Parallelogram Law Example 7, page 1 of 2


7. Resolve the 120-lb force into components
acting in the u and v directions.
120 lb

25
u
40

1 Construct a parallelogram with


the 120-lb force as a diagonal.
v
40

120 lb

Rv

Draw a line from the tip of the


force vector parallel to v.

25
u
Ru

3 Draw another line from the


tip but parallel to u.

40

53

4 Label the components Ru and Rv.

2.1 Adding Forces by the Parallelogram Law Example 7, page 2 of 2


5 Analyze the triangle forming the left-hand half of the
parallelogram.
180 40 25 = 115
Ru
40

120 lb
Rv

25

Calculate Rv from the law of sines.


sin 115
sin 40
=
Rv
120 lb
Solving gives

6 Calculate Ru from the law of sines.

Rv = 169.2 lb

sin 25 sin 40
=
Ru
120 lb
Solving gives
Ru = 78.9 lb

Ans.

54

Ans.

2.1 Adding Forces by the Parallelogram Law Example 8, page 1 of 2


8. Resolve the 4-kN
horizontal force into
components along truss
members AB and AC.

4 kN

12
5
35
C

Construct a parallelogram with the


4-kN force as the diagonal.
2

Extend line AC.

12
3

Draw a line from the tip of the


force vector parallel to AB.

A
4 kN

Draw another line from


the tip but parallel to AC.

12
5
B

35
C

55

2.1 Adding Forces by the Parallelogram Law Example 8, page 2 of 2


6 Analyze the triangle forming the upper half of the
parallelogram (The drawing has been enlarged for clarity).

5 Label the components RB and RC.


RC

12
5

35
A

RC

4 kN
RB

12
RB

12
5
B

35

35

4 kN

7 Geometry
= tan-1

12
= 67.38
5

= 180

35

67.38

8 Law of sines to calculate RC


67.38
sin
sin
=
4 kN 77.62
RC
Solving gives
RC = 3.78 kN

Ans.

= 77.62
9 Law of sines to calculate RB
sin 35
RB
Solving gives

= sin
4 kN

RB = 2.35 kN

56

77.62

Ans.

2.1 Adding Forces by the Parallelogram Law Example 9, page 1 of 3


9. Find two forces, one acting along rod AB and one
along rod CB, which when added, are equivalent to the
200-N vertical force.
200 N

30

40
C

57

2.1 Adding Forces by the Parallelogram Law Example 9, page 2 of 3


1

Construct a parallelogram with


sides parallel to AB and BC and
with the 200-N force as a
diagonal.
30

4 Draw a line from the tail of the


force vector parallel to AB.
5 Draw another line from the
tail but parallel to BC.

200 N

Extend BC.

30

40

Extend AB.

6 Label the sides of the


parallelogram RA and RC.
RA
30

30
A

40
C

RC
200 N

RC
RA
30

40
B

58

2.1 Adding Forces by the Parallelogram Law Example 9, page 3 of 3


7 Analyze the triangle
formed by the
left-hand side of the
parallelogram.

RA
10
180

50

(30 + 40) = 60

30
200 N

40
8 Angles are equal

RC

9
40

90

40 = 50

11 Law of sines to calculate RA

12 Law of sines to calculate RC


sin 60 = sin (30 + 40)
RC
200 N
Solving gives

sin 50 = sin (30 + 40)


RA
200 N
Solving gives
RA = 163.0 N

RC = 184.3 N

Ans.

59

Ans.

2.2 Rectangular Components in Two-Dimensional Force Systems

60

2.2 Rectangular Components in Two-Dimensional Force Systems Procedures and Strategies, page 1 of 1
Procedures and Strategies for Solving Problems Involving
Rectangular Components in Two-Dimensional Force
Systems

y
F
Fy = F sin

1. Two situations commonly arise in which the rectangular


components are to be computed.
a) The force is defined by its magnitude F and the angle
that the force makes with the positive x axis. In this case,
use the equations
Fx = F cos

Fx = F cos
y

Fy = F sin
5

b) The force is defined by a magnitude F and a slope


triangle. The components can be computed by multiplying
the magnitude F by the ratios of the slope triangle the ratio
of the horizontal side to the hypotenuse gives the horizontal
component of the force. The ratio of the vertical side gives
the vertical component of the force.

13

5F
Fy =

13

12
12F
Fx =

13
x

2. To add forces simply


a) add all x components to obtain Rx, the x component of the
resultant R, and

y
2

b) add all y components to obtain Ry, the y component of the


resultant R.

R = Rx + R y

= tan-1 Rx
Ry

The magnitude and direction of the resultant can be found


from a right triangle with R, Rx, and Ry as its sides.

Ry
Rx

61

2.2 Rectangular Components in Two-Dimensional Force Systems Problem Statement for Example 1
1. Express the 5-kN force in terms of x and y components.
y

F = 5 kN
30
x

62

2.2 Rectangular Components in Two-Dimensional Force Systems Problem Statement for Example 2
2. Resolve the 20-lb force into x and y components.
F = 20 lb
y

3
4

63

2.2 Rectangular Components in Two-Dimensional Force Systems Problem Statement for Example 3
3. Express the 260-N force in terms of components
parallel and perpendicular to the inclined plane.
F = 260 N

12
5

64

2.2 Rectangular Components in Two-Dimensional Force Systems Problem Statement for Example 4
4. Determine the components of the 160-N force
perpendicular and parallel to the axis of the nail.

F = 160 N
15
20

65

2.2 Rectangular Components in Two-Dimensional Force Systems Problem Statement for Example 5
5. The connecting rod AB exerts a 2-kN force on the
crankshaft at B. Resolve this force into components
acting perpendicular to BC and along BC.

F = 2 kN
20
B
30
C

66

2.2 Rectangular Components in Two-Dimensional Force Systems Problem Statement for Example 6
6. Guy wire AB exerts a horizontal component of force of 0.5 kN
on the utility pole. Determine the total force from the wire acting
on the point of attachment, A. Assume that the force is directed
along the wire from A to B.

10 m
B

5m

67

2.2 Rectangular Components in Two-Dimensional Force Systems Problem Statement for Example 7
7. If the vertical component of the force F
applied to the ring is 10 lb, determine the
magnitude F and also the horizontal component.
F
30

68

2.2 Rectangular Components in Two-Dimensional Force Systems Problem Statement for Example 8
8. The weight W is supported by the boom AB and
cable AC. Knowing that the horizontal and vertical
components of the cable force at A are 5 kN and 3 kN
as shown, determine the distance d.
C
Fcable

5 kN

3 kN
A

W
B

10 m

69

2.2 Rectangular Components in Two-Dimensional Force Systems Problem Statement for Example 9
9. Determine the magnitude and direction of
the resultant force acting on the hook.
20 lb

35
x
5
12
104 lb

70

2.2 Rectangular Components in Two-Dimensional Force Systems Problem Statement for Example 10
10. Determine the magnitude and direction
of the resultant force acting on the beam.
15 kN

8 kN

3
40

11 kN

71

2.2 Rectangular Components in Two-Dimensional Force Systems Problem Statement for Example 11
11. Determine the magnitude and direction
of the resultant force acting on the particle.
y

(5 m, 3 m)

x
25 N
50 N

(6 m, 2 m)
80 N

m, 6 m)

72

2.2 Rectangular Components in Two-Dimensional Force Systems Problem Statement for Example 12
12. Three forces support the weight W shown. Determine
the value of F, given that the resultant of the three forces
is vertical. Also determine the value of W.
y
F
20 N

120 N
40
30
x

15
W

73

2.2 Rectangular Components in Two-Dimensional Force Systems Problem Statement for Example 13
13. The resultant, R, of the forces A and B acting on the
bracket is known to be a force of magnitude 300 lb
making an angle of 40 with the horizontal direction as
shown. Determine the magnitude of A and B.
y
B
300 lb (resultant, R)
70

40
A

74

2.2 Rectangular Components in Two-Dimensional Force Systems Problem Statement for Example 14
14. To support the 100-N block as shown, the resultant of
the 50-N force and the force F must be a 100-N force
directed horizontally to the right. Determine F and .
F

Pulley

60

50 N
100 N

75

2.2 Rectangular Components in Two-Dimensional Force Systems Example 1, page 1 of 3


1. Express the 5-kN force in terms of x and y components.
y

F = 5 kN
30
x

Construct a parallelogram with the


5-kN force as the diagonal and with
sides in the x and y directions.

Because the x and y axes are


perpendicular, the parallelogram
is a special case a rectangle.

Analyze the triangle


forming the lower half of
the rectangle.

Fx
F = 5 kN
Fy

30

y component, Fy
x component, Fx

5 kN
Fy

30
Fx

76

2.2 Rectangular Components in Two-Dimensional Force Systems Example 1, page 2 of 3


4

Calculate Fx from the definition of the cosine:


cos

A
C

Similarly calculate Fy from the definition of the


sine:

sin

so
so

A = C cos

B
C

B = C sin
In words,
In words ,
side adjacent (to angle) = hypotenuse
times cosine of angle.

side opposite (to angle) = hypotenuse


times sine of angle.

(Memorize this you will use this relation many


times in a course in statics; you don't want to have to
think it out each time)

(Memorize this.)
Applying this equation to the force triangle gives:

Applying this equation to the force triangle gives:


Fy = (5 kN) sin 30
Fx = (5 kN) cos 30

= 4.33 kN

= 2.50 kN

(1)

77

(2)

2.2 Rectangular Components in Two-Dimensional Force Systems Example 1, page 3 of 3


6 Thus we have resolved the 5-kN force
into x and y components.
y
F = 5 kN

Fy = 2.50 kN

30
x
Fx = 4.33 kN

In terms of base vectors, the force is

F=

4.33i + 2.50j kN

Ans.

j
x

8 The minus sign indicates that the x component


points in the negative x direction.

78

2.2 Rectangular Components in Two-Dimensional Force Systems Example 2, page 1 of 4


2. Resolve the 20-lb force into x and y components.
F = 20 lb
y

3
4

Construct a parallelogram (rectangle)


with the 20-lb force as the diagonal.

Analyze the triangle forming the


lower half of the rectangle.

Fx

20 lb

F = 20 lb

Equal angles

Fy

Fy

Fx

Fx

79

Fy

2.2 Rectangular Components in Two-Dimensional Force Systems Example 2, page 2 of 4


3

Side adjacent = hypotenuse cos


or
Fx = (20 lb) cos

(1)

Similarly, side opposite = hypotenuse sin


or
Fy = (20 lb) sin

(2)
32 + 42 = 5

From the "slope triangle," we see


cos
sin

4
5
3
5

(3)
3
4

Note that we do not have to calculate ;


we already have what we need, sin and cos .

Fx

80

Fy

2.2 Rectangular Components in Two-Dimensional Force Systems Example 2, page 3 of 4

Using Eq. 3 in Eq. 1 gives

F = 20 lb

Fx = (20 lb) ( 4 )
5
= 16 lb

(5)

Fx

In general, then, get the horizontal force


component by multiplying the force by
the horizontal side of the slope triangle
divided by the hypotenuse (Memorize
this result; it is used frequently).

81

Slope triangle

2.2 Rectangular Components in Two-Dimensional Force Systems Example 2, page 4 of 4

Similarly, using Eq. 4 in Eq. 2 gives

F = 20 lb

Fy = (20 lb) ( 3 )
5
= 12 lb

Fy

(6)

8 In general, get the vertical force


component by multiplying the force by
the vertical side of the slope triangle
divided by the hypotenuse (Memorize
this result).
y
F = 20 lb
9

Eqs. 5 and 6 now give the components in


terms of base vectors as

F = {16i + 12j} lb

Ans.

12 lb

j
i

82

x
16 lb

2.2 Rectangular Components in Two-Dimensional Force Systems Example 3, page 1 of 2


3. Express the 260-N force in terms of components
parallel and perpendicular to the inclined plane.
F = 260 N

12
5

Introduce an inclined x and y coordinate system.


y

Construct a parallelogram (rectangle) with the


260-N force as a diagonal.

F = 260 N

Fx
Fy

12

12

5
5
Fx
x

83

Fy

2.2 Rectangular Components in Two-Dimensional Force Systems Example 3, page 2 of 2


3

Analyze the triangle forming the lower half


of the rectangle.
F = 260 N
2

5 + 12 = 13
13
5

Fy
12

Fx
4

Fx = (260 N)( 5 ) = 100 N


13

Fy = (260 N)( 12 ) = 240 N


13
y
6

Representation in terms of components:

j
i

F = { 100i

240j} N

Ans.

x
7

Fx points in the negative x direction, and


Fy points in the negative y direction.

84

2.2 Rectangular Components in Two-Dimensional Force Systems Example 4, page 1 of 3


4. Determine the components of the 160-N force
perpendicular and parallel to the axis of the nail.

F = 160 N
15
20

Introduce an inclined x and y coordinate system.


y

F = 160 N
15
20

85

2.2 Rectangular Components in Two-Dimensional Force Systems Example 4, page 2 of 3


2

Geometry

F = 160 N
4 Total angle

20
15

15

= 20 + 15

Equal angles

= 35

Draw a parallelogram (rectangle)


with the 160-N force as a diagonal.

Fx
F=
Fy

160

35

86

2.2 Rectangular Components in Two-Dimensional Force Systems Example 4, page 3 of 3


6

Analyze the triangle forming the


bottom half of the rectangle.

F=
Fy

160

35
Fx = (160 N) cos 35

Fy = (160 N) sin 35

= 131.1 N

= 91.8 N

x
15

j
7

In terms of base vectors,

F = { 131.1i

91.8j} N

i
Ans.

87

2.2 Rectangular Components in Two-Dimensional Force Systems Example 5, page 1 of 3


5. The connecting rod AB exerts a 2-kN force on the
crankshaft at B. Resolve this force into components
acting perpendicular to BC and along BC.

F = 2 kN
20
B
30
C

88

2.2 Rectangular Components in Two-Dimensional Force Systems Example 5, page 2 of 3


1

Introduce an inclined x and y


coordinate system.

2 Calculate angles
2 kN
3

Equal

5
30

20
A

Calculate the sum: 20 + 30 = 50

B
2 kN

30
y

x
Equal

20
C

20
B
30
x
C

Calculate components
2 kN

y
50
(2 kN) cos 50 = 1.286 kN

(2 kN) sin 50 = 1.532 kN


B
C

89

2.2 Rectangular Components in Two-Dimensional Force Systems Example 5, page 3 of 3


7

In terms of base vectors,

F = 1.532i

1.286j kN

Ans.

30

90

2.2 Rectangular Components in Two-Dimensional Force Systems Example 6, page 1 of 3


6. Guy wire AB exerts a horizontal component of force of 0.5 kN
on the utility pole. Determine the total force from the wire acting
on the point of attachment, A. Assume that the force is directed
along the wire from A to B.

10 m
B

5m

91

2.2 Rectangular Components in Two-Dimensional Force Systems Example 6, page 2 of 3


1

Express the guy-wire force F in


terms of rectangular components.
2
A

Fx = 0.5 kN

Fy

10 m

The horizontal component of


force is known to be 0.5 kN.

5m

92

2.2 Rectangular Components in Two-Dimensional Force Systems Example 6, page 3 of 3


3

Relate Fx to F through geometry.

10 m
B

= tan-1

10 m
= 63.43
5m
Fx = 0.5 kN
5 0.5 kN = F cos
63.43

Fy

Solving gives

5m

F = 1.118 kN

93

Ans.

2.2 Rectangular Components in Two-Dimensional Force Systems Example 7, page 1 of 1


7. If the vertical component of the force F
applied to the ring is 10 lb, determine the
magnitude F and also the horizontal component.
F

Relate F to Fy.

30
10 lb = F sin 30
F

Fy = 10 lb

Therefore,

30
Fx

Express F in terms of rectangular components.

F = 20 lb

Ans.

Relate Fx to F.
Fx = F cos 30

Fy
30

= (20 lb) cos 30

Fx

= 17.32 lb
y
x

94

Ans.

2.2 Rectangular Components in Two-Dimensional Force Systems Example 8, page 1 of 2


8. The weight W is supported by the boom AB and
cable AC. Knowing that the horizontal and vertical
components of the cable force at A are 5 kN and 3 kN
as shown, determine the distance d.

Calculate the angle between Fcable


and its horizontal component.
C

C
Fcable

5 kN

Fcable

d
3 kN
A

5 kN

3 kN
A

W
W
B
B
10 m
10 m
Fcable
3 kN
5 kN
= tan-1

3 kN
5 kN

= 30.96

95

2.2 Rectangular Components in Two-Dimensional Force Systems Example 8, page 2 of 2


2

Use

to calculate d.

C
d
= 30.96
10 m

d = (10 m) tan 30.96


= 6.0 m
3

Ans.

The same result could also have been


obtained by using similar triangles.
3 kN
d
=
5 kN 10 m

C
d

Therefore,
3 kN
5 kN

d = 6.0 m (same as before)

10 m

96

2.2 Rectangular Components in Two-Dimensional Force Systems Example 9, page 1 of 2


9. Determine the magnitude and direction of
the resultant force acting on the hook.
20 lb

Calculate the x and y components of the resultant R by summing the


components of the given forces algebraically.

35
x

Rx = Fx: Rx = 16.38 lb + 96 lb = 112.38 lb

12
104 lb

Ry = Fy: Ry = 11.47 lb

Express the forces in x and y components.


(20 lb) sin 35 = 11.47 lb
20 lb
(20 lb) cos 35 = 16.38 lb
35
5

13
12

(104 lb)( 5 ) = 40 lb
13

(104 lb)( 12 ) = 96 lb
13
104 lb
52 + 122 = 13

97

40 lb = 28.53 lb = 28.53 lb
(arrow indicates negative y direction)

2.2 Rectangular Components in Two-Dimensional Force Systems Example 9, page 2 of 2


3

Calculate the magnitude and direction of the resultant R.


y

112.38 lb
28.53 lb

R=

(112.38 lb)2 + (28.53 lb)2

= 115.9 lb
= tan-1 28.53 lb
112.38 lb
= 14.2

14.2

Ans.
115.9 lb

98

2.2 Rectangular Components in Two-Dimensional Force Systems Example 10, page 1 of 2


10. Determine the magnitude and direction
of the resultant force acting on the beam.

(8 kN) sin 40 = 5.142 kN

15 kN

8 kN

Resolve the forces into x and y components.

3
40

8 kN

15 kN
5 3
4

40

(15 kN)( 4 ) = 12 kN
5
x

11 kN
(8 kN) cos 40 = 6.128 kN

Calculate the x and y components of the resultant by summing the


components of the given forces algebraically.
Force points left
+

11 kN

(15 kN)( 3 ) = 9 kN
5

Rx = Fx: Rx = 6.128 kN
Ry = Fy: Ry = 5.142 kN

12 kN = 5.872 kN = 5.872 kN
9 kN + 11 kN = 3.142 kN = 3.142 kN
Force points down

99

2.2 Rectangular Components in Two-Dimensional Force Systems Example 10, page 2 of 2


Calculate the magnitude and direction of the resultant.

5.872 kN
3.142 kN

R=

(5.872 kN)2 + (3.142 kN)2

= 6.66 kN
= tan-1

3.142 kN
5.872 kN

= 28.2

28.2
Ans.
6.66 kN

100

2.2 Rectangular Components in Two-Dimensional Force Systems Example 11, page 1 of 5


11. Determine the magnitude and direction
of the resultant force acting on the particle.
y

(5 m, 3 m)

x
25 N
50 N

(6 m, 2 m)
80 N

m, 6 m)

101

2.2 Rectangular Components in Two-Dimensional Force Systems Example 11, page 2 of 5


1

We want to compute the x and y components of each


force. To do that, we first must compute some angles.
y

Components
of 80-N force.

Angle for
80-N force.

(80 N) sin 26.57 = 35.78 N


x

80 N

(80 N) cos 26.57 = 71.55 N


6m
= 26.57

m, 6 m)
80 N

3m
= tan-1 3 m
6m
= 26.57

102

2.2 Rectangular Components in Two-Dimensional Force Systems Example 11, page 3 of 5


2

Angle and components for 50-N force.

Components
of 50-N force.

Angle for
50-N force

(50 N) cos 30.96 = 42.88 N


3m

x
x

= 30.96

5m
50 N

50 N
= tan-1 3 m
5m
= 30.96

(50 N) sin 30.96 = 25.72 N

103

2.2 Rectangular Components in Two-Dimensional Force Systems Example 11, page 4 of 5


3

Angle and components for 25-N force.

y
y
Angle for
25-N force.

Components
of 25-N force.

6m

(25 N) cos 18.43 = 23.72 N

x
x

2m

25 N
= 18.43

25 N
(6 m, 2 m)
(25 N) sin 18.43 = 7.90 N
= tan-1 2 m
6m
= 18.43

104

2.2 Rectangular Components in Two-Dimensional Force Systems Example 11, page 5 of 5


Sum the components algebraically.
y

35.78 N
42.88 N

23.72 N

Rx = Fx: Rx = 35.78 N

42.88 N + 23.72 = 16.62 N

Ry = Fy: Ry = 71.55 N

25.72 N

7.90 N = 37.93 N

7.90 N
25.72 N

71.55 N

Calculate the magnitude and direction of the resultant R.

R=

R
37.93 N

(37.93 N)2 + (16.62)2

= 41.4 N
= tan-1

16.62 N

37.93 N
16.62 N

= 66.3

41.4 N
Ans.
66.3

105

2.2 Rectangular Components in Two-Dimensional Force Systems Example 12, page 1 of 3


12. Three forces support the weight W shown. Determine
the value of F, given that the resultant of the three forces
is vertical. Also determine the value of W.
y
F
20 N

120 N
40
30
x

15
W

Express the forces in x and y components (For clarity, the


components of the unknown force, F, are shown separately).
y

y
(120 N) sin 30 = 60 N
F
(20 N) sin 15 = 5.176 N

120 N

20 N

40

F cos 40

30
x

15
(20 N) cos 15 = 19.32 N

(120 N) cos 30 = 103.92 N

106

x
F sin 40
W

2.2 Rectangular Components in Two-Dimensional Force Systems Example 12, page 2 of 3


2

Sum the components algebraically.


Rx = Fx: Rx = 103.92 N

19.32 N

F sin 40

Ry = Fy: Ry = 60 N + 5.176 N + F cos 40

Use the fact that the resultant is known to be


vertical, so Rx = 0.
Eq. 1 becomes
Rx = 103.92 N

(1)
(2)

19.32 N

F sin 40

0
R = Ry

Solving gives

Rx = 0

F = 131.61 N

Ans.

x
W

Substitute this value of F into Eq. 2 and compute Ry:


Ry = 60 N + 5.176 N + F cos 40 = 166.0 N
131.61 N

107

2.2 Rectangular Components in Two-Dimensional Force Systems Example 12, page 3 of 3


5

The resultant upward force must balance the weight W, so


W = Ry = 166.0 N

Ans.

R = Ry = 166.0 N

108

2.2 Rectangular Components in Two-Dimensional Force Systems Example 13, page 1 of 2


13. The resultant, R, of the forces A and B acting on the
bracket is known to be a force of magnitude 300 lb
making an angle of 40 with the horizontal direction as
shown. Determine the magnitude of A and B.
y
B
300 lb (resultant, R)
70

40
x

Express the forces in x and y components.


y

B sin 70
B
70
B cos 70

(300 lb) sin 40 = 192.84 lb

Ry (y component of 300-lb resultant)

300 lb
40
A

(300 lb) cos 40 = 229.81 lb


x

109

Rx (x component of 300-lb resultant)

2.2 Rectangular Components in Two-Dimensional Force Systems Example 13, page 2 of 2


2
+

Rx is the algebraic sum of x components of A and B:


Rx = Fx: Rx = A

B cos 70

(1)

Similarly for Ry:

229.81 lb

Ry = Fy: Ry = B sin 70

(2)

192.84 lb

Solving Eqs. 1 and 2 simultaneously gives:


A = 300 lb

Ans.

B = 205 lb

Ans.

110

2.2 Rectangular Components in Two-Dimensional Force Systems Example 14, page 1 of 2


14. To support the 100-N block as shown, the resultant of
the 50-N force and the force F must be a 100-N force
directed horizontally to the right. Determine F and .
F

Pulley

60
1

Express the forces in x and y components.

50 N

y
F

100 N
F sin

F cos

60

x
(50 N) cos 60 = 25 N

(50 N) sin 60 = 43.30 N


50 N

111

2.2 Rectangular Components in Two-Dimensional Force Systems Example 14, page 2 of 2


Sum the components algebraically.

3 And then substitute for F in Eq. 3:

Rx = Fx: Rx = F cos

25 N

(1)

Ry = Fy: Ry = F sin

43.30 N

(2)

100 N = F cos

43.30 N
sin

Because the resultant is known to be horizontal,


Ry = 0, and the magnitude R is thus equal to the
horizontal component Rx alone, that is, R = Rx.
We also know, however, that the magnitude of
the resultant is 100 N, so R = Rx = 100 N. Thus
Eqs. 1 and 2 become

Substituting
cos
sin

1
tan

gives
43.30 N
+ 25 N
tan

100 N =
100 N = F cos

+ 25 N

+ 25 N

(3)

and solving gives


0 = F sin

43.30 N

(4)
= 30.0

The best way to solve these equations is to use a


calculator that can solve two simultaneous
nonlinear equations. Alternatively, solve Eq. 4
for F:
F=

43.30 N
sin

Using the result in Eq. 5 gives


F=

43.30 N
= 86.6 N
sin

(5)
86.6 N
30

112

Ans.

2.3 Rectangular Components in Three-Dimensional Force Systems

113

2.3 Rectangular Components in Three-Dimensional Force Systems Procedures and Strategies, page 1 of 1
Procedures and Strategies for Solving Problems Involving
Rectangular Components in Three-Dimensional Force Systems

1. If the magnitude F of a force and its direction angles, x y, and z, are


known, then compute the components of the force from the equations

F
y

Fx = F cos

Fy = F cos

Fz = F cos

Fx

If only two angles are known, then find the third angle from the equation
cos2

+ cos2

+ cos2

=1

2. If the magnitude F is known and the direction of the force is defined


through its projection on a horizontal plane, then compute the horizontal
components by projecting the projection onto the horizontal axes.
3. If the rectangular components Fx, Fy, and Fz are known, then compute
the magnitude F of the force from the equation
F=

Fx + Fy +

= Fx/F

F cos

(F sin ) sin
x

(F sin ) cos

cos

= Fy/F

cos

= Fz/F

4. To compute the resultant of several force, express each force in


rectangular component and add the components:
Rx = Fx

Fz2

and the direction angles from


cos

Ry = Fy

Rz = Fz

114

F sin

2.3 Rectangular Components in Three-Dimensional Force Systems Problem Statement. for Example 1
1. Express the force F in terms of x, y, and z components.
y
F = 200 lb

45
120
60

O
x

115

2.3 Rectangular Components in Three-Dimensional Force Systems Problem Statement for Example 2
2. Express F in terms of x, y, and z components.
y
F = 50 N
40

O
z

35
B
x

116

2.3 Rectangular Components in Three-Dimensional Force Systems Problem Statement. for Example 3
3. Express F in terms of x, y, and z components.
y
z

O
25
B
x
70

F = 8 kN

117

2.3 Rectangular Components in Three-Dimensional Force Systems Problem Statement. for Example 4
4. Determine the x, y, and z components of the 26-N force shown.
Also determine the coordinate direction angles of the force.
y

F = 26 N

12
5
13

A
20

118

2.3 Rectangular Components in Three-Dimensional Force Systems Problem Statement. for Example 5
5. Determine the magnitude and coordinate direction angles
of the resultant of the three forces acting on the mast.
y
F3 = 40 N

O
30
80
z

F2 = 60 N

20

F1 = 100 N

119

2.3 Rectangular Components in Three-Dimensional Force Systems Problem Statement. for Example 6
6. Determine the magnitude and coordinate direction angles
of the resultant of the forces acting on the eye-bolt.
F1 = 650 N

y
12
13 5
30

F2 = 800 N

O
F3 = 300 N

70

50

z
x

120

2.3 Rectangular Components in Three-Dimensional Force Systems Problem Statement. for Example 7
7. A 300-lb vertical force is required to pull the pipe out of the ground. Determine
the magnitude and direction angles of the force F2 which, when applied together
with the 150-lb force F1 shown, will produce a 300-lb vertical resultant.
y

F1 = 150 lb
45
60

F2

O
60

x
z

121

2.3 Rectangular Components in Three-Dimensional Force Systems Problem Statement. for Example 8
8. Two forces, F1 and F2 act on the bracket as shown. If the
resultant of F1 and F2 lies in the xy plane, determine the
magnitude of F2. Also determine the magnitude of the resultant.

F1 = 60 N

60
50
O
F2
z
x

122

2.3 Rectangular Components in Three-Dimensional Force Systems Example 1, page 1 of 2


1. Express the force F in terms of x, y, and z components.
y
F = 200 lb

2 Calculate the x component.


45
F = 200 lb
120
60

O
3

= 180

120 = 60

x
z
120
1

View of plane formed by the x axis and F.


d

F = 200 lb
4

d = (200 lb) cos 60 = 100 lb

5 The component points in the negative direction, so


Fx = 100 lb
120
O

123

Ans.

2.3 Rectangular Components in Three-Dimensional Force Systems Example 1, page 2 of 2


6

View of plane formed by the y axis and F


y
y component

7
F = 200 lb

Fy = (200 lb) cos 45 = 141.4 lb

Ans.

Fy
45

O
8

View of plane formed by the z axis and F.


F = 200 lb

z component

Fz = (200 lb) cos 60 = 100 lb

Ans.

60
z
Fz

10 F = Fxi + Fyj + Fzk


= { 100i + 141.4j + 100k} lb

124

Ans.

2.3 Rectangular Components in Three-Dimensional Force Systems Example 2, page 1 of 2


2. Express F in terms of x, y, and z components.
y
F = 50 N
40

O
z

35
B
x

1 View of plane formed by OA, F, and the y axis.


y
2

Fy = (50 N) cos 40 = 38.3 N

Ans.

F = 50 N
3

40

Fy

A
O

FOA

125

FOA = (50 N) sin 40 = 32.14 N

2.3 Rectangular Components in Three-Dimensional Force Systems Example 2, page 2 of 2


4 View of xz plane from above
A
6

FBA = (32.14 N) sin 35 = 18.4 N


32.14 N

FBA
7

Fz = 18.4 N

35

negative direction

y O

Fx

z
5

Fx = (32.14 N) cos 35 = 26.3 N

F = Fxi + Fyj + Fzk


= {26.3i + 38.3j

18.4k} N

Ans.

126

Ans.

2.3 Rectangular Components in Three-Dimensional Force Systems Example 3, page 1 of 2


3. Express F in terms of x, y, and z components.
y
z

O
25
B
x
70

F = 8 kN
1

View of the plane formed by OA, F, and the y axis.


3

y
FOA

O
2

(8 kN) cos 70 = 2.74 kN


Fy

Fy = 2.74 kN

70

Ans.

Negative direction

127

F = 8 kN

FOA = (8 kN) sin 70 = 7.518 kN


A

2.3 Rectangular Components in Three-Dimensional Force Systems Example 3, page 2 of 2


5 View of the xz plane from above.
6

Fz = (7.518 kN) sin 25


= 3.18 kN

Fx

25

Fx = (7.518 kN) cos 25 = 6.81 kN


B

Fz

Ans.

7.518 kN
z

F = Fxi + Fyj + Fzk


= {6.81i

2.74j + 3.18k} kN

Ans.

128

2.3 Rectangular Components in Three-Dimensional Force Systems Example 4, page 1 of 4


4. Determine the x, y, and z components of the 26-N force shown.
Also determine the coordinate direction angles of the force.
y

F = 26 N

12
5
13

A
20

B
1

View of the plane formed by OA, F, and the y axis.


y

F = 26 N

Fy = (26 N)( 5 )
13
= 10 N

Ans.

12
5
13

A
FOA

129

Fy
3

FOA = (26 N)( 12 )


13
= 24 N

2.3 Rectangular Components in Three-Dimensional Force Systems Example 4, page 2 of 4


7

4 View of the xz plane as seen from above

Determine the x coordinate direction angle, .


y

(24 N) sin 20 = 8.21 N


Fx = 8.21 N

5
y, O

Fx

Ans.

20

Fz = (24 N) cos 20 = 22.6 N

Fx = 8.21 N

F = 26 N

Negative direction
Ans.

Fz

x
24 N
8 View of the plane formed by the x axis and the force F.

8.21 N
z

9
x

10

The direction angle is measured


from the the positive part of the
axis. Here it is , not .

= 180

26 N
= 180
= 180
= 108.4

130

cos-1 8.21 N
26 N
71.59
Ans.

2.3 Rectangular Components in Three-Dimensional Force Systems Example 4, page 3 of 4


11 Determine the y coordinate direction angle, .
y

F = 26 N

Fy = 10 N

B
12 View of the plane formed by the y axis and the force F.
y

13
10 N

26 N
O

131

= cos-1 10 N
26 N
= 67.4
Ans.

2.3 Rectangular Components in Three-Dimensional Force Systems Example 4, page 4 of 4


14 Determine the z coordinate direction angle, .
y

15 View of the plane formed by the z axis and the force F.


26 N

16

F = 26 N
z

= cos-1 22.6 N
26 N
= 29.6
Ans.

O
22.6 N

Fz = 22.6 N
17 Observation: the calculations for , , and can be
summarized by the general formulas
cos

Fx
F

cos

Fy
F

cos =

Fz
F

The algebraic signs of Fx, Fy, and Fz must be included


when using these formulas.

132

2.3 Rectangular Components in Three-Dimensional Force Systems Example 5, page 1 of 3


5. Determine the magnitude and coordinate direction angles
of the resultant of the three forces acting on the mast.
y
F3 = 40 N

Express F1 in rectangular components.

(100 N) sin 80 = 98.48 N

y
O

3 F1z = (98.48 N) cos 20 = 92.54 N

O
30
80
z

F2 = 60 N
80
x

20

20
F1 = 100 N
F1 = 100 N
4
5

F1y = (100 N) cos 80 = 17.36 N

F1x = (98.48 N) sin 20 = 33.68 N

In vector form,

F1 = {33.68i

133

17.36j + 92.54k} N

(1)

2.3 Rectangular Components in Three-Dimensional Force Systems Example 5, page 2 of 3


7

Express F2 and F3 in rectangular components.


y
F3 = 40 N
8

F2y = (60 N) sin 30 = 30 N

O
10 F3y = 40 N

30

F2 = 60 N
9

F2x = (60 N) cos 30 = 51.96 N

x
11 In vector form,

F2 = {51.96i + 30j} N

(2)

F3 = {40j} N

(3)

12 Use Eqs. 1, 2, and 3 to compute the resultant, R.

R = F1 + F2 + F3
= {33.68i
13 Collect coeffiecients
of i, j, and k.

17.36j + 92.54k} N + {51.96i + 30j} N + {40j} N

= {33.68 + 51.96}i N + { 17.36 + 30 + 40}j N + {92.54k} N


= {85.64i + 52.64j + 92.54k} N

134

2.3 Rectangular Components in Three-Dimensional Force Systems Example 5, page 3 of 3


14 Magnitude of resultant
R=

(85.64 N)2 + (52.64 N)2 + (92.54 N)2

= 136.6 N

Ans.

15 Coordinate direction angles


cos

Rx
85.64 N
=
=
R
136.6 N

cos

Ry
52.64 N
= R =
136.6 N

cos =

Rz
92.54 N
=
136.6 N
R

Solving for the angles gives


= 51.2

Ans.

= 67.4

Ans.

= 47.4

Ans.

135

2.3 Rectangular Components in Three-Dimensional Force Systems Example 6, page 1 of 4


6. Determine the magnitude and coordinate direction angles
of the resultant of the forces acting on the eye-bolt.
F1 = 650 N

y
12
13 5
30

F1 = 650 N

F2 = 800 N

O
F3 = 300 N

Express F1 in rectangular components.

70

5
F1y = (650 N)(
) = 250 N
13

50

12
13 5

z
30
x

3
4

(650 N)( 12 ) = 600 N


13

F1x = (600 N) cos 30 = 519.6 N

5 F1z = (600 N) sin 30 = 300 N


6

In vector form,

F1 = { 519.6i + 250j

136

O
z

300k} N

(1)

2.3 Rectangular Components in Three-Dimensional Force Systems Example 6, page 2 of 4


7 Express F2 in rectangular components.

10 Since we are not given

F2y = F2 cos

2,

we cannot compute F2y from the formula

But since we know the magnitude F2 = 800 N, we can solve for F2y
from the formula for the magnitude of a vector in terms of its
rectangular components:
2

F2 = F2x2 + F2y2 + F2z2


F2 = 800 N

O
70

Substituting for F2, F2x, and F2z gives

50

(800 N) = (514.2 N)2 + F2y2 + (273.6 N)2

and solving for F2y gives


F2y = 548.4 N

x
8 F2x = (800 N) cos 50 = 514.2 N
9

Alternatively, we could have solved for


satsified by the direction angles:
(cos 50 )2 + (cos

F2z = (800 N) cos 70 = 273.6 N

Solving gives

2
2)

by using the identity

+ (cos 70 )2 = 1

= 46.73 and thus

F2y = (800 N) cos 46.73 = 548.3 N


This is the same result as before, allowing for round-off error.

137

2.3 Rectangular Components in Three-Dimensional Force Systems Example 6, page 3 of 4


y
12 In vector form, F2 can be written as

11 The figure shows that F2y is


positive, so choose the plus sign:

F2 = {514.2i + 548.4j + 273.6k} N

F2y

F2y = +548.4 N

(2)

F2

O
z

13 Also, in vector form, F3 is


x

F3 = {300k} N

(3)

F3 = 300 N
z

14 Use Eqs. 1, 2, and 3, to compute the resultant R.

R = F1 + F2 + F3
= { 519.6i + 250j

300k} N + {514.2i + 548.4j + 273.6k} N + {300k} N

= { 519.6 + 514.2}i N + {250 + 548.4}j N + { 300 + 273.6 + 300}k N


= { 5.4i + 798.4j + 273.6k} N

138

2.3 Rectangular Components in Three-Dimensional Force Systems Example 6, page 4 of 4


15 Magnitude of resultant
R = ( 5.4 N)2 + (798.4 N)2 + (273.6 N)2
= 844.0 N

Ans.

16 Coordinate direction angles


Rx
5.4 N
cos = R =
844.0 N
cos

Ry
798.4 N
= R = 844.0 N

cos =

Rz
273.6 N
=
R
844.0 N

Solving gives

= 90.4

Ans.

= 18.9

Ans.

= 71.1

Ans.

139

2.3 Rectangular Components in Three-Dimensional Force Systems Example 7, page 1 of 3


7. A 300-lb vertical force is required to pull the pipe out of the ground. Determine
the magnitude and direction angles of the force F2 which, when applied together
with the 150-lb force F1 shown, will produce a 300-lb vertical resultant.
y
F1 = 150 lb
45

Express F1 in rectangular components

60
F2

F1x = F1 cos

1=

(150 lb) cos 60 = 75 lb

F1y = F1 cos

1=

(150 lb) cos 60 = 75 lb

F1z = F1 cos

1=

(150 lb) cos 135 = 106.1 lb

O
60

F1 = 150 lb

1=

180

45

45 = 135

1 = 60

( 1 measured from positive z)


1=

60

O
z
2

F1 = {75i + 75j

140

In vector form,
106.1k} lb

(1)

2.3 Rectangular Components in Three-Dimensional Force Systems Example 7, page 2 of 3


3

Express F2 in terms of components,

F2 = F2xi + F2yj + F2zk

4
(2)

Equating coefficients of i gives


0 = 75 lb + F2x
Solving gives

Use Eqs. 1 and 2 to compute the resultant,

F2x = 75 lb

R = F1 + F2
= {75i + 75j

106.1k} lb

Equating coefficients of j, and then k, gives

+ {F2xi + F j + F2zk}
2y

300 = 75 lb + F2y
0 = 106.1 lb + F2z

= {75 lb + F2x}i + {75 lb + F }j


2y
+ { 106.1 lb + F2z}k

(4)

Solving gives

(3)

Now we use the fact that R is known to be vertical, directed upwards


with a magnitude of 300 lb:

R = 0i + 300j + 0k
So Eq. 3 becomes
0i + 300j + 0k ={75 lb + F2x}i + {75 lb + F2y}j
+ { 106.1 lb + F2z}k

141

F2y = 225 lb

(5)

F2z = 106.1 lb

(6)

2.3 Rectangular Components in Three-Dimensional Force Systems Example 7, page 3 of 3


5

Magnitude of F2
F2 = (F2x)2 + (F2y)2 + (F2z)2
=

( 75 lb)2 + (225 lb)2 + (106.1 lb)2


ns.

= 259.8 lb
Coordinate direction angles
F2x
75 lb
cos 2 = F =
259.8 lb
2
F2y

2=

cos

F2z
106.1 lb
=
2
F2 = 259.8 lb

F2

225 lb
259.8 lb

cos

F2

Solving gives
2

= 106.8

Ans.

= 30.0

Ans.

2
2

= 65.9

Ans.

O
z

142

2.3 Rectangular Components in Three-Dimensional Force Systems Example 8, page 1 of 3


8. Two forces, F1 and F2 act on the bracket as shown. If the
resultant of F1 and F2 lies in the xy plane, determine the
magnitude of F2. Also determine the magnitude of the resultant.

F1 = 60 N
y

60
50
O
F2
z
x

143

2.3 Rectangular Components in Three-Dimensional Force Systems Example 8, page 2 of 3


1

Express F1 in rectangular components:


F1x = F1 cos

1=

F1y = F1 cos

(60 N) cos 130 = 38.57 N

= (60 N) cos 60 = 30 N

(1)
F1 = 60 N

(2)

F1y

Since we do not know the value of 1, we cannot compute F1z


from

= 60
F1z = F1 cos

50

F1x

Instead we can use the equation for the magnitude of F1:

F1z

1
2

F1 = (F1x) + (F1y) + (F1z)

60 N = ( 38.57 N)2 + (30 N)2 + (F1z)2

1=

180

50 = 130

z
x

Solving gives
2

F1z = 34.82 N

F1z points in the negative direction so


choose the minus sign
F1z = 34.82 N

In vector form, from Eqs. 1, 2, and 3,

F1 = { 38.57i + 30j

144

(3)

34.82k} N

(4)

2.3 Rectangular Components in Three-Dimensional Force Systems Example 8, page 3 of 3


4

Now, Eq. 4 and the vector form of F2,

Equating coefficients of k gives


0 = F2

F2 = F2k
or

can be used to compute the resultant, R:

F2 = 34.82 N

R = F1 + F2
= { 38.57i + 30j

34.82 N

34.82k} N + F2k

(5)

Finally, the magnitude of R is


R = (Rx)2 + (Ry)2 + (Rz)2

Because the resultant R lies in the x-y plane,


Rz = 0

= ( 38.57 N)2 + (30 N)2 + (0)2

(6)

Combining Eqs. 5 and 6 gives


Rxi + Ryj + 0k ={ 38.57i + 30j + (F2

Ans.

= 48.9 N
34.82)k} N

Equating coefficients of i gives


Rx = 38.57 N
Equating coefficients of j gives
Ry = 30 N

145

Ans.

2.4 Position Vectors. Use in Defining Force Vectors.

146

2.4 Position Vectors. Use in Defining Force Vectors. Procedures and Strategies, page 1 of 2
Procedures and Strategies for Solving Problems
Involving Position Vectors.

B(5 m, 9 m, 8 m)

1. The components of the position vector from point A to


point B can be found in two ways.

rAB
A( 1 m, 5 m, 4 m)

a) Subtract the coordinates of point A from the


coordinates of point B. Note that the tail coordinates
must be subtracted from the head coordinates.

x
z

b) Determine the movement required in each coordinate


direction to go from point A to point B.

rAB = [5

( 1) i +(9

= {6i + 4j

5)j + ( 8

4)k

12k} m

1. Move 6 m in the +x direction (x component of +6).


2. Move 4 m in the +y direction (y component of +4).
3. Move 12 m in the z direction (z component of 12).

147

2.4 Position Vectors. Use in Defining Force Vectors. Procedures and Strategies, page 2 of 2
2. If the line of action of a force of magnitude F passes
through two points, A and B, then the position
vector from A to B can be used to find the
components of the force vector by using the
following procedure:
a) Express the position vector from A to B in
rectangular component form (If the force points
from B to A, then the position vector should
point from B to A).

B(5 m, 9 m, 8 m)

y
F = 42 N
A( 1 m, 5 m, 4 m)

x
z
B(5 m, 9 m, 8 m)

b) Find a unit vector parallel to the position vector


by dividing each component of the position
vector by its magnitude.
c) Multiply each component of the unit vector by F
to find the component of the force.

rAB = {6i + 4j 12k} m


(Shown above)
A(- 1 m, 5 m, 4 m)
x
z
rAB =

(6)2 + (4)2 + ( 12)2 = 14 m

r
6i + 4j 12k
uAB = rAB =
= 3 i+ 2j
7
7
AB
14
FAB = 42uAB = 42( 3 i + 2 j
7
7
= {18i + 12j

148

36k} N

6 k)
7

6 k
7

2.4 Position Vectors. Use in Defining Force Vectors. Problem Statement for Example 1
1. Determine a) the position vector directed from point A to point B and
b) the position vector directed from point B to point A.
y

B (11, 10)

A (3, 4)

149

2.4 Position Vectors. Use in Defining Force Vectors. Problem Statement for Example 2
2. Determine the position vector directed from point B to
point A. Also determine the distance between B and A.
y
4m
2m
6m
1m
A

x
3m

7m

150

2.4 Position Vectors. Use in Defining Force Vectors. Problem Statement for Example 3
3. Determine a) the position vector rAB from A to B, b) unit vector u in
the same direction, and c) the direction cosines of rAB.
y
B
8m
2m

30

3m
x
4m
50
z

151

2.4 Position Vectors. Use in Defining Force Vectors. Problem Statement for Example 4
4. The radio tower is supported by three guy wires. If the tension in
wire CD is 1.6 kN, determine the components of the force from CD
acting on the tower at D. Also determine the coordinate direction
angles of the force vector.
y
D
A
30 m

x
6m
C

4m
B
z

152

2.4 Position Vectors. Use in Defining Force Vectors. Problem Statement for Example 5
5. The awning is supported by cables CD
and AB. If the tension in CD is 80 lb,
determine the components of the force
from the cable acting on the awning at C.

2 ft
B
D

C
3 ft
5 ft
x

153

2.4 Position Vectors. Use in Defining Force Vectors. Problem Statement for Example 6
6. A cylindrical reactor vessel is being lifted by a crane connected to
three 2-m long cables. If the tension in cable AC is 2 kN, determine
the components of the force exerted by the cable on the vessel at C.
y
A
B
2m
120
D
z

Radius = 1.5 m

O
120

C
x

154

2.4 Position Vectors. Use in Defining Force Vectors. Problem Statement for Example 7
7. Strut EF is employed as a temporary support to help retaining wall ABCD
resist soil pressure. If the compressive force in the strut is FFE = 3 kip,
determine the components of the force from the strut acting on the wall at E.
y

E
8 ft
Soil pressure

D
6 ft
F
3 ft

155

2.4 Position Vectors. Use in Defining Force Vectors. Problem Statement for Example 8
8. If the axial compression forces in the legs of the tripod cooking-stand
are denoted FA, FB, and FC respectively, determine the resultant force
acting on the connection at D in terms of FA, FB, and FC.
y
D

25 in.
C
18 in.
x

A
18 in.
14 in.

14 in.

156

2.4 Position Vectors. Use in Defining Force Vectors. Problem Statement for Example 9
9. The 20-m boom DCE is supported by a ball-and-socket joint at D and two
cables attached at the midpoint C of the boom. If the tension in each cable is
500 N and the coordinates direction angles of the boom are 60, 45, and 60
as shown, determine the resultant of the cable forces acting on the boom.
y
3m

3m

B
20 m

C
12 m
45
60

60

8m

x
8m

157

2.4 Position Vectors. Use in Defining Force Vectors. Problem Statement for Example 10
y

10. A 12-ft wide movie screen is suspended in a


corner of a room by four wires. The ends C and F of
the screen are located the same distance, L, from the
corner. If the tension in wires DO and DA are 12 lb
and 8 lb respectively, determine the resultant force
from the wires acting on handle D of the screen.

O
L

A
z

2 ft

C
4 ft

158

4 ft

F
4 ft

2 ft

2.4 Position Vectors. Use in Defining Force Vectors. Example 1, page 1 of 2


1. Determine a) the position vector directed from point A to point B and
b) the position vector directed from point B to point A.
y

B (11, 10)

A (3, 4)

x
1

Part a)
Draw the vector from A to B.
y

B (11, 10)

rAB

rAB = B's coordinates minus A's coordinates


= (xB

xA)i + (yB

= (11

3)i + (10

= 8i + 6j

A (3, 4)

j
i

159

yA)j
4)j
Ans.

2.4 Position Vectors. Use in Defining Force Vectors. Example 1, page 2 of 2


3

Part b)
Draw the vector from B to A.
y

B (11, 10)

rBA
A (3, 4)

j
i

rBA = A's coordinates minus B's coordinates


= (3
= 8i

11)i + (4
6j

10)j
Ans.

Observation: Since rAB equals the negative value of rBA, it is easy to


make a mistake and get rBA when you intended to calculate rAB. The
rule to remember is "Subtract the coordinates of the tail of the vector."

160

2.4 Position Vectors. Use in Defining Force Vectors. Example 2, page 1 of 2


2. Determine the position vector directed from point B to
point A. Also determine the distance between B and A.
y
4m
2m
6m
1m
A

x
3m

7m

161

2.4 Position Vectors. Use in Defining Force Vectors. Example 2, page 2 of 2


1

Determine the coordinates of A and B.


y
4m
2m
6m
1m

A
( 4 m, 2 m, 7 m)

x
3m

rBA

7m

(6 m, 3 m, 1 m)
2

Draw the vector from B to A.

Subtract tail coordinates (point B) from head coordinates (point A).

rBA = ( 4 m

6 m)i + [2 m

= { 10i + 5j + 8k} m

Distance between B and A = magnitude of rBA


Three-dimensional
Pythagorean theorem

( 10)2 + (5)2 + (8)2

= 13.75 m

Ans.

162

( 3 m)]j + [7 m

( 1 m)]k
Ans.

2.4 Position Vectors. Use in Defining Force Vectors. Example 3, page 1 of 4


3. Determine a) the position vector rAB from A to B, b) unit vector u in
the same direction, and c) the direction cosines of rAB.
y
B
8m
2m

30

3m
x
4m
50
z
1

Determine coordinates of A and B.


B

Coordinates of B: ( 3.064 m, 6.928 m, 2.571 m)

Coordinates of A: (4 m, 2 m, 3 m)

y
8m

30

(8 m) cos 30 = 6.928 m

A
2m

3 (8 m) sin 30 = 4 m

3m
4m
5 (4 m) cos 50 = 2.571 m
50
z

(4 m) sin 50 = 3.064 m

163

2.4 Position Vectors. Use in Defining Force Vectors. Example 3, page 2 of 4


y

B ( 3.064 m, 6.928 m, 2.571 m)

Draw rAB

A (4 m, 2 m, 3 m)

x
z

Determine the components of rAB by subtracting tail coordinates (point A)


from head coordinates (point B):
point A

rAB = ( 3.064 m
point B

4 m)i + (6.928 m

= { 7.064i + 4.928j + 5.571k} m

2 m)j + [2.571 m
(1)

( 3 m)]k
Ans.

rAB = { 7.064i + 4.928j + 5.571k} m


A

10 Next, determine the unit vector in the same direction as rAB.


The best way is to use a calculator with a built-in function for
calculating unit vectors. You simply input rAB and the
calculator gives you back the components of the unit vector.

164

2.4 Position Vectors. Use in Defining Force Vectors. Example 3, page 3 of 4


11 If you do not have such a calculator, proceed as follows:
Calculate rAB, the length of rAB.
rAB =

( 7.064)2 + (4.928)2 + (5.571)2

= 10.26 m

(2)

rAB

uAB
A

12 Divide by rAB to get the unit vector in the same direction.


rAB
uAB = r
AB
=

{ 7.064i + 4.928j + 5.571k} m


10.26 m
z

5.571 m
7.064 m
4.928 m
= { 10.26 m i + 10.26 m j + 10.26 m k
= 0.689i + 0.480j + 0.543k

(3)

13 Finally, to determine the directions cosines, recall the


definition of the x direction cosine of rAB:
Ans.
x component of rAB
cos =
magnitude of rAB
7.064 m
=
10.26 m

Eq. 1
Eq. 2

= 0.689
But this is the x component of the unit vector, by Eq. 3:

uAB = 0.689i + 0.480j + 0.543k

165

(3)

2.4 Position Vectors. Use in Defining Force Vectors. Example 3, page 4 of 4


14 Thus in general the components of a unit vector are also the direction
cosines of any vector pointing in the same direction as the unit vector.
For our particular problem, since

uAB = 0.689i + 0.480j + 0.543k,


the direction cosines of rAB are
cos

= 0.689

Ans.

cos

= +0.480

Ans.

cos = +0.543

Ans.

Obviously, we could use these equations to solve for the coordinate


direction angles , , and .

166

2.4 Position Vectors. Use in Defining Force Vectors. Example 4, page 1 of 5


4. The radio tower is supported by three guy wires. If the tension in
wire CD is 1.6 kN, determine the components of the force from CD
acting on the tower at D. Also determine the coordinate direction
angles of the force vector.
y
D
A
30 m

x
6m
C

4m
B
z

167

2.4 Position Vectors. Use in Defining Force Vectors. Example 4, page 2 of 5


1

Since the force in the wire is tension,


the force from the wire acting on the
tower at D is directed from D to C .

y
D

F = 1.6 kN
2

Draw the position vector in the


same direction as the force.

30 m

rDC
x
6m
C

3 Determine the coordinates

4m
B

y
D (0, 30 m, 0)

z
F = 1.6 kN

30 m

rDC
x
6m
C

4m
B

( 4 m, 0, 6 m)
z

168

2.4 Position Vectors. Use in Defining Force Vectors. Example 4, page 3 of 5


y
D (0, 30 m, 0)
F = 1.6 kN
A
30 m

rDC
x
6m
C

4m
B

( 4 m, 0, 6 m)

4 Determine the components of rDC by subtracting tail


coordinates (point D) from head coordinates (point C):
point D

rDC = ( 4 m
point C

= { 4i

0)i + (0

30 m)j + (6 m

0)k

30j + 6k} m

169

2.4 Position Vectors. Use in Defining Force Vectors. Example 4, page 4 of 5


5

Determine the unit vector by dividing rDC by its length.

rDC = { 4i

30j + 6k} m

F = 1.6 kN
A

( 4)2 + ( 30)2 + (6)2

rDC =

y
(0, 30 m, 0)

30 m

u
= 30.85 m

rDC
uDC = r
DC
=

4i

rDC

= 0.130i

30j + 6k
30.85

6m
C

0.972j + 0.194k

4m
B

( 4 m, 0, 6 m)

The force vector has the same direction as u and also has a
magnitude of 1.6 kN, so

F = (1.6 kN)u
= (1.6 kN)( 0.130i
= { 0.207i

0.972j + 0.194k)

1.556j + 0.311k} kN

Ans.

170

2.4 Position Vectors. Use in Defining Force Vectors. Example 4, page 5 of 5


7

To find the coordinate direction angles of F, recall that the


components of a unit vector are the direction cosines of all vectors
pointing in the same direction as the unit vector. For our particular
problem, since u (= 0.130i 0.972j + 0.194k) and F are in the
same direction, we have for the direction angles , , and ,
cos

y
D

= 0.130

u
cos

= 0.972

cos = 0.194

rDC
x

Solving gives
= 97.5

Ans.

= 166.5

Ans.

Ans.

C
z

171

2.4 Position Vectors. Use in Defining Force Vectors. Example 5, page 1 of 4


5. The awning is supported by cables CD
and AB. If the tension in CD is 80 lb,
determine the components of the force
from the cable acting on the awning at C.

2 ft
B
D

C
3 ft
5 ft
x

172

2.4 Position Vectors. Use in Defining Force Vectors. Example 5, page 2 of 4


y
1

Since the cable is in tension, the cable force acting on


point C of the awning is directed from C to D. The
position vector rCD points in the same direction.
2 ft
B
D

rCD
F = 80 lb
C
3 ft
5 ft

173

2.4 Position Vectors. Use in Defining Force Vectors. Example 5, page 3 of 4


2

We cannot determine the coordinates of C and D because


we are given no distances from the coordinate axes.

However, we can still determine the


components of rCD because we are given the
x, y, and z differences in the coordinates of C
and D.

rCD = ( 5 ft)i + (3 ft)j + ( 2 ft)k


( 2 ft)k

5 Determine the unit vector in the


direction of rCD.

2 ft
B
D

rCD

(3 ft)j

r
uCD = r CD
CD

F = 80 lb

{ 5i + 3j

2k }

( 5)2 + (3)2 + ( 2)2

uCD

( 5 ft)i
3 ft

Length
of rCD

C
5 ft

x
4 The figure shows that rCD points in
the negative x direction, so the x
component is negative. Analogously,
the y component is positive and the z
component negative.

174

= 0.811i + 0.487j
0.324k

2.4 Position Vectors. Use in Defining Force Vectors. Example 5, page 4 of 4


6

The force vector has the same direction as the unit vector and
also has a magnitude of 80 lb, so

F = (80 lb)uCD
= (80 lb)( 0.811i + 0.487j
= { 64.9i + 38.9j

0.324k)

26.0k} lb

Ans.

175

2.4 Position Vectors. Use in Defining Force Vectors. Example 6, page 1 of 3


6. A cylindrical reactor vessel is being lifted by a crane connected to
three 2-m long cables. If the tension in cable AC is 2 kN, determine
the components of the force exerted by the cable on the vessel at C.
y
A
B
2m
120
D
z

Radius = 1.5 m

O
120

C
x

176

2.4 Position Vectors. Use in Defining Force Vectors. Example 6, page 2 of 3


y

1 Since the cable is in tension, the force FCA of the cable


acting on the vessel at C is directed from C to A. The
position vector rCA is also directed from C to A.

(0, yA, 0) A

rCA

B
2m
120
D

Radius = 1.5 m
FCA = 2 kN

120

C (xC, 0, zC)
3 Determine the coordinates of C.

x
Top view of vessel

Determine the coordinates of A


A
yA = (2 m)2 (1.5 m)2
yA

2m

Radius = 1.5 m C

B
120

90 = 30

(1.5 m) sin 30 = 0.75 m

y, A, O

= 1.323 m

zC = 0.75 m

120
(1.5 m) cos 30 = 1.299 m

Radius = 1.5 m

C
xC = 1.299 m
D
z

177

2.4 Position Vectors. Use in Defining Force Vectors. Example 6, page 3 of 3


y
(0, 1.323 m, 0) A
B
2m
120
D
z

O
120

rCA
Radius = 1.5 m
FCA = 2 kN
C (1.299 m, 0, 0.75 m)
x

Determine rCA by subtracting coordinates of C from A:

rCA = (0

1.299 m)i + (1.323 m

0)j + [0

( 0.75 m)]k

= { 1.299i + 1.323j + 0.75k} m


5

6 Divide by the magnitude of rCA


to obtain the unit vector.
(Actually we could have saved
work here by noting that the
magnitude of rCA is already
known: the length of cable CA
was given as 2 m.)

The force FCA is given by its magnitude, 2 kN, multiplied by


the unit vector in the direction of rCA:

FCA = (2 kN)

1.299i + 1.323j + 0.75k


2

)
2

( 1.299) + (1.323) + (0.75)


= { 1.299i + 1.323j + 0.75k} kN

178

Ans.

2.4 Position Vectors. Use in Defining Force Vectors. Example 7, page 1 of 3


7. Strut EF is employed as a temporary support to help retaining wall ABCD
resist soil pressure. If the compressive force in the strut is FFE = 3 kip,
determine the components of the force from the strut acting on the wall at E.
y

E
8 ft
Soil pressure

D
6 ft
F
3 ft

179

2.4 Position Vectors. Use in Defining Force Vectors. Example 7, page 2 of 3


y
Soil pressure
z

B
E
8 ft

FFE = 3 kip

rFE
D

C
6 ft
F
3 ft

Since the force in the strut is compressive, the force from the
strut acting on point E is directed from F to E. The position
vector rFE is then chosen in the same direction as the force.

180

2.4 Position Vectors. Use in Defining Force Vectors. Example 7, page 3 of 3


y
Soil pressure
z

B
E
8 ft
FFE = 3 kip
( 8 ft)k
(6 ft)j

rFE

2
C

We don't know the coordinates of E and F


so we can't subtract coordinates to get rFE.

6 ft
F (?, ?, ?)
( 3 ft)i
3 ft
3

Even though we don't know the coordinates


of points E and F, we do know the differences
in these coordinates, so we can obtain the
components of rFE directly from the
dimensions.

rFE = { 3i + 6j

Since the force has magnitude 3 kip and direction given by the
unit vector in direction of rFE, we have:

8k} ft

FFE = (3 kip)

3 i + 6j

( 3)2 + (6)2 + ( 8)2

= { 0.862i + 1.724j

181

8k

2.299k} kip

Ans.

2.4 Position Vectors. Use in Defining Force Vectors. Example 8, page 1 of 5


8. If the axial compression forces in the legs of the tripod cooking-stand
are denoted FA, FB, and FC respectively, determine the resultant force
acting on the connection at D in terms of FA, FB, and FC.
y
D

25 in.
C
18 in.
x

A
18 in.
14 in.

14 in.

182

2.4 Position Vectors. Use in Defining Force Vectors. Example 8, page 2 of 5


1

Because the forces in the legs are compressive, the forces from
the legs acting on the connection at D are directed towards D.
y
D
FC
FA

rCD

FB
25 in.

rAD

C
18 in.
x

14 in.

rBD
14 in.

18 in.
B

z
2

Draw the position vectors in the same direction as the forces.

183

2.4 Position Vectors. Use in Defining Force Vectors. Example 8, page 3 of 5


3

Determine the coordinates of the ends of the position vectors.


y
D (0, 25 in., 0)
FC
FA

rCD

FB
25 in.

rAD
18 in.
A
( 14 in., 0, 0)

14 in.

rBD
14 in.

18 in.
B (14 in., 0, 18 in.)

184

(14 in., 0, 18 in.)

2.4 Position Vectors. Use in Defining Force Vectors. Example 8, page 4 of 5


4

Subtract coordinates to get position vectors.


5

y
D (0, 25 in., 0)

rCD = (0

14 in.)i + (25 in.

0)j + [0

( 18 in.)]k

= { 14i + 25j + 18k} in.

FC
FA

rCD

FB
25 in.

rAD

(14 in., 0, 18 in.)

18 in.
A
( 14 in., 0, 0)

14 in.

rBD
14 in.

18 in.
B (14 in., 0, 18 in.)

z
7

rAD = [0

( 14 in.)]i + (25 in.

0)j + (0

0)k

= {14i + 25j} in.

185

x
6 rBD = (0

14 in.)i + (25 in.

= { 14i + 25j

18k} in.

0)j + [0

18 in.)]k

2.4 Position Vectors. Use in Defining Force Vectors. Example 8, page 5 of 5


8

Force vectors = magnitude

FA = FA

unit vector

14i + 25j

Resultant = sum of forces

R = FA + FB + FC

(14) + (25)

= 0.489 FAi + 0.873 FAj


= 0.489 FAi + 0.873 FAj

FB = FB

14i + 25j

0.414 FBi + 0.739 FBj

18k

0.414 FCi + 0.739 FCj + 0.532 FCk

( 14) + (25) + ( 18)


= 0.414 FBi + 0.739 FBj

FC = FC

0.532 FBk

14i + 25j + 18k

)
2

( 14) + (25) + (18)

= 0.414 FCi + 0.739 FCj + 0.532 FCk

10 Re-arrange by combining coefficients of i, then of j, and then of k:

R = (0.489 FA

0.414 FB

0.414 FC)i

+ (0.873 FA + 0.739 FB + 0.739 FC)j


+ ( 0.532 FB + 0.532 FC)k

0.532 FBk

Ans.

186

2.4 Position Vectors. Use in Defining Force Vectors. Example 9, page 1 of 5


9. The 20-m boom DCE is supported by a ball-and-socket joint at D and two
cables attached at the midpoint C of the boom. If the tension in each cable is
500 N and the coordinates direction angles of the boom are 60, 45, and 60
as shown, determine the resultant of the cable forces acting on the boom.
y
3m

3m

B
20 m

C
12 m
45
60

60

8m

x
8m

187

2.4 Position Vectors. Use in Defining Force Vectors. Example 9, page 2 of 5


1 To compute the resultant of the cable forces, we need to
express them in rectangular components. But to do that,
we first need to obtain the coordinates of point C.

To obtain the coordinates of C, we


can use the vector sum equation:

rC = rD + rDC

(1)

45

From the figure, we see

rD = {8i + 8k} m

rC
rDC
rD
60

60

x
8m
4

8m

(2)

To obtain rDC, recall the definition of


coordinate direction angles, for example, :
cos

188

x component of rDC
magnitude of rDC

2.4 Position Vectors. Use in Defining Force Vectors. Example 9, page 3 of 5


5

Solving the equation for the x component of rDC gives


x component of rDC = cos 60

(magnitude of rDC)

or,

rDCx = 5 m

(3)

Similarly

rDCy = cos 45

(4)
(10 m)

=5m

(Divide by 2 because point


C is the midpoint of the
20-m boom)

(10 m)

= 7.071 m

rDCz = cos 60

20 m
= 10 m,
2

(5)

Eqs. 2-5 can now be used in Eq. 1:

rC = rD + rDC
= {8i + 8k} m + {5i + 7.071j + 5k} m
= {13i + 7.071j + 13k} m

(6)

189

2.4 Position Vectors. Use in Defining Force Vectors. Example 9, page 4 of 5


7

Since the cables are in tension, the cable forces acting on the boom
are directed away from C. Position vectors rCA and rCB are
similarly drawn away from C towards the supports at A and B.
y
3m
(0, 12 m, 3 m)
A

3m

FCB = 500 N

B
rCB

rCA

9
8

The coordinates of C are the


components of rC, Eq. 6.

Determine the coordinates of A.

FCA = 500 N

(13 m, 7.071 m, 13 m)

12 m
x

10 To obtain rCA, subtract tail coordinates from head coordinates:

rCA = (0

13 m)i + (12 m

= { 13i + 4.929j

190

7.071 m)j + (3 m

10k} m

13 m)k

2.4 Position Vectors. Use in Defining Force Vectors. Example 9, page 5 of 5


11 The force in cable CA is then

FCA = (500 N)

13i + 4.929j

10k
2

12 To obtain the components of FCB, first determine the


coordinates of B. Then subtract coordinates of C from
coordinates of B to get rCB:

)
2

( 13) + (4.929) + ( 10)


= { 379.5i + 143.9j

(0, 12 m, 3 m)
A

13 m)i + (12 m

= { 10i + 4.929j

y
3m

rCB = (3m

292.0k} N

3m

(3 m, 12 m, 0)

13 m)k

13k} N

and so

B F = 500 N
CB
rCB

rCA

7.071 m)j + (0

FCB = (500 N)

10i + 4.929j

13k
2

)
2

( 10) + (4.929) + ( 13)


= { 292.0i + 143.9j

379.5k} N

(8)

FCA = 500 N

(13 m, 7.071 m, 13 m)

12 m

13 From Eqs. 7 and 8, the resultant of the cable forces acting


at C is

R = FCA + FCB
x

= { 379.5i + 143.9j
+ { 292.0i + 143.9j

= { 672i + 288j

292.0k} N
379.5k} N

672k} N

D
14 Equal, as we would expect from symmetry.

191

Ans.

2.4 Position Vectors. Use in Defining Force Vectors. Example 10, page 1 of 8
y

10. A 12-ft wide movie screen is suspended in a


corner of a room by four wires. The ends C and F of
the screen are located the same distance, L, from the
corner. If the tension in wires DO and DA are 12 lb
and 8 lb respectively, determine the resultant force
from the wires acting on handle D of the screen.

O
L

A
z

2 ft

C
4 ft

192

4 ft

F
4 ft

2 ft

2.4 Position Vectors. Use in Defining Force Vectors. Example 10, page 2 of 8
y

1 To compute the resultant of the forces, we


must express them in rectangular components.
But to do that, we first have to find the
coordinates of point D.

2 To obtain the coordinates of point D,


use the vector sum equation

rOD = rOC + rCD

rOC
A
z

2 ft
C

rOD
rCD

4 ft

4 ft

193

F
4 ft

2 ft

(1)

2.4 Position Vectors. Use in Defining Force Vectors. Example 10, page 3 of 8
3

First, locate point C, using a view


looking down on the xz plane:

O, y

B, F x
O

45
L

45

4 ft = 12 ft

rOC
A

A, C

z
4

2 ft

rCD
4 ft

Since L = (12 ft) cos 45 = 8.485 ft,

rOC = { 2j + 8.485k} ft

rOD

(2)

194

D
4 ft

F
4 ft

2 ft

2.4 Position Vectors. Use in Defining Force Vectors. Example 10, page 4 of 8
5

To find the components of rCD, note that it is has magnitude 4 ft,


is parallel to the xz plane (so has no y component) and makes a
45 angle with the z axis. Thus

rCD = (4 ft)(cos 45i

sin 45k)

O
L

= {2.828i

2.828k} ft

(3)

rOC
A
O, y

B, F x
45

2 ft
C

rOD
rCD
4 ft

45
D

rCD
A, C
z
View looking down on the xz plane

195

2 ft

E
4 ft

4 ft

2.4 Position Vectors. Use in Defining Force Vectors. Example 10, page 5 of 8
6

Now, determine rOD from Eq. 1:

rOD = rOC + rCD

(Eq. 1 repeated)

by Eq. 2

by Eq. 3

O
8.485 ft

8.485 ft

= { 2j + 8.485k} ft
+ {2.828i 2.828k} ft
= {2.828i

2j + 5.657k} ft

rOC

(4)

rOD

A
z

2 ft

rCD

C
4 ft

196

4 ft

4 ft

2 ft

2.4 Position Vectors. Use in Defining Force Vectors. Example 10, page 6 of 8
y

rDO

FDO

A
z

2 ft
C

r
FDO = (12 lb) r DO
DO

B
E

D
4 ft

Next express the forces in component form. Begin by


showing the force FDO directed away from the
connection D, because the cable DO is in tension.
FDO can be expressed in terms of a unit vector parallel
to the position vector rDO. The position vector rDO
has the opposite sense of the position vector rOD, so
rDO = rOD. Thus, taking into account that the
magnitude of FDO was given as 12 lb, we have

4 ft

2 ft

r
= (12 lb) r OD
OD
by Eq. 4

4 ft
(12 lb)

(2.828i

2j + 5.657k)

(2.828)2 + ( 2)2 + (5.657)2


= { 5.116i + 3.618j

197

10.234k} lb

(5)

2.4 Position Vectors. Use in Defining Force Vectors. Example 10, page 7 of 8
y

Coordinates of point
A: (0, 0, 8.485 ft)
O

Next find the components of the force FDA of


cable DA acting on handle D. Since FDA is in
the same direction as the position vector rDA,
we first find the components of rDA:

8.485 ft

rDA= coordinates of point A


coordinates of point D
by Eq. 4

rDA
A
z

FDA

2 ft

C
4 ft

4 ft

F
4 ft

= {8.485k}
2 ft

{2.828i

2j + 5.657k}

= { 2.828i + 2j + 2.828k} ft

FDA can be expressed in terms of a unit vector


parallel to the position vector rDA. Thus,
taking into account that the magnitude of FDA
was given as 8 lb, we have
rDA
FDA = (8 lb) rDA
(8 lb)

2.828i + 2j + 2.828k
( 2.828)2 + (2)2 + (2.828)2

= { 5.059i + 3.578j + 5.059k}

198

(6)

2.4 Position Vectors. Use in Defining Force Vectors. Example 10, page 8 of 8
9

The resultant is the sum of the forces:

R = FDA + FDO
by Eq. 6

by Eq. 5

= { 5.059i + 3.578j + 5.059k} lb + { 5.116i + 3.618j


= { 10.18i + 7.20j

5.17k} lb

10.234k} lb
Ans.

199

2.5 Applications of Dot Products

200

2.5 Applications of Dot Products Procedures and Strategies, page 1 of 1


Procedures and Strategies for Solving Problems Involving
Application of Dot Products

A
B

1. Compute the angle between two vectors by using the


formula
cos-1

AB
AB

Original position

Position after B has been


moved

is the angle between the vectors when they are placed


tail-to-tail. The dot product is calculated by using
rectangular components:

A B = AxBx + AyBy + AzBz


u

2. Compute the projection, say, Fu, of a force F in a


direction specified by a unit vector u by using the formula

F u = (F)(1) cos
Fu = F u
A vector representation of the projection is given by Fuu.

201

= Fu

2.5 Applications of Dot Products Problem Statement for Example 1


1. Two lines intersect in the plane as shown.
Determine a) the angle , and b) the angle .
y

B (4 m, 12 m)
A (9 m, 11 m)

C (5 m, 6 m)

202

2.5 Applications of Dot Products Problem Statement for Example 2


2. A billboard is braced in back by struts AB and AC.
Determine the angle between the struts.
y

10 m
4m

C
z

7m
A

3m
x

203

2.5 Applications of Dot Products Problem Statement for Example 3


3. A bin is constructed by cutting off the corner of a box. Top
ABC is glued to the box, and lid ADC pivots about hinge AC. To
make the lid, we need to know the values of the angles at A, D,
and C. Determine these angles.
y
B
200 mm
C
hinge

125 mm
125 mm
30 mm

x
180 mm

204

2.5 Applications of Dot Products Problem Statement for Example 4


4. Determine the angle

between the forces.

F1 = 800 N

70
F2 = 300 N
(7 m, 3 m, 6 m)
25
z
x

205

2.5 Applications of Dot Products Problem Statement for Example 5


5. A crate is suspended from supports at B, C, and D. The tension in cable AB is
known to be F = 800 N. Determine
a) the angle between cables AB and AC;
b) the component FAC of the 800-N force that acts in the direction of cable AC;
c) the component FAD of the 800-N force that acts in the direction of cable AD; and
d) the component of the 800-N force that acts in the vertical direction.
y

C
4m

5m

3.5 m

7m
B
3m

2m
x
9m
F = 800 N
A

206

2.5 Applications of Dot Products Problem Statement for Example 6


6. End B of the beam shown is subjected to an upward force F = 4 kN. Determine the
components of F parallel and perpendicular to the long axis of the beam, AB.
y

F = {4j} kN

B
5m

A
z
6m
2m

207

2.5 Applications of Dot Products Problem Statement for Example 7


7. When a certain load P is applied to
the bar BCD, a tension of F = 60 N is
produced in cable DA. Determine the
components of F parallel and
perpendicular to segment CD. Neglect
the thickness of the bar.

y
200 mm
A

450 mm
B
F = 60 N
D
z
300 mm
400 mm

C
30

208

2.5 Applications of Dot Products Example 1, page 1 of 3


1. Two lines intersect in the plane as shown. Determine a) the angle , and b) the angle
.
y
B (4 m, 12 m)
A (9 m, 11 m)

C (5 m, 6 m)

x
1 Draw position vectors a and b.
y
B (4 m, 12 m)

2 Calculate components.

a = (9 m

5 m)i + (11 m

6 m)j

A (9 m, 11 m)
= {4i + 5j} m

b = (4 m
C (5 m, 6 m)

5 m)i + (12 m

= { i + 6j } m

209

(1)
6 m)j
(2)

2.5 Applications of Dot Products Example 1, page 2 of 3


3

Apply the definition of the vector dot product:

ab = ab cos

cos

(3)

ab = axbx + ayby

Angle between tails of vector a and b

Solving gives

Writing the dot product in terms of rectangular


components gives

Using this in the numerator of Eq. 4 gives

ab
ab

cos

axbx + ayby
ab

(5)

Recalling Eqs. 1 and 2,

a = {4i + 5j} m

(Eq. 1 repeated)

b = { i + 6j } m

(Eq. 2 repeated)

we can evaluate Eq. 5 as

B (4 m, 12 m)

cos

A (9 m, 11 m)

(4)( 1) + (5)(6)
42 + 52

( 1)2 + 62

Solving gives

= 48.1

= 48.1

Ans.

C (5 m, 6 m)
5

can now be calculated:


= 180

210

= 131.9

48.1
Ans.

2.5 Applications of Dot Products Example 1, page 3 of 3


6 We could have used the dot product to calculate directly, but to do
so, we must define a new vector, a'
a. This is because the dot
product formula gives us the angle between the tails of the vectors:
cos

(a')b
ab

(6)
y

Using

a' = a,
= { 4i

B (4 m, 12 m)

A (9 m, 11 m)
= 48.1

5j } m

and

C (5 m, 6 m)

b = { i + 6j} m
in Eq. 6 gives
cos

)( 1) + (
2

( 5) + ( 4)

a' = a

)(6)
2

( 1) + 6

Solving gives
= 131.9 as before.

211

2.5 Applications of Dot Products Example 2, page 1 of 3


2. A billboard is braced in back by struts AB and AC.
Determine the angle between the struts.
y

10 m
4m

C
z

7m
A

3m
x

212

2.5 Applications of Dot Products Example 2, page 2 of 3


1

Introduce position vectors rAC and rAB.

Determine their rectangular components.

rAB = (0

7 m)i + (4 m

= { 7i + 4j

rAC = (10 m
rAB

= {3i + 4j

10 m

4m

rAC

z
7m
A

3m
x

213

0)j + (0

3 m)k

3k} m

7 m)i + (4 m
3k} m

0)j + (0

3 m)k

2.5 Applications of Dot Products Example 2, page 3 of 3


3 Use the vector dot product equation:
cos

=
=

4 Multiply x component by x
component, y by y, and z by z.

rACrAB
rAC rAB
{3i + 4j

3k}{ 7i + 4j

3k}

32 + 42 + ( 3)2 ( 7)2 + 42 + ( 3)2


(3)( 7) + (4)(4) + ( 3)( 3)
34

74

= 0.0797
5 Solving cos

= 0.0797 gives

= 85.4

Ans.

6 This is a tedious and error-prone calculation. A much better


approach is to use a calculator with built-in functions for
evaluating the dot product and magnitude (called the "norm"
function in some calculators). If such a calculator is available, all
you need to do is enter once the components for rAC and the
components for rAB. The built-in functions perform all the
arithmetic. As a result, typical errors such as missing a minus sign
or mis-copying a number from one line to the next are avoided.

214

2.5 Applications of Dot Products Example 3, page 1 of 3


3. A bin is constructed by cutting off the corner of a box. Top
ABC is glued to the box, and lid ADC pivots about hinge AC. To
make the lid, we need to know the values of the angles at A, D,
and C. Determine these angles.

y
B
200 mm
C
Hinge

125 mm
125 mm

30 mm

Introduce position vectors rDA and rDC

x
180 mm

B
200 mm
C
A
125 mm

rDC

rDA

125 mm

Determine their rectangular components.

rDA = (0
D

30 mm)j

= { 180i + 95j} mm

30 mm
x

180 mm

180 mm)i + (125 mm

rDC = (125 mm

= {95j

215

30 mm)j + (0

200k} mm

(1)
200 mm)k
(2)

2.5 Applications of Dot Products Example 3, page 2 of 3


3 Use the vector dot product equation:
cos

5 Next determine the angle at A. Begin by


introducing position vectors rAC and rAD.
y

rDArDC
rDA rDC
{ 180i + 95j}{95j

200k}

200 mm

( 180)2 + 952 952 + ( 200)2

rAC

( 180)(0) + (95)(95) + (0)( 200)


(203.5)(221.4)
= 0.2003
=

= 78.45

125 mm

125 mm

rAD

4 Solving gives
D

(3)

30 mm

Ans.

x
180 mm

= 180
6 Note that we use vector rAD,
not rDA, because the dot
product formula gives the
angle between the tails of
the vectors. If we use rDA
and rAC, we would get the
supplement of A, not A.

rAC
A

rDA

216

2.5 Applications of Dot Products Example 3, page 3 of 3


7

Determine the rectangular components of rAC and rAD.

rAC = (180 mm
= {180i

0)i + (0

200k} mm

8 Use the vector dot product equation:

200 mm)k

cos

rACrAD
rAC rAD

(4)

{180i

(180)(180) + (0)( 95) + ( 200)(0)


(269.1)(203.5)
= 0.5917

= { 180i + 95j} mm
95j} mm

95j}

1802 + ( 200)2 1802 + ( 95)2

rAD = rDA

= {180i

200k}{180i

=
(5)
Solving gives
A

= 53.72

(6)

Ans.

Finally, we could compute C, the angle at C, by


using position vectors and the dot product, but it's
easier to use the fact that the angles of a triangle add
to 180:
A

= 180
53.72 by Eq. 6
78.45 by Eq. 3

or
C

= 180
= 47.8

217

Ans.

2.5 Applications of Dot Products Example 4, page 1 of 4


4. Determine the angle

between the forces.

y
F1 = 800 N

70
F2 = 300 N
(7 m, 3 m, 6 m)
25
z
x

218

2.5 Applications of Dot Products Example 4, page 2 of 4


1 Determine the components of F1.
y
F1 = 800 N

2 F1y = (800 N) cos 70


= 273.6 N

70
x

(800 N) sin 70 = 751.8 N


25

5 F1z = (751.8 N) cos 25

= 681.4 N

4 F1x = (751.8 N) sin 25


= 317.7 N

6 In component form

F1 = { 317.7i + 273.6j + 681.4k} N

219

(1)

2.5 Applications of Dot Products Example 4, page 3 of 4


7 Determine the components of F2.

Using Eqs. 1 and 2 (repeated here),

y
F2 = 300 N

A(7 m, 3 m, 6 m)

F1 = { 317.7i + 273.6j + 681.4k} N

(1)

F2 = {216.6i + 92.8j + 185.7k} N

(2)

we can evaluate

cos

from the vector dot product equation

F1F2
F1 F2

(3)

The formula for the magnitude of F1, appearing in the denominator of


Eq. 3 is
x

8 F2 = (300 N)

unit vector pointing from O to A.

= (300 N)

{7i + 3j
2

( 317.7)2 + (273.6)2 + (681.4)2

F1 =

but we don't have to evaluate this expression because we were initially


given F1 = 800 N (see steps 2 - 6 above). Similarly, we were initially
given F2 = 300 N. Using these observations together with Eqs. 1 and 2
in Eq. 3 gives

6k }
cos

7 +3 +6

= {216.6i + 92.8j + 185.7k} N

(2)

F1F2
F1 F2

( 317.7)(216.6) + (273.6)(92.8) + (681.4)(185.7)


(800)(300)

Solving gives

220

= 69.7

Ans.

2.5 Applications of Dot Products Example 4, page 4 of 4


10 Observation: Eq. 3 can be rearranged to give
cos

F1F2
F1 F2

=(

F
F1
)( 2 )
F1
F2

= (unit vector in F1 direction)(unit vector in F2 direction)


That is, the magnitudes of the forces, 800 N and 300 N, divide out.
We could have saved some multiplications by simply replacing "800
N" by "1 N" and "300 N" by "1 N" at the beginning of the problem.

221

2.5 Applications of Dot Products Example 5, page 1 of 7


5. A crate is suspended from supports at B, C, and D. The tension in cable AB is
known to be F = 800 N. Determine
a) the angle between cables AB and AC;
b) the component FAC of the 800-N force that acts in the direction of cable AC;
c) the component FAD of the 800-N force that acts in the direction of cable AD; and
d) the component of the 800-N force that acts in the vertical direction.
y

C
4m

5m

3.5 m

7m
B
3m

2m
x
9m
F = 800 N
A

222

2.5 Applications of Dot Products Example 5, page 2 of 7


1 Introduce position vectors rAB and rAC.

2 Determine their rectangular components

rAB = ( 3 m
C

rAC

0)k

= { 3i + 9j + 3.5k} m

rAC = (

0)i + [4 m

= {13j

(1)

( 9 m)]j + ( 5 m

0)k

5k } m

(2)

3 Use the vector dot product equation:

rAB

cos

BC

3m

=
z
x
=
9m

( 9 m)]j + (3.5 m

4m

5m

3.5 m

0)i + [0

rABrAC
rAB rAC
{ 3i + 9j + 3.5k}{13j

5k}

( 3)2 + 92 + 3.52 132 + ( 5)2


( 3)(0) + (9)(13) + (3.5)( 5)

F = 800 N

102.25

194

= 0.7065

A
4

Solving gives
BC

223

= 45.1

Ans.

(3)

2.5 Applications of Dot Products Example 5, page 3 of 7

4m

5 To calculate FAC, the component of F in the direction


of AC, consider the plane formed by AB and AC:
C

5m

3.5 m

7m
B
D

3m

F = 800 N

2m
z
x
FAC

FAC = (800 N) cos 45.1


= 565 N

F = 800 N
9m

BC

FAC

224

Ans.

= 45.1

2.5 Applications of Dot Products Example 5, page 4 of 7

6 To calculate FAD, the component of F in the direction


of AD, consider the plane formed by AB and AD:

4m

B
5m

3.5 m

D
7m

uAD
D

3m

F = 800 N

2m

uAD

F = 800 N

FAD
A

9m

BD

FAD
A

From the figure, we see that


FAD = F cos

225

BD

(4)

2.5 Applications of Dot Products Example 5, page 5 of 7


8 But we don't know BD. We can, however, still use Eq. 4 by
introducing the dot product:
FAD = F cos
=F

(4)
insert a factor of 1

BD

cos BD
magnitude of unit vector in AD direction

=F

uAD

= FuAD

cos BD
definition of dot product
(5)

= FxuADx + FyuADy + FzuADz

(6)

Thus we can calculate FAD, if we express F and uAD in


rectangular components.

226

2.5 Applications of Dot Products Example 5, page 6 of 7


9 Since F points from A to B, we can express it as

F = (800 N)
= (800 N)
C

4m

= (800 N)

unit vector pointing from A to B

rAB
rAB

by Eq. 1

3i + 9j + 3.5k
( 3)2 + 92 + 3.52

= { 237.3i + 712.0j + 276.9k} N

5m

3.5 m

7m

(6)

10 To obtain uAD in component form, introduce the


position vector from A to D,

rAB
3m

rAD

rAD = (7 m

= {7i

2m

uAD
x
F = 800 N
9m

9j

FAD
=

0)k

2k} m

rAD
rAD
7i + 9j

2k

72 + 92 + ( 2)2

= 0.605i + 0.777j

227

( 9)]j + ( 2 m

Then

uAD =
A

0)i + [0

0.173k

(7)

2.5 Applications of Dot Products Example 5, page 7 of 7


11 Substituting for F and uAD from Eqs. 6 and 7 in Eq. 5 gives
FAD = FxuADx + FyuADy + FzuADz

(6)

= ( 237.3)(0.605) + (712.0)(0.777) + (276.9)( 0.173)


= 362 N

Ans.

Finally, to find the vertical component of F, we can use the equation


Fvertical = Fuvertical
= [{ 237.3i + 712.0j + 276.9k} N]j
= 712 N

Ans.

Of course, we could have simply read this value directly from the j
component of F. The dot product equation was not needed.
Nevertheless, it is worthwhile to confirm that the general principle
component = F(unit vector in direction of component)
gives the x, y, and z components of F when the unit vectors are i, j,
and k respectively.

228

The unit vector in the vertical


direction is the base vector, j.

2.5 Applications of Dot Products Example 6, page 1 of 4


6. End B of the beam shown is subjected to an upward force F = 4 kN. Determine the
components of F parallel and perpendicular to the long axis of the beam, AB.
y

F = {4j} kN

B
5m

A
z
6m
2m

229

2.5 Applications of Dot Products Example 6, page 2 of 4


1 Resolve F into components parallel (F| |) and
perpendicular (F ) to the AB direction.

F = {4 j} kN
2 Introduce a unit vector pointing in the direction
from A to B.

3 Then F| | (the component of F in


the direction of u) is given by

F| | = Fu

F| |

A
z

230

(1)

2.5 Applications of Dot Products Example 6, page 3 of 4


4

Express u in rectangular component form by first introducing


the position vector rAB.

5 Now find the components of u

u=

rAB

rAB
rAB
6i + 5j + 2k
62 + 52 + 22

B
= 0.744i + 0.620j + 0.248k

(2)

A
z

6 Eq. 1 now becomes


5m

F| | = Fu

6m
2m

(1)

F| | = [{4j} kN]{0.744i + 0.620j + 0.248k}


= (0)(0.744) + (4)(0.620) + (0)(0.248)
or
F| | = 2.480 kN

231

(3)

Ans.

2.5 Applications of Dot Products Example 6, page 4 of 4


7 F can be computed by the Pythagorean theorem.
F =
or

F2

8 Finally, we can express F and F| | in rectangular component


form if we note first that F| | is parallel to u so that
A scalar (number) multiplying a vector
u from Eq. 2
F| | = F| | u

F 2
||

2.480 kN by Eq. 3
4 kN (given)
F = 3.14 kN
Ans.

= (2.480 kN){0.744i + 0.620j + 0.248k}


= {1.845i + 1.538j + 0.615k} kN

(4)

Ans.

9 We get F from the vector sum

F = F| | + F
y

so

F =F

F| |

= {4j} kN
B

{1.845i + 1.538j + 0.615k} kN

F| |

= { 1.845i + 2.462j

F
A
z

232

F| |

0.615k} kN

Ans.

2.5 Applications of Dot Products Example 7, page 1 of 6


7. When a certain load P is applied to
the bar BCD, a tension of F = 60 N is
produced in cable DA. Determine the
components of F parallel and
perpendicular to segment CD. Neglect
the thickness of the bar.

y
200 mm
A

450 mm
B
F = 60 N
D
z
300 mm
400 mm

C
30

233

2.5 Applications of Dot Products Example 7, page 2 of 6


y
200 mm

450 mm

1 Resolve F into components parallel (F| |) and


perpendicular (F ) to the CD direction.

F = 60 N

2 Introduce a unit vector u pointing


in the direction from C to D.
u

F| |
z
300 mm
400 mm

D
3 Then F| | (the component in the
direction parallel to u) is given by

C
30

F| | = Fu
x

234

(1)

2.5 Applications of Dot Products Example 7, page 3 of 6


y

200 mm

A
4 To express F in rectangular components,
first introduce the position vector rDA.

rDA

450 mm
B
F = 60 N
D
z

6 Determine the y
coordinate of D
(neglect the thickness
of the member BCD):

300 mm
400 mm

C
30

(300 mm) sin 30 = 150 mm


5 Determine the x coordinate of D:

(300 mm) cos 30

400 mm + (300 mm) cos 30 = 659.8 mm

235

2.5 Applications of Dot Products Example 7, page 4 of 6


y
200 mm

7 Express rDA in rectangular components

rDA = (0

659.8 mm)i + (450 mm

150 mm)j + (200 mm

0)k

A
= { 659.8i + 300j + 200k} mm

rDA

8 The force F has magnitude 60 N and points from D to A so

F = (60 N)
450 mm

= (60 N)

659.8i + 300j + 200k


( 659.8)2 + 3002 + 2002

= { 52.65i + 23.94j + 15.96k} N

F = 60 N

rCD

D (659.8 mm, 150 mm, 0)

300 mm
400 mm

rDA
rDA

9 To express the unit vector u in rectangular


components, first introduce the position vector
rCD from C to D:

C
30
x

rCD = (659.8 mm 400 mm)i


+ (150 mm 0)j + (0 0)k
= {259.8i + 150j} mm

236

(2)

2.5 Applications of Dot Products Example 7, page 5 of 6


10 Now find the components of u

u=
=

rCD
rCD
259.8i + 150j
259.82 + 1502

= 0.866i + 0.500j

(3)

11 Eq. 1 now becomes


F| | = Fu

by Eq. 3

by Eq. 2
F| | = 33.63 N

F| | = { 52.65i + 23.94j + 15.96k}{0.866i + 0.500j}


F = 60 N
= 33.63 N

(4)

Ans.
12 The magnitude of F can be computed
by the Pythagorean theorem.

Minus sign means F| | is in the


opposite direction to u.

F =

602

= 49.7 N

237

33.632
Ans.

2.5 Applications of Dot Products Example 7, page 6 of 6


y

13 We can express F| | and F in rectangular component


form, if we note first that F| | is parallel to u so that
by Eq. 3

F| | = F| | u

by Eq. 4
= ( 33.63 N){0.866i + 0.500j}

= { 29.12i

16.82j} N

(5)

Ans.

B
F

14 Finally we can get F from the vector sum

F| |
z
D

300 mm
P

F = F| | + F

u
Thus

F =F

F| |

= { 52.65i + 23.94j + 15.96k} N


{ 29.12i 16.82j} N

30
x

238

= { 23.5i + 40.8j + 16.0k} N

Ans.

3. Equilibrium of a Particle

239

3.1 Particles and Two-Dimensional Force Systems

240

3.1 Particles and Two-Dimensional Force Systems Procedures and Strategies, page 1 of 1
Procedures and Strategies for Solving Problems Involving Equilibrium
of a Particle in a Two-Dimensional Force System
1. Establish x and y coordinates.
2. Construct a free-body diagram showing and labeling all forces acting on
the particle, including the weight, if the weight or the mass is given.
Assign the sense of forces arbitrarily (If you guess wrong, the magnitude of
the force will be found to be negative, when you solve the equations).
3. Resolve each force into x and y components and set the sum of each
component to zero:
Fx = 0

Fy = 0

4. Solve the equations. Note that if a force is found to be negative, it has a


sense opposite to that shown on the free-body diagram.
5. If a spring is present, it exerts a force F = kx, where k is the spring
constant and x is the displacement of the end of the spring.

241

3.1 Particles and Two-Dimensional Force Systems Problem Statement for Example 1
1. Determine the tension in cables AB and BC.

30

90 lb

242

3.1 Particles and Two-Dimensional Force Systems Problem Statement for Example 2
2. The spring balance reads 500 N. Determine the tensions in cords AB and BD.

A
1.5 m
C
Spring
balance

2m

B
3m
D

2m

4m

243

3.1 Particles and Two-Dimensional Force Systems Problem Statement for Example 3
3. The 10-kg block is supported by two identical
springs. The unstretched length of each spring is
500 mm. Calculate the spring constant k.

40 40

B
600 mm

C
10 kg

244

3.1 Particles and Two-Dimensional Force Systems Problem Statement for Example 4
4. Determine the value of angle and the
tension in the cables required for equilibrium.

D
C
B

A
10

20
150 lb

60 lb

245

3.1 Particles and Two-Dimensional Force Systems Problem Statement for Example 5
5. The collar at B weighs 5 lb and can slide freely
over the rod. If spring constant k is 2 lb/in.,
determine the unstretched length of the spring.

20
B

14 in.

35

246

3.1 Particles and Two-Dimensional Force Systems Problem Statement for Example 6
6. Two 2-g spheres are suspended by light cords and then given
electrostatic charges of opposite sign. Calculate the attractive
force, F, acting between the spheres.
A

30

30

247

3.1 Particles and Two-Dimensional Force Systems Problem Statement for Example 7
7. The figure shows top and side views of a control knob A that slides
smoothly in a slot cut in a panel and is attached to a spring underneath the
panel. Determine the force in the spring and the distance s, for which the
knob will be in equilibrium when a 0.5-lb force is applied. The
unstretched length of the spring is 4 in., and the weight of the knob is
negligible.

0.5 lb

0.5 lb

Top view

4 in.

k = 5 lb/in.

B
Side view

248

3.1 Particles in Two-Dimensional Force Systems Example 1, page 1 of 1


1. Determine the tension in cables AB and BC.

30

B
1

90 lb

Free-body diagram of connection B


y
FBC

Equilibrium equations for connection B


FAB

Fx = 0:

FAB cos 30 + FBC cos 60 = 0

Fy = 0: FAB sin 30 + FBC sin 60

30

180

30

30

90 lb = 0

Solving these equations gives


FAB = 45.0 lb

Ans.

FBC = 77.9 lb

Ans.

90 lb

249

90 = 60

3.1 Particles in Two-Dimensional Force Systems Example 2, page 1 of 2


2. The spring balance reads 500 N. Determine the tensions in cords AB and BD.

A
1.5 m
C
Spring
balance

2m

B
3m
D

2m

4m

250

3.1 Particles in Two-Dimensional Force Systems Example 2, page 2 of 2


Free body diagram of connection B
y

FAB

Force in spring =
tension of 500 N.

Geometry

2 m +1.5 m
= 3.5 m

3.5 m )
= 60.26
2m
2m
= tan-1 ( 4 m ) = 26.57
C

= tan-1 (

2m

FBD
2m
2

4m

Equilibrium equations for connection B


4

Fx = 0

FAB cos

Fy = 0: FAB sin

+ (500 N) cos
+ (500 N) sin

=0

(1)

FBD = 0

(2)

251

Substituting
60.26 and
Eqs. 1 and 2 and solving gives

= 26.57 in

FAB = 901 N

Ans.

FBD = 1006 N

Ans.

3.1 Particles in Two-Dimensional Force Systems Example 3, page 1 of 2


3. The 10-kg block is supported by two identical
springs. The unstretched length of each spring is
500 mm. Calculate the spring constant k.

Free body diagram of connection C


y

FAC
40
40 40

40

B
600 mm

40

40

10 kg

Weight = mg = (10 kg)(9.81 m/s2) = 98.1 N

2
+

FBC

252

Equilibrium equations for connection C:


Fx = 0

FAC cos 40 + FBC cos 40 = 0

Fy = 0: FAC sin 40 + FBC sin 40

98.1 N = 0

(1)
(2)

3.1 Particles in Two-Dimensional Force Systems Example 3, page 2 of 2


3

Geometry

Extension of spring = stretched length


= 933.4 mm

600 mm
40

unstretched length

500 mm

= 433.4 mm
5

FBC = force in spring BC


=k

(extension of spring)

=k

(433.4 mm)

C
mm
BC = 600
sin 40 = 933.4 mm
6

(3)

Solving Eqs. (1), (2), and (3) gives


FAC = 76.3 N = FBC
k = 0.176 N/mm = 0.176 kN/m

253

Ans.

3.1 Particles in Two-Dimensional Force Systems Example 4, page 1 of 2


4. Determine the value of angle and the
tension in the cables required for equilibrium.

Free body diagram of connection B


y

D
FBC
C
B

A
10

20

10

20

B
FAB

150 lb

10

60 lb
60 lb

2
+

Fx = 0:

FAB cos 10 + FBC cos 20 = 0

(1)

Equilibrium equations for connection B:

Fy = 0:

FAB sin 10 + FBC sin 20

(2)

60 lb = 0

Solving Eqs. 1 and 2 gives

254

FAB = 324.7 lb

Ans.

FBC = 340.3 lb

Ans.

3.1 Particles in Two-Dimensional Force Systems Example 4, page 2 of 2


Free body diagram of connection C
y

Equilibrium equations for connection C

4
+

Fx = 0:

(340.3 lb) cos 20 + FCD cos

Fy = 0:

(340.3 lb) sin 20 + FCD sin

FCD

=0

(5)

150 lb = 0

(6)

Eqs. 5 and 6 can be solved with a calculator that can handle two
simultaneous nonlinear equations.
20

20

x
Alternatively re-write Eqs. 5 and 6 as

FBC = 340.3 lb
150 lb

FCD cos

= (340.3) cos 20

(7)

FCD sin

= (340.3) sin 20 +150

(8)

Eliminate FCD by dividing Eq. 8 by Eq. 7:


FCD sin
FCD cos

(9)

(340.3) sin 20 + 150


(340.3) cos 20

tan
5

Solving Eq. 9 gives


= 39.8

Ans.

Using this result in Eq. 7 gives


FCD = 416 lb

255

Ans.

3.1 Particles in Two-Dimensional Force Systems Example 5, page 1 of 2


5. The collar at B weighs 5 lb and can slide freely
over the rod. If spring constant k is 2 lb/in.,
determine the unstretched length of the spring.

Free body diagram of collar B


y

20
2
B

Since the collar


can slide freely
over the rod, only
a normal force N
is present; no
friction force
(which would be
parallel to the rod)
is present.

14 in.

35

256

x
B

5 lb

Equilibrium equations for B:

Fspring

Fx = 0:

N sin

Fy = 0: N cos

+ Fspring cos
+ Fspring sin

=0
lb = 0

(1)
(2)

3.1 Particles in Two-Dimensional Force Systems Example 5, page 2 of 2


4

Geometry:

and

35

Geometry: stretched length, BC


C
D
35
125
35 = 125
stretched length = BC
6

D
180

20

= 35

20
20
35

Solving gives BC = 9.802 in.

55 35
35
7

Law of sines:
sin 125
sin 35
= BC
14 in.

= 20 + 35 = 55

B
A

14 in.

Using = 55 and = 35 in Eqs. 1 and 2 and solving


simultaneously gives

3.052 lb = (2 lb/in.)(9.802 in.


unstretched length).

N = 3.052 lb and Fspring = 3.052 lb

Solving gives

Fspring is related to the extension of the spring:


Fspring = k

(stretched length
unstretched length)

Substituting "stretched length" = BC = 9.802 in.,


k = 2 lb/in., and Fspring = 3.052 lb in Eq. 3 gives

unstretched length = 8.28 in.


(3)

257

Ans.

3.1 Particles in Two-Dimensional Force Systems Example 6, page 1 of 1


6. Two 2-g spheres are suspended by light cords and then given
electrostatic charges of opposite sign. Calculate the attractive
force, F, acting between the spheres.

30

30

Free body diagram of sphere B


y

FAB

30
F

30
B
B

Weight = mg = (2 g)(1 kg/1000 g)(9.81 m/s2)


= 0.01962 kgm/s2 = 0.01962 N

Equilibrium equations for sphere B


+

Fx = 0:

FAB cos 30 + F = 0

Fy = 0: FAB sin 30

(1)

0.01962 N = 0

(2)

Solving Eqs. 1 and 2 gives


FAB = 0.0392 N = 39.2 mN
F = 0.0340 N = 34.0 mN

258

Ans.

3.1 Particles in Two-Dimensional Force Systems Example 7, page 1 of 2


7. The figure shows top and side views of a control knob A that slides
smoothly in a slot cut in a panel and is attached to a spring underneath the
panel. Determine the force in the spring and the distance s, for which the
knob will be in equilibrium when a 0.5-lb force is applied. The
unstretched length of the spring is 4 in., and the weight of the knob is
negligible.

Free body diagram of control knob A


(Side view)
y

0.5 lb
A
A

0.5 lb

Fspring
2

Top view

4 in.

0.5 lb

k = 5 lb/in.

B
Side view

259

N
Since the knob "slides
smoothly", only a normal
force N is present; no
friction force is present.

Equilibrium Equations for control knob A:

Fx = 0

Fspring cos

Fy = 0:

Fspring sin

0.5 lb = 0
+N=0

(1)
(2)

3.1 Particles in Two-Dimensional Force Systems Example 7, page 2 of 2


4 Geometry

5 Substituting the "stretched length" expression given in Eq. 4


into the force-extension relationship for a spring gives

A
Fspring = k

4 in.

tan

4 in.
s

(stretched length

= (5 lb/in.)

(3)

4 in.
sin

unstretched length)
4 in.)

(5)

Recall that Eq. 1 is


Fspring cos

4 in.
sin

(Eq. 1 repeated)

Substituting the expression for Fspring from Eq. 5 into Eq. 1

stretched length of spring = AB


=

0.5 = 0

and multiplying through by sin

(4)

5(4

4 sin ) cos

gives

0.5 sin

=0

(6)

Eq. 6 is nonlinear and must be solved numerically by, for


example, using the solver on a calculator. The result is =
69.1. Using the value = 69.1 in Eqs. 2, 3, and 4 and then
solving gives
N = 1.31 lb

260

Fspring = 1.40 lb

Ans.

s = 1.53 in.

Ans.

3.2 Particles and Three-Dimensional Force Systems

261

3.2 Particles and Three-Dimensional Force Systems Procedures and Strategies, page 1 of 1
Procedures and Strategies for Solving Problems Involving
Equilibrium of a Particle in a Three-Dimensional Force System
1. Establish an x, y, and z coordinate system.
2. Construct a free-body diagram showing and labeling all forces acting on
the particle, including the weight, if the weight or the mass is given.
Assign the sense of forces arbitrarily (If you guess wrong, the magnitude of
the force will be found to be negative, when you solve the equations).
3. Resolve each force into x, y and z components and set the sum of each
component to zero:
Fx = 0

Fy = 0

Fz = 0

4. Solve the equations. Note that if a force is found to be negative, it has a


sense opposite to that shown on the free-body diagram.

262

3.2 Particles and Three-Dimensional Force Systems Problem Statement for Example 1
1. The 150 lb block is supported by pole AB and cables AC
and AD. The force in the pole can be considered to act
along the axis of the pole. Determine the forces in the
cables and the pole.
y

4 ft

5ft

C
D

A
8 ft
B
150 lb

263

3.2 Particles and Three-Dimensional Force Systems Problem Statement for Example 2
2. The chandelier has a mass of 10 kg and is
supported by three wires arranged as shown.
Determine the force in each wire.
y

A
300 mm

B
O

Radius = 100 mm

50
50
z

C
x

264

3.2 Particles and Three-Dimensional Force Systems Problem Statement for Example 3
3. The small 50-lb block rests on the smooth
inclined plane ACDE and is attached to a cord AB.
Determine the value of the applied force P
necessary to keep the block in equilibrium. P acts
parallel to the z axis.
C
y

cord
30

A
15

50 lb

z
E

265

3.2 Particles and Three-Dimensional Force Systems Problem Statement for Example 4
4. The 400-kg precast concrete section is being
hoisted by a vertical force T from a crane.
Determine the tension in cables AB, AC, and AD.
y

T
A
5m

3m
C
3m
z

1m
1m

266

3.2 Particles and Three-Dimensional Force Systems Problem Statement for Example 5
5. Members CA and DA
of the derrick exert
compressive forces along
their respective axes.
Determine the values of
these forces and the
tension in cable AB.

B
10 m
300 kg
5m
D

3m
z
3m

3m

267

3.2 Particles and Three-Dimensional Force Systems Problem Statement for Example 6
6. The 200-lb plate is supported by three cables tied to ring A,
upon which a 200-lb vertical force acts. Determine the tension
in each cable. The edges of the plate are parallel to the
coordinate axes, and the center of the plate at E lies directly
below A.
y

F = 200 lb
120

x
45

60

z
C

65
70
E
40
D
B

50

268

3.2 Particles and Three-Dimensional Force Systems Problem Statement for Example 7
7. The compressive force in pole AO acts along the axis
of the pole and has a magnitude of 900 lb. Determine the
tension in the guy wires AC, AB, and AD.
y
A
B
65

O
70

60
20

40

D
z

269

3.2 Particles in Three-Dimensional Force Systems Example 1, page 1 of 4


1. The 150 lb block is supported by pole AB and cables AC
and AD. The force in the pole can be considered to act
along the axis of the pole. Determine the forces in the
cables and the pole.
y

4 ft

5ft

C
D

A
8 ft
B
150 lb

270

3.2 Particles in Three-Dimensional Force Systems Example 1, page 2 of 4


1

Free body diagram of connection at A

FAC

FAD
A

4 ft

5 ft

FBA
150 lb

2
3

The forces in cables AC and AD are tensile so are


directed away from A.

C
D

Equilibrium equation:

FAC + FAD + FBA

{150j} lb = 0

(1)
8 ft

4 Express the forces in terms of rectangular


components.

B
Cable AC is parallel to the x axis so

FAC = FACi

(2)

The minus sign indicates that the force is in the


negative x direction. Similarly, cable AD is parallel
to the z axis so

FAD = FADk

(3)

271

150 lb

3.2 Particles in Three-Dimensional Force Systems Example 1, page 3 of 4


y

The components of FBA can be derived from a position


vector going from B to A.

rBA = coordinates of A
4 ft

5 ft

= {5i + 4k} ft

coordinates of B

{ 8j} ft

= {5i + 8j + 4k} ft

The magnitude of rBA is

C
D

rBA =

52 + 82 + 42

= 10.2470 ft

rBA

A
The unit vector in the direction of BA is

8 ft

uBA = rBA/rBA

B
150 lb

5i + 8j + 4k
10.247

= 0.4879i + 0.7807j + 0.3904k

272

3.2 Particles in Three-Dimensional Force Systems Example 1, page 4 of 4


6

Since the force FBA is in the same direction as uBA, we can write

FBA = FBAuBA
= FBA(0.4879i + 0.7807j + 0.3904k)
= 0.4879FBAi + 0.7807FBAj + 0.3904 FBAk

(4)

Substituting the component forms of the forces given by Eqs. 2, 3,


and 4 into Eq. 1 gives

FAC + FAD + FBA

{150j} lb = 0

(Eq. 1 repeated)

or
FACi

FADk + (0.4879FBAi + 0.7807FBAj +


0.3904FBAk) 150j = 0

Rearranging by collecting coefficients of i, j, and k gives


( FAC + 0.4879FBA)i + ( 150 lb + 0.7807FBA)j
+ ( FAD + 0.3904FBA)k = 0

273

7 Since each component must vanish independently,


we have
Fx = 0:

FAC + 0.4879FBA = 0

(5)

Fy = 0:

150 lb + 0.7807FBA = 0

(6)

Fz = 0:

FAD + 0.3904FBA = 0

(7)

Solving Eqs. 5, 6, and 7 simultaneously gives


FAC = 93.7 lb

Ans.

FAD = 75.0 lb

Ans.

FBA = 192.1 lb

Ans.

3.2 Particles in Three-Dimensional Force Systems Example 2, page 1 of 5


2. The chandelier has a mass of 10 kg and is
supported by three wires arranged as shown.
Determine the force in each wire.
y

A
300 mm

B
O

Radius = 100 mm

50
50
z

C
x

274

3.2 Particles in Three-Dimensional Force Systems Example 2, page 2 of 5


1

Free body diagram

Determine the rectangular components of FBA.

FBA = FBA sin j + FBA cos k

FBA

FDA

y
B

Weight

300 mm

FCA
x

FBA

B
100 mm

Weight = mg
5

Geometry

= (10 kg)(9.81 m/s )

= tan-1

= 98.1 N
2

The forces in the wires are all tensile, so they are directed
away from the body (the chandelier).

Equilibrium equations

FBA + FCA + FDA

{98.1j} N = 0

(1)

275

300 mm
= 71.5651
100 mm

(2)

3.2 Particles in Three-Dimensional Force Systems Example 2, page 3 of 5


6

Substituting

= 71.5651 in Eq. 2 gives

FBA = FBA sin 71.5651j + FBA cos 71.5651k


or
9

FBA = 0.9487FBAj + 0.3162FBAk

The vertical component is

(3)
FDAy = FDA sin 71.5651

y
or

A
7

Determine the components of FDA

FDAy = 0.9487FDA

(4)

FDA

= 71.5651
D

z
8

50

D
x

z
10 The component in the horizontal plane (the xz plane) is

The angle is the same as we computed before.

FDA cos 71.5651 = 0.3162FDA

276

3.2 Particles in Three-Dimensional Force Systems Example 2, page 4 of 5


11

12 Combining Eqs. 4, 5, and 6 gives

x and z components of FDA

FDA = 0.2422FDAi + 0.9487FDAj

0.2032FDAk

(7)

0.3162FDA
O

13 The easiest way to get FCA is to note that it differs from FDA only by a
change in sign of the x coordinate.

50

FCA = 0.2422FCAi + 0.9487FCAj

0.2032FCAk

(8)

D
y
z
FDAx = (0.3162FAD) sin 50 = 0.2422FDA

(5)

FDAz = (0.3162FAD) cos 50 = 0.2032FDA

(6)

FDA

FCA
z

277

3.2 Particles in Three-Dimensional Force Systems Example 2, page 5 of 5


14 Substituting the component form of the forces into the equilibrium equation gives

FBA + FCA + FDA

98.1j = 0

(Eq. 9 repeated)

or
(0.9487FBAj + 0.3162FBAk) + ( 0.2422FCAi + 0.9487FCAj 0.2032FCAk)
+ (0.2422FDAi + 0.9487FDAj 0.2032FDAk) {98.1j} N = 0

(10)

Rearranging by collecting coefficients of i, j, and k gives


( 0.2422FCA + 0.2422FDA)i + (0.9487FBA + 0.9487FCA + 0.9487FDA
98.1 N)j + (0.3162FBA 0.2032FCA 0.2032FDA)k = 0

(11)

Since each component must vanish independently, we have


Fx = 0:

0.2422FCA + 0.2422 FDA = 0

Fy = 0: 0.9487FBA + 0.9487FCA + 0.9487FDA


Fz = 0: 0.3162FBA

0.2032FCA

(12)
98.1 N = 0

0.2032FDA = 0

(13)
(14)

Solving Eqs. 12, 13, and 14 simultaneously gives


FBA = 40.5 N

Ans.

FCA = 31.5 N

Ans.

FDA = 31.5 N

Ans.

278

3.2 Particles in Three-Dimensional Force Systems Example 3, page 1 of 3


3. The small 50-lb block rests on the smooth
inclined plane ACDE and is attached to a cord AB.
Determine the value of the applied force P
necessary to keep the block in equilibrium. P acts
parallel to the z axis.
C
y

cord
30

50 lb

Free body diagram of block


y

15

W = weight of block

z
E

Tension in T
cord AB
P

Normal force from the


inclined plane. Since the
plane is smooth, no friction
forces are present.

Equilibrium equation:

F = 0: W + P + N + T = 0

279

(1)

3.2 Particles in Three-Dimensional Force Systems Example 3, page 2 of 3


3 We must express the forces in terms of
rectangular coordinates. Thus

W = {50j} lb

Compute the components of T.


C

Tz = T cos 30

(2)

P = Pk

(3)
30

The components of the force N


perpendicular to the plane can be
determined from the figure below.

Ty = (T sin 30) sin 15


15

N = (N cos 75)i + (N sin 75)j

(4)

T
B
D

T sin 30

N
Tx = (T sin 30) cos 15

N sin 75
y

Thus,

75

15

T = (T sin 30) cos 15i + (T sin 30) sin 15j


+ T cos 30k
(5)

15
x

N cos 75

280

3.2 Particles in Three-Dimensional Force Systems Example 3, page 3 of 3


6

Substituting the rectangular component forms of the forces in Eq. 1 gives


50j

Pk + (N cos 75)i + (N sin 75)j

(T sin 30 cos 15)i + (T sin 30 sin 15)j + (T cos 30)k = 0

(6)

Rearranging by collecting i, j, and k components gives


(N cos 75

T sin 30 cos 15)i + ( 50 + N sin 75 + T sin 30 sin 15)j + ( P + T cos 30)k = 0

(7)

Since each component must vanish independently we have


Fx = 0: N cos 75

T sin 30 cos 15 = 0

(8)

Fy = 0:

50 + N sin 75 + T sin 30 sin 15 = 0

(9)

Fz = 0:

P + T cos 30 = 0

(10)

Solving Eqs. 8, 9, and 10 simultaneously gives


N = 48.3 lb

Ans.

T = 25.9 lb

Ans.

P = 22.4 lb

Ans.

281

3.2 Particles in Three-Dimensional Force Systems Example 4, page 1 of 7


4. The 400-kg precast concrete section is being
hoisted by a vertical force T from a crane.
Determine the tension in cables AB, AC, and AD.
y

T
A
5m

3m
C
3m
z

1m
1m

282

3.2 Particles in Three-Dimensional Force Systems Example 4, page 2 of 7


Free body diagram of hook, cables, and concrete section.
y
T
A
5m

Fy = 0: T

3924 N = 0

Thus the vertical force from the crane


equals the weight being supported:

3m
C
z

Equilibrium equation

T = 3924 N

3m

1m
1m

Weight of concrete section = (400 kg)(9.81 m/s2 )


= 3924 N

283

3.2 Particles in Three-Dimensional Force Systems Example 4, page 3 of 7


4

Free body diagram of hook A


T = 3924 N

A
z

FAD

FAC
FAB

Equilibrium equation
{3924j} N + FAD + FAB + FAC = 0

(1)

284

3.2 Particles in Three-Dimensional Force Systems Example 4, page 4 of 7


6

Express the forces in terms of rectangular


components.

First let

rAD = position vector to D from A (the force acting on the hook at A


points to D from A)
= coordinates of D
= { 3i} m

coordinates of A

{5j} m

y
= 3i
A

uAD

5j

The length of rAD is


rAD =

3)2 + ( 5)2

= 5.8310 m

rAD

5m

The unit vector in the direction of AD is,


rAD
uAD = r
AD

D
3m

=
C

z
B

3i 5j
5.8310

= 0.5145i

285

0.8575j

3.2 Particles in Three-Dimensional Force Systems Example 4, page 5 of 7


8

Since the force FAD is in the same direction as uAD, we can write

FAD = FAD uAD


= FAD( 0.5145i
= 0.5145FADi

0.857j)
0.8575FADj

uAB

(2)

Next let

rAB = position vector to B from A


= {3i + k} m
= 3i

5m

{5j} m

rAB

5j + k

The length of rAB is


rAB =

3m

3)2 + ( 5)2 + (1)2

x
B 1m

= 5.9161 m
The unit vector in the direction of AB is

rAB
uAB = rAB
=

3i

5j + k
5.9160

= 0.5071i

0.8452j + 0.1690k

286

3.2 Particles in Three-Dimensional Force Systems Example 4, page 6 of 7


9 Thus,
y

FAB = FABuAB
= 0.5071FABi

0.8452FABj + 0.1690FABk

(3)

We can save work if we note that the position vector rAC differs
from rAB only by the sign of the z component:

rAB = 3i

5j + k

rAC = 3i

5j

rAC

5m

rAB

So the force FAC must also differ from FAB only by the sign of the z
component:

FAB = 0.5071FABi

0.8452FABj + 0.1690FABk

3m
z

FAC = 0.5071FACi

0.8452FACj

0.1690FACk

(4)

287

1m
x
B 1m

3.2 Particles in Three-Dimensional Force Systems Example 4, page 7 of 7


10 The equilibrium equation, Eq. 1, was
{3924j} N + FAD + FAB + FAC = 0

(Eq. 1 repeated)

Substituting the component forms of the forces given by Eqs. 2, 3, and 4 into Eq. 1 gives
3924j

0.5145FADi

0.8575FADj + 0.5071FABi

0.8452FABj + 0.1690FABk + 0.5071FACi

0.8452FACj

0.1690FACk = 0

Rearranging by collecting the i, j, and k components gives


( 0.5145FAD + 0.5071FAB + 0.5071FAC)i + (3924

0.8575FAD

Since each component must vanish independently, we have


Fx = 0:

0.5145FAD + 0.5071FAB + 0.5071FAC = 0

Fy = 0: 3924

0.8575FAD

Fz = 0: 0.1690FAB

0.8452FAB

0.8452FAC = 0

0.1690FAC = 0

(5)
(6)
(7)

Solving Eqs. 5, 6, and 7 simultaneously gives


FAB = 1.161 kN

Ans.

FAC = 1.161 kN

Ans.

FAD = 2.29 kN

Ans.

288

0.8452FAB

0.8452FAC)j + (0.1690FAB

0.1690FAC)k = 0

3.2 Particles in Three-Dimensional Force Systems Example 5, page 1 of 5


5. Members CA and DA
of the derrick exert
compressive forces along
their respective axes.
Determine the values of
these forces and the
tension in cable AB.

y
B

1
x

10 m

300 kg

FCA
(compression)
5m
D

Free body diagram of


connection at A.

FAB (tension)
FDA
(compression)

x
Weight = (300 kg)(9.81 m/s2) = 2943 N

3m
2

Equilibrium equation
{2943j} N + FAB + FCA + FDA = 0

3m

3m

289

(1)

3.2 Particles in Three-Dimensional Force Systems Example 5, page 2 of 5


A

uAB

5 Thus define
rAB = coordinates of B

= { 5k} m
= 10j

coordinates of A

{10j + 3k} m

8k

The length of rAB is

rAB

rAB =

10 m

( 10)2 + ( 8)2

= 12.8062 m
B

The unit vector in the direction of AB is


rAB
uAB = r
AB

5m

3m

10j 8k
= 12.8062

= 0.7809j

Express the forces in terms of rectangular components.

The components of FAB will be derived from the


position vector to B from A because the force FAB
points to B from A.

0.6247k

Since the force is in the same direction as uAB, we can


write

FAB = FAB uAB

290

= FAB( 0.7809j

0.6247k)

= 0.7809FABj

0.6247FABk

(2)

3.2 Particles in Three-Dimensional Force Systems Example 5, page 3 of 5


6
A

Next let

rCA = position vector to A from C


= coordinates of A
= {10j + 3k} m

coordinates of C

{ 3i} m

= 3i + 10j + 3k

rCA

The unit vector in the direction of rCA is,


10 m

uCA =

3i + 10j + 3k
3)2 + ( 10)2 + (3)2

uCA
= 0.2762i

3m
C

x
3m

0.9206j + 0.2762k

so the force FCA can be expressed as

FCA = FCA uCA


= 0.2762FCAi + 0.9206 FCAj + 0.2762FCAk

291

(3)

3.2 Particles in Three-Dimensional Force Systems Example 5, page 4 of 5


7

To determine the components of FDA, we can make use of the


observation that the position vectors rCA and rDA differ only by the
sign of the x component:

rCA = +3i + 10j + 3k


rDA

rDA = 3i + 10j + 3k

rCA

So the force FCA must also differ from FDA only by the sign of the x
component:

10 m

FCA = +0.2762FCAi + 0.9206FCAj + 0.2762FCAk


FDA = 0.2762FDAi + 0.9206FDAj + 0.2762FDAk

(4)

3m
3m

Substituting the component forms of the forces given by Eqs. 2, 3, and 4 into Eq. 1 gives
2943j + FAB + FCA + FDA = 0

(Eq. 1 repeated)

or
2943j + ( 0.7809FABj

0.6247FABk) + (0.2762FCAi + 0.9206FCAj + 0.2762FCAk)

+ ( 0.2762FDAi + 0.9206FDAj + 0.2762FDAk) = 0

292

x
D

C
z

3m

3.2 Particles in Three-Dimensional Force Systems Example 5, page 5 of 5


Rearranging by collecting the i, j, and k components gives
(0.2762FCA

0.2762FDA)i + ( 0.7809FAB

2943 + 0.9206FCA + 0.9206FDA)j

+ ( 0.6247FAB + 0.2762FCA + 0.2762FDA)k = 0


9

Since each component must vanish independently, we have


Fx = 0: 0.2762FCA

0.2762FDA = 0

(5)

Fy = 0:

0.7809FAB

2943 + 0.9206FCA + 0.9206FDA = 0

Fz = 0:

0.6247FAB + 0.2762FCA + 0.2762FDA = 0

(7)

Solving Eqs. 5, 6, and 7 simultaneously gives


FAB = 2261 N = 2.26 kN

Ans

FCA = 2558 N = 2.56 kN

Ans

FDA = 2558 N = 2.56 kN

Ans

(6)

293

3.2 Particles in Three-Dimensional Force Systems Example 6, page 1 of 7


6. The 200-lb plate is supported by three cables tied to ring A,
upon which a 200-lb vertical force acts. Determine the tension
in each cable. The edges of the plate are parallel to the
coordinate axes, and the center of the plate at E lies directly
below A.
y
Free body diagram of ring A

200 lb

F = 200 lb
y
120

x
z

45

FAD

FAB
FAC

60

Equilibrium equation

C
{200j} lb + FAB + FAC + FAD = 0

65
70
E
40
D
B

50

294

(1)

3.2 Particles in Three-Dimensional Force Systems Example 6, page 2 of 7


3
y

We must use the given angles to express the


forces in rectangular components. Begin with
the force cable AD exerts on ring A.
y

F = 200 lb
A

120

= 120

45

x
= 45

60
z
z

FAD

4 The rectangular components of FAD are

C
65

FAD = FAD cos

70

From the diagram,


E

i + FAD cos
= 45,

j + FAD cos k

= 120, and = 60, thus

FAD = FAD cos 45 i + FAD cos 120 j + FAD cos 60 k

40

= 0.7071FADi

D
B

= 60

50

295

0.5FADj + 0.5FADk

(2)

3.2 Particles in Three-Dimensional Force Systems Example 6, page 3 of 7


5

Next determine the rectangular


components of FAB.

The component parallel to the


horizontal plane is

FABxz = FAB cos 40

x
40

= 0.7660FAB

The component normal to the


horizontal plane is
FABy = FAB sin 40
= 0.6428FAB

FAB
E
40
B

50

296

3.2 Particles in Three-Dimensional Force Systems Example 6, page 4 of 7


8

The x and z components of FAB can be now be found:


FABx = 0.7660FAB cos 50 = 0.4924FAB
FABz = 0.7660FAB sin 50 = 0.5868FAB

9
y
A

50

The x, y, and z components of FAB are now known and so we can


write

FAB = 0.4924FABi

FABxz = 0.7660FAB

E
40
B

50

297

0.6428FABj

0.5868FABk

(3)

3.2 Particles in Three-Dimensional Force Systems Example 6, page 5 of 7


10 Next, determine the rectangular components of FAC.
11 The component parallel to the
horizontal plane is
y
FACxz = FAC cos 70 = 0.3420 FAC
A

70

12 The vertical component of FAC is

FAC
z

FACy = FAC sin 70 = 0.9397FAC


70
C

298

3.2 Particles in Three-Dimensional Force Systems Example 6, page 6 of 7


y

FACxz = 0.3420FAC

13 The x and z components of FAC are


65

FACx = (0.3420FAC) cos 65 = 0.1445FAC


FACz = (0.3420FAC) sin 65 = 0.3100FAC

z
C
65

14 The x, y, and z components of FAC are now known and so we can write

FAC = 0.1445F ACi

0.9397FACj 0.3100FACk

(4)

299

3.2 Particles in Three-Dimensional Force Systems Example 6, page 7 of 7


16 Solving Eqs. 5, 6, and 7 simultaneously gives

15 The equilibrium equation is


200j + FAB + FAC + FAD = 0

(Eq. 1 repeated)

or, replacing FAB by Eq. 3, FAC by Eq. 4, and FAD by Eq. 2,


200j + ( 0.4924FABi

0.6428FABj

0.5868FABk)

+ ( 0.1445FACi

0.9397FACj

0.3100FACk)

+ ( 0.7071FADi

0.5FADj + 0.5FADk) = 0

Rearranging by collecting coefficients of i, j, and k gives


( 0.4924FAB
+ (200

0.1445FAC + 0.7071FAD)i

0.6428FAB

+ (0.5868FAB

0.9397FAC

0.50FAD)j

0.3100FAC + 0.50FAD)k = 0

Since each component must vanish independently, we have


Fx

0:

0.4924FAB

Fy

0: 200

Fz

0: 0.5868FAB

0.1445FAC + 0.7071FAD = 0

0.6428FAB

0.9397FAC

0.5FAD = 0

0.3100FAC + 0.5FAD = 0

(5)
(6)
(7)

300

FAB = 35.2 lb

Ans.

FAC = 158.5 lb

Ans.

FAD = 56.9 lb

Ans.

3.2 Particles in Three-Dimensional Force Systems Example 7, page 1 of 5


7. The compressive force in pole AO acts along the axis
of the pole and has a magnitude of 900 lb. Determine the
tension in the guy wires AC, AB, and AD.
y
A
B
1

65

Free body diagram of connection at A


A
y
x

20

FAD
900 lb

70

60
C

FAB

FAC

40

Compressive force, so
pole pushes up on A
D

Equilibrium equation
{900j} lb + FAC + FAD + FAB = 0

301

(1)

We must use the given angles to express the forces


in rectangular components.

3.2 Particles in Three-Dimensional Force Systems Example 7, page 2 of 5


y
A
B
65
4 To determine the rectangular components of FAB,
consider a view of the xz plane as seen from the
positive x axis. Thus
O
70

60
20

FAB = FAB sin 65j


= 0.9063FABj

FAB cos 65k


0.4236FABk

(2)

40
y
D
A

FAB

65
z

302

x, O

3.2 Particles in Three-Dimensional Force Systems Example 7, page 3 of 5


Next determine the rectangular components of FAC.
The vertical component is

The x and z components of FAC are

FACy = FACy sin 60

FACx = 0.5FAC cos 20

= 0.8660FAC

= 0.4698FAC

FACz = 0.5FAC sin 20

A
6
60

= 0.1710FAC

The component parallel to the


horizontal (xz) plane is

A
20
0.5 FAC

FACxz = FAC cos 60


= 0.5FAC

FAC
O

60

C
20

C
z
z

303

Thus all components of FAC are known:

FAC = 0.4698FACi
0.8660FACj
+ 0.1710FACk

(3)

3.2 Particles in Three-Dimensional Force Systems Example 7, page 4 of 5


9

Finally, we determine the rectangular


components of FAD. The vertical component is

FADy = FAD sin 70


A

= 0.9397FAD
y

40
10 The component parallel to
the horizontal (x-z) plane is

0.3420FAD

FADxz = FAD cos 70


O

= 0.3420FAD

11 Now, the x and z components of FAD are


70 FAD

FADx = 0.3420FAD sin 40 = 0.2198FAD

O
z

FADz = 0.3420FAD cos 40 = 0.2620FAD


D

12 Therefore,

70

40

FAD = 0.2198FADi

0.9397FADj + 0.2620FADk

304

(4)

3.2 Particles in Three-Dimensional Force Systems Example 7, page 5 of 5


13 Substituting the component form of the forces (Eqs. 2, 3, and 4) into the equilibrium equation, Eq. 1, gives
900j + FAB + FAC + FAD = 0
900j + ( 0.9063FABj
+ ( 0.4698FACi
+ (0.2198FADi

(Eq. 1 repeated)

0.4226FABk)

0.8660FACj + 0.1710FACk)
0.9397FADj + 0.2620FADk) = 0

(5)

Rearranging Eq. 5 by collecting coefficients of i, j, and k gives


( 0.4698FAC + 0.2198FAD)i
+ (900

0.9063FAB

0.8660FAC

0.9397FAD)j

+ ( 0.4226FAB + 0.1710FAC + 0.2620FAD)k = 0

(6)

Since each component must vanish independently, we have


Fx = 0:

0.4698FAC + 0.2198FAD =0

Fy = 0: 900
Fz = 0:

0.9063FAB

0.8660FAC

(7)
0.9397FAD = 0

(8)

0.4226FAB + 0.1710FAC + 0.2620FAD = 0

(9)

Solving Eqs. 7, 8, and 9 simultaneously gives


FAB = 350 lb, FAC = 203 lb, and FAD = 433 lb

Ans.

305

4. Moments and Resultants of Force Systems

306

4.1 Moments in Two-Dimensional Force Systems

307

4.1 Moments in Two-Dimensional Force Systems Procedures and Strategies, page 1 of 1


Procedures and Strategies for Solving Problems Involving
Moments in a Two-Dimensional Force System

The moment of a force F about a point O can be computed in


three ways:

O
d

1. MO = F d,
where d = perpendicular distance from O to the line of
action of the force;
F

2. MO = F
d,
where F = component of force perpendicular to the
line connecting point O with the point of application A
of the force, and d = distance OA; and

F
d
A

3. MO = Fy dx Fx dy,
where Fx and Fy are the rectangular components of the
force and dx and dy are the x and y distances between O
and A.

Which way is best depends on what information is given in


the problem. For example, if you are given x and y distances
from point O to the force, use the third approach above.

F
Fy
dx

Fx
dy

308

4.1 Moments in Two-Dimensional Force Systems Problem Statement for Example 1


1. Determine the moment of the 40-N force about the bolt at A when
the wrench is in position a) and in position b).
200 mm

HEAVY DUTY 250 MM

CHINA

A
a)

y
x

DROP FORGED STEEL 10"

40 N

40 N

A
b)

309

4.1 Moments in Two-Dimensional Force Systems Problem Statement for Example 2


2. Determine the moment of the 10-lb force
about a) end A and b) end B of the beam.
10 lb

4 ft

8 ft

310

4.1 Moments in Two-Dimensional Force Systems Problem Statement for Example 3


3. Determine the moment of the force about
the bolt at A for cases a) and b).
200 mm

30

200 mm
50 N
DROP FORGED STEEL 10"

DROP FORGED STEEL 10"

A
30
a)

b)

311

50 N

string tied
to end of
wrench

4.1 Moments in Two-Dimensional Force Systems Problem Statement for Example 4


4. Determine the moment of the 4-kN force about corner A of the plate a) by
using the definition MA= Fd, and b) by using the components of the force.

4 kN

60

4m
D

3m

312

4.1 Moments in Two-Dimensional Force Systems Problem Statement for Example 5


5. Boom AB and cable CB support a crate of weight W.
The force from the cable acting on the boom is known to
be 5 kN and is directed from B to C. Determine the
moment produced by this force about A.

B
12
13

5m

5
C

40
W
A

313

4.1 Moments in Two-Dimensional Force Systems Problem Statement for Example 6


6. A window is propped open by a stick AB. If a 150-lbin. moment about the
hinge is required to keep the window open, determine the force in the stick.
Assume the force is directed along the axis of the stick.

24 in.

30 in.
A

12 in.
B

314

4.1 Moments in Two-Dimensional Force Systems Example 1, page 1 of 3


1. Determine the moment of the 40-N force about the bolt at A when
the wrench is in position a) and in position b).
200 mm

HEAVY DUTY 250 MM

CHINA

A
a)

y
x
200 mm

DROP FORGED STEEL 10"

40 N

40 N

Position a)

A
200 mm

HEAVY DUTY 250 MM

CHINA

AV
HE

TY
DU

M
250

If the wrench could


rotate freely about A
(the bolt offered no
resistance), then a
counterclockwise
rotation would be
produced.

INA

40 N

CH

315

b)

4.1 Moments in Two-Dimensional Force Systems Example 1, page 2 of 3

Calculate the moment about A.


MA= +(40 N) (200 mm) = 8000 Nmm = 8 Nm

Ans.

Force distance to line of action of force


= magnitude of moment

Sign convention: a counterclockwise


rotation is called positive.

CHINA

Observation: The force points in the negative y


direction but MA is still positive . The sign
convention for moment is distinct from the sign
convention for force.

HEAVY DUTY 250 MM

y
40 N
x

316

4.1 Moments in Two-Dimensional Force Systems Example 1, page 3 of 3


Position b)

10"

A clockwise rotation
would occur, if the bolt
offered no resistance.

FO
RG
ED
ST
EE
L

DR

200 mm

OP

DROP FORGED STEEL 10"

40 N

Calculate the moment about A.


MA=

(40 N) (200 mm) = 8000 Nmm = 8 Nm

Ans.
40 N

9
Negative moment because the clockwise
rotation of the wrench is opposite what we
called positive moment.

Observation: The force points in


the positive x direction, but the
moment is still negative. This is
not a contradiction. Two sign
conventions exist, one for
moments and one for forces.

DROP FORGED STEEL 10"

y
x

317

4.1 Moments in Two-Dimensional Force Systems Example 2, page 1 of 3


2. Determine the moment of the 10-lb force
about a) end A and b) end B of the beam.
10 lb

4 ft

8 ft

318

4.1 Moments in Two-Dimensional Force Systems Example 2, page 2 of 3


a) Moment about A

Rotation
about A

If the beam could rotate freely about


point A (no constraint present at B),
then a clockwise rotation would be
produced.

10 lb

Sign convention: counterclockwise rotation is positive.

4 ft

MA= (10 lb) (4 ft) = 40 lbft

Ans.

Minus sign because rotation of beam is clockwise.

319

4.1 Moments in Two-Dimensional Force Systems Example 2, page 3 of 3


b) Moment about B

If the beam could rotate freely about


point B (no constraint present at A),
then a counterclockwise rotation
would be produced.

10 lb

8 ft

MB= +(10 lb) (8 ft) = 80 lbft

Ans.

Plus sign because rotation of beam is counterclockwise.

Observation: The same force can produce both a


positive or a negative moment it depends on the
location of the point about which the moment is
calculated.

320

4.1 Moments in Two-Dimensional Force Systems Example 3, page 1 of 4


3. Determine the moment of the force about
the bolt at A for cases a) and b).
200 mm

30

200 mm
50 N
DROP FORGED STEEL 10"

DROP FORGED STEEL 10"

A
30
a)

b)

Case a)

Draw the line of action


of the force.

String tied
to end of
wrench

50 N

Draw a line from A perpendicular


to the line of action.

50 N

50 N

DROP FORGED STEEL 10"

DROP FORGED STEEL 10"

30

30

321

4.1 Moments in Two-Dimensional Force Systems Example 3, page 2 of 4


Use trigonometry to calculate the "moment arm"
perpendicular distance d from A to the line of action.

200 mm
4

50 N

Calculate the moment.

DROP FORGED STEEL 10"

MA= (50 N)(100 mm)

30
= 5000 Nmm
d = (200 mm) sin 30
=

Nm

= 100 mm

Case b)

Draw the line of action.

DROP FORGED STEEL 10"

DROP FORGED STEEL 10"

A
30

30
string
50 N

50 N

322

Ans.

4.1 Moments in Two-Dimensional Force Systems Example 3, page 3 of 4


7

Draw a line from A perpendicular to the


line of action and calculate the
perpendicular distance.

200 mm
DROP FORGED STEEL 10"

30
8

d = (200 mm) sin 30

Calculate the moment.

50 N
= 100 mm

MA= (50 N)(100 mm)


= 5 Nm

323

Ans.

4.1 Moments in Two-Dimensional Force Systems Example 3, page 4 of 4


9

Observation: Same moment no matter where


the force acts along its line of action.

50 N
30
Rod attached to
end of wrench

DROP FORGED STEEL 10"

A
30
50 N
DROP FORGED STEEL 10"

DROP FORGED STEEL 10"

DROP FORGED STEEL 10"

30
30

string

50 N

10

This observation is called the "principle of


transmissibility" the force can "slide" along
its line of action without affecting the
moment produced.

324

50 N

4.1 Moments in Two-Dimensional Force Systems Example 4, page 1 of 4


4. Determine the moment of the 4-kN force about corner A of the plate a) by
using the definition MA= Fd, and b) by using the components of the force.

4 kN

60

4m
D

3m

325

4.1 Moments in Two-Dimensional Force Systems Example 4, page 2 of 4


1

Part a): Use geometry to compute d in MA= Fd.


y

4 kN
4m

AE = 4 m
=4m

EB

= 2.268 m

Draw a line
from A to
the line of
action.

3m

60

60

d = AE sin 60
7

x
4

d
2

60

1.732 m

Calculate moment.
MA= F

= 1.964 m
= (4 kN)

(1.964 m)

= 7.86 kNm

326

Distance EB =

Draw the line of action of the


force.

= (2.268 m) sin 60

Ans.

3m
= 1.732 m
tan 60

4.1 Moments in Two-Dimensional Force Systems Example 4, page 3 of 4


Part b): Express the force in terms of
rectangular components.
(4 kN) sin 60 = 3.464 kN
y

4 kN
4m

60

(4 kN) cos 60 = 2 kN
10 Tends to rotate the
plate clockwise
about A, so
negative.

3m

A
9

Calculate moment.
MA = Fd = + (3.464 kN)(4m)

(2 kN)(3m) = 7.86 kNm

11 Same answer as computed before.

327

Ans.

4.1 Moments in Two-Dimensional Force Systems Example 4, page 4 of 4


12 Observation: Calculating MA was much easier when using components
(part b) because we did not have to compute any distances they were
given. In general, since horizontal and vertical distances are often given
(or easily computed) in a problem statement, calculating moments by
using horizontal and vertical components of force is often the best
procedure to follow. It all depends, however, on what data are given.

13 In the problem below, use MA = Fd (express


force in components)

14 In this problem, use MA = Fd (do not express force in


components)

4 kN

40 kN
60

4m
D

A
IN
CH
0
25

M
M

TY
DU
Y
AV
HE

3m

200 mm
A
x

328

4.1 Moments in Two-Dimensional Force Systems Example 5, page 1 of 5


5. Boom AB and cable CB support a crate of weight W.
The force from the cable acting on the boom is known to
be 5 kN and is directed from B to C. Determine the
moment produced by this force about A.

B
12
13

5m

5
C

40
W
A

329

4.1 Moments in Two-Dimensional Force Systems Example 5, page 2 of 5


1

Express the 5-kN force in terms of


components at B:
2
12 (5 kN)
13

B
5 (5 kN)
13

Geometry

5 kN

40

12
13

(5 m) cos 40 = 3.830 m

5
C

(5 m) sin 40 = 3.214 m

5m

40

40
A

Calculate the moment.


MA = Fd = 5 (5 kN)(3.830 m)
13

12 (5 kN)(3.214 m) = 7.47 kNm


13

330

Ans.

(1)

4.1 Moments in Two-Dimensional Force Systems Example 5, page 3 of 5


4

Alternative solution. Slide the force to C


along the line of action.

12
13 (5 kN)
5 kN
12

40

5 (5 kN)
13

13

331

Line of action

4.1 Moments in Two-Dimensional Force Systems Example 5, page 4 of 5


B

Calculate the moment.


MA = Fd =

12 (5 kN)d + 5 (5 kN)(0)
CA
13
13

12 (5 kN)
13

5 (5 kN)
13

dCA

40

8
7

Geometry
(5 m) cos 40 = 3.830 m

Use similar triangles:


3.830 m
d
=
12
5
Solve and get

B
12
40

13

(2)

d = 1.596 m

(5 m) sin 40 = 3.214 m

C
5m

10

dCA
40

11 dCA = 3.214 m
A

= 1.618 m

332

1.596 m

This component of force


passes through point A so
its moment arm is zero.

4.1 Moments in Two-Dimensional Force Systems Example 5, page 5 of 5

12 Substituting dCA = 1.618 m in Eq. 2 gives


MA=

12 (5 kN)(1.618 m) = 7.47 kNm


13

Same result as before.

13 Observation: Even though sliding the force


vector to point C saved us some work because
one of the force components did not then
appear in the moment equation, this advantage
was not sufficient to compensate for the
additional geometry calculations that were
required; the first solution was easier.

333

4.1 Moments in Two-Dimensional Force Systems Example 6, page 1 of 6


6. A window is propped open by a stick AB. If a 150-lbin. moment about the
hinge is required to keep the window open, determine the force in the stick.
Assume the force is directed along the axis of the stick.

24 in.

30 in.
A

12 in.
B

334

4.1 Moments in Two-Dimensional Force Systems Example 6, page 2 of 6


Express the force of the stick acting
on the window in terms of
components at A.

Calculate MC.
MC = Fd = (FAB cos )d1 + (FAB sin )d2

FAB cos

d1

d2
FAB cos

FAB
A

FAB sin

FAB sin

Line of action of FAB


B
B

335

(1)

4.1 Moments in Two-Dimensional Force Systems Example 6, page 3 of 6


3

Geometry

Law of cosines for triangle ABC (calculate )


242 = 302 +122

2(30)(12) cos

Solving gives

= 9.119 in.
C

= 49.458

24 in.
7

d2 = 30 in.

7.800 in.

= 22.200 in.
30 in.
30 in.

A
12 in.
12 in.
B
B
6

(12 in.) cos 49.458 = 7.800 in.


= 49.458

336

d1 = (12 in.) sin 49.458

4.1 Moments in Two-Dimensional Force Systems Example 6, page 4 of 6


8

Substitute = 49.458, d1 = 9.119 in., d2 = 22.200 in., and MC = 150 lbin


(the required moment) into the equation for MC, Eq. 1:
MC = (FAB cos )d1 + (FAB sin )d2

(Eq. 1 repeated)

150 lbin = (FAB cos 49.458)(9.119 in.) + (FAB sin 49.458)(22.200 in.)
Solving gives
FAB = 6.58 lb

Ans.

337

4.1 Moments in Two-Dimensional Force Systems Example 6, page 5 of 6


9

Alternative solution. Slide the force along its


line of action to B and express it in terms of
components.
C

10 Calculate the moment about C.


MC = (FBA sin )(30 in.) + (FBA cos )(0)

30 in.

FBA cos

A
11 Moment arm is zero (line of
action passes through C).

F BA
B

FBA sin
12

Eliminating the moment


caused by FBA cos is the
reason we moved the force
to point B.

338

4.1 Moments in Two-Dimensional Force Systems Example 6, page 6 of 6


13 Equating MC to the known value 150 lbin.
gives
150 lbin. = (FBA sin )(30 in.) + 0
and substituting = 49.458 from Eq. 1 and
then solving yields:
FBA = 6.58 lb

(Same answer as before.)

14 Observation: Because the 30-in. moment arm


was given, the alternative solution method was
slightly easier than the first solution method
(Note that in both methods, we have to use the
law of cosines to calculate ).

339

4.2 Moments in Three-Dimensional Force Systems

340

4.2 Moments in Three-Dimensional Force Systems Procedures and Strategies, page 1 of 2


y

Procedures and Strategies for Solving Problems Involving


Moments in Three-Dimensional Force Systems

A possible
choice for r

1. To calculate the moment of a force F about a point O,


a) Express F in rectangular component form.
b) Define a position vector r, with tail at point O and head at any point
along the line of action of F. If you have more than one possible
choice for r, choose the one that gives the simplest form for r.
Express r in rectangular component form.

r
O

2. To determine the shortest distance d between a point A and a given


line, assume that a force of unknown magnitude F acts along the line,
and then make use of the fact that two different formulas exist for
calculating the magnitude of the moment MA about A:
MA = Fd

c) Evaluate the cross product: MO = r x F (If you have a scientific


graphing calculator, use its built-in cross-product function).
z

Best choice for r because r has a simple form


(only one component an x component)

(1)

F = Fu

and
MA = r x F

where r is a positive vector with tail at A and head on the line. Then
a) find a unit vector u parallel to the line, and express it in rectangular
component form;

r
d

b) express the force as F = Fu,

341

4.2 Moments in Three-Dimensional Force Systems Procedures and Strategies, page 2 of 2


c) express r in rectangular component form,
d) compute the cross product

r x F = r x (Fu)
= F(r x u)
Then Eqs. 1 and 2 give
Fd = F r x u
Cancel F.
Thus to calculate the shortest distance d from point
A to a line, just calculate the magnitude of the cross
product of r and u.

342

4.2 Moments in Three-Dimensional Force Systems Problem Statement for Example 1

1. Use the cross-product definition of the moment of a force to determine the


moment of the force about point A. Also, compare the sign of the result with
that obtained from the scalar definition of positive moment,
M = Fd.
y
200 mm

30

CHINA

HEAVY DUTY 250 MM

F = 50 N

x
A

343

4.2 Moments in Three-Dimensional Force Systems Problem Statement for Example 2


2. A force F = 20 N is applied to the end of a string of length L. The other end of
the string is tied to the handle of a wrench as shown. Use the cross-product
definition of the moment to determine the moment of F about point A. Discuss the
effect of distance L on your answer.
y
200 mm
L
CHINA

HEAVY DUTY 250 MM

30

C
F = 20 N

344

4.2 Moments in Three-Dimensional Force Systems Problem Statement for Example 3


3. A shower/bathtub grab bar is being pulled by a force F = 30 lb as
shown. Determine the moment of F about the support A. Also determine
the coordinate direction angles of the moment vector and interpret the
result.

5 in.
y
16 in.

40

8 in.
x
A

60
F = 30 lb
z

345

4.2 Moments in Three-Dimensional Force Systems Problem Statement for Example 4


4. A force F = 15 N acting parallel to the z axis is applied to the
handle of a socket wrench to turn a bolt at A. Determine the
moment of the force about the point A. Also, state which
component of the moment tends to turn the bolt.
y
80 mm

x
C
A
z
100 mm

F = 15 N

346

4.2 Moments in Three-Dimensional Force Systems Problem Statement for Example 5


5. Pulley B is used to drive pulley C. Determine the resultant
moment about bearing A produced by the belt forces acting on
pulley B. Also, interpret your result.

Q = 55 N

30

Radius = 70 mm
A

Belt forces

z
E

P = 30 N
40 mm

347

4.2 Moments in Three-Dimensional Force Systems Problem Statement for Example 6


6. A child on a bicycle collides with a mailbox and
exerts the force F shown. If the base of the pole at O
will fail if the magnitude of the moment there exceeds
60 Nm, determine if the mailbox will fall over.
y

F = {80i + 12j

10k} N

900 mm

250 mm

75 mm

348

4.2 Moments in Three-Dimensional Force Systems Problem Statement for Example 7


7. Copper tubing emerges from the wall at A and is subjected to a
force F at its free end B. The tubing will fail if the magnitude of
the moment at A exceeds 3 Nm. Determine the largest value of
the force F that can be applied to the free end of the tubing.

200 mm

A
35

300 mm

y
B

x
30

250 mm

40

349

4.2 Moments in Three-Dimensional Force Systems Problem Statement for Example 8


8. Two forces, P = 60 N and Q = 80 N act on the vertices of a cube as
shown. Determine the moment of each force about point O, if the length
of each edge of the cube is 2 m. Also, determine the shortest distance
from O to the line BF.

y
Each edge is
2 m long.

E
P = 60 N

Q = 80 N

D
G
O

350

4.2 Moments in Three-Dimensional Force Systems Problem Statement for Example 9


9. Determine the moment about the screw at A of the force F = 2 N
applied to the sheet-metal bracket shown. Also, determine the shortest
distance from A to the line connecting B and C.
y
50 mm 60 mm
30 mm
x

A
80 mm

z
70 mm
F=2N
B

25

100 mm

60 mm

351

4.2 Moments in Three-Dimensional Force Systems Problem Statement for Example 10


10. If the tension in the cable BC is T = 80 lb,
determine the moment about point A of the cable force
acting on the frame at point B. Also, determine the
shortest distance from A to the line through B and C.
y
C
12 in.

T = 80 lb
x

A
18 in.

10 in.

32 in.
z

352

4.2 Moments in Three-Dimensional Force Systems Example 1, page 1 of 3

1. Use the cross-product definition of the moment of a force to determine the


moment of the force about point A. Also, compare the sign of the result with
that obtained from the scalar definition of positive moment,
M = Fd.
y
200 mm

F = 50 N

HEAVY DUTY 250 MM

30

CHINA

B
z

1
y

Introduce a position vector rAB


with tail at A and head at B.

Express the force in rectangular components.


y
F = 50 N

200 mm

(50 N) sin 30 = 25 N

30

HEAVY DUTY 250 MM

CHINA

(50 N) cos 30 = 43.30 N

rAB = { 200i} mm
z

353

F = {43.30i

25j} N

(1)

4.2 Moments in Three-Dimensional Force Systems Example 1, page 2 of 3


3

Use the vector cross product to compute the moment


about point A.

MA = rAB

A memory aid for cross products:

j
= { 200i} mm

{43.30i

= [ 200(43.30) i

25j} N

i + ( 200)( 25) i

= 0, because cross
product of parallel
vectors is zero
4

i
j ] Nmm
k

= k, by the right-hand rule

Assign a plus sign if the product is in


the order indicated by the arrowheads;
minus sign otherwise

Apply the
right-hand rule.

i
j

j = +k
from i to j

i
k

i
k

fingers of right hand


curl from i to j
6
thumb points out
of i-j plane

k= j

MA = ( 200 mm)( 25 N)k


= {5000k} Nmm
= {5k} Nm

354

Ans.

4.2 Moments in Three-Dimensional Force Systems Example 1, page 3 of 3


7

Display the MA vector (a double-headed arrow)


y
50 N
HEAVY DUTY 250 MM

30

CHINA

A
z

MA = {+5k} Nm

direction of fingers

thumb points
out-of-plane

y
50 N
HEAVY DUTY 250 MM

CHINA

30

MA = {+5k} Nm

355

The 50-N force tends to rotate the


wrench counterclockwise about
the axis (through A) defined by
the unit vector k. By the
right-hand rule, this definition of
positive moment reduces to the
usual sign convention for positive
moment in coplanar problems.

Ans.

4.2 Moments in Three-Dimensional Force Systems Example 2, page 1 of 4


2. A force F = 20 N is applied to the end of a string of length L. The other end of
the string is tied to the handle of a wrench as shown. Use the cross-product
definition of the moment to determine the moment of F about point A. Discuss the
effect of distance L on your answer.
y
200 mm
L
HEAVY DUTY 250 MM

CHINA

30

C
z
1

Introduce a position vector rAC


with head at C and tail at A.

B
HEAVY DUTY 250 MM

CHINA

F = 20 N

30

rAC
C
F = 20 N

356

4.2 Moments in Three-Dimensional Force Systems Example 2, page 2 of 4


Determine the rectangular components of rAC.
y

3
L cos 30
CHINA

30

L sin 30

(L sin 30)j

200 mm

B
4

rAC = (L cos 30 + 200 mm)i

HEAVY DUTY 250 MM

rAC
C

L
z

Determine the rectangular


components of the force F

x
C
30
F = 20 N

F = (20 N) cos 30 i

357

(20 N) sin 30 j

(2)

(1)

4.2 Moments in Three-Dimensional Force Systems Example 2, page 3 of 4


7

Use the cross product to compute the moment.

MA = rAC

= {( L cos 30
= [( L cos 30

200)i

L sin 30 j} mm

00)( 20 cos 30)(i

{ 20 cos 30 i

20 sin 30 j} N

i)
j

=0

i
+ ( L cos 30

00)( 20 sin 30)(i

j)

=k
+ ( L sin 30 )( 20 cos 30)(j

i)

= k

+ ( L sin 30)( 20 sin 30)(j

j) ] Nmm

=0
= k[( L cos 30

200)( 20 sin 30)

( L sin 30)( 20 cos 30)] Nmm

= k[( L cos 30)( 20 sin 30) 200( 20 sin 30)


( L sin 30)( 20 cos 30)] Nmm
The terms involving L drop out
= {2000k} Nmm = {2k} Nm

Ans.

358

4.2 Moments in Three-Dimensional Force Systems Example 2, page 4 of 4


Discussion: Distance L does not appear in the answer
for MA. Thus the result is valid for all values of L,
and so the head of the position vector can be located
anywhere (for example, points 1, 2, 3, or C in the
figure) on the line of action of the force.

Note: All position vectors


and force vectors are in
the xy plane.

y
1
2
B

rA2

CHINA

30

rAC
F

rA3

x
A

30

Ans.

rA1

HEAVY DUTY 250 MM

359

4.2 Moments in Three-Dimensional Force Systems Example 3, page 1 of 6


3. A shower/bathtub grab bar is being pulled by a force F = 30 lb as
shown. Determine the moment of F about the support A. Also determine
the coordinate direction angles of the moment vector and interpret the
result.

5 in.
y
16 in.

40

8 in.
x
A

60
F = 30 lb
z

360

4.2 Moments in Three-Dimensional Force Systems Example 3, page 2 of 6


1

Introduce a position vector with tail at A and head at B.

rAB = { 16i + 8j + 5k} in.

(1)

5 in.
y
16 in.

40

8 in.

rAB
A

60
F = 30 lb
z

361

4.2 Moments in Three-Dimensional Force Systems Example 3, page 3 of 6


2

Determine the rectangular


components of the force F.
y
6

Fx = (30 lb)(sin 60)(cos 40)


= 19.90 lb

(4)

Fz = (30 lb)(sin 60)(sin 40)


= 16.70 lb
4

40
x

(3)
(30 lb) sin 60

60
7
F = 30 lb

F = { 19.90i

z
3

Fy = (30 lb) cos 60


= 15 lb

Component form of F,
from Eqs. 2-4:

(2)

362

15j + 16.70k} lb

(5)

4.2 Moments in Three-Dimensional Force Systems Example 3, page 4 of 6

Calculate the moment.

MA = rAB

= { 16i + 8j + 5k }

{ 19.90i

Because both vectors each have three non-zero


components, evaluating the cross products is
easier if we use the determinant form.

15j + 16.70k}

MA =

16

15

16.70

19.90

10 Expand the determinant in terms of the first row,


remembering to insert the minus sign for the j term.

MA =

15

16.70

16

= i[8(16.70)

5( 15)]

16

19.90

15

+k
19.90

j[ 16(16.70)

16.70

5( 19.90)] + k[ 16( 15)

= {208.6i +167.7j + 399.2k} lbin.

8( 19.90)]
Ans.

363

4.2 Moments in Three-Dimensional Force Systems Example 3, page 5 of 6


11 Observation: The above procedure is tedious and error prone. A much
better approach is to use a calculator with a built-in function for evaluating
the cross product of two vectors. If such a calculator is available, all you
need to do is enter the components of the vectors and then let the calculator
perform the arithmetic. As a result, typical errors such as missing a minus
sign or mis-copying a number from one line to the next are avoided.
To determine the coordinate direction angles, first determine the
magnitude of the moment.
MA =

(208.6)2 + (167.7)2 + (399.2)2

= 480.6 lbin.
Coordinate direction angles
= cos-1

MA x

= cos-1

MA y

= cos-1

MA z

MA
MA
MA

= cos-1 208.6 = 64.3


480.6

Ans.

= cos-1 167.7 = 69.6


480.6

Ans.

= cos-1 399.2 = 33.8


480.6

Ans.

364

4.2 Moments in Three-Dimensional Force Systems Example 3, page 6 of 6


5 in.
y
Axis of rotation
16 in.

40

69.6
x

64.3
A

MA = 480.6 lbin.

33.8

60
F = 30 lb
z

12 Interpretation: The force F = 30 lb


produces a moment of 480.6 lbin. about
point A. This moment tends to rotate the
grab bar about an axis defined by the
moment vector.

365

Ans.

4.2 Moments in Three-Dimensional Force Systems Example 4, page 1 of 3


4. A force F = 15 N acting parallel to the z axis is applied to the
handle of a socket wrench to turn a bolt at A. Determine the
moment of the force about the point A. Also, state which
component of the moment tends to turn the bolt.
y
80 mm

x
C
A
z
100 mm

F = 15 N

366

4.2 Moments in Three-Dimensional Force Systems Example 4, page 2 of 3


y
1

80 mm

Introduce a position vector


with head at B and tail at A.

rAB = {80i

100j} mm
x

C
A
z
2

Calculate the moment.

MA = rAB
= (80i

100 mm

rAB

F
100j)

{ 15k}

(F points in negative z-direction.)

F = 15 N
= 80( 15)(i

k) + ( 100)( 15)(j
= j

k)
=i

= ( 1200)( j) + (1500)i

j
i

= {1500i + 1200j} Nmm


= {1.5i + 1.2j} Nm

Ans.

367

4.2 Moments in Three-Dimensional Force Systems Example 4, page 3 of 3


3

Display the moment vector.


y
MA

80 mm

MAy = 1.2 Nm

C
A

MAx = 1.5 Nm

z
100 mm
4

The component that tends to


rotate the shaft AC of the
wrench about the x axis
(and thus turn the bolt) is
MA x = 1.5 Nm

B
15 N

Ans.

368

4.2 Moments in Three-Dimensional Force Systems Example 5, page 1 of 6


5. Pulley B is used to drive pulley C. Determine the resultant
moment about bearing A produced by the belt forces acting on
pulley B. Also, interpret your result.

Q = 55 N

30

Radius = 70 mm
A
C

Belt forces

z
E

P = 30 N
40 mm

369

4.2 Moments in Three-Dimensional Force Systems Example 5, page 2 of 6


1

Introduce position vectors with


tails at A and heads at E and F.

Q = 55 N

30

Radius = 70 mm

rAF

Belt forces

rAE
E
2
P = 30 N
40 mm

370

From the figure,

rAE = {40i

70j} mm

(1)

4.2 Moments in Three-Dimensional Force Systems Example 5, page 3 of 6


y

Q = 55 N
F

30
A

Radius = 70 mm

rAF
B

Belt forces

rAE

E
P = 30 N
3

40 mm

To determine the components of rAF, consider a


view of the pulley from the positive x axis:
6

Q = 55 N
7

35k} mm

(3)

In component form, from Eqs. 2 and 3,

rAF = {40i + 60.62j

rAFz = (70 mm) cos 60 = 35 mm

(4)

30

Angle = 90

30 = 60

F
5

rAFy = (70 mm) sin 60


= 60.62 mm

z
B
P = 30 N

371

Radius = 70 mm

(2)

4.2 Moments in Three-Dimensional Force Systems Example 5, page 4 of 6


y

Q = 55 N

30

F
Radius = 70 mm

rAF
A

Belt forces

rAE
E
8
P = 30 N
40 mm

Express the forces in rectangular


components.

Q = (55 N) sin 30j + (55 N) cos 30k


= {27.5j + 47.63k} N

P = {30k} N

372

(5)
(6)

4.2 Moments in Three-Dimensional Force Systems Example 5, page 5 of 6


9

Calculate the resultant moment.

MA= rAF Q + rAE P

MA =

40

60.62

35

27.5

60.62

40

70

30

40

60.62

27.5

47.63

35

40

=i

35

j
27.5

47.63

70

+i

+k
0

47.63

40

j
0

30

= i[60.62(47.63)

0
( 35)(27.5)]

+ i[ 70(30)

40

70

+k
30

j[40(47.63)

0(0)]

= {1750i

3105j + 1100k} Nmm

= {1.750i

3.105j + 1.100k} Nm

j[40(30)

( 35)(0)] + k[40(27.5)
0(0)] + k[40(0)

Ans.

373

70)(0)]

60.62(0)]

4.2 Moments in Three-Dimensional Force Systems Example 5, page 6 of 6


y

55 N

30

Mx = 1.750 Nm
B

Belt forces

30 N

10 The magnitude of the moment is


MA =

40 mm

(1.750)2 + ( 3.105)2 + (1.100)2

= 3.73 Nm

11 Interpretation: The belt forces acting on pulley B tend to


rotate the entire structure with a 3.73 Nm moment about an
axis defined by the direction of the moment vector MA.
The 1.750 Nm x component of MA is the component of
moment that rotates the shaft and drives pulley C.

374

Ans.

4.2 Moments in Three-Dimensional Force Systems Example 6, page 1 of 3


6. A child on a bicycle collides with a mailbox and
exerts the force F shown. If the base of the pole at O
will fail if the magnitude of the moment there exceeds
60 Nm, determine if the mailbox will fall over.
y

F = {80i + 12j

10k} N

900 mm

250 mm

75 mm

375

4.2 Moments in Three-Dimensional Force Systems Example 6, page 2 of 3


y

F = {80i + 12j
1

10k} N

Introduce a position vector with tail at O and


head at A.

rOA = {250i + 900j + 75k} mm


A
2

Calculate the moment.

MO = rOA F

rOA

250

900

75

80

12

10

900 mm

O
900

75

250

=i

75

j
12

10

= i[900( 10)

80

(12)(75)]

= { 9900i + 8500j
= { 9.9i + 8.5j

250

900

80

12

+k
10

j[250( 10)

80(75)] + k[250(12)

69000k} Nmm

69.0k} Nm

376

250 mm

80(900)]

75 mm

4.2 Moments in Three-Dimensional Force Systems Example 6, page 3 of 3


3

Magnitude of moment
MO =

( 9.9)2 + (8.5)2 + ( 69.0)2

= 70.2 Nm

Because 70.2 Nm exceeds the 60 Nm


maximum allowable moment, the
mailbox will fall over.

Ans.

377

4.2 Moments in Three-Dimensional Force Systems Example 7, page 1 of 6


7. Copper tubing emerges from the wall at A and is subjected to a
force F at its free end B. The tubing will fail if the magnitude of
the moment at A exceeds 3 Nm. Determine the largest value of
the force F that can be applied to the free end of the tubing.

200 mm

A
35

300 mm

y
B

x
30

250 mm

40

378

4.2 Moments in Three-Dimensional Force Systems Example 7, page 2 of 6


1

Introduce a position vector rAB


with tail at A and head at B.

200 mm

rAB
35

300 mm

y
B
x
30

250 mm

40

z
F

379

4.2 Moments in Three-Dimensional Force Systems Example 7, page 3 of 6


2

Determine the
components of rAB.

rABx = 200 mm + 250 mm = 450 mm

200 mm

(1)

250 mm
C

rAB
35

300 mm

B
x
30

D
250 mm

40

z
F

380

4.2 Moments in Three-Dimensional Force Systems Example 7, page 4 of 6


4

To determine the y and z components of rAB,


consider a view from the positive x axis.
y
C, A
35

rAB

rABy = (300 mm) sin 35


= 172.1 mm

300 mm

z
x, B, D

rABz = (300 mm) cos 35 = 245.7 mm

In component form, from Eqs. 1-3,

rAB = {450i

172.1j + 245.7k} mm

(3)

(4)

381

(2)

4.2 Moments in Three-Dimensional Force Systems Example 7, page 5 of 6


8

Determine the components of F.


12 Fx = F sin 40 cos 30
y

= 0.5567F

(7)

11 Fz = F sin 40 sin 30

x
30

= 0.3214F
40

10 F sin 40

Fy = F cos 40
= 0.7660F

F
13 In component form, from Eqs. 5-7,

F = F{0.5567i

0.7660j + 0.3214k}

(8)

382

(5)

(6)

4.2 Moments in Three-Dimensional Force Systems Example 7, page 6 of 6


14 Calculate the resultant moment.

15 Compute the magnitude.

MA = rAB F

MA =

= F (132.9)2 + ( 7.8)2 + ( 248.9)2

450

172.1

245.7

0.5567F

172.1

= 282.3F Nmm
= 0.2823F Nm

0.7660F 0.3214F

245.7

=i

450

245.7

j
0.7660F 0.3214F

= (132.9F)i

(7.8F)j

(132.9F)2 + ( 7.8F)2 + ( 248.9F)2

450

172.1

0.5567F

0.7660F

+k
0.5567F 0.3214F

(248.9F)k

16 Equate MA to the largest allowable moment, 3 Nm:


MA = 3 Nm
0.2823F Nm
Solving gives
F = 10.6 N

383

Ans.

4.2 Moments in Three-Dimensional Force Systems Example 8, page 1 of 6


8. Two forces, P = 60 N and Q = 80 N act on the vertices of a cube as
shown. Determine the moment of each force about point O, if the length
of each edge of the cube is 2 m. Also, determine the shortest distance
from O to the line BF.

y
Each edge is
2 m long.

E
P = 60 N

Q = 80 N

D
G
O

384

4.2 Moments in Three-Dimensional Force Systems Example 8, page 2 of 6


y

Each edge is
2 m long.
1

To determine the
moment of the force
P about point O, we
have several choices
of position vector
(Recall that the head
of the vector can lie
anywhere on the line
of action of P).

rOE

P = 60 N

Q = 80 N

rOD
G
O

z
2

Select the position vector with


the simpler form.

rOD = {2i + 2j +2k} m


rOE = {2j} m

(1)
Simpler (only one component)

385

4.2 Moments in Three-Dimensional Force Systems Example 8, page 3 of 6


y

Each edge is
2 m long.
3

rDE

Determine the components of


the force P by introducing a
position vector rDE:

rDE = { 2i

Q = 80 N

P = 60 N
D

2k} m
G
O

z
4

The force P has magnitude 60 N and


points in the direction from D to E, so

r
P = (60 N) r DE
DE
= (60 N)

= { 42.43i

386

2i

2k

( 2)2 + ( 2)2
42.43k} N

(2)

4.2 Moments in Three-Dimensional Force Systems Example 8, page 4 of 6


5

Calculate the moment.

MOP = rOE P
= {2j}

{ 42.43i

= (2)( 42.43)(j

i)

42.43k}
(2)(42.43)(j

Since rOE has only one component, it is easier to


calculate cross products of base vectors individually
rather than use the determinant approach to calculating
the cross product.

k)
j

= k

= { 84.86i + 84.86k} Nm

=i

Ans.

387

4.2 Moments in Three-Dimensional Force Systems Example 8, page 5 of 6


y
6

To determine the moment of the force Q


about point O, we could introduce either
position vector rOB or rOF. Let's
arbitrarily choose rOB.
C

rOF
8

rOB

MOQ = rOB Q
i

56.57

56.57

rFB

Calculate the moment.

Q = 80 N

P = 60 N

rOB = {2i + 2k} m

Each edge is
2 m long.

=i

Determine the components of


force Q by noting it has
magnitude 80 N and points from
F to B so

rFB
Q = (80 N) r
FB
2

j
56.57 56.57

56.57

+k
0

56.57

= (80 N)

2j + 2k
( 2)2 + (2)2

= { 56.57j + 56.57k} N
= {113.14i

113.14j

113.14k} Nm

(3)

Ans.

388

4.2 Moments in Three-Dimensional Force Systems Example 8, page 6 of 6


9

Finally, to determine the


shortest distance from point O
to line FB, consider the plane
defined by O and FB:

Q = 80 N

Each edge is
2 m long.

d1

E
P = 60 N

Q = 80 N

d2
C

d
d3

10 From the figure it is clear that of the various distances


d1, d2, d3, ..., from O to line FB, the perpendicular
distance d is the shortest. But we also know that, by
the scalar definition of moment, the moment of Q
about point O equals the perpendicular distance from O
to the line of action of Q times Q. That is,
MQO = Qd
Thus,
d=

MQO
Q
(113.14)2 + ( 113.14)2 + ( 113.14)2
80

= 2.45 m

Ans.

389

z A

4.2 Moments in Three-Dimensional Force Systems Example 9, page 1 of 6


9. Determine the moment about the screw at A of the force F = 2 N
applied to the sheet-metal bracket shown. Also, determine the shortest
distance from A to the line connecting B and C.
y
50 mm 60 mm
30 mm
x

A
80 mm

z
70 mm
F=2N
B

25

100 mm

60 mm

390

4.2 Moments in Three-Dimensional Force Systems Example 9, page 2 of 6


y
1

50 mm 60 mm

To calculate the moment


we could use either the
position vector rAB or rAC
.

rAC
x

rAB
z
2 rAC has a single component,

rAC = (50 mm + 60 mm)i


F=2N
B

= { 110i} mm

25

while rAB has two components, so let's


choose the simpler, rAC.

391

(1)

4.2 Moments in Three-Dimensional Force Systems Example 9, page 3 of 6


3

y
50 mm 60 mm

Next, we need to calculate the components


of the force, F. To do this, we first need to
calculate the coordinates of point B. By
inspection, the x coordinate of B is 60 mm.

30 mm
y

A
80 mm

z
4 To find the y and z
coordinates of point B,
consider a view from
the positive x-axis.

70 mm
30 mm
F=2N
80 mm

25

100 mm

70 mm
60 mm

100 mm
25
(100 mm) sin 25 = 42.26 mm

B
6

z coordinate of B

(100 mm) cos 25


= 90.63 mm + 80 mm
= 90.63 mm
= 170.63 mm

392

y coordinate of B
= 30 mm

70 mm

= 142.26 mm

42.26 mm

4.2 Moments in Three-Dimensional Force Systems Example 9, page 4 of 6


7

To determine the components of the force, introduce the


position vector rBC.

rBC = ( 50 mm

y
50 mm 60 mm

60 mm)i

+ [0

( 142.26 mm)]j

+ (0

170.63 mm)k

= { 110i + 142.26j

30 mm

C coordinates
( 50 mm, 0, 0)

170.63k} mm

A
80 mm

(2)
z

70 mm

rBC
F=2N
8

The force F has magnitude 2 N and


points from B to C so

110i + 142.26j

170.63k

( 110)2 + (142.26)2 + ( 170.63)2

= { 0.887i + 1.148j

100 mm

B coordinates (60, 142.26 mm, 170.63 mm)


60 mm

r
F = (2 N) r BC
BC
= (2 N)

25

1.377k} N

393

4.2 Moments in Three-Dimensional Force Systems Example 9, page 5 of 6


9

Calculate the moment.

MA = rAC

= { 110i}

{ 0.887i + 1.148j

= ( 110)( 0.887)(i

1.377k}

i) + ( 110)(1.148)(i

j) + ( 110)( 1.377)(i

=k

=0

k)

= j

j
i

= { 151.4j

126.3k} Nmm

(3)

Ans.

394

4.2 Moments in Three-Dimensional Force Systems Example 9, page 6 of 6


y

10 To determine the shortest distance


between point A and line BC, consider the
plane formed by A and BC.
C

50 mm 60 mm
30 mm

80 mm
z
70 mm

F=2N

F=2N
25

B
B

100 mm

60 mm

11 Magnitude of moment about A = Fd


or,

by Eq. 3

( 151.4)2 + ( 126.3)2 N mm = (2 N)d


Solving gives,
d = 98.6 mm

395

Ans.

4.2 Moments in Three-Dimensional Force Systems Example 10, page 1 of 5


10. If the tension in the cable BC is T = 80 lb,
determine the moment about point A of the cable force
acting on the frame at point B. Also, determine the
shortest distance from A to the line through B and C.
y
C
12 in.

T = 80 lb
x

A
18 in.

10 in.

32 in.
z

396

4.2 Moments in Three-Dimensional Force Systems Example 10, page 2 of 5


2
y
1

To calculate the
moment, we could
use either position
vector rAB or rAC.

Since rAC has only one


component while rAB has three,
let's use rAC.

rAC = {12j} in.

C
12 in.

(1)

T = 80 lb

rAC

rAB

10 in.
18 in.

32 in.
z

397

4.2 Moments in Three-Dimensional Force Systems Example 10, page 3 of 5


3

To determine the components of the force, T,


introduce the position vector rBC.

rBC = (0

32 in.)i + (12 in.

= { 32i + 2j

10 in.)j + (0

18 in.)k

18k} in.

y
C

rBC
12 in.

T = 80 lb
x

A
18 in.

4 The force T has magnitude 80 lb and


points from B to C so

10 in.

r
T = (80 lb) r BC
BC

32 in.
z

= (80 lb)

32i + 2j

( 32)2 + 22 + ( 18)2

= { 69.62i + 4.35j

398

18k

39.16k} lb

4.2 Moments in Three-Dimensional Force Systems Example 10, page 4 of 5


5

Calculate the moment.

MA = rAC

T
by Eq. 1

= 12j

{ 69.62i + 4.35j

= (12)( 69.62)(j

39.16k}

i) + (12)(4.35)(j

= k

j) + (12)( 39.16)(j

k)

=i

=0

j
i

= { 469.92i + 835.44k} lbin.

Ans.

(2)

399

4.2 Moments in Three-Dimensional Force Systems Example 10, page 5 of 5


y
C
12 in.

T = 80 lb
x

A
18 in.

10 in.

32 in.
z
6

Finally, to determine the shortest distance


between point A and line BC, consider the
plane formed by A and BC.

Extension of BC

or,

by Eq. 2

( 469.92)2 + (835.44)2 = (80 lb)d


Solving gives,

B
C

Magnitude of moment about A = Td

T = 80 lb

d = 11.98 in.

d
A

400

Ans.

4.3 Moment of a Couple

401

4.3 Moment of a Couple Procedures and Strategies, page 1 of 1

e
d

MA = Fd

Calculate the moment of a couple by summing


moments of the two forces about any point.
The result is independent of which point you
choose, but often you can save work by
choosing the point to lie on the line of action of
one of the forces.

MB = Fd

Procedures and Strategies for Solving Problems


Involving Couples

MC = Fe + F(e + d)
= Fd

402

4.3 Moment of a Couple Problem Statement for Example 1


1. Two swimmers on opposite sides of a boat
attempt to turn the boat by pushing as shown.
Determine the couple moment about
a) point A on the bow,
b) point B on the stern, and
c) point C.

A
1m
50 N

Also state what general principles your results


demonstrate.

2m
C

50 N

1.5 m

403

4.3 Moment of a Couple Problem Statement for Example 2


2. Determine the magnitude and sense of the resultant couple
moment acting on the rectangular plate.
30 N
20 N
8m

30
A

D
4m

C
30
20 N

30 N

404

4.3 Moment of a Couple Problem Statement for Example 3


3. Determine the value of the force P such that the
resultant couple moment of the two couples acting on
the beam is 900 lbft clockwise.

5 ft

4 ft
P
B

7 ft
P

200 lb
5 4
E 3

A
C
5 4
3
200 lb

0.5 ft

405

4.3 Moment of a Couple Problem Statement for Example 4


4. The wrench applies a 10 Nm couple
moment to the bolt. To prevent the plate
from rotating, two 2-N forces are applied as
shown. Determine the distance s such that the
resultant couple moment acting on the plate
and bolt is zero.

10 Nm

C
A

2N

2N
s

406

4.3 Moment of a Couple Problem Statement for Example 5


5. Two cords are wrapped around pegs attached to a
board as shown. Determine the value of such that P is
as small as possible while still producing a resultant
couple moment of zero. Also determine the value of P
corresponding to this value of . Neglect the size of the
pegs.

80 N
A

B
P

2m

C
80 N

4m

407

4.3 Moment of a Couple Problem Statement for Example 6


6. A plumber uses two pipe wrenches so that he can
loosen pipe BC from pipe AB without also
loosening pipe AB from the connection at the wall,
A. Determine the moment of the forces about a) A
and b) D. Also state what general principles your
results demonstrate.
y
130 mm

250 mm

x
A
80 N
175 mm

z
D

80 N

408

4.3 Moment of a Couple Problem Statement for Example 7


7. Determine the couple moment produced by the two
forces applied to the handle of the crank as shown.
y
40
F = 20 N
300 mm

25

25

40
G = 20 N
z

409

4.3 Moment of a Couple Problem Statement for Example 8


8. Determine the resultant couple moment of the two couples acting
on the block.
y

B
6 lb

20 in.

5 in.
x

6 lb
C

10 lb

30
E
z

10 lb

410

4.3 Moment of a Couple Problem Statement for Example 9


9. Force F is applied to the doorknob as shown. Determine
where on the surface of the door a force G = F should be
applied so that the resulting couple moment has a y
component of 2 Nm but no x or z components.
y

F = {16i + 12k} N

70 mm
650 mm

x
700 mm
z

411

4.3 Moment of a Couple Example 1, page 1 of 3


1. Two swimmers on opposite sides of a boat
attempt to turn the boat by pushing as shown.
Determine the couple moment about
a) point A on the bow,
b) point B on the stern, and
c) point C.

A
1m
50 N

Also state what general principles your results


demonstrate.

2m
C

50 N

1.5 m

412

4.3 Moment of a Couple Example 1, page 2 of 3


1

Calculate the moment about A.


2
MA = (50 N)(1 m)

Calculate the moment about B.

(50 N)(1 m + 2 m)
MB =

= 100 Nm

(50 N)(2 m + 1.5 m)

(50 N)(1.5 m)

ns.
= 100 Nm

ns.

A
1m
50 N

50 N

2m
C

2m
C

50 N

50 N

1.5 m

413

4.3 Moment of a Couple Example 1, page 3 of 3


3

Moment about C
MC = (50 N)(2 m)

50 N
= 100 Nm

ns.

2m
C

50 N

General principles demonstrated:


1) The couple moment is the same about every
point, and
ns.
2) The calculation of the couple moment is
simplified if the moment is calculated about a
point on the line of action of one of the forces
making up the couple.

414

4.3 Moment of a Couple Example 2, page 1 of 3


2. Determine the magnitude and sense of the resultant couple
moment acting on the rectangular plate.
30 N
20 N
8m

30
A

1
D
4m

20 N

30 N
(20 N) sin 30
= 10 N

30

30
20 N
30 N

Resolve the inclined forces into rectangular


components.

(20 N) cos 30
= 17.32 N

(20 N) cos 30
= 17.32 N
B

C
30
(20 N) sin30
= 10 N

30 N

415

20 N

4.3 Moment of a Couple Example 2, page 2 of 3


2 Calculate the moments
3 Moment of the couple formed by the forces at B and D.
30 N

10 N

MD = (30 N)(8 m)

8m
17.32 N

= 240 Nm

This moment value would be the same for any other point
besides D.

4m

30 N

17.32 N

10 N

4 Moment of the couple formed by the forces at A and C.


MC = (17.32 N)(4 m)

(1)

(10 N)(8 m)

= 10.72 Nm

(2)

This moment value would be the same for any other point besides C.

416

4.3 Moment of a Couple Example 2, page 3 of 3


5 Since both couple moments are the same about all
points, we can move them to any arbitrary point
we choose and then add them to get the resultant
couple moment.
M = MC + MD
= 10.72 + 240
= 229 Nm

ns.

No subscript because
valid for all points

D
229 Nm

B
6

Arbitrary location

417

4.3 Moment of a Couple Example 3, page 1 of 2


3. Determine the value of the force P such that the
resultant couple moment of the two couples acting on
the beam is 900 lbft clockwise.

5 ft

4 ft
P
B

7 ft
P

200 lb
5 4
E 3

A
C
5 4
3
200 lb

0.5 ft

418

4.3 Moment of a Couple Example 3, page 2 of 2


5 ft
P

7 ft
160 lb

P
0.5 ft

120 lb

A
120 lb

C
1
160 lb

MD =

P(5 ft)

(1)

Since the couple moments are the same about all


points, we can move them both to any arbitrary
point we choose and then add them to get the
resultant couple moment:
M = P(5) + 1860

Compute the moment of the couple at B and D.

(3)

Compute the moment of the couple at C and E.


ME = (120 lb)(0.5 ft) + (160 lb)(5 ft + 7 ft)

Since the resultant couple moment is specified to


be 900 lbft clockwise, Eq. 3 becomes

= 1860 lbft

900 lbft = P(5) + 1860


Solving gives
P = 552 lb

ns.

419

(2)

4.3 Moment of a Couple Example 4, page 1 of 2


4. The wrench applies a 10 Nm couple
moment to the bolt. To prevent the plate
from rotating, two 2-N forces are applied as
shown. Determine the distance s such that the
resultant couple moment acting on the plate
and bolt is zero.

10 Nm

C
A

2N

2N
s

420

4.3 Moment of a Couple Example 4, page 2 of 2

To calculate the moment of the couple, sum the moments of


the two forces at A and B with respect to the point A:
MA = (2 N)s

10 Nm

C
A

Since couple moments are the same about any point, the two
couple moments of (2 N)s and 10 Nm can be considered to act
at the same point and thus added to give the resultant moment,
M (Note that no subscript is needed on M, since M is
independent of where the moment is calculated):
M = (2 N)s

2N

10 Nm

2N
s

Because the resultant moment, M, is to be zero, the latter


equation becomes
0 = (2 N)s

10

Solving gives
s=5m

421

ns.

4.3 Moment of a Couple Example 5, page 1 of 5


5. Two cords are wrapped around pegs attached to a
board as shown. Determine the value of such that P is
as small as possible while still producing a resultant
couple moment of zero. Also determine the value of P
corresponding to this value of . Neglect the size of the
pegs.

80 N
A

B
P

2m

C
80 N

4m

422

4.3 Moment of a Couple Example 5, page 2 of 5

80 N
B

1
2m

Moment of the couple at B and C.


MC = (80 N)(2 m)
= 160 Nm

(1)

80 N

2
A
P

P
D

423

To balance the 160 Nm couple moment computed in


Eq. 1, the moment of the couple at A and D must be
160 Nm. To achieve this value of couple moment
with the smallest possible value of force P, P must be
perpendicular to line AD

4.3 Moment of a Couple Example 5, page 3 of 5

A
2m

P
C

Now use geometry to find .

Equal angles

tan-1 4 m
2m

4m

= 63.4

(2)

A
6
2m P

2
2
distance AD = (2 m) + (4 m)

= 4.472 m

P
7
C

Moment of couple A and D

MD = P

4m

distance AD

= P(4.472 m)

424

(3)

ns.

4.3 Moment of a Couple Example 5, page 4 of 5


8

The resultant couple moment must be zero:


M = MC + MD = 0

(4)

or,
160 + P(4.472 m) = 0
Solving gives
P = 35.8 N
9

ns.

Alternative solution. Do not assume that force P is


perpendicular to line AD. Calculate the couple moment of P
about point D in terms of the unknown angle .
MD = (P cos )(2 m) + (P sin )(4 m)

by Eq. 1
160

2m

80
cos + 2 sin

P sin

P
D

(Eq. 4 repeated)
by Eq. 5
(P cos )(2) + (P sin )(4)

4m

Solving for P gives


P=

(5)

Now substitute the expressions for MC and MD into Eq. 4:


MC + MD = 0

P cos

(6)

425

4.3 Moment of a Couple Example 5, page 5 of 5


10 To find the minimum value of P, use
dP = 0
d
From Eq. 6,
dP = d
d
d

cos

80
+ 2 sin

=0

so
80 ( sin + 2 cos )
=0
(cos + 2 sin )2
Thus
sin
tan

+ 2 cos

=0

=2

= tan-1 2
= 63.4
This is the same result as Eq. 2.

426

4.3 Moment of a Couple Example 6, page 1 of 3


6. A plumber uses two pipe wrenches so that he can
loosen pipe BC from pipe AB without also
loosening pipe AB from the connection at the wall,
A. Determine the moment of the forces about a) A
and b) D. Also state what general principles your
results demonstrate.
y
130 mm

250 mm

x
A
80 N
175 mm

z
D

80 N

427

4.3 Moment of a Couple Example 6, page 2 of 3


y

130 mm

250 mm
1

Part a): Determine the moment about


point A. Introduce position vectors
from point A to points D and E
respectively.
x

= {380i

rBD

175j} mm

rAE = {130 mm + 250 mm}i

80 N

rAD

rAD = {130i

175j} mm

(1)
(175 mm}j
(2)

175 mm
D

80 N
2

Use the cross product definition of moment.

MA = rAD
= {130i

{80k} N + rAE
175j}

= 130(80)i

{ 80k} N

{80k} + {380i

k 175(80)j
= j

175j}

k + 380( 80)i

Equal magnitude, opposite


sign, so cancel out

{ 80k}

k 175( 80)j
= j

= [130(80) + 380( 80)]( j)

k
i

= {20 000j} Nmm

k
= {20j} Nm

428

4.3 Moment of a Couple Example 6, page 3 of 3


y
3

250 mm
B

Part b): To determine the moment about


point D, first introduce a position vector
with tail at D and head at E.

rDE = {250i} mm

MD

80 N

175 mm
D
5

rDE

4
E

Calculate the moment.

MD = rDE

80 N
The two forces (the couple) produce a moment that tends to rotate the
entire pipe assembly about the vertical axis (j component). The force
applied at E also produces a moment on pipe BC about the x axis, while
the force applied at D produces a moment on the other pipe, AB, about
the x axis. Since the forces have opposite senses, the moments have
opposite senses and thus the total moment applied (to the entire pipe)
about the x axis adds to zero (no i component in MD).

{ 80k} N

= {250i}

{ 80k}

= 250( 80)i

k
= j

= {20000j} N mm

i
j

= {20j} N m

General principles demonstrated:

Same result as in Part a).

1) The couple moment is the same about every point, and


2) the calculation of the couple moment is simplified, if the moment is
calculated about a point on the line of action of one of the forces
making up the couple.

429

4.3 Moment of a Couple Example 7, page 1 of 3


7. Determine the couple moment produced by the two
forces applied to the handle of the crank as shown.
y
40
F = 20 N
300 mm

25

25

40
G = 20 N
z

430

4.3 Moment of a Couple Example 7, page 2 of 3


1 Express the force F in rectangular components.
2

Fy = (20 N) cos 40
40

= 15.32 N

F = 20 N

25

(20 N) sin 40

A
= 12.86 N

z
5

Fx = (12.86 N) cos 25

Fz = (12.86 N) sin 25
= 11.66 N
= 5.43 N
6

In component form,

F = {11.66i

15.32j + 5.43k} N

431

4.3 Moment of a Couple Example 7, page 3 of 3


7

Introduce a position vector with tail at B and head at A

Calculate the couple moment.

rBA = { 300i} mm

M = rBA

= { 300i}

{11.66i

15.32j + 5.43k}

40

= 300(11.66)i

i 300( 15.32)i

j 300( 5.43)i

F = 20 N
=0

300 mm

25

rBA B

= j

= {1629j + 4596k} Nmm

25

=k

j
= {1.63j + 4.60k} Nm

ns.

k
40
G = 20 N

432

Because the couple has both j and k


components, it tends to cause rotation
about both the y and z axes.

4.3 Moment of a Couple Example 8, page 1 of 6


8. Determine the resultant couple moment of the two couples acting
on the block.
y

B
6 lb

20 in.

5 in.
x

6 lb
C

10 lb

30
E
z

10 lb

433

4.3 Moment of a Couple Example 8, page 2 of 6


y
1
6 lb
A

20 in.

5 in.

rAC

Introduce a position vector


from a point, A, on the line of
action of the { 6i} lb force, to
a point C, on the line of action
of the {+6i} lb force.

rAC = {20k} in.

6 lb

10 lb

D
2 Calculate the moment of the 6-lb couple.
30

zz

10 lb

M6 = rAC

{6i} lb

= {20k}

{6i}

= 20(6) k

i
j

= {120j} lbin.

434

j
k

4.3 Moment of a Couple Example 8, page 3 of 6


y
3
6 lb

20 in.

5 in.
x

rDF = { 5j + 8.660k} in.

6 lb

10 lb
C

Introduce a position vector


from a point, D, on the line of
action of the { 10i} lb force,
to a point, F, on the line of
action of the {+10i} lb force.

D, C

rDF

30
E

10 lb

F, E

z
4

5 in.

30

Calculate the moment of the 10-lb couple.

M10 = rDF

5 in. = 8.660 in.


tan 30

{10i}

= { 5j + 8.660k}
= 5(10)j

i + 8.660(10)k

= k

{10i}

=j

= {86.60j + 50k} lbin.

435

View from positive x axis


looking back on yz plane

4.3 Moment of a Couple Example 8, page 4 of 6


5

MR

Compute the resultant couple moment from Eqs. 1


and 2,

= {120j} + {86.60j + 50k}


= {206.6j + 50k} lbin.

M6

M10

MR = M6 + M10

ns.

The two couples combine to produce a rotation of


the block about the axis defined by the MR vector.
Since MR is the same for all points, it can be
considered to act at an arbitrary point on top of the
block.

30
E
z

436

4.3 Moment of a Couple Example 8, page 5 of 6


y

Alternative solution that does not use the cross product


definition of a moment:

6 lb
20 in.

B
A
120 lbin

Using the right hand rule and the scalar equation M = Fd,
we can see that the couple moment produced by the 6-lb
couple has magnitude 6 lb 20 in. = 120 lbin. and can be
represented as a vector perpendicular to the plane of the
forces (plane ABDC) and pointing up.

5 in.
x

6 lb

7
30

Similarly, the 10-lb couple produces a couple moment with


magnitude 10 lb distance DF = 10 lb 10 in. = 100 lbin
and with direction perpendicular to the plane CDFE.

y
5 in. = 10 in.
sin 30

z
20 in.

100 lbin

5 in.

100 lbin

D, C
x
5 in.

30

10 lb

F, E
View from positive x axis looking back on yz plane
30

E
z

10 lb

437

4.3 Moment of a Couple Example 8, page 6 of 6


8

To add the 100 lbin. couple moment and the 120 lbin. couple
moment vectorially, we first have to express the 100 lbin.
couple moment, which is perpendicular to FD, in terms of its
y and z components.

11 Moment vector perpendicular


to top surface (plane ABDC)
of block

100 lbin
(100 lbin)(sin 60) = 86.6 lbin
10 90

30 = 60

MR = 86.6j + 50.0k + 120j

D, C

= {206.6j + 50k} lbin.

30
(100 lbin.)(cos 60) = 50.0 lbin.
30
F, E
y
z
9

12 In component form,

View from positive x axis looking back on yz plane

438

Same answer as before.

4.3 Moment of a Couple Example 9, page 1 of 5


9. Force F is applied to the doorknob as shown. Determine
where on the surface of the door a force G = F should be
applied so that the resulting couple moment has a y
component of 2 Nm but no x or z components.
y

F = {16i + 12k} N

70 mm
650 mm

x
700 mm
z

439

4.3 Moment of a Couple Example 9, page 2 of 5


y

F = {16i + 12k} N
1
70 mm
650 mm

Introduce a force G.

G= F

rBA

= {16i + 12k} N

B(xB, yB, 0)

G= F
700 mm

= { 16i
x

12k} N

G acts at a point B(xB, yB, 0) on the surface


of the door. G and F form a couple.

z
2

Draw a position vector rBA from point B on the surface of the door
to point A on the doorknob, and determine its rectangular
components.

rBA = (700 mm

xB)i + (650 mm

yB)j + (70 mm

440

0)k

4.3 Moment of a Couple Example 9, page 3 of 5


3

Calculate the couple moment.

M = rBA

F
i

(700

xB)

(650

16

k
yB)

(650

yB)

= i [(650

yB)(12)

= i [(650

yB)(12)]

12

70

(700

xB)

= i
0

70

12
70(0)]

16

j[(700

70

(700
+k

12
xB)(12)

70(16)] + k[(700

j [( 12xB + 7280)] + k [ (650

yB)(12) = 0

(2)

yB)(16) = 0

(3)

and Mz = 0, so
Mz = (650

(650

16

Use the fact that we know Mx = 0, so


Mx = (650

xB)

441

yB)(16)]

yB)

0
xB)(0)

(650
(1)

yB)(16)]

4.3 Moment of a Couple Example 9, page 4 of 5


5 Solving Eq. 2 gives
yB = 650 mm

ns.

This value of yB also satisfies Eq. 3:


(650

yB)(16) = 0

(Eq. 3 repeated)

650
The y component of M, from Eq. 1, is
My = ( 12xB + 7280) Nmm

(4)

Since My was specified at the beginning of the problem as


My = 2 Nm =

2000 Nmm

Eq. 4 becomes
2000 = ( 12xB + 7280)
Solving gives
xB = 440 mm

ns.

442

4.3 Moment of a Couple Example 9, page 5 of 5


y

M = { 2j} Nm
The couple moment
tends to open the door.

F = {16i + 12k} N
B

440 mm
650 mm
6
x
700 mm
z

443

Thus force G is applied at the same


height as F but closer to the hinges.

G = { 16i

12k} N

4.4 Moment of a Force About a Line

444

4.4 Moment of a Force About a Line Procedures and Strategies, page 1 of 1


Procedures and Strategies for Solving Problems
Involving Moment of a Force About a Line
1. Express the force F in rectangular component form.

2. Find the unit vector u along the line and express it in


rectangular component form.

u
3. Construct a position vector r with tail at any point on the
line and head at any point on the line of action of the
force .

F
r

4. Evaluate the scalar triple product:

ML = u r x F
Note: The choice of sense of u is arbitrary. If ML turns out
to be positive, then the moment vector MLu has the same
sense as u; if ML turns out to be negative, then MLu has the
opposite sense as u .

Line L

Moment vector MLu

445

4.4 Moment of a Force About a Line Problem Statement for Example 1


1. Force F is applied to the end of gearshift
lever CDE. Determine the moment of F
about shaft AB. State which way the lever
will rotate about AB.
y

F = {2i

j + 0.5k} lb

6 in.
40

D
B

5 in.
A
z

C
x

446

4.4 Moment of a Force About a Line Problem Statement for Example 2


2. Thread is pulled off a spool as shown.
Determine the moment of the force F
about the axis of the spool.
y

30 mm

F = 0.25 N

60

40

100 mm
x

447

4.4 Moment of a Force About a Line Problem Statement for Example 3


3. The lug wrench is used to turn the lug nut holding the wheel
on the hub. Determine the moment of the force about the axis of
the bolt to which the nut is fastened.
y

F = {3i + 10j

5k } N

x
z
0.3 m

0.15 m

0.25 m

448

4.4 Moment of a Force About a Line Problem Statement for Example 4


4. Forces P1, P2, P3, each of magnitude P, act on the edges of a
cube of side "a" as shown. Determine the moment of each force
about diagonal CB. Also express each moment in vector form.
y
Force P2 lies on the z axis.
P2 = P

G
P1 = P
H

a
A
F
z

P3 = P
B

449

4.4 Moment of a Force About a Line Problem Statement for Example 5


5. Determine the moment about the line through
points A and B of the force shown. Express the
result in vector form.
y
C
10 m

15 m
F = 6 kN

18 m
A
z

12 m
20 m

450

4.4 Moment of a Force About a Line Problem Statement for Example 6


6. An electrical conduit is held in place by brackets A, D
and E. The effect of the bracket at A on the conduit can be
represented by the force F as shown. Determine the
resulting moment that F produces that tends to twist the
portion CG of the conduit about its axis.
y

3m
E

D
C
2.5 m

F = { 20i +12j + 16k} N


B

3.5 m

A
z

x
2m

451

4.4 Moment of a Force About a Line Example 1, page 1 of 3


1. Force F is applied to the end of gearshift
lever CDE. Determine the moment of F
about shaft AB. State which way the lever
will rotate about AB.
y

F = {2i

j + 0.5k} lb

6 in.

F = {2i
40

j + 0.5k} lb

E
E

5 in.

A
z

1 The moment ML of a force F about a line L is


ML = u r

5 in.

where u is a unit vector along L, and r is a


position vector from any point on L to any
point on the line of action of F.

452

Here, the unit vector, u, is just the base


vector, k.

4.4 Moment of a Force About a Line Example 1, page 2 of 3


y
F = {2i

j + 0.5k} lb

6 in.
40

D
B

5 in.

MAB = u rCE

A
z

The best way to evaluate the scalar triple product is to use


a calculator that has built-in functions for the dot and cross
products. If such a calculator is not available, then
evaluate the triple product by using a determinant in which
the successive rows are u, r, and F.

9.596

0.5

rCE
3

3.857
2

Choose position vector r as rCE:


9.596

r = rCE

= 0

=0

0.5

0 + (1)[3.857( 1)

= 23.0 lbin.

453

3.857

0
1

= (6 in.)(sin 40)i + [5 in. + (6 in.)(cos 40)]j


= {3.857i + 9.596j} in.

3.857

9.596

+1
2

9.596(2)]

0.5

4.4 Moment of a Force About a Line Example 1, page 3 of 3


5

y
E

The negative sign in MAB = 23.0 lbin.


indicates that MAB has a sense opposite to
u. This moment will cause the lever CDE
to rotate clockwise about AB, when
viewed from end A.

ns.

C
A

B
23.0 lbin.
6

We could have saved a little work by noting that

MAB = u rCE

= k rCE

= MC (moment of F about C)
= k component of MC
So we only need to calculate MC and look at its k
component; we didn't need to evaluate the determinant for
the scalar triple product, u rCE F.

454

4.4 Moment of a Force About a Line Example 2, page 1 of 3


2. Thread is pulled off a spool as shown.
Determine the moment of the force F
about the axis of the spool.
y

30 mm

F = 0.25 N

60
1

40

100 mm
x

The moment ML of a force F about a line L is


ML = u r

where u is a unit vector along L and r is a


position vector from any point on L to any
point on the line of action of F.

455

4.4 Moment of a Force About a Line Example 2, page 2 of 3


y
F = 0.25 N
3

The best choice


(simplest form) for r
is r = {30i} mm.

60
2

Fy = (0.25 N) cos 60 = 0.1250 N

(0.25 N) sin 60 = 0.2165 N

Fz = (0.2165 N) sin 40 = 0.1392 N

30 mm

The unit vector


u is just the base
vector, u j.

40
7

Fx = (0.2165 N) cos 40 = 0.1658 N

In vector form,

x
z

F = {0.1658i + 0.1250j

456

0.1392k} N

4.4 Moment of a Force About a Line Example 2, page 3 of 3


9

Maxis of bobbin = u r

30

0.1658 0.125
0

0.1392

0.1250
= 0

30

= 0
0.1392

[30( 0.1392)

30

+0
0.1658

0.1392

0.1658

0(0.1658)] + 0
ns.

= 4.18 Nmm

457

0.1250

4.4 Moment of a Force About a Line Example 3, page 1 of 2


3. The lug wrench is used to turn the lug nut holding the wheel
on the hub. Determine the moment of the force about the axis of
the bolt to which the nut is fastened.
y

F = {3i + 10j

5k } N

The moment ML of a force F about a line L is


ML = u r

x
z
0.3 m

0.15 m

0.25 m

458

where u is a unit vector along L and r is a


position vector from any point on L to any
point on the line of action of F.

4.4 Moment of a Force About a Line Example 3, page 2 of 2


2

Since we want the moment about the bolt axis, which


is the x axis, the unit vector u is the base vector:

u
3

F = {3i + 10j

Choose r as the position vector from A to B:

5k} N

r = rAB
= {0.3i + 0.15j + 0.25k} m
Mbolt = u rAB

= i {0.3i + 0.15j + 0.25k} {3i + 10j


B

0.3

0.15

0.25

10

5k }

x
=
x
z
0.3 m

0.15 m

0.25 m

0.15

0.25

=1
10
= 1[0.15(5)
= 1.75 Nm

459

0.3

0.25

0
5
0.25(10)]

0.3

0.15

10

+0
3

0+0
Ans.

4.4 Moment of a Force About a Line Example 4, page 1 of 5


4. Forces P1, P2, P3, each of magnitude P, act on the edges of a
cube of side "a" as shown. Determine the moment of each force
about diagonal CB. Also express each moment in vector form.
y
Force P2 lies on the z axis.
P2 = P

G
P1 = P
H

The moment ML of a force F about a line L is


ML = u r

a
A
F
z

where u is a unit vector along L and r is a


position vector from any point on L to any
point on the line of action of F.

P3 = P
B

460

4.4 Moment of a Force About a Line Example 4, page 2 of 5


2

To determine the unit vector u, first define the position vector rCB.

rCB = ai

aj

Thus

r
u = r CB
CB
=
=

ai

aj

P1 = P

a2 + ( a2)

rCB
P2 = P

a
A
F
z

P3 = P
B

461

4.4 Moment of a Force About a Line Example 4, page 3 of 5


3

To calculate the moment of P1 about line CB, the


best choice (simplest form) for r is rCG (Note
that G lies on the line of action of P1).
y
r = rCG
G

= ( a)k

rCG

For force P1,


MCB = u rCG

Vector form of the force P1 = Pi

P1 = P
H

u
a
A

MCB
F

1
2
0

1
2
0

1
2

=0+

P1

0
0
1

a
0

1
2

[0(0)

0
P

a
0 0
+0
0
P 0

a) P)] + 0

aP

ns.

x
6

The vector representation of the moment about CB is


just the product of the magnitude of the moment, M CB,
times the unit vector u, since u is parallel to CB. Thus

MCB =

462

aP

u=

2
aP
( i + j)
2

aP

ns.

4.4 Moment of a Force About a Line Example 4, page 4 of 5


7

To calculate the moment of P2 about line CB, a


good choice for r is rCA (Note that A is on the
line of action of P2).

For force P2,

MCB = u rCA

r = rCA
= aj

P2

1
2
0

1
2
a

G
=
H

rCA

MCB
u P2 = P

1 0
2 0

a 0
0 P

1
[ a) P)
2
aP

0(0)]

0
P

+0

0
0

a
0

0+0

ns.

A
z

1
2

rBA

rBA would be
an equally
good choice

10 Vector form

MCB =

B
Vector form of
the force
P2 = Pk

=
=

463

aP

2
aP

aP (
2

i+j)

ns.

4.4 Moment of a Force About a Line Example 4, page 5 of 5


y
11 To determine the moment of P3 about line CB, note that
the line of action of P3 passes through point B. Thus
the force does not tend to rotate the cube about line CB:

MCB = 0

ns.

and in vector form

MCB = 0

ns.

a
In mathematical terms, we could choose a position
vector of zero length (from B to B), r = 0.

A
F
z

P3 = P
B

x
This would give

MCB = u r

=u0

=0

464

4.4 Moment of a Force About a Line Example 5, page 1 of 4


5. Determine the moment about the line through
points A and B of the force shown. Express the
result in vector form.
y
C
10 m
1

A
z

12 m
20 m

ML = u r

15 m
F = 6 kN

18 m

The moment ML of a force F about a line L is

where u is a unit vector along L and r is a


position vector from any point on L to any
point on the line of action of F.
x

465

4.4 Moment of a Force About a Line Example 5, page 2 of 4


y
C

10 m

The best choice (simplest form) for r is rBC

rBC = [10 m

( 20 m)]j

= {30j} m

15 m
F = 6 kN

18 m
A
z

u
20 m

Note that point C lies on the line of action of the force.

rBC
rAB

12 m
B

To determine the unit vector u along AB, first introduce


position vector rAB.

rAB = (0

20 m)j + (0

= { 20j

18k} m

Then,

rAB
u= r
AB
20j
=

466

18k

( 20)2 + (
0.7433j

)2
0.6690k

18 m)k

4.4 Moment of a Force About a Line Example 5, page 3 of 4


y
C
4 To express the force in rectangular components, first
introduce a position vector from D to C.

10 m

rDC

rDC = { 15i + 10j


15 m
F = 6 kN

18 m
A

Then the force is,

F = (6 kN)
z

12 m
20 m

12k} m

x
= (6 kN)

rDC
rDC
15i + 10j

12k

( 15)2 + (10)2 + ( 12)2


= { 4.1558i + 2.7705j

467

3.3247k} kN

4.4 Moment of a Force About a Line Example 5, page 4 of 4


5

MAB = u rBC

F
0

0.7433

30

4.1558
30

2.7705

3.3247
0
( 0.

2.7705

0( 4.1558)]

3.3247

0.6690[0(2.7705)

4.1558
30( 4.1558)]

MAB is the moment in the direction of


line AB, which is defined by the unit
vector, u.

MAB = MAB u
0.6690k}

= {62.0j + 55.8k} kNm

ns.

468

30

0.6690)

ns.

Vector form

.41){ 0.7433j

0
+

4.1558

83.41 kN.m

=(

33)

3.3247

= 0 + 0.7433[0( 3.3247)

= 0

0.6690

2.7705

4.4 Moment of a Force About a Line Example 6, page 1 of 3


6. An electrical conduit is held in place by brackets A, D
and E. The effect of the bracket at A on the conduit can be
represented by the force F as shown. Determine the
resulting moment that F produces that tends to twist the
portion CG of the conduit about its axis.
y

3m
E

D
C

1
2.5 m

ML = u r

F = { 20i +12j + 16k} N


B

where u is a unit vector along L and r is a


position vector from any point on L to any
point on the line of action of F.

3.5 m

A
z

The moment ML of a force F about a line L is

x
2m

469

4.4 Moment of a Force About a Line Example 6, page 2 of 3


y

To determine the unit vector u, first


introduce a position vector from C to G.

3m
E

D
C

rCA

F = { 20i +12j
+ 16k} N

rCG = {3i + 2.5j} m

rCG

Then the unit vector along CG is,


u

u = CG
rCG

2.5 m

=
3.5 m

3i + 2.5j
(3)2 + (2.5)2

A
z

= {0.7682i + 0.6402j}

x
2m

Choose the position vector r as

rCA
= { 3.5j + 2k} m

470

4.4 Moment of a Force About a Line Example 6, page 3 of 3


4

MCG = u rCA F
0.7682 0.6402
=

3.5

20

0
2

12

16

3.5

3.5

16

20

12

0.6402 0(16)

(2)( 20)

0.6402

= 0.7682
12

16

= 0.7682[ 3.5(16)

+0
20

2(12)]

= 87.1 Nm

ns.

471

4.5 Equivalent Force-Couple Systems

472

4.5 Equivalent Force-Couple Systems Procedures and Strategies, page 1 of 2


Procedures and Strategies for Solving Problems Involving
Equivalent Force-Couple Systems
1. To move a force from a point A on a rigid-body to a
point B and maintain equivalence, first calculate the
moment M of the force about B. Then at B apply the
force and a couple moment of magnitude M.

F
A
d

Equivalent

A
M = Fd
B

473

4.5 Equivalent Force-Couple Systems Procedures and Strategies, page 2 of 2


2. To reduce a system of forces to a single equivalent
force and couple moment, apply the procedure
described in paragraph 1 above to move each force to a
common point. Then add the forces to obtain a single
resultant force R and add the couple moments to obtain
a single resultant couple moment M.

F3

F1
A

3. To replace a system of coplanar forces by a single


force that is statically equivalent, first apply the
procedure described in paragraph 2 above to produce a
resultant force R and couple moment M at a point O.
Then move the force R to a new line of action such that
R produces a moment, about O, of magnitude M.

F2
Equivalent
R

d=

Equivalent

M
R
R

d
O

474

4.5 Equivalent Force-Couple Systems Problem Statement for Example 1


1. Replace the force at A by an equivalent
force and couple moment at point O.
y

4
3

6m

4m

20 N

x
0

475

4.5 Equivalent Force-Couple Systems Problem Statement for Example 2


2. The 60-N force acts at point A on the lever as shown.
Replace the force at A by a force and couple moment
acting at point O that will have an equivalent effect.
O

60

A
200 mm

60 N

476

4.5 Equivalent Force-Couple Systems Problem Statement for Example 3


3. Replace the 2-lb force acting on the end of the bottle
opener by an equivalent force and couple moment
acting on the underside of the bottle cap at A. Use your
results to explain how a bottle opener works.

3 in.
B
2 lb

477

4.5 Equivalent Force-Couple Systems Problem Statement for Example 4


4. Replace the given forces and couple moment by a
resultant force and couple moment at A.
260 lb

120 lb

12 13
5

800 lbft
A

2 ft

4 ft

7 ft

478

4.5 Equivalent Force-Couple Systems Problem Statement for Example 5


5. Replace the force F = 3 kN acting on corner A of the
block by an equivalent force and couple moment acting
at the center C of the block.
y
F = 3 kN

A
x

150 mm
150 mm
100 mm

100 mm

479

4.5 Equivalent Force-Couple Systems Problem Statement for Example 6


6. Replace the forces acting on the ice auger by an
equivalent force and couple moment acting at A.
y

4 lb
C

B
7 lb

6 in.
48 in.

480

4.5 Equivalent Force-Couple Systems Problem Statement for Example 7


7. Replace the forces and couple moment by a single force and
specify where it acts.
3 kip

4 kip

40

20 kipft

2 ft

8 ft

4 ft

3 ft

481

4.5 Equivalent Force-Couple Systems Problem Statement for Example 8


8. Replace the forces acting on the frame by a single force
and specify where its line of action intersects
a) member BCD and b) member AB.
y

400 N
600 N

300 N

D
2m
800 N
2m
E

A
4m

4m

482

4.5 Equivalent Force-Couple Systems Problem Statement for Example 9


9. Determine the value of force P such that the line of
action of the resultant of the forces acting on the truss
passes through the support at H. Also determine the
magnitude of the resultant.
30

160 lb

200 lb

180 lb

260 lb

8 ft
G

F
6 ft

6 ft

H
6 ft

6 ft

483

4.5 Equivalent Force-Couple Systems Problem Statement for Example 10


10. A machine part is loaded as shown. The part is to be attached to
a supporting structure by a single bolt. Determine the equation of
the line defining possible positions of the bolt for which the given
loading would not cause the part to rotate. Also, determine the
magnitude and direction of the resultant force.
y
80 N

60 N
60

90 Nm
0.3 m

0.5 m

20 Nm

x
0.4 m

0.6 m

484

4.5 Equivalent Force-Couple Systems Problem Statement for Example 11


11. The rectangular foundation mat supports the four
column loads shown. Determine the magnitude,
direction, and point of application of a single force that
would be equivalent to the given system of forces.
y
30 kip
15 kip
A
20 kip

11 kip

B
z
C

10 ft

D
3 ft

5 ft

8 ft

485

4.5 Equivalent Force-Couple Systems Problem Statement for Example 12


12. Three signs are supported by an arch over a highway and are
acted upon by the wind forces shown. Replace the forces by an
equivalent force and specify its point of application.
y
B

300 N

A
850 N
3.5 m

400 N

4m 6m

x
5m

4m

1m

486

4.5 Equivalent Force-Couple Systems Problem Statement for Example 13


13. Three forces act on the pipe assembly. Determine the
magnitude of forces P and Q if the resultant of all three
forces is to act on point A. Also determine the magnitude
of the resultant.
y

O
P
B

A
3m

200 N

1.5 m
2m
D

1m

487

4.5 Equivalent Force-Couple Systems Problem Statement for Example 14


14. The end plate of a pressurized tank is held in place by forces from three bolts.
Determine the required value of bolt-force P and angle if the resultant of the
three bolt forces is to act through the center of the plate at O.

700 N

0.5 m
x

40

C
P

500 N

488

4.5 Equivalent Force-Couple Systems Problem Statement for Example 15


15. Replace the system of forces by a wrench.
Determine the pitch and axis of the wrench.
y
B

12 m
140 N
O
40 N

4m
6m

60 N
z

489

4.5 Equivalent Force-Couple Systems Problem Statement for Example 16


16. In a machining operation, holes are simultaneously drilled at points
A and B of the wedge. The drill at A produces a force and couple
moment perpendicular to the planar surface at A. The force and couple
moment at B are similarly perpendicular to the planar surface at B.
Replace the forces and couple moments by a wrench. Determine
a) the magnitude R of the resultant force,
b) the pitch of the wrench,
c) the axis of the wrench, and
d) the point where the axis intersects the x-z plane.
y

1.5 in.

M1 60 lb in.

M2

F1 4 lb
B

80 lb in.

D
x

30

3 in.

F2
1 in.

6 in.
C

4 in.

4 in.

490

6 lb

4.5 Equivalent Force-Couple Systems Example 1, page 1 of 1


1. Replace the force at A by an equivalent
force and couple moment at point O.
y

4
3

6m

Calculate the moment about O.


MO = (16 N)(4 m) + (12 N)(6 m)
= 136 Nm

4m

20 N

x
0
3

Display the equivalent force and couple moment


at point O.

Express the force in rectangular components.


y

(20 N) 4 = 16 N
5
A

6m

136 Nm

4
3

4m

20 N
(20 N) 3 = 12 N
5

3
x

491

20 N

4
5

x
0

ns.

4.5 Equivalent Force-Couple Systems Example 2, page 1 of 1


2. The 60-N force acts at point A on the lever as shown.
Replace the force at A by a force and couple moment
acting at point O that will have an equivalent effect.

Calculate the moment about O.


MO = (60 N)(200 mm) cos 60
= 6000 Nmm
= 6 Nm

60

A
200 mm

60 N
O
(200 mm) cos 60

200 mm

60

A
60 N
2

Display the equivalent force


and couple moment at point O.

60 N
6Nm

ns.
A

492

4.5 Equivalent Force-Couple Systems Example 3, page 1 of 1


3. Replace the 2-lb force acting on the end of the bottle
opener by an equivalent force and couple moment
acting on the underside of the bottle cap at A. Use your
results to explain how a bottle opener works.

3 in.
1
B

Calculate the couple moment about point A.


MA = (2 lb)(3 in.)

2 lb

= 6 lbin.
2

Display the equivalent force


and couple moment at point A.
6 lbin.

ns.

B
2 lb

The bottle opener works by pulling (with a 2-lb


force) on the edge of the cap while
simultaneously twisting (with a 6 lbin. moment)
the entire cap away from the top of the bottle.

493

ns.

4.5 Equivalent Force-Couple Systems Example 4, page 1 of 2


4. Replace the given forces and couple moment by a
resultant force and couple moment at A.
260 lb

120 lb

12 13
5

800 lbft
A

2 ft

7 ft

4 ft

Calculate the resultant force


Rx = Fx: Rx = 100 lb

Express the inclined force in rectangular


components.

Ry = Fy: Ry = 240 lb
= 360 lb

12 (260 lb) = 240 lb


13

= 360 lb

120 lb
12 13
5

800 lbft
A

Calculate the resultant couple moment about A.


MRA = MA
= (240 lb) (2 ft + 4 ft + 7 ft)
+ (120 lb) (4 ft + 7 ft) 800 lbft

5 (260 lb) = 100 lb


13
2 ft

20 lb

4 ft

7 ft

= 3640 lbft

494

4.5 Equivalent Force-Couple Systems Example 4, page 2 of 2


4

Determine the magnitude and direction of the


resultant force.
100 lb

360 lb

R = (360)2 + (100)2
= 374 lb

ns.

= tan-1 360 = 74.5


100

ns.

Display the equivalent force and couple


moment at A.
374 lb

74.5
A
ns.

3640 lbft

495

4.5 Equivalent Force-Couple Systems Example 5, page 1 of 3


5. Replace the force F = 3 kN acting on corner A of the
block by an equivalent force and couple moment acting
at the center C of the block.
y
F = 3 kN

A
x

150 mm
150 mm
100 mm

100 mm

496

4.5 Equivalent Force-Couple Systems Example 5, page 2 of 3


y
F = 3 kN
1
A

rCA
x

Calculate the couple moment about C.

MC = rCA

= {100i

150k}mm

= 100(

150(

3j}kN

z
=k

= i

150 mm
150 mm
100 mm

= { 450i

300k}kNmm

= { 450i

300k}N m

100 mm

497

Ans.

4.5 Equivalent Force-Couple Systems Example 5, page 3 of 3


2

Display the equivalent force and couple moment at C.

F = { 3j}kN
MC = { 450i

Ans.

300k}Nm
A
x

150 mm
150 mm
100 mm

100 mm

498

4.5 Equivalent Force-Couple Systems Example 6, page 1 of 4


6. Replace the forces acting on the ice auger by an
equivalent force and couple moment acting at A.
y

4 lb
C

B
7 lb

6 in.
48 in.

499

4.5 Equivalent Force-Couple Systems Example 6, page 2 of 4


y

4 lb
C

Calculate the resultant force


Rx = Fx: Rx = 0

7 lb

Ry = Fy: Ry = 4 lb
6 in.

Rz = Fz: Rz = 7 lb
48 in.

500

4.5 Equivalent Force-Couple Systems Example 6, page 3 of 4


y
2

MAR = MA

4 lb
C

Calculate the resultant couple moment about A.

= rAC

{ 4j} lb + rAB

{7k} lb

= 0, because rAC and { 4j} are parallel,


or, what amounts to the same thing,
the line of action of the { 4j} lb force
passes throughout point A.

rAC

7 lb

rAB

= 0 + { 6i + 48j} in.

{7k} lb

= 6(7)i

= j

48 in.

6 in.

k + 48(7)j

= {336i + 42j} lbin.

501

=i

4.5 Equivalent Force-Couple Systems Example 6, page 4 of 4


y
C

MAR = {336i + 42j} lbin.


A

R={

ns.

j + 7k} lb

502

4.5 Equivalent Force-Couple Systems Example 7, page 1 of 3


7. Replace the forces and couple moment by a single force and
specify where it acts.
3 kip

4 kip

40

20 kipft

2 ft

8 ft

4 ft

3 ft

503

4.5 Equivalent Force-Couple Systems Example 7, page 2 of 3


(4 kip) cos 40 = 3.064 kip
3 kip
y

(4 kip) sin 40 = 2.571 kip

4 kip
B

20 kip ft

Resolve the inclined force into


rectangular components.

40
2 ft
2

8 ft

4 ft

3 ft

Calculate the resultant force


Rx = Fx: Rx = 3.064 kip = 3.064 kip
Ry = Fy: Ry = 3 kip

2.571 kip = 5.571 kip

= tan-1 5.571 = 61.2


3.064

3.064 kip
R=

(3.064) + (5.571)

= 6.358 kip

ns.

504

5.571 kip

ns.

4.5 Equivalent Force-Couple Systems Example 7, page 3 of 3


3 kip
3.064 kip

3.064 kip
4 kip
20 kipft

2.571 kip
D

= 61.2

40
2 ft

8 ft

This is the original force-couple moment


system.

We equate the moment of the new


system, about point A, to the moment
of the original force-couple system
(The choice of point A for summing
moments is arbitrary; any other point
would work as well, except that we
must use the same point for both the
original system and the new system.)

5.571 kip
D

4 ft

3 ft

R = 6.358 kip
This is a new force-couple system that we want to make
equivalent to the original force-couple system. We
already know that the forces are equivalent because R
is the resultant of the forces in the original system.
Now we have to make sure that the moment is also
equivalent. We do this by placing R at some unknown
distance d from the left end and then choosing d so that
the moment of this new system is the same as that of
the original system.

MAR = MA
or,
(5.571 kip)d = 20 kipft

(3 kip)(2 ft)

Solving gives
d = 2.10 ft

505

Ans.

(2.571 kip)(2 ft + 8 ft)

4.5 Equivalent Force-Couple Systems Example 8, page 1 of 5


8. Replace the forces acting on the frame by a single force
and specify where its line of action intersects
a) member BCD and b) member AB.
y

400 N
600 N

300 N

Determine the resultant force.


Rx = Fx: Rx = 800 N = 800 N

2m

Ry = Fy: Ry = 400 N

800 N
2m
E

600 N 300 N

= 1300 N
x
= 1300 N

4m

4m
800 N

R=
1300 N

506

(800)2 + (1300)2 = 1526 N

= tan-1 1300 = 58.4


800

Ans.
Ans.

4.5 Equivalent Force-Couple Systems Example 8, page 2 of 5

800 N

58.4

300 N

600 N

400 N

d
D

D
2m

1300 N
R = 1526 N
A

800 N
E

2m
E

A
2

Part a) To determine where the line of


action of the resultant force intersects
member BCD, place the force on BCD,
an unknown distance d from point B.

4m

4m

Choose d so that the moment about B of the resultant


force equals the moment of the original force system.
(The choice of point B was arbitrary.)
MBR = MB
or
(1300 N)d = (600 N)(4 m)
Solving gives
d = 4.92 m

507

Ans.

(300 N)(4 m + 4 m)

(800 N)(2 m)

4.5 Equivalent Force-Couple Systems Example 8, page 3 of 5


B
d'
800 N

1300 N
A

300 N

600 N

400 N

58.4

2m

R = 1526 N
800 N
4

Part b) To determine where the line of


action of the resultant force intersects
member AB, place the force a distance
d' from point B.

A
4m

2m
x

4m

Choose d' so that the moment about B of the resultant


force equals the moment of the original force system.

MBR = MB
or
(800 N)d' = (600 N)(4 m)
Solving gives
d' = 8.0 m

508

Ans.

(300 N)(4 m + 4 m)

(800 N)(2 m)

4.5 Equivalent Force-Couple Systems Example 8, page 4 of 5


B

4m
d' = 8 m

800 N
1300 N

58.4

R = 1526 N

The intersection of the line of action with a line drawn


through A and B occurs below point B.

509

4.5 Equivalent Force-Couple Systems Example 8, page 5 of 5


7

Alternative solution for part b. Once we know where


the line of action intersects member BCD, we can
use geometry to find the intersection with AB.

d = 4.92 m
B

58.4
d'

1300 N
E

From triangle CBG,


tan 58.4 =

d'
4.92

Solving gives,
d' = 8.0 m
(same result as before)

510

line of action of
resultant force

4.5 Equivalent Force-Couple Systems Example 9, page 1 of 4


9. Determine the value of force P such that the line of
action of the resultant of the forces acting on the truss
passes through the support at H. Also determine the
magnitude of the resultant.
30

160 lb

200 lb

180 lb

260 lb

8 ft
G

F
6 ft

6 ft

H
6 ft

6 ft

511

4.5 Equivalent Force-Couple Systems Example 9, page 2 of 4


160 lb

200 lb

180 lb

(260 lb) sin 30


= 130 lb
(260 lb) cos 30
= 225.2 lb

A
B

30
8 ft
F

260 lb G
6 ft

H
6 ft

6 ft

Determine the resultant force.


Rx = Fx: Rx = 130 lb

Ry = Fy: Ry = 225.2 lb

160 lb

200 lb

180 lb

= 765.2 lb
= 765.2 lb

(2)

512

J
6 ft

4.5 Equivalent Force-Couple Systems Example 9, page 3 of 4


160 lb

200 lb

180 lb

130 lb
A

225.2 lb

8 ft
G

F
6 ft

H
6 ft

6 ft

6 ft

Choose force P so that the


moment about H of the
resultant force equals the
moment of the original force
system about H. Any other
point besides H could be
used, but H has the advantage
that the moment of the
resultant R is zero, since R is
known (as part of the
statement of the problem) to
pass through H.

MHR = MH :
0 = (225.2 lb)(6 ft + 6 ft)
+ (160 lb)(6 ft)

G
Ry = 765.2 lb
Resultant R

I
Rx = 130 lb
= 130 lb

(130 lb)(8 ft)

(180 lb)(6 ft) + P(8)

Solving gives

P = 192.8 lb

( 192.8 lb)

= 322.8 lb

= 192.8 lb
ns.

513

ns.

4.5 Equivalent Force-Couple Systems Example 9, page 4 of 4


4

Magnitude of resultant.
From Eqs. 1 and 2,
R=

(322.8)2 + ( 765.2)2 = 831 lb

ns.

514

4.5 Equivalent Force-Couple Systems Example 10, page 1 of 4


10. A machine part is loaded as shown. The part is to be attached to
a supporting structure by a single bolt. Determine the equation of
the line defining possible positions of the bolt for which the given
loading would not cause the part to rotate. Also, determine the
magnitude and direction of the resultant force.
y
80 N

60 N
60

90 Nm
0.3 m

0.5 m

20 Nm

x
0.4 m

0.6 m

515

4.5 Equivalent Force-Couple Systems Example 10, page 2 of 4


y
(80 N) cos 60 = 40 N
90 Nm

60 N

60
80 N

Determine the resultant force.

(80 N) sin 60 = 69.28 N

Rx = Fx: Rx = 40 N = 40 N
Ry = Fy: Ry = 69.28 N
20 Nm

60 N

= 129.28 N

= 129.28 N
40 N

R=

(40)2 + (129.28)2 = 135.33 N

= tan-1 129.28 = 72.8


40
129.28 N

516

Ans.
Ans.

4.5 Equivalent Force-Couple Systems Example 10, page 3 of 4


y

60 N

40 N

90 Nm
60

40 N (x, y)

0.3 m

80 N

72.8

69.28 N

O
0.6 m

Rotation is caused by moment.


If the given force-couple
system is replaced by a single
equivalent force (the resultant)
and a specified line of action,
then the moment would be zero
about any point on the line of
action. So the line of action is
the line on which the bolt
should be placed to prevent
rotation.

x
x

3
2

x
0.4 m

129.28 N

0.5 m

20 Nm

To find the equation of the line of action, place the resultant at a


general point (x, y). Then equate moments about, say, point O for the
resultant (the figure on the right) and the original loading (the figure
on the left):
MOR = MO
or
(40 N)y (129.28 N)x = 90 Nm 20 Nm + (40 N)(0.3 m + 0.5 m)
(69.28 N)(0.4 m) (60 N)(0.4 m + 0.6 m)

517

4.5 Equivalent Force-Couple Systems Example 10, page 4 of 4


4

Solving for y gives the equation of a line


y = 3.232 x + 0.357

All points at the top of the machine part have a y coordinate of


y = 0.3 m + 0.5 m

Ans.

= 0.8 m.

This line defines the possible locations of


the bolt.

Substituting y = 0.8 m into the equation of the line for the bolt
locations,
y = 3.32 x + 0.357

0.133 m

and solving for x gives


x = 0.133 m

0.3 m

0.5 m
0.357 m
0

x
0.4 m

0.6 m

518

4.5 Equivalent Force-Couple Systems Example 11, page 1 of 4


11. The rectangular foundation mat supports the four
column loads shown. Determine the magnitude,
direction, and point of application of a single force that
would be equivalent to the given system of forces.
y
30 kip
15 kip
A
20 kip

11 kip

B
z
C

10 ft

D
3 ft

5 ft

8 ft
1

Determine the resultant.


Ry = Fy: Ry = 30 kip
= 76 kip
= 76 kip

519

15 kip

20 kip

11 kip

4.5 Equivalent Force-Couple Systems Example 11, page 2 of 4


2 To make the resultant
force R equivalent to
the original system of
forces, place R at the
point (x, 0, z) and then
determine values of x
and z such that R
z
produces the same
moments about the x
and z axes as the given
forces produce.

y
30 kip
15 kip

A
11 kip

20 kip
B
z

10 ft

8 ft

20 kip

8 ft

(x, 0, z)
C

R = 76 kip

30 kip
x, A

C, B

First, equate moments about the x axis. We can use


either the scalar definition of moment, M = Fd, or the
vector product definition. Let's use the scalar definition.

15 kip
z

x
B

y
11 kip

R = 76 kip

3 ft
5 ft

C, B

x, A

3 ft

z
View from positive x axis

MxR = Fd: (76 kip)z = (20 kip)(8 ft + 3 ft) + (15 kip)(8 ft + 3 ft)
+ (11 kip)(3 ft ) + (30 kip)(0)

520

(1)

4.5 Equivalent Force-Couple Systems Example 11, page 3 of 4


5 Solving Eq. 1 gives

z = 5.5 ft

30 kip

Ans.

R = 76 kip

Next, equate moments about the z axis.

15 kip

A
20 kip

11 kip

z
(x, 0, z)

z
C

10 ft

D
3 ft

5 ft

8 ft
y

6 Use two-dimensional views.

30 kip

15 kip

z, B, A
10 ft

MzR = Fd:

20 kip

11 kip

D
5 ft

y
R = 76 kip

z, B, A
x
View from positive z axis

(76 kip)x = (20 kip)(10 ft)

521

(11 kip)(10 ft + 5 ft)

(2)

4.5 Equivalent Force-Couple Systems Example 11, page 4 of 4


8 Solving Eq. 2 gives
x = 4.80 ft

Ans.

522

4.5 Equivalent Force-Couple Systems Example 12, page 1 of 3


12. Three signs are supported by an arch over a highway and are
acted upon by the wind forces shown. Replace the forces by an
equivalent force and specify its point of application.
y
B

300 N

A
850 N
3.5 m

4m 6m

400 N
O

x
5m

4m

1m

Determine the resultant.


Rz = Fz: Rz = 300 N

850 N

400 N

= 1550 N
= 1.55 kN

Ans.

523

4.5 Equivalent Force-Couple Systems Example 12, page 2 of 3


2

Place the resultant R at the arbitrary point (x, y, 0) and then


determine values of x and y such that R produces the same
moments about the x and y axes as the given forces produce.
y
B

300 N

(x, y, 0)

A
850 N
3.5 m

4m 6m

400 N
O

1550 N
O

x
x

4m

5m
z

1m

First equate moments about the x axis.

850 N
400 N
300 N

4m

3.5 m

View from the


positive x axis

x,O

z
4

6m

R = 1550 N

MxR = Fd: (1550 N)y = (850 N)(6 m)

(400 N)(4 m)

524

(300 N)(3.5 m)

x,O

(1)

4.5 Equivalent Force-Couple Systems Example 12, page 3 of 3


5

Solving Eq. 1 gives


y = 5.0 m

Ans.

Next, equate moments about the y axis.

y,O

x
A

5m

4m

1m

=
400 N

850 N

300 N

y, O

View from the


positive y axis

R = 1550 N
x

MxR = Fd: (1550 N)x = (300 N)(1 m) + (850 N)(1 m + 5 m) + (400 N)(1 m + 5 m + 4 m)
Solving gives
x = 6.06 m

Ans.

525

4.5 Equivalent Force-Couple Systems Example 13, page 1 of 6


13. Three forces act on the pipe assembly. Determine the
magnitude of forces P and Q if the resultant of all three
forces is to act on point A. Also determine the magnitude
of the resultant.
y

O
P
B

A
3m

200 N

1.5 m
2m
D

1m

1 Express the resultant R in terms of P and Q.


R = Fy: R = P + Q + 200 N

526

(1)

4.5 Equivalent Force-Couple Systems Example 13, page 2 of 6


2

Place the resultant R at point A and then determine values of


P and Q such that R produces the same moment about the x
and z axes as the given forces, P, Q, and 200 N, produce.
y
3
O

We can use either the scalar definition of moment M = Fd,


or the vector definition, M = r F. Let's use the vector
definition and calculate moments with respect to point O.

rOB
rOC
rOD
z

Q
O

3m
200 N

R=( P

1.5 m

200 N)j

2m
D

1m

rOA
4

Introduce position vectors, all with tails at point O.

A
x

3m

rOB = {3i} m

rOC = {5i + 1.5k} m


D

rOD = {5i + 2.5k} m


rOA = {3i + 1.5k} m

527

1.5 m

4.5 Equivalent Force-Couple Systems Example 13, page 3 of 6


5

For equivalence, equate the moment of the resultant R


about point O to the moment of the given forces about O.

MOR = MO: rOA

R = rOB

{ Pj} + rOC

{ Qj} + rOD

{ 200j}

or
{3i + 1.5k}
3( P

( P
200)i

200)j = {3i}

j + 1.5( P

{ Pj} + {5i + 1.5k}


200)k

=k

j = 3( P)i

= i

{ Qj} + {5i + 2.5k}

j + 5( Q)i

=k

j + 1.5( Q)k

=k

Equating coefficients of i gives


1.5(P + Q + 200) = 1.5Q + 500

(2)

Equating coefficients of k gives


3(P + Q + 200) = 3P

5Q

1000

(3)

528

5Q

1000)k

j + 5( 200)i

= i

Collecting coefficients of i, j, and k gives


1.5(P + Q + 200)i 3(P + Q + 200)k = (1.5Q + 500)i + ( 3P

{ 200j}

j + 2.5( 200)k

=k

= i

4.5 Equivalent Force-Couple Systems Example 13, page 4 of 6


7

Solving Eqs. 2 and 3 gives,


P = 133.3 N

ns.

Q = 200 N = 200 N

ns.

Using these values in Eq. 1 gives


8

P and Q were defined as


downward directed
forces.

R = P + Q + 200 N
133.3

200
ns.

= 133.3 N

529

4.5 Equivalent Force-Couple Systems Example 13, page 5 of 6


9

Alternative solution: the computation are simplified


somewhat if we sum moments about point A instead
of point O.
y

O
O
P

rAC

rAB
z

3m

rAD

200 N

1.5 m
D

2m
D

10

1m

Introduce position vectors, all with


tails at point A.

rAB = { 1.5k} m
rAC = {2i} m
11 Equate the moment of R about point A to
the moment of the given forces about A.

530

rAD = {2i + k} m

4.5 Equivalent Force-Couple Systems Example 13, page 6 of 6


MAR =
12

Because R passes
through point A, the
moment is zero.

MA: 0 = rAB

{ Pj} + rAC

{ Qj} + rAD

{ 200j}

or

0 = { 1.5k}

{ Pj} + {2i}

{ Qj} + {2i + k}

{ 200j}

j
0 =

5( P)k

j + 2( Q)i

= i

j + 2( 200)i

=k

0i + 0j = ( 1.5P + 200)i + ( 2Q

=k
400)k

Equating coefficients of i gives


0 = 1.5P + 200

(4)

Equating coefficients of k gives


0 = 2Q

400

(5)

Solving Eqs. 4 and 5 gives,


P=
Q=

Same result as before.

531

j + ( 200)k
= i

j
k

4.5 Equivalent Force-Couple Systems Example 14, page 1 of 4


14. The end plate of a pressurized tank is held in place by forces from three bolts.
Determine the required value of bolt-force P and angle if the resultant of the
three bolt forces is to act through the center of the plate at O.

700 N

0.5 m
x

40

C
P

500 N

532

4.5 Equivalent Force-Couple Systems Example 14, page 2 of 4


1 Place the resultant, R, at point O and then equate the moment of R about O (which is
zero because R passes through O) to the moment of the given forces about O.
y

700 N

rOA
0.5 m

rOC
C
P

rOB
500 N

0.5 m
x

40
C

R
B

B
2 Introduce position vectors,
all with tails at point O.
rOA = {0.5j} m

rOB = {0.5 cos 40i


rOC = { 0.5 cos i

0.5 sin 40j} m = {0.3830i


0.5 sin j} m

533

0.3214j} m

4.5 Equivalent Force-Couple Systems Example 14, page 3 of 4


3

Equate moments about O.

MOR = MO: 0 = rOA

{ 700k} N + rOB

{ 500k} N + rOC

{ Pk} N

R passes through O, so
produces zero moment.

or

0 = {0.5j}

{ 700k} + {0.3830i

0 = 0.5( 700)j

k + 0.3830( 500)i
=i

0.3214j}

{ 500k} +

0.3214( 500)j

= j

{ 0.5 cos

=i

0.5 cos

0.5 sin

( P)i

j}

{ Pk }

0.5 sin

( P)j

= j

=i

Collecting coefficients of i and j gives


0i + 0j = [0.5( 700)

0.3214( 500)

0.5 sin

( P)]i + [0.3830(500)

0.5 cos

(P)]j

(1)

Equating coefficients of i gives


0 = [0.5( 700)

0.3214( 500)

0.5 sin

( P)]

or, after some arithmetic,


P sin

= 378.6

(2)

534

4.5 Equivalent Force-Couple Systems Example 14, page 4 of 4


5

Equating coefficients of j in Eq. 1 gives


0 = 0.3830(500)

0.5 cos (P)

or, after some arithmetic,


P cos

= 383.0

(3)

Eqs. 2 and 3 are best solved by using a calculator capable of solving simultaneous
nonlinear equations. Alternatively, dividing Eq. 2 by Eq. 3 gives
P sin
P cos

378.6
383.0

tan
Solving for

gives,
ns.

= 44.7
Substituting for
P cos

in Eq. 3 gives,

= 383.0
44.7

Solving gives
P = 539 N

ns.

535

4.5 Equivalent Force-Couple Systems Example 15, page 1 of 6


15. Replace the system of forces by a wrench.
Determine the pitch and axis of the wrench.
y
B

12 m
140 N
O
40 N

4m
6m

60 N
z

536

4.5 Equivalent Force-Couple Systems Example 15, page 2 of 6


1

Express the 140-N force in terms of


rectangular components. First introduce a
position vector rABfrom A to B.

rAB

{4i + 12j

6k } m

rAB

12 m

2 Thus the vector form of the 140-N force is

FAB = (140 N)

rAB
rAB

140 N
4i + 12j

= (140 N)

6k

40 N

42 + 122 + ( 6)2
= {40i + 120j
3

60k} N

(1)

120j

60 N

F
{40i + 120j

60k} N

6m

The resultant of all three forces is

4m

14

Forces along x and z axes

{40i} N + {60k} N
(2)

537

4.5 Equivalent Force-Couple Systems Example 15, page 3 of 6


6

Moment arm is zero.


(40 N force passes through O)
y
B

D
5

Moment resultant about O:

MOR

rOA

FAB + rOA

(60k) + 0

(6k)

(40i +120j

60k) + (6k)

( 40i)

12 m

6(40) k

(60k)

i + 6(120) k

j + 6( 60) k

k + 6(60) k
0

k
FAB

O
40 N

{ 720i + 240j} N m

6m
60 N
z

538

rOA

{6k} m

4.5 Equivalent Force-Couple Systems Example 15, page 4 of 6


7

Express MOR in terms of components parallel M|| and


perpendicular M to R.
y

M || {240j} N m
R

MOR

{120j} N

{ 720i +240j} N m

M = { 720i} N m
z
8

By definition,
pitch of wrench, p,

M||
R

539

240
120

2m

Ans.

4.5 Equivalent Force-Couple Systems Example 15, page 5 of 6


9

Now move R to a new line of action such that in the


new location R will produce a moment (about O)
equal to M .

10

Equate the moment of R in the new position to M

rOP

Old line of action of R

y Old position of R
New position of R

M = { 720i} N m

or
O
{xi + zk}

{120j}

720i

{120j} N

rOP
x(120) i

j + z(120) k j
k
i

(120z)i + (120x)k

720i
z
rOP

720i + 0k

P(x, 0, z)
(Intersection of new line
of action with x-z plane)
{xi + zk} m

Equating coefficients of i gives


120z = 720

New line of action of R

z = 720 = 6 m
120
Equating coefficients of k gives
120x = 0
x=0

540

4.5 Equivalent Force-Couple Systems Example 15, page 6 of 6


11 The axis of the wrench is a vertical line
(same direction as R {120j}) passing
through the point (0, 0, 6 m)

Ans.
y

M|| {240j} N m

O
12 Because a couple moment has the same value
about all points it can be moved to the new
line of action of R to form the wrench.

6m
Ans.

{120j} N
z

541

4.5 Equivalent Force-Couple Systems Example 16, page 1 of 14


16. In a machining operation, holes are simultaneously drilled at points
A and B of the wedge. The drill at A produces a force and couple
moment perpendicular to the planar surface at A. The force and couple
moment at B are similarly perpendicular to the planar surface at B.
Replace the forces and couple moments by a wrench. Determine
a) the magnitude R of the resultant force,
b) the pitch of the wrench,
c) the axis of the wrench, and
d) the point where the axis intersects the x-z plane.
y

1.5 in.

M1 60 lb in.

M2

F1 4 lb
B

80 lb in.

D
x

30

3 in.

F2
1 in.

6 in.
C

4 in.

4 in.

542

6 lb

4.5 Equivalent Force-Couple Systems Example 16, page 2 of 14


1

Express the forces and couple moments in rectangular components.


M2

60 lb in.

2 Geometry

60

F1 4 lb
E

E
90

30

60

30

Equal

60

30
C

z
x, D

x, D

View from positive x axis

F1

(4 lb) sin 60j


{ 3.464j

M1

2k} lb

(60 lb in.) sin 60j


{ 51.962j

543

(4 lb) cos 60k


(1)
(60 lb in.) cos 60k

30k} lb in.

(2)

4.5 Equivalent Force-Couple Systems Example 16, page 3 of 14


y

M2

80 lb in.

B
F2

F2
M2

{ 6i} lb
{ 80i} lb in.

544

6 lb

(3)
(4)

4.5 Equivalent Force-Couple Systems Example 16, page 4 of 14


5

Next, determine the coordinates of points A and B.


y

E
6

Ay

(3 in.) sin 30
Az

1.5 in.

(6 in. + 1 in.)

2.598 in.

x
4.402 in.

y
A

7
E

C
4 in.

3 in.

4 in.

1.5 in.
B

z
8

Ax

4 in.

Bx

4 in. + 4 in.
8 in.

30

x, D
6 in.

(3 in.) cos 30 2.598 in.

545

1 in.

By

1.5 in.

Bz

1 in.

4.5 Equivalent Force-Couple Systems Example 16, page 5 of 14


9

Introduce position vectors from O, the origin of coordinates, to A and B.


y

10

rOA

Axi + Ayj + Azk


{4i + 1.5j + 4.402k} in.

(5)

B
O

x
A

11 rOB

Bxi + Byj + Bzk


{8i + 1.5j + k} in.

546

(6)

4.5 Equivalent Force-Couple Systems Example 16, page 6 of 14


12 Resultant force

F: R

{ 3.464j
{ 6i

2k} + { 6i}

3.464j

2k} lb

(7)

13 Resultant moment about point O

MOR
i
MOR =

MO: MOR
j

1.5

3.464

rOA

F1 + rOB

k
4.402
2

F2
i

1.5

Carrying out the cross products and simplifying gives

MOR

{12.249i + 2j

4.856k}

(8)

547

4.5 Equivalent Force-Couple Systems Example 16, page 7 of 14


14 Display R and MOR with respect to the xyz axes.
y

MOR

{ 6i

3.464j

2k} lb

548

{12.249i + 2j

4.856k}

4.5 Equivalent Force-Couple Systems Example 16, page 8 of 14


15 Two intersecting vectors define a plane. Display the plane defined by R and MOR.
Q

R
MOR

549

4.5 Equivalent Force-Couple Systems Example 16, page 9 of 14


16 Express MOR in terms of components
parallel M || and perpendicular M to R.

17 Unit vector in R direction


R
u
R

By Eq. 7 for R,
R

M
R

( 6)2 + ( 3.464)2 + ( 2)2


7.211 lb

Ans.

(9)

Thus

MOR

M||
18 M ||

0.832i

0.480j

M|| u
( 9.806){ 0.832i

3.464j
7.211

2k

0.277k

(10)

component of MOR in direction of u


{12.249i + 2j

19 In vector form, since M|| lies in the


direction of the unit vector u,

M||

6i

MOR u

R
R

4.856k}{ 0.832i

0.480j

0.277k}

Performing the multiplications and


then simplifying gives
0.480j

0.277k}

{8.159i + 4.707j + 2.716k} lb in.

M||
(12)

550

9.806 lb in.

(11)

4.5 Equivalent Force-Couple Systems Example 16, page 10 of 14


20 The component of MOR perpendicular to R can now be
computed by starting with the vector sum

MOR

M ||

and rearranging to get

M = MOR

M||
by Eq.12

by Eq.8

{12.249i + 2j
{4.090i

4.856k}

2.707j

{8.159i + 4.707j + 2.716k}

7.572k} lb in.

(13)

551

4.5 Equivalent Force-Couple Systems Example 16, page 11 of 14


21 Now we move the force R to a new line of
action such that R will produce a moment
about O exactly equal to M

Old position of R
Q

M
R

Old line of action of R


O

M||
R
Parallel

New line of action of R

O
New position of R
(below plane SPQR)

552

4.5 Equivalent Force-Couple Systems Example 16, page 12 of 14


y

22 View in terms of xyz axes

Old line of action of R

New line of action of R


O

Old position of R

rOP
23 Because the force R, in its new position, is to create a
moment about O equal to M , we can write

z
Parallel

rOP
or,

by Eq. 13

by Eq.7
{xi + zk}

{ 6i

3.464j

2k}

4.090i

2.707j

7.572k

New position of R
Performing the multiplications and simplifying gives
P(x, 0, z)-intersection of new line
of action with the x-z plane
3.464zi + (2x

553

6z)j

3.464xk

4.090i

2.707j

7.572k

4.5 Equivalent Force-Couple Systems Example 16, page 13 of 14


24 Equating coefficients of i gives
3.464z

4.090

(14)

Similarly for j and k


2x

6z

3.464x

2.707

(15)

7.572

(16)

These are three equations in only two unknowns, but the


equations are not all independent. Eqs 14 and 16 imply
z
x

4.090 1.181
3.464
7.572 2.186
3.464

(17)
(18)

Substituting these values into the left hand side of Eq.15 gives
2x

6z

2(2.186)

6(1.181)

2.714

Thus Eq.15 is satisfied (Round-off, error leads to 2.714 rather than 2.707).

554

4.5 Equivalent Force-Couple Systems Example 16, page 14 of 14


26 The axis of the wrench is a line passing through the
point (2.186 in., 0, 1.181 in.) with direction, by Eq. 10,

0.832i

0.480j

Ans.

0.277k

2.186 in.
x

1.181 in.
z

Ru

M||

M || u
by Eq. 11
( 9.805 lb in.)u

by Eq. 9

27 Pitch of wrench

M ||
R
9.806 lb in.
7.211 lb
1.360 in.

Ans.

(7.211 lb)u
25 Because a couple moment is the same about all
points, we can move M || to point P. Similarly
we can slide R to P along R's line of action, by
the principle of transmissibility.

28 Summary: The effect of the drilling operations on the two


surfaces of the wedge is to push the wedge down the
wrench axis (direction of u) while simultaneously causing
the wedge to tend to rotate counterclockwise (as viewed
from a position above the x-z plane) about the wrench axis.

555

4.6 Distributed Loads on Beams

556

4.6 Distributed Loads on Beams Procedures and Strategies, page 1 of 1


Procedures and Strategies for Solving Problems Involving Distributed
Loads Acting on Beams

Area = wL

1. To calculate the resultant R of a distributed load, calculate the


area under the load curve w(x) either by integration or by using the
formulas for area (for a rectangle or triangle, for example):

w
L

R = w(x) dx = Area

R = wL

2. To locate the line of action, locate the centroid of the area under
the load curve either by integration or by using the formulas for
locating centroids of simple shapes.

xR =

xw(x) dx
w(x) dx

wo

L
R=

2L
3

557

L
3

1
woL
2

4.6 Distributed Loads on Beams Problem Statement for Example 1


1. Determine a) the magnitude and location of the resultant
of the distributed load, and b) the reactions at the supports.
2 kN/m

B
4m

2m

558

4.6 Distributed Loads on Beams Problem Statement for Example 2


2. Determine a) the magnitude and location of the resultant of
the distributed load, and b) the reactions at the support.
4 kN/m

B
6m

559

4.6 Distributed Loads on Beams Problem Statement for Example 3


3. Determine a) the magnitude and location of the resultant
of the distributed load, and b) the reactions at the supports.
200 lb/ft

50 lb/ft
A

B
12 ft

560

4.6 Distributed Loads on Beams Problem Statement for Example 4


4. Determine the reactions at the supports.
900 N/m
300 N/m

250 N/m
B

A
3m

1.5 m

561

4.6 Distributed Loads on Beams Problem Statement for Example 5


5. Determine the reactions at the support.
2 kip/ft

3 ft

2 kip/ft

9 ft

562

4.6 Distributed Loads on Beams Problem Statement for Example 6


6. Determine the reactions at the supports.
w = wo sin

x
L

w
x

B
L

563

4.6 Distributed Loads on Beams Problem Statement for Example 7


7. Determine the reactions at the supports.
w = 2 x2

2x + 4 kN/m
14.4 kN/m

40 kN

w
4 kN/m
A

B
3m

5m

564

4.6 Distributed Loads on Beams Problem Statement for Example 8


8. The lift force acting on an airplane wing can be modeled by
the equation shown. Determine the force and moment at the
point where the wing is attached to the fuselage.

3m
A

p(x) = [1500

565

10(x2 + 4)] N/m

4.6 Distributed Loads on Beams Problem Statement for Example 9


9. Determine the reactions at the supports.
120 lb/ft
B

360 lb/ft
9 ft

4 ft

566

4.6 Distributed Loads on Beams Problem Statement for Example 10


10. Determine the reactions at the supports.
4 kip/ft
3 kip ft
A

B
6 ft

2 ft

2 ft

567

4.6 Distributed Loads on Beams Problem Statement for Example 11


11. Determine w1 and w2 so that the beam is in equilibrium.
8 kN

w1

w2
4m

2m

568

4.6 Distributed Loads on Beams Problem Statement for Example 12


12. The forces from the supports are approximately represented as
triangular distributed loads. Determine the values of w1 and w2.
3 kN/m
8 kN m
A

B
C
w2
w1
0.3 m

0.4 m
1m

4m

1m

569

2m

4.6 Distributed Loads on Beams Example 1, page 1 of 3


1. Determine a) the magnitude and location of the resultant
of the distributed load, and b) the reactions at the supports.
2 kN/m

B
4m

2m

Distributed load diagram. The magnitude R


of the resultant of the distributed load equals
the area under the load curve.

R = Area
= (2 kN/m)(6 m)

2 kN/m
A
B
6m

570

= 12 kN

Ans.

4.6 Distributed Loads on Beams Example 1, page 2 of 3


3

The line of action of the resultant, R, passes through the centroid of the load area.
4

3m

R = 12 kN

6m
= 3 m from A
2

B
6m

Free-body diagram

3m
R = 12 kN

Ax

A
Ay

Centroid of rectangle lies

By
4m

2m

571

Ans

4.6 Distributed Loads on Beams Example 1, page 3 of 3


6

Equations of equilibrium,

Fx = 0: Ax = 0
Fy = 0: Ay + By

12 kN = 0

MA = 0: (4 m)(By)

(3 m)(12 kN) =0

Solving gives
Ax = 0

Ans.

Ay = 3 kN

Ans.

By = 9 kN

Ans.

572

4.6 Distributed Loads on Beams Example 2, page 1 of 3


2. Determine a) the magnitude and location of the resultant of
the distributed load, and b) the reactions at the support.
4 kN/m

B
6m

Distributed load diagram. The magnitude R of


the resultant of the distributed load equals the
area under the load curve.
2

R = Area
=

1
(4 kN/m)(6 m)
2

= 12 kN

4 kN/m

Ans.

B
6m

573

4.6 Distributed Loads on Beams Example 2, page 2 of 3


3

The line of action of the resultant, R, passes


through the centroid of the triangle.
4
y

Centroid of triangle lies


1 (6 m) = 2 m from B
3

2m

Ans.

R = 12 kN
A
x

Free-body diagram. Since the support at A is fixed,


three reactions are present: Ax, Ay, and MA.
R = 12 kN
A

Ax

B
MA

2m
Ay
6m

574

4.6 Distributed Loads on Beams Example 2, page 3 of 3


Equations of equilibrium:

Fx = 0: Ax = 0
Fy = 0: Ay

12 kN = 0

MA = 0: (12 kN)(4 m) + MA = 0
Solving gives
Ax = 0

Ans.

Ay = 12 kN

Ans.

MA = 48 kN m

Ans.

575

4.6 Distributed Loads on Beams Example 3, page 1 of 3


3. Determine a) the magnitude and location of the resultant
of the distributed load, and b) the reactions at the supports.
200 lb/ft

50 lb/ft
A

B
12 ft

Distributed load diagram. The trapezoid loading area can


be divided into rectangular and triangular loadings.
2 F2 = Area2
1
=
(150 lb/ft)(12 ft)
2

= 900 lb

F1 = Area1
= (50 lb/ft)(12 ft)

150 lb/ft

= 600 lb

(2)
(1)

50 lb/ft
A

B
12 ft

576

4.6 Distributed Loads on Beams Example 3, page 2 of 3


A single resultant, R, can be
calculated as:

The lines of action of F1 and F2 pass through the centroids of the


rectangular and triangular loading areas respectively.
xR

R = Fy
= F1 + F2

F1 = 600 lb
F2 = 900 lb

= 600 lb + 900 lb
= 1500 lb

Ans.

B
4 ft
6 ft

Centroid of triangle lies

Centroid of rectangle lies 1 (12 ft) = 6 ft from A


2

1 (12 ft) = 4 ft from A


3
8

To be equivalent, the moment about A produced by R must equal


the sum of the moments about A produced by F1 and F2.

xR R = (6 ft)F1 (4 ft)F2

where xR is the distance of R from the support A.


Hence,
xR (1500 lb) = (6 ft)(600 lb)
xR = 4.8 ft

577

Ans.

(4 ft)(900 lb)

4.6 Distributed Loads on Beams Example 3, page 3 of 3


9

Free- body diagram


xR = 4.8 ft
R = 1500 lb
Ax

Ay

By

12 ft

Fx = 0: Ax = 0

Fy = 0: Ay + By

10 Equations of equilibrium

MA = 0: (12 ft)(By)

1500 lb = 0
(1500 lb)(4.8 ft) = 0

Solving gives
Ax = 0

Ans.

Ay = 900 lb

Ans.

By = 600 lb

Ans.

578

4.6 Distributed Loads on Beams Example 4, page 1 of 3


4. Determine the reactions at the supports.
900 N/m
300 N/m

250 N/m
B

A
3m

1.5 m

Distributed load diagram.


(2)

300 N/m

(1)

(900 N/m

300 N/m) = 600 N/m

(3)

250 N/m
B

A
3m
2

Resultant of load No. 1


F1 = Area1
= (300 N/m)(3 m)
= 900 N

1.5 m

Resultant of load No. 2


F2 = Area2
1
=
(600 N/m)(3 m)
2

Resultant of load No. 3


F3 = Area3
= (250 N/m)(1.5 m)
= 375 N

= 900 N

579

4.6 Distributed Loads on Beams Example 4, page 2 of 3


5

The lines of action of resultants F1, F2, and F3 pass through the
centroid of their loading areas.

Centroid of rectangle 1 lies

3.75 m
2m
1.5 m

3m
= 1.5 m from A
2

F1

F2

F3

(2)
(1)

Centroid of rectangle 3 lies

7
(3)

3m+

A
3m

1.5 m
8

Ax

F2 = 900 N

Centroid of triangle 2 lies


2
(3 m) = 2 m from A
3

Free body diagram

F1 = 900 N

1.5 m
= 3.75 m from A
2

F3 = 375 N

Ay

By
1.5 m
2m
3.75 m
4.5 m

580

4.6 Distributed Loads on Beams Example 4, page 3 of 3

Fx = 0: Ax = 0

Fy = 0: Ay + By

10 Equations of equilibrium

MA =

900 N

(900 N)(1.5 m)

900 N

375 N

(900 N)(2 m)

(375 N)(3.75 m) + By(4.5 m) = 0

Solving gives
Ax = 0

Ans.

Ay = 1162 N

Ans.

By = 1012 N

Ans.

581

4.6 Distributed Loads on Beams Example 5, page 1 of 3


5. Determine the reactions at the support.
2 kip/ft

3 ft

2 kip/ft

9 ft
1

Distributed load diagram

2 kip/ft

F1 = Area1
1
=
(12 ft)(2 kip/ft)
2
= 12 kip

(1)
A

3
(2)
2 kip/ft

F2 = Area2
1
=
(9 ft)(2 kip/ft)
2
= 9 kip

3 ft

9 ft

582

4.6 Distributed Loads on Beams Example 5, page 2 of 3


4

The lines of actions of the resultants pass through the centroids of the loading areas.
5

F1

Centroid of triangle 1 lies


1
(12 ft) = 4 ft from A
3

4 ft

A
F2

3 ft + 2 (9 ft) = 9 ft from A
3

9 ft
3 ft
7

9 ft

Free- body diagram


F1 = 12 kip
4 ft
MA

Ax

Centroid of triangle 2 lies

A
Ay
F2 = 9 kip

9 ft
12 ft

583

4.6 Distributed Loads on Beams Example 5, page 3 of 3


Equations of equilibrium

Fx = 0: Ax = 0
Fy = 0: Ay

MA = 0: MA

12 kip + 9 kip = 0
12 kip(4 ft) + 9 kip(9 ft) = 0

Solving gives
Ax = 0

Ans.

Ay = 3 kip

Ans.

MA = 33 kip ft

Ans.

584

4.6 Distributed Loads on Beams Example 6, page 1 of 3


6. Determine the reactions at the supports.
w = wo sin

x
L

w
x

B
L
1

Distributed load diagram. The magnitude and location of the


resultant of the given load will be determined by integration.
w = wo sin

x
L

dA = w dx
x

B
dx

The resultant force R =


R=

L
wo
0

dA =

L
0

w dx

x L 2woL
L
sin x dx = wo[
cos
] =
L 0
L

585

4.6 Distributed Loads on Beams Example 6, page 2 of 3


3

Resultant force diagram. The location xR of R measured from A is


x dA
xR =
=
dA

L
0x

wo sin Lx dx
=L
2
2woL

xR

B
L

Free- body diagram


R=

2woL

Ax

By

Ay
L
2

L
2

586

4.6 Distributed Loads on Beams Example 6, page 3 of 3


5

Equations of equilibrium

Fx = 0: Ax = 0
Fy = 0: Ay + By

MA = 0: ByL

2woL
(

=0

2woL L
)
=0
2

Solving gives
Ax = 0

Ans.

Ay = woL

Ans.

By = woL

Ans.

587

4.6 Distributed Loads on Beams Example 7, page 1 of 2


7. Determine the reactions at the supports.
w = 2 x2

2x + 4 kN/m
14.4 kN/m

40 kN

w
4 kN/m
A

B
3m

5m

1 Distributed load diagram. The magnitude and location of the force


represented by the distributed load will be determined by integration.
w = 2 x2

2x + 4 kN/m
14.4 kN/m

w
4 kN/m

dA = w dx

B
x

dx

8m

588

4.6 Distributed Loads on Beams Example 7, page 2 of 2


3

2 The force F = dA = w dx
F=

8
02

x2

Free- body diagram


40 kN

2x + 4 dx = 60.44 kN
Ax

and F acts through the point defined by x1:


x1 =

x dA
dA

F = 60.44 kN

8
0

2x x2 2x + 4 dx 305.09
=
= 5.05 m
60.44
60.44

B
Ay

By

3m
5.05 m

Equations of equilibrium

B
3m

Fx = 0: Ax = 0

8m

F = 60.44 kN

Fy = 0: Ay + By

40 kN

MA = 0: By(8 m)

x1
40 kN

60.44 kN = 0

40 kN(3 m)

60.44 kN(5.05 m) = 0

Solving gives

589

Ax = 0

Ans.

Ay = 47.3 kN

Ans.

By = 53.1 kN

Ans.

4.6 Distributed Loads on Beams Example 8, page 1 of 3


8. The lift force acting on an airplane wing can be modeled by
the equation shown. Determine the force and moment at the
point where the wing is attached to the fuselage.

3m

p(x) = [1500

Distributed load diagram. The magnitude and location of


the resultant force will be determine by integration.
x

10(x2 + 4)] N/m

3m
dx
dA = p(x) dx
p(x) = [1500

10(x2 + 4)] N/m

590

4.6 Distributed Loads on Beams Example 8, page 2 of 3


2

The resultant force R is equal to the area under the loading curve.
R=

dA =

3
0

[1500

10(x2 + 4)] dx = 4290 N

The line of action of R passes through the point defined by xR :


3
0

x dA
xR =
=
dA

x[1500

10(x2 + 4)] dx 6368


=
= 1.484 m
4290
4290
_
x

3m

FR

Free- body diagram


1.484 m

Ax
MA

Ay

4290 N
3m

591

4.6 Distributed Loads on Beams Example 8, page 3 of 3


Equations of equilibrium
+

Fx = 0: Ax = 0

Fy = 0: Ay + 4290 N = 0

MA = 0: MA + (4290 N)(1.484 m) = 0
Solving gives
Ax = 0

Ans.

Ay = 4290 N = 4.29 kN

Ans.

MA = 6366 N m = 6.37 kN m

Ans.

592

4.6 Distributed Loads on Beams Example 9, page 1 of 3


9. Determine the reactions at the supports.
120 lb/ft
B

360 lb/ft
9 ft
1

4 ft

Distributed load diagram. The distance x will be determined later.


2

120 lb/ft

(1)

F1 = Area1
1
=
(120 lb/ft)(x)
2
9 ft x
B

A
x

3
(2)

593

360 lb/ft

F2 = Area2
1
=
(360 lb/ft)(9 ft
2

x)

4.6 Distributed Loads on Beams Example 9, page 2 of 3


4

From similar triangles,

x
9 x
=
120 lb/ft
360 lb/ft

Solving gives
x = 2.25 ft.
1
(120 lb/ft)(2.25 ft) = 135 lb.
2
1
The force represented by lower triangle F2 =
(360 lb/ft)(9 ft 2.25 ft) = 1215 lb.
2
The force represented by upper triangle F1 =

The lines of action of these forces pass through the centroids of the corresponding areas.
1
(2.25 m) = 0.75 m from A
6 Centroid of triangle 1 lies
3
F1 = 135 lb
0.75 ft
9 ft 2.25 ft = 6.75 f t
B

A
2.25 ft

F2 = 1215 lb

6.75 ft
7

Centroid of rectangle 2 lies


2.25 ft +

594

2
(6.75 ft) = 6.75 ft from A
3

4.6 Distributed Loads on Beams Example 9, page 3 of 3


8

Free-body diagram
0.75 ft

Ax

135 lb

By

Ay
1215 lb
6.75 ft
13 ft
9
+

Fx = 0: Ax = 0

Fy = 0: Ay

Equations of equilibrium

MA = 0:

135 lb + 1215 lb

By = 0

(135 lb)(0.75 ft) + (1215 lb)(6.75 ft)

By(13 ft) = 0

Solving we get
Ax = 0

Ans.

Ay = 457 lb

Ans.

By = 623 lb

Ans.

595

4.6 Distributed Loads on Beams Example 10, page 1 of 3


10. Determine the reactions at the supports.
4 kip/ft
3 kip ft
A

B
6 ft

2 ft

2 ft

Distributed load diagram

Resultant
F = Area

4 kip/ft

B
6 ft

596

1
(4 kip/ft)(6 ft)
2

= 12 kip

4.6 Distributed Loads on Beams Example 10, page 2 of 3


3

The line of action of the resultant F passes through the centroid of the loading area.
4

4 ft
F

Centroid of triangle lies


2
(6 ft) = 4 ft from A
3

4 kip/ft

B
6 ft

Free-body diagram
4 ft

F = 12 kip
3 kip ft

Ax

A
B
Ay

By
6 ft + 2 ft = 8 ft

2 ft

597

4.6 Distributed Loads on Beams Example 10, page 3 of 3


Equations of equilibrium
+

Fx = 0: Ax = 0

Fy = 0: Ay

MA = 0:

12 kip + By = 0

(12 kip)(4 ft)

(3 kip ft) + By(6 ft + 2 ft + 2 ft) = 0

Solving gives
Ax = 0

Ans.

Ay = 6.90 kip

Ans.

By = 5.10 kip

Ans.

598

4.6 Distributed Loads on Beams Example 11, page 1 of 3


11. Determine w1 and w2 so that the beam is in equilibrium.
8 kN

w1

w2
4m

2m

Resultant of load No. 1


F1 = Area1

Distributed load diagram. The trapezoidal area can


be divided into rectangular and triangular areas.

= (w1)(6 m)
8 kN

= 6w1
3 Resultant of load No. 2
B

A
w1

F2 = Area2
1
=
(w2
2

(1)
(2)

= 3(w2
w2

6m

599

w1

w1)(6 m)
w1)

4.6 Distributed Loads on Beams Example 11, page 2 of 3


The lines of action of the resultants pass through the
centroids of their areas.
8 kN

6w1

3(w2

w1)

3m
4m
5

6 Centroid of triangle 2 lies


2
(6 m) = 4 m from A
3

Centroid of rectangle 1 lies


6m
= 3m from A
2
8 kN

Free-body diagram

6w1

3(w2

w1)

3m
4m

600

4.6 Distributed Loads on Beams Example 11, page 3 of 3

Equations of equilibrium
Fy = 0: 6w1 + 3(w2

w1)

MA = 0: 6w1(3 m) + 3(w2

8 kN = 0
w1)(4 m)

8 kN(4 m) = 0

Solving gives
w1 = 0

Ans.

w2 = 83 kN/m

Ans.

601

4.6 Distributed Loads on Beams Example 12, page 1 of 3


12. The forces from the supports are approximately represented as
triangular distributed loads. Determine the values of w1 and w2.
3 kN/m
8 kN m
A

B
C
w2
w1
0.3 m

0.4 m

4m

1m
1

2m

1m

Distributed load diagram.


3 kN/m

(3)

B
C

(1)
w1

w2
2

F1 = Area1
1
=
w (0.4 m)
2 1
= 0.2w1

F3 = Area3
= (3 kN/m)(4 m)
= 12 kN

602

(2)
4

F2 = Area2
1
=
w (0.3 m)
2 2
= 0.15w2

4.6 Distributed Loads on Beams Example 12, page 2 of 3


The lines of action of the resultant forces pass
through the centroids of their loading areas.

6
3.4 m

Centroid of rectangle lies

6.4 m
0.4 m + 1 m +

12 kN

4m
= 3.4 m from A
2

8 kN m

B
C

0.2 w1

0.15 w2

0.267 m
8.5 m
7

Centroid of triangle 1 lies


2
(0.4 m) = 0.267 m from A
3

Centroid of triangle 2 lies


0.4 m + 1 m + 4 m + 1 m + 2 m + 1 (0.3 m) = 8.5 m from A
3

603

4.6 Distributed Loads on Beams Example 12, page 3 of 3


9

Free- body diagram


6.4 m
3.4 m

12 kN

8 kN m

B
C

0.2 w1

0.15 w2

0.267 m
8.5 m

Fy = 0: 0.2w1 + 0.15w2

10 Equations of equilibrium
12 kN = 0

MA = 0: (0.2w1)(0.267 m)

(12 kN)(3.4 m)

8 kN m + (0.15w2 )(8.5 m) = 0

Solving gives
w1 = 32.3 kN/m

Ans.

w2 = 36.9 kN/m

Ans.

604

5. Equilibrium of a Rigid Body

605

5.1 Constraints and Static Determinacy

606

5.1 Constraints and Static Determinacy Procedures and Strategies, page 1 of 1


C

Procedures and Strategies for Solving Problems Involving Constraints and


Static Determinacy
To determine the type of constraints,
1. draw a free-body diagram but do not include the applied forces
affect the type of constraint and

they do not

RC

2. count the number of reaction components:


a) If the number of components is two or less, then the body is partially
constrained.

C
A

b) If the number of components is three or more, and if either 1) all force


components are parallel, or 2) the lines of action of all force components
pass through the same point and no reaction couple-moment is present, then
the body is improperly constrained.

RA

To determine if a body that is known to be in equilibrium under a set of applied loads


is statically determinate or indeterminate,
1. draw a free-body diagram.
y

b) If the number is three or less, write three equilibrium equations and attempt
to solve them. If a unique solution can be found, then the body is determinate.
Otherwise it is indeterminate.

607

RB

c) If the body is neither partially nor improperly constrained, then it is


properly constrained .

2. count the number of reaction components:


a) If the number is greater than three, the body is indeterminate.

RC
C

x
RAx

A
RAy

5.1 Constraints and Static Determinacy Problem Statement for Example 1


1. In each case, determine if the beam is partially,
improperly, or properly constrained.
2 kN

a)
C

A
500 N
45
b)
A

B
200 lb

c)
A

2 kN m
d)
A

C
B

608

5.1 Constraints and Static Determinacy Problem Statement for Example 2


2. In each case, determine if the rigid body is partially,
improperly, or properly constrained.

a)

C
6 kN m

3 kN

b)

B
C

300 N
4m
c)

4m
C

A
45

45

609

5.1 Constraints and Static Determinacy Problem Statement for Example 3


3. In each case, determine if the rectangular plate is
partially, improperly, or properly constrained.
10 kN

a)

10 kN

c)

400 lb

b)

d)

70 lb

610

5.1 Constraints and Static Determinacy Problem Statement for Example 4


4. In each case, determine if the rectangular plate is
partially, improperly, or properly constrained.
900 N
a)

c)
D

45

B
A
4 kN

200 N
b)
C

611

45

45

5.1 Constraints and Static Determinacy Problem Statement for Example 5


5. In each case, determine if the rigid body is
partially, improperly, or properly constrained.
200 N

a)

45
Semicircles with
common center at O

6N

b)

20 N

B
A
C

612

5.1 Constraints and Static Determinacy Problem Statement for Example 6


6. In each case, determine if the rigid body is
partially, improperly, or properly constrained.
10 lb

Collar slides freely


on smooth rod

a)
D

45

b)

Pin slides freely


in smooth slot
B

C
45
C
45
50 N

613

5.1 Constraints and Static Determinacy Problem Statement for Example 7


7. Determine if the rigid body is partially,
improperly, or properly constrained.

45

45
A

20 kN

3m

45

3m

45

614

5.1 Constraints and Static Determinacy Problem Statement for Example 8


8. In each case, determine if the rigid body is
statically determinate or indeterminate.
a)
6 kip

4 ft
A

5 ft

3 ft

4 ft

4 kip

b)

B
4 ft
A

5 ft

3 ft

4 ft

615

5.1 Constraints and Static Determinacy Problem Statement for Example 9


9. In each case, determine if the rigid body
is statically determinate or indeterminate.
a)

400 N
1m

2m

B
4m

b)

2 kN

2 kN

C
D

2m

2m

2m

616

5.1 Constraints and Static Determinacy Problem Statement for Example 10


b)

10. In each case, determine if the rigid body


is statically determinate or indeterminate.
a)

Collar slides freely


on smooth rod

A
2m
B

3m

3m

600 N

400 N

5 kN
45

2m

6m

E
C
D
C
D
3m

3m

40 N

c)

B
E

6m

A
2m
1m

D
4m

617

C
2m

5.1 Constraints and Static Determinacy Example 1, page 1 of 9


1. In each case, determine if the beam is partially,
improperly, or properly constrained.
2 kN

a)
C

A
500 N
45
b)
A

B
200 lb

c)
A

2 kN m
d)
A

C
B

618

5.1 Constraints and Static Determinacy Example 1, page 2 of 9


Definitions: a rigid body in two dimensions is said to be

Partially constrained, if it has two or fewer reaction


components (There are not enough reactions to prevent
motion under all possible loading conditions);
Improperly constrained, if it has three or more reaction
components and either a) the reaction forces are all parallel,
or b) the lines of action of the reaction forces intersect at a
common point, and no couple moment is present (There are
enough reactions, but they are not properly arranged to
prevent motion under all possible loading conditions); and
Properly constrained, if it is neither partially nor
improperly constrained (There are enough reactions and they
are properly arranged to prevent motion under all possible
loading conditions):

619

5.1 Constraints and Static Determinacy Example 1, page 3 of 9


2 kN
a)

Show the reactions. But do not show the applied 2-kN


force: the reaction forces, not the applied forces,
determine the constraints of the body.

RA

RC

RB

Two or fewer reactions?


No. Conclusion: not
partially constrained.

620

5.1 Constraints and Static Determinacy Example 1, page 4 of 9

RA

RC

RB

All reactions parallel?


Yes. Conclusion:
improperly constrained

Ans.

(Horizontal translation not prevented.)

621

5.1 Constraints and Static Determinacy Example 1, page 5 of 9


500 N
b)

45
A

Show the reactions.


MB
RBx
B

RBy

622

Two or fewer reactions?


No. Conclusion: not
partially constrained.

5.1 Constraints and Static Determinacy Example 1, page 6 of 9


MB
RBx

3 All reactions parallel?


No. Conclusion: can't translate
(There is no direction in which the
body could translate without the
translation being opposed by one
or more of the reaction forces).
5

RBy
4
MB
RBx
B

Lines of action intersect


in common point?
Yes but a moment
reaction is also present
and it prevents rotation.
Conclusion: can't
rotate.

RBy

623

Because the body is neither


partially constrained nor
improperly constrained (can
neither translate nor rotate), it is
properly constrained
All motion is prevented.

Ans.

5.1 Constraints and Static Determinacy Example 1, page 7 of 9


200 lb
c)

1 Show the reactions.


A

RBx

RA

RBy

2 Two or fewer reactions?


No. Conclusion: not
partially constrained.

624

5.1 Constraints and Static Determinacy Example 1, page 8 of 9


A

RBx

RA

RBy

All reactions parallel?


No. Conclusion: can't translate
(There is no direction in which the
body could translate without the
translation being opposed by one
or more of the reaction forces).

RBx

5
RA

RBy

Lines of action intersect


in common point?
No. Conclusion: can't
rotate (For example,
RA prevents rotation
about point B).

625

Because the body is neither partially


constrained nor improperly
constrained (can neither translate
nor rotate), it is
properly constrained
All motion is prevented.

Ans.

5.1 Constraints and Static Determinacy Example 1, page 9 of 9


d)

2 kN m

Show the reactions.

RC

RA

Two or fewer reactions?


Yes. Conclusion:
partially constrained

Ans.

(Horizontal translation is not prevented.)


A

626

5.1 Constraints and Static Determinacy Example 2, page 1 of 9


2. In each case, determine if the rigid body is partially,
improperly, or properly constrained.

a)

C
6 kN m

3 kN

b)

B
C

300 N
4m
c)

4m
C

A
45

45

627

5.1 Constraints and Static Determinacy Example 2, page 2 of 9


Definitions: a rigid body in two dimensions is said to be

Partially constrained, if it has two or fewer reaction


components (There are not enough reactions to prevent
motion under all possible loading conditions);
Improperly constrained, if it has three or more reaction
components and either a) the reaction forces are all parallel,
or b) the lines of action of the reaction forces intersect at a
common point, and no couple moment is present (There are
enough reactions, but they are not properly arranged to
prevent motion under all possible loading conditions); and
Properly constrained, if it is neither partially nor
improperly constrained (There are enough reactions and they
are properly arranged to prevent motion under all possible
loading conditions):

628

5.1 Constraints and Static Determinacy Example 2, page 3 of 9

a)
A

B
C
6 kN m

Show the reactions.


MC

RC

RA
2

Two or fewer reactions?


No. Conclusion: not
partially constrained.

629

5.1 Constraints and Static Determinacy Example 2, page 4 of 9


MC
RC

A
C

RA

All reactions parallel?


No. Conclusion: can't translate
(There is no direction in which the
body could translate without the
translation being opposed by one
or more of the reaction forces).
5

MC
RC

A
C
4
RA

Lines of action intersect in


common point? Yes (at
point A) but a moment
reaction is also present and
it prevents rotation.
Conclusion: can't rotate.

630

Because the body is neither


partially constrained nor improperly
constrained (can neither translate
nor rotate), it is
properly constrained
All motion is prevented.

Ans.

5.1 Constraints and Static Determinacy Example 2, page 5 of 9


3 kN
A

b)

B
C

Show the reactions.


MC
RDx

RA

RDy

RC

Two or fewer reactions?


No. Conclusion: not
partially constrained.

631

5.1 Constraints and Static Determinacy Example 2, page 6 of 9


MC
RDx
A

RA

RDy

RC

All reactions parallel?


No. Conclusion: can't translate
(There is no direction in which the
body could translate without the
translation being opposed by one
or more of the reaction forces).
MC

RDx
A

RA
RA

RDy

RC

Because the body is neither


partially constrained nor improperly
constrained (can neither translate
nor rotate), it is
properly constrained
All motion is prevented.

Lines of action intersect in common


point? No. Conclusion: can't
rotate.

632

Ans.

5.1 Constraints and Static Determinacy Example 2, page 7 of 9


300 N

c)
4m

4m

C
B

45

Show the reactions.

4m
A

45

4m
C

45

45
RC

RA
RB
2 Two or fewer reactions?
No. Conclusion: not
partially constrained.

633

5.1 Constraints and Static Determinacy Example 2, page 8 of 9


4m

4m

45

45
RC

RA
RB

All reactions parallel?


No. Conclusion: can't translate
(There is no direction in which the
body could translate without the
translation being opposed by one
or more of the reaction forces).

634

5.1 Constraints and Static Determinacy Example 2, page 9 of 9


D

4
4m
A

4m
B

improperly constrained

C
45

45

RC
RA
RB
D

Lines of action intersect in common point?


Yes. Conclusion:

B
45

45

635

Ans.

Small (actually infinitesimal) rotation


about D is not prevented. The short links
at A and C would have to change
orientation before they could restrain the
rotation; they could not restrain the
rotation in their original position.

5.1 Constraints and Static Determinacy Example 3, page 1 of 9


3. In each case, determine if the rectangular plate is
partially, improperly, or properly constrained.
10 kN

a)

10 kN

c)

400 lb

b)

d)

70 lb

636

5.1 Constraints and Static Determinacy Example 3, page 2 of 9


Definitions: a rigid body in two dimensions is said to be

Partially constrained, if it has two or fewer reaction


components (There are not enough reactions to prevent
motion under all possible loading conditions);
Improperly constrained, if it has three or more reaction
components and either a) the reaction forces are all parallel,
or b) the lines of action of the reaction forces intersect at a
common point, and no couple moment is present (There are
enough reactions, but they are not properly arranged to
prevent motion under all possible loading conditions); and
Properly constrained, if it is neither partially nor
improperly constrained (There are enough reactions and they
are properly arranged to prevent motion under all possible
loading conditions):

637

5.1 Constraints and Static Determinacy Example 3, page 3 of 9


10 kN
a)

Show the reactions. But do not show the


applied 10-kN force: the reaction forces, not
the applied forces, determine the constraints of
the body.
C
D
R
D

RAx

A
B
2
RAy

Two or fewer reactions?


No. Conclusion: not
partially constrained.

638

5.1 Constraints and Static Determinacy Example 3, page 4 of 9

3
C

D
RD

RAx

A
B

RAy

D
RD

RAx

All reactions parallel?


No. Conclusion: can't
translate (There is no
direction in which the
body could translate
without the translation
being opposed by one
or more of the reaction
forces).

Lines of action intersect


in common point?
No. Conclusion: can't
rotate (For example,
RD prevents rotation
about A).

A
B

RAy

639

5 Because the body is neither


partially constrained nor
improperly constrained (can
neither translate nor rotate), it is
properly constrained
All motion is prevented.

Ans.

5.1 Constraints and Static Determinacy Example 3, page 5 of 9


b)

70 lb

FD

Show the reactions.


C

D
2

FA

Two or fewer reactions?


Yes. Conclusion:
partially constrained

Ans.

Small (actually infinitesimal) rotation about B is not


prevented.
C
D
A
B

640

5.1 Constraints and Static Determinacy Example 3, page 6 of 9


c)

10 kN

Show the reactions.

RD

RB

RA

641

Two or fewer reactions?


No. Conclusion: not
partially constrained.

5.1 Constraints and Static Determinacy Example 3, page 7 of 9


3
C

RD

All reactions parallel?


No. Conclusion: can't translate
(There is no direction in which the
body could translate without the
translation being opposed by one
or more of the reaction forces).

RB
5 Because the body is neither
partially constrained nor
improperly constrained (can
neither translate nor rotate), it is

RA
4
C

RD

Lines of action intersect


in common point?
No. Conclusion: can't
rotate (For example,
RD prevents rotation
about A).

RB

RA

642

properly constrained
All motion is prevented.

Ans.

5.1 Constraints and Static Determinacy Example 3, page 8 of 9


400 lb
d)
D

1
RC

Show the reactions.


RD

RA

2 Two or fewer reactions?


No. Conclusion: not
partially constrained.

RB

643

5.1 Constraints and Static Determinacy Example 3, page 9 of 9


RC

RD

RA
A

RC

RD

RA
A

3 All reactions parallel?


No. Conclusion: can't translate
(There is no direction in which the
body could translate without the
translation being opposed by one
or more of the reaction forces).
RB

4 Lines of action intersect


in common point?
No. Conclusion: can't
rotate (For example,
RD prevents rotation
about A).

RB

644

5 Because the body is neither


partially constrained nor
improperly constrained (can
neither translate nor rotate), it is
properly constrained
All motion is prevented.

Ans.

5.1 Constraints and Static Determinacy Example 4, page 1 of 8


4. In each case, determine if the rectangular plate is
partially, improperly, or properly constrained.
900 N
a)

c)
D

45

B
A
4 kN

200 N
b)
C

645

45

45

5.1 Constraints and Static Determinacy Example 4, page 2 of 8


Definitions: a rigid body in two dimensions is said to be

Partially constrained, if it has two or fewer reaction


components (There are not enough reactions to prevent
motion under all possible loading conditions);
Improperly constrained, if it has three or more reaction
components and either a) the reaction forces are all parallel,
or b) the lines of action of the reaction forces intersect at a
common point, and no couple moment is present (There are
enough reactions, but they are not properly arranged to
prevent motion under all possible loading conditions); and
Properly constrained, if it is neither partially nor
improperly constrained (There are enough reactions and they
are properly arranged to prevent motion under all possible
loading conditions):

646

5.1 Constraints and Static Determinacy Example 4, page 3 of 8


a)
C

B
4 kN

1
RC

Show the reactions.


RD

B
RA

2 Two or fewer reactions?


No. Conclusion: not
partially constrained.

RB

647

5.1 Constraints and Static Determinacy Example 4, page 4 of 8


RC

RD

All reactions parallel?


Yes. Conclusion:
improperly constrained

(Horizontal translation
is not prevented.)

RA

RB

648

Ans.

5.1 Constraints and Static Determinacy Example 4, page 5 of 8


200 N

b)
C

Show the reactions.

RC
2

Two or fewer reactions?


No. Conclusion: not
partially constrained.

RA

RB

RC
3
C

RA

RB

649

All reactions parallel?


No. Conclusion: can't translate
(There is no direction in which the
body could translate without the
translation being opposed by one
or more of the reaction forces).

5.1 Constraints and Static Determinacy Example 4, page 6 of 8


RC

RA
A

RB
4

Lines of action intersect in common point?


Yes. Conclusion:
improperly constrained

Ans.

Small (actually infinitesimal) rotation about


D is not prevented.
C
A
D

650

5.1 Constraints and Static Determinacy Example 4, page 7 of 8


900 N
c)
C

45

45
A

45

2
1

Show the reactions.


C

RD
45

45
RA

45
RB

651

Two or fewer reactions?


No. Conclusion: not
partially constrained.

5.1 Constraints and Static Determinacy Example 4, page 8 of 8


RD
C

45

3
A

45

45
RB

RA

All reactions parallel?


Yes. Conclusion:
improperly constrained

Ans.

Translation in a direction 45 from the


horizontal is not prevented. The short
links at A, B, and D would have to
change orientation before they could
restrain the translation; they could not
restrain the translation in their original
position.

A
B
45

652

5.1 Constraints and Static Determinacy Example 5, page 1 of 7


5. In each case, determine if the rigid body is
partially, improperly, or properly constrained.
200 N

a)

45
Semicircles with
common center at O

6N

b)

20 N

B
A
C

653

5.1 Constraints and Static Determinacy Example 5, page 2 of 7


Definitions: a rigid body in two dimensions is said to be

Partially constrained, if it has two or fewer reaction


components (There are not enough reactions to prevent
motion under all possible loading conditions);
Improperly constrained, if it has three or more reaction
components and either a) the reaction forces are all parallel,
or b) the lines of action of the reaction forces intersect at a
common point, and no couple moment is present (There are
enough reactions, but they are not properly arranged to
prevent motion under all possible loading conditions); and
Properly constrained, if it is neither partially nor
improperly constrained (There are enough reactions and they
are properly arranged to prevent motion under all possible
loading conditions):

654

5.1 Constraints and Static Determinacy Example 5, page 3 of 7


200 N
a)
C

45
Semicircles with
common center at O

Show the reactions. But do not show the


applied 200-N force: the reaction forces, not
the applied forces, determine the constraints of
the body.

RA
A

RC
45

RB

655

Two or fewer reactions?


No. Conclusion: not
partially constrained.

5.1 Constraints and Static Determinacy Example 5, page 4 of 7


O

RA
A

RC
45

C
3

RB

656

All reactions parallel?


No. Conclusion: can't translate
(There is no direction in which the
body could translate without the
translation being opposed by one
or more of the reaction forces).

5.1 Constraints and Static Determinacy Example 5, page 5 of 7


4

improperly constrained

RA
A

Lines of action intersect in common point?


Yes. Conclusion:

45

RC

Rotation about O is not prevented.

RB

O
45

657

Ans.

5.1 Constraints and Static Determinacy Example 5, page 6 of 7


6N
b)
D

20 N
B
A
C

RD
1

Show the reactions.

C
RA

RC

658

Two or fewer reactions?


No. Conclusion: not
partially constrained.

5.1 Constraints and Static Determinacy Example 5, page 7 of 7


D

RD

3
A

RA

RC

All reactions parallel?


No. Conclusion: can't translate
(There is no direction in which
the body could translate
without the translation being
opposed by one or more of the
reaction forces).

5 Because the body is neither


partially constrained nor
improperly constrained (can
neither translate nor rotate), it is
properly constrained

RD
All motion is prevented.
4
A

RA

Lines of action intersect


in common point?
No. Conclusion: can't
rotate (For example,
RD prevents rotation
about A).

RC

659

Ans.

5.1 Constraints and Static Determinacy Example 6, page 1 of 7


6. In each case, determine if the rigid body is
partially, improperly, or properly constrained.
10 lb

Collar slides freely


on smooth rod

a)
D

45

b)

Pin slides freely


in smooth slot
B

C
45
C
45
50 N

660

5.1 Constraints and Static Determinacy Example 6, page 2 of 7


Definitions: a rigid body in two dimensions is said to be

Partially constrained, if it has two or fewer reaction


components (There are not enough reactions to prevent
motion under all possible loading conditions);
Improperly constrained, if it has three or more reaction
components and either a) the reaction forces are all parallel,
or b) the lines of action of the reaction forces intersect at a
common point, and no couple moment is present (There are
enough reactions, but they are not properly arranged to
prevent motion under all possible loading conditions); and
Properly constrained, if it is neither partially nor
improperly constrained (There are enough reactions and they
are properly arranged to prevent motion under all possible
loading conditions):

661

5.1 Constraints and Static Determinacy Example 6, page 3 of 7


10 lb
a)
D

Show the reactions.

RD
2 Two or fewer reactions?
No. Conclusion: not
partially constrained.

B
RA

C
RB

RC

662

5.1 Constraints and Static Determinacy Example 6, page 4 of 7


RD
D

All reactions parallel?


Yes. Conclusion:
improperly constrained

RA

RB

RC
(Horizontal translation
is not prevented.)

663

Ans.

5.1 Constraints and Static Determinacy Example 6, page 5 of 7

b)
Collar slides freely
on smooth rod

45
Pin slides freely
in smooth slot
B

45
C
45
50 N

664

5.1 Constraints and Static Determinacy Example 6, page 6 of 7


1
RA

Show the reactions.

45

2
A

Two or fewer reactions?


No. Conclusion: not
partially constrained.

RB

45
B
RA
45

45

All reactions parallel?


No. Conclusion: can't translate
(There is no direction in which the
body could translate without the
translation being opposed by one
or more of the reaction forces).

A
RB

45 R
C

45
B

45

C
45

665

RC

5.1 Constraints and Static Determinacy Example 6, page 7 of 7


RA
45
4

A
RB

improperly constrained

45

45
45

RC
B

C
45

666

Ans.

Small (actually infinitesimal) rotation


about B is not prevented. The collar at A
and the pin at C would have to move
slightly before they could restrain the
rotation; they could not restrain the
rotation in their original position.

45

Lines of action intersect in common point?


Yes. Conclusion:

5.1 Constraints and Static Determinacy Example 7, page 1 of 4


7. Determine if the rigid body is partially,
improperly, or properly constrained.

45

45
A

20 kN

3m

45

3m

45

667

5.1 Constraints and Static Determinacy Example 7, page 2 of 5


Definitions: a rigid body in two dimensions is said to be

Partially constrained, if it has two or fewer reaction


components (There are not enough reactions to prevent
motion under all possible loading conditions);
Improperly constrained, if it has three or more reaction
components and either a) the reaction forces are all parallel,
or b) the lines of action of the reaction forces intersect at a
common point, and no couple moment is present (There are
enough reactions, but they are not properly arranged to
prevent motion under all possible loading conditions); and
Properly constrained, if it is neither partially nor
improperly constrained (There are enough reactions and they
are properly arranged to prevent motion under all possible
loading conditions):

668

5.1 Constraints and Static Determinacy Example 7, page 3 of 4


1

Show the reactions.

RA

RC
A

45

45

2
3m

D
45

45

RA

RC

3m
RD

RB
A

45

45

3m

D
45

45
3m
RB

Two or fewer reactions?


No. Conclusion: not
partially constrained.

RD

669

All reactions parallel?


No. Conclusion: can't translate
(There is no direction in which the
body could translate without the
translation being opposed by one
or more of the reaction forces).

5.1 Constraints and Static Determinacy Example 7, page 4 of 4


RA

RC
A

45

45

improperly constrained

3m

D
45

45

45

3m
RB

Ans.

Small (actually infinitesimal) rotation about E is not


prevented. The short links at A, B, C, and D would
have to change orientation before they could restrain
the rotation; they could not restrain the rotation in
their original position.

Lines of action intersect in common point?


Yes. Conclusion:

45

RD

A
C
3m
E
B
D
3m

45

670

45

5.1 Constraints and Static Determinacy Example 8, page 1 of 7


8. In each case, determine if the rigid body is
statically determinate or indeterminate.
a)
6 kip

4 ft
A

5 ft

3 ft

4 ft

4 kip

b)

B
4 ft
A

5 ft

3 ft

4 ft

671

5.1 Constraints and Static Determinacy Example 8, page 2 of 7


Definitions: a rigid body in equilibrium under a given loading is said to be

Statically determinate, if a unique set of reaction components can


be found by solving the equations of statics alone (No additional
equations such as are introduced in a course in mechanics of
materials are needed); or
Statically indeterminate, if not all reaction components can be
found uniquely by solving the equations of statics alone.

672

5.1 Constraints and Static Determinacy Example 8, page 3 of 7


a)
6 kip

4 ft
A

5 ft

3 ft

4 ft

Draw a free-body diagram.


6 kip

By

Cy

4 ft
A

statically indeterminate.
3

Ax
Ay

5 ft

3 ft

There are four unknown reactions: Ax Ay, By, and Cy.


But there are at most three equations of equilibrium
for a body in two dimensions. Because there are
more unknowns than equations, a unique solution
cannot be found; the rigid body is

4 ft

673

Ans.

Observation No. 1: In situations such as this, it is


tempting to write four equilibrium equations by, for
example, writing two force equations and moment
equations about two different points. But this
approach will not work, as will now be shown.

5.1 Constraints and Static Determinacy Example 8, page 4 of 7


6 kip

By

Cy

4 ft
A
Ax

4
4 ft

Fx = 0: Ax + 6 kip = 0

(1)

3 ft

Fy = 0: Ay + By + Cy = 0

(2)

5 ft

Write four equilibrium equations:

MA = 0: By(5 ft + 3 ft) + Cy(5 ft + 3 ft + 4 ft)


(6 kip)(4 ft) = 0

(3)

MC = 0: Ax(4 ft)

(4)

Ay

Ay(5 ft + 3 ft + 4 ft)

By(4 ft) = 0

We will now show that Eq. 4 is not independent of Eqs. 1-3. Multiply Eq. 1 by
4, Eq. 2 by 12, Eq. 3 by 1 and add the results to obtain
4(Ax + 6)

12(Ay + By + Cy) + [By(5 + 3) + Cy(5 + 3 + 4)

(6)(4)] = 0

The last equation simplifies to


Ax(4)

Ay(12)

By(4) + Cy( 12 + 5 + 3 + 4) + [4(6)

6(4)] = 0

which is equivalent to Eq. 4. This is a general result: You can write as many
equilibrium equations as you want, but at most only three of them will be
independent (for two-dimensional problems).

674

5.1 Constraints and Static Determinacy Example 8, page 5 of 7


6

Observation No. 2: The difficulty is not that we can't find a solution. We can find
as many solutions as we want. For example, here are two:
Solution No. 1: Ax = 6 kip, Ay =

3 kip, By = 3 kip, and Cy = 0.

Solution No. 2: Ax = 6 kip, Ay =

2 kip, By = 0, and Cy = 2 kip.

The difficulty is that we can't find the unique solution, because it is not
determined by the equations of statics alone, which is another way of saying that
the structure is statically indeterminate. To find the unique solution, we would
have to supplement the equations of statics by introducing an equation describing
material behavior the subject of a course in mechanics of materials.

675

5.1 Constraints and Static Determinacy Example 8, page 6 of 7


4 kip

b)

B
4 ft
A

5 ft

4 ft

There are three unknown reactions, Ay, By, and Cy, and
for a body in two dimensions, there are at most three
equations of equilibrium, so it appears that the body may
be statically determinate. However, let's write the
equilibrium equations as a check:

4 kip

Fx = 0: 0 = 0

Draw a free-body diagram.

Fy = 0: Ay + By + Cy

3 ft

MA = 0: By(5 ft + 3 ft) + Cy(5 ft + 3 ft + 4 ft)


(4 kip)(5 ft) = 0

(1)
4 kip = 0

(2)

(3)

4 ft
By

Only two of the equations are independent, since Eq. 1 is


0 = 0. Conclusion: we do not have enough equations to
solve for the three reactions, so the rigid body is

Cy

A
Ay

5 ft

3 ft

statically indeterminate.

4 ft

676

Ans.

5.1 Constraints and Static Determinacy Example 8, page 7 of 7


3

Observation: The reason that the body is


statically indeterminate even though there are
only three unknown reactions is that the
structure is improperly constrained (all
reactions are parallel). In general, an
improperly constrained body (in equilibrium
under a set of loads) is always statically
indeterminate because one of the equilibrium
equations is always trivial; for example, the
sum of forces in the direction perpendicular to
the (parallel) reactions reduces to 0 = 0, or the
sum of the moments about a point where all
the reaction forces intersect reduces to 0 = 0.
A partially constrained body (in equilibrium
under a set of loads), on the other hand, may
or may not be statically determinate.
4 kN
2 kN

Partially constrained and


statically determinate

677

Partially constrained and


statically indeterminate

5.1 Constraints and Static Determinacy Example 9, page 1 of 4


9. In each case, determine if the rigid body
is statically determinate or indeterminate.
a)

400 N
1m

2m

B
4m

b)

2 kN

2 kN

C
D

2m

2m

2m

678

5.1 Constraints and Static Determinacy Example 9, page 2 of 4


Definitions: a rigid body in equilibrium under a given loading is said to be

Statically determinate, if a unique set of reaction components can


be found by solving the equations of statics alone (No additional
equations such as are introduced in a course in mechanics of
materials are needed); or
Statically indeterminate, if not all reaction components can be
found uniquely by solving the equations of statics alone.

679

5.1 Constraints and Static Determinacy Example 9, page 3 of 4


a)

400 N
1m

2m
A

B
4m
2

400 N

Fx = 0:

Draw a free-body diagram.

Fy = 0: Ay + By = 0

There are three unknowns reactions, Ay, By, and Cx, so the body will
be statically determinate if the equilibrium equations are independent.
Consider the following equilibrium equations:

MA = 0: By(4 m) + Cx(2 m)

1m
Cx
2m

Cx + 400 N = 0

(1)
(2)
(400 N)(2 m + 1 m) = 0

(3)

Solving these equations yields the unique solution


A

B
Ay

By
4m

Ay = 100 N, By = 100 m, and Cx = 400 N


Thus the body is
statically determinate.

680

Ans.

5.1 Constraints and Static Determinacy Example 9, page 4 of 4


b)

2 kN
A

2 kN

C
D

2m

2m

2
Draw a free-body diagram.

MA

Ax

2m

2 kN

2 kN

statically indeterminate.

A
Ay

Dy
2m

2m

There are four unknowns reactions, Ax, Ay, Dy,


and MA, and at most three equations of equilibrium
exist, so the body is

2m

681

Ans.

5.1 Constraints and Static Determinacy Example 10, page 1 of 6


b)

10. In each case, determine if the rigid body


is statically determinate or indeterminate.
a)

Collar slides freely


on smooth rod

A
2m
B

3m

3m

600 N

400 N

5 kN
45

2m

6m

E
C
D
C
D
3m

3m

40 N

c)

B
E

6m

A
2m
1m

D
4m

682

C
2m

5.1 Constraints and Static Determinacy Example 10, page 2 of 6


Definitions: a rigid body in equilibrium under a given loading is said to be

Statically determinate, if a unique set of reaction components can


be found by solving the equations of statics alone (No additional
equations such as are introduced in a course in mechanics of
materials are needed); or
Statically indeterminate, if not all reaction components can be
found uniquely by solving the equations of statics alone.

683

5.1 Constraints and Static Determinacy Example 10, page 3 of 6


a)

Collar slides freely


on smooth rod

A
2m
B

5 kN
45

2m

E
C
D

3m

3m
1

Draw a free-body diagram.


2 There are four unknown reactions, Ax, Cy, Dy, and FE, and
at most three equations of equilibrium exist, so the body is

Ax
2m

statically indeterminate.

5 kN

2m

45

FE

Dy

Cy
3m

3m

684

Ans.

5.1 Constraints and Static Determinacy Example 10, page 4 of 6


b)

3m

3m

400 N

600 N

400 N

3m

3m

600 N
A

Draw a free-body diagram.

6m

6m

C
C

Fx = 0: 0 = 0

There are two unknown reactions, Cy and Dy. Writing equilibrium equations gives

Fy = 0: Cy

Observation: There are only two


unknown reactions because the
body is partially constrained.
However, the loading happens to
be such that the two nontrivial
equations of equilibrium can be
satisfied. If the loading consisted
of a horizontal load, then Eq. 1
could not be satisfied, and the
body would not be in equilibrium.

Dy

Cy
2

(1)
600 N

400 N + Dy = 0

MD = 0: Cy(3 m + 3m)

(600 N)(3 m) = 0

(2)
(3)

Solving these equations gives the unique solution Cy = 300 N and Dy = 700 N, so
the body is
statically determinate.

685

Ans.

5.1 Constraints and Static Determinacy Example 10, page 5 of 6


1
3

40 N

c)

Draw a free-body diagram.


FB

B
E

E
FA

6m

A
C

1m

4m

2m

2m

cos
sin

= 1/ 5

10

= 1/ 10

sin

= 3/ 10

D 1
E

686

2m

FC

There are three unknown reactions, FA, FB, and FC.


Writing equilibrium equations gives

Geometry
cos

4m

Fx = 0: FA cos

A
2

= 2/ 5

1m

2
3

6m

Fy = 0:

FA sin

2m

40 N

MA = 0:

FC + (40 N) cos

=0

(1)

FB + (40 N) sin

=0

(2)

FB(4 m) FC(2 m)
+ (40 N) sin (4 m + 2 m)
(40 N) cos (6 m 2 m) = 0

(3)

5.1 Constraints and Static Determinacy Example 10, page 6 of 6


4

If a calculator is used to invert the matrix of


coefficients in the equations, the calculator will
return an error message. That is, the equations
are redundant and do not have a unique
solution. Thus the body is
statically indeterminate.

Ans.

The source of the redundancy can be seen by


extending the lines of action of all the forces
and observing that they intersect in a common
point. Thus summing moments about this
common point would produce the trivial
equilibrium equation, 0 = 0. Hence this body
has only two independent equations of
equilibrium.

FB

40 N
1

B
E
FA

6m

A
2m

1
D

2
4m

1m

687

C
2m

FC

5.2 Rigid Bodies and Two-Dimensional Force Systems

688

5.2 Rigid Bodies and Two-Dimensional Force Systems Procedures and Strategies, page 1 of 1
Procedures and Strategies for Solving Problems Involving
Equilibrium of a Rigid Body in a Two-Dimensional Force System

1. Draw a free-body diagram by showing all external forces:


a) applied loads and couple moments
b) weight of the body if the weight or mass is given
c) reactions from the supports. If a constraint prevents motion in a certain
direction, then show a reaction force acting in that direction on the
free-body diagram. If a constraint prevents rotation about a certain axis,
then show a reaction couple-moment on the free-body diagram.

Ay
P

Ax
2. Introduce sufficient dimensions on the diagram to allow for calculating the
moments of forces.
3. Write three equations of equilibrium.
Notes:
a) In writing moment equilibrium equations, remember that a couple moment
can be considered to act anywhere on the free-body diagram.

Bx
P
A

b) Do not show internal forces on a free-body diagram because they occur in


equal and opposite sense and thus will cancel out.
c) At most, three equations of equilibrium are independent. If you find that
you have more unknowns than independent equations, then look for an error
in your free-body diagram you may have made a mistake in representing
the reaction forces from the supports. Do not write an additional equilibrium
equation by, for example, summing moments about a new point. Such an
equation will not be independent of the equilibrium equations that you have
already written.

689

MA
Ax
Ay

5.2 Rigid Bodies and Two-Dimensional Force Systems Problem Statement for Example 1
1. Determine the reactions at A and B. The
weight of the rod is negligible.

3 ft
40 lb

B
30

3 ft
30
A

690

5.2 Rigid Bodies and Two-Dimensional Force Systems Problem Statement for Example 2
2. Determine the reactions at A and B. The
weight of the rod is negligible.
A

3 kip
5 ft
B
4 ft

4 ft

691

5.2 Rigid Bodies and Two-Dimensional Force Systems Problem Statement for Example 3
3. Determine the reactions at supports A and C.
A

400 lb
4 ft
30
C

5 ft

692

5.2 Rigid Bodies and Two-Dimensional Force Systems Problem Statement for Example 4
4. The automobile shown is being pushed up the incline at
a constant velocity by a force, P, from a tow truck. The
2,600-lb weight of the car acts at the center of gravity, O,
and the friction forces acting on the wheels are negligible.
Determine the force P and the forces from the ground
acting on the individual tires.

O
20 in.

P
12 in.
10

55 in.

693

15 in.

5.2 Rigid Bodies and Two-Dimensional Force Systems Problem Statement for Example 5
5. Plate ABCD is supported by cord EC and a pin at A.
Determine the tension in the cord and the horizontal and
vertical reactions at A. The weight of the plate is negligible.
E

4 kN

20

2m
5 kNm

B
3m

694

5.2 Rigid Bodies and Two-Dimensional Force Systems Problem Statement for Example 6
6. Member ABC has the shape of a quarter circle centered at O.
Determine the reactions at each of the roller supports.
O

3
ft
45
B
60 lb

A
45

695

5.2 Rigid Bodies and Two-Dimensional Force Systems Problem Statement for Example 7
7. The end of pole DA is buried deep in the ground. The
tension in cable BE is 2 kip and in cable CF is 3 kip.
Determine the reactions at A.
D
6 kip
2 ft

2 ft

5 ft
E

8 ft

12 ft

696

5.2 Rigid Bodies and Two-Dimensional Force Systems Problem Statement for Example 8
8. A smooth slot has been cut in the plate shown, and
a pin, C, fixed to a support behind the plate, fits in
the slot. Determine the forces acting on the plate at
the supports A and B and from pin C.
4 lbft
B
D
1.5 ft
15

C
1.5 ft

A
2 lb
3 ft

2 ft

697

5.2 Rigid Bodies and Two-Dimensional Force Systems Problem Statement for Example 9
9. Pin C is rigidly attached to the plate and can slide freely
in the slot cut in member DB. Determine the reaction
force at the pin support at A and the force transmitted at C.
100 mm

30
425 mm

E
12 Nm

698

5.2 Rigid Bodies and Two-Dimensional Force Systems Problem Statement for Example 10
10. Member ABD is supported by a pin at A and a cord attached at
B and D. The cord passes over a frictionless pulley at C.
Determine the reaction at A and the tension in the cord.
C
3 kNm

A
2m

2m

60 40

3m

5 kN

699

5.2 Rigid Bodies and Two-Dimensional Force Systems Problem Statement for Example 11
11. The rigid bar AB is supported by two rollers attached to its
ends at A and B. If the bar is in equilibrium in the position
shown, determine the inclination, , of the inclined plane.

7 in.

20 lb
3 in.
25
A

700

5.2 Rigid Bodies and Two-Dimensional Force Systems Problem Statement for Example 12
12. The uniform square plate ABC of mass 10 kg is supported
by a vertical cord at B. Determine the tension in the cord and
the forces from the walls if the walls are smooth at the contact
points A and C.
D

20

B
C

701

5.2 Rigid Bodies in Two-Dimensional Force Systems Example 1, page 1 of 1


1. Determine the reactions at A and
B. The weight of the rod is
negligible.

3 ft

40 lb

30

3 ft
30
A
3
1

Free-body diagram of rod AB.

The force from the roller must be perpendicular


to the plane upon which the roller rests.

x
B

40 lb

Equilibrium equations for rod AB:


+

Fx = 0: Ax = 0

Fy = 0: Ay

By

40 lb)(3 ft) + By(3 ft + 3 ft) = 0

3 ft

Ax

5
Ay

MA = 0:

40 lb + By = 0

3 ft

The force components at A can be assumed to act in any


direction we choose. Choosing them in the direction of the
inclined coordinate system simplifies the calculation a little.

702

Solving these equations simultaneously gives


Ax = 0

Ans.

Ay = 20.0 lb

Ans.

By = 20.0 lb

Ans.

5.2 Rigid Bodies in Two-Dimensional Force Systems Example 2, page 1 of 1


2. Determine the reactions at A and B. The
weight of the rod is negligible.
A
Equilibrium equations for member AB:

4
+

3 kip

Fx = 0: Ax + Bx = 0

5 ft

Fy = 0:

4 ft

Ax

Ax(5 ft) + (3 kip)(4 ft) = 0

4 ft

Free-body diagram of member AB


A 2

MB = 0:

3 kip + By = 0

Solving these equations simultaneously gives

The force from the roller at A


must be perpendicular to the plane
upon which the roller rests.

5 ft
3 kip
Bx

B
By
4 ft

4 ft

The pin support at B


provides two force
components.

703

Ax = 2.4 kip

Ans.

Bx = 2.4 kip

Ans.

By = 3.0 kip

Ans.

5.2 Rigid Bodies in Two-Dimensional Force Systems Example 3, page 1 of 3


3. Determine the reactions at supports A and C.
A

400 lb

4 ft
30

5 ft

FA
2

The force from the roller


is perpendicular to the
inclined plane. The
angle will be
determined later.

Free-body diagram of member ABC

400 lb
3
B

The pin support provides two


components of reaction.
C

Cy

704

Cx

5.2 Rigid Bodies in Two-Dimensional Force Systems Example 3, page 2 of 3


4

Free-body diagram showing horizontal and vertical components


of the unknown reaction force FA.
A

FA cos
400 lb

FA sin

4 ft

30

Cx
(4 ft)/tan 30

5 ft

Cy

Fx = 0: FA cos

+ Cx = 0

(1)

Fy = 0: FA sin

+ Cy

400 lb = 0

(2)

FA cos (4 ft) + (400 lb)(5 ft)


FA sin 5 ft 4 ft)/tan 30] = 0

(3)

Equilibrium equations for member ABC:

MC = 0:

705

5.2 Rigid Bodies in Two-Dimensional Force Systems Example 3, page 3 of 3


30

Geometry
7

30 = 60

= 90

30
B

Substituting = 60 in Equations 1, 2, and 3


and solving gives
FA = 162.2 lb

Ans.

Cx = 81.1 lb

Ans.

Cy = 260 lb

Ans.

Free-body diagram showing forces


with correct senses
A
9

60
FA = 162.2 lb

400 lb

We had assumed that Cx pointed to the right, but


solving the equilibrium equations gave a negative
value for Cx, so our assumption was wrong. Cx
points to left as shown.
C
|Cx| = 81.1 lb

Cy = 260 lb

706

5.2 Rigid Bodies in Two-Dimensional Force Systems Example 4, page 1 of 2


4. The automobile shown is being pushed up the incline at
a constant velocity by a force, P, from a tow truck. The
2,600-lb weight of the car acts at the center of gravity, O,
and the friction forces acting on the wheels are negligible.
Determine the force P and the forces from the ground
acting on the individual tires.

O
20 in.

P
12 in.
10

55 in.

15 in.

Free-body diagram of car


y
x
O

It is convenient to use x and y coordinates that


are parallel and perpendicular to the incline,
because the forces from the tow truck and
from the ground acting on the tires are parallel
and perpendicular to the incline.

B
2,600 lb

P
A

2 FB

2 FA

707

If FB is the force acting on each front


wheel, then 2FB is the resultant force
acting on the body.

5.2 Rigid Bodies in Two-Dimensional Force Systems Example 4, page 2 of 2


4

Free-body diagram of car, with dimensions shown


y
x
10

O
20 in

20 in.

12 in = 8 in

2,600 lb

12 in.

2 FB

2 FA

15 in.

55 in.

Equilibrium equations for the car:

Geometry

Fx = 0: P

Fy = 0: 2FA + 2FB

= 90

10 = 80

10

(2,600 lb) cos

MO = 0: P(8 in.)

(2,600 lb) sin

=0

2FA(55 in.) + 2FB(15 in.) = 0

Solving simultaneously, with

708

=0

= 80, gives

P = 451 lb

Ans.

FA = 300 lb

Ans.

FB = 980 lb

Ans.

5.2 Rigid Bodies in Two-Dimensional Force Systems Example 5, page 1 of 3


5. Plate ABCD is supported by cord EC and a pin at A.
Determine the tension in the cord and the horizontal and
vertical reactions at A. The weight of the plate is negligible.
E

4 kN

20

2m
5 kNm

B
3m

709

5.2 Rigid Bodies in Two-Dimensional Force Systems Example 5, page 2 of 3


1 Free-body diagram of plate ABCD
T
4 kN

20

The tension T points away from the


plate and is collinear with the cord.

Equilibrium equations for plate ABCD

2m
5 kNm

Fx = 0: Ax + 4 kN

3m

Ay

Ax

T cos 20 = 0

Fy = 0: Ay + T sin 20 = 0
MA = 0:

4 kN)(2 m) + (T cos 20)(2 m) + (T sin 20)(3 m)

The 5 kNm couple-moment at corner B can be


considered to act anywhere; in particular, then, we can
consider it to act at point A, about which we are
summing moments. So we just insert "5 kNm" into the
equation. The minus sign signifies that the 5 kNm
couple-moment has a sense opposite to what we have
chosen to call a positive sense.

710

5 kNm = 0

5.2 Rigid Bodies in Two-Dimensional Force Systems Example 5, page 3 of 3


5

Solving these equations simultaneously gives


T = 4.47 kN

Free-body diagram of plate ABCD showing


correct senses of forces.

Ans.
T = 4.47 kN

Ax = 0.20 kN

Ans.

Ay = 1.53 kN

Ans.

4 kN

20
C

5 kN m
A
Ax = 0.20 kN

Ay = 1.53 kN
7

Initially we arbitrarily assumed Ay points upward.


Solving the equilibrium equations gave Ay = 1.53 kN.
The minus sign shows that our assumption was wrong,
and the correct sense of Ay is down, as shown on this
free body.

711

5.2 Rigid Bodies in Two-Dimensional Force Systems Example 6, page 1 of 2


6. Member ABC has the shape of a quarter circle centered at O.
Determine the reactions at each of the roller supports.
O

3
ft
45
B
60 lb

A
45

712

5.2 Rigid Bodies in Two-Dimensional Force Systems Example 6, page 2 of 2


1 Free-body diagram of member ABC
FC

3 ft

A
60 lb

45

FA

FB

The force from the


roller must be
perpendicular to the
plane upon which
the roller rests. This
is true at all three
points, A, B, and C.

Equilibrium equations for plate member ABC


+

Fx = 0: 60 lb

Fy = 0: FA + FB cos 45

FB sin 45 = 0

MO = 0: (60 lb)(3 ft)

FC = 0

FC(3 ft) = 0

Solving these equations simultaneously gives


4

The lines of action of the forces FA


and FB go through point O, so FA
and FB have no moment about O.

713

FA = 0

Ans.

FB = 84.9 lb

Ans.

FC = 60.0 lb

Ans.

5.2 Rigid Bodies in Two-Dimensional Force Systems Example 7, page 1 of 3


7. The end of pole DA is buried deep in the ground. The
tension in cable BE is 2 kip and in cable CF is 3 kip.
Determine the reactions at A.
D
6 kip
2 ft

2 ft

5 ft
E

8 ft

12 ft

714

5.2 Rigid Bodies in Two-Dimensional Force Systems Example 7, page 2 of 3


Free-body diagram of pole ABCD
D
6 kip
C

2 ft

TCF = 3 kip

TBE = 2 kip
5 ft

2
A

Ax
Ay
3

The tensions
point away from
the pole and are
collinear with the
cables.

Because the end of the pole is buried


in the ground, not only do two force
components Ax and Ay act at the end
of the pole, but also a couple moment
M acts to prevent the pole from
rotating.

Equilibrium equations for pole ABCD


+

Fx = 0: Ax

(2 kip) sin

2 ft

Fy = 0: Ay

(2 kip) cos

(3 kip) cos

+ 6 kip = 0
=0

MA = 0: M + (2 kip)(sin )(5 ft) (3 kip)(sin


(6 kip)(5 ft + 2 ft + 2 ft) = 0
Geometry

+ (3 kip) sin

8 ft )
= 58.00
5 ft
= tan-1 ( 12 ft ) = 59.74
5 ft + 2 ft

= tan-1 (
C

2 ft

(5 ft + 2 ft)

5 ft
A
E

715

8 ft

F
12 ft

5.2 Rigid Bodies in Two-Dimensional Force Systems Example 7, page 3 of 3


6

Solving these equations, with

= 58.00 and

= 59.74, gives

Ax = 6.9 kip

Ans.

Ay = 2.6 kip

Ans.

M = 63.7 kipft

Ans.

Free-body diagram showing the


correct sense of the forces.
D
6 kip
C
B

8
3 kip

2 kip

Initially we assumed that Ax acted to the


right. Solving the equilibrium equations
gave a negative value for Ax, so the
correct sense of Ax must be to the left as
shown here.

M = 63.7 kipft
Ax = 6.9 kip
Ay = 2.6 kip

716

5.2 Rigid Bodies in Two-Dimensional Force Systems Example 8, page 1 of 3


8. A smooth slot has been cut in the plate shown, and
a pin, C, fixed to a support behind the plate, fits in
the slot. Determine the forces acting on the plate at
the supports A and B and from pin C.
4 lbft
B
D
1.5 ft
15

C
1.5 ft

A
2 lb
3 ft

2 ft

717

5.2 Rigid Bodies in Two-Dimensional Force Systems Example 8, page 2 of 3


1

Free-body diagram of plate ABD


2

Because the slot is smooth, no


friction force acts on it from the pin.
The only force from the pin must be
the normal force, NC. That is, the
force is perpendicular to the slot.
Here its direction has been arbitrarily
assumed to be up and to the right.

The force from the roller supports at B and A


are perpendicular to the surface upon which
the roller rests.

Equilibrium equations for plate ABD:

4 lbft
NC

FB

Fx = 0: 2 lb

1.5 ft

Fy = 0: FA + NC sin

FB + NC cos

=0

C
1.5 ft

A
2 lb
FA

3 ft

=0

MA = 0: 4 lbft (NC cos )(1.5 ft ) + (NC sin


+ FB (1.5 ft + 1.5 ft) = 0

(3 ft)

2 ft
5

The 4 lbft couple moment at corner D can be


considered to act anywhere; in particular, then, we
can consider it to act at point A, about which we are
summing moments. So we just insert "4 lbft" into
the equation.

718

5.2 Rigid Bodies in Two-Dimensional Force Systems Example 8, page 3 of 3


6

Geometry

= 90

15 C

Substituting the value = 75 into the equilibrium


equations and solving simultaneously, gives

15 = 75

15

FA = 2.94 lb

Ans.

FB = 1.212 lb

Ans.

NC = 3.04 lb

Ans.

Free-body diagram of plate ABD showing correct senses of forces

4 lbft
B
D

FB = 1.212 lb

NC = 3.04 lb
9
C

A
2 lb

Initially we arbitrarily assumed NC points upward


and to the right. Solving the equilibrium
equations gave NC = 3.04 lb. The minus sign
shows that our assumption was wrong, and the
correct direction of NC is down and to the left as
shown on this free body.

FA = 2.94 lb

719

5.2 Rigid Bodies in Two-Dimensional Force Systems Example 9, page 1 of 3


9. Pin C is rigidly attached to the plate and can slide freely
in the slot cut in member DB. Determine the reaction
force at the pin support at A and the force transmitted at C.
100 mm

30
425 mm

E
12 Nm

720

5.2 Rigid Bodies in Two-Dimensional Force Systems Example 9, page 2 of 3


1

Free-body diagram of plate ACE


100 mm

C
Equilibrium equations for plate ACE

NC

Fx = 0: Ax

NC sin

=0

Fy = 0: Ay

NC cos

=0

425 mm

3
30

MA = 0:

12 Nm

(NC sin

Ax

12 Nm
Ay
2

Because the slot is smooth, no friction force acts on


it from the pin. The only force from the pin must
be the normal force, that is, the force is
perpendicular to the slot. Here its direction has
been arbitrarily assumed to be up and to the left.

721

(0.425 m) + (NC cos )(0.1 m) = 0

5.2 Rigid Bodies in Two-Dimensional Force Systems Example 9, page 3 of 3


4

90

Geometry

30 = 60

Substituting = 30 into the equilibrium equations and


solving simultaneously gives

30

Ax = 20.1 N

Ans.

Ay = 34.7 N

Ans.

NC = 40.1 N

Ans.

D
= 90

60 = 30
6

Free-body diagram of plate ACE showing correct


senses of forces
C

NC = 40.1 N

30
D
7

The sense of the force


vector has been reversed
because solving the
equilibrium equations
gives a minus sign.

722

Ax = 20.1 N
Ay = 34.7 N

12 Nm

5.2 Rigid Bodies in Two-Dimensional Force Systems Example 10, page 1 of 2


10. Member ABD is supported by a pin at A and a cord attached at
B and D. The cord passes over a frictionless pulley at C.
Determine the reaction at A and the tension in the cord.
C
3 kNm

A
2m

2m

60 40

3m

5 kN

Moment equilibrium for pulley C: The moment about the


center produced by T1 must equal the moment produced by
T2. That is, T1r = T2r. Canceling the r's gives T1 = T2. In
fact, it's always true that for a frictionless pulley, the
tensions on the two sides of the pulley must be equal.

Radius r
T1

T2

723

5.2 Rigid Bodies in Two-Dimensional Force Systems Example 10, page 2 of 2


Free-body diagram of beam ABD

Because the tensions on the two sides


of pulley C are equal, the same tension
T acts on point B as on point D.

T
3 kNm

Ax
Ay

4m

T
60 40
3m

5 kN

Equilibrium equations for beam ABD


Fx = 0: Ax + T cos 60

Fy = 0: Ay + T sin 60 +T sin 40

MA = 0:

T cos 40 = 0
5=0

3 kNm + (T sin 60)(4 m)


+ (T sin 40)(4 m + 3 m)
(5 kN)(4 m + 3 m) = 0

Solving these equations simultaneously gives


Ax = 1.269 kN

Ans.

Ay = 2.20 kN

Ans.

T = 4.77 kN

Ans.

724

5.2 Rigid Bodies in Two-Dimensional Force Systems Example 11, page 1 of 3


11. The rigid bar AB is supported by two rollers attached to its
ends at A and B. If the bar is in equilibrium in the position
shown, determine the inclination, , of the inclined plane.

7 in.

20 lb
3 in.
25
A

725

5.2 Rigid Bodies in Two-Dimensional Force Systems Example 11, page 2 of 3


1

Free-body diagram of bar AB


FB

B
2

The force acting on roller A is perpendicular


to the wall; the same is true for roller B.

Equilibrium equations for rod AB:

20 lb

Fx = 0: FB cos
Fy = 0:

25

7 in.

3 in.

FB sin

+ FA cos
+ FA sin

20 lb = 0
=0

MB = 0: (FA cos )(3 in. + 7 in.)

A
FA

726

(1)
(2)

(20 lb)(7 in.) = 0

(3)

5.2 Rigid Bodies in Two-Dimensional Force Systems Example 11, page 3 of 3


4

Geometry
= 90
=

(90

)
5

Substituting

= 25 in Eq. 3 and solving gives FA = 15.447 lb.

Substituting

= 25,

FB cos

= , and FA = 15.447 lb in Eqs. 1 and 2 gives

+ (15.447 lb) cos 25 = 20 lb

(4)

90
FB sin

+ (15.447 lb) sin 25 = 0

(5)

Eqs. 4 and 5 can be solved with a calculator that can handle two
simultaneous nonlinear equations.

C
6

Alternatively, rewrite Eqs. 4 and 5 so that the constant terms are


on the right:

25
90

25 = 65

FB cos

= 20 lb

(15.447 lb) cos 25

FB sin

= (15.447 lb) sin 25

(7)

Now eliminate FB by dividing Eq. 7 by Eq. 6:

A
= 90

FB sin
(15.447 lb) sin 25
=
FB cos
20 lb (15.447 lb) cos 25

65 = 25

tan
Solving for gives
= 47.4

727

(6)

Ans.

5.2 Rigid Bodies in Two-Dimensional Force Systems Example 12, page 1 of 3


12. The uniform square plate ABC of mass 10 kg is supported
by a vertical cord at B. Determine the tension in the cord and
the forces from the walls if the walls are smooth at the contact
points A and C.
D

20

B
C

728

5.2 Rigid Bodies in Two-Dimensional Force Systems Example 12, page 2 of 3


1

Free-body diagram of plate ABC


2

FBD

Since the wall is smooth, no friction force acts. Only


the normal force FC is present at C. Similarly only
the normal force FA is present at A.

d1
B
FC
C
Center of Mass
G

d3
4

Weight = mg = (10 kg)(9.81m/s2) = 98.1 N

Fx = 0: FA

Equilibrium equations for plate ABC:

d2
Fy = 0: FBD

FA

729

FC = 0

(1)

98.1 N = 0

MA = 0: (FBD)(d1) + (FC)(d3)

(2)
(98.1 N)(d2) = 0

(3)

5.2 Rigid Bodies in Two-Dimensional Force Systems Example 12, page 3 of 3


5

Geometry - calculation of d1
d1

Geometry - calculation of d3

d1 = L sin 20

L2 + L2 = L 2
L

20

L
20

d3

45
25

A
A

d3 = (L 2) sin 25

Geometry - calculation of d2
B
L cos 45

20

L
G

45

90
A

20

45 = 25

d2
d2 = (L cos 45)(cos 25)

730

Substituting these expressions for d1, d2, and


d3 into Eq. 3, canceling L, and solving Eqs. 1,
2, and 3 simultaneously gives
FA = 49.0 N

Ans.

FC = 49.0 N

Ans.

FBD = 98.1 N

Ans.

5.3 Rigid Bodies and Three-Dimensional Force Systems

731

5.3 Rigid Bodies and Three-Dimensional Force Systems Procedures and Strategies, page 1 of 1
x
A

Ball and socket


support

Short links
P

2. Write six equations of equilibrium:


Fy = 0
My = 0

C
E

1. Draw a free-body diagram, paying special


attention to the reactions from the supports. If a
constraint prevents motion in a certain direction,
then show a reaction force acting in that direction
on the free-body diagram. If a constraint prevents
rotation about a certain axis, then show a reaction
couple-moment on the free-body diagram.

Fx = 0
Mx = 0

Procedures and Strategies for Solving


Problems Involving Equilibrium of a Rigid
Body in a Three-Dimensional Force System

Ax

Fz = 0
Mz = 0

Ez
E

Bx

Notes:

Ex

D
Ey
Dy

a) If the problem is such that distances and force


components are easy to determine, then
calculate moments in scalar form (M = Fd).
Otherwise use the cross product definition of
moments ( M = r F).

My

Fy

Journal bearing
Fx

b) In some problems, you can save work by


summing moments about an axis other than a
coordinate axis.

Mx

732

5.3 Rigid Bodies and Three-Dimensional Force Systems Problem Statement for Example 1
1. For the rigid frame shown, determine the
reactions at the knife-edge supports A, B, D.
Neglect the weight of the frame.
y

80 N
A
x

D
C

200 mm

150 mm
E
150 mm

400 mm

24 N

600 mm

300 mm

733

5.3 Rigid Bodies and Three-Dimensional Force Systems Problem Statement for Example 2
2. A 10-kg block rests on top of a triangular plate of negligible
weight. The plate is supported by vertical wires AD, OB, and EC.
Determine the x and z coordinates of the 10-kg block if the
tensions in wires AD and CE both equal 20 N. Also determine
the tension in wire OB.
y

400 mm
O
D

300 mm

E
z

10 kg

734

5.3 Rigid Bodies and Three-Dimensional Force Systems Problem Statement for Example 3
3. To hoist the 180-lb load, a vertical force P is applied to the
crank of the windlass shown. The bearing at A exerts forces
normal and parallel to the shaft; the bearing at B exerts only
forces normal to the shaft no axial force. Determine the
magnitude of P and the forces exerted by the bearings when
the crank is in the position shown.

12 in.

24 in.

10 in.
Radius = 8 in.
Bearing B

Bearing A

B
60
C

18 in.

180 lb
P

P
View as seen from
the right end.

735

5.3 Rigid Bodies and Three-Dimensional Force Systems Problem Statement for Example 4
4. The box is supported by short links at the
corners. Determine the forces in the links.
y

30

20 lb
42 lb

E
O

5 ft

30 C

6 ft
B

6 ft

z
4 ft

736

5.3 Rigid Bodies and Three-Dimensional Force Systems Problem Statement for Example 5
5. A force F = {20i 40j 30k} N acts at the midpoint D of rod ADB.
End B of the rod rests on a smooth horizontal plate and is attached to
cord BC. End A is attached to a ball-and-socket support. Determine
the force in the cord and the reactions at A and B.
y

F = {20i

40j + 30k} N
3m

D
x

C
Cord
4m

5m
z

737

5.3 Rigid Bodies and Three-Dimensional Force Systems Problem Statement for Example 6
6. The boom AB is supported by a ball-and-socket at A
and guy wires CF and DE. Determine the components
of the reactions at A and the forces in the guy wires.
y
1m
D
4m

3m
2m
A

E
F

2m
2m

50 kg
3m

738

5.3 Rigid Bodies and Three-Dimensional Force Systems Problem Statement for Example 7
7. The uniform 15-kg lid of the box is supported by a stick CD
and short hinges at A and B. Assuming that hinge B transmits
no axial force and that the line of action of the force from the
stick coincides with the long axis of the stick, determine the
reactions at the hinges and the force transmitted by the stick at C.
y

250 mm

250 mm
400 mm

C
A

50

x
30 mm

30 mm

739

5.3 Rigid Bodies and Three-Dimensional Force Systems Problem Statement for Example 8
8. The uniform bar AB weighs 20-lb, is 4-ft long, and is supported
by two cords and by smooth surfaces at A and B. Determine the
forces in the cords and the reactions at A and B.
y

Cord

4 ft
O

30
60
Cord
A

740

5.3 Rigid Bodies in Three-Dimensional Force Systems Example 1, page 1 of 5


1. For the rigid frame shown, determine the
reactions at the knife-edge supports A, B, D.
Neglect the weight of the frame.
y

80 N
A
x

D
C

200 mm

150 mm
E
150 mm

400 mm

24 N

600 mm

300 mm

741

5.3 Rigid Bodies in Three-Dimensional Force Systems Example 1, page 2 of 5


Free-body diagram of frame.

80 N

FA

FD

E
24 N

FB

Since all forces are vertical and dimensions are given in


coordinate directions, a scalar rather than a vector
approach is probably best.

Sum forces

Fy = 0: FA + FB + FD

80

742

24 N = 0

(1)

5.3 Rigid Bodies in Three-Dimensional Force Systems Example 1, page 3 of 5


4

y
80 N

To calculate moments acting about the x axis, consider the view seen by an
observer located on the positive x axis and looking back at the yz plane.

D
C

200 mm

150 mm
FD

FA

150 mm

24 N

B
FB

400 mm

600 mm

300 mm
5

150 mm

Sum moments about the x axis.


Mx = 0:

80 N

FB (150 mm + 150 mm) + 24 N (200 mm)


+ 80 N (150 mm) = 0

Solving gives
FB = 56.0 N

z
Ans.

View from the positive x axis


y
150 mm

C
x,D,A

B
FB

24 N

FD
200 mm

743

FA

5.3 Rigid Bodies in Three-Dimensional Force Systems Example 1, page 4 of 5


y

7
80 N

To calculate moments about the z axis, consider the view seen by an observer
located on the positive z axis and looking back at the xy plane.

A
C

200 mm

150 mm

FA

FD
150 mm

400 mm

FB

600 mm

300 mm

80 (400 mm) + 56 N (400 mm)


+ FD (400 mm + 600 mm)
24 N (400 mm + 600 mm
+ 300 mm) = 0

View from the positive z axis.

C
z,A

Solving gives
FD = 40.8 N

80 N

Sum moments about the z axis.


Mz = 0:

E
24 N

y
9

FA
Ans.

400 mm

744

FB = 56.0 N

FD

24 N

600 mm

300 mm

5.3 Rigid Bodies in Three-Dimensional Force Systems Example 1, page 5 of 5


10 Using FB = 56.0 N and FD = 40.8 N in Eq. 1 gives
FA + FB + FD

80 N

60 N = 0

(Eq. 1 repeated)

56.0 N 40.8 N
Solving gives
FA = 7.2 N

Ans.

745

5.3 Rigid Bodies in Three-Dimensional Force Systems Example 2, page 1 of 4


2. A 10-kg block rests on top of a triangular plate of negligible
weight. The plate is supported by vertical wires AD, OB, and EC.
Determine the x and z coordinates of the 10-kg block if the
tensions in wires AD and CE both equal 20 N. Also determine
the tension in wire OB.
y

400 mm
O
D

300 mm

E
z

10 kg

746

5.3 Rigid Bodies in Three-Dimensional Force Systems Example 2, page 2 of 4


1

Free-body diagram of plate and 10-kg block.


y
C
400 mm
TOB

20 N

300 mm
z
x
10 kg

20 N

Weight = (10 kg)(9.81 m/s2)


= 98.10 N

A
2
z
3

Since all forces are parallel to a single coordinate (y) axis


and dimensions are given parallel to coordinate directions, a
scalar rather vector approach is probably best.

Sum forces
Fy = 0: TOB + 20 N + 20 N

98.10 N

Solving gives
TOB = 58.1 N

Ans

747

5.3 Rigid Bodies in Three-Dimensional Force Systems Example 2, page 3 of 4


4

To calculate x and z, we need to use moment


equations of equilibrium.
y
400 mm
TOB

20 N
98.10 N

300 mm

z
x
5

20 N
D
z

View from the positive x axis

To calculate moments about


the x axis, consider the view
seen by an observer located
on the positive x axis and
looking back at the yz plane.

300 mm
7

z
98.10 N

20 N

Sum moments about the positive x axis.


Mx = 0: (98.10 N)(z)

(20 N)(300 mm) = 0

Solving gives
z

x, E, O

748

z = 61.2 mm

Ans.

5.3 Rigid Bodies in Three-Dimensional Force Systems Example 2, page 4 of 4


y
400 mm
TOB

20 N
98.10 N

300 mm
O

z
x
20 N
D
9
8

To calculate moments about the z axis,


consider the view seen by an observer located
on the positive z axis and looking back at the
xy plane.

View from the positive z axis

20 N

98.10 N

TOB

20 N

10 Sum moments about the z axis.

Mz = 0: 20 N (400 mm)

(98.10 N)x = 0

E
z, D, O
x

Solving gives
x = 81.6 mm

400 mm

Ans.

749

5.3 Rigid Bodies in Three-Dimensional Force Systems Example 3, page 1 of 6


3. To hoist the 180-lb load, a vertical force P is applied to the
crank of the windlass shown. The bearing at A exerts forces
normal and parallel to the shaft; the bearing at B exerts only
forces normal to the shaft no axial force. Determine the
magnitude of P and the forces exerted by the bearings when
the crank is in the position shown.

12 in.

24 in.

10 in.
Radius = 8 in.
Bearing B

Bearing A

B
60
C

18 in.

180 lb
P

P
View as seen from
the right end.

750

5.3 Rigid Bodies in Three-Dimensional Force Systems Example 3, page 2 of 6


1 Free-body diagram of windlass
y
12 in.

24 in.

10 in.

Az
Bz
A

B
x

Ax
Ay

By
2

180 lb

z
3

No x component of force is
shown because the bearing B
transmits no axial force

Fx = 0: Ax = 0

Fy = 0: Ay + By

Sum forces.

Fz = 0: Az + Bz = 0

(1)
180 lb + P

2)
(3)

751

5.3 Rigid Bodies in Three-Dimensional Force Systems Example 3, page 3 of 6


y
4
12 in.

24 in.

To calculate moments acting


about the x axis, consider the
view seen by an observer
located on the positive x axis
and looking back at the yz
plane.

10 in.

Az

Bz

B
x

Ax
Ay

By
C
180 lb

View from the positive x axis


(18 in.) cos 60
= 9 in.

y
Radius
= 8 in.

x,B,A

z
60
6

Mx = 0: (180 lb)(8 in.)

P(9 in.) = 0
180 lb

Solving gives
P = 160 lb

18 in.

Ans.
P

752

5.3 Rigid Bodies in Three-Dimensional Force Systems Example 3, page 4 of 6


7 To calculate the moments acting about the y axis, consider
the view seen by an observer located on the positive y axis
and looking back at the xz plane.

12 in.

24 in.

10 in.

Az

Bz
B

Ax
By

Ay

z
180 lb

36 in.

Az

9 Sum moments about the y axis.

Bz

x
B

y, A

My = 0:

Bz (36 in.) = 0

So
8

View from the positive y axis

Bz = 0

753

Ans.

5.3 Rigid Bodies in Three-Dimensional Force Systems Example 3, page 5 of 6


y
12 in.

24 in.

10 in.
Bz

Az
A

Ax
Ay

By
180 lb

z
10 To calculate the moments about the z axis, consider
the view seen by an observer located on the positive
z axis and looking back at the xy plane.

11 View from the positive z axis


B

z,A

x
12 Sum moments about the z axis.
By

180 lb

Ay

P = 160 lb
12 in.

24 in.

Mz = 0:

(180 lb)(12 in.) + By (12 in. + 24 in.)


(160 lb)(12 in. + 24 in. + 10 in.) = 0

Solving gives
By = 144.4 lb

10 in.

754

Ans.

5.3 Rigid Bodies in Three-Dimensional Force Systems Example 3, page 6 of 6


13

Substituting P = 160 lb, By = 144.4 lb, and Bz = 0


in Eqs. 1, 2, and 3 and solving gives
Ax = 0

Ans.

Ay = 164.4 lb

Ans.

Az = 0

Ans.

755

5.3 Rigid Bodies in Three-Dimensional Force Systems Example 4, page 1 of 6


4. The box is supported by short links at the
corners. Determine the forces in the links.
y

30

20 lb
42 lb

E
O

5 ft

30 C

6 ft
B

6 ft

z
4 ft

756

5.3 Rigid Bodies in Three-Dimensional Force Systems Example 4, page 2 of 6


1

Free-body diagram of box

FG
G

30
5 ft

20 lb
E

42 lb

FE

6 ft
B

FA

FB

6 ft

Sum forces.

FO

Fx = 0: 20 lb

Fy = 0: FC sin 30 + FG sin 30

FC
30 C

Fz = 0: FB + 42 lb = 0

4 ft

757

FC cos 30

FG cos 30 = 0
FA

FO

(1)
FE

(2)
(3)

5.3 Rigid Bodies in Three-Dimensional Force Systems Example 4, page 3 of 6


To calculate the moments about the x axis, consider the view seen by an
observer located on the positive x axis and looking back at the yz plane.
y

FG
G

30
5 ft

20 lb

FC sin 30

42 lb

42 lb

FC
D

30 C

View from the positive x axis


y

FO

H,G

D,C

FE
5 ft
z

6 ft

B,A
x,E,O

12 ft

FA

FB
4 ft

6 ft

Sum moments about the x axis.

FA

Mx = 0: FA(12 ft)

758

FC sin 30 (12 ft) + 42 lb (5 ft) = 0

(4)

5.3 Rigid Bodies in Three-Dimensional Force Systems Example 4, page 4 of 6


6
y

FG

To calculate the moments about


the y axis, consider the view seen
by an observer located on the
positive y axis and looking back
at the xz plane.

4 ft
y,G,O

H,E

6 ft

30
20 lb

5 ft

20 lb

42 lb

42 lb

FC

FC cos 30
D

30 C

C,A

6 ft

FA

6 ft

FB

View from the positive y axis

Sum moments about the y axis.

FB

D,B

FE

FO

z
A

6 ft

4 ft

759

My = 0:

FC cos 30 (6 ft + 6 ft) FB (4 ft)


20 lb (6 ft) 42 lb (4 ft) = 0

(5)

5.3 Rigid Bodies in Three-Dimensional Force Systems Example 4, page 5 of 6


y

To calculate moments about the z axis, consider the view


seen by an observer located on the positive z axis and
looking back at the xy plane.
y

C,G

20 lb

D,H

FC cos 30 FG cos 30
FG
G

5 ft

30
z,A,O
20 lb
E

42 lb

FE

FC

B,E

5 ft

4 ft
D

30 C

FE

FO

10 View from the positive z axis

6 ft

11 Sum the moments about the z axis.

B
A
z

FA

FB

6 ft

4 ft

760

Mz = 0: FG cos 30 (5 ft) + FC cos 30 (5 ft)


20 lb (5 ft) FE (4 ft) 0

(6)

5.3 Rigid Bodies in Three-Dimensional Force Systems Example 4, page 6 of 6


12 Solving the six equilibrium equations
20

FC cos 30

FG cos 30 = 0

FC sin 30 + FG sin 30

FA

FO

(1)
FE = 0

FB + 42 = 0

(3)

(42)(5)

(4)

(12 FC) sin 30 + 12 FA = 0

(12 FC) cos 30

4FB + (6)(20)

(5 FC) cos 30 + (5 FG) cos 30

(42)(4) = 0
(20)(5)

(5)

4FE = 0

(6)

gives
FA = 11.7 lb

Ans

FB = 42.0 lb

Ans.

FC = 11.5 lb

Ans.

FE = 0

Ans.

FG = 11.5 lb

Ans.

FO = 23.3 lb

Ans.

761

5.3 Rigid Bodies in Three-Dimensional Force Systems Example 5, page 1 of 6


5. A force F = {20i 40j 30k} N acts at the midpoint D of rod ADB.
End B of the rod rests on a smooth horizontal plate and is attached to
cord BC. End A is attached to a ball-and-socket support. Determine
the force in the cord and the reactions at A and B.
y

F = {20i

40j + 30k} N
3m

D
x

C
Cord
4m

5m
z

762

5.3 Rigid Bodies in Three-Dimensional Force Systems Example 5, page 2 of 6


y

Azk

Axi

3m

Free-body diagram of rod

Ax, Ay, and Az are the forces from the


ball-and-socket joint at A. No moment is
transmitted at a ball-and-socket joint.

Ayj

F = {20i

40j + 30k} N

D
x

O
4m
Tk
B
5m

z
5

Tk = 0

Collecting the coefficients of i, j, and k gives


(Ax + 20)i + (Ay + B

Normal force from plate

F = 0: Axi + Ayj + Azk + Bj


+ {20i 40j + 30k}N

40)j + (Az + 30

T)k = 0

763

Tension in cord

Bj
4

Sum forces.

Equating coefficients to zero gives


Fx = 0: Ax + 20 = 0

(1)

Fy = 0: Ay + B

40 = 0

(2)

Fz = 0: Az + 30

T=0

(3)

5.3 Rigid Bodies in Three-Dimensional Force Systems Example 5, page 3 of 6


7

Next, calculate moments. To do so,


first introduce position vectors from A
to B and from A to D.

Azk

Axi

F = {20i
3m
9

rAD = position vector from A to


midpoint of rod
rAB
=
2
=

{5i

= {2.5i

40j + 30k} N

Ayj

rAB = {5i

3j + 4 k } m

D
x

O
4m

3j + 4k}
2

Tk
B

1.5j + 2k} m
5m

764

Bj

5.3 Rigid Bodies in Three-Dimensional Force Systems Example 5, page 4 of 6


10 Sum moments about point A.

MA = 0: rAD

F + rAB

{Bj

k} = 0

or,
{2.5i

1.5j + 2k}

{20i

40j

30k} + { i

3j+ 4k}

{Bj

k} = 0

Evaluating the cross products gives

0=

2.5

1.5

20

40

30

1.5

40

30

=i

2.5

20

30

= i[( 1.5)(30)

20

40

j
B

1.5

+k

+i

2.5

( 40)(2)]

j[2.5(30)

+ i[( 3)( T)

+k

B(4)]

20(2)] + k[2.5( 40)

j[5( T)

0(4)] + k[5(B)

765

20( 1.5)]
0)( 3)]

5.3 Rigid Bodies in Three-Dimensional Force Systems Example 5, page 5 of 6


11

Carrying out the multiplications and collecting coefficients of i,


j, and k gives

0 = (35 + 3T 4B)i + ( 35 + 5T)j + ( 70 + 5B)k


Equating each coefficient to zero gives
Mx = 0:

35 + 3T

4B = 0

(4)

My = 0:

35 + 5T = 0

(5)

Mz = 0:

70 + 5B = 0

(6)

Solving Eq. 5 gives


T=7N

Ans.

and solving either Eq. 4 or Eq. 6 (one of the equations are


redundant) gives
B = 14 N

Ans.

Eqs. 1, 2, and 3 are


Fx = 0: Ax + 20 = 0

(Eq. 1 repeated)

Fy = 0: Ay + B

40 = 0

(Eq. 2 repeated)

Fz = 0: Az + 30

T=0

(Eq. 3 repeated)

766

12 Substituting T = 14 N and B = 8 N and solving Eqs.


1, 2, and 3 gives
Ax = 20 N

Ans.

Ay = 26 N

Ans.

Az = 23 N

Ans.

5.3 Rigid Bodies in Three-Dimensional Force Systems Example 5, page 6 of 6


y

y' Azk

z'
Axi

F
Ayj
D
x

Tk

Bj

x'

13 Why is one of the moment equations, Mx = 0 and Mz = 0, redundant? Because the


lines of action of all forces pass through the axis of the rod. Thus if, instead of x, y,
and z axes, we had chosen x', y', and z' axes as shown above, then since all forces
intersect the x' axis, the moment equation Mx' = 0 would reduce to 0 = 0. That is,
one of the moment equations is not an independent equation, and there are only two
non-trivial moment equations: My' = 0 and Mz' = 0. General conclusion: In three
dimensions, there are at most six independent equations of equilibrium.

767

5.3 Rigid Bodies in Three-Dimensional Force Systems Example 6, page 1 of 6


6. The boom AB is supported by a ball-and-socket at A
and guy wires CF and DE. Determine the components
of the reactions at A and the forces in the guy wires.
y
1m
D
4m

3m
2m
A

E
F

2m
2m

50 kg
3m

768

5.3 Rigid Bodies in Three-Dimensional Force Systems Example 6, page 2 of 6


1

Free-body diagram

y
Azk

FED A
Axi

FFC
E

2
Ayj

x, y, and z components of force


at a ball-and-socket joint

F
B

z
Cable force supporting 50-kg block
= 50 kg

9.81 m/s2

Sum forces.

F = 0: FFC + FED + Axi + Ayj + Azk

= 490.5 N
4

{490.5j} N = 0

(1)

Next we have to express FFC and FED in rectangular components.

769

5.3 Rigid Bodies in Three-Dimensional Force Systems Example 6, page 3 of 6


y
1m
D
C

rED

rFC

2m

Introduce position vectors in the same direction as the forces.

4m

rFC = ( 4 m

3m

0)i + (2 m

= { 4i + 2j
A

FED

FFC

0)j + (0

2m

2 m)k

4k}m

rED = (1 m

= {i + 3j

0)i + (3 m

0)j + (0

2 m)k

2k}m

F
6
B

2m

Force = (magnitude)(unit vector):

r
FFC = (FFC)( FC )
rFC

2m
z

= FFC (

50 kg
3m

4i + 2j

4k

( 4)2 + (2)2 + ( 4)2

= FFC{ 0.6667i + 0.3333j

770

0.6667k}

(2)

5.3 Rigid Bodies in Three-Dimensional Force Systems Example 6, page 4 of 6


r
FED = (FED)( r ED )
ED
= FED (

i + 3j
2

2k
2

)
2

(1) + (3) + ( 2)
= FED{0.2673i + 0.8018j
7

0.5345k)

(3)

Substitute Eqs. 2 and 3 into the equilibrium equation, Eq. 1:


FFC + FED + Axi + Ayj + Azk

(490.5)j = 0

FED{0.2673i + 0.8018j

FFC{ 0.6667i + 0.3333j

(Eq. 1 repeated)

0.5345k}

0.6667k}

Collecting coefficients of i, then of j, and then of k gives

( 0.6667FFC + 0.2673FED + Ax)i + (0.3333FFC + 0.8018FED


+ Ay

490.5)j + ( 0.6667FFC

0.5345FED + Az)k = 0

Equating each coefficient to zero gives


Fx = 0:

0.6667FFC + 0.2673FED + Ax = 0

(4)

Fy = 0: 0.3333FFC + 0.8018FED + Ay 490.5 = 0

Fz = 0:

(5)

0.6667FFC + 0.5345FED + Az = 0

(6)

771

5.3 Rigid Bodies in Three-Dimensional Force Systems Example 6, page 5 of 6


y

Az
rAE

FED
A

FFC
rAF

Ax

To calculate moments, first introduce position vectors

rAE = {2k} m

Ay

rAF = (2m + 2m)k = {4k} m

rAB = (2m + 2m + 3m)k = {7k} m


9

Sum moments.

2m

rAB

MA = 0: rAE

FED + rAF

FFC + rAB

{ 490.5j} = 0

2m

or,
3m

{2k} FED{0.2673i 0.8018j 0.5345k}


+ {4k} FFC{ 0.6667i + 0.3333j 0.6667k}
+ {7k} { 490.5j} = 0
Expanding the cross products gives
2FED(0.2673)k

i + 2FED(0.8018)k

= i

=j

+ 4FFC(0.3333)k

j + 2FED( 0.5345)k

j + 4FFC( 0.6667)k
= i

772

k + 4FFC( 0.6667)k
=0

k + 7( 490.5 k
=0

j=0
= i

i
=j

5.3 Rigid Bodies in Three-Dimensional Force Systems Example 6, page 6 of 6


10 Carrying out the multiplications and collecting coefficients of i and then j gives
( 1.6036FED

1.3332FFC + 3433.5)i + (0.5334FED

2.6668FFC)j = 0

Equating each coefficient to zero gives


Mx = 0:

.6036FED

My = 0: 0.5334FED

1.3332FFC + 3433.5 = 0

(7)

2.6668FFC = 0

(8)

Solving Eqs. 7 and 8 simultaneously gives


FED = 1835.841 N = 1.836 kN

Ans.

FFC = 367.196 N = 0.367 kN

Ans.

Using these results in Eqs. 4, 5, and 6 gives


Ax = 245.911 N = 0.246 kN

Ans.

Ay = 1103.863 N = 1.104 kN

Ans.

Az = 736.448 N = 0.736 kN

Ans.

773

5.3 Rigid Bodies in Three-Dimensional Force Systems Example 7, page 1 of 8


7. The uniform 15-kg lid of the box is supported by a stick CD
and short hinges at A and B. Assuming that hinge B transmits
no axial force and that the line of action of the force from the
stick coincides with the long axis of the stick, determine the
reactions at the hinges and the force transmitted by the stick at C.
y

250 mm

250 mm
400 mm

C
A

50

x
30 mm

30 mm

774

5.3 Rigid Bodies in Three-Dimensional Force Systems Example 7, page 2 of 8


1

Free-body diagram of lid


y
2

FC = force from
stick acting on
lid at point C.

Weight = (15 kg)(9.81 m/s2) = 147.15 N

250 mm
C
FC
Ayj

400 mm
= 200 mm
2

400 mm
E (center
of lid)

Byj

50

Axi

rDC

Azk

Bzk
400 mm
4

30 mm

440 mm

30 mm

Bx = 0 because we are
told that hinge B
transmits no axial force

z
5

Sum forces.

F = 0: Axi + Ayj + Azk + Byj + Bzk + FC


6

{147.15j} N = 0

The force FC must be expressed in rectangular components. We know


that, in terms of the position vector rDC:

FC = FC

rDC
rDC

775

(1)

5.3 Rigid Bodies in Three-Dimensional Force Systems Example 7, page 3 of 8


y

y
250 mm

400 mm

rDC

400 sin 50 mm
= 306.4 mm
50

x, B, A

500 mm

400 cos 50 mm
= 257.1 mm

View of xy plane as seen from positive z axis

400 mm
View of yz plane as seen from positive x axis

7 The force FC must be expressed in rectangular


components. We know that, in terms of the
position vector rDC:

r
FC = FC DC
rDC

(2)

Thus, Eq. 2 gives

FC = FC

rDC
rDC

Here,

rDC = coordinates of C
= {(250

= FC

coordinates of D

500)i + (306.4

0)j + (257.1

{ 250i + 306.4j

142.9k}

( 250)2 + (306.4)2 + ( 142.9)2


400)k} m

776

= FC{ 0 5946i + 0.7287j

0.3399k}

(3)

5.3 Rigid Bodies in Three-Dimensional Force Systems Example 7, page 4 of 8


9 This expression for FC can now be substituted into Eq. 1:
Axi + Ayj + Azk + Byj + Bzk + FC

{147.15j} = 0

(Eq. 1 repeated)

FC{ 0.5946i + 0.7287j

0.3399k

Collecting coefficients of i, then j, and then k gives


(Ax

0.5946 FC)i + (Ay + By + 0.7287 FC 147.15)j


+ (Az + Bz 0.3399 FC)k = 0

Equating each coefficient to zero gives


Fx = 0: Ax

0.5946FC = 0

(4)

Fy = 0: Ay + By + 0.7287FC 147.15 = 0

(5)

Fz = 0: Az + Bz

(6)

0.3399FC = 0

777

5.3 Rigid Bodies in Three-Dimensional Force Systems Example 7, page 5 of 8


10 Next, moment equations must be written. If we choose point A as the point about which
to sum moments, then the resulting equations will be simplified, since Ax, Ay, and Az will
not appear. Thus we need the position vectors from A to the lines of action of the forces.
y

y
440 mm
= 220 mm
2

C
Center of lid

400 mm
2
= 200 mm

400 mm

rAE

Byj

rAB

A
147.15 N

200 sin 50 mm
= 153.2 mm
B

50
x, B, A

Bzk
200 cos 50 mm
= 128.6 mm
D

440 mm

View of the yz plane as seen


from the positive x axis.

11 From the figures,

rAE = {220i +153.2j


rAB = {440i} mm

778

128.6k} mm

(7)
(8)

5.3 Rigid Bodies in Three-Dimensional Force Systems Example 7, page 6 of 8


y

12 Line of action of force FC

FC

E
Byj

B
Bzk 400 mm

rAD

440 mm

30 mm

13 We can use position vector rAD from A to D (to


calculate the moment of the force FC) because the
line of action of FC passes through D.

rAD = {(440 + 30)i +400k} mm

779

(9)

5.3 Rigid Bodies in Three-Dimensional Force Systems Example 7, page 7 of 8


14 Sum moments about A.

MA = rAE

{ 147.15j}N + rAB

{Byj + Bzk} + rAD

FC

Substituting from Eqs. 8, 9, and 10 for rAE, rAB, and rAD and from Eq. 3 for FC gives

MA = {220i
+ {470i

153.2j

400k}

128.6k}

{ 147.15j} + {440i}

FC{ 0 5946i + 0.7287j

{Byj

zk}

k}

Expanding the cross products gives

MA = 220( 147.15)i

j + 153.2( 147.15)j

=k

+ 470( 0.5946FC)i

+ 400( 0.5946FC)k
=j

j + 470( 0.3399FC)i

=k

i + 400(0.7287FC)k

j + 440(By)i

= i

=0

i + 470(0.7287FC i

=0

j + 128.6( 147.15)k

k
= j

j + 400( 0.3399FC)k

= i

=0

780

j + 440(Bz)i
=k

k
= j

5.3 Rigid Bodies in Three-Dimensional Force Systems Example 7, page 8 of 8


15 Carrying out the multiplications and then collecting coefficients of i, j, and k gives

MA = (18923.490

291.480FC)i + ( 440Bz

78.087FC)j + ( 32373.000 + 440By + 342.489FC)k

Setting coefficients of i, j, and k to zero gives


18923.490
440Bz

291.480FC = 0

(10)

78.087FC = 0

(11)

32373.000 + 440By + 342.489FC = 0

(12)

Solving Eqs. 10, 11, and 12 simultaneously gives


FC = 64.922 N

Ans.

By = 23.041 N

Ans.

Bz = 11.522 N

Ans.

Substituting these results in Eqs. 4, 5, and 6, and then solving gives


Ax = 38.6 N

Ans.

Ay = 76.8 N

Ans.

Az = 33.6 N

Ans.

781

5.3 Rigid Bodies in Three-Dimensional Force Systems Example 8, page 1 of 8


8. The uniform bar AB weighs 20-lb, is 4-ft long, and is supported
by two cords and by smooth surfaces at A and B. Determine the
forces in the cords and the reactions at A and B.
y

Cord

4 ft
O
30

x
60

Cord
A

782

5.3 Rigid Bodies in Three-Dimensional Force Systems Example 8, page 2 of 8


1

Free-body diagram of AB
y

NBk (normal force only, "smooth" surface so


no friction force no force parallel to the wall)

TBi (tension in cord)

M (midpoint of AB)

{ 20j} lb
(weight)

TA

Tension in cord

NAj (normal force only, because surface is smooth)

Sum forces.

F = 0: NBk + TBi + TA + NAj + { 20j} lb = 0

(1)

783

5.3 Rigid Bodies in Three-Dimensional Force Systems Example 8, page 3 of 8


y

To find the rectangular


components of the force TA,
introduce a position vector rAO
from A to O.

Cord

Magnitude rAO = 4 ft cos 60

4 ft
O

= 2 ft

30

rAO
4

x component of rAO = (2 ft) cos 30

x
60

Cord
A

TA

= 1.7321 ft

TA = (TA)(unit vector along AO)


rAO
= TA r
AO
= TA

1.7321i
2

= 0.8660TAi

784

k
0.5TAk

z component of rAO = (2 ft) sin 30


= 1 ft

Since the x and z components of rOA are


now known, rOA can be expressed as
rAO = { 1.7321i

k} ft.

(2)

5.3 Rigid Bodies in Three-Dimensional Force Systems Example 8, page 4 of 8


8

The expression that has just been found for TAin terms of
rectangular components can now be substituted into the sum of
forces equation, Eq. 1:
NBk + TBi + TA + NAj

20j = 0

0.8660TAi

(Eq. 1 repeated)

0.5TAk

Collecting coefficients of i, then j, and then k gives


(TB

0.8660TA)i + ( NA

20)j + (NB

0.5TA)k = 0

Each coefficient must equal zero so


TB

0.8660TA = 0

(3)

NA

20 = 0

(4)

NB

0.5TA = 0

(5)

Three equations, four unknowns. An additional equation is needed


and will be obtained by writing a moment equation.

785

5.3 Rigid Bodies in Three-Dimensional Force Systems Example 8, page 5 of 8


To write the moment equation, first
introduce position vectors from O
to B, from O to A, and from O to
the line of action of the weight.

10

11 The position vector from O to A, rOA, can be found by


noting that it is just the negative of the position vector from
A to O, rAO, and we have already found rAO ( Eq. 2):

rOA =

rOB = (2ft + 2ft) sin 60j


= {3.4641j} ft

rAO
Eq. 2

(6)

= { 1.7321i

k}

= {1.7321i + k}ft

12 MM' is the line of action of the weight


of the rod. By similar triangles AMM'
and ABO, point M' must lie at the
midpoint of OA (because M lies at the
midpoint of the rod). Thus
1
rOM' = 2 rOA
= 12 {1.7321i + k}
x

B
2 ft

rOB
M
O
2 ft

rOM'

= {0.8660i + 0.5k} ft

60
M'

rOA

(7)

786

(8)

5.3 Rigid Bodies in Three-Dimensional Force Systems Example 8, page 6 of 8


y
NBk
B

TBi

rOB
M
O

rOM'

M'

TA
rOA

{ 20j} lb

NA j
z
13 Sum moments about the origin O.

MO = 0: rOB
{3.4641j}

NBk + TBi) + rOM'

= 0 because rOA and TAare parallel


{ 20j} + (rOA

NBk + TBi) + {0.8660i + 0.5k}

TA) + rOA

NA j = 0

{ 20j} + {1.7321i + k}

787

NA j = 0

5.3 Rigid Bodies in Three-Dimensional Force Systems Example 8, page 7 of 8


14 Carrying out the cross products gives
3.4641NB j

k + 3.4641TB j

i + 0.8660( 20)i

= k

=i

j + 0.5( 20)k

j + 1.7321NA i

= i

=k

j + NA k

=k

= i

Carrying out the multiplications and collecting coefficients of i, then j, and then k gives
(3.4641NB + 10

NA)i + ( 3.4641TB 17.3200 + 1.7321NA)k = 0

Equating coefficients of i and then of k to zero gives


3.4641NB + 10

NA = 0

(9)

3.4641TB 17.3200 + 1.7321NA = 0

(10)

These equations must be solved simultaneously with Eqs. 3, 4, and 5:


TB

0.8660TA = 0

(Eq. 3 repeated)

NA

20 = 0

(Eq. 4 repeated)

NB

0.5TA = 0

(Eq. 5s repeated)

788

j=0

5.3 Rigid Bodies in Three-Dimensional Force Systems Example 8, page 8 of 8


15 Eqs. 9, 10, 3, 4, and 5 constitute five equations in four
unknowns. One equation must be redundant. Solving
these equations gives
NA = 20 lb

Ans.

NB = 2.89 lb

Ans.

TB = 5 lb

Ans.

TA = 5.77 lb

Ans.

789

6. Structural Applications

790

6.1 Frames and Machines

791

6.1 Frames and Machines Procedures and Strategies, page 1 of 3


Procedures and Strategies for Solving Problems Involving
Analysis of Frames and Machine

To solve problems involving frictionless pulleys,

1. Identify each rope and assign a unique label


such as T1, T2, ...., for the tension throughout the
length of the rope. Label the tensions on both
sides of pulleys with the same Ti, since tension
is the same on both sides of a frictionless
pulley.

T1

T1

T1

B
T2

T2

2. Draw free-body diagrams and write equilibrium


equations for various pulleys until you get
enough equations to solve for all the unknowns.
In drawing the diagrams, remember that tension
acting on a portion of a rope is always directed
away from the rope.

C
T1

T1

T2

792

6.1 Frames and Machines Procedures and Strategies, page 2 of 3


To solve problems involving frames and machines other than pulleys,
1. Look for two-force members. Do not draw free-body diagram of these
members.
2. Select one of the multi-force members that is acted upon by forces you are
to determine that is, forces which were requested in the problem statement
and draw a free-body diagram of this member.
a) On the diagram, show the force from any connected two-force member
as a single unknown magnitude (not as two unknown components) and a
known line of action (passing through the two load points of the two-force
member).
b) Assign the senses of all unknown forces arbitrarily if you guess
wrong, at the end of the solution process the forces will be found to be
negative.
3. Count the number of unknowns. If it exceeds the number of equilibrium
equations available, draw a free-body diagram of another multi-force
member. This time, however, you cannot assign the senses of all the forces
arbitrarily forces that have already been shown on another free-body
diagram must now be shown with the opposite sense on the new diagram.
4. Continue to count the number of unknowns and add free-body diagrams
until you have enough equations to solve for all the unknowns. Remember
that sometimes you can save work by using a free body of the whole frame.

793

6.1 Frames and Machines Procedures and Strategies, page 3 of 3


5. Write equilibrium equations. Try to find
points for summing moments that will lead to
an equation with a single unknown. A point
where the lines of action of several forces
intersect would be an example.
Note: Special care must be given to situations where
three or more members are connected by a pin. One
approach is to assign the pin arbitrarily to one of the
free bodies. An alternative approach is to draw a
separate free-body diagram of the pin. Drawing a
separate free-body diagram increases the number of
unknowns but will help you understand which forces
act on which bodies.

P
A

C
B

Free-body diagram of AB with pin included at B


P
A
Ax

By
B
Bx

Ay

FBD (Force from


two-force member
BD acting on pin B)

Free-body diagram of BC without including pin at B


Q
By
B

Bx
Force FBD is not present since
it acts on pin B, but pin B is
not part of this free body

794

Cy

6.1 Frames and Machines Problem Statement for Example 1


1. A pair of pliers is used to hold a short, smooth
rod in position. Determine the force that the pliers
applies to the rod.

18 lb

C
A
0.25 in.

D
E
1.5 in.

3 in.
18 lb

795

6.1 Frames and Machines Problem Statement for Example 2


2. Determine the force that the nutcracker
applies to the top and bottom of the nut.
A

20 lb
C

1 in

D
1.5 in.

F
5 in.

20 lb

796

6.1 Frames and Machines Problem Statement for Example 3


3. Determine the support force at A and the
tension in the cord. The pulleys are frictionless,
and their weight can be neglected.

90 kg

797

6.1 Frames and Machines Problem Statement for Example 4


4. Determine the values of the weight W and the
support forces at A and B. The pulleys are frictionless,
and their weight can be neglected.

B
C

400 lb
W
A

798

6.1 Frames and Machines Problem Statement for Example 5


5. Determine the force P and the force in the connecting
bar EF. The pulleys are frictionless and their weight can
be neglected.

400 lb

799

6.1 Frames and Machines Problem Statement for Example 6


6. Determine the values of the reactions at the
supports, A and B. The pulleys are frictionless,
and their weight can be neglected.
G

H
C

E
A

100 kg

800

6.1 Frames and Machines Problem Statement for Example 7


7. While standing on a 50-lb platform, a 150-lb man supports
himself by pulling with force P on the rope. Determine the value of
P and the force that the man's feet exert on the platform. Assume
that the pulley is frictionless, the platform remains horizontal, and
the inclination of the rope can be neglected.

801

6.1 Frames and Machines Problem Statement for Example 8


8. A 180-lb man supports himself, while standing in the middle of a
20-lb platform, by pulling on the ropes. Determine the force that the
man's feet exert on the platform. The pulleys are frictionless, each
weighs 10 lb, and the platform is of uniform density.

D
B

F= 0

F
3 ft

3 ft

802

6.1 Frames and Machines Problem Statement for Example 9


9. The frame is pin-connected at D and at the supports
A and B. Determine the reactions at A and B.
6 ft
10 kip

6 ft

9 ft

803

6.1 Frames and Machines Problem Statement for Example 10


10. A trailer is connected to a truck by a ball-and-socket
trailer hitch B. Determine the force of each tire on the
ground, when the truck and trailer are parked.

5000 lb
900 lb

6 ft
1.5 ft

4 ft

4.5 ft

2 ft

804

6.1 Frames and Machines Problem Statement for Example 11


11. Determine the couple moment M applied to the crankshaft
that will keep the piston motionless when a 400 psi pressure acts
on the top of the piston. The diameter of the piston is 3 in., and
the piston slides with negligible friction in the cylinder.

400 psi
C

8 in.

B
M

4 in.
A

2.5 in

805

6.1 Frames and Machines Problem Statement for Example 12


12. Determine the force in spring AD for the wheel
suspension-system shown. The ground exerts a
900-lb vertical force on the wheel.
6 in.
B

10 in.

8 in.
900 lb

7 in.

3 in.

806

6.1 Frames and Machines Problem Statement for Example 13


13. Determine the reactions at A and C.
B
2m
2 kN

3 kN

3m
A

40

60
C

807

6.1 Frames and Machines Problem Statement for Example 14


14. Determine the force P required to keep the two 20-lb, uniform rods
of length L in equilibrium. The rods are prevented from moving out of
the plane of the figure.
Pin-connection
B

30

30

Smooth floor

808

6.1 Frames and Machines Problem Statement for Example 15


15. Determine the resultant force applied to the
material at A by the crushing mechanism. Neglect
friction acting on the sides of the piston.
200 mm
C

D
500 mm

300 N
325 mm
B

270 mm

809

6.1 Frames and Machines Problem Statement for Example 16


16. Determine the forces transmitted by the rollers. Horizontal
motion is prevented by the short links at A, E, and F.
4 kN

Beam 1

Beam 2

Beam 3

6m

B
D

C
G

3m

3m

810

6.1 Frames and Machines Problem Statement for Example 17


17. Determine the resultant force
transmitted by the pin at D in the tongs.
400 lb
A
45

45

C
14 in.

4 in.
D

18 in.

400 lb

8 in.

8 in.

811

6.1 Frames and Machines Problem Statement for Example 18


18. Determine all forces acting on member FCB.
The 2-kip force is applied to pin F.
F

2 kip

6 ft
4 kipft

C
3 ft
B
8 ft

4 ft

3 ft

812

6.1 Frames and Machines Problem Statement for Example 19


19. Determine all forces acting on pin D.
Members CD and DE are quarter-circle arcs.
C

14 in.
14 in.
40 lb

14 in.
D

20 in.

813

6.1 Frames and Machines Problem Statement for Example 20


20. Members AB, BC, and BED are connected at pin B.
Determine the reactions at the supports A, C, and D.
D
30 in.

100 lb
60
A

B
E
25 in.
C

25 in.

25 in.

50 in.

814

6.1 Frames and Machines Problem Statement for Example 21


21. Pin C is attached to member BD and slides freely
in the slot cut in member AE. Determine the tension
in the cord BE and the force transmitted by pin C.
A

100 lb

5 ft
C
5 ft
D
E
3 ft

3 ft

815

6.1 Frames and Machines Problem Statement for Example 22


22. The sleeve C is pinned to bar AE and can slide feely
on the smooth bar BD. Calculate reaction components
from the supports at D and E.
A

200 mm
C

200 mm

30 kg
D

300 mm

300 mm

816

6.1 Frames and Machines Problem Statement for Example 23


23. Determine all the forces acting on pin B.
E
1 ft
0.5 ft

A
B

C
2 ft

40 lb
D
3 ft

3 ft

817

6.1 Frames and Machines Problem Statement for Example 24


24. The smooth cylinder is supported by two pinned
members held together by spring AB. The floor upon
which the members rest is smooth. Determine the
unstretched length of the spring.

5 in.-diameter
100 lb
O

24 in.
k = 25 lb/in.
B

6 in.

6 in.

818

6.1 Frames and Machines Problem Statement for Example 25


25. Determine all forces acting on pin D.
A

Radius = 100 mm

300 mm
B

250 mm
20 kg

400 mm

819

6.1 Frames and Machines Problem Statement for Example 26


26. Motor E and pulley B weigh 25 lb and 3 lb respectively
and are supported by the two-dimensional frame shown.
When the motor is turned off, the belt tension is 2 lb. The
15-lb force acts on member ADG, not on pin A. Determine
the tension in cable FG and all forces acting on member ADG.

15 lb

6-in. dia.
B
C

6-in. dia.
3.5 ft
E
D

3.5 ft

H
2 ft

2 ft

820

6.1 Frames and Machines Example 1, page 1 of 2


1. A pair of pliers is used to hold a short, smooth
rod in position. Determine the force that the pliers
applies to the rod.

18 lb

C
A
0.25 in.

D
E
1.5 in.

3 in.
18 lb

821

6.1 Frames and Machines Example 1, page 2 of 2


1
2

Free-body diagram of member DBC

Since the rod is smooth, no


friction force acts between the
rod and the jaw of the pliers;
the only force is the normal
force.
By

18 lb

FD
Bx

0.25 in.
3
+

Fx = 0: Bx = 0
Fy = 0:

Equilibrium equations for member DBC:

1.5 in.

3 in.

FD + By

MB = 0: FD(1.5 in.)

18 lb = 0
(18 lb)(3 in.) = 0

Solving these equations simultaneously gives

822

Bx = 0

Ans.

By = 54 lb

Ans.

FD = 36 lb

Ans.

6.1 Frames and Machines Example 2, page 1 of 3


2. Determine the force that the nutcracker
applies to the top and bottom of the nut.
A

20 lb
C

1 in

1.5 in.
1

5 in.

20 lb

Free-body diagram of member AD. Note that it is a 2-force member (a


member loaded at two and only two points). Generally, you do not need to
draw a free-body diagram of a two-force member; it is shown here to illustrate
what we can conclude about the direction of the forces acting on the member.
Ay
A

Because AD is a two-force member, Ax is zero (If


it were not zero, the sum of moments about D
wouldn't be zero, and the body would spin).

Because AD is a two-force member, Dx is zero (If


it were not zero, the sum of moments about A
wouldn't be zero, and the body would spin).

Because AD is a two-force member, the force at D


must be equal and opposite to the force at A.

Ax = 0

Dx = 0
Dy = Ay

823

6.1 Frames and Machines Example 2, page 2 of 3


20 lb

Free-body diagram of ABC

5
A

B
Bx

Ay

By
1.5 in.

5 in.

Ay pointed up on the free body of AD, so by Newton's Third


Law, it must point down on the neighboring body, ABC.

Equilibrium equations for member ABC.


Fx = 0: Bx = 0:
Fy = 0:

Ay + By

MB = 0: Ay(1.5 in.)

20 lb = 0
(20 lb)(5 in.) = 0

Solving these equations simultaneously gives


Bx = 0

Ans.

Ay = 66.7 lb

Ans.

By = 86.7 lb

Ans.

824

6.1 Frames and Machines Example 2, page 3 of 3


8

Free-body diagram of nut


By = 86.67 lb

Equilibrium equation for the nut:

Ey

Fy = -86.7 + Ey = 0
Solving gives
Ey = 86.7 lb

Ans.

10 Note that the resultant force applied


to the nut is zero, since the forces
acting on the top and bottom cancel.

825

6.1 Frames and Machines Example 3, page 1 of 2


3. Determine the support force at A and the
tension in the cord. The pulleys are frictionless,
and their weight can be neglected.
A

C
1

Tension is same on
both sides of pulley C.

Tension is same on
both sides of pulley D.

D
T

T
90 kg
D

90 kg

826

6.1 Frames and Machines Example 3, page 2 of 2

T
Tension forces are directed
away from the body.

6
5
D

Free-body diagram of pulley C

Equilibrium equation for pulley D:


Fy = 0: 3T

FA

882.9 N = 0

Solving gives
T = 294.3 N

Ans.

90 kg

T = 294.3 N T = 294.3 N

Weight of block = mg

Equilibrium equation for pulley C

= (90 kg)(9.81 m/s2)

Free-body diagram of pulley D. This


free body is chosen first because only
one unknown force, T, is present.

= 882.9 N

Fy = 0: FA

2(294.3 N) = 0

Solving gives
FA = 589 N

827

Ans.

6.1 Frames and Machines Example 4, page 1 of 2


4. Determine the values of the weight W and the
support forces at A and B. The pulleys are frictionless,
and their weight can be neglected.

1
T = 400 lb

D
T* = W

T = 400 lb

400 lb

400 lb

Tension is same,
400 lb, on both
sides of pulley C.

828

T* = W

Tension is same, T* = W,
on both sides of pulley D.
Note that this is a different
cord than the cord that goes
around pulley C. That is
why the tension is labeled
T* rather than T.

6.1 Frames and Machines Example 4, page 2 of 2


3

Free-body diagram of pulley D. This free


body is chosen first because only one
unknown force (W) is present.

Free-body diagram of pulley C.


FB

T = 400 lb

C
D
4 The tension forces are
directed away from the body.

T = 400 lb

T* = W

T* = W

Solving gives

Thus the support force at A is 200 lb

Fy = 0: FB

2(400 lb) = 0

FB = 800 lb

2W = 0

W = 200 lb

Equilibrium equation for pulley C:

Solving gives

Equilibrium equation for pulley D:


Fy = 0: 400

T = 400 lb

Ans.
Ans.

829

Ans.

6.1 Frames and Machines Example 5, page 1 of 3


5. Determine the force P and the force in the connecting
bar EF. The pulleys are frictionless and their weight can
be neglected.

400 lb

830

6.1 Frames and Machines Example 5, page 2 of 3


B

A
3

Tension is same, P, on
both sides of pulley C.

Tension is same, P, on
both sides of pulley D.

P
P
P

Tension is same, P, on
both sides of pulley E .

P
2

P
P
F

400 lb

831

Tension is same, P, on
both sides of pulley F.

6.1 Frames and Machines Example 5, page 3 of 3


Free-body diagram of pulley F. This free body is
chosen because it involves only the force P and the
force in the bar EF. The support forces at A and B,
which we have not been asked to calculate, do not
appear.
P

Fy = 0: 2P + FEF

400 lb = 0

(1)

Free-body diagram of pulley E. This free body is


chosen because it involves only the forces P and FEF.

FEF

Fy = 0: 2P

FEF = 0

(2)

Solving Eqs. 1 and 2 simultaneously gives


400 lb

P = 100 lb

Ans.

FEF = 200 lb

Ans.

FEF

832

6.1 Frames and Machines Example 6, page 1 of 2


6. Determine the values of the reactions at the
supports, A and B. The pulleys are frictionless,
and their weight can be neglected.
G

H
C

T*

T*

T*

T*

F
E
A
T
100 kg

833

Tensions on both
sides of pulleys C,
D, and E are equal.
Note that this is a
different cord than
the cord wrapped
around pulley F, so
the label T* is used
instead of T.

T
F

100 kg

Tensions on
both sides of
pulley F are
equal.

6.1 Frames and Machines Example 6, page 2 of 2


3

Free-body diagram of pulley F. This


free body is chosen first because only
one unknown force, T, is present.
T

Free-body diagram for pulley E

T*

Equilibrium equation for pulley F:


Fy = 0: 2T

T*

981 N = 0
E

Solving gives
T = 490.5 N
Reaction at B = T = 490.5 N

Ans.
T = 490.5 N

Tension in cable supporting the


weight = mg = (100 kg)(9.81 m/s2)
= 981 N

Equilibrium equation for pulley E:


Fy = 0: 2T*

490.5 N = 0

Solving gives
T* = 245.25 N
Reaction at A is T* = 245

834

Ans.

6.1 Frames and Machines Example 7, page 1 of 2


7. While standing on a 50-lb platform, a 150-lb man supports
himself by pulling with force P on the rope. Determine the value of
P and the force that the man's feet exert on the platform. Assume
that the pulley is frictionless, the platform remains horizontal, and
the inclination of the rope can be neglected.

835

Tension is same, P, on
both sides of pulley.

6.1 Frames and Machines Example 7, page 2 of 2


2

Free-body diagram of man and platform.


This free body is chosen first because only
one unknown force (P) is present.
P

Free-body diagram of platform. Note carefully that the


150-lb weight of the man is not included in this free
body because the man is not part of the free body.

P = 100 lb

Center of
gravity of man

Weight of man
= 150 lb

Ffeet

Weight of platform = 50 lb

Weight of platform = 50 lb

Equilibrium equation for man and platform:


Fy = 2P

150 lb 50 lb = 0

Solving gives
P = 100 lb

Equilibrium equation for platform:


Fy = 100 lb

Ffeet 50 lb = 0

Solving gives
Ans.

Ffeet = 50 lb

836

Ans.

6.1 Frames and Machines Example 8, page 1 of 3


8. A 180-lb man supports himself, while standing in the middle of a
20-lb platform, by pulling on the ropes. Determine the force that the
man's feet exert on the platform. The pulleys are frictionless, each
weighs 10 lb, and the platform is of uniform density.

D
B

F= 0

A
2
E

F
3 ft

3 ft

Tension T* is
same on both
sides of pulleys
A and B.

D
B

T*

F= 0

837

Tension T is
same on both
sides of pulleys
C and D.

6.1 Frames and Machines Example 8, page 2 of 3


Free-body diagram of platform. This free body
is chosen first because it will allow us to show
that T = T*. Note carefully that the weight of the
man is not shown because the man is not part of
the free body.

T*

T
Ffeet

O
3 ft

Now we ask, "Is there another free-body diagram that


we can choose that will not introduce any new
unknowns?" The answer is "Yes." Pass a section
through the ropes as shown in the figure below, and
draw the free-body diagram of the part of the system
below the section.

3 ft
A

Weight of platform = 20 lb
B

Equilibrium equations for platform:


Fy = 0: T* + T

MO = 0:

Ffeet

3T* + 3T = 0

20 lb = 0

(1)
(2)

F= 0

Solving Eq. 2 gives T = T*, a result we expected because


of symmetry.
E

F
3 ft

838

3 ft

6.1 Frames and Machines Example 8, page 3 of 3


Free-body diagram of part below section.
Because of symmetry, T* = T.
T T T
T* T* T* T T T

Equilibrium equation for part below section:

Fy = 0: 6T

180 lb

20 lb

20 lb = 0

Solving gives
F= 0

T = 36.67 lb.
Using this result in Eq. 1 gives
E

F
3 ft

Ffeet = 53.3 lb

Ans.

3 ft

Weight of man = 180 lb


Weight of platform = 20 lb
Weight of two pulleys = 2(10 lb) = 20 lb

839

6.1 Frames and Machines Example 9, page 1 of 3


9. The frame is pin-connected at D and at the supports
A and B. Determine the reactions at A and B.
6 ft
10 kip

6 ft

Free-body diagram of member DEB


6 ft

FBD
D

9 ft

B
9 ft
2

DEB is a two-force
member (loaded at
two and only two
points, and no couple
moment acts).

B
FBD

840

No equilibrium equation needs to be written. Because DEB


is a two-force member, we know that the forces at B and D
are equal, opposite, and have a line of action passing
through B and D. Usually, it is unnecessary to draw a
free-body diagram of a two-force member; it was drawn
here to emphasize that the line of action of the forces is
known.

6.1 Frames and Machines Example 9, page 2 of 3


Free-body diagram of member ACD

10 kip

FBD

9 ft

Geometry

7
B

Ax
6 ft

6 ft

Ay

Because BD is a two-force member, only one


unknown (the magnitude of FBD) is shown on the
free-body diagram at D.

= tan-1 6 = 33.69
9

Equilibrium equationd for member ACD:


Fx = 0: 10 kip + Ax
Fy = 0: Ay + FBD cos

MB = 0:

FBD sin

(10 kip)(9 ft)

=0

=0
Ay (6 ft + 6 ft) = 0

(1)
8

(2)
(3)

Solving Eqs. 1, 2, and 3 simultaneously gives


Ax = 5.0 kip

Ans.

Ay = 7.5 kip

Ans.

FBD = 9.014 kip

841

6.1 Frames and Machines Example 9, page 3 of 3


9

The horizontal and vertical components of FBD are


equal to the components of the force from the support
acting on the frame at B.
FBD = 9.014 kip

By = (9.014 kip) cos 33.69 = 7.5 kip

Ans.

= 33.69
B
Bx = (9.014 kip) sin 33.69 = 5.0 kip

Ans.

842

6.1 Frames and Machines Example 10, page 1 of 3


10. A trailer is connected to a truck by a ball-and-socket
trailer hitch B. Determine the force of each tire on the
ground, when the truck and trailer are parked.

5000 lb
900 lb

6 ft
1.5 ft

4 ft

4.5 ft

2 ft

843

6.1 Frames and Machines Example 10, page 2 of 3


1

Free-body diagram of truck and trailer

5000 lb
900 lb

A
2FA
10 ft

2FC

2FD

4.5 ft

1.5 ft

2 ft

If FD represents the force acting


on one front wheel, then 2FD
represents the resultant acting on
both front wheels.

Equilibrium equations for truck and trailer:


Fy = 0: 2FA
MA = 0:

900 lb + 2FC

5000 lb + 2FD = 0

(900 lb)(1.5 ft) + 2FC (1.5 ft + 10 ft)


5000 lb (1.5 ft +10 ft + 4.5 ft)
+ 2FD (1.5 ft +10 ft + 4.5 ft + 2 ft) = 0

(1)
4
(2)

844

Two equations in three


unknowns, so an additional
equation is needed.

6.1 Frames and Machines Example 10, page 3 of 3


Free-body diagram of trailer alone

Equilibrium equation for trailer alone:

MB = 0:

FA(1.5 ft + 6 ft) + (900 lb)(6 ft) = 0

Solving gives

900 lb

B
FA = 360 lb

A
FB
2FA

Ans.

Substituting this result into Eqs. 1 and 2 and solving


simultaneously gives

6 ft
1.5 ft

FC = 915 lb

Ans.

FD = 1675 lb

Ans.

845

(3)

6.1 Frames and Machines Example 11, page 1 of 3


11. Determine the couple moment M applied to the crankshaft
that will keep the piston motionless when a 400 psi pressure acts
on the top of the piston. The diameter of the piston is 3 in., and
the piston slides with negligible friction in the cylinder.

400 psi
C

8 in.

B
M

4 in.
A

2.5 in

846

6.1 Frames and Machines Example 11, page 2 of 3


1

Free-body diagram of entire mechanism


2

Resultant of pressure = (400 psi)( (3 in./2)2 = 2827 lb

3
N

8 in.

Equilibrium equation for the entire mechanism


(Because we weren't asked to determine Ax and
Ay, we don't bother to write the equilibrium
equations for the horizontal and vertical forces):

Since friction is negligible, only the normal


force, N, acts on the side of the piston.

MA = 0: N(8 in. + 4 in.)

M=0

4 in.

Ax
A
4
Ay

Because member AB is not a two-force member


(a couple moment acts on it), the line of action
of the reaction force acting at A is not known.
Thus two unknown components, Ax and Ay, are
shown.

847

(1)

6.1 Frames and Machines Example 11, page 3 of 3


6

Free-body diagram of piston


2827 lb

FBC

7 Equilibrium equation for the piston


+

Fx = N + FBC sin

Fy = FBC cos

Geometry

=0

(2)

2827 lb = 0

(3)

Solving Eqs. 2 and 3, with

= 17.35, gives

FBC = 2,960 lb
N = 883 lb
Substituting N = 883 lb in Eq. 1 and solving gives

8 in.
= tan-1 (

2.5 in. )
= 17.35
8 in.

M = 10,600 lbin.

B 2.5 in.

848

Ans.

6.1 Frames and Machines Example 12, page 1 of 3


12. Determine the force in spring AD for the wheel
suspension-system shown. The ground exerts a
900-lb vertical force on the wheel.
6 in.
B

10 in.

8 in.
900 lb

7 in.

3 in.

849

6.1 Frames and Machines Example 12, page 2 of 3


Free-body diagram of wheel and axle
FAB

Because member AB is a
two-force member, only one
unknown force component, FAB,
is shown acting at B.

10 in.
4

E
3
Ey
8 in.

Because member EDC is not a


two-force member (it is loaded at
three points), two unknown
components, Ex and Ey, are
shown acting at E.

Equilibrium equations for the wheel and axle:


+

Fx = 0: Ex

Ex

Fy = 0: Ey + 900 lb = 0

900 lb

ME = 0:

FAB = 0

FAB(10 in.)

(900 lb)(8 in.) = 0

Solving simultaneously gives


Ex = 720 lb
Ey = 900 lb
FAB = 720 lb

850

6.1 Frames and Machines Example 12, page 3 of 3


Free-body diagram of member CDE
Fspring
Ex

Cy

7
Cx

Equilibrium equation for member CDE

MC = 0:

(900 lb)(7 in. + 3 in.) + Fspring(3in.) = 0

C
Solving gives
7 in.

3 in.

Ey = 900 lb
6

Fspring = 3,000 lb

Newton's third law says that Ey should be


directed downward on this diagram because it
was directed upwards on the free-body diagram
of the wheel and axle. But we found that Ey
was negative, so we now reverse its downward
direction.

851

Ans.

6.1 Frames and Machines Example 13, page 1 of 3


13. Determine the reactions at A and C.
B
2m
2 kN

3 kN

3m
A

40

60
C

852

6.1 Frames and Machines Example 13, page 2 of 3


1

Free-body diagram of entire frame


B
2m
2 kN

3 kN

3m
Ax

40

60
C
3

Cx
L1

L2

Ay

Geometry

Cy
2m+3m=5m

Equilibrium equation for the entire frame

Fx = 0: Ax + Cx + 2 kN

3 kN = 0

(1)

Fy = 0: Ay + Cy = 0

(2)

MA = 0:

(3)

(2 kN)(3 m) + (3 kN)(3 m) + Cy(L1 + L2) = 0

5m
L1 = ( tan 40 ) = 5.959 m

Three equations but four unknowns, so


another free-body diagram is needed.

853

60

40

L1

L2

5m
L2 = ( tan 60 ) = 2.887 m

6.1 Frames and Machines Example 13, page 3 of 3


Free-body diagram of member BC

Bx
2m

By

3m
60

L2 = 2.887 m

MB = 0: Cy(2.887 m) + Cx(3 m + 2 m)

(3 kN)(2 m) = 0

C
Cx

L2

Since we were not asked to find Bx and By, point B is a


good point for summing moments because Bx and By
will not appear in the equation:

3 kN

Cy

Solving Eqs. 1, 2, 3 and 4 gives


Ax = 0.396 kN

Ans.

Ay = 0.339 kN

Ans.

Cx = 1.396 kN

Ans.

Cy = 0.339 kN

Ans.

854

(4)

6.1 Frames and Machines Example 14, page 1 of 3


14. Determine the force P required to keep the two 20-lb, uniform rods
of length L in equilibrium. The rods are prevented from moving out of
the plane of the figure.
Pin-connection
B

30

30

Smooth floor

855

6.1 Frames and Machines Example 14, page 2 of 3


1

Free-body diagram of the two rods together


B

30

Weight = 20 lb

30

C
3

N
2
Because the floor is smooth,
no friction force is shown;
only a normal force is present.

Cx
Equilibrium equation for the two rods together. Because
four unknown forces are present in the free-body
diagram, we know that we must draw an additional
diagram to get enough equations. Note also that we were
not asked to compute Cx and Cy, so we would like to
save work, if we can, by not writing any equilibrium
equations involving these quantities. Of course, if it
turns out later that we need the values of Cx and Cy to
determine P, then we can return to this free body and
write the equations at that time.

Cy
L cos 30
2

MC = 0: 20 lb ( 3 L ) cos 30 + (20 lb) ( L) cos 30


2
2
L
N (4
) cos 30 = 0
2
Solving gives (note that L cancels out)

Weight = 20 lb

N = 20 lb

856

6.1 Frames and Machines Example 14, page 3 of 3


4

Free-body diagram of rod AB


By
B

Bx

20 lb
L sin 30
30

5
N = 20 lb

L cos 30
2

A
Equilibrium equation for rod AB. Summing moments about B
eliminates Bx and By and gives us one equation in one
unknown, P. Thus we don't have to compute forces Cx, Cy, Bx,
and By.
MB = 0: P(L sin 30) + (20 lb) L cos 30 (20 lb)(L cos 30) = 0
2

Solving gives
P = 17.32 lb

857

Ans.

6.1 Frames and Machines Example 15, page 1 of 2


15. Determine the resultant force applied to the
material at A by the crushing mechanism. Neglect
friction acting on the sides of the piston.
200 mm
1
C

Free-body diagram of piston (This is a good free body


to use first, since it shows Fcrush, the quantity that we
want to calculate.)
Ay

D
500 mm

300 N

325 mm
B

Ax and Ay are the


force components
from member
ABD acting on
the piston.

NA

Ax
A

Normal force
from the wall

270 mm
Fcrush
A

858

Equilibrium equations for piston. Two equations, but


four unknowns, so additional equations are needed.
+

Fx = 0:

NA + Ax = 0

(1)

Fy = 0: Fcrush + Ay = 0

(2)

6.1 Frames and Machines Example 15, page 2 of 2


5 Free-body diagram of member ABD

BC is a
two-force
member.

Geometry
C

FBC
B

325 mm

300 N
500 mm
200 mm

270 mm
B
270 mm

Ax

mm
= tan-1 ( 500
200 mm ) = 68.20

200 mm
Ay
Equilibrium equations for member ABD
Fx = 0:

Ax + FBC cos

Fy = 0:

Ay

MB = 0:

Ax(270 mm) + Ay(200 mm)


+ (300 N) (325 mm) = 0

FBC sin

(300 N) cos

=0

(3)

+ (300 N) sin

=0

(4)

mm
= tan-1 (200
270 mm) = 36.53

Substituting = 68.20 and = 36.53 into


Eqs. 3, 4, and 5 and solving Eqs. 1-5
simultaneously gives
Ax = NA = 16.5 N
FBC = 693 N

(5)

Ay = 465 N
Fcrush = 465 N

859

Ans.

6.1 Frames and Machines Example 16, page 1 of 3


16. Determine the forces transmitted by the rollers. Horizontal
motion is prevented by the short links at A, E, and F.
4 kN

Beam 1

Beam 2

Beam 3

6m

B
D

C
G

3m

3m

860

6.1 Frames and Machines Example 16, page 2 of 3


Free-body diagram of beam 1 (This is a good
free body to begin with, since only three
unknown forces are present).
4 kN

Ax

Beam 1

Ex

6m

Ax = 0

Fy = 0: Ay + By

4
(1)

4 kN = 0

MA = 0: By(9 m + 3 m)

(2)

(4 kN)(9 m) = 0

(3)

Solving simultaneously gives

Cy
6m

Equilibrium equations for beam 2:


Fx = 0:

Ex = 0

Fy = 0: Cy + Dy

Fx = 0:

D
Dy

3m

Equilibrium equations for beam 1:

Beam 2

By
9m

By = 3 kN

Ay = 1 kN

Ay

Free-body diagram of beam 2

MC = 0:

(4)
1 kN

Dy(6 m)

3 kN = 0

(5)

(1 kN)(6 m + 6 m) = 0 (6)

Solving simultaneously gives

Ax = 0

Ex = 0

Ay = 1 kN

Ans.

Cy = 2 kN

Ans.

By = 3 kN

Ans.

Dy = 2 kN

Ans.

861

6.1 Frames and Machines Example 16, page 3 of 3


Free-body diagram of beam 3
Dy = 2 kN

Fx

Fy

Beam 3

6m

Cy = 2 kN

6m

Gy

Equilibrium equations for beam 3


+

Fx = 0:

Fx = 0

Fy = 0: Fy

2 kN

(7)
2 kN + Gy = 0

MG = 0: (2 kN)(6 m)

Fy(6 m + 6 m) = 0

(8)
(9)

Solving simultaneously gives


Fx = 0
Fy = 1 kN

Ans.

Gy = 3 kN

Ans.

862

6.1 Frames and Machines Example 17, page 1 of 4


17. Determine the resultant force
transmitted by the pin at D in the tongs.
400 lb
A
45

45

C
14 in.

4 in.
D

18 in.

400 lb

8 in.

8 in.

863

6.1 Frames and Machines Example 17, page 2 of 4


Free-body diagram of member BDF (This is a
good free body to begin with, since it shows the
components of force transmitted by pin D).
FAB

2
45

14 in.

AB is a two-force member, so only one unknown,


FAB, is shown (the magnitude of the force is
unknown; the line of action of the force is known).

Dy
Dx

Equilibrium equations for member BDF:

Fx = 0: FAB cos 45 + Dx + Fx = 0

(1)

Fy = 0: FAB sin 45 + Dy

(2)

18 in.

Fx

MD = 0: Fx(18 in.) Fy(8 in.) FAB cos 45(14 in.)


FAB sin 45(4 in.) = 0

(3)

F
3

4 in. 8 in.

Fy = 0

Fy

The tongs must depend on a friction


force to be able to lift the weight;
both a normal force, Fx, and a friction
force, Fy, are present.

864

Three equations but five unknowns so at


least one more free body is needed (to obtain
more equations of equilibrium).

6.1 Frames and Machines Example 17, page 3 of 4


8

Free-body diagram of connection A.


400 lb

Since both AB and AC


are two-force members,
only two unknowns
(magnitudes of FAB and
FAC) appear on this free
body.

A
FAB

Equilibrium equations for connection A:


+

Fx = 0:

Fy = 0: 400 lb

FAC

FAB cos 45 + FAC cos 45 = 0


FAB sin 45

FAC sin 45 = 0

(4)
(5)

Solving gives
FAB = 282.8 lb

45

45
FAC = 282.8 lb

Free-body diagram of block EF


8 in.

8 in.

10 Equilibrium equations for block EF:


Fx

Fy = 0: Ey + Fy

Ex
E

Ey

ME = 0:

(400 lb)(8 in.) + Fy(16 in.) = 0

Solving gives

Fy

Ey = 200 lb

400 lb

Fy = 200 lb

865

400 lb = 0

(6)
(7)

6.1 Frames and Machines Example 17, page 4 of 4


11 Substituting FAB = 282.8 lb and Fy = 200 lb
into Eqs. 1, 2, and 3 and solving gives
Dx = 488.9 lb
Dy = 0.0 lb
Fx = 288.9 lb
The resultant force transmitted at D is, then,
Dx2+ Dy2

( 488.9 lb)2 + (0.0)2

= 489

lb

Ans.

866

6.1 Frames and Machines Example 18, page 1 of 5


18. Determine all forces acting on member FCB.
The 2-kip force is applied to pin F.
F

2 kip

6 ft
4 kipft

C
3 ft
B
8 ft

4 ft

3 ft

867

6.1 Frames and Machines Example 18, page 2 of 5


1

Free-body diagram of member FCB not including the pin at F.


Fy

Fx and Fy are forces from the pin


acting on member FCB. The force
from member FD and the 2-kip force
are not shown since they act on the
pin, not on member FCB.

2
Fx

6 ft
Cy
Cx

Equilibrium equations for member FCB:


+

Fx = 0: Fx + Cx = 0

(1)

Fy = 0: Fy + Cy + By = 0

(2)

MC = 0:

(3)

Fx (6 ft) = 0

4
By

Eqs. 1 and 3 imply


Fx = 0

Ans.

Cx = 0

Ans.

868

Three unknowns, Fy, Cy, and By, remain to


be determined, so at least one additional
free-body diagram is needed.

6.1 Frames and Machines Example 18, page 3 of 5


Free-body diagram of pin at F (This is a good choice
for the next free body to use, since it will allow us to
calculate Fy).
Fy

2 kip

Fx = 0
8
FDF

DF is a two-force
member, so DF's line
of action is known.

= tan-1 (
6 ft

Fx = 0:

2 kip + FDF sin

Equilibrium equations for pin at F:


+

Fy = 0:

Fy

FDF cos

=0
=0

Geometry

4 ft
) = 33.69
6 ft

(4)
C

(5)

D
4 ft

Substituting = 33.69 in Eqs. 4 and 5 and solving


simultaneously gives
FDF = 3.606 kip
Fy = 3 kip

Ans.

10 Two unknowns, Cy and By, remain to be determined.

869

6.1 Frames and Machines Example 18, page 4 of 5


12 Equilibrium equation for entire frame:

11 Free-body diagram of entire frame (This free-body


diagram will allow us to determine By).
2 kip

ME = 0:

4 kipft + (2 kip)(6 ft)


By (7 ft) = 0

(6)

Solving gives
By = 1.143 kip

6 ft
4 kipft

Substituting By = 1.143 kip and Fy = 3 kip into Eq. 2


D

Ans.

E
Fy + Cy + By = 0

Ex

(Eq. 2 repeated)

and solving gives


Ey

Cy = 1.857 kip
7 ft
By

870

Ans.

6.1 Frames and Machines Example 18, page 5 of 5


13 Free-body diagram of member FCB (not
including pin F) showing correct senses
Fy = 3 kip
F

Fx = 0

Cy = 1.857 kip
C

Cx = 0

B
By = 1.143 kip

871

6.1 Frames and Machines Example 19, page 1 of 3


19. Determine all forces acting on pin D.
Members CD and DE are quarter-circle arcs.
C

14 in.
14 in.
40 lb

14 in.

20 in.
1

Three-dimensional free-body diagram of pin D


y

A
FDE 2

x
z

FCD

FBD
4

BD is also a two-force member.


Since member BD is horizontal,
force FBD is also horizontal.

872

DE is a two-force member, so only one


unknown component, FDE, is shown (the
line of action of the force is known to
pass through E and D).

CD is also a two-force member (The line of action


of FCD is known to pass through C and D).

6.1 Frames and Machines Example 19, page 2 of 3


Two-dimensional free-body diagram of pin D

E
14 in.

FDE

FCD
45
FBD

14 in.

Equilibrium equations for pin D:

45

Fx = 0: FBD + FCD cos 45


Fy = 0:

FCD sin 45

FDE cos 45 = 0

FDE sin 45 = 0

(1)
(2)

Two equations but three unknowns so at least one more


free body is needed (to obtain another equation of
equilibrium).

873

6.1 Frames and Machines Example 19, page 3 of 3


8 Free-body diagram of portion ABCD of
frame including pin at D

To simplify the calculation of moments, FDE has been


expressed in terms of horizontal and vertical components.

C
10 Equilibrium equation for ABCD:

FDE sin 45

FDE cos 45
40 lb

MA = 0:

(40 lb)(20 in.) + FDE (cos 45)(20 in.)


FDE(sin 45)(14 in.) = 0

(3)

Solving Eq. 3 gives


FDE = 188.6 lb

20 in.

A
Ax

Ans.

Using this result in Eqs. 1 and 2 and solving simultaneously gives


FBD = 267 lb

Ans.

FCD = 188.6 lb

Ans.

14 in.
Ay

FDE = 188.6 lb
y

11 Free-body diagram of pin D showing


correct senses of forces

FBD = 267 lb
z
FCD = 188.6 lb

874

6.1 Frames and Machines Example 20, page 1 of 3


20. Members AB, BC, and BED are connected at pin B.
Determine the reactions at the supports A, C, and D.
D
30 in.

100 lb
60
A

B
E
25 in.
C

25 in.

25 in.

50 in.

875

6.1 Frames and Machines Example 20, page 2 of 3


1

Free-body diagram of entire frame

BD is a two-force member, so only one unknown


the magnitude of the force is shown; the line of
action of the force is known.

FBD
D

100 lb
60
B

25 in.

25 in.

50 in.

BC is also a two-force member.

FBC

Equilibrium equations for entire frame:

4
+

Fx = 0: 100 cos 60 FBD cos


FBC cos = 0

Ay

876

Fy = 0:
MB = 0:

100 sin 60 + Ay
+ FBC sin = 0

(1)

FBD sin

Ay (25 in. + 25 in.)


+ (100 sin 60)(25 in.) = 0

Three equations and three unknowns so no more


free bodies are needed.

(2)

(3)

6.1 Frames and Machines Example 20, page 3 of 3


6

Geometry

30 in.
B

= tan-1( 30 in. ) = 30.96


50 in.
-1 25 in.
= tan (
) = 26.57
50 in.

E
25 in.

50 in.

Substituting = 30.96 and = 26.57 in Eqs. 1, 2,


and 3 and solving simultaneously gives
FBC = 74.5 lb

Ans.

FBD = 19.40 lb

Ans.

Ay = 43.3 lb

Ans.

877

6.1 Frames and Machines Example 21, page 1 of 3


21. Pin C is attached to member BD and slides freely
in the slot cut in member AE. Determine the tension
in the cord BE and the force transmitted by pin C.
A

100 lb

5 ft
C
5 ft
D
E
3 ft

3 ft

878

6.1 Frames and Machines Example 21, page 2 of 3


Free-body diagram of member
AE, including pin at E.
A
Ax

5 ft

Since we assumed that the pin at E is


part of the free body, then the tension
T in the cord must be shown (if, as
seems reasonable, we assume that the
cord is attached to the pin)

C
LCE

FC

5 ft

5 ft

Ey

Ex
E
Because the pin slides freely in
the slot, only a normal
component, FC, is present (No
friction acts).

E
3 ft
= tan-1 ( 5 ft ) = 59.04
3 ft
= 90

Equilibrium equation for member AE (Since we weren't asked to


calculate Ex and Ey, we won't write any additional equilibrium
equations that would introduce Ex and Ey as additional
unknowns):

ME = 0:

Geometry

Ax(5 ft + 5 ft) + FC LCE = 0

(1)

879

LCE =

= 30.96

5 ft
= 5.831 ft
sin

We need another free-body one from


which we can calculate Ax easily.

6.1 Frames and Machines Example 21, page 3 of 3


7

Free-body diagram of entire frame (We


can calculate Ax from this free body).
A

Ax

Free-body diagram of member DCB, including pin at


D (We can calculate T easily from this free body)
B

100 lb

100 lb
5 ft

= 30.96

10 ft

FC = 171.5 lb

5 ft
Dx

Dx

FDE
D
Dy

Dy

3 ft

Equilibrium equation for entire frame:

3 ft

11 Equilibrium equation for member DCB


MD = 0:

Ax(10 ft) + (100 lb)(10 ft) = 0

(2)

10 The force FC
acting on the
pin acts at
the angle
previously
calculated.

Solving gives Ax = 100 lb and using this value in Eq. 1


gives
FC = 171.5 lb

MD = 0:

(171.5 lb)(cos 30.96)(5 ft)


+ (171.5 lb)(sin 30.96)(3 ft)
+ (100 lb)(5 ft + 5 ft) T(3 ft + 3 ft) = 0

Solving gives

Ans.

T = 88.2 lb

880

Ans.

6.1 Frames and Machines Example 22, page 1 of 4


22. The sleeve C is pinned to bar AE and can slide feely
on the smooth bar BD. Calculate reaction components
from the supports at D and E.
A

200 mm
C

200 mm

30 kg
D

300 mm

300 mm

881

6.1 Frames and Machines Example 22, page 2 of 4


1

Free-body diagram of entire frame


A

Weight = mg
C
2

= (30 kg)(9.81 m/s )

400 mm

= 294.3 N

Dy

300 mm

300 mm

E
Ey

Fx = 0: Dx + Ex = 0

(1)

Ex

Fy = 0: Dy + Ey

(2)

Dx

Equilibrium equation for entire frame

294.3 N = 0

MD = 0: Ey(300 mm + 300 mm)


(294.3 N)(300 mm) = 0

Solving Eqs. 2 and 3 gives

882

Dy = 147.2 N

Ans.

Ey = 147.2 N

Ans.

(3)
Dx and Ex remain
unknown, so at least
one more free-body
diagram is needed.

6.1 Frames and Machines Example 22, page 3 of 4


4

Free-body diagram of member DCB

5
FAB

Member AB is a two-force member so only one


unknown (the magnitude) is shown.

B
Since the sleeve slides freely, it exerts a normal force, FC ,
only (no friction force parallel to member DCB). Note that
FC does not lie in the direction of member ACB.

Equilibrium equations for member DC:

400 mm

FC

Dy = 147.2 N
600 mm

Dx

Fy = 0: 147.2 N

FC sin

=0

MB = 0: Dx(400 mm) + (FC)LBC


(147.2 N) (600 mm) = 0

Geometry

200 mm
C

= tan-1(
= 90
LBC =

300 mm

(5)

LBC

883

(4)

200 mm
) = 33.69
300 mm
= 56.31

300 mm
= 360.56 mm
cos 33.69

6.1 Frames and Machines Example 22, page 4 of 4


9

Substituting = 56.31 and LBC = 360.56 mm into


Eqs. 4 and 5 and solving simultaneously gives
FC = 176.9 N
Dx = 61.3 N

Ans.

Then Eq. 1 gives


Ex = Dx
Ex = 61.3 N

Ans.

884

6.1 Frames and Machines Example 23, page 1 of 4


23. Determine all the forces acting on pin B.
E
1 ft
0.5 ft

A
B

C
2 ft

40 lb
D
3 ft

3 ft

Three-dimensional free-body diagram of pin B


y

Bx
3

BE is a two-force
member, so only one
unknown, FBE, (the
magnitude of the
force) is shown. The
line of action of the
force is known.
4

885

FBE

x
By

B`x
B`y

z
2 Bx and By represent
the force components
from member ABC.

B`x and B`y represent the force


components from the pulley.

6.1 Frames and Machines Example 23, page 2 of 4


Two-dimensional free-body diagram of pin B

B`x

Bx
FBE

8 Two equations but five unknowns so at least one more


free body is needed (to obtain additional equations of
equilibrium).

By

B`y

Free-body diagram of pulley, not including pin at B


40 lb

Equilibrium equations for pin B:

Fx = 0: Bx + B`x + FBE cos

= 0

(1)

Fy = 0: By + B`y + FBE sin

=0

(2)

10 B`x and B`y represent the forces


from the pin acting on the pulley.
40 lb

Geometry

B`y

11 Equilibrium equations for the pulley:


+

Fx = 0:

B`x + 40 lb = 0

B`x

Fy = 0:

40

1.5 ft
B

Solving simultaneously gives

3 ft
= tan-1(

B`y = 0

1.5 ft.
) = 26.57
3 ft.

886

B`x = 40.0 lb

Ans.

B`y = 40.0 lb

Ans.

6.1 Frames and Machines Example 23, page 3 of 4


12 Free-body diagram of member ABC not including pin at B
Cy
A

Bx
C

Cx

By
Ay

13 Bx and By represent the forces from


the pin acting on member ABC.
14 Equilibrium equation for member ABC:
+

Fx = 0:

Bx + Cx = 0

(5)

15 This free body introduces three additional


unknowns (Cx, Cy, and Ay), so it looks like a
poor choice. But by expressing Bx in terms of
Cx through Eq. 5, we only need one more free
body to complete the solution.

887

6.1 Frames and Machines Example 23, page 4 of 4


16 Free-body diagram of member DCE

18 Equilibrium equation for member DCE:


E

= 26.57
FBE

1 ft

40 lb
Cx

(6)

Substituting B`x = 40 lb in Eq. 1 and then solving Eqs. 1, 5,


and 6 simultaneously gives

0.5 ft
17 The tension in the
cord is 40 lb on both
sides of the pulley, so
the tension force
acting on member
DCE is 40 lb

MD = 0: Cx(2 ft) + (40 lb)(2 ft + 0.5 ft)


+ (FBE cos )(2 ft + 0.5 ft + 1 ft) = 0

C
Cx = 26.7 lb
Cy

2 ft
D

Bx = 26.7 lb

Ans.

FBE = 14.91 lb

Ans.

Dx
Substituting B`y = 40 lb and FBE = 14.91 lb in Eq. 2 and
then solving for By gives

Dy

By = 46.7 lb

Ans.

19 Free-body diagram of pin B with correct sense of forces


FBE = 14.91 lb

Bx = 26.7 lb
By = 46.7 lb

B`x = 40 lb

x
B`y = 40 lb

888

6.1 Frames and Machines Example 24, page 1 of 4


24. The smooth cylinder is supported by two pinned
members held together by spring AB. The floor upon
which the members rest is smooth. Determine the
unstretched length of the spring.

5 in.-diameter
100 lb
O

24 in.
1
k = 25 lb/in.
B

The unstretched length of the spring is


related to the force in the spring:
Fspring = k
=k

6 in.

6 in.

(extension of spring)
(Lstretched

Lunstretched)

= (25 lb/in.)[(6 in. + 6 in.)

Lunstretched]

So to calculate Lunstretched we first must calculate Fspring.

889

(1)

6.1 Frames and Machines Example 24, page 2 of 4


2

Free-body diagram of member BCD

Cy

FD
LCD

Equilibrium equation for member BCD. Since we weren't


asked to calculate Cx and Cy, we won't write equilibrium
equations involving them. If we find later that we need to
know these values, we can always return to this free body
and write additional equations.

Cx
Radius =

MC = 0: FD(LCD) + FB(6 in.)

Fspring(24 in.) = 0

(2)

Geometry

5 in.
= 2.5 in.
2
D

24 in.

6 By similar triangles,

LCD

LCD
2.5 in.
=
24 in.
6 in.

Solving gives LCD = 10 in.

Fspring
B
6 in.

24 in.

FB
3

Since the cylinder is smooth, only a force FD


normal to the member is present (no friction
force parallel to the member)

6 in.
7

890

To calculate Fspring from Eq. 2, we need first to calculate


FD and FB.

6.1 Frames and Machines Example 24, page 3 of 4


8

Free-body diagram of the cylinder (This free


body will give us FD).

10 Geometry

100 lb
D

C
= tan-1 (

FD

FE

24 in.
) = 75.96
6 in.
24 in.

9
+

Fx = 0: FD sin

FE sin

Equilibrium equation for the cylinder:

Fy = 0: FD cos

+ FE cos

=0

(3)
6 in.

100 lb = 0

(4)
11 Substituting = 75.96 in Eqs. 3 and 4 and
solving gives
FE = 206.1 lb
FD = 206.1 lb

891

6.1 Frames and Machines Example 24, page 4 of 4


13 Equilibrium equation for the entire structure:

12 Free-body diagram of the entire structure


(This free body will give us FB).

MA = 0:

(100 lb)(6 in.) + FB (6 in. + 6 in.) = 0

100 lb
Solving gives
FB = 50 lb
14 Substituting LCD = 10 in., FB = 50 lb, and FD = 206.1 lb into Eq. 2:
FD(LCD) + FB(6 in.)

Fspring(24 in.) = 0

(Eq. 2 repeated)

and solving gives


Fspring = 98.38 lb
Substituting Fspring = 98.38 lb into Eq. 1:
Fspring = (25 lb/in.)(12 in

(Eq. 1 repeated)

B
6 in.

FA

Lunstretched)

and solving gives

6 in.
FB

Lunstretched = 8.07 in.

892

Ans.

6.1 Frames and Machines Example 25, page 1 of 3


25. Determine all forces acting on pin D.
A

Radius = 100 mm

300 mm
B

1 Three-dimensional free-body diagram of pin D


250 mm
20 kg

400 mm

Dx

Dx and Dy represent
the force components
from the pulley.

x
FBD

Dy
3

FCD
4

Similiarly, CD is a
two-force member, so only
one unknown (the
magnitude of the force) is
shown; the direction of the
force is known.

893

BD is a two-force member,
so only one unknown
component (the magnitude
of the force) is shown ; the
line of action is known.

6.1 Frames and Machines Example 25, page 2 of 3


5

Two-dimensional free-body diagram of pin D

Free-body diagram of pulley


T = 196.2 N

Dy

FBD

Dx
Dx

FCD
9

Equilibrium equation for pin D:

6
+

Fx = 0: FBD + Dx + FCD cos

Fy = 0: FCD sin

+ Dy = 0

The tension is the same on


both sides of the pulley.
Dy

(1)
(2)

Weight = mg
= (20 kg)(9.81m/s2)
= 196.2N

Geometry

10 Equilibrium equations for the pulley:

400

32

250
= tan-1( 250 ) = 32.01
400
C

894

Fx = 0:

Dx

(196.2 N) cos

=0

Fy = 0:

Dy

196.2 N + (196.2 N) sin

(3)

=0
=0

(4)

6.1 Frames and Machines Example 25, page 3 of 3


11 Geometry

13 Substituting = 25.33 in Eqs. 3 and 4 and


solving simultaneously gives

300
300

Dx = 177.3 N

Ans.

Dy = 112.3 N

Ans.

100 cos
Radius = 100

Substituting these values and = 32.01 into


Eqs. 1 and 2 and solving simultaneously gives
100 cos

100 sin

400

FBD = 2.2 N

Ans.

FCD = 212 N

Ans.

400 + 100 sin


12 From the figure above, we see that
following equation:
tan

=(

300 100 cos


400 + 100 sin

Solving numerically gives

14 Free-body diagram of pin D with correct


sense of forces
must satisfy the
Dy = 112 N
y

Dx = 177 N
x

= 25.33
z

FBD = 2.2 N
FCD = 212 N

895

6.1 Frames and Machines Example 26, page 1 of 7


26. Motor E and pulley B weigh 25 lb and 3 lb respectively
and are supported by the two-dimensional frame shown.
When the motor is turned off, the belt tension is 2 lb. The
15-lb force acts on member ADG, not on pin A. Determine
the tension in cable FG and all forces acting on member ADG.

15 lb

6-in. dia.
B
C

6-in. dia.
3.5 ft
E
D

3.5 ft

H
2 ft

2 ft

896

6.1 Frames and Machines Example 26, page 2 of 7


1

Free-body diagram of member ADG, not including pin G and not


including pin A (Since we are asked to find all the forces acting
on member ADG, ADG is the best choice for the first free-body
diagram).
Ay
A

Ax

3.5 ft
Dy
D

Dx

2 Forces Gx and Gy are from the pin G


acting on the member ADG. The force
from the cable does not appear here,
since the cable is attached to the pin, and
the pin is not part of the free body.

3 Equilibrium equations for ADG:

3.5 ft
Gy
Gx

15 lb

Fx = 0: 15 lb + Ax + Dx + Gx = 0

(1)

Fy = 0: Ay + Dy + Gy = 0

(2)

MG = 0:

15 lb(3.5 ft + 3.5 ft)

897

Ax(3.5 ft + 3.5 ft)

Dx(3.5 ft) = 0

(3)

6.1 Frames and Machines Example 26, page 3 of 7


Free-body diagram of pin G.
5
G`y
6

G'x and G'y are


forces from the
Gy
pin support.

The tension T comes from the cable.


Since we are asked to determine T,
pin G is a good choice for the next
free-body diagram.

Gx

Equilibrium equations for pin G:

G`x
Here the forces Gx and Gy from
member ADG are equal and opposite
those specified in the free-body
diagram for member ADG.

Fx = 0:

Gx + G`x + T cos

=0

(4)

Fy = 0:

Gy + G`y + T sin

=0

(5)

Geometry

F
= tan-1(

3.5
) = 41.19
4
3.5

H
4

898

6.1 Frames and Machines Example 26, page 4 of 7


10 Free-body diagram of entire body.
Using this free body next will allow
us to determine G'x and G'y.
Weight = 3 lb

15 lb

6-in dia
B
C

6-in dia
3.5 ft
E
D

3.5 ft
Weight = 25 lb

Fx = 0: 15 lb + G`x = 0
MH = 0:

15 lb(3.5 ft + 3.5 ft)


+ 3 lb(2 ft) 25 lb(2 ft)
G`y(2 ft + 2 ft) = 0

Solving Eqs. 6 and 7 gives

G
G`y

11 Only the forces G'x and G'y


from the support are shown.
The forces Gx, Gy, and T are
internal forces for this free
body and so are not shown.
G`x

12 Equilibrium equations for the entire body


(Only two equations are needed, because we
don't need to calculate the reaction Hy.):

G`x = 15 lb
2 ft

2 ft
Hy

899

G`y = 12.25 lb

(6)

(7)

6.1 Frames and Machines Example 26, page 5 of 7


13 Free-body diagram of upper shelf and pulley.
This free body will allow us to calculate Ay.
Weight = 3 lb

14 The diameter of the pulley is 6 in. so


the radius is 3 in. (= 0.25 ft).
6 in. dia
B

Ax
2 lb

2 lb

Cx
15 Equilibrium equation for the upper shelf and pulley:

Ay

Cy

2 ft 0.25 ft
= 1.75 ft
4 ft

MC = 0: Ay(4 ft) + (3 lb)(1.75 ft + 0.25 ft) + (2 lb)(1.75 ft)


+ (2 lb)(1.75 ft + 0.25 ft + 0.25 ft) = 0

Solving gives
Ay = 3.5 lb

900

Ans.

6.1 Frames and Machines Example 26, page 6 of 7


16 This free body of the lower shelf and motor
will allow us to calculate Dy.
2 lb

2 lb
6 in. dia (= 0.5 ft)
E

Dx

2 ft
Dy

Fy

Weight = 25 lb
1.75 ft

1.75 ft
4 ft

17 Since pin F is not


included in the
Fx
free-body, the
tension of the cable
is not shown
(Although it doesn't
really affect our
calculation, since
we sum moments
about point F and
no force at F would
enter the equation).

Ay + Dy + Gy = 0

(Eq. 2 repeated)

Substituting Ay = 3.5 lb and Dy = 10.5 lb and


then solving gives
Gy = 14 lb

Ans.

20 Recall that Eq. 5 is

18 Equilibrium equation for the lower shelf and motor:

19 Recall that Eq. 2 is

Gy + G`y + T sin

MF = 0: Dy(4 ft) + (25 lb)(2 ft)


(2lb)(1.75 ft + 0.5 ft)
(2 lb)(1.75 ft) = 0

=0

(Eq. 5 repeated)

Substituting Gy = 14 lb, G`y = 12.25 lb, and


= 41.19 and then solving gives

Solving gives
T = 39.86 lb
Dy = 10.5 lb

Ans.

901

Ans.

6.1 Frames and Machines Example 26, page 7 of 7


21 Recall that Eq. 4 is
Gx + G`x + T cos

=0

(Eq. 4 repeated)

Substituting G`x = 15.0 lb, T = 39.86 lb and


then solving gives
Gx = 15 lb

= 41.19 and
23 Free-body diagram of member
ADG not including pin G (correct
sense of forces are shown).

Ans.

Ay = 3.5 lb

22 Finally recall that Eqs. 1 and 3 are


15 lb + Ax + Dx + Gx = 0
15 lb(7 ft)

Ax(7 ft)

(Eq. 1 repeated)

15 lb

Ax = 0

Dx(3.5 ft) = 0 (Eq. 3 repeated)

Substituting Gx = 15.0 lb and solving simultaneously gives


Dy = 10.5 lb
Ax = 0

Ans.

Dx = 30.0 lb

Ans.

Dx = 30.0 lb

24 Force G'x, G'y, and T are not


shown, since they act on pin
G, not on member ADG.

902

Gy = 14.0 lb
Gx = 15.0 lb

6.2 Trusses: Method of Joints and Zero-Force Members

903

6.2 Trusses: Method of Joints and Zero-Force Members Procedures and Strategies, page 1 of 2
Procedures and Strategies for Solving Problems
Involving Trusses: the Method of Joints

10 kN
C

1. Examine the truss and try to find a joint at which


you can calculate at least one member force.
Such joints occur where

a) two members meet and an external load is


present,
b) two members meet at a support (the reaction
forces at the support must first be found by
using a free-body analysis of the entire truss),
and
c) three members meet and two are collinear.

A
G

E
B

2. Draw a free-body diagram of the joint and show


all unknown forces to be tensile (directed away
from the joint). Write the Fx = 0 = Fy
equilibrium equations and solve for the unknown
member force or forces. If a member force turns
out to be negative, then the force is compressive.

2 kip

4 kip

2 kip

K
I

x
E

60
F
60

30

D
B

904

2 kip

2 kip

Summing forces in the


(inclined) y direction will
give an equation with a single
unknown, FIF, so FIF can be
found easily.
A

6.2 Trusses: Method of Joints and Zero-Force Members Procedures and Strategies, page 2 of 2
3. Examine the truss again, as you did in Step 1, but
take into account that one or more member forces
are known because you just now found them.
Repeat Step 2, being careful to show compressive
forces pointing towards the joint, tensile forces away
from the joint. Proceed in this manner joint-by-joint
through the entire truss.

4 kN

4 kN
D

60

4. You can save work by identifying zero-force


members by inspection. The common situations in
which zero-force members occur are where

B
2 kip

a) two members meet at an unloaded joint, and


b) three members meet at an unloaded joint, and
two of the members are collinear the
non-collinear member is a zero-force member.

J
I

A
B

905

6.2 Trusses: Method of Joints and Zero-Force Members Problem Statement for Example 1
1. Determine the force in each member of the truss and
state whether the force is tension or compression.
C

10 kN

3m
A

B
5m

906

6.2 Trusses: Method of Joints and Zero-Force Members Problem Statement for Example 2
2. Determine the force in each member of the truss and state
whether the force is tension or compression.
F

14 ft

10 ft

2 kip

4 kip
10 ft

2 kip
10 ft

10 ft

907

6.2 Trusses: Method of Joints and Zero-Force Members Problem Statement for Example 3
3. Determine the force in each member of
the truss and state whether the force is
tension or compression.
B

900 lb

30

A
60
12 ft

400 lb
10 ft

908

6.2 Trusses: Method of Joints and Zero-Force Members Problem Statement for Example 4
4. Determine the force in each member of the truss and state whether
the force is tension or compression. The truss is symmetric.
6 ft

6 ft

6 ft

6 ft

6 ft

6 ft

2 kip
2 kip

2 kip
K

2 kip

2 kip

I
E
A

60
F
60

30

D
B

909

6.2 Trusses: Method of Joints and Zero-Force Members Problem Statement for Example 5
5. Determine the force in each member of the truss and
state whether the force is tension or compression.

100 lb

1 ft
E

2.5 ft
A

4 ft

2 ft

4 ft

910

6.2 Trusses: Method of Joints and Zero-Force Members Problem Statement for Example 6
6. Determine the force in each member of the truss and
state whether the force is tension or compression.
2 kip

J
I

14 ft
L

A
B

16 ft

16 ft

911

6.2 Trusses: Method of Joints and Zero-Force Members Problem Statement for Example 7
7. Determine the force in each member and state
whether the force is tension or compression.
4 kN
H

I
60

B
2m

2m

912

6.2 Trusses: Method of Joints and Zero-Force Members Example 1, page 1 of 3


1. Determine the force in each member of the truss and
state whether the force is tension or compression.
C

10 kN

3m
A

B
5m

Free-body diagram of entire truss. Calculating the reactions


is a good place to start because they are usually easy to
compute, and they can be used in the equilibrium equations
for the joints where the reactions act.
10 kN
C

2
+

Fx = 0: Ax + 10 kN = 0

(1)

Fy = 0: Ay + By = 0

(2)

MA = 0:

(3)

3m

Equilibrium equations for entire truss

Solving these equations simultaneously gives

Ax
Ay

(10 kN)(3 m) + By(5 m) = 0

By

Ax = 10 kN, Ay = 6 kN, and By = 6 kN

5m

913

6.2 Trusses: Method of Joints and Zero-Force Members Example 1, page 2 of 3


3

Free-body diagram of joint C


10 kN

Geometry
= tan-1( 5 ) = 59.04
3

3m

FAC
FBC

B
5m

Equilibrium equations for joint C. It is a good idea


to assume all members in tension (forces point away
from the joint, not towards it). Then, after solving
the equilibrium equations, you will know
immediately that any member force found to be
negative must be compression.

Using = 59.04 in Eqs. 4 and 5 and solving


simultaneously gives
FAC = 11.66 kN (T)

Ans.

FBC = 6.0 kN = 6.0 kN (C)

Ans.

and
Fx = 0: 10 kN
Fy = 0:

FAC sin

FAC cos

=0

(4)

FBC = 0

(5)

Writing "(T)" after the numerical value shows that


the member is in tension. We had arbitrarily
assumed member BC to be in tension. We then
found that the member force was negative, so we
know that our assumption was wrong. Member BC
is in compression, and we show this by writing a
positive "6.0" followed by "(C)".

914

6.2 Trusses: Method of Joints and Zero-Force Members Example 1, page 3 of 3


Free-body diagram of joint B

FBC = 6 kN
B

FAB

8 The force FBCis directed toward


the joint because member BC is
known to be in compression.

By = 6 kN

9
+

Equilibrium equation for joint B


Fx = 0:

10 An "Answer diagram" summarizes the analysis


of the entire truss (All forces are in kN).

FAB = 0
C
10 kN

Solving gives
FAB = 0

T)

(
.66

Ans.

11
A

6.0 (C)
0

10
6

915

6.2 Trusses: Method of Joints and Zero-Force Members Example 2, page 1 of 7


2. Determine the force in each member of the truss and state
whether the force is tension or compression.
F

14 ft

10 ft

2 kip

4 kip
10 ft

2 kip
10 ft

10 ft

916

6.2 Trusses: Method of Joints and Zero-Force Members Example 2, page 2 of 7


Free-body diagram of entire truss.
Calculating the reactions is usually a
good way to start the analysis.

14 ft

Ax

Ey

Ay
4 kip

2 kip
10 ft

Equilibrium equations for entire truss


Fx = 0: Ax = 0
Fy = 0: Ay + Ey

MA = 0:

10 ft

10 ft
(1)

kip

2 kip)(10 ft)

kip

kip = 0

(4 kip)(20 ft)

(2)

(2 kip)(30 ft) + Ey(40 ft) = 0

Solving simultaneously gives


Ax = 0, Ay = 4.0 kip, and Ey = 4.0 kip.

917

2 kip

(3)

10 ft

6.2 Trusses: Method of Joints and Zero-Force Members Example 2, page 3 of 7


Free-body diagram of joint E. This joint is
chosen because only two unknown forces are
present. Thus we know that we can solve for
these forces because two equations of
equilibrium are available for the joint. Note
also that we assume that both unknown forces
are in tension (directed away from the joint).

Geometry
= tan-1( 14 ) = 54.46
10
H

14 ft

FEH
E
D

FDE
Ey = 4 kip

6
4

Equilibrium equations for joint E

Fx = 0:

FDE

Fy = 0: FEH sin

FEH cos

=0

(4)

4 kip = 0

(5)

918

E
10 ft

Using = 54.46 in Eqs. 4 and 5 and solving


simultaneously gives
FDE = 2.857 kip (T)

Ans.

FEH = 4.916 kip = 4.916 kip (C)

Ans.

We arbitrarily assumed member EH to be in tension. We


then found that the member force was negative, so we
know that our assumption was wrong. Member EH is in
compression, and we show this by writing a positive
"4.916" followed by "(C)".

6.2 Trusses: Method of Joints and Zero-Force Members Example 2, page 4 of 7


9

8 Use a free-body diagram of joint H next because only


two member forces are unknown.

FEH = 4.916 kip (C)


G

Free-body diagram of joint H

FGH

FDH

2 kip

4 kip

2 kip

11 Equilibrium equations for joint H


FDE = 2.857 kip (T)

Fx = 0:

FGH

(4.916 kip) cos 54.46 = 0

(6)

10 As before, we assume
that the unknown
FEH = 4.916 kip (C)
member forces (GH and
DH in this instance) are
= 54.46
tension, so are directed
away from the joint.
The force in member
EH has already been
found to be 4.916 kip
compression, so it is
directed towards the
joint, not away from it.

Fy = 0:

FDH + (4.916 kip) sin 54.46 = 0

(7)

Solving simultaneously gives


FGH = 2.858 kip = 2.858 kip (C)

Ans.

FDH = 4.0 kip (T)

Ans.

and

919

6.2 Trusses: Method of Joints and Zero-Force Members Example 2, page 5 of 7


FGH = 2.858 kip(C)
G

13 Free-body diagram of joint D

FEH = 4.916 kip (C)


H

FDH = 4.0 kip

FDG
= 54.46
FCD

FDE = 2.857 kip


2 kip

E
B

14 As before, we assume that the unknown


member forces are tension, so are directed away
from the joint. The forces in members DH and
DE have already been found to be tension and
so are directed away from the joint.

D
FDE = 2.857 kip (T)

4 kip

2 kip

12 Use a free-body
diagram of joint
D because only
two member
forces are
unknown.

15 Equilibrium equations for joint D


+

Fx = 0:

2 kip

Fy = 0: FDG sin(54.46) + 4.0 kip

FCD

FDG cos(54.46) + 2.857 kip = 0


2 kip = 0

(8)
(9)

Solving simultaneously gives


FCD = 4.286 kip (T)

Ans.

FDG = 2.458 kip = 2.458 kip (C)

Ans.

and

920

6.2 Trusses: Method of Joints and Zero-Force Members Example 2, page 6 of 7


FGH = 2.857 kip(C)
G

FEH = 4.916 kip (C)


H
FDH = 4.0 kip (T)
FDG= 2.458 kip (C)

E
C

4 kip

2 kip

2 kip

FCD = 4.286 kip (T)

FDE = 2.857 kip (T)


19 Equilibrium equations for joint C.

16 Use a free-body diagram of


joint C because only two
member forces are
unknown.
17 Free-body diagram of joint C
FCG
FBC

C
4 kip

FCD = 4.286 kip (T)

18 The unknown forces in


members CG and BC are
assumed to be tension, so
are directed away from
the joint. The force in
member CD has already
been found to be 4.286
kip (T).

921

Fx = 0:

Fy = 0: FCG

FBC + 4.286 kip = 0


4 kip = 0

(10)
(11)

Solving gives
FBC = 4.286 kip (T)

Ans.

FCG = 4 kip (T)

Ans.

and

6.2 Trusses: Method of Joints and Zero-Force Members Example 2, page 7 of 7


20 All remaining bar forces follow from symmetry.

All forces in kips.

Answer diagram

4.0 (T)

4.00 (T)

C)
6(
2.4

4.0 (T)
2

2.86 (T)

4.29 (T)

C)
2(

4.29 (T)

2.86 (T)

4.9

C)

4.9
2(

2.86 (C)

6(
2.4

C)

2.86 (C)

922

6.2 Trusses: Method of Joints and Zero-Force Members Example 3, page 1 of 4


3. Determine the force in each member of
the truss and state whether the force is
tension or compression.

900 lb

30

A
60
12 ft

400 lb
10 ft

923

6.2 Trusses: Method of Joints and Zero-Force Members Example 3, page 2 of 4


1

Free body-diagram of entire truss

Because AB is a two-force member, the line of


action of FAB must pass through A and B.

900 lb
30
B

FAB
60

Ex
Ey

400 lb

Equilibrium equations for entire truss

Fx = 0:

FAB sin 60 + Ex + (900 lb) cos 30 = 0

(1)

Fy = 0:

FAB cos 60 + Ey + (900 lb) sin 30

(2)

12 ft

MC = 0: (400 lb)(10 ft + FAB cos 60(10 ft) + Ex(12 ft) = 0

Solving simultaneously gives


10 ft

400 lb = 0

FAB = 347.8 lb
Ex = 478.2 lb,
Ey = 123.9 lb

924

(3)

6.2 Trusses: Method of Joints and Zero-Force Members Example 3, page 3 of 4


4

Free-body diagram of joint D. Joint D is chosen


because only two member forces are unknown there.

Free-body diagram of joint C. Joint C is chosen because


only two member forces are unknown there.
(900 lb) sin 30 = 450.0 lb

FBD
5
FDE

The unknown forces


have been assumed to
be tension.

400 lb
6

FBC
8

(900 lb) cos 30= 779.4 lb

The unknown
forces have
been assumed
to be tension.

FCE

Equilibrium equations for joint D


Equilibrium equations for joint C

9
Fx = 0: FDE = 0
Fy = 0: FBD

400 lb = 0

(4)
+

Fx = 0:

FBC +779.4 lb = 0

(6)

Fy = 0:

FCE + 450 lb = 0

(7)

(5)

Solving gives
Solving gives
FBD = 400 lb (T)
FDE = 0

Ans.
FBC = 779.4 lb (T)

Ans.

FCE = 450.0 lb (T)

Ans.

Ans.

925

6.2 Trusses: Method of Joints and Zero-Force Members Example 3, page 4 of 4


10 Free-body diagram of joint B. Only one
member force is unknown at this joint.
B
FAB = 347.8 lb

FBC = 779.4 lb

60

FBE

13 Using

FBD = 400 lb

FBE = 746.9 lb = 747 lb (C)

Fx =0: FBE cos


(347.8 lb) sin 60
+ 779.4 lb = 0

900 lb

(8)
B

12 Geometry
= tan-1( 12 )
10
= 50.19

10 ft

Ans.

14 Answer diagram (all forces in lb)

11 Equilibrium equations for joint B


+

= 50.19 in Eq. 8 and then solving gives

779 (T)

C
347.8 lb

60
747 (C)
450 (T)

12 ft

400 (T)

E
478

E
124
400

926

30

6.2 Trusses: Method of Joints and Zero-Force Members Example 4, page 1 of 9


4. Determine the force in each member of the truss and state whether
the force is tension or compression. The truss is symmetric.
6 ft

6 ft

6 ft

6 ft

6 ft

6 ft

2 kip
2 kip

2 kip
K

2 kip

2 kip

I
E
A

60
F
60

30

D
B

927

6.2 Trusses: Method of Joints and Zero-Force Members Example 4, page 2 of 9


Free-body diagram of entire truss

6 ft

6 ft

6 ft

6 ft

6 ft

6 ft

2 kip
2 kip

2 kip
2 Equilibrium equations for entire truss

2 kip

Ax

60
F
60

30

Fy = 0: Ay + Dy 2 kip
2 kip 2 kip 2 kip

D
Ay

Fx = 0: Ax = 0

C
Dy

2 kip

MA = 0: 2(kip)(6 ft) 2 kip(12 ft)


2(kip)(18 ft) 2 kip(24 ft)
2(kip)(30 ft) + Dy(36 ft) = 0

Solving simultaneously gives


Ax = 0
Ay = 5 kip
Dy = 5 kip

928

2 kip = 0

6.2 Trusses: Method of Joints and Zero-Force Members Example 4, page 3 of 9


2 kip
2 kip

2 kip
K

2 kip

2 kip

I
60

F
60

30

A
Ax = 0

J
H

D
C

Ay = 5 kip

Dy = 5 kip

Use a free-body diagram of joint A because


only two unknown member forces are
present.

Free-body diagram of joint A.


FAE

30

Fx = 0: FAB + FAE cos 30 = 0

5 Equilibrium equations for joint A

Fy = 0: FAE sin 30 + 5 kip = 0


Solving simultaneously gives

A
FAB
Ay = 5 kip

FAB = 8.660 kip (T)

Ans.

FAE = 10 kip = 10 kip (C)

Ans.

929

6.2 Trusses: Method of Joints and Zero-Force Members Example 4, page 4 of 9


6

Three unknown member forces are present at joint I, but


two of them, FEI and FIK , are collinear, so summing
forces perpendicular to FEI and FIK would give an
equation with FFI as the only unknown.

2 kip
2 kip

2 kip
K

2 kip

2 kip

I
7

Free-body diagram of joint I


2 kip
y

A
FIK
FEI

60

x
Ax = 0

30

F
60

D
C

Ay = 5 kip

Dy = 5 kip

I
FFI

Geometry of members at joint I


y
= 60

60 E
A

30

I
60

x
9

30
60
F

= 30 + 60
= 90
So member FI is
perpendicular to the x axis.
Thus the member force FFI
lies on the y axis.

930

Equilibrium equations for joint I


Fy = 0:

FFI sin 90

(2 kip) sin 60 = 0

Solving gives
FFI = 1.732 = 1.732 kip (C)

Ans.

6.2 Trusses: Method of Joints and Zero-Force Members Example 4, page 5 of 9


FFI = 1.732 kip (C)
(already known)

2 kip
2 kip

2 kip
K

2 kip

2 kip

I
E

60
F
60

30

A
Ax = 0

Ay = 5 kip

12 Free-body diagram of joint F

FFI = 1.732 kip (C)

Dy = 5 kip
y

10 Use the same technique at


joint F as was used at joint I:
sum forces perpendicular to
collinear members BF and FK.

60

FFK

FEF
60

11 Geometry of members at joint F


I
= 180 (60 + 60)
60
= 60
E
F
60
60

FBF

13 Equilibrium equations for joint F

Fy = 0: FEF sin 60

(1.732 kip) sin 60 = 0

Solving simultaneously gives


FEF = 1.732 kip (T)

931

Ans.

6.2 Trusses: Method of Joints and Zero-Force Members Example 4, page 6 of 9


FAE = 10.0 kip (C)
(already known)

2 kip
2 kip

2 kip
K

2 kip

2 kip

I
60

F
60

30

D
C

Ay = 5 kip

14 At joint E, now only two member


forces, FEI and FEB, are unknown.
15 Free-body diagram of joint E
2 kip
60

FEI

30
FEF = 1.732 kip (T)
30
FBE
FAE = 10 kip(C)
E
30

Dy = 5 kip

FEF = 1.732 kip (T)


(already known)

16 Equilibrium equations for joint E


+

Ax = 0

Fx = 0: (10 kip) cos 30 + FEI cos 30 + FBE cos 30 + 1.732 kip = 0


Fy = 0: (10 kip) sin 30 + FEI sin 30

FBE sin 30

Solving simultaneously gives


FEI = 9.0 kip = 9.0 kip (C)

Ans.

FBE = 3.0 kip = 3.0 kip (C)

Ans.

932

2 kip = 0

6.2 Trusses: Method of Joints and Zero-Force Members Example 4, page 7 of 9


2 kip
2 kip

FBE = 3 kip (C)


(already known)
2 kip

2 kip
K

2 kip

I
E
A

60
F
60

30

D
C

Ax = 0

Dy = 5 kip

FAB = 8.660 kip (T)


Ay = 5 kip
(already known)

17 At joint B, now only two member forces, FBF and


FBC, are unknown.

19 Equilibrium equations for joint B


+

Fx = 0: 3 kip) cos 30

8.660 kip + FBF cos 60 + FBC = 0

18 Free-body diagram of joint B


FBF
FBE = 3 kip(C)
30
FAB = 8.660 kip (T)

3 kip) sin 30 + FBF sin 60 = 0

Solving simultaneously gives

60
B

Fy = 0:

FBC

933

FBF = 1.732 kip (T)

Ans.

FBC = 5.196 kip (T)

Ans.

6.2 Trusses: Method of Joints and Zero-Force Members Example 4, page 8 of 9


20 The remaining unknown member forces, FIK and FFK,
can be found by re-using the free-body diagrams of
joints I and F.

23 Free-body diagram of joint F


FFI = 1.732 kip (C)

21 Free-body diagram of joint I


y

y
2 kip

FFK

x
F

FEF = 1.732 kip (T)

FIK

60
FEI = 9.0 kip (C)

I
FBF = 1.732 kip (T)

FFI = 1.732 kip (C)

24 Equilibrium equation for joint F

22 Equilibrium equations for joint I


Fx = 0: 9.0 kip

Fx = 0: FFK

(2 kip) cos 60 + FIK = 0

Solving gives
FIK = 8.0 kip = 8.0 kip (C)

1.732 kip (1.732 kip) cos 60


(1.732 kip) cos 60 = 0

Solving gives
Ans.

FFK = 3.464 kip (T)

934

Ans.

6.2 Trusses: Method of Joints and Zero-Force Members Example 4, page 9 of 9


25 By symmetry, all forces on the right half of
the truss are also known.
Answer diagram
2

8.66 (T)

6(
T)

1.7

32

3.4

32

(T)

1.7

C)

0 (C
)

1.732 (T)

(T)

0(

9.0

32

3.0

1.7

0
10.

1.732 (T)

T)

T)

(
32
1.7

C)

(
.00

8.0

0 (C
)

(C)

6(

3.4

0
8.0

(C)

(T)

All forces in kips

5.20 (T)

10.
00

C)
0(

3.0

8.66 (T)

(C)
D

935

6.2 Trusses: Method of Joints and Zero-Force Members Example 5, page 1 of 8


5. Determine the force in each member of the truss and
state whether the force is tension or compression.

100 lb

1 ft
E

2.5 ft
A

4 ft

2 ft

4 ft

936

6.2 Trusses: Method of Joints and Zero-Force Members Example 5, page 2 of 8


Free-body diagram of entire truss.
B
100 lb

D
1 ft
E

2.5 ft
A
F

Ax
Ay

2 ft

Fy

4 ft

Equilibrium equations for entire truss


Fx = 0: Ax + 100 lb = 0
Fy = 0: Ay + Fy = 0

4 ft

MA = 0:

100 lb)(1 ft + 2.5 ft) + Fy(4 ft +2 ft + 4 ft) = 0

Solving these equations simultaneously gives


Ax = 100 lb
Ay = 35 lb
Fy = 35 lb

937

6.2 Trusses: Method of Joints and Zero-Force Members Example 5, page 3 of 8


Free-body diagram
of entire truss.
100 lb

1 ft

Geometry

2.5 ft
4
Ax = 100 lb
A

Only two unknown member


forces act at joint F.

4 ft

F
E

3.5 ft

Fy = 35 lb

Ay = 35 lb
4 ft

2 ft

2.5 ft

4 ft

F
5

Free-body diagram of joint F.


FDF
FEF
F

4 ft
2.5 ft
) = 32.01
4 ft
= tan-1( 4 ft ) = 48.81
3.5 ft

= tan-1(

Fy = 35 lb
6

Equilibrium equations for joint F


Fx = 0:

FEF cos

Fy = 0: FEF sin

FDF sin
+ FDF cos

=0
+ 35 lb = 0

938

Solving the equilibrium equations with


= 32.01 and = 48.81 gives
FEF = 165.10 lb (T)

Ans.

FDF = 186.03 lb = 186.03 lb (C)

Ans.

6.2 Trusses: Method of Joints and Zero-Force Members Example 5, page 4 of 8


9

Free-body diagram
of entire truss.
100 lb

1 ft
E

2.5 ft
Only two unknown member
forces act at joint A.
Ay = 35 lb
4 ft

F
Fy = 35 lb

2 ft

4 ft

10 Free-body diagram of joint A


FAB

11 Equilibrium equations for joint A

FAC

Ax = 100 lb

Fx = 0:

Ax = 100 lb
A

Fy = 0: FAC sin

A
Ay = 35 lb
By symmetry, the angles and at joint A are the
same as we calculated at joint F. Thus

100 lb + FAC cos

+ FAB sin

+ FAB cos

Solving the above equations with


and = 48.81 gives

=0

35 lb = 0
= 32.01

= 32.01

FAC = 247.70 lb (T)

Ans.

= 48.81

FAB = 146.23 lb = 146.23 lb (C)

Ans.

939

6.2 Trusses: Method of Joints and Zero-Force Members Example 5, page 5 of 8


12 Free-body diagram of entire truss
B

13 Only two unknown member


forces act at joint D.

100 lb
FDF = 186.03 lb (C)
(already known)
48.81
Ax = 100 lb
A

Ay = 35 lb

Fy = 35 lb

FBD

15 Equilibrium equations for joint D.

Fx = 0:

FBD

14 Free-body diagram of joint D

Fy = 0:

FDE + (186.03 lb) cos 48.81 = 0

48.81
FDF = 186.03 lb (C)

(186.03 lb) sin 48.81 = 0

Solving these equations gives

FDE

940

FBD = 139.99 lb = 139.99 lb (C)

Ans.

FDE = 122.51 lb (T)

Ans.

6.2 Trusses: Method of Joints and Zero-Force Members Example 5, page 6 of 8


16 Free-body diagram of entire truss
B

100 lb
FAC = 247.70 lb (T)
(already known)

Ax = 100 lb
A
Ay = 35 lb

32.01

Fy = 35 lb

FCE

Fx = 0:

(247.70 lb) cos 32.01 + FCE = 0

FBC

FAC = 247.70 lb (T)

19 Equilibrium equations for joint C.

18 Free-body diagram of joint C.

17 Only two unknown member


forces act at joint C

Fy = 0:

(247.70 lb) sin 32.01 + FBC = 0

Solving these equations gives

941

FCE = 210.04 lb (T)

Ans.

FBC = 131.23 lb (T)

Ans.

6.2 Trusses: Method of Joints and Zero-Force Members Example 5, page 7 of 8


20 Free-body diagram of entire truss
B

FDE = 122.51 lb (T)


(already known)
FEF = 165.10 lb (T)
(already known)

100 lb
E

Ax = 100 lb

FCE = 210.04 lb (T)


(already known)

A
Ay = 35 lb

32.01
F

21 At joint E, member BE
is the only unknown
member force.

Fy = 35 lb

24 Geometry

22 Free-body diagram of joint E.

FBE
FCE = 210.04 lb (T)

FDE = 122.51 lb (T)


1 ft
E
32.01

C
FEF = 165.10 lb (T)

Fx = 0:

FBE cos

2 ft

25 Substituting = 26.57 in the equation for joint E


and solving gives

23 Equilibrium equation for joint E.


+

= tan-1( 1 ft ) = 26.57
2 ft

210.04 lb + (165.10 lb) cos 32.01 = 0

942

FBE = 78.31 lb = 78.31 lb (C)

Ans.

6.2 Trusses: Method of Joints and Zero-Force Members Example 5, page 8 of 8


26 Answer diagram
All forces in lb
100

78 (

C)

131 (T)

6
14

)
(C

140 (C)

210 (T)

123 (T)
E

18

6(

C)

16
5

(T)
8
4
2

(T
)

100
A

35

35

943

6.2 Trusses: Method of Joints and Zero-Force Members Example 6, page 1 of 9


6. Determine the force in each member of the truss and state
whether the force is tension or compression.
2 kip

J
I

14 ft
L

A
B

16 ft

16 ft

944

6.2 Trusses: Method of Joints and Zero-Force Members Example 6, page 2 of 9


2 Equilibrium equations for entire truss

Free-body diagram of entire truss

Fx = 0: Ax = 0

Fy = 0: Ay + Gy

2 kip

J
I

MA = 0:

2 kip(16 ft) + Gy(16 ft + 16 ft) = 0

Solving simultaneously gives

Ax = 0
Ay = 1 kip

14 ft
H

Gy = 1 kip.

L
G

A
Ay

Ax
16 ft

2 kip = 0

16 ft

945

F
Gy

6.2 Trusses: Method of Joints and Zero-Force Members Example 6, page 3 of 9


2 kip

6 Geometry
J

= tan-1( 14 ft ) = 41.19
16 ft

J
I

K
14 ft

14 ft
H

G
B
Ay = 1 kip

16 ft

Gy = 1 kip
16 ft

Free-body
diagram of joint G

16 ft
3

FGL
G
FFG
Gy = 1 kip

Only two unknown member


forces act at joint G.
Equilibrium equations for joint G

Fx = 0:

Fy = 0: FGL sin

FFG

FGL cos

=0

+ 1 kip = 0

946

Solving simultaneously gives


FFG = 1.143 kip (T)

Ans.

FGL = 1.518 kip = 1.518 kip (C)

Ans.

6.2 Trusses: Method of Joints and Zero-Force Members Example 6, page 4 of 9


2 kip
J
I

14 ft
8
H

L
G

A
C

Gy = 1 kip
16 ft

16 ft
9

Free-body diagram of joint F


FFL

FEF

10 Equilibrium equation for joint F

B
Ay = 1 kip

At joint F, no external forces


act, three members meet, and
two of these members are
collinear. So FL is a
zero-force member, as will
now be shown.

Fy = 0: FFL sin
Since sin

FFG = 1.143 kip


(already known)

947

=0

0, it follows that

FFL = 0

Ans.

6.2 Trusses: Method of Joints and Zero-Force Members Example 6, page 5 of 9


2 kip
J
I

12 At joint L, no external forces act, three


members meet, and two of these
members are collinear. So EL is a
zero-force member:

14 ft

FEL = 0

Ans.

H
L
A

G
B
Ay = 1 kip

11 Member LF has been omitted


because it is a zero-force member.

F
Gy = 1 kip

16 ft

16 ft

948

6.2 Trusses: Method of Joints and Zero-Force Members Example 6, page 6 of 9


2 kip

J
I

14 ft
H

L
G

A
B
Ay = 1 kip

F
Gy = 1 kip

16 ft

16 ft

14 Consideration of joint E shows that EK is a zero-force member:


FEK = 0

Ans.

But then consideration of joint K shows that DK is also a


zero-force member:
FDK = 0

Ans.

949

13 Members EL and FL have been omitted


because they are zero-force members.

6.2 Trusses: Method of Joints and Zero-Force Members Example 6, page 7 of 9


2 kip
17 You cannot conclude that member
DJ is a zero-force member by
looking at end J. (Instead, look at
end D.)

J
I

K
15 All zero-force members in the right
half of the truss have been omitted.

A
B
Ay = 1 kip

16 Because of symmetry, the members in


the left half of the truss must also be
zero-force and so can be omitted, too.

G
Gy = 1 kip

18 Consideration of joint D shows that DJ


must be a zero-force member:
FDJ = 0

950

Ans.

6.2 Trusses: Method of Joints and Zero-Force Members Example 6, page 8 of 9


20 Free-body diagram of joint L

2 kip

FKL
19 Zero-force member DJ
has been omitted.

L
J

FKL = FGL

FGL

K
21 We have previously shown that
FGL = 1.518 kip (C)

Consideration of free-body diagrams of K and L


show that

A
B
Ay = 1 kip

Ans.

FKL = FKJ = 1.518 kip (C)

Ans.

Gy = 1 kip

24 By symmetry,
FAH = 1.518 kip = FHI = FIJ

23 We have previously shown that

Ans

FFG = 1.143 kip (T)

22 Free-body diagram joint F

Ans.

Consideration of free-body diagrams of all joints in the lower


chord, B, C, D, E, and F, shows that all member forces there
must equal 1.143 kip (T).

F
FEF = FFG

951

6.2 Trusses: Method of Joints and Zero-Force Members Example 6, page 9 of 9


25 Answer diagram
2
All forces in kips
J
I

K
1.518 (C)

1.518 (C)
0
0

0
H

L
0

F
1

1
1.143 (T)

952

6.2 Trusses: Method of Joints and Zero-Force Members Example 7, page 1 of 7


7. Determine the force in each member and
state whether the force is tension or
compression.

Free-body diagram
of entire truss

4 kN

4 kN

I
60

I
60

G
Ax

A
Ay

B
2m

2m

Cy

Equilibrium equations for entire truss

Fx = 0: Ax = 0

2m
Fy = 0: Ay + Cy

2m

4 kN = 0

MA = 0: ( 4 kN)(2 m) + Cy(2 m + 2 m) = 0

Solving simultaneously gives


Ax = 0, Ay = 2 kN, and Cy = 2 kN.

953

6.2 Trusses: Method of Joints and Zero-Force Members Example 7, page 2 of 7


3

Free-body diagram of entire truss.

4 Two members meet at joint J, they are not collinear,


and no external force acts at the joint, so members IJ
and FJ must be zero-force members.

4 kN
H

Free-body diagram of joint J

60

FIJ
60

FFJ

Ax

A
Ay = 2 kN

Equilibrium equations for joint J

Fx = 0:

FIJ FFJ cos 60 = 0

Fy = 0:

FFJ sin 60 = 0

Solving simultaneously gives

Cy = 2 kN

954

FIJ = 0

Ans.

FFJ = 0

Ans.

6.2 Trusses: Method of Joints and Zero-Force Members Example 7, page 3 of 7


7 Two members meet at the joint, they are not
collinear, and no external forces act, so the
members carry zero force.

Members IJ and FJ have been omitted


because they are zero-force members.

4 kN
H

8 The same argument at G shows FG and


CG are zero-force members.

Ax

A
Ay = 2 kN

Cy = 2 kN

955

6.2 Trusses: Method of Joints and Zero-Force Members Example 7, page 4 of 7


4 kN

10 Free-body diagram of joint C


FCF

9 All members identified


as zero-force have been
omitted.

60
FBC

Cy = 2 kN

A
B

C
Cy = 2 kN

Ay = 2 kN

Fx = 0:

11 Equilibrium equations for joint C

Fy = 0: FCF sin 60 + 2 kN = 0

FBC FCF cos 60 = 0

Solving simultaneously gives

12 By symmetry,
FAE = 2.309 kN (C)
FAB = 1.155 kN (T)

956

FCF = 2.309 kN = 2.309 kN (C)

Ans.

FBC = 1.155 kN (T)

Ans.

6.2 Trusses: Method of Joints and Zero-Force Members Example 7, page 5 of 7


4 kN
I

14 Free-body diagram of joint B


FBF

FBE
A
B

C
Cy = 2 kN

60
FAB = 1.155 kN (T)

Ay = 2 kN
13

FBC = 1.155 kN (T)

Equilibrium equations for joint B


+

Fx = 0:

FAB = 1.155 kN (T)


(already known)

FBC = 1.155 kN (T)


(already known)

60

Fy = 0: FBE sin 60 + FBF sin 60 = 0

FBE cos 60 + FBF cos 60

1.155 kN + 1.155 kN = 0

Solving simultaneously gives


FBE = 0

Ans

FBF = 0

Ans.

957

6.2 Trusses: Method of Joints and Zero-Force Members Example 7, page 6 of 7


15 Zero-force members
BE and BF have
been omitted.

16 At joint F, no external force


acts, three members meet, and
two of these members are
4 kN
collinear, so member EF is a
zero-force member.

17 Free-body diagram of joint F


y
FFI
x

I
F
FEF = 0
(zero-force member)
F

FCF = 2.309 kN (C)

18 Equilibrium equations for joint F


Fy = 0: FFI + 2.309 kN (C) = 0

B
Ay = 2 kN

Cy = 2 kN

Solving gives
FFI = 2.309 kN = 2.309 kN (C)

Ans.

Then by symmetry
FEI = FFI = 2.309 kN (C)

958

Ans.

6.2 Trusses: Method of Joints and Zero-Force Members Example 7, page 7 of 7


19 Answer diagram
All forces in kN
4

1
2.3

2.3
1
E

C)

2.3

1(

A 1.155 (T)
2

2.3

1(
C)

0
0

)
(C

(C
)

B 1.155 (T)
2

959

6.3 Trusses: Method of Sections

960

6.3 Trusses: Method of Sections Procedures and Strategies, page 1 of 2


Procedures and Strategies for Solving Problems
Involving Trusses: the Method of Sections

1. Pass a section through members where member forces


are to be calculated.

A
B

2. Draw the free-body diagram of the portion of the truss


on one side of the section (You can decide which side to
choose by noting which side will be easier to analyze, for
example, which side has fewer forces acting on it). On the
diagram, be sure to include the (unknown) forces from the
members through which the section passed.

C
2 kN

2 kN

Section

3. Write equations of equilibrium for the portion of the


truss that you have isolated in the free-body diagram.

4. If support reaction-forces appear in the equations of


equilibrium, then you must draw a free-body diagram of
the entire truss, write equilibrium equations and solve for
the support reactions. You can then use these values in the
equations you wrote in Step 3.

FEF
FEC

A
B
RAy

FBC

2 kN
Find from the free-body diagram
of the entire truss.

961

6.3 Trusses: Method of Sections Procedures and Strategies, page 2 of 2


Notes:

A
a) Look for special situations in which you can save
work. For example, if two of three unknown forces at
a section are parallel, then summing forces in a
direction perpendicular to the two parallel forces will
give an equation involving only one unknown the
non-parallel force. Similarly, if the lines of action of
two unknown forces at a section intersect, then
summing moments about the point of intersection
will eliminate these forces from the moment equation.

F
E

Section

b) The method of sections is ideally suited for


problems in which only a small number (one to three)
member forces are to be determined. The method can
be used, however, in combination with the method of
joints to analyze complicated trusses.

4 kN

FBC
FAE

FBE
ME = 0

c) Sections need not be straight lines. Sometimes using


curved sections is advantageous.

D
FDE
RDy

FBC is the only


unknown in the
equation.

Find from the free-body


diagram of the entire truss.

962

6.3 Trusses: Method of Sections Problem Statement for Example 1


2 kN
G

1. Determine the force in members AC, CD, and DF, and


state whether the force is tension or compression.

5m

4 kN
E

5m

6 kN
C

5m

3m

963

6.3 Trusses: Method of Sections Problem Statement for Example 2


2. Determine the force in members CD, CH, and GH, and state whether the force is tension or compression.
G

8 ft
A

E
800 lb

12 ft

800 lb

12 ft

800 lb

12 ft

964

12 ft

6.3 Trusses: Method of Sections Problem Statement for Example 3


3. The diagonal members are not connected to each other where they cross. Determine the force
in members BG, CF, and FG, and state whether the force is tension or compression.
4m

4m

4m

3 kN

3 kN

3 kN

2.5 m
D

965

6.3 Trusses: Method of Sections Problem Statement for Example 4


4. Determine the force in members CE, EF, HF, and CF, and state whether the force is tension or compression.
1.5 m
3m
6 kN
I

3m
4 kN
G

3m

E
3m

D
C
3m
A

B
6m

966

6.3 Trusses: Method of Sections Problem Statement for Example 5


5. Determine the force in members RS, LS, FL, and EF, and state whether the force is tension or compression.
N

2m
I

2m
G

A
C

4 kN
3m

3m

3m

4 kN
3m

F
4 kN
3m

967

3m

6.3 Trusses: Method of Sections Problem Statement for Example 6


6. Determine the force in members TU, EF, and EU. State whether the force is tension or compression.
T

5m
N

5m
A

M
B

12 panels @ 4 m each

H
10 kN

968

6.3 Trusses: Method of Sections Problem Statement for Example 7


7. Determine the force in
members KM, LM, and
DK. State whether the
force is tension or
compression.

2 kN

2 kN

2 kN

2 kN

J K

I
2 kN
6m

2 kN

2 kN

A
2m

2m

B
2m

2m

2m

2m

E
2m

2m

969

2m

2m

6.3 Trusses: Method of Sections Problem Statement for Example 8


8. Determine the force in members GH,
CD, and CH. State whether the force is
tension or compression. Also, find the
tension in the cable.
Cable

30

3 ft
A
C

B
5 ft

5 ft

5 ft

5 ft

5 ft

5 kip

970

6.3 Trusses: Method of Sections Example 1, page 1 of 2


2 kN
G

1. Determine the force in members AC, CD, and DF, and


state whether the force is tension or compression.

2 kN
G

5m

4 kN
E

4 kN
E

5m
1
6 kN
C

Pass a section through the three


members whose forces are to be
determined.

6 kN
C

5m

3m

971

6.3 Trusses: Method of Sections Example 1, page 2 of 2


2

Free-body diagram of portion of truss above the section


4

2 kN
G

Equations of equilibrium for the portion of the


truss (Note that moments are summed about
point D, even though point D is not part of the
free body):
+

Fx = 0: 2 kN + 4 kN + FCD = 0

5m

Fy = 0:

4 kN
E

FAC

FDF = 0

MD = 0: FAC(3 m)

(2 kN)(5 m + 5 m)
(4 kN)(5 m) = 0

At each cut through a


member, a force is shown
to represent the effect of
the portion of the member
on one side of the section
pulling on the portion on C
the other side. It is
FCD
convenient to always
assume the force to be
FAC
tension.

Solving simultaneously gives

FDF
5m

3m

972

FAC = 13.33 kN (T)

Ans.

FCD = 6.0 kN = 6.0 kN (C)

Ans.

FDF = 13.33 kN = 13.33 kN (C)

Ans.

We had assumed member CD to be in tension.


Calculations showed that FCD is negative, so our
assumption was wrong: CD must be in compression.
Similarly DF must be in compression.

6.3 Trusses: Method of Sections Example 2, page 1 of 3


2. Determine the force in members CD, CH, and GH, and state whether the force is tension or compression.
G

8 ft
A

E
D

B
800 lb

12 ft

800 lb

12 ft

800 lb

12 ft

12 ft
1

Pass a section through the three


members whose forces are to be
determined.

E
D

B
800 lb

800 lb

973

800 lb

6.3 Trusses: Method of Sections Example 2, page 2 of 3


Free-body diagram of portion of truss to right of section
FGH

Equations of equilibrium for the portion of the


truss:

4
8 ft
E

FCD
D

Fx = 0:

FGH

FCH

Fy = 0:

FCH cos

MH = 0:

FCH sin

FCD = 0

(1)

800 lb + Ey = 0

(2)

FCD (8 ft) + Ey (12 ft) = 0

(3)

800 lb
Ey
12 ft

Geometry

= tan-1 12 ft = 56.31
8 ft

3 At each cut through a member, a force is shown.

8 ft
C
D
12 ft
6

974

Three equations but four unknowns, so another


equation is needed.

6.3 Trusses: Method of Sections Example 2, page 3 of 3


Free-body diagram of entire truss.

8 ft
E

Ax

B
800 lb

D
800 lb

800 lb

Ey

Ay
12 ft

12 ft

12 ft

Equilibrium equation for entire truss. This will give the needed fourth equation.

12 ft

MA = 0:

(800 lb)(12 ft)

(800 lb)(2

12 ft)

(800 lb)(3

12 ft) + Ey(4

12 ft) = 0

Solving gives Ey = 1,200 lb.


9

Substituting Ey = 1,200 lb into Eqs. 1, 2, and 3 and solving simultaneously gives


FCD = 1,800 lb (T)

Ans.

FCH = 721 lb (T)

Ans.

FGH = 2,400 lb = 2,400 lb (C)

Ans.

975

6.3 Trusses: Method of Sections Example 3, page 1 of 3


3. The diagonal members are not connected to each other where they cross. Determine the force
in members BG, CF, and FG, and state whether the force is tension or compression.
4m

4m

4m

3 kN

3 kN

3 kN

2.5 m
D

1
E

3 kN

3 kN

3 kN

Pass a section through the three members


whose forces are to be determined.
H

976

6.3 Trusses: Method of Sections Example 3, page 2 of 3


Free-body diagram of portion of truss to right of section

4m
FFG

FBG
2.5 m

FCF

Geometry
= tan-1

D
C

4m
= 58.0
2.5 m

4m
3 kN

Dy

At each cut through a member, a force is shown

2.5 m
Equations of equilibrium for the portion of the truss:

FFG

Fy = 0:

FBG cos

Fx = 0:

MG = 0:

FBG sin

FCF sin

FCF sin

+ FCF cos

+ Dy

=0

(1)

3 kN = 0

(2)

(2.5 m) + Dy(4 m) = 0

(3)

977

6 Three equations but four unknowns, so


another equation is needed.

6.3 Trusses: Method of Sections Example 3, page 3 of 3


7

Free-body diagram of entire truss (This will give the needed fourth equation).
4m

4m

4m

Ey
Ex

3 kN

3 kN

3 kN

2.5 m
D

Equilibrium equation for entire truss.

Dy

ME = 0:

(3 kN)(4 m)

(3 kN)(2

4 m) + Dy(3

Solving gives Dy = 3.0 kN. Then substituting


simultaneously gives

4 m) = 0

= 58.0 and Dy = 3.0 kN into Eqs. 1, 2, and 3 and solving

FBG = 5.66 kN (T)

Ans.

FCF = 5.66 kN (T)

Ans.

FFG = 9.6 kN = 9.6 kN (C)

Ans.

978

6.3 Trusses: Method of Sections Example 4, page 1 of 4


4. Determine the force in members CE, EF, HF, and CF, and state whether the force is tension or compression.
1.5 m
3m
6 kN
6 kN
I

3m
4 kN

4 kN
G

3m

3m
D

D
C

C
3m
A

B
6m

6m

979

Pass a section through at


least some of the members
whose forces are to be
determined. The general
idea is to choose as few
members as possible --three
in this instance-- because
each time a member is cut
by a section, an additional
unknown is introduced into
the equilibrium equations.

6.3 Trusses: Method of Sections Example 4, page 2 of 4


2

Free-body diagram of portion of truss above section (Using


the upper portion of the truss rather than the lower
eliminates the need to calculate the reactions at the bottom
of the truss).
LGH
6 kN

J
I
3m

4 kN
G

3m

FFH
FEF
F

E
LEF

Equations of equilibrium for the portion of the truss:


MG = 0:

(6 kN)(3 m) + FEF(3 m)

FCE

MF = 0:

(6 kN)(2

Fx = 0:

FCE sin

3 m)

(LGH) = 0

4 kN)(3 m) + FCE cos

+ FEF + FFH sin

980

FFH cos

(LEF) = 0

+ 4 kN + 6 kN = 0

(1)
(2)
(3)

6.3 Trusses: Method of Sections Example 4, page 3 of 4


Geometry

5
3m

3m
G

(6 m) tan
3m

3m
3m

+ (3 m) tan

= 3.75 m

LEF = 3 m + (6 m) tan

+ (6 m) tan

= 4.50 m

Substituting these values for , LGH, and LEF into Eqs. 1, 2, and
3 and solving simultaneously gives:
FCE = 10.75 kN

Ans.

FEF =

Ans.

3m

3m
3m

C
3m

3m

3m

kN = 7.33 kN (C)

FFH = 10.75 kN = 10.75 kN (C)

A 1.5 m

LGH = 3 m + (3 m) tan

3m

3m

(3 m) tan

1.5 m
= tan-1 BB' = tan-1
= 7.125
JB'
4 3m

B' 1.5 m B

981

Ans.

6.3 Trusses: Method of Sections Example 4, page 4 of 4


Free-body diagram of joint F.
This free body will enable us to
calculate the remaining unknown
force the force in member CF.

Equilibrium equations for joint F


+

Fx = 0:

FCF cos

Fy = 0:

FCF sin

+ FDF sin 7.125 + (10.75 kN)(sin 7.125)


FDF cos 7.125

7.33 kN = 0

(10.75 kN)(cos 7.125) = 0

FFH = 10.75 kN (C)


9

Geometry

FEF = 7.33 kN (C)


F
FCF

LEF = 4.50 m

(3 m) tan

E
FDF
3m

C
= 7.125

= tan-1

10 Substituting

3m
= 31.61
(3 m) tan 7.125 + 4.50 m

= 31.608 into Eqs. 4 and 5 and solving simultaneously gives:

FDF = 14.97 kN = 14.97 kN (C)


FCF = 7.99 kN (T)

982

Ans.

(4)
(5)

6.3 Trusses: Method of Sections Example 5, page 1 of 4


5. Determine the force in members RS, LS, FL, and EF, and state whether the force is tension or compression.
N

2m
I

2m
G

A
C

4 kN
3m

3m

4 kN

4 kN

3m

3m

3m

3m

1
N

G
B

D
4 kN

E
4 kN

4 kN

983

Pass a section through the four


members whose forces are to be
determined. It does not appear
possible to find a section that cuts
only three of these members.

6.3 Trusses: Method of Sections Example 5, page 2 of 4


Free body diagram of truss portion to right of section line
FRS

2m

FLS

FFL

2m
G

FEF

F
Gy
3m

Equations of equilibrium for the portion of the truss:

MS = 0:

MF = 0: FRS(2

FEF(2

Fy = 0: FFL

2 m) + Gy(3 m) = 0
2 m) + G y(3 m) = 0

FLS + Gy = 0

(1)
(2)
(3)

4 Three equations with five unknowns so two more


equations are needed.

984

6.3 Trusses: Method of Sections Example 5, page 3 of 4


Free-body diagram of entire truss (This free body will enable us to calculate the reaction at G).

T
2m

M
2m

Ax

C
4 kN

Ay
3m

3m

4 kN

4 kN

3m

3m

3m

Gy
3m

Equation of equilibrium for the entire truss.

MA = 0:

(4 kN)(2

3 m)

(4 kN)(3

3 m)

(4 kN)(4

3 m) + Gy(18 m) = 0

Solving gives
Gy = 6 kN
Substituting Gy = 6 kN into Eqs. 1 and 2 and solving gives:
FEF = 4.5 kN (T)

Ans.

FRS = 4.5 kN = 4.5 kN (C)

Ans.

985

(4)

6.3 Trusses: Method of Sections Example 5, page 4 of 4


7

Free-body diagram of joint S. This free body will


enable us to calculate the force in member LS.
FRS = 4.5 kN (C)

10 Free body diagram of joint T


FST

FST

T
FMT

FMS
FLS
8

Equations of equilibrium for joint S. Note that there


are three unknowns but only two equations.

Fx = 0: 4.5 kN + FST + FMS cos

Fy = 0:

FLS

FMS sin

=0

=0

11 Two members meet at joint T, they are not collinear and no


external force acts at joint T, so members ST and MT are zeroforce members.

(5)
(6)

Substituting FST = 0 in Eq. 5 and solving Eqs. 5 and 6


simultaneously gives:
FMS = 5.41 kN = 5.41 kN (C)

Geometry
S

FLS = 3.0 kN (T)

Ans.

T
2m
M

3m
-1 2 m
= tan
= 33.69
3m

12 Substituting FLS = 3.0 kN and Gy = 6 kN into Eq. 3 and


solving gives:
FFL = 3.0 kN = 3.0 kN (C)

986

Ans.

6.3 Trusses: Method of Sections Example 6, page 1 of 4


6. Determine the force in members TU, EF, and EU. State whether the force is tension or compression.
T

5m
N

5m
A

M
C

10 kN

12 panels @ 4 m each

M
B

1 Even though we were not asked to determine the force in member EP,
we have to pass the section through it because we must make the
section go completely through the truss.

987

H
10 kN

6.3 Trusses: Method of Sections Example 6, page 2 of 4


Free-body diagram of portion of the truss to the left of the section
FTU
T
5m

3
O

FEU

Fx = 0: Ax + FTU + FEP cos

+ FEF = 0

(1)

Equations of equilibrium for the portion of the truss:

Fy = 0: Ay + FEU + FEP sin

=0

(2)

FEP
5m
Ax

FEF
C

Ay
4m

4m

4m

ME = 0:

Ay (4

4 m)

FTU(2

5 m) = 0

Three equations with six unknowns so three more


equations are needed.

Geometry
P

4m

5m
E
F
4m

988

(3)

= tan-1 5 m = 51.34
4m

6.3 Trusses: Method of Sections Example 6, page 3 of 4


Free-body diagram of entire truss (This free body will enable us to calculate the reactions at support A).

Ax

M
B

Ay

H
10 kN

L
My

12 panels @ 4 m each
Equations of equilibrium for entire truss. Note that we
only write two equations because we only need to
calculate Ax and Ay, since only Ax and Ay appear in
Eqs. 1, 2, and 3.

Fx = 0: Ax = 0
MM = 0: (10 kN)(6

(4)
4 m)

Ay(12

Consideration of joint F shows that member FP is a zero-force


member, so FFP = 0.
But if member FP is removed (because it is a zero-force
member), consideration of joint P shows that member EP is also
a zero-force member, so FEP = 0.

4 m) = 0 (5)

Solving gives Ax = 0 and Ay = 5 kN.

989

6.3 Trusses: Method of Sections Example 6, page 4 of 4


9

Substituting
= 51.34,
Ax = 0,
Ay = 5 kN,
and
FEP = 0
into Eqs. 1, 2, and 3, and solving gives:
FTU = 8 kN = 8 kN (C)

Ans.

FEF = 8 kN (T)

Ans.

FEU = 5 kN = 5 kN (C)

Ans.

990

6.3 Trusses: Method of Sections Example 7, page 1 of 6


7. Determine the force in
members KM, LM, and
DK. State whether the
force is tension or
compression.

2 kN

2 kN

2 kN

2 kN

J K

I
2 kN
6m

2 kN

2 kN

A
2m

2m

B
2m

2m

2m

2m

2m

2m

2 kN

2 kN

2m

2m

We choose a section that cuts at least some of the


members whose forces are to be determined. But the
section should cut as few other members as possible,
2 kN since each time a member is cut, an additional
unknown appears in the equilibrium equations.
1

2 kN

I
L

J K

2 kN

2 kN

2 kN

A
B

991

6.3 Trusses: Method of Sections Example 7, page 2 of 6


2

Free-body diagram of portion of truss to right of section.


It is not essential but we can save some work if we use
the principle of transmissibility as shown in Step 3.

Same free body as in Step 2, but now the force FKM has been
moved along its line of action to joint D (principle of
transmissibility) and then expressed in terms of vertical and
horizontal components. Similarly FML is moved to joint F.

2 kN

FKM

2 kN

FML
L

2 kN

K
FKM sin

2 kN
H
FCD
D

H
FCD

FKM cos

FML cos
Fy
2m

+
+

Equations of equilibrium for free body in Step 3. Note that


because we were not asked to determine FCD, we choose two
moment equations in which FCD does not appear.
MF = 0: (2 kN)(2 m) + (2 kN)(3
MD = 0:

(2 kN)(2 m)

(2 kN)(3

2 m)
2 m)

FKM sin
(2kN)(4

2 kN
F

Fy
4

FML sin

2 kN

(4

2m

2 m) = 0

2 m) + Fy(4

5 Two equations but three unknown forces, so another


equilibrium equation is needed.

992

2m

2m

(1)
2 m) + FML sin

(4

2 m) = 0

(2)

6.3 Trusses: Method of Sections Example 7, page 3 of 6


6

Geometry
M

6m

D
8m
2m
6m
= tan-1 ( 2 m ) = 71.56
6m
= tan-1 ( 2 m + 8 m ) = 30.96

993

6.3 Trusses: Method of Sections Example 7, page 4 of 6


7

2 kN

Free-body diagram of entire


truss. This free body will
enable us to calculate the
reaction at support F.

2 kN

2 kN

I
I

2 kN

J K

2 kN

2 kN

2 kN

G
Ax

A
D

Fy

Ay
2m

2m

2m

2m

2m

2m

2m

2m

2m

Equilibrium equation for entire truss

2m

MA = 0:

(2 kN)(2 m) (2 kN)(3 2 m) (2 kN)(5 2 m)


(2 kN)(7 2 m) (2 kN)(9 2 m) (2 kN)(10 2 m) + Fy(10

9 Solving gives Fy = 7 kN. Substituting Fy = 7 kN, = 71.56, and


= 30.96 in Eqs. 1 and 2, and solving simultaneously gives:
FKM = 2.11 kN (T)

Ans.

FML = 5.83 kN = 5.83 kN (C)

Ans.

994

2 m) = 0

(3)

6.3 Trusses: Method of Sections Example 7, page 5 of 6


11 Since there are only two unknown forces, FKL and FDK ,
we could write force-equilibrium equations in the x
and y directions and then solve them simultaneously.
However, we can save work by noticing that a
zero-force member is present.

10 Free-body diagram of joint K (This free body will enable


us to calculate the force in member DK).
y
FKM = 2.11 kN (T)
x
K

FKL

FDK

12 Free- body diagram of entire truss

2 kN

2 kN

2 kN

13 Consideration of joint K shows that KL must


be a zero-force member, so FKL = 0.

2 kN

I
L

J K

2 kN

2 kN

2 kN

A
B

Ax

Ay

Fy

995

6.3 Trusses: Method of Sections Example 7, page 6 of 6


14 Free-body diagram of joint K (repeated)
y
FKM = 2.11 kN (T)
x
K

FKL

FDK

Fy = 2.11 kN

FDK = 0

(4)

Solving gives
FDK = 2.11 kN (T)

Ans.

996

6.3 Trusses: Method of Sections Example 8, page 1 of 3


8. Determine the force in members GH,
CD, and CH. State whether the force is
tension or compression. Also, find the
tension in the cable.
Cable

30

3 ft
A
5 ft

Cable

B
5 ft

5 ft

5 ft

5 ft

5 kip
1 The section must pass through the
cable. Otherwise the portion of the
truss to the left of the section could
not be isolated as a free body.

30

A
B
5 kip

997

6.3 Trusses: Method of Sections Example 8, page 2 of 3


2 Free-body diagram of portion of truss to left of section
T
F

3 The tension in the cable is


one of the unknowns.

30

FGH
FCH

3 ft
A
5 ft

FCD

B
5 ft

Equations of equilibrium for the portion of the truss:

MC = 0:

MG = 0: (5 kip)(2

5 kip

Fy = 0:

Geometry
5 ft
G
3 ft

T cos 30(3 ft)

FGH(3 ft) + (5 kip)(2

5 ft) + FCD(3 ft) + FCH sin

5 kip + T sin 30 + FCH cos

H
5 ft
= tan-1( 3 ft ) = 59.04

998

5 ft) = 0
(3 ft) = 0

(1)
(2)
(3)

6 Three equations but four


unknown forces, so another
equilibrium equation is needed.

6.3 Trusses: Method of Sections Example 8, page 3 of 3


Free-body diagram of entire truss

T
30

Jx

3 ft
A
C

B
5 ft

5 ft

5 ft

5 ft

5 ft

5 kip

Equation of equilibrium for the entire truss. Only one equation is


used because we need to calculate T only; the reactions at J are not
needed.

MJ = 0: (5 kip)(5

5 ft)

T sin 30(3

T = 16.67 kip

5 ft) = 0

Jy

(4)
Ans.

9
Substituting = 59.04 and T = 16.67 kip into Eqs. 1, 2, and 3 and
solving simultaneously gives:
FGH = 2.23 kip (T)

Ans.

FCD = 11.11 kip = 11.11 kip (C)

Ans.

FCH = 6.48 kip = 6.48 kip (C)

Ans.

999

6.4 Space Trusses

1000

6.4 Space Trusses Procedures and Strategies, page 1 of 1


Procedures and Strategies for Solving Problems
Involving Space Trusses
Space truss problems can be solved by the same
methods the method of joints and the method of
sections as are used in solving planar trusses.
The only difference are 1) There are three rather
than two equilibrium equations for each joint; and
2) in planar truss problems you can sometimes save
work by looking for a point through which the
lines of action of several unknown force pass and
then summing moments about this point because
the unknown forces will not appear in the moment
equation. However, in space truss problem, you
look for a line through which the lines of action of
several unknown forces pass, and then you sum
moments about the line, rather than about a point.

1001

6.4 Space Trusses Problem Statement for Example 1


1. The truss is supported by short links at B and D and by a
ball and socket at C. Determine the force in each member,
and state whether the force is tension or compression.
y
4m

4m

C
2m

5m
B
x

7m

z
200 N

1002

6.4 Space Trusses Problem Statement for Example 2


2. Determine the force in each member of the space truss, and
state whether the force is tension or compression. The truss is
supported by short links at C and D and by ball-and-socket
supports at A and E.
y

4m
B
A

4m

3m

3m
E
D
700 N
z

5m

1003

6.4 Space Trusses Problem Statement for Example 3


3. The truss is supported by ball-and-socket
joints at A, C, D, and F. Determine the force
in each member and state whether the force
is tension or compression.
y
10 ft

8 ft

C
x

6 ft
B
5 kip
2 kip
z

4 kip

1004

6.4 Space Trusses Problem Statement for Example 4


4. Determine the force in each member of the space
truss. State whether the force is tension or
compression. The supports at C and D are short
links; at A a ball-and-socket support is present.
y
400 N

2m

800 N
B

5m

A
4m
D
8m
z

1005

6.4 Space Trusses Problem Statement for Example 5


5. The truss is supported by ball-and-socket
joints at A, B, C, and D. Determine the forces
in members FE and EC. State whether the
forces are tension or compression.

y
800 N

800 N
800 N

800 N
G

H
E

8m
B

x
4m

2m
4m
A

3m

5m

3m

1006

6.4 Space Trusses Example 1, page 1 of 6


1. The truss is supported by short links at B and D and by a
ball and socket at C. Determine the force in each member,
and state whether the force is tension or compression.
y
4m

4m

C
2m

5m
B
x

7m

z
200 N

1007

6.4 Space Trusses Example 1, page 2 of 6


1

At joint A, only three unknown forces are


present, so begin the analysis there.
4m

4m

Free-body diagram of joint A.

FAD

FAB
FAC

C
2m

5m
B
x

200 N
Equilibrium equation

FAD + FAC + FAB

{200j}N= 0

(1)
A
z
200 N

1008

7m

6.4 Space Trusses Example 1, page 3 of 6


y
3

Express the forces in component form.


4m

FAD = FAD
= FAD

unit vector pointing from A to D


4i + 5j

7k
D

42 + 52 + 72

= FAD( 0.4216i + 0.5270j

0.7379k)

C
2m

(2)

FAC = FAC(0.4216i + 0.5270j


3j

0.7379k)

(3)

7k
A

32 + 72
z

= FAB(0.3939j

0.9191k)

5m
B
x

FAC and FAD differ only in sign of x component

FAB = FAB

4m

(4)
200 N

1009

7m

6.4 Space Trusses Example 1, page 4 of 6


4

Substitute the component forms of the force vectors into Eq.1:

FAD + FAC + FAB

200j = 0

0.4216FADi + 0.5270FADj

(Eq. 1 repeated)
0.7379FADk

+ 0.4216FACi + 0.5270FACj

0.7379FACk)

+ 0.3939FABj

200j = 0

0.9191FABk

Or, in scalar form,


Fx = 0:

0.4216FAD + 0.4216FAC = 0

(5)

Fy = 0: 0.5270FAD + 0.5270FAC + 0.3939 FAB 200 = 0

(6)

Fz = 0:

(7)

0.7379FAD

0.7379FAC 0.9191FAB = 0

Solving gives
FAD = 474.3616 N (T)

Ans.

(8)

FAC = 474.3616 N (T)

Ans.

(9)

FAB = 761.5773 N = 761.5773 N (C)

Ans.

(10)

1010

6.4 Space Trusses Example 1, page 5 of 6


y
4m

5 Free-body diagram of joint B. Now that the force


in member AB is known, the member forces FBD
and FBC can be found.

4m

Bzk (Reaction force)


D

C
2m

FBC = FBC

FBD

5m

42 + 22
= FBC (0.8944i + 0.4472j)

4i + 2j

FBD = FBD

FBA = FAB (Force of member AB on joint B is equal


and opposite to force on joint A.)

4i + 2j
42 + 22

= FAB(0.9191j
= FBD( 0.8944i + 0.4472j)

7m

761.5773 N, by Eq. 10
= (300j

0.3939k), by Eq. 4

700k) N

200 N

7 Solving gives

Summing the forces in the x and y directions gives

Fx = 0: FBC(0.8944)

FBD(0.8944) = 0

Fy = 0: FBC(0.4472) + FBD(0.4472) + 300 = 0

1011

FBC = 335.4102 N = 335.4102 N (C)

Ans.

FBD = 335.4102 N = 335.4102 N (C)

Ans.

(11)

6.4 Space Trusses Example 1, page 6 of 6


y
4m

8 Free-body diagram of joint D. Now that the force


in member BD is known, the remaining unknown
member force FDC can be found.

4m

Reaction forces

Dyj
Dzk

C
2m

5m

FDC = FDCi
FDB = FBD

= ( 335.4102)( 0.8944i + 0.4472j)


= 300i +

FDA = FAD
= FAD( 0.4216i + 0.5270j
A
z

by Eq. 11

0.7379k), by Eq. 2

474.3616 N, by Eq. 8

7m
= (200i

250j + 350k) N

200 N

9 Summing the x components of the forces acting on


joint D gives
+

Fx = 0: 200

300 + FDC = 0

Solving gives
FDC = 100 N (T)

1012

Ans.

6.4 Space Trusses Example 2, page 1 of 4


2. Determine the force in each member of the space truss, and
state whether the force is tension or compression. The truss is
supported by short links at C and D and by ball-and-socket
supports at A and E.
y

Only three unknowns two member forces and a


reaction force are present at D, so a free-body
diagram of D is a good place to start.

4m
Free-body diagram of joint D

B
A

4m

FDB

Dzk
(Reaction force
from link at D)

3m

FDE

3m

E
700 N

D
700 N
z

5m

Equilibrium equation

FDB + FDE {700j}N+ Dzk = 0

1013

(1)

6.4 Space Trusses Example 2, page 2 of 4


2 Express the forces FDB and FDEin component form.

FDB = FDB

unit vector pointing from D to B


4m
5i + 4j

= FDB

3k
B

52 + 42 + 32

= FDB( 0.7071i + 0.5657j

0.4243k)

FDE = FDEi

(2)

4m

3m

(3)
3m

Substitute the component form of the force vectors in Eqs. 2


and 3 into Eq. 1:

E
D

FDB + FDE 700j + Dzk = 0

(Eq. 1 repeated)

700 N
z

or
0.7071FDBi + 0.5657 FDBj 0.4243FDBk
FDEi 700j + Dzk = 0

In scalar form,
Fx = 0:

Solving gives
FDB = 1237.4369 N (T)

0.7071FDB

Fy = 0: 0.5657FDB
Fz = 0:

5m

FDE = 0

Ans.

(4)

Ans.

(5)

FDE = 875 N

700 = 0

= 875 N (C)

0.4243FDB + Dz = 0

Dz = 525 N

1014

6.4 Space Trusses Example 2, page 3 of 4


y

Now that the force in member DB is known,


there are only three unknowns at joint B.

4m
Free-body diagram of joint B
B

FBA

4m

FBC

3m

FBD

FBE

3m

Equilibrium equation

FBE + FBC + FBA + FBD = 0


5

Express each force in component form

FBE = FBE

700 N
z

5m

4 j + 3k
42 + 32

= FBE ( 0.8j + 0.6k)

FBC = FBC

(6)

4j

3k

(7)

FBA = FBA i

(9)

FBD = FDB

1237.4369 N, by Eq. 4
= FDB( 0.7071i + 0.5657j 0.4243k), by Eq. 2

4 +3

= FBC ( 0.8j

Force of member
BD on joint B is
equal and opposite
force of member
BD on joint D.

0.6k)

= {875i

(8)

1015

700j + 525k} N

(10)

6.4 Space Trusses Example 2, page 4 of 4


6

Substitute the component form of the force vectors in Eq. 7-10


into Eq. 6:

FBE + FBC + FBA + FBD = 0

Free-body diagram of joint C.


y
B

(Eq. 6 repeated)

or,
0.8FBEj + 0.6FBEk 0.8FBCj 0.6FBCk
FBAi 875i 700j + 525k = 0

4m
FBC = 0

In scalar form,
Fx = 0:

FBA + 875 = 0

Fy = 0:

0.8FBE

x, C

z
FEC

Cyj

0.8FBC 700 = 0

Fz = 0: 0.6FBE 0.6FBC

3m

525 = 0
Fz = 0: FEC + 0 = 0

Solving gives
FBE = 875 N = 875 N (C)

Ans.

FBA = 875 N (T)

Ans.

FBC = 0

Ans.

Thus
FEC = 0

1016

Ans.

6.4 Space Trusses Example 3, page 1 of 6


3. The truss is supported by ball-and-socket
joints at A, C, D, and F. Determine the force
in each member and state whether the force
is tension or compression.
y
10 ft

8 ft

C
x

6 ft
B
5 kip
2 kip
z

4 kip

1017

6.4 Space Trusses Example 3, page 2 of 6


1

At joint B, only three unknown forces are


present, so begin the analysis there.

10 ft
2

Free-body diagram of joint B

FBA

FBE
8 ft

FBC
B

5 kip

2 kip

4 kip

A
3

Equilibrium equation:

FBA + FBE + FBC + {5i} kip {4j} kip


+ 2k} kip = 0

(1)
6 ft

4 Express the forces in component form


B

FBA = FBA j

(2)

FBE = FBE k

(3)

FBC = FBC

10i + 6j
2

5 kip
z

2 kip 4 kip

(4)

10 + 6

= FBC(0.8575i + 0.5145j)

(5)

1018

6.4 Space Trusses Example 3, page 3 of 6


5

Substitute the component form of the force vectors in Eqs.


2-5 into Eq. 1:

FBA + FBE + FBC + 5i


+ 2k = 0
FBAj

4j
(Eq. 1 repeated)

FBEk + 0.8575FBCi + 0.5145FBCj


+ 5i 4j + 2k = 0

In scalar form,
Fx = 0: 0.8575FBC + 5 = 0
Fy = 0: FBA + 0.5145FBC
Fz = 0:

(6)
4=0

(7)

FBE + 2 = 0

(8)

Solving gives
FBA = 7 kips (T)

Ans.

(9)

Ans.

(10)

Ans.

(11)

FBC = 5.8310 kip (T)


= 5.8310 kip (C)
FBE = 2 kip (T)

1019

6.4 Space Trusses Example 3, page 4 of 6


y

6 Free-body diagram of joint E. Now that the force


in member BE is known, the forces in members
ED, EC, and EF can be found.

10 ft

FED
8 ft

FEF
E

FEC
C

FEB

Equilibrium equation

FEB + FED + FEF + FEC = 0

(12)
6 ft
B
5 kip
2 kip
4 kip

1020

6.4 Space Trusses Example 3, page 5 of 6


y

7 Express each force in component form.

10 ft

FEB = force in direction opposite to FBE


= FBE
by Eqs. 3 and 11
= { 2}k kip

FED = FEDj

8 ft

(13)
(14)
C

FEF = FEF

10i + 6j

102 + 62

= FEF(0.8575i + 0.5145j)

(15)
6 ft

FEC = FEC

10i + 6j + 8k

102 + 62 + 82

5 kip
2 kip

= FEC(0.7071i + 0.4243j
+ 0.5657k)

(16)

4 kip

1021

6.4 Space Trusses Example 3, page 6 of 6


8

Substitute the component form of the force vector in Eqs. 13-16 into Eq. 12:

FEB + FED + FEF + FEC = 0

(Eq. 12 repeated)

2k + FEDj + 0.8575FEFi + 0.5145FEFj + 0.7071FECi


+ 0.4243FECj + 0.5657FECk = 0
In scalar form,
Fx = 0: 0.8575FEF + 0.7071FEC = 0
Fy = 0: FED + 0.5145FEF + 0.4243FEC = 0
Fz = 0: 2 + 0.5657FEC = 0
Solving gives
FEC = 3.54 kip = 3.54 kip (C)

Ans.

FED = 0

Ans.

FEF = 2.92 kip (T)

Ans.

1022

6.4 Space Trusses Example 4, page 1 of 7


4. Determine the force in each member of the space
truss. State whether the force is tension or
compression. The supports at C and D are short
links; at A a ball-and-socket support is present.
y
400 N

2m

800 N
B

5m

A
4m
D
8m
z

1023

6.4 Space Trusses Example 4, page 2 of 7


1 Joint B is the only joint with no more
than three unknown forces, so begin
with a free-body diagram of joint B.

y
400 N

2m

800 N
B

2 Free-body diagram of joint B


800 N

5m

B
C

400 N

FBC
FBA

FBD

4m
D

Equilibrium equation
8m

FBA + FBD + FBC

{800j}N + (400k}N= 0

(1)

1024

6.4 Space Trusses Example 4, page 3 of 7


3

Express the forces in component form.

FBA = FBA
= FBA

2m

800 N
B

2i

5j

22 + 52

6i

0.9285j)

5m

(2)

5j

62 + 52

= FBC(0.7682i

FBD = FBD

400 N

unit vector pointing from B to A

= FBA( 0.3714i

FBC = FBC

6i

A
0.6402j)

(3)

5j + 4k

62 + 52 + 42

= FBD(0.6838i

0.5698j + 0.4558k)

4m
D

z
(4)

1025

8m

6.4 Space Trusses Example 4, page 4 of 7


4

Substituting the component form of the forces in Eqs. 2, 3, and 4 into


Eq. 1 gives

FBA + FBD + FBC

800j + 400k = 0

(Eq. 1 repeated)

or, in scalar form,


Fx = 0:

0.3714FBA + 0.7682FBC + 0.6838FBD = 0

(5)

Fy = 0:

0.9285FBA

(6)

0.6402FBC

0.5698FBD

800 = 0

Fz = 0: 0.4558 FBD + 400 = 0

(7)

Solving Eqs. 5, 6, and 7 simultaneously gives


FBA = 646.2198 N = 646.2198 N (C)

Ans.

(8).

FBC = 468.6150 N (T)

Ans.

(9)

FBD = 877.4964 N = 877.4964 N (C)

Ans.

(10)

1026

6.4 Space Trusses Example 4, page 5 of 7


6 Now that the force in member BC is known, there are
only three unknowns at joint C.

y
400 N

2m

Free-body diagram of joint C

800 N
B

FCB = FBC
= (468.6150 N)(0.7682i

FCAi

0.6402j), by Eq. 3

5m

= 360i + 300j
FCDk

Cyj (Reaction from link C)


A

Equilibrium equations:
Fx = 0:

FCA

4m
D

360 = 0
8m

Fy = 0: Cy + 300 = 0
z

Fz = 0: FCD = 0
7 Solving simultaneously gives
FCA = 360 N
= 360 N (C)
FCD = 0
Cy = 300 N

Ans.

(11)

Ans.

(12)
(13)

1027

6.4 Space Trusses Example 4, page 6 of 7


8

Now that the forces in members CD and BD are known,


a free-body diagram of joint D will give an equation for
the remaining unknown member force, FDA.

y
400 N

2m

800 N
B

Free-body diagram of joint D

FDB

FCD = 0

FDA

5m

Dxi

C
Dyj

Reactions from links


A

4m
D

10 Equilibrium equation:

FDA + FDB + Dxi + Dyj = 0

(14)

8m
z

11 Express the forces in component form.

FDB = FBD
= FBD(0.6838i

0.5698j + 0.4558k)

(Eq. 4 repeated)

877.4964 N, by Eq. 10
= {600i

500j + 400k} N

12 FDA = FDA

8i

4k

82 + 42

= FDA( 0.8944i

(15)

1028

0.4472k )

(16)

6.4 Space Trusses Example 4, page 7 of 7


13 Substitute the component form of the force vectors in Eqs. 15 and
16 into Eq. 14:

FDA + FDB + Dxi + Dyj = 0

(Eq. 14 repeated)

or
0.8944FDAi 0.4472FDAk + 600i
+ 400k + Dxi + Dyj = 0

500j

Then the summation of the z components gives


Fz = 0:

0.4472FDA + 400 = 0

Solving gives
FDA = 894 N (T)

Ans.

1029

6.4 Space Trusses Example 5, page 1 of 5


5. The truss is supported by ball-and-socket
joints at A, B, C, and D. Determine the forces
in members FE and EC. State whether the
forces are tension or compression.

y
800 N

800 N
800 N

800 N
G

H
E

8m
B

x
4m

2m
4m
A

3m

5m

3m

1030

6.4 Space Trusses Example 5, page 2 of 5


y
800 N

800 N
800 N
800 N
Section

H
E

8m
B

x
4m

2m
4m
A

3m
z

5m

3m

1 Because we have been asked to find only two


member forces, the method of sections will be
used. Pass a section through the truss that cuts
members DF, CF, FG, and FE.

1031

6.4 Space Trusses Example 5, page 3 of 5


2

Free-body diagram of portion of truss on


side of section that includes joint F.
y
800 N

FFG (parallel to the z axis)

FFE

FBD
3
+

8m

FEF can be computed by summing moments about the z axis:


Mz = 0:

(800 N)(3 m)

FFE(8 m) = 0

Solving gives
FFE = 300 N = 300 N (C)

3m
D
z

1032

Ans.

6.4 Space Trusses Example 5, page 4 of 5


y
800 N

800 N
800 N
800 N

H
Section

8m
B

x
4m

2m
4m
A

3m
z

5m

3m

4 To calculate the force in member EC, pass a


section through the truss that cuts members EC,
EF, EA, and EH.

1033

6.4 Space Trusses Example 5, page 5 of 5


5

Free-body diagram of portion of truss on


side of section that includes joint E

FEC can be found by summing moments about line AB. To


sum moments, first we have to express FEC in component
form.
8i 8j 6k
FEC = FEC
82 + 82 + 62

800 N
FEC (Parallel to line AB)
E

FFE = 300 N (C)


y

Position
vector
from B
to E

FEC
FEA
8m

= FEC( 0.6247i

0.6247j

0.4685k)

The moment of FEC about the line AB is given by


MAB = u rBE FEC

B
x

6m
Unit vector
parallel to AB

= k { 3i + 8j + 6k} m
FEC( 0.6247i

0.6247j

0.4685k)

= 6.8716 FEC

8m
z

7
A
3m
+

Sum moments about AB, calling a moment positive if it


produces a counterclockwise moment about AB, as
viewed from A looking back at B.
MAB = (800 N)(3 m)

300 N)(8 m) + 6.8716 FEC = 0

Solving gives
FEC = 0

1034

Ans.

6.5 Cables: Concentrated Loads

1035

6.5 Cables: Concentrated Loads Procedures and Strategies, page 1 of 3


Procedures and Strategies for Solving Problems Involving
Cables With Concentrated Loads

3m

1. Pass sections through the cable at


points where something about the
cable geometry is known. For
example, consider the following
situations:
a) The location of a load point is
known. Pass a section through
the cable close to the point, draw
a free-body diagram of either the
portion of the cable to the left or
to the right of the section, and
write M = 0 about the point.
This gives an equation involving
the reaction components at one
of the supports. Then writing
three equilibrium equations for a
free body of the entire cable will
allow you to solve
simultaneously for all reaction
components

3m

3m

3m

A
4m

C
3m

Ey
Ex

1.6 kN

2 kN

3m

3 kN
Section

E
TBC

4m

MC = 0 Equation
involving Ex and Ey
only

C
2 kN
3 kN

3m

3m

3m

3m
Ay

Ey
Ex
Fx = 0
Fy = 0
MB = 0

Three
equations
involving Ex,
Ey, Ax, and Ay
.

1036

A
B

D
C

1.6 kN

2 kN
3 kN

Ax

6.5 Cables: Concentrated Loads Procedures and Strategies, page 2 of 3


b) The horizontal and vertical
distances between two adjacent
load points are known.
Consider a free body of the
cable segment between the load
points (Do not include the load
points in the free body.) and sum
moments about either end to
obtain an equation relating the x
and y components of the tension
in the cable segment. Next pass
a section through the segment,
and consider a free body of
either the part of the cable to the
left or right of the section. Write
Fx = 0 and Fy = 0. These
equations, together with three
equilibrium equations for a free
body of the entire cable, will
allow you to solve for the
support reactions.

3m

3m

3m

3m

A
D

1.5 m
C

1.6 kN

2 kN
3 kN
TBAy
TCBx 1.5 m
C

TCBy

Section

TBAx

3m

Point on BC just to left of B

Point on BC just to right of C


MB = 0 Equation involving
TCBx and TCBy only.

Ey
Ex

TCBy

D
2 kN

TBCx

C
3 kN

1037

Fx = 0
Fy = 0

Two equations
involving Ex, Ey,
TCBx, and TCBy.

6.5 Cables: Concentrated Loads Procedures and Strategies, page 3 of 3


2. Once the support-reaction components are
known, you can determine the elevation of any
load point by passing a section through the cable
near the load point, considering a free-body
diagram of the part of the cable on either side of
the section, and then writing M = 0 about the
section. The unknown elevation will occur as a
moment arm in this equation and can thus be
found.

Now known

Ey
Ex

yD
D
3m

MD = 0

1038

TDC
2 kN

Equation involving yD.

6.5 Cables: Concentrated Loads Problem Statement for Example 1


1. For the cable system shown, determine the
reactions at support A and the distance yC.

4m

3m

2m
A

D
2.5 m

yC

B
C

1.4 kN

2 kN

1039

6.5 Cables: Concentrated Loads Problem Statement for Example 2


2. The horizontal force P is applied to end A of the
cable as shown. Determine the value of P and the
distance d required to keep the cable system in the
configuration shown. Also determine the total
length of the cable.
d

4m

12 m
15 m

A
3.5 kN

2 kN

1040

6.5 Cables: Concentrated Loads Problem Statement for Example 3


3. The cable supports the 150 N and 500 N
loads shown. Determine the distance xC and
the tension in each segment of the cable.

4m
A
150 N

2m

6m

500 N

C
3m
D
xC

1041

6.5 Cables: Concentrated Loads Problem Statement for Example 4


4. For the cable system shown, determine the distance yC for
which segment BC will be horizontal. Also determine yD.
E

3m

A
yD

yC

1.2 kN
2.2 kN
0.25 m

2m

1.8 kN
2m
0.5 m

1042

6.5 Cables: Concentrated Loads Problem Statement for Example 5


5. For the cable system shown,
determine the value of the forces PB
and PD necessary to maintain the
given configuration.

PB

A
2m

5m
C
200 N
7m
D
2m

PD

4m

4m

2m

3m

1043

6.5 Cables: Concentrated Loads Problem Statement for Example 6


6. For the cable system shown, determine
distance yB and the tension in each segment.
1.5 ft
A
D
3.5 ft
yB
C
B

50 lb

80 lb

2 ft

3.5 ft

5 ft

1044

6.5 Cables: Concentrated Loads Problem Statement for Example 7


7. The cable supports the four forces shown.
Determine the maximum tension in the cable.
18 ft

18 ft

18 ft

18 ft

18 ft

15 ft
F

35 ft
B
C

E
D

2 kip
2 kip

2 kip

2 kip

1045

6.5 Cables: Concentrated Loads Example 1, page 1 of 3


1. For the cable system shown, determine the
reactions at support A and the distance yC.

4m

3m

2m
A

D
2.5 m

yC

Free-body diagram of AB
Ay

2m

C
Ax
1.4 kN

2 kN

2.5 m
B

Strategy: Note that both


horizontal and vertical
distances between A and B are
known. Thus summing
moments about B, for a
free-body AB, would give an
equation involving Ax and Ay
only.

TBC

1.4 kN
3 Equilibrium equation

1046

MB = 0: Ax(2.5 m)

Ay(2 m) = 0

(1)

6.5 Cables: Concentrated Loads Example 1, page 2 of 3


4

We can get another equation involving the reaction at


A as the only unknown by summing moments about
D for a free body of the entire cable.

Free-body diagram of entire cable


6 Equilibrium equation

Dy

Ay

Ax

4m

3m

2m
A

Dx

MD = 0: (1.4 kN)(3 m + 4 m)
+ (2 kN)(4 m)
y(2 m + 3 m + 4 m) = 0

Solving gives
y

= 1.978 kN

Ans.

Using this result in Eq. 1 gives


C
Ax(2.5)

1.4 kN

(2)

y(2)

=0

( Eq. 1 repeated)

1.978 kN
2 kN

Solving gives
Ax = 1.582 kN

1047

Ans.

6.5 Cables: Concentrated Loads Example 1, page 3 of 3


7 Now that the values of Ax and Ay are known,
summing moments about C, for a free body ABC,
will give an equation with a single unknown, yc.

8 Free-body diagram of ABC


Ay

1.978 kN

2m
1.582 kN

3m

A
yC
B

9
TCD

Equilibrium equation

Ax

MC = 0: (1.4 kN)(3 m)
(1.978 kN)(2 m + 3 m)
+ (1.582 kN) yc) = 0

Solving gives
1.4 kN

yc = 3.60 m

2 kN

1048

Ans.

6.5 Cables: Concentrated Loads Example 2, page 1 of 3


2. The horizontal force P is applied to end A of the
cable as shown. Determine the value of P and the
distance d required to keep the cable system in the
configuration shown. Also determine the total
length of the cable.
d

4m

12 m
15 m

1
B

A
3.5 kN

2 kN

1049

Strategy: Note that we know the


horizontal distance between points A
and B, and we can compute the vertical
distance between these points. Thus
summing moments about B for a free
body AB, will give an equation from
which P can be found.

6.5 Cables: Concentrated Loads Example 2, page 2 of 3


4

2 Free-body diagram of AB
TBC

Now that the value of P is known, we can sum


moments about C, for a free-body of the whole
cable, to obtain an equation for d.

B
15 m
P

12 m = 3 m

Free-body diagram of ABC


Cy

3.5 kN

4m

2 kN

Cx

4m

3 Equilibrium equation for AB


MB = 0: (2 kN)(4 m)

P(3 m) = 0
15 m

Solving gives
Ans.

P = 2.667 kN

6 Equilibrium equation for ABC

MC = 0:

(2.667 kN)(15 m) + (2 kN)(4 m + d)


+ (3.5 kN)(d) = 0

Solving gives
P = 2.667 kN

d = 5.819 m
3.5 kN

2 kN

1050

Ans.

6.5 Cables: Concentrated Loads Example 2, page 3 of 3


7 The total length of the cable can be
found by applying the Pythagorean
Theorem to segment AB and to BC

Geometry

LTotal = LAB + LBC


=

(4 m)2 + (3 m)2 + (5.819 m)2 + (12 m)2

12 m
= 5 m + 13.34 m
= 18.34 m
B
15 m

12 m

3m

A
4m

5.819 m

1051

Ans.

6.5 Cables: Concentrated Loads Example 3, page 1 of 5


3. The cable supports the 150 N and 500 N
loads shown. Determine the distance xC and
the tension in each segment of the cable.

Strategy: Note that both the horizontal


and vertical distances between A and B
are known. Thus summing moments
about B, for a free-body AB would give
an equation involving the reaction
components at A (Ax and Ay), only; no
other unknowns are present.

Free-body diagram of AB

4m
A
150 N

2m

4m

6m

Ay
C
Ax

3m
150 N

D
xC

2m

B
TBC
3

Equilibrium equation

500 N

1052

MB = 0: Ay (4 m)

(2 m) = 0

(1)

6.5 Cables: Concentrated Loads Example 3, page 2 of 5


4

Now summing moments about D, for a free body of the entire


cable, will give another equation for Ax and Ay.

Free-body diagram of entire cable


Ay
A
150 N

Equilibrium equation

Ax

MD = 0:

x (2 m + 6 m + 3 m)
(500 N)(3 m) = 0

(150 N)(6 m + 3 m)

Solving gives
2m

Ax = 259.09 N

(2)

Substituting this in Eq. 1 gives


6m

4 Ay 2 Ax= 0

(Eq. 1 repeated)

259.09 N
500 N

Solving gives

3m
D

Ay = 129.55 N
Dx

Dy

1053

(3)

6.5 Cables: Concentrated Loads Example 3, page 3 of 5


7

Now that Ax and Ay are known, we can calculate xC by


summing moments about C for a free-body ABC.

8 Free-body diagram of ABC

Ay
A

129.55 N

Ax

259.09 N
2m

150 N

10 We can get the tension in AB by considering a


free body of support A.
11 Free-body diagram of A for calculating tension TAB

Ay
TABx

6m

TAB
500 N

Ax
TABy

12 Summing horizontal and vertical forces gives

C
TCD

TABx = 259.09 N

(4)

xC

TABy = 129.55 N

(5)

and the magnitude of the tension in AB is

Equilibrium equation

129.55 N

TAB = (TABx)2 + (TABy)2

MC = 0: (150 N)(6 m) (259.09 N)(2 m + 6 m)


+ (129.54 N)(xC) = 0

= (259.09 N)2 + (129.55)2

Solving gives
xC = 9.05 m

= 290 N
Ans.

1054

Ans.

259.09 N

6.5 Cables: Concentrated Loads Example 3, page 4 of 5


13 Free-bodies of B and C will yield the
values of the tension in BC and CD
14 Free-body diagram of B for
calculating tension TBC
TABy
TBCx

129.55 N

B
TABx

150 N

TBC

259.09 N

TBCy

15 Summing vertical forces gives


TBCy = 129.55 N
Summing horizontal forces gives
+

Fx = 0:

150 N

TBCx + 259.09 N = 0

16 The magnitude of the tension in BC is


TBC =
=

Solving gives

(TBCx)2 + (TBCy)2
(109.09 N)2 + (129.55 N)2

= 169.4 N

TBCx = 109.09 N

1055

Ans.

6.5 Cables: Concentrated Loads Example 3, page 5 of 5


17 Free-body diagram of C for
calculating tension TCD
TBCy

129.55 N
C TCDx

500 N

4m
TBCx

109.09 N

TCD

TCDy

150 N

2m

Summing vertical forces gives


6m

TCDy = 129.55 N
Summing horizontal forces gives
+

Fx = 0:

500 N

500 N

TCDx + 109.09 N = 0

3m

Solving gives

D
xC

TCDx = 390.91 N
The magnitude of the tension in CD is
TCD =
=

(TCDx)2 + (TCDy)2
(390.91 N)2 + (129.55)2

= 412 N

Ans.

1056

6.5 Cables: Concentrated Loads Example 4, page 1 of 6


4. For the cable system shown, determine the distance yC for
which segment BC will be horizontal. Also determine yD.
E

3m

A
yD

yC

1.2 kN
2.2 kN
0.25 m

2m

1.8 kN
2m
0.5 m

1 Strategy: We have to make use of the fact


that segment BC is horizontal. One way to
do this is to pass a section through BC and
then consider the portion of the cable to the
left of the section.

1057

6.5 Cables: Concentrated Loads Example 4, page 2 of 6


2 Free-body diagram of AB
E

Ay
Ax

A
TBC (horizontal)

3m

2.2 kN
3

Equilibrium equation for AB (Because BC is


horizontal, we use the sum of vertical forces
so that the unknown tension TBC will not
appear in the equation.):

yD

yC

1.2 kN
+

Fx = 0: Ay

2.2 kN = 0

2.2 kN

Solving gives
Ay = 2.2 kN

0.25 m

2m

1.8 kN
2m
0.5 m

(1)

1058

6.5 Cables: Concentrated Loads Example 4, page 3 of 6


4

Now that Ay is known, we can solve for


Ax by summing moments about E for a
free-body diagram of the entire cable.
Ey

Free-body diagram of entire cable

E
Ex

3m
2.2 kN

Equilibrium equation for entire cable

Ay

Ax

A
yC

Solving gives

1.2 kN
2.2 kN

1.8 kN

Ax = 1.517 kN
0.5 m

0.25 m

ME = 0: (1.2 kN)(0.5 m)
+ (1.8 kN)(0.5 m + 2 m)
+ (2.2 kN)(0.5 m + 2 m + 2 m)
(2.2 kN)(0.5 m + 2 m + 2 m + 0.25 m)
x(3 m) = 0

2m

2m

1059

(2)

6.5 Cables: Concentrated Loads Example 4, page 4 of 6


7

Summing moments about B for a free body AB


will now give us the value of yC.

Free-body diagram of AB
Ay

2.2 kN
3m

A
Ax

1.517 kN

yB

TBC

yC (because BC is horizontal)

A
yB

2.2 kN

yD

yC

B
0.25 m

1.2 kN
2.2 kN

Equilibrium equation for AB


0.25 m

1.8 kN

MB = 0: (1.517 kN)(yC)

2m
0.5 m

(2.2 kN)(0.25 m) = 0

Solving gives
yC = 0.36 m

2m

Ans.

1060

6.5 Cables: Concentrated Loads Example 4, page 5 of 6


10 Finally, summing moments about D
for a free body ABCD will give an
equation for yD.
11 Free-body diagram of ABCD
Ay

2.2 kN

TDE

A
yD

1.517 kN

12 Equilibrium equation
D

Ax

1.2 kN
2.2 kN
0.25 m

2m

1.8 kN

MD = 0: (1.517 kN)(yD)
+ (2.2 kN)(2 m + 2 m)
+ (1.8 kN)(2 m)
(2.2 kN)(0.25 m + 2 m + 2 m) = 0

Solving gives

2m

yD = 2.01 m

1061

Ans.

6.5 Cables: Concentrated Loads Example 4, page 6 of 6


E

D
3m
2.01 m
0.36 m

1.2 kN

2.2 kN

13 The minus sign in yD = 2.01 m


indicates point D lies above point A,
not below it, as was assumed in
drawing the free-body diagram.

1.8 kN

1062

6.5 Cables: Concentrated Loads Example 5, page 1 of 4


5. For the cable system shown,
determine the value of the forces PB
and PD necessary to maintain the
given configuration.

PB

A
2m

5m
C
200 N
7m
D
2m

PD

1
4m

4m

2m

3m

1063

Strategy: Equilibrium equations for a free


body of the entire cable would involve
six unknowns (the components of the
support reactions: Ax, Ay, Ex, and Ey;
plus PB and PD). Thus using a free body
of the entire cable does not look like a
good place to start.

6.5 Cables: Concentrated Loads Example 5, page 2 of 4


Ay

A better place to start is to observe that we


know the horizontal and vertical distances
between points B, C and D. Thus we can
calculate the angles that segments BC and DC
make with the horizontal, and then we can use
the equilibrium equation for connector C to find
the tension in BC and in CD. Once these
tensions are known, we can use the equilibrium
equations for segments AB and ED to determine
PB and PD.

PB

Ax
2m
B
5m
BC
DC

C
200 N

3
7m

B
D

2m

Geometry

PD

Ex

5m

BC

1
= tan ( 5 m ) = 59.04
3m

(1)

DC

1
= tan ( 7 m ) = 54.46
5m

(2)

BC

5m

3m

Ey
4m

2m+3m

4m

2m

3m

DC

7m

1064

6.5 Cables: Concentrated Loads Example 5, page 3 of 4


4

Free-body diagram of C
6

TBC

Free-body diagram of AB
4m

BC
DC

59.04

Ay

200 N

54.46
Ax
TDC

PB

2m
B

Fx = 0:

5 Equilibrium equations for C

Fy = 0: TBC sin 59.04

TBC cos 59.04

BC

TDC cos 54.46 + 200 = 0

59.04

TDC = 187.02 N

(4)

Equilibrium equation for AB

Solving gives
(3)

177.46 N

TDC sin 54.46 = 0

TBC = 177.46 N

TBC

MA = 0: PB(4 m) + (177.46 N)(cos 59.04)(2 m)


(177.46 N)(sin 59.04)(4 m) = 0

Solving gives
PB = 106.5 N

1065

Ans.

6.5 Cables: Concentrated Loads Example 5, page 4 of 4


8

Free-body diagram of ED
C

DC

54.46
D

2m

Ex

TDC

187.02 N
PD

Ey

8m

ME = 0:

PD(2 m) (187.02 N)(cos 54.46 )(2 m)


+ (187.02 N)(sin 54.46 )(8 m) = 0

Solving gives
PD = 500 N

Ans.

1066

6.5 Cables: Concentrated Loads Example 6, page 1 of 8


6. For the cable system shown, determine
distance yB and the tension in each segment.
1.5 ft
A
D
3.5 ft
yB
C
B

50 lb

80 lb

2 ft

3.5 ft

1
5 ft

Strategy: If we can compute the reactions


at supports A and D, then we can compute
the tensions in AB and CD. Let's start
with support D. Note that both horizontal
and vertical distances between C and D
are known. Thus summing moments
about C, for the free body CD, would give
an equation involving Dx and Dy only.

1067

6.5 Cables: Concentrated Loads Example 6, page 2 of 8


2

Free-body diagram of CD

We can get another equation involving reactions at Dx and


Dy as the only unknowns by summing moments about A
for a free body of the entire cable.

Free-body diagram of entire cable


Ay

5 ft
Dy

Dx
Dy
3.5 ft

Ax

1.5 ft
Dx

C
TBC

50 lb
C
Equilibrium equation
MC = 0: Dy(5 ft)

B
Dx(3.5 ft) = 0

50 lb

(1)
80 lb
2 ft
6

1068

3.5 ft

5 ft

Equation of equilibrium

MA = 0: Dy(2 ft + 3.5 ft + 5 ft) Dx(1.5 ft)


(50 lb)(2 ft + 3.5 ft) 80 lb(2 ft) = 0

(2)

6.5 Cables: Concentrated Loads Example 6, page 3 of 8


Solving Eqs. 1 and 2 simultaneously gives
Dx = 49.15 lb

(3)

Dy = 34.41 lb

(4)

Now that the values of Dx and Dy are


known, summing moments about B, for a
free body BCD, will give an equation with
a single unknown, yB.

Free-body diagram BCD

Dy

34.41 lb

D
Dx
TAB

49.15 lb

3.5 ft
yB

10
C

MB = 0:

(50 lb)(3.5 ft) + (34.41 lb)(3.5 ft + 5 ft)

(49.15 lb)(yB) = 0

Solving gives
B

50 lb

yB = 2.39 ft

80 lb
3.5 ft

5 ft

1069

Ans

(5)

6.5 Cables: Concentrated Loads Example 6, page 4 of 8


11 We still must find the tension in each cable segment.
A free-body consisting of support D will give the
tension in CD.
1.5 ft

12 Free-body diagram of D for determining tension TCD


Dy 34.41 lb
TCDx

D
Dx

TCD

49.15 lb

3.5 ft
yB

TCDy

Summing x forces and then y forces gives


B
TCDx = 49.15 lb

(6)

TCDy = 34.41 lb

(7)

80 lb

The magnitude of the tension in CD is then


2 ft
TCD =
=

(TCDx)2 + (TCDy)2
(49.15 lb)2 + (34.41 lb)2

= 60.0 lb

50 lb

Ans.

1070

3.5 ft

5 ft

6.5 Cables: Concentrated Loads Example 6, page 5 of 8


13 A free-body diagram of connection C will give
the tension in cable segment BC.

1.5 ft
A

14 Free-body diagram of C for determining tension TBC


TCDy

34.41 lb (Eq. 6)
3.5 ft

TBCx

yB

TCDx

49.15 lb (Eq. 7)

TBCy

TBC

50 lb

50 lb
80 lb
Summing horizontal forces gives
TBCx = 49.15 lb

2 ft

(8)

Summing vertical forces gives


Fy = 0: 34.41 lb

TBCy

50 lb = 0

Solving gives
TBCy = 15.59 lb

(9)

1071

3.5 ft

5 ft

6.5 Cables: Concentrated Loads Example 6, page 6 of 8


15 Why did TBCy turn out
to be negative? Because
in drawing the tension
TBC on the free-body
diagram of C, we
assumed that B was
below C:

A
Free-body diagram of C
D

CDy

3.5 ft
yB

CDx

C
TBC (Pulls down on C)
B
80 lb

16 But this assumption was


wrong, as we should have
realized, since we have
already calculated yB and
found yB = 2.39 ft (Eq. 5).
The corrected free-body
diagram shows TBC pulling
up on C, which would lead
to a positive value of TBCy

50 lb

50 lb

Corrected free-body diagram of C


CDy

D
2.39 ft
3.5 ft
B
B lies
above C

C
80 lb

50 lb

1072

TBC (Pulls up on C)
C

50 lb

CDx

6.5 Cables: Concentrated Loads Example 6, page 7 of 8


17 The magnitude of the tension in BC is
TBC

(TBCx)2 + (TBCy)2

(49.15 lb)2 + ( 15.59 lb)2

= 51.56 lb

Ans.

18 The tension in AB can be found from a free-body


diagram of connector B.
19 Free-body diagram of B for calculating TAB

20 Summing x forces gives


TABx = 49.15 lb

TAB
TABy

Summing y forces gives


TABy = 15.59 lb + 80 lb

TABx

TBCx 49.15 lb
B
TBCy +15.59 lb

= 95.59 lb
The magnitude of the tension in AB is then
TAB =

80 lb

Since TBCy points down,


we are assuming correctly
that point C lies below B.

(TABx)2 + (TABy)2
(49.15 lb)2+ (95.59 lb)2

= 107.5 lb

1073

Ans.

6.5 Cables: Concentrated Loads Example 6, page 8 of 8


21 The tension in the cable segments could have
been calculated by other approaches, for
example, by calculating the angle of inclination
of each segment and then summing moments
about one end of the segment.

1.5 ft
A
D

Example:
Ay
3.5 ft
yB
Ax

C
B

(known)

50 lb

B
TBC (unknown)

80 lb

80 lb
2 ft
MA = 0 gives an equation that will give us the
value of TBC. Proceeding to the next cable segment,
BC, and summing moments about end B of that
segment would give the value of the tension in that
segment, etc.

1074

3.5 ft

5 ft

6.5 Cables: Concentrated Loads Example 7, page 1 of 7


7. The cable supports the four forces shown.
Determine the maximum tension in the cable.
18 ft

18 ft

18 ft

18 ft

18 ft

15 ft
F

35 ft
B
C

E
D

2 kip
2 kip

2 kip

2 kip
1

Strategy: To find the maximum tension, Tmax in


the cable, we could find the tension in each of
the five segments and then pick the largest.
However, this is a tedious and time-consuming
approach, and we can find Tmax more easily if we
note two facts: 1) The horizontal component of
tension is the same in all cable segments, and 2)
the maximum tension occurs in the cable
segment with the maximum slope.

1075

6.5 Cables: Concentrated Loads Example 7, page 2 of 7


Demonstration that horizontal components
of tension are equal:

3 Demonstration that Tmax occurs in the segment


with the greatest slope:

Free-body diagram of B
TAB

TABy
Thorizontal
B

TABx
TBCy

TBCx

Since Thorizontal is the same for all segments, it


follow that T will be largest where cos is
smallest, that is, where is the largest the
steepest slope.

TBC
2 kip

Fx = 0:

For each cable segments,


Thorizontal
T =
cos

TABx + TBCx = 0

Thus
TABx = TBCx
That is, the horizontal components are
equal. A similar argument holds for points
C, D, and E.

1076

6.5 Cables: Concentrated Loads Example 7, page 3 of 7


4 Thus the problem of finding Tmax has now
been reduced to determining whether the slope
is greater at A than at B.
5 To determine A, we can find Ax and Ay
first. To do this, sum moments about F for
a free-body consisting of the entire cable.
Free-body diagram of entire cable
18 ft

18 ft

18 ft

18 ft

18 ft

Ay
Fy
A

15 ft

35 ft

Fx

B
2 kip

6 Equilibrium equation for cable

Ax

2 kip

2 kip
2 kip

MF = 0: Ax(15 ft)
+ (2 kip)(4
+ (2 kip)(3
+ (2 kip)(2
+ (2 kip)(1

1077

y(5

18 ft)

18 ft)
18 ft)
18 ft)
18 ft) = 0

(1)

6.5 Cables: Concentrated Loads Example 7, page 4 of 7


7

To obtain another equation for Ax and y, pass a


section through segment DE and consider a free
body ABCD.

Free-body diagram of ABCD


18 ft

18 ft

18 ft

Ay

Equilibrium equation for ABCD

Ax

35 ft
B
C

TDE
D

2 kip
2 kip

MD = 0: Ax(35 ft)
18 ft)
y(3
+ (2 kip) (2 18 ft)
+ (2 kip) (18 ft) = 0

(2)

Solving Eqs. 1 and 2 simultaneously gives


Ax = 4.154 kip

(3)

= 4.692 kip

(4)

2 kip

1078

6.5 Cables: Concentrated Loads Example 7, page 5 of 7


10 To find the slope at F, consider a free body of the
entire cable.

11 Free-body diagram of entire cable


Ay

4.154 kip

Fy

A
Fx

B
C

12 Equilibrium equations for entire cable

D
2 kip
2 kip

2 kip
2 kip

Fx = 0:

Ax

4.692 kip

Fy = 0: 4.692 kip + Fy

4.154 kip + Fx = 0
(4

2 kip) = 0

Solving gives

1079

Fx = 4.154 kip

(5)

Fy = 3.308 kip

(6)

6.5 Cables: Concentrated Loads Example 7, page 6 of 7


13 Compare slopes at A and F

Ay

4.692 kip

Fy

3.308 kip

Ax

4.154 kip

1
= tan ( 4.692 ) = 48.5
4.154

14 Thus

is greater than

Fx

4.154 kip

TFE

TAB
A

1
= tan ( 3.308 ) = 38.5
4.154

and it follows that Tmax occurs at A.

Tmax = magnitude of the reaction force at A


=

(Ax)2 + (Ay)2

(4.154 kip)2 + (4.692 kip)2

= 6.27 kip

Ans.

1080

6.5 Cables: Concentrated Loads Example 7, page 7 of 7


15 Of course, once we had calculated the components of the
reaction at F,
Fx = 4.154 kip

(Eq. 5 repeated)

Fy = 3.308 kip

(Eq. 6 repeated)

we could have computed the tension in EF:


TEF =

(4.154 kip)2 + (3.308 kip)2

= 5.31 kip
and then we could have compared TEF with the tension in
TAB. That is, in this particular example, we didn't have to
compare slopes, but we did it to illustrate the principle
that Tmax occurs where the slope is a maximum.

1081

6.6 Cables: Uniform Loads

1082

6.6 Cables: Uniform Loads Procedures and Strategies, page 1 of 3


Procedures and Strategies for Solving Problems
Involving Cables With Uniform Loads

1. Draw a free-body diagram of the entire cable and write


three equilibrium equations involving the four reaction
components at the supports.

2m
A

2. Obtain a fourth equation by using additional information


given in the problem, such as the following:

w kN/m
6m

a) The location of the low point of the cable is


specified. In this case, pass a section through the low
point, consider a free body of the portion of the cable
on one side of the section, and write M = 0 about the
low point.

Section through low


point
By

B
2m
To
w
6m

1083

Bx

6.6 Cables: Uniform Loads Procedures and Strategies, page 2 of 3


b) The tension in the cable is known at a support. In
this case, draw a free-body diagram of a short piece
of cable at the support, and write Fx = 0 and Fy = 0.

c) The slope of the cable at a support is known. In


this case, proceed as in case b).

3. Once the support reactions are known, you can solve for
all other quantities such as cable tensions or the sag by
considering free-body diagrams of the supports or of
portions of the cable found by passing a section through the
cable and isolating the portion on one side.

w kN/m

By
T known, unknown, so one
additional unknown and two
additional equations.

Bx

By
known (slope = tan ), T
unknown, so one additional
unknown and two additional
equations.

1084

B
T

Bx

6.6 Cables: Uniform Loads Procedures and Strategies, page 3 of 3


Notes:
a) You can solve most uniform-load cable problems by
using free bodies and equilibrium equations only. You
do not need to use the equation for the cable shape,
y = wx2/(2To)

Cable

(1)

unless the cable length, s, is part of the problem, in


which case you must use Eq. 1 together with the
equation for calculating s.
ds =

[(dx)2 + (dy)2]

(2)

In using Eq. 1, take into account that the origin of the


xy-coordinate system is at the low point of the
parabola, but the low point and hence the origin need
not by part of the cable, since the cable coincides with
only a part of the parabola.

Parabolic Curve y =

b) You can find the maximum tension in the cable by


calculating the tension at the point where the slope is a
maximum.
c) In some problems you can save some work by using
the fact that the horizontal component, To, of the cable
tension is constant throughout the cable.

1085

wx2
2T o

6.6 Cables: Uniform Loads Problem Statement for Example 1


1. Each cable of the center span of the suspension
bridge supports a uniform load of 10 kip/ft along
the horizontal. If the span is 3,800 ft and the sag
400 ft, determine the maximum and minimum
tensions in the cable.

3800 ft
A

400 ft

1086

6.6 Cables: Uniform Loads Problem Statement for Example 2


2. A length of oil pipeline weighing 3200 lb is supported by a system
of cables as shown. Determine a) the distance h to the lowest point C
on the cable and b) the maximum tension in the cable.
B
8 ft

C
h

30 ft

50 ft

1087

6.6 Cables: Uniform Loads Problem Statement for Example 3


3. An 18-lb washer load of wet beach-towels is hung on a clothesline
to dry. Each beach towel is 72-in. long, and the clothesline is tied to
supports located 75 in. above the ground. If the breaking strength of
the clothesline is 280 lb, determine if the clothesline can be tightened
enough to keep the middle towel from touching the ground.
28 ft

A
75 in

75 in

1088

6.6 Cables: Uniform Loads Problem Statement for Example 4


4. Chains AB and BC are attached to a roller support at B
as shown. The chains support beams that have mass per
length of 0.5 kg/m. Determine the maximum tension in
each chain and the sag s in chain BC.
80 m

60 m

3m
A

0.5 kg/m

s
B

0.5 kg/m

1089

6.6 Cables: Uniform Loads Problem Statement for Example 5


5. Cable AB supports a uniformly distributed mass of
0.2 kg/m. The slope of the cable at B is known to be
40. Determine the maximum tension in the cable and
the length of the cable.

40
B

2m
A

0.2 kg/m
3m

1090

6.6 Cables: Uniform Loads Problem Statement for Example 6


6. Determine the largest uniform load, w lb/ft, that the cable
can support if it will fail at a tension of 3,000 lb. Also
determine the location of the low point C of the cable.
A
4 ft

6 ft
C

w lb/ft

48 ft

1091

6.6 Cables: Uniform Loads Problem Statement for Example 7


7. The cable system shown supports a uniformly
distributed mass of 5 kg/m along the horizontal.
Determine the tension at B and the length of portion AB
of the cable. Assume that the pulleys are frictionless.
A
4m
B

100 kg

5 kg/m

20 m

1092

6.6 Cables: Uniform Loads Problem Statement for Example 8


8. The chain AB supports a horizontal, uniform beam of
mass per length 22 kg/m. If the maximum allowable
tension in the chain is 7 kN, determine distances dA and
dB of the supports above the low point C of the chain.
Also determine the length of the chain.
y
30 m

16 m

dA
B
dB
x

22 kg/m

1093

6.6 Cables: Uniform Loads Problem Statement for Example 9


9. A 40-m length of rope has a uniformly distributed
mass of 0.1 kg/m and has one end fixed and the other
end attached to a cart as shown. Determine the distance
d and the sag h when the cable and cart are in
equilibrium under the force supplied by the 10-kg load.
d
A

C
h

10 kg

1094

6.6 Cables: Uniform Loads Example 1, page 1 of 4


1. Each cable of the center span of the suspension
bridge supports a uniform load of 10 kip/ft along
the horizontal. If the span is 3,800 ft and the sag
400 ft, determine the maximum and minimum
tensions in the cable.

3800 ft
A

400 ft

1095

6.6 Cables: Uniform Loads Example 1, page 2 of 4

1 The maximum tension occurs where the slope is


greatest points A and B.
3800 ft
A

400 ft

2 The minimum tension occurs where the slope is least


low point of the cable.

1096

the

6.6 Cables: Uniform Loads Example 1, page 3 of 4


3800 ft

400 ft
C

3
7

Free-body diagram of CB

Resultant load
TBy

(10 kip/ft)(1900 ft)

5 Tension components at B
= 19,000 kip

B
400 ft
C

To
4 Tension at low point
(minimum tension)

TBx
8 (1900 ft)/2

950 ft (Resultant acts through middle of


1900 ft span)

6 (3800 ft)/2 = 1900 ft


(Because of symmetry, we know
that the low point occurs at the
middle of the 3800-ft span.)

1097

6.6 Cables: Uniform Loads Example 1, page 4 of 4


9

Fx = 0:

Fy = 0: TBy 19,000 kip = 0

Equilibrium equation
T + TBx = 0

MB = 0: (19,000 kip)(950 ft)

T (400 ft) = 0

Solving gives
T = 45,125 kip (minimum tension in cable)

Ans.

TBx = 45,125 kip


TBy = 19,000 kip
The maximum tension is the resultant of TBx and TBy:
TBy

19,000 kip Tmax


B

TBx = 45,125 kip

Tmax = (45,125 kip)2 + (19,000 kip)2


= 49,000 kip

Ans.

1098

6.6 Cables: Uniform Loads Example 2, page 1 of 4


2. A length of oil pipeline weighing 3200 lb is supported by a system
of cables as shown. Determine a) the distance h to the lowest point C
on the cable and b) the maximum tension in the cable.
B
8 ft

C
h

30 ft

50 ft

1099

6.6 Cables: Uniform Loads Example 2, page 2 of 4


The maximum tension occurs at B, where the
slope is largest, so let's draw a free-body
diagram of the entire system, including the
tension components at B:

Equilibrium equation

4
(80 ft)/2

40 ft

40 ft
By

Ay
B

8 ft
Ax

Bx

A
C
2

3200 lb (Weight of pipeline acts through


midpoint of 30 ft + 50 ft 80-ft span,
not through low point, C)

1100

MA = 0: By(40 ft + 40 ft)
Bx(8 ft) (3,200 lb)(40 ft) = 0
We could write two additional equilibrium
equation, but they would introduce two
additional unknowns, Ax and Ay, so nothing
would be gained. An additional free-body
diagram involving Bx and By is needed.

(1)

6.6 Cables: Uniform Loads Example 2, page 3 of 4


5

Pass a vertical section through the low point, C, and


draw a free-body diagram of the portion of the
system to the right of C.
By
Ay
B

7 Free-body diagram of BC

Bx
By

8 ft

A
Ax

(50 ft)/2
25 ft

50 ft

Bx

h
To
(To is horizontal
because C is
the minimum
point on the
curve.)

h + 8 ft

Weight

distributed load
40 lb/ft

length of pipe

50 ft

2000 lb
50 ft
9

80 ft

The 3200-lb weight spread over an 80-ft length


of pipe is equivalent to a uniformly distributed
load of 3200 lb/80 ft = 40 lb/ft.

Equilibrium equation for free-body BC

1101

Fx = 0:

T + Bx = 0

Fy = 0: By

(2)

2000 lb = 0

MB = 0: (2000 lb)(25 ft)

(3)
T (h + 8 ft) = 0

(4)

6.6 Cables: Uniform Loads Example 2, page 4 of 4


10 Eq. 4 is nonlinear, but Eqs. 1-3 are linear and can be easily
solved to give
Bx = 4000 lb
By = 2000 lb
T = 4000 lb
Using the value of T = 4000 lb in Eq. 4 gives
(2000 lb)(25 ft)

T (h + 8 ft) = 0

(Eq. 4 repeated)

Solving gives
h = 4.5 ft

Ans.

11 The maximum tension is the resultant of Bx and By:


By

2000 lb

Tmax
Bx

4000 lb

B
Tmax = (4000 lb)2 + (2000 lb)2
= 4470 lb

Ans.

1102

6.6 Cables: Uniform Loads Example 3, page 1 of 2


3. An 18-lb washer load of wet beach-towels is hung on a clothesline
to dry. Each beach towel is 72-in. long, and the clothesline is tied to
supports located 75 in. above the ground. If the breaking strength of
the clothesline is 280 lb, determine if the clothesline can be tightened
enough to keep the middle towel from touching the ground.
28 ft

A
75 in

75 in

1 The towel near the middle of the clothesline span will touch the
ground if the sag is 75 in. 72 in. = 3 in. We can now work the
problem either of two ways: 1) assume the sag = 3 in., compute
the corresponding maximum tension, and compare it to the
280-lb breaking strength; or 2) assume the maximum tension is
280 lb, compute the corresponding sag, and compare it to the
maximum permissible sag of 3 in.

1103

6.6 Cables: Uniform Loads Example 3, page 2 of 2


Let's arbitrarily choose the first approach assume a sag
= 3 in. Then a free-body diagram of the right half of the
clothesline and hanging towels would appear like this:

(28 ft)/4

7 ft

(18 lb)/2
9 lb

(28 ft)/4

Bx = 252 lb
By = 9 lb

7 ft

T = 252 lb
By

The maximum tension occurs where the slope is


greatest at B (or A):
Bx

3 in

To

Solving Eq. 1-3 gives

By

9 lb

Fx = 0:

T + Bx = 0

(1)

Fy = 0:

9 lb + By = 0

(2)

MB = 0: 9 lb(7 ft)
T (3 in. 1 ft/ 12 in.) = 0

Bx

252 lb

Since the maximum tension, 252.2 lb, is less than


the breaking strength, 280 lb, the clothesline can
be tightened enough to keep the towel from
touching the ground. Note, however, that the
poles supporting the clothesline must be
well-anchored in the ground since they must
resist a horizontal force of Bx = 252 lb acting at
the point where the clothesline is attached.

Equilibrium equations

Tmax = (252 lb)2 + (9 lb)2 = 252.2 lb

(3)

1104

6.6 Cables: Uniform Loads Example 4, page 1 of 4


4. Chains AB and BC are attached to a roller support at B
as shown. The chains support beams that have mass per
length of 0.5 kg/m. Determine the maximum tension in
each chain and the sag s in chain BC.
80 m

60 m

3m
A

0.5 kg/m

s
B

0.5 kg/m

1 The maximum tension in chain AB occurs where the


slope is greatest, points A and B.

1105

6.6 Cables: Uniform Loads Example 4, page 2 of 4

We know the location of the low point of chain AB, so


let's choose a free-body diagram of the right half of
the chain because then we will know the distances
appearing in the moment equation.
By symmetry, we know the low point occurs at mid span:
(80 m)/2

40 m

(40 m)/2

Equilibrium equation

Fx = 0:

Fy = 0: By

MB = 0:

T + Bx = 0
196.2 N = 0

(2)

T (3 m)
+ (196.2 N)(20 m) = 0

(3)

20 m
6 Solving Eqs. 1-3 gives
T = 1308 N

By

Bx = 1308 N

3 Sag distance is given: 3 m


To

By = 196.2 N
B

Bx

Resultant force
(0.5 kg/m)(9.81 m/s2)(40 m)
196.2 N

1106

(1)

6.6 Cables: Uniform Loads Example 4, page 3 of 4


7 Maximum tension in AB
By

196.2 N
B

Bx

1308 N
9

Next consider a free-body diagram of the


left half of chain BC:
(60 m)/2 30 m

Equilibrium equation
+

(30 m)/2

Ans.

15 m

(1308 N)2 + (196.2 N)2 = 1323 N

Tmax =

B'y
B'x
T'o

B
s
Resultant weight

(0.5 kg/m)(9.81 m/s2)(30 m)


147.15 N

1107

Fx = 0:

Bx + T = 0

Fy = 0: By

147.15 N = 0

MB = 0: T (s)

(147.15 N)(15 m) = 0

(4)
(5)
(6)

6.6 Cables: Uniform Loads Example 4, page 4 of 4


10 Eqs. 4-6 are three equations in four unknowns, Bx ,
By , T and s. An additional free-body is needed.
11 The free-body diagram below shows that no
horizontal force acts on the base of the roller
support at B.
B
To

T'o

1308 N
196.2 N

147.15 N

FB, vertical force


acting on roller support
12 Equilibrium equation
+

Fx = 0:

1308 N + T = 0

(7)

13 Maximum tension in BC

Solving Eqs. 4-7 simultaneously gives


Tmax

B'y

147.15 N

T = 1308 N
s = 1.688 m
Bx = 1308 N

B'x

Ans.

Tmax =

By = 147.15 N

1308 N

(1308 N)2 + (147.15 N)2

= 1316 N

1108

Ans.

6.6 Cables: Uniform Loads Example 5, page 1 of 5


5. Cable AB supports a uniformly distributed mass of
0.2 kg/m. The slope of the cable at B is known to be
40. Determine the maximum tension in the cable and
the length of the cable.

40
B

2m
A

0.2 kg/m
3m

1 The maximum tension occurs at B, where the slope of


the cable is the greatest.

1109

6.6 Cables: Uniform Loads Example 5, page 2 of 5


2

Free-body diagram
3m

(3 m)/2

Tmax
40

Tmax sin 40
B

2m

1.5 m

Tmax cos 40

Ay

Ax

Weight = (0.2 kg/m)(9.81 m/s2)(3 m)


= 5.886 N

Equilibrium equation

MA = 0: (Tmax sin 40)(3 m)


(Tmax cos 40)(2 m)
(5.886 N)(1.5 m) = 0

4 To compute the length of the cable, we need


to use the equation of the cable curve,

(1)

Solving gives
Tmax = 22.28 N

wx2
y=
2T

Ans.

1110

(2)

6.6 Cables: Uniform Loads Example 5, page 3 of 5


5 Here, w is the distributed load per horizontal meter,
w = (0.2 kg/m)(9.81 m/s2)
= 1.962 N/m

(3)

The quantity T is the horizontal component of the cable tension.


Since T is the same at all points, we can evaluate it at support B:
T = Tmax cos 40
= (22.28 N) cos 40

(4)

Substituting Eqs. 3 and 4 in Eq. 2 gives


1.962 N/m by Eq. 3
y =

wx2
2T

(Eq. 2 repeated)
(22.28 N) cos 40 by Eq. 4

or,
y = 0.057478x2

(5)

We will also need the slope,


dy
= 2(0.057478x)
dx
= 0.114956x

(6)

1111

6.6 Cables: Uniform Loads Example 5, page 4 of 5


6

(Change the variable of


integration from s to x)

The length of the cable is


sAB =
=

A ds

B
A

ds
(dx)2 + (dy)2

dy

dx
3m

x
= x B 1 + ( dy )2 dx
A
dx

40

0.114956x by Eq. 6

B(xB, yB)

Thus the length of the cable can be expressed as


xB
sAB = xA 1 + (0.114956x)2 dx

(7)

To evaluate this integral, we have to find the values of xA


and xB. From the figure, we see that
xA = xB

3m

0.05784 x2

A(xA, yA)

(8)
x
40 dy
7

1112

Also at B, the slope is known:


dy
= tan 40
dx

dx
(9)

6.6 Cables: Uniform Loads Example 5, page 5 of 5


8 Using Eq. 6 to evaluate the left-hand side of Eq. 9 gives

dy
) = tan 40
dx B

(Eq. 9 repeated)

0.114956xB, by Eq. 6
Solving for xB gives
xB = (tan 40)/0.114956
= 7.299311 m
and using this result in Eq. 8 gives
xA = xB

3m

= 7.299311

= 4.299311 m
By Eq. 7, then, the length of the cable is

9 This integral is best evaluated numerically with the


integral function of a calculator.

7.299311

sAB =

4.299311

1+ (0.114956x)2 dx

The result is
sAB = 3.61 m

1113

Ans.

6.6 Cables: Uniform Loads Example 6, page 1 of 5


6. Determine the largest uniform load, w lb/ft, that the cable
can support if it will fail at a tension of 3,000 lb. Also
determine the location of the low point C of the cable.
A
4 ft

6 ft
C

w lb/ft

48 ft

1114

6.6 Cables: Uniform Loads Example 6, page 2 of 5


1 The maximum tension in the cable occurs where the
slope is greatest, point A. The cable will fail if the
tension there exceeds 3,000 lb. A free-body diagram of
the entire system, with a maximum tension of 3,000 lb at
A, would appear as below:
The line of action of the weight of the cable does
not pass through the low point C, because the ends
of the cable are at different elevations.
48 ft
(48 ft)/2

24 ft

24 ft

(3000 lb) sin

(3000 lb) cos

By

4 ft

As part of our solution


to the problem, we will
calculate .

Bx
Resultant load
w lb/ft

6 ft

48 ft
2

Moment equilibrium equation

(48w) lb

1115

MB = 0: (48w)(24 ft) (3000 lb)(sin )(48 ft)


+ (3000 lb)(cos )(4 ft) = 0

(1)

6.6 Cables: Uniform Loads Example 6, page 3 of 5


Moment equilibrium equation

MB = 0: (48w)(24 ft) (3000 lb)(sin )(48 ft)


+ (3000 lb)(cos )(4 ft) = 0

(1)

We could write two additional equilibrium equation, but they


would introduce two additional unknowns, Bx and By, so there
is no advantage gained. Instead, we need another free-body
diagram.
3
(3000 lb) sin

Pass a vertical section through


the low point, C.
By

A
(3000 lb) cos

4 ft

B
Bx
C

d
(Unknown distance)

1116

6 ft

6.6 Cables: Uniform Loads Example 6, page 4 of 5


4

Free-body diagram of portion to left of section

(3000 lb) sin


A

Eqs. 1-3 are three nonlinear equations in three unknowns,


, w, and d. These equations are best solved with a
calculator capable of solving simultaneous nonlinear
equations. Alternatively, proceed as follows. First note
that Eq. 2,

(3000 lb) cos


4 ft + 6 ft

10 ft

C
Resultant
w d

wd = 0

(Eq. 2 repeated)

can be solved for d:


To
d = (3000 sin )/w

(4)

This equation can be used to eliminate d from Eq. 3:

d/2

d/2

3000 sin

3000 cos (10) (3000 sin )d


+ wd2/2 = 0
(Eq. 3 repeated)

d
Equation of equilibrium
Fy = 0: (3000 lb)(sin )

wd = 0

MC = 0: (3000 lb)(cos )(10 ft)


(3000 lb)(sin )d
+ (wd)(d/2) = 0

(3000 sin )/w

(2)

(3000 sin )/w

Multiplying through by w and combining terms gives


(3)
(30,000 cos )w

We could write a third equilibrium equation, but it


would introduce an additional unknown, T so no
advantage would be gained.

1117

(3,000 sin )2/2 = 0

(5)

6.6 Cables: Uniform Loads Example 6, page 5 of 5


7

Next note that Eq.1,

(48)(24)w (3,000 sin )(48)


+ (3,000 cos )(4) = 0

Using this value for

in Eq. 6 then gives w:

w = 125 sin

10.4167 cos

(Eq. 6 repeated)

(Eq. 1 repeated)
= 125 sin 36.48

10.4167 cos 36.48

can be solved for w:


= 65.94 lb/ft
w = 125 sin

10.4167 cos

Ans.

(6)
Eq. 4 then gives distance d:

and this equation can be used to eliminate w from Eq. 5:


d=
(30,000 cos ) w

(3,000 sin )2/2 = 0

(Eq. 5 repeated)
=

125 sin

10.4167 cos

3000 sin
w

3000 sin 36.48


65.94

= 27.0 ft
Carrying out the multiplication gives
(3.75

106) cos sin


(0.312501
6
2
(4.5 10 ) sin = 0

106) cos2
(7)

Dividing both sides by 106 gives


(3.75) cos sin
(0.312501) cos2
(4.5) sin2 = 0

(8)

Solving this equation by trial and error gives


= 36.48

1118

(Eq. 4 repeated)

Ans.

6.6 Cables: Uniform Loads Example 7, page 1 of 4


7. The cable system shown supports a uniformly
distributed mass of 5 kg/m along the horizontal.
Determine the tension at B and the length of portion AB
of the cable. Assume that the pulleys are frictionless.
A
4m
B

100 kg

5 kg/m

20 m

The 100-kg mass produces a cable


tension of 100 kg 9.81 m/s2 = 981 N
at point A.

1119

6.6 Cables: Uniform Loads Example 7, page 2 of 4


2

Free-body diagram
20 m
10 m
981 N
(981 N) sin

(981 N) cos

By
4m
B

Bx

Resultant weight of cable between A and B


(20 m)(9.81 m/s2)(5 kg/m)
981 N

Equilibrium equation

Fx = 0:

(981 N) cos

Fy = 0: (981 N) sin

+ Bx = 0
+ By

981 N = 0

MB = 0: (981 N) cos (4 m)
(981 N) sin (20 m)
+ (981 N)(10 m) = 0

(1)

Eqs. 1-3 are best solved with a calculator that solves


simultaneous nonlinear equations. Alternatively, Eq. 3
involves only one unknown, , and can be solved by
trial-and-error to yield
= 40.67

(2)

(4)

Using this value in Eq. 1 and 2 then leads to


(3)

1120

Bx = 744.1 N

(5)

By = 341.7 N

(6)

6.6 Cables: Uniform Loads Example 7, page 3 of 4


5

The tension at B is then

Substituting Eqs. 8 and 9 in Eq. 7 gives,


49.05 N/m

Bx + By =

(744.1 N) + (341.7 N)

= 818.8 N
6

wx2
y=
2T

Ans.

744.1 N

or,
y = 0.03296 x2

To compute the length of the cable, we need to use the


equation of the cable curve,
wx2
y=
2T

(Eq. 7 repeated)

(10)

We will need the equation for the slope:


dy
dx = 2(0.3296 x) = 0.06592 x

(7)

(Change the
variable of
integration
from s to x)

Here, w is the distributed load per horizontal meter,


8

The length of the cable is

w = (5 kg/m)(9.81 m/s )
sAB =
= 49.05 N/m

(8)
=

The quantity T is the horizontal component of the cable


tension. Since T is the same at all points, we can evaluate it
at support B:

744.1 N by Eq. 5

(9)

ds

B
A

(dx) + (dy)

ds
dx

xA
xB

1+(

dy 2
) dx
dx
(0.06592 x)2, by Eq. 11

1121

dy

or
sAB =

T = Bx

(11)

(12)

6.6 Cables: Uniform Loads Example 7, page 4 of 4


11 Solving for xA gives

9 To evaluate the integral for sAB, we have to find the values


of xA and xB.
y

xA = 13.03436 m

0.03296x2

(14)

Using this result in Eq. 13 gives


y

A(xA, yA)

40.67

xB = xA + 20 m
=

(Eq. 13 repeated)

13.03436 m + 20 m

B(xB, yB)
x
20 m

6.96564

sAB =

xB = xA + 20 m

dy
= tan 40.67
dx

(13)
dx

1 + (0.06592 x)2 dx

-13.03436

This integral is best evaluated numerically with the


integral function of a calculator. The result is

40.67
dy

sAB = 21.7 m

Using Eq. 11 to evaluate the left hand-side of this equation gives


(

(15)

By Eq. 12, then, the length of cable is

10 From the figure, we see that

Also at A, the slope is known, so

= 6.96564 m

dy
) = tan 40.67
dx A
0.06592xA, by Eq. 11

1122

Ans.

6.6 Cables: Uniform Loads Example 8, page 1 of 6


8. The chain AB supports a horizontal, uniform beam of
mass per length 22 kg/m. If the maximum allowable
tension in the chain is 7 kN, determine distances dA and
dB of the supports above the low point C of the chain.
Also determine the length of the chain.
y
16 m

30 m
A

1 The maximum tension occurs at support A,


where the slope of the chain is greatest.

dA
B
dB
x

22 kg/m

1123

6.6 Cables: Uniform Loads Example 8, page 2 of 6


Free-body diagram of entire system.
Ay

Ax

30 m + 16 m 46 m
(46 m)/2

23 m

dA

By

dB

Bx
4

Equation of equilibrium

Resultant weight
(22 kg/m)(9.81 m/s2)(46 m)

MB = 0: Ax(dA dB) Ay(46 m)


+ (9.928 kN)(23 m) = 0

(1)

We could write two additional equations of


equilibrium, but they would contain two additional
unknowns, Bx and By, so nothing would be gained.

9928 N
9.928 kN

1124

6.6 Cables: Uniform Loads Example 8, page 3 of 6


5 To obtain an additional free-body diagram, pass a
vertical section through the low point of the chain.

Free-body diagram of portion of chain to left of section.


(30 m)/2

30 m
Ay

Ay

Ax

15 m

Ax

By

dA

dA

B
Bx

Resultant weight

To

(22 kg/m)(9.81 m/s2)(30 m)


6475 N
6.475 kN

22 kg/m
30 m

1125

6.6 Cables: Uniform Loads Example 8, page 4 of 6


7

Equation of equilibrium
Fx = 0:

Ax + T = 0

Fy = 0: Ay

6.475 kN = 0

MC = 0: Ax(dA) Ay(30 m)
+ 6.475 kN)(15 m) = 0

Ax dA

Ay(30) + (6.475)(15) = 0

(2)

2.660 kN

(3)

Solving gives

(Eq. 4 repeated)

6.475 kN

T = 2.660 kN

(7)

dA = 36.52 m

(8)

(4)

Equation 3 gives

Ans.

Distance dB can now be found by substitution in Eq. 1:


Ay = 6.475 kN

(5)
Ax (dA

Since we know that the maximum tension of 7 kN occurs


at A and is the resultant of Ax and Ay, we have

dB)

2.660 kN
by Eq. 6

Ax2 + Ay2 = 7 kN

Ay(46) + (9.928)(23) = 0
36.52 m
by Eq. 8

6.475 kN
by Eq. 5

Solving gives
Substituting Ay = 6.475 kN and solving for Ax gives
dB = 10.4 m
Ax = 2.660 kN

(6)

Substituting for Ax and Ay in Eqs. 2 and 4 gives


Ax + T = 0

(Eq. 2 repeated)
2.660 kN

1126

Ans.

6.6 Cables: Uniform Loads Example 8, page 5 of 6


9 To compute the length of the chain, we need to use the
equation of the chain curve:

10 We will need the equation for the slope,


dy
= 2(0.040564 x) = 0.08113 x
dx

wx
y=
2T

(9)

The length of the chain is

Here, w is the distributed load per horizontal meter,

sAB =

w = (22 kg/m)(9.81 m/s2)

ds

(12)

(Change the variable of


integration from s to x)
ds

dy

dx
= 215.8 N/m

= 0.2158 kN/m

or

The quantity T has been given in Eq. 7

xA
sAB =

(Eq. 7 repeated)

0.2158 kN/m by Eq. 10


(Eq. 9 repeated)

2.660 kN by Eq. 7
or,
y = 0.040564 x2

1+(
xB

Substituting into Eq. 9 gives

wx2
y=
2T

(dx)2 + (dy)2

(10)

T = 2.660 kN

(11)

1127

dy 2
) dx
dx

(13)

6.6 Cables: Uniform Loads Example 8, page 6 of 6


12 The integral for the cable length, Eq. 13, can now be
written as

11 Since the location of the low point of the chain is


known, xA and xB are known:
A(xA

30 m)

0.040564 x2

16

sAB =
y

1+(
-30

dy 2
) dx
dx

(Eq. 13 repeated)

(0.08113x)2, by Eq. 12
This integral is best evaluated numerically with the integral
function of a calculator. The result is
sAB = 69.2 m
B(xB

16 m)

30 m

16 m

1128

Ans.

6.6 Cables: Uniform Loads Example 9, page 1 of 4


9. A 40-m length of rope has a uniformly distributed
mass of 0.1 kg/m and has one end fixed and the other
end attached to a cart as shown. Determine the distance
d and the sag h when the cable and cart are in
equilibrium under the force supplied by the 10-kg load.
d
A

C
h

10 kg
1

The total weight of the rope is 40 m 0.1 kg/m 9.81 m/s2 =


39.24 N, and the force of the cart acting horizontally on the
rope is 10 kg 9.81 m/s2 = 98.1 N, or more than twice as much.
Thus it seems reasonable to assume that the sag, h, is small
compared to the span d, h
d, and the weight of the rope is
well-approximated as a uniformly distributed load along the
horizontal of
w = (0.1 kg/m)

(9.81 m/s2) = 0.981 N/m

(1)

At the end of the problem, we can check the reasonableness of


these assumptions.

1129

6.6 Cables: Uniform Loads Example 9, page 2 of 4


2

The equation describing the shape of the rope is


y=

wx2
2T

(2)

T is the horizontal component of tension in the rope


and can be found by considering free bodies of the
cart and rope.

Free-body diagram of rope


Cy

By
B

C
Cx

Bx To
(Horizontal
component
of tension)

Free-body diagram of cart

Bx

(10 kg) (9.81 m/s2)


98.1 N

6 The equation of the rope is now


By
0.981 N/m by Eq. 1
Vertical force from
ground acting on cart

Fx = 0:

y=

wx2
2T

98.1 N by Eq. 3

98.1 N + Bx = 0
or,

Solving gives
Bx = 98.1 N = T

(Eq. 2 repeated)

y = 0.005 x2

(3)

We will also need the equation for the slope,


dy
= 2(0.005 x) = 0.01x
dx

1130

(4)

(5)

6.6 Cables: Uniform Loads Example 9, page 3 of 4


7

The equation for the length of the slope is


C

sBC =

ds

ds

dy

dx
C

(dx)2 + (dy)2

xC
=

xB

1+(

dy 2
) dx
dx

(6)

xC and xB can be expressed in terms of the unknown, d:


y
d/2

d/2

B
d/2

xB

C
xC d/2

The best way to solve for d in Eq. 7 is to use the solver on a


calculator and to use the calculator integral function to input
Eq. 7 in the solver. Alternatively, use a table of integrals to
evaluate the integral in Eq. 7 to get
40 =

d
1 + (0.005d)2
2

x
+ (50) ln
Thus Eq. 6 can be written
d/2

sBC =

1+(
-d/2

Rope length = 40 m

0.005d + 1 + (0.005d)2

dy 2
) dx (Eq. 6 repeated)
dx

Solving this equation by trial-and-error gives


d = 39.74 m

(0.01 x) by Eq. 5

or,
d/2

40 =

-d/2

0.005d + 1 + (0.005d)2

1 + (0.01x)2 dx

(7)

1131

Ans.

(8)

6.6 Cables: Uniform Loads Example 9, page 4 of 4


9 The sag, h, can now be found from Eq. 4:
y = 0.005x2

(Eq. 4 repeated)
d 39.74
=
by Eq. 8
2
2

h
y

h
x

The result is
h = 1.974 m

Ans.

Note that the sag is much smaller than the span,


h = 1.974 m << 39.74 m = d, as we assumed in
the beginning of the problem.

1132

6.7 Cables: Catenaries

1133

6.7 Cables: Catenaries Procedures and Strategies, page 1 of 2


Procedures and Strategies for Solving Problems Involving Catenary Cables
y

To solve catenary-cable problems, you must determine the


"parameter of the catenary," c.
1. If the coordinates x and y of a point on the cable are
known, determine c from
y = c cosh (x/c)

(1)

(2)

3. If the y coordinate of a point on the cable and the length


s are known, determine c from
y2 - s2 = c2

(3)

4. If the tension T is known at a point on the cable,


determine the y coordinate of the point from
T = wy

(x, y) To

2. If the x coordinate of a point on the cable and the


corresponding cable length s are known, determine c from
s = sinh (x/c)

(4)

Then if either x or s is known, then use either Eq. 1 or 3 to


solve for c.

1134

tension
in cable
horizontal
component
of tension

6.7 Cables: Catenaries Procedures and Strategies, page 2 of 2


5. If the slope of the cable is known at a point on the cable
with coordinate x, then differentiate Eq. 1 to obtain an
equation that can be solved for c.
Frequently, solving catenary-cable problem requires
finding roots of nonlinear algebraic equations. You can
usually handle these problems by using the solver on your
calculator, but some equations have multiple roots, and
you must determine which of the roots are physically
realistic.

1135

6.7 Cables: Catenaries Problem Statement for Example 1


1. An electric power line of length 140 m and mass per
unit length of 3 kg/m is to be suspended between two
towers 120 m apart and of the same height. Determine
the sag and maximum tension in the power line.
120 m
A

1136

6.7 Cables: Catenaries Problem Statement for Example 2


2. A cable is supported at two points 400 ft apart and at
the same elevation. If the sag is 40 ft and the weight per
unit length of the cable is 4 lb/ft, determine the length of
the cable and the tension at the low point, C.
400 ft
40 ft
A

B
C

1137

6.7 Cables: Catenaries Problem Statement for Example 3


3. A 20-m chain is suspended between two points at
the same elevation and with a sag of 6 m as shown.
If the total mass of the chain 45 kg, determine the
distance between the supports. Also determine the
maximum tension.

B
6m

1138

6.7 Cables: Catenaries Problem Statement for Example 4


4. A certain cable will break if the maximum tension
exceeds 500 N. If the cable is 50-m long and has a
mass of 50 kg, determine the greatest span possible.
Also determine the sag.

Supports A and B are at the same elevation.

1139

6.7 Cables: Catenaries Problem Statement for Example 5


5. The cable is attached to a fixed support at A and a
moveable support at B. If the cable is 80-ft long, weighs
0.3 lb/ft, and spans 50 ft, determine the force F holding
the moveable support in place. Also determine the sag.

F (force)

1140

6.7 Cables: Catenaries Problem Statement for Example 6


6. The cable is attached to a fixed support at
A and a moveable support B. If the cable is
40-m long and has a mass of 0.4 kg/m,
determine the span and sag.
A

50 N

1141

6.7 Cables: Catenaries Problem Statement for Example 7


7. A cable goes over a frictionless pulley at B and
supports a block of mass m. The other end of the cable
is pulled by a horizontal force P. If the cable has a mass
per length of 0.3 kg/m, determine values of P and m that
will maintain the cable in the position shown.
B

5m
A

P (horizontal)
10 m

1142

6.7 Cables: Catenaries Problem Statement for Example 8


8. A chain makes angles of 30 and 60 at its
supports as shown. Determine the location of the
low point C of the chain relative to A. Also
determine the tension at support A, if the cable has
a mass per length of 0.6 kg/m.
Supports A and B
are at different
elevation.

60
B

30
A

20 m

1143

6.7 Cables: Catenaries Problem Statement for Example 9


9. A wire weighing 0.2 lb/ft is attached to a moveable
support at A and makes an angle of 55 at a fixed
support at B. Supports A and B are at different
elevations. Determine the location of the low point C
of the wire relative to support B. Also, determine the
tension in the wire at C.

A
8 lb
B

55

1144

6.7 Cables: Catenaries Problem Statement for Example 10


10. Determine the location of the low point C relative to
the support at A. Also determine the tension at C, if the
mass of the cable per unit length is 1 kg/m.
A
B

2m
1m

C
10 m

1145

6.7 Cables: Catenaries Example 1, page 1 of 4


1. An electric power line of length 140 m and mass per
unit length of 3 kg/m is to be suspended between two
towers 120 m apart and of the same height. Determine
the sag and maximum tension in the power line.
120 m
A

1146

6.7 Cables: Catenaries Example 1, page 2 of 4


1 Many problems involving catenary cables can be
solved using the following formulas:
s = c sinh (x/c)

(1)

y2

(2)

s2 = c2

W = ws

(3)

y = c cosh (x/c)

(4)

To = wc

(5)

T = wy

(6)

where

T
s

(x, y) To

w = weight of cable per unit length of cable,


and

c
x

W = weight of length of cable from low


point to a point a distance s along
the cable.
Other quantities are defined in the figure to the
right.

1147

tension
in cable
horizontal
component
of tension

6.7 Cables: Catenaries Example 1, page 3 of 4


sB
A

(140 m)/2
70 m
B

2 The sag, h, can be found from Eq. 2, provided that we


can determine the distance, c:
yB2

Sag
xB

(120 m)/2
60 m

sB2 = c2

(Eq. 2 evaluated at point B)

or,
(h + c)2

yB

(70 m)2 = c2

(7)

The distance, c, can be determined from Eq. 1:

sB = c sinh (xB/c)
x

(Eq. 1 evaluated at point B)

or,
70 m = c sinh (60 m/c)

120 m

(8)

This equation must be solved numerically for c. An initial


estimate for c, when the solver on a calculator is to be
used, could be
c = sB = 70 m
The solution to Eq. 8 is
c = 61.45 m
Another possible solution is c = 61.45 m, but this has no
physical meaning.

1148

6.7 Cables: Catenaries Example 1, page 4 of 4


3 Substituting c = 61.45 m into Eq. 7 gives
(h + c)2

(70 m) = c2

(h + 61.45 m)2

(Eq. 7 repeated)

The maximum tension, Tmax, occurs where the cable


has its steepest slope, point B ( or point A).

(70 m)2 = (61.45 m)2

Tension
A

Solving gives the sag

h = 154.6 m

max

31.70 m

and
h = 31.70 m

yB

Ans.
c

61.45 m

The negative root has no physical meaning.


x
5 The maximum tension can be calculated from Eq. 6:
Tmax = wyB

(Eq. 6 evaluated at point B)

w is given.
= (3 kg/m)(9.81 m/s2) (31.70 m + 61.45 m)
= 2740 N
= 2.74 kN

1149

Ans.

6.7 Cables: Catenaries Example 2, page 1 of 4


2. A cable is supported at two points 400 ft apart and at
the same elevation. If the sag is 40 ft and the weight per
unit length of the cable is 4 lb/ft, determine the length of
the cable and the tension at the low point, C.
400 ft
40 ft
A

B
C

1150

6.7 Cables: Catenaries Example 2, page 2 of 4


1

Many problems involving catenary cables can be


solved using the following formulas:
s = c sinh (x/c)

(1)

y2

(2)

s2 = c2

W = ws

(3)

y = c cosh (x/c)

(4)

To = wc

(5)
y

T = wy

(6)

where

(x, y) To

w = weight of cable per unit length of cable,


and

c
x

W = weight of length of cable from low


point to a point a distance s along
the cable.
Other quantities are defined in the figure to the
right.

1151

tension
in cable
horizontal
component
of tension

6.7 Cables: Catenaries Example 2, page 3 of 4


3 The distance c can be determined from Eq. 4:

2 The length of cable, sB, from the low point to point B can be
found from Eq. 1, provided that we can determine the
distance c:
sB = c sinh (xB/c)

yB = c cosh (xB/c)

(Eq. 1 evaluated at point B)

= c sinh (200 ft/c)

y
sB

or,
c + 40 ft = c cosh (200 ft/c)

(7)

(8)

This equation must be solved numerically. An initial


estimate for c, when the solver on a calculator is to be
used, could be

400 ft
A

(Eq. 4 evaluated at point B)

B
xB

c = sag = 40 ft.

(400 ft)/2

Sag

200 ft

40 ft

c
xB

200 ft
c

yB

1152

6.7 Cables: Catenaries Example 2, page 4 of 4


4

The solution to Eq. 8 is


c = 506.53 ft
Using this value of c in Eq. 7 gives
sB = c sinh (200 ft/c)

(Eq. 7 repeated)

= (506.53 ft) sinh (200 ft/506.53 ft)


= 205.237 ft
By symmetry, the total length of cable is twice sB:
sTotal = 2

205.237 ft

= 410 ft

Ans.

Because the tension at the low point of the cable is


horizontal, it can be found from Eq. 5:
To = wc
w is given.

(Eq. 5 repeated)

= (4 lb/ft)(506.53 ft)
= 2,025 lb

Ans.

1153

6.7 Cables: Catenaries Example 3, page 1 of 4


3. A 20-m chain is suspended between two points at
the same elevation and with a sag of 6 m as shown.
If the total mass of the chain 45 kg, determine the
distance between the supports. Also determine the
maximum tension.

B
6m

1154

6.7 Cables: Catenaries Example 3, page 2 of 4


1 Many problems involving catenary cables can be
solved using the following formulas:
s = c sinh (x/c)

(1)

y2

(2)

s2 = c2

W = ws

(3)

y = c cosh (x/c)

(4)

To = wc

(5)

T = wy

(6)

where

T
s

(x, y) To

w = weight of cable per unit length of cable,


and

c
x

W = weight of length of cable from low


point to a point a distance s along
the cable.
Other quantities are defined in the figure to the
right.

1155

tension
in cable
horizontal
component
of tension

6.7 Cables: Catenaries Example 3, page 3 of 4


B

sB
y

A
sB

(20 m)/2

10 m
B

10 m
Sag

6m

xB

yB

x
2 The distance between the supports is 2xB, and xB can be
found from Eq. 1, provided that we can determine the
distance c;
sB = c sinh (xB/c)

c
x
3 The distance, c, can be determined from Eq. 2:

(Eq. 1 evaluated at point B)

yB2

10 m

sB2 = c2

or

This equation can be solved explicitly for xB by


rearranging it as

(6 m + c)2

(10 m)2 = c2

or

sinh (xB/c) = 10 m/c

36 + 12 c + c2

which implies

100 = c2

The c2 terms cancel and the resulting linear equation


has the solution

xB/c = sinh-1(10 m/c)


so

c = 5.333 m
xB = c sinh-1(10 m/c)

(Eq. 2 evaluated at point B)

(7)

1156

6.7 Cables: Catenaries Example 3, page 4 of 4


4 Substituting this value of c into Eq. 7 gives
xB = c sinh-1(10 m/c)

(Eq. 7 repeated)

= (5.333 m) sinh-1(10 m/5.333 m)


= 7.393 m
Thus the distance between supports, 2xB, can be found:
2xB = 2(7.393 m)
= 14.786 m

Ans.

The maximum tension occurs where the slope of the cable is a


maximum, at end B (or at A). Eq. 6 gives
Tmax = wyB

T
A

(Eq. 6 evaluated at point B)


Sag

Total weight of the cable


y
Total length of the cable B

(45 kg)(9.81 m/s2)


(6 m + 5.333 m)
20 m

= 250 N

Tmax

6m
yB

5.333 m
x

Ans.

1157

6.7 Cables: Catenaries Example 4, page 1 of 6


4. A certain cable will break if the maximum tension
exceeds 500 N. If the cable is 50-m long and has a
mass of 50 kg, determine the greatest span possible.
Also determine the sag.

Supports A and B are at the same elevation.

1158

6.7 Cables: Catenaries Example 4, page 2 of 6


1 Many problems involving catenary cables can be
solved using the following formulas:
s = c sinh (x/c)

(1)

y2

(2)

s2 = c2

W = ws

(3)

y = c cosh (x/c)

(4)

To = wc

(5)

T = wy

(6)

y
where

T
s

(x, y) To

w = weight of cable per unit length of cable,


and
c
x

W = weight of length of cable from low


point to a point a distance s along
the cable.
Other quantities are defined in the figure to the
right.

1159

tension
in cable
horizontal
component
of tension

6.7 Cables: Catenaries Example 4, page 3 of 6


2 The maximum tension has been specified (500 N), so
a good place to start our solution is to see how we can
use the fact that Tmax = 500 N. Eq. 6 relates the
tension, T, to the y coordinate of a point on the curve:
T = wy

(Eq. 6 repeated)

Since Tmax occurs where the curve has its steepest


slope, Eq. 6 gives Tmax at point B:
Tmax = wyB

(Eq. 6 evaluated at point B)


T

3 Thus because we know the maximum tension, we can


compute yB:

Tmax

yB = Tmax
w

=
yB
=

Tmax
Total weight of the cable
Total length of the cable
500 N
(50 kg)(9.81 m/s2)

Given

= 50.97 m

1160

50 m
(7)

6.7 Cables: Catenaries Example 4, page 4 of 6


4 The distance between supports is 2xB, so we need to use the
value of yB to determine xB. This can be done by using Eq. 4,
provided that we can determine c:
yB = c cosh (xB/c)

(Eq. 4 evaluated at point B)

We can solve this equation explicitly for xB by rewriting it as

2 xB
(By symmetry)
y

cosh (xB/c) = yB/c


so
xB/c = cosh-1(yB/c)
c

or,
xB = c cosh-1(yB/c)

xB

yB

(8)
x

1161

6.7 Cables: Catenaries Example 4, page 5 of 6


5 The distance, c, can be calculated from Eq. 2:
yB2

sB2 = c2

(Eq. 2 evaluated at point B)

sB

or,

A
(50.97 m)2

(50 m)/2

25 m

(25 m)2 = c2

The solution is
c = 44.42 m
The negative root has no physical meaning.

yB

50.97 m

Substituting c = 44.42 m and yB = 50.97 m in Eq. 8


gives
xB = c cosh-1(yB/c)

(Eq. 8 repeated)

-1

= (44.42 m) cosh (50.97 m/44.42 m)


= 23.836 m
So the distance between supports, 2xB, is known:
2xB = 2(23.836 m)
= 47.7 m

Ans.

1162

6.7 Cables: Catenaries Example 4, page 6 of 6


6 Since c and yB are known, the sag h can be computed:
h = yB

= (50.97 m)

(44.42 m)

= 6.55 m
A

Ans.
y

B
Sag

44.42 m

yB

50.97 m

1163

6.7 Cables: Catenaries Example 5, page 1 of 4


5. The cable is attached to a fixed support at A and a
moveable support at B. If the cable is 80-ft long, weighs
0.3 lb/ft, and spans 50 ft, determine the force F holding
the moveable support in place. Also determine the sag.

F (force)

1164

6.7 Cables: Catenaries Example 5, page 2 of 4


1

Many problems involving catenary cables can be


solved using the following formulas:
s = c sinh (x/c)

(1)

y2

(2)

s2 = c2

W = ws

(3)

y = c cosh (x/c)

(4)

To = wc

(5)

T = wy

(6)

where

(x, y) To

w = weight of cable per unit length of cable,


and

c
x

W = weight of length of cable from low


point to a point a distance s along
the cable.
Other quantities are defined in the figure to the
right.

1165

tension
in cable
horizontal
component
of tension

6.7 Cables: Catenaries Example 5, page 3 of 4


2

The force F acting on the moveable support at B equals the


horizontal component, T , of tension in the cable, F = T .
Eq. 5 can be used to calculate T , provided that we can
determine the distance, c:
T = wc

(Eq. 5 repeated)
Given

= (0.3 lb/ft)c

(7)
sB

The distance, c, can be calculated from Eq. 1:


sB = c sinh (xB/c)

(80 ft)/2

(Eq. 1 evaluated at point B)


c

or,

x
40 ft = c sinh (25 ft/c)

xB

(8)

This equation must be solved numerically. An initial


estimate for c could be c = xB = 25 m, when finding c with
the use of the solver on a calculator. The solution to Eq. 8 is
c = 14.229 ft
The negative root has no physical meaning.

1166

50 ft

(50 ft)/2
25 ft

40 ft

6.7 Cables: Catenaries Example 5, page 4 of 4


3 Using c = 14.229 ft in Eq. 7 gives
T = (0.3 lb/ft)c

(Eq. 7 repeated)

= (0.3 lb/ft)(14.229 ft)


= 4.27 lb
Since F = T , we have
F = 4.27 lb

Ans.
y

The sag, h, can be calculated from Eq. 4 and the known


value of c:

xB

A
h = yB

25 ft

c
Sag

By Eq. 4

h
yB

= c cosh (xB/c)

= (14.229 ft) cosh (25 ft/14.229 ft)


= 28.2 ft

14.229 ft

14.229 ft
x

Ans.

1167

6.7 Cables: Catenaries Example 6, page 1 of 4


6. The cable is attached to a fixed support at
A and a moveable support B. If the cable is
40-m long and has a mass of 0.4 kg/m,
determine the span and sag.
A

50 N

1168

6.7 Cables: Catenaries Example 6, page 2 of 4


1 Many problems involving catenary cables can be
solved using the following formulas:
s = c sinh (x/c)

(1)

y2

(2)

s2 = c2

W = ws

(3)

y = c cosh (x/c)

(4)

To = wc

(5)

T = wy

(6)

where

T
s

(x, y) To

w = weight of cable per unit length of cable,


and

c
x

W = weight of length of cable from low


point to a point a distance s along
the cable.
Other quantities are defined in the figure to the
right.

1169

tension
in cable
horizontal
component
of tension

6.7 Cables: Catenaries Example 6, page 3 of 4


2 The span is 2xB. The coordinate, xB, can be found from
Eq. 1, provided that the distance c can be found:
sB = c sinh (xB/c)

(Eq. 1 evaluated at point B)

This equation can be rewritten to give xB explicitly:


sinh (xB/c) = sB/c
so
A

xB/c = sinh-1 (sB/c)

2 xB
(by symmetry)
y

B
sB

or
xB = c sinh-1 (sB/c)
= c sinh-1 (20 m/c)

xB

(7)
x

1170

(40 m)/2

20 m

6.7 Cables: Catenaries Example 6, page 4 of 4


3 Because the 50-N force acting on the moveable
support equals the horizontal component, T , of the
tension in the cable, Eq. 5 with T = 50 N can be used
to solve for c:
T = wc

(Eq. 5 repeated)

or
50 N = (0.4 kg/m)(9.81 m/s2) c
Solving gives
c = 12.742 m
Using this value of c in Eq. 7 gives
xB = c sinh-1 (20 m/c)

(Eq. 7 repeated)

= (12.472 m) sinh-1 (20 m/12.472 m)


= 15.708 m
So the span is
Span = 2xB
= 2(15.708 m)
= 31.4 m

Ans.

1171

6.7 Cables: Catenaries Example 7, page 1 of 4


7. A cable goes over a frictionless pulley at B and
supports a block of mass m. The other end of the cable
is pulled by a horizontal force P. If the cable has a mass
per length of 0.3 kg/m, determine values of P and m that
will maintain the cable in the position shown.
B

5m
A

P (horizontal)
10 m

1172

6.7 Cables: Catenaries Example 7, page 2 of 4


1 Many problems involving catenary cables can be
solved using the following formulas:
s = c sinh (x/c)

(1)

y2

(2)

s2 = c2

W = ws

(3)

y = c cosh (x/c)

(4)

To = wc

(5)

T = wy

(6)

where

(x, y) To

w = weight of cable per unit length of cable,


and

c
x

W = weight of length of cable from low


point to a point a distance s along
the cable.
Other quantities are defined in the figure to the
right.

1173

tension
in cable
horizontal
component
of tension

6.7 Cables: Catenaries Example 7, page 3 of 4


y

2 The force P equals T , the horizontal component of


the cable tension given by Eq. 5:

xB = 10 m
T = wc

(Eq. 5 repeated)

so, with T = P
P = wc

5m

(7)

Here,

A
w = (0.3 kg/m)(9.81 m/s )
= 2.943 N/m

yB

(8)

The value of c in Eq. 7 can be found from Eq. 4:


yB = c cosh (xB/c)

(Eq. 4 repeated)

or,
5 m + c = c cosh (10 m/c)
Solving numerically gives

Using this value of c in Eq. 7 gives


P = wc

(Eq. 7 repeated)
by Eq. 8

c = 10.743 m

= (2.943 N/m)(10.743 m)
= 31.617 N

1174

Ans.

6.7 Cables: Catenaries Example 7, page 4 of 4


Pulley B

4 The cable tension at B must equal the weight, mg:


TB = mg
TB

Thus the mass is


m = TB/g

mg
by Eq. 6

TB

= wyB/g

mg

by Eq. 8
5m

= (2.943 N/m)(5 m + 10.743 m) /9.81 N/m2


= 4.72 kg

Ans.

yB

10.743 m

1175

6.7 Cables: Catenaries Example 8, page 1 of 5


8. A chain makes angles of 30 and 60 at its
supports as shown. Determine the location of the
low point C of the chain relative to A. Also
determine the tension at support A, if the cable has
a mass per length of 0.6 kg/m.
Supports A and B
are at different
elevation.

60
B

30
A

20 m

1176

6.7 Cables: Catenaries Example 8, page 2 of 5


1 Many problems involving catenary cables can be
solved using the following formulas:
s = c sinh (x/c)

(1)

y2

(2)

s2 = c2

W = ws

(3)

y = c cosh (x/c)

(4)

To = wc

(5)

T = wy

(6)

where

T
s

(x, y) To

w = weight of cable per unit length of cable,


and

c
x

W = weight of length of cable from low


point to a point a distance s along
the cable.
Other quantities are defined in the figure to the
right.

1177

tension
in cable
horizontal
component
of tension

6.7 Cables: Catenaries Example 8, page 3 of 5


y

2 The geometric data are shown in the figure. To


determine the location of the low point C relative to A,
we need to determine the coordinates xA and yA. We
can get an equation for xA by using the fact that the
slope is known at A:
dy
tan 30 =
dx A

60
B (xB, yB)

30
A (xA, yA)

by Eq. 4

d
= dx c cosh (x/c)
A

= sinh (xA/c)
Solving for xA gives
xA = c sinh-1( tan 30)

A
(7)

Similarly at point B, we have


xB = c sinh-1(tan 60)

(8)

1178

dx
30

20 m
dy (Slope is negative)

6.7 Cables: Catenaries Example 8, page 4 of 5


3

The coordinates xA and xB are related to the 20-m span


through the equation
xB

60
B (xB, yB)

xA = 20 m
30
A (xA, yA)

or,
by Eq. 8
c sinh-1(tan 60)

by Eq. 7

c sinh-1( tan 30) = 20

Since this equation is linear in c, it is easily solved to


give c = 10.717 m. Eq. 7 then gives
xA = c sinh-1( tan 30)

(Eq. 7 repeated)
20 m

= (10.717 m) sinh-1( tan 30)


= 5.887 m

Ans.

The y coordinate of point A can now be calculated from


Eq. 4:
yA = c cosh (xA/c) (Eq. 4 evaluated at point A)
= (10.717 m) cosh ( 5.887 m/10.717 m)
= 12.375 m

(9)

1179

6.7 Cables: Catenaries Example 8, page 5 of 5


4 The vertical distance between support A and the low
point C is given by

y
B

5.887 m
d = yA c
d

by Eq. 9
= 12.375 m

10.717 m

= 1.658 m

Ans.

yA

c = 10.717 m

The tension at A is given by Eq. 6:


TA = wyA
(w given)

(Eq. 6 evaluated at point A)


by Eq. 9

= (0.6 kg/m)(9.81 m/s2) (12.375 m)


= 72.8 N

Ans.

1180

6.7 Cables: Catenaries Example 9, page 1 of 4


9. A wire weighing 0.2 lb/ft is attached to a moveable
support at A and makes an angle of 55 at a fixed
support at B. Supports A and B are at different
elevations. Determine the location of the low point C
of the wire relative to support B. Also, determine the
tension in the wire at C.

A
8 lb
B

55

1181

6.7 Cables: Catenaries Example 9, page 2 of 4


1 Many problems involving catenary cables can be
solved using the following formulas:
s = c sinh (x/c)

(1)

y2

(2)

s2 = c2

W = ws

(3)

y = c cosh (x/c)

(4)

To = wc

(5)

T = wy

(6)

where

T
s

(x, y) To

w = weight of cable per unit length of cable,


and

c
x

W = weight of length of cable from low


point to a point a distance s along
the cable.
Other quantities are defined in the figure to the
right.

1182

tension
in cable
horizontal
component
of tension

6.7 Cables: Catenaries Example 9, page 3 of 4


y

2 To determine the location of the low point, C, relative to


the support at B, we need to determine the coordinates xB
and yB. We can get an equation for xB by using the fact
that the slope is known at B:
dy
tan 55 =
dy
55
dx B
B
dx
by Eq. 4
d
= dx c cosh (x/c)
B

A
55
B (xB, yB)

C
c
x

= sinh (xB/c)
Thus
xB = c sinh-1( tan 55)

(7)
A

The value of c occurring in Eq. 7 can be found by


observing that the 8-lb force acting at support A equals T ,
the horizontal component of tension at A, so Eq. 5 gives

To

8 lb
B

T = wc
8 lb

(Eq. 5 repeated)
w

0.2 lb/ft

Solving gives
c = 40 ft

0.2 lb/ft

(8)

1183

6.7 Cables: Catenaries Example 9, page 4 of 4


3

Using this result, c = 40 ft, in Eq. 7 gives


xB = c sinh-1(tan 55)

A
(Eq. 7 repeated)

xB

46.169 ft

-1

= (40 ft) sinh (tan 55)


= 46.169 ft

B
Ans.

The vertical distance between B and C is

yB

C
d = yB

c = 40 ft

by Eq. 4
= c cosh (xB/c)

x
c

= (40 ft) cosh (46.169 ft/40 ft)


= 29.7 ft

40 ft
Ans.
Free body diagram of portion AC of cable

Since point C is the low point of the cable, the tension


there is horizontal and so must equal the horizontal
component of tension at A which is known to be 8 lb.
That is,
TC = 8 lb

A
8 lb
Weight of portion
AC of cable

Ans.
Vertical force
acting on support

1184

TC

6.7 Cables: Catenaries Example 10, page 1 of 6


10. Determine the location of the low point C relative to
the support at A. Also determine the tension at C, if the
mass of the cable per unit length is 1 kg/m.
A
B

2m
1m

C
10 m

1185

6.7 Cables: Catenaries Example 10, page 2 of 6


1 Many problems involving catenary cables can be
solved using the following formulas:
s = c sinh (x/c)

(1)

y2

(2)

s2 = c2

W = ws

(3)

y = c cosh (x/c)

(4)

To = wc

(5)

T = wy

(6)

where

T
s

(x, y) To

w = weight of cable per unit length of cable,


and

c
x

W = weight of length of cable from low


point to a point a distance s along
the cable.
Other quantities are defined in the figure to the
right.

1186

tension
in cable
horizontal
component
of tension

6.7 Cables: Catenaries Example 10, page 3 of 6


2 The geometric data are shown in the figure.
To locate the low point, C, relative to support A, we
must determine the coordinate xA. We know that xA is
related to yA through Eq. 4:

A(xA, yA)
2m+1m

y
B(xB, yB)

3m

1m

yA = c cosh (xA/c) (Eq. 4 evaluated at point A)


And this can be solved for xA to give
-1

xA = c cosh (yA/c)

c
(7)

The minus sign must be inserted here to make xA


negative because we will consider "cosh-1 " to be the
principal value of the inverse cosh function, and the
principal value is always positive.

x
10 m

cosh-1 v
(principal value)

cosh u

Branch of cosh function


corresponding to the
principal value of the inverse
function
u

v
1
Note that cosh-1 v is not defined for v < 1

1187

6.7 Cables: Catenaries Example 10, page 4 of 6


3

An equation analogous to Eq. 7 can be written for point B:


xB = c cosh-1(yB/c)

(8)
A

Here no minus sign has to be inserted, because xB is positive.


The figure shows that
xB

y
B

3m

By Eq. 8

yA

By Eq. 7
-1

c cosh (yB/c)

1m

xA = 10 m

-1

c cosh (yA/c) = 10 m

yB

or
c cosh-1(

1m+c
c

+ c cosh-1( 3 mc + c

10 m

10 m = 0

(9)

This equation must be solved numerically for c. It can be difficult to


solve with the solver on a calculator because the cosh-1 (u) function is
undefined for u 1, and, as a result, the solver may fail if, during the
solution procedure, an attempt is made to evaluate cosh-1 (u) for u 1.

1188

6.7 Cables: Catenaries Example 10, page 5 of 6


4

One approach to finding the solution of Eq. 9 is to define


the left-hand side of the equation as a function,
f(z)

z cosh-1 (

1+z
z

) + z cosh-1 ( 3 +z z )

10

And then use the calculator to plot f(z) versus z. The point
z = zr where the f(z) curve crosses the z axis is the
approximate root of Eq. 9, that is, where f(zr) = 0.
Another approach to finding the solution of Eq. 9 is simply
to experiment with initial estimates for c while using the
solver on a calculator. For example, since the span was
given as 10 m, we might try c = 10 m, 50 m, 100 m as
successive initial estimates.
The solution to Eq. 9 is
A

c = 7.044 m

y
B

3m

Eq. 7 then gives

C
xA = c cosh (yA/c)
= c cosh ( 3 + c
c

(Eq. 7 repeated)
yA

c = 7.044 m

= (7.044 m) cosh ( 3 + 7.044 m )


7.044 m
= 15.5 m
Ans.

1189

6.7 Cables: Catenaries Example 10, page 6 of 6


A

5 At the low point of the cable, the tension is horizontal, so


it can be calculated from Eq. 5:

y
B

To = wc

(Eq. 5 repeated)

To

Given
= [(1 kg/m)(9.81 m/s2)](7.044 m)
= 69.1 N

7.044 m

Ans.
x

1190

7. Friction

1191

7.1 Friction: Basic Applications

1192

7.1 Friction: Basic Applications Procedures and Strategies, page 1 of 1


Procedures and Strategies for Solving Problems Involving Friction:
Basic Applications

1. Draw free-body diagrams, taking care to choose the sense of the


friction forces so that they oppose actual or impending relative
motion at surfaces of contact.

2. Write equilibrium equations for each free body.


3. Count the number of unknowns and compare it with the number of
equilibrium equations.
a) If the numbers are equal, solve the equations and then compare
each friction force, f i, with the maximum possible friction force
that the surfaces can produce, f i-max
Ni. If f i < f i-max, then the
bodies are in equilibrium, if f i > f i-max, then the friction forces
are not large enough to keep the bodies from moving. The
equilibrium equations cannot be satisfied.
b) If the number of unknowns is greater than the number of
equilibrium equations, then you must obtain additional equations
by assuming slip impends at some surfaces (so f i = f i-max at
surface i) or perhaps tipping impends (thus, for example, the
location of a normal force is known to be at a corner). Solve the
entire set of equilibrium and friction equations, and then check
that your solution is physically possible, for example, that
f i f i-max everywhere. If the solution is possible, you are done.
Otherwise, you must consider a different case of impending
motion, for example, tipping rather than sliding.

1193

W
Tipping does not
impend, so x > 0.
N
Assume sliding
x
impends, so f = f max
( N)

P
Tipping impends,
so x = 0, and the
normal force N
acts at the corner.

W
f
N

7.1 Friction: Basic Applications Problem Statement for Example 1


1. The uniform ladder is 2-m long and makes an angle
of = 60 with the floor. If the wall at B is smooth and
the coefficient of static friction at A is A= 0.3,
determine if the ladder can remain in the position shown.

B
Mass of ladder = 10 kg

1194

7.1 Friction: Basic Applications Problem Statement for Example 2


2. The uniform ladder is 2-m long and the wall at B
is smooth. If the coefficient of static friction at A is
A = 0.2, determine the smallest angle for which
the ladder can remain in the position shown.

B
Mass of ladder = 10 kg

1195

7.1 Friction: Basic Applications Problem Statement for Example 3


3. The uniform ladder is 2-m long. The
coefficient of static friction at A is A = 0.6 and
at B is B = 0.4. Determine the smallest angle, ,
for which the ladder can remain in the position
shown.

B
Mass of ladder = 10 kg

1196

7.1 Friction: Basic Applications Problem Statement for Example 4


4. Four round pegs A, B, C, and D are attached to
the bracket and loosely straddle the vertical pole.
When a 100-N force is applied as shown, the
bracket rotates slightly and friction forces
develop between pegs B, C, and the pole. If the
coefficient of static friction between the pegs and
the pole is determine the smallest value of
for which the bracket will support the load.
Neglect the effect of the rotation of the bracket
on the distances shown.
100 N
50 mm
D

B
100 mm

300 mm

1197

7.1 Friction: Basic Applications Problem Statement for Example 5


5. Arm ABC acts as a brake on the wheel. The
coefficient of static friction at B is B = 0.4.
Determine the largest moment M that can act on the
wheel without causing it to slip.
Radius = 200 mm

100 N

M
O

300 mm
B
A

400 mm

200 mm

1198

7.1 Friction: Basic Applications Problem Statement for Example 6


6. The uniform block is initially at rest when a 10-lb
force is applied. The coefficient of static friction
between the block and the plane is = 0.6.
Determine if the block will move.
1 ft

10 lb

20 lb
(weight)

2 ft

1199

7.1 Friction: Basic Applications Problem Statement for Example 7


7. The uniform block is initially at rest when the force P
is applied. The coefficient of static friction between the
block and the plane is = 0.6. Determine the minimum
value of P that will cause the block to move.
1 ft

20 lb
(weight)

2 ft

1200

7.1 Friction: Basic Applications Problem Statement for Example 8


8. The cylinder is intially at rest when a horizontal
force P is applied. The coefficients of static friction
at A and B are A = 0.3 and B = 0.6. Determine
the minimum value of P that will cause the cylinder
to move.
Radius = 0.2 m
P

0.3 m

B
20 kg
A

1201

7.1 Friction: Basic Applications Problem Statement for Example 9


9. The small block B rests on top of the large block A. The
coefficients of static friction are shown in the figure.
Determine the smallest value of applied force P that will
keep block A from sliding down the inclined plane.
Frictionless
pulley
Cord
B

= 0.3
= 0.2

B
10 kg
60 kg
P

30

1202

7.1 Friction: Basic Applications Problem Statement for Example 10


10. The three blocks are stationary when the force P is applied. The
coefficients of static friction for each pair of surfaces are given in the
figure. Determine the smallest value of P for which motion will
occur. The blocks are sufficiently long that tipping will not occur.
P

10 kg

AB

= 0.8

10 kg

BC

= 0.3

10 kg

= 0.15

1203

7.1 Friction: Basic Applications Problem Statement for Example 11


11. The two cylinders shown are initially at rest when
horizontal forces of magnitude P/2 are applied to the ends
of the axle in the lower cylinder. The coefficients of
static friction for each pair of surfaces are given in the
figure. Determine the largest value of P that can be
applied without moving the cylinders up the inclined
plane.
P/2
Radius of each cylinder = 300 mm
B

C = 0.5

= 0.4
B

Mass of each cylinder = 50 kg

C
A
A

= 0.6

P/2
25

1204

7.1 Friction: Basic Applications Example 1, page 1 of 2


1. The uniform ladder is 2-m long and makes an angle of = 60
with the floor. If the wall at B is smooth and the coefficient of
static friction at A is A = 0.3, determine if the ladder can remain
in the position shown.

B
Mass of ladder = 10 kg

Free-body diagram

No friction force is
present because the
wall is smooth.

NB

Fx = 0: NB

fA = 0

(2 m)(sin 60) = 1.732 m

Fy = 0: NA

98.1 N = 0

MA = 0: (98.1 N)(0.5 m)

1m

A
Weight = mg
2
= (10 kg)(9.81 m/s )
60
= 98.1 N

fA

4 Impending motion

NA
5

(1 m)(cos 60) = 0.5 m

Equations of equilibrium:

B
1m
Because the ladder is
uniform, the weight
acts through the center.

The friction force must


be drawn in a direction
opposing the
impending motion.

NB(1.732 m) = 0

Results of solving the above


equations of equilibrium:
NA = 98.1 N
NB = 28.3 N
f A = 28.3 N

1205

7.1 Friction: Basic Applications Example 1, page 2 of 2


8

Compute the maximum possible friction force that


the surfaces can develop at point A.
f A-max

A NA

= (0.3)(98.1 N) = 29.4 N

To determine whether or not the ladder will stay in


the original position, the friction force found from the
equilibrium equations, f A = 28.3 N, must be
compared with the maximum force that the surfaces
at A can develop:
f A-max = 29.4 N
Since
f A = 28.3 N < 29.4 N = f A-max
the surfaces are able to develop enough friction force
and the ladder will stay in equilibrium.

1206

7.1 Friction: Basic Applications Example 2, page 1 of 2


2. The uniform ladder is 2-m long and the wall at B
is smooth. If the coefficient of static friction at A is
A = 0.2, determine the smallest angle for which
the ladder can remain in the position shown.

2
1

Free-body diagram

Mass of ladder = 10 kg

NB

No friction force is
present because the
wall is smooth.

1m
(2 m) sin
1m

Equations of equilibrium:
Fx = 0: NB

fA = 0

Fy = 0: NA

98.1 N = 0

MA = 0: (98.1 N)(1 m) cos

Weight = mg
= (10 kg)(9.81 m/s2)
= 98.1 N
(1)

fA
NA

Impending motion

(2)
NB(2 m) sin

=0

(3)

Three equations, but four unknowns: NB, f A, NA, and


An additional equation is needed.

1207

4
(1 m) cos

The friction force


must be drawn in a
direction opposing the
motion.

7.1 Friction: Basic Applications Example 2, page 2 of 2


The fourth equation comes from the condition of
impending slip at point A, because if slip is just about to
occur, then the friction force, f A, is at its maximum value,
which is ANA:
fA

f A-max

A NA

= (0.2)NA

Using f A = 19.62 N in Eq. 1 gives


NB = f A
= 19.62 N

(4)
Using NB = 19.62 N in Eq. 3 gives

Three of the four equations are linear but the moment


equation, Eq. 3, is nonlinear because cos and sin
appear.

(98.1 N) cos

(19.62 N)(2) sin

Dividing through by cos

MA = 0: (98.1 N)(1m) cos


- NB(2 m) sin = 0

sin
cos

(Eq. 3 repeated)

The easiest way to solve these equations is to use the


general equation solver on a calculator. Alternatively,
manipulate the equations as follows.

=0

and rearranging gives

98.1 = 2.5
(19.62)(2)

Replacing the left-hand side of this equation by tan


gives
tan

First note that Eq. 2 implies that

= 2.5

which implies

NA = 98.1 N

68.2

Then using this value for NA in Eq. 4 gives


f A = NA
= (0.2)(98.1 N)
= 19.62 N

1208

Ans.

7.1 Friction: Basic Applications Example 3, page 1 of 2


3. The uniform ladder is 2-m long. The
coefficient of static friction at A is A = 0.6 and
at B is B = 0.4. Determine the smallest angle, ,
for which the ladder can remain in the position
shown.

3
1

Free-body diagram

Friction force, f B, is
present and opposes
the possible motion.

B
NB

Mass of ladder = 10 kg

1m

Impending
motion
(2 m) sin

fB
1m
Weight = mg
= (10 kg)(9.81 m/s2)
= 98.1 N

fA

Equations of equilibrium:
+

NA
Fx = 0: NB

fA = 0

(1)

Fy = 0: NA + f B

(1 m) cos
98.1 N = 0

MA = 0: (98.1 N)(1 m) cos


NB(2 m) sin = 0

(2)

(2 m) cos

f B(2 m) cos
(3)

6 Three equilibrium equations but five unknowns: NA,


NB, f A, f B, and two more equations are needed.

1209

Friction force
opposes the
possible motion.

7.1 Friction: Basic Applications Example 3, page 2 of 2


7

The two additional equations come from the condition of


impending slip at points A and B, because if slip is just about
to occur, then the friction forces, f A and f B, are at their
maximum values, ANA and NB
fA

f A-max

A NA

fB

f B-max

BNB

= 0.6NA

(4)

= 0.4NB

(5)

Four of the five equations are linear but the moment equation
Eq. 3 is nonlinear (sin and cos are present). To solve these
equations, use the general equation solver on your calculator or
manipulate the equation algebraically and use a trig identity
such as tan = sin /cos .
Results of solving the five equations (three equilibrium and
two friction equations) given above:
NA = 79.1 N
NB = 47.5 N
f A = 47.5 N
f B = 19.0 N
= 32.3

Ans.

1210

7.1 Friction: Basic Applications Example 4, page 1 of 3


4. Four round pegs A, B, C, and D are attached to
the bracket and loosely straddle the vertical pole.
When a 100-N force is applied as shown, the
bracket rotates slightly and friction forces
develop between pegs B, C, and the pole. If the
coefficient of static friction between the pegs and
the pole is determine the smallest value of
for which the bracket will support the load.
Neglect the effect of the rotation of the bracket
on the distances shown.

Movement of bracket exaggerated for clarity

100 N
Bracket rotates a
small amount

Peg D loses contact with the pole


50 mm
100 N
D

B
100 mm
D

300 mm

Peg A loses contact with the pole

1211

7.1 Friction: Basic Applications Example 4, page 2 of 3


Free-body diagram

The friction forces, f B and f C,


resist the motion by pushing the
bracket up.

P = 100 N

B
Impending
motion of
bracket

Equations of equilibrium:

NB
100 mm

fB
C

NC
5

fC
300 mm

As the bracket inclines slightly, the pegs at A and D


lose contact with the pole. That is why no forces
appear at A and D on the free-body diagram.

50 mm

The normal forces,


NB and NC, are
directed from the
pole to the pegs .

1212

Fx = 0: NB

NC = 0

Fy = 0: f B + f C

100 N = 0

MC = 0: (100 N)(300 mm) + f B(50 mm)


NB(100 mm) = 0

(1)
(2)

(3)

7.1 Friction: Basic Applications Example 4, page 3 of 3


8

There are only three equations of equilibrium but four


unknowns (f B, NB, f C, and NC), so at least one more
equation is needed. The additional equation comes from the
condition of impending slip at B, but if the bracket is going
to slip at B, it will also slip at C. So we have two additional
equations and one additional unknown, :
f B = f B-max

NB

(4)

f C = f C-max

NC

(5)

Solving Eqs. 1-5 gives the results below (Note that Eqs. 4
and 5 are nonlinear because multiplies NB and NC):
f B = 50 N
NB = 325 N
f C = 50 N
NC = 325 N
= 0.154

Ans.

1213

7.1 Friction: Basic Applications Example 5, page 1 of 2


5. Arm ABC acts as a brake on the wheel. The
coefficient of static friction at B is B = 0.4.
Determine the largest moment M that can act on the
wheel without causing it to slip.
C

100 N

M
O

Free-body diagram of
wheel.

M
Ox

300 mm

NB

B
A

Impending motion
of point on outer
surface of wheel

Oy

400 mm

3 The friction force f B


opposes the motion.

fB
200 mm

200 mm

Radius = 200 mm

1214

Equation of moment equilibrium for the wheel


(Since we were not asked to compute the reactions
Ox and Oy, we do not need to write the
force-equilibrium equations.):
MO = 0: f B(200 mm)

M=0

(1)

7.1 Friction: Basic Applications Example 5, page 2 of 2


5
6

The sense of the friction


force on the brake can be
determined by Newton's
Third Law (equal and
opposite to the force on
the wheel).

Free-body diagram of arm ABC.


fB

100 N
300 mm

Ay
NB

400 mm
A
Ax
200 mm

Equation of moment equilibrium for the brake (Since we


were not asked to compute the reactions Ax and Ay, we do
not need to write the force-equilibrium equations.):

8 The third equation follows from the condition


that slip impends at B:

MA = 0: f B(200 mm) NB(400 mm)


+ (100 N)(300 mm + 400 mm) = 0 (2)

fB

Thus far we have two equations but three unknowns (M, f


B, and NB), so another equation is needed.

f B-max

BNB

= 0.4NB

(3)

Solving Eqs. 1-3 simultaneously yields


f B = 87.5 N
NB = 218.8 N
M = 17 500 Nmm = 17.5 Nm

1215

Ans.

7.1 Friction: Basic Applications Example 6, page 1 of 4


6. The uniform block is initially at rest when a 10-lb
force is applied. The coefficient of static friction
between the block and the plane is = 0.6.
Determine if the block will move.
1 ft

10 lb

20 lb
(weight)

2 ft

1216

7.1 Friction: Basic Applications Example 6, page 2 of 4


1

Free-body diagram

Free-body diagram showing resultant forces


10 lb

1 ft
10 lb

20 lb
2 ft
20 lb

fd

The resultant of the


distributed force f d is f.

f
A

0.5 ft

B
3 The distributed friction force f d
opposes possible slip to the
right; the distributed normal
force Nd opposes possible
tipping of the block.

Nd
As the block is pushed to the right by the 10-lb force, the floor opposes the
possible motion by providing a distributed reaction force. The component of
this reaction force parallel to the floor is the distributed friction force f d, and
the component normal to the floor is the distributed normal force Nd.

1217

x
N

The resultant of the distributed force


Nd is N. Because Nd opposes possible
tipping of the block, it is not uniform
but is greater near the right-hand side
of the base of the block to balance the
tendency to tip. Thus the resultant N
does not act at the middle of the base
but instead acts at some unknown
distance, x, from the middle.

7.1 Friction: Basic Applications Example 6, page 3 of 4


7

Equations of equilibrium:
+

Fy = 0: N

Fx = 0: 10 lb

MA = 0:

f=0

20 lb = 0
(20 lb)(0.5 ft)

(10 lb)(2 ft) + N(0.5 ft + x) = 0

Solving these equations gives


f = 10 lb
N = 20 lb
x = 1 ft

9 These are the values required if the system is to


stay in equilibrium, that is, not move. To
determine if the system can produce the 10-lb
friction force f required to keep the system in
equilibrium, we have to compare f with the
maximum possible value of the friction force:
f max

N = (0.6)(20 lb) = 12 lb

Because f = 10 lb is less than the 12 lb maximum


possible force, the surfaces can develop enough
force to balance forces in the x direction (thus the
block will not slide to the right).

1218

7.1 Friction: Basic Applications Example 6, page 4 of 4


10 lb

10 We next consider whether or not the block will tip.


Recall that solving the equilibrium equations gave the result
x = 1 ft. That is, to maintain equilibrium, the normal force
N must act at the location shown, 0.5 ft to the right of the
block. But this is impossible because N is the normal force
from the ground acting up on the block; the farthest N can
act is at the right hand corner, B. Thus the block will tip
because N cannot act far enough to the right to prevent it.

20 lb

f
0.5 ft

0.5 ft

N (impossible location
because outside the base of
the block)

x = 1 ft

1219

7.1 Friction: Basic Applications Example 7, page 1 of 6


7. The uniform block is initially at rest when the force P
is applied. The coefficient of static friction between the
block and the plane is = 0.6. Determine the minimum
value of P that will cause the block to move.
1 ft

20 lb
(weight)

2 ft

1220

7.1 Friction: Basic Applications Example 7, page 2 of 6


1

Free-body diagram

Free-body diagram showing resultant forces


P

1 ft
P

20 lb
2 ft
20 lb

fd

The resultant of the


distributed force f d is f. A
f

B
0.5 ft

x
N

Nd
2

As the block is pushed to the right by the force P, the floor opposes the
possible motion by providing a distributed reaction force. The component
of this reaction force parallel to the floor is the distributed friction force, f d,
and the component normal to the floor is the distributed normal force Nd.

1221

The resultant of the distributed force Nd


is N. Because Nd opposes possible
tipping of the block, it is not uniform
but is greater near the right-hand side of
the base of the block to balance the
tendency to tip. Thus the resultant N
does not act at the middle of the base
but instead acts at some unknown
distance, x, from the middle.

7.1 Friction: Basic Applications Example 7, page 3 of 6


6
+

Fx = 0: P

f=0

(1)

Fy = 0: N

20 lb = 0

(2)

Equations of equilibrium:

MA = 0:

(20 lb)(0.5 ft) P(2 ft)


+ N(0.5 ft + x) = 0

Case 1: Sliding
1 ft

(3)
P

Three equations but four unknowns (P, f, N and x),


so one more equation is needed.

The fourth equation comes from considering


possible impending motion. There are two cases to
consider: sliding and tipping.
20 lb
(weight)

1222

7.1 Friction: Basic Applications Example 7, page 4 of 6


10 Case 2: Tipping

11 We have to analyze each case separately. Let's (arbitrarily)


choose Case 1 first. If sliding impends, then

f = f max

N = 0.6N

12 Solving Eqs. 1-4 simultaneously gives


P = 12 lb
N = 20 lb

20 lb
(weight)

f = 12 lb
x = 1.2 ft

1223

(4)

7.1 Friction: Basic Applications Example 7, page 5 of 6


13 Free body diagram for Case 1 (Sliding impends)
P = 12 lb
14 But this diagram shows that the only way the equilibrium
equations for Case 1 can be satisfied is if the normal force N
lies to the right of the block (x = 1.2 ft). Since this is
impossible, the Case 1 assumption that sliding impends must
be incorrect.

20 lb

N = 20 lb

f = 12 lb
0.5 ft
x = 1.2 ft

1224

7.1 Friction: Basic Applications Example 7, page 6 of 6


15 Free body diagram for Case 2 (Tipping impends)
0.5 ft

0.5 ft
16 Since the block is just about to tip, it loses contact with the
floor except at the corner B, where the normal force N is
concentrated. Since N acts at the corner, we know
x = 0.5 ft

(5)

Solving the equilibrium equations, Eqs. 1, 2, and 3,


simultaneously with Eq. 5 gives

20 lb

f = 5 lb
N = 20 lb
P = 5 lb
B

A
f

Ans.

Since there were only two possibilities, sliding and tipping, and
we eliminated sliding, we know that the above result P = 5 lb is
correct. However, we can also check our work by verifying
that the friction force f is less than the maximum possible
value:
f = 5 lb < f max

1225

N = (0.6)(20 lb) = 12 lb. (OK)

7.1 Friction: Basic Applications Example 8, page 1 of 3


8. The cylinder is initially at rest when a horizontal
force P is applied. The coefficients of static friction
at A and B are A = 0.3 and B = 0.6. Determine
the minimum value of P that will cause the cylinder
to move.
Radius = 0.2 m
P
1
0.3 m

Free-body diagram

B
20 kg
A

Weight = mg
= (20 kg)(9.81 m/s2)
= 196.2 N

0.3 m

fB

A
fA
5 The friction force from the floor
opposes the motion of point A on
the cylinder.

NA

1226

NB

Possible motion of
point B on cylinder.
Force P tends to
rotate the cylinder
clockwise.

0.2 m
3 The friction force from
the wall opposes the
motion of point B on
the cylinder.

Possible motion of point A.

7.1 Friction: Basic Applications Example 8, page 2 of 3


6

Equilibrium equations

Fx = 0: P + f A
Fy = 0:

NB = 0

(1)

196.2 N + f B + NA = 0

(2)

MA = 0: f B(0.2 m) + NB(0.2 m)

P(0.3 m) = 0

Case 2

The cylinder spins


about its center.

f A = f A-max

A NA

= 0.3NA

(4)

f B = f B-max

BNB

= 0.6NB

(5)

(3)

There are three equations and five unknowns (P, f A, NA, f B, NB),
so two more equations are needed. The two additional equations
come from considering possible impending motion. There are
two cases to consider:
Case 1

We have to analyze each case separately. Let's


(arbitrarily) choose Case 1 first. Thus if the cylinder
is about to slip about its center, then slip impends
simultaneously at points A and B, so

Solving Eqs. 1-5 simultaneously gives


P = 554 N
f A = 34.6 N

The cylinder rolls up the


wall without slipping.

f B = 312 N
NA = 115 N
NB = 519 N

B
A

A negative normal force, NA, is impossible (The


floor can't pull down on the cylinder), so the
assumption of slip at both A and B must be wrong.

1227

7.1 Friction: Basic Applications Example 8, page 3 of 3


8

Next consider Case 2 the cylinder is about to roll up the wall. Thus
the cylinder is about to lose contact with the floor at point A, and so
the friction and normal forces there are zero:
fA = 0

(6)

NA = 0

(7)

Solving the equilibrium equations, Eqs. 1, 2, and 3, simultaneously


with Eqs. 6 and 7 gives
fA = 0
NA = 0
f B = 196 N
NB = 392 N
P = 392 N

Ans.

Since there were only two possibilities, spinning about the cylinder
center or rolling up the wall, and we eliminated spinning, the above
result P = 392 N must be correct. However, we can also check our
work by comparing the friction force, f B, with the maximum possible
value:
f B = 196 N < f B-max

BNB

= (0.6)(392 N) = 235 N (OK)

1228

7.1 Friction: Basic Applications Example 9, page 1 of 4


9. The small block B rests on top of the large block A. The
coefficients of static friction are shown in the figure.
Determine the smallest value of applied force P that will
keep block A from sliding down the inclined plane.
Frictionless
pulley
Cord
B

= 0.3
= 0.2

B
10 kg
60 kg
P

30

1229

7.1 Friction: Basic Applications Example 9, page 2 of 4


1

Free-body diagram of block B


Weight = mg
= (10 kg)(9.81 m/s2)
= 98.1 N
y

Tension in cord, T

3
4

Impending motion of
B
block B relative to
block A (If block A
moves down the plane,
block B must move up
the plane.)
N
Normal force B
from block A
5

The numerical value of


will be calculated later.

P
fB

30
x

The friction force from block A opposes the


impending motion of block B up the incline.

Equations of equilibrium for block B. We assume that the blocks will


not tip because they are much longer than they are high; thus no
moment equation is needed (Since no dimensions are given, we could
not write a moment equation even if we wanted to).

Fx =0: P
Fy = 0: NB

T + f B + (98.1 N) sin
(98.1 N) cos

=0

It's convenient to use an inclined


xy coordinate system.

=0

(1)
(2)

1230

7.1 Friction: Basic Applications Example 9, page 3 of 4


7

Geometry
= 90
y

60 = 30

Free-body diagram of block A


13 Weight = mg
= (60 kg)(9.81 m/s2)
= 588.6 N

60
10 Friction force from block B opposes
motion of block A.
30

30
x

fB
A

[Weight of block A alone (Note


that the weight of block B is not
included because block B is not
part of this free-body. The effect
of the weight of block B is
= 30 transmitted through the normal
force, NB.)]
y
NB
9 Impending motion
of block A relative
to block B.

fA
12 Friction force from
inclined plane opposes
NA
motion of block A.
Normal force from
inclined plane
11 Impending motion of block A
relative to inclined plane.

1231

30
x

7.1 Friction: Basic Applications Example 9, page 4 of 4


Free body diagram of block A repeated

14 Equilibrium equations for block A


+

Fx =0: (588.6 N) sin 30

fA

fB

T=0

(3)

fB

T
+

Fy = 0:

(588.6 N) cos 30 + NA

NB = 0

(4)

Four equations in six unknowns (T, P, f A, NA, f B, NB). Two


more equations come from the condition of impending sliding
between the blocks and between block A and the plane:
fA

f A-max

A NA

= 0.2NA

fB

f B-max

BNB = 0.3NB

fA

588.6 N

= 30
y
NB

(5)
NA
(6)

Solving Eqs. 1-6 simultaneously gives


f A = 119 N
NA = 595 N
f B = 25 N
NB = 85 N
T = 150 N
P = 75 N

30

Ans.

1232

7.1 Friction: Basic Applications Example 10, page 1 of 8


10. The three blocks are stationary when the force P is applied. The
coefficients of static friction for each pair of surfaces are given in the
figure. Determine the smallest value of P for which motion will
occur. The blocks are sufficiently long that tipping will not occur.

10 kg

AB

= 0.8

10 kg

BC

= 0.3

10 kg

= 0.15

Free-body diagram of block A


Weight = mg
= (10 kg)(9.81 m/s2)
= 98.1 N

4
+

Fx = 0: P

Fy = 0: NAB

A
f AB
2
NAB

Equilibrium equations for block A:

f AB = 0

(1)

98.1 N = 0

The last equation gives

Impending motion of
block A relative to B

3 Friction opposes the motion

1233

NAB = 98.1 N

(2)

7.1 Friction: Basic Applications Example 10, page 2 of 8


5

Free-body diagram of block B

Impending motion of block B


relative to block A (An observer
on A would see B moving in this
direction.)

NAB = 98.1 N

9 Friction force opposes


relative motion

f AB
B
6

f BC

Impending motion of
block B relative to C

Weight = 98.1 N
NBC

Fx = 0: f AB

10 Equilibrium equations for block B:

Fy = 0:

f BC = 0

98.1 N

(3)

98.1 N + NBC = 0

The last equation gives


NBC = 196.2 N

(4)

1234

Friction force opposes


relative motion

7.1 Friction: Basic Applications Example 10, page 3 of 8


11 Free-body diagram of block C
14 Impending motion of block C
relative to block B (An observer
on B would see C moving in this
direction.)

NBC = 196.2 N
15 Friction force opposes
relative motion
f BC
C
12 Impending motion of
block C relative to
floor

fC
Weight = 98.1 N
NC

13 Friction force opposes


relative motion

Fx = 0: f BC

16 Equilibrium equations for block C:

Fy = 0:

fC = 0

98.1 N

(5)

196.2 N + NC = 0

The last equation gives


NC = 294.3 N

(6)

1235

7.1 Friction: Basic Applications Example 10, page 4 of 8


17

We now have six equilibrium equations but seven


unknowns (P, f AB, NAB, f BC, NBC, f C, NC), so another
equation is needed.

18 The seventh equation comes from the condition of


impending slip. We have to consider three cases:

Case 1
Impending
motion

A
B

Stationary

Case 2
A

Impending motion: blocks A and B move together

B
C

Stationary
Case 3
A
B
C

1236

Impending
motion: blocks
A, B and C
move together

7.1 Friction: Basic Applications Example 10, page 5 of 8


19 Analyze each case separately.
22 For block B, Eq. 3 is
f AB

f BC = 0

20 Case 1
So for equilibrium,

A
B

f BC = f AB

Stationary

= 78.5 N
Let's compare this with the maximum possible friction
force:

21 Slip impends so
f AB = f AB-max
ABNAB

f BC-max

by Eq. 2
= (0.8)(98.1 N) = 78.5 N

(7)

We have to check to see if the surfaces of contact between


blocks B and C develop enough friction force to keep block B
stationary.

1237

BCNBC

= (0.3)(196.2 N) = 58.9 N

(8)

So the surfaces can develop only 58.9 N while 78.5 N


are needed for equilibrium. Thus block B will move,
contrary to our assumption for Case 1.

7.1 Friction: Basic Applications Example 10, page 6 of 8


23 Case 2
A

Impending
motion
together

25 For block C, Eq. 5 is


f BC

B
C

fC = 0

So for equilibrium,

Stationary

f C = f BC
= 58.9 N

24 Slip impends so

Compare this with the maximum possible friction force

f BC = f BC-max

f C-max

by Eq. 8
= 58.9 N

CNC

by Eq. 6
= (0.15)(294.3 N) = 44.1 N

(9)

So the surfaces can develop only 44.1 N while


58.9 N are needed for equilibrium. Thus block C will
move, contrary to our assumption for Case 2.

We have to check to see if the surfaces of contact between


block C and the ground develop enough friction force to
keep block C stationary.

1238

7.1 Friction: Basic Applications Example 10, page 7 of 8


28 Eq. 5 gives f BC:

26 Case 3
A

Impending
motion together

f BC

fC = 0

Thus

C
f BC = f C

by Eq. 10

= 44.1 N

(11)

27 Slip impends so
and so
f C = f C-max = 44.1 N

by Eq. 8

(10)

44.1 N = f BC < f BC-max = 58.9 N

by Eq. 9

(OK)

Eq. 3 gives f AB:

We don't have to check that the surfaces of contact


between blocks A and B and between B and C develop
enough friction to keep A and B in equilibrium, since there
were only three cases of possible motion, and we showed
that the first two cases were impossible. Nonetheless, we
can verify that our work is correct by showing that the
friction forces acting between A and B and between B and
C are less than their maximum possible values.

f AB

f BC = 0

Thus
f AB = f BC
= 44.1 N

(12)

and so
by Eq. 7
44.1 N = f AB < f AB-max = 78.5 N

1239

(OK)

7.1 Friction: Basic Applications Example 10, page 8 of 8


29 Thus the surfaces of contact between blocks A
and B and between B and C can develop
enough friction to keep blocks A, B, and C
moving together as a unit.
30 Finally, we can calculate P from Eq. 1:
P
or,

f AB = 0
by Eq. 12

P = f AB = 44.1 N

Ans.

31 Why didn't we consider a case like this?


Case 4

32 Answer: No matter what the impending motion is,


there are only seven unknown forces (f AB, NAB, f BC, NBC,
f C, NC, and P). Since these seven unknowns must
satisfy the six equations of equilibrium, the unknowns
can be chosen to satisfy only one additional equation
a friction equation. In the unlikely event that the
masses and 's just happen to have values such that the
seven forces simultaneously satisfy the six equilibrium
equations and two friction equations, then one of the
eight equations must be redundant.
Applying this reasoning to Case 4, we see that if forces
exist that satisfy Case 4's equations, then these forces
must be identical to the forces satisfying the equations
for Case 1 (slip between A and B) and Case 2 (slip
between B and C). Since solving Case 4 would give
the same answer as solving Case 1 (or Case 2), we
don't have to consider Case 4. A similar argument can
be made for other possible motions.

Impending motion of A relative to B

Impending motion of B relative to C

Stationary

1240

7.1 Friction: Basic Applications Example 11, page 1 of 9


11. The two cylinders shown are initially at rest when
horizontal forces of magnitude P/2 are applied to the ends
of the axle in the lower cylinder. The coefficients of
static friction for each pair of surfaces are given in the
figure. Determine the largest value of P that can be
applied without moving the cylinders up the inclined
plane.
P/2
Radius of each cylinder = 300 mm
B

= 0.4

= 0.5
Mass of each cylinder = 50 kg

C
A
A

= 0.6

P/2
25

1241

7.1 Friction: Basic Applications Example 11, page 2 of 9


1

Impending motion of point C on lower


cylinder relative to upper cylinder (An
observer on the upper cylinder would
see this motion as the lower cylinder
moves).

Free-body diagram of lower cylinder


y
NC

Weight = mg
= (50 kg)(9.81 m/s2)
= 490.5 N

C
6 The friction force from f C
the upper cylinder
opposes the relative
motion of point C on the
lower cylinder.
fA

4 The numerical values of


calculated later.

and

will be

A
25
x

Radius = 300 mm
8

The friction force from the


plane opposes the motion
of point A on the cylinder.

NA
7

Impending motion of point A on


cylinder. The x component of
the applied force, P cos , is
pushing the cylinder up the
plane.

1242

It is convenient to use an
inclined xy coordinate-system.

7.1 Friction: Basic Applications Example 11, page 3 of 9


9

Equilibrium equations for cylinder:

Fx = 0: (490.5 N) sin
Fy = 0:

90.5 N) cos

MO = 0: f A(300 mm)

P cos
P sin

Free-body diagram of lower cylinder repeated


+ f A + NC = 0

(1)

+ f C + NA = 0

f C(300 mm) = 0

(2)

NC

490.5 N

(3)
C
fC

10 Geometry

= 90
= 25

65

O
y
A

fA
65

25
x
Radius = 300 mm

NA
= 25

25

1243

7.1 Friction: Basic Applications Example 11, page 4 of 9


11 Free-body diagram of upper cylinder
y

Radius = 300 mm

Weight = 490.5 N

= 25

P
fC
C

fB

15 The friction force


from the plane
opposes the motion
up the plane.

13 The friction force from the lower


cylinder opposes the relative
motion of point C on the upper
cylinder.
.
NC

12 Impending motion of point C on upper


cylinder relative to lower cylinder (An
NB
observer on the lower cylinder would
see this motion as the upper cylinder
moves).
14 Impending motion of point B as
normal force NC pushes the
upper cylinder up the plane.

1244

7.1 Friction: Basic Applications Example 11, page 5 of 9


Free-body diagram of upper cylinder repeated

16 Equilibrium equations

Fx = 0: (490.5 N) sin 25 + f B
Fy = 0:

(490.5 N) cos 25

MO' = 0: f B(300 mm)

NC = 0

(4)

f C + NB = 0

(5)

f C(300 mm) = 0

490.5 N

(6)

17 Thus far we have six equation but seven unknowns (P, f A, NA,
f B, NB, f C, NC), so another equation is needed.
The seventh equation comes from the condition of impending
slip. We have to consider only two cases:
1. slip occurs at point B (and simultaneously rolling
occurs about points A and C).

25

P
C

fB

2. slip occurs at point C (and simultaneously rolling


occurs about points A and B).
Slip at point A will be discussed later.

1245

NC
x

Radius = 300 mm
NB

fC

7.1 Friction: Basic Applications Example 11, page 6 of 9


18 Case 1
Before motion
O'
C

After motion
A
O'
B
20 Slip

For impending slip at B,


f B = f B-max

BNB

= 0.4NB

Displacement of point O
(Point O moves up the plane)
A

(7)

19 Rolling without slipping (The radial


line OA on the lower cylinder
rotates through the same angle, , as
the radial line O'C on the upper
cylinder.)

1246

7.1 Friction: Basic Applications Example 11, page 7 of 9


21 Solving Eqs. 1-7 simultaneously gives
f A = 296 N

NA = 618 N

f B = 296 N

NB = 741 N

f C = 296 N

NC = 504 N

P = 1111 N

22 We must check that the surfaces at A and C can provide enough friction
force to prevent slip and allow rolling:
fA

296

f A-max

A NA

= (0.6)(618 N) = 371 N (OK)

f C-max
CNC = (0.5)(504 N) = 252 N (Not enough! We need
f C = 296 N for equilibrium.)
So the assumption of impending slip at B is wrong.

1247

7.1 Friction: Basic Applications Example 11, page 8 of 9


23 Case 2 (Slip at C, rolling at A
and B)
Before motion
O'
C

After motion
A
O'

C
O

25 Rolling without
slipping

Displacement
of point O
A

26 Slip
24 Rolling without
slipping

For impending slip at C,


f C = f C-max

CNC

= 0.5NC

(8)

1248

7.1 Friction: Basic Applications Example 11, page 9 of 9


27 Solving the six equilibrium equations, Eqs. 1-6, plus Eq. 8
yields
f A = 207 N

NA = 624 N

f B = 207 N

NB = 652 N

f C = 207 N

NC = 415 N
P = 915 N

29 What about slip occurring at point A only? Well if the


lower cylinder moves, then the upper cylinder must also
move. But the only way that the upper cylinder can move
is if either 1) it slips at point B, or 2) it slips at point C.
Thus the case of slip impending at point A alone is
impossible and does not have to be considered.
What about simultaneous slip at A and B? Answer: we
have already found values of the seven unknowns in the
problem that satisfy the six equilibrium equations and the
equation for slip at B. In the unlikely case that the seven
values happen to satisfy an eighth equation (slip at A), then
that equation must be redundant, and the solution for the
eight equations is the same as we have already found for
the seven equations.

Ans.

28 The above answers must be correct since we eliminated


the only other possible case where slip impends. But we
can check our results by verifying that the friction forces
at A and B are less than their maximum possible values.
f A = 207 N

f A-max

A NA

An analogous statement can be made for the case of


simultaneous slip at A and C.

= (0.6)(624 N)

= 374 N (OK)
f B = 207 N

f B-max

BNB

= (0.4)(652 N)

= 261 N (OK)

1249

7.2 Wedges

1250

7.2 Wedges Procedures and Strategies, page 1 of 1


Procedures and Strategies for Solving Problems
Involving Friction: Wedges
1. Draw a free-body diagram of the bodies involved and write
equilibrium equations.
2. Show friction forces opposing impending motion. If a wedge
is to be inserted or removed, then you know that slip impends
on the surfaces of the wedge, and thus f = f max (
) on
these surfaces. If you do not know if slip impends, then solve
the friction-wedge problem by using the same approach used
for basic applications of friction equations..

1251

7.2 Wedges Problem Statement for Example 1


1. If the coefficient of static friction equals 0.3 for all
surfaces of contact, determine the smallest value of force P
necessary to raise the block A. Neglect the weight of the
wedge B.

A
300 kg

10
B

1252

7.2 Wedges Problem Statement for Example 2


2. Wedges A and B are to be glued together. Determine the
minimum coefficient of static friction required, if clamp
CDE is to be able to hold the wedges in the position shown,
while the glue dries.
A

C
D

10
10
B

1253

7.2 Wedges Problem Statement for Example 3


3. Determine the smallest values of forces P1 and P2
required to raise block A while preventing A from
moving horizontally. The coefficient of static friction
for all surfaces of contact is 0.3, and the weight of
wedges B and C is negligible compared to the weight
of block A.
A
2 kip

P2

P1
C

1254

7.2 Wedges Problem Statement for Example 4


4. If the coefficient of static friction for all surfaces of
contact is 0.25, determine the smallest value of the forces P
that will move wedge B upward.
B
200 kg
P

C
20 kg

75

75

20 kg

1255

7.2 Wedges Problem Statement for Example 5


5. The cylinder D, which is connected by a pin at A to the
triangular plate C, is being raised by the wedge B. Neglecting the
weight of the wedge and the plate, determine the minimum force
P necessary to raise the cylinder if the coefficient of static friction
is 0.3 for the surfaces of contact of the wedge.

400 lb
A

D
r = 10 in

C
B

10

1256

7.2 Wedges Problem Statement for Example 6


6. To split the log shown, a 120-lb force is applied to the top of the
wedge, which causes the wedge to be about to slip farther into the log.
Determine the friction and normal forces acting on the sides of the
wedge, if the coefficient of static friction is 0.6. Also determine if the
wedge will pop out of the log if the force is removed. Neglect the
weight of the wedge.
120 lb
Wedge angle = 8

1257

7.2 Wedges Problem Statement for Example 7


7. The end A of the beam needs to be raised slightly to make it level. If
the coefficient of static friction of the contact surfaces of the wedge is
0.3, determine the smallest value of the horizontal force P that will raise
end A. The weight and size of the wedge are negligible. Also, if the
force P is removed, determine if the wedge will remain in place, that is, is
the wedge self-locking?
60 lb/ft

A
P

15 ft

1258

7.2 Wedges Example 1, page 1 of 4


1. If the coefficient of static friction equals 0.3 for all
surfaces of contact, determine the smallest value of force P
necessary to raise the block A. Neglect the weight of the
wedge B.

A
300 kg

10
B

1259

7.2 Wedges Example 1, page 2 of 4


Free-body diagram
of block A

Equations of equilibrium for block A

Fx = 0: NA

f AB cos 10

NAB sin

Fy = 0:

f AB sin 10

NAB cos

=0

(1)

y
2

Impending motion of
left surface of block A
relative to wall

fA

2.943 kN = 0

(2)

Impending motion so,


f A = f A-max

NA = 0.3NA

(3)

A
f AB = f AB-max
NA
fA

(300 kg)(9.81 m/s2)


= 2.943 kN

f AB

NAB = 0.3NAB

x
4

Impending motion of lower surface of block A relative


to block B (This is the motion that an observer sitting
on block B would see as he observes block A move
past.)

10
3

The friction force from the


wall opposes the relative
motion of block B.

NAB

5 The friction force from block B opposes the relative


motion of block A.

1260

(4)

7.2 Wedges Example 1, page 3 of 4


7

Geometry
10

Free-body diagram of wedge B

80

NAB = 3.716 N

10
10 Impending motion
of top surface of
block B relative to
block A.

f AB = 1.115 N

10
P

B
= 90

fB

Using = 10 in Eqs. 1- 4 and solving


simultaneously gives

NB
12 Impending motion of lower
surface of block B relative to
floor

f A = 0.523 N

11 The friction force f AB from


block A opposes the
relative motion of block B.
13 The friction force f B from the
floor opposes the relative motion
of block B.

NA = 1.743 N
f AB = 1.115 N
NAB = 3.716 N

14 Equations of equilibrium for block B


+

Fx = 0: f B + (1.115 N) cos 10 + (3.716 N) sin 10

80 = 10

Fy = 0: NB + (1.115 N) sin 10

1261

P=0

(3.716 N) cos 10 = 0

(5)
(6)

7.2 Wedges Example 1, page 4 of 4


15 Slip impends between block B and the floor, so
f B = f B-max

NB = 0.3NB

(7)

Solving Eqs. 5, 6, and 7 simultaneously gives


f B = 1.04 N
NB = 3.47 N
P = 2.78 N

Ans.

1262

7.2 Wedges Example 2, page 1 of 3


2. Wedges A and B are to be glued together. Determine the
minimum coefficient of static friction required, if clamp
CDE is to be able to hold the wedges in the position shown,
while the glue dries.
A

C
D

10
10
B

1263

7.2 Wedges Example 2, page 2 of 3


1

Free-body diagram
of wedges

Impending motion of
upper surface of wedge A
relative to the upper jaw
of the clamp (The wedge
is just about to slip out
from the jaws of the
clamp.)

Impending motion of
lower surface of wedge
B relative to the lower
jaw of the clamp.

N
3 The friction force from the
upper jaw of the clamp opposes
the relative motion of block A.
A

10
10

f
The friction force from the
lower jaw of the clamp opposes
the relative motion of block B.
N
6

Because of symmetry, the normal force N and friction


force f acting on the bottom of the wedge are given the
same variable names as those on the top.
Equations of equilibrium

Fx = 0: 2N sin

Fy = 0: N cos

2f cos 10 = 0
N cos

1264

f sin 10 + f sin 10 = 0

(1)
(2)

7.2 Wedges Example 2, page 3 of 3


8

Since we have already used symmetry in


labeling the forces on the free-body diagram, the
Fy equilibrium equation degenerates to 0 = 0
and gives us no new information.

Geometry

11 Eqs. 1 and 3 constitute two equations in three


unknowns (f, N, and ), so it appears that we can't
solve the problem. But we have not been asked to
find f and N, only . Thus solve Eq. 1 with = 10
for the ratio f/N:
f
= tan 10
N

(4)

Also solve Eq. 3 for f/N:


=10

90

10 = 80
f
N=

10

(5)

Eqs. 4 and 5 imply that


10 Slip impends between both jaws of the clamp and
the wedge, and thus
f = f max

= tan 10
= 0.176

(3)

Ans.

This result is independent of the clamp force N. That


is, no matter how strong the clamp spring is, if the
coefficient of friction is less than 0.176, then the
clamp will slip.

1265

7.2 Wedges Example 3, page 1 of 3


3. Determine the smallest values of forces P1 and P2
required to raise block A while preventing A from
moving horizontally. The coefficient of static friction
for all surfaces of contact is 0.3, and the weight of
wedges B and C is negligible compared to the weight
of block A.

Free-body diagram of block A


A
2 kip

A
f AB

2 kip

NAB
7

P2

P1
C

2 Equations of equilibrium for block A


+

Fx = 0:

f AB = 0

(1)

Therefore, f AB = 0
Fy = 0: NAB

2 kip = 0

(2)

Solving gives
NAB = 2 kip
3 The friction force f AB has to be zero, since we know that block A is
not to move horizontally and no other horizontal force acts. In fact, we
could have just shown f AB = 0 on the free body initially.

1266

7.2 Wedges Example 3, page 2 of 3


4 Free-body diagram of block B

Geometry
7
7

NAB = 2 kip
7

P2

90

7 = 83

f BC
NBC
6

Equations of equilibrium for block B

The friction force


from block C
opposes the relative
motion of block B.

5 Impending motion
of lower surface of
block B relative to
block C.

Fx = 0: f BC cos 7 + NBC sin


Fy = 0:

f BC sin 7 + NBC cos

P2 = 0
2 kip = 0

10 Solving Eqs. 3, 4, and 5 with


(3)

NBC = 0.3NBC

83 = 7

= 7 gives

NBC = 2.092 kip

(4)

f BC = 0.628 kip = 628 lb

Slip impends between blocks B and C, so


f BC = f BC-max

= 90

P2 = 0.878 kip = 878 lb


(5)

1267

Ans.

7.2 Wedges Example 3, page 3 of 3


11 Free-body diagram of block C
y
= 7

NBC = 2.092 kip

f BC

13 The friction force f BC from


block B opposes the relative
motion of block C.
12 Impending motion of upper
surface of C relative to B

P1

x
fC

15 The friction force f C


from the floor
opposes the relative
motion of block B.

14 Impending motion of lower


surface of C relative to the floor
NC

Fx = 0:

16 Equilibrium equations for block C

Fy = 0: f BC sin 7

f BC cos 7

(2.092 kip) sin 7 + P1 f C = 0

(6)

(2.092 kip) cos 7 + NC = 0

(7)
f C = 0.6 kip = 600 lb

Slip impends between block C and the floor, so


f C = f C-max

17 Solving Eqs. 6, 7, and 8


simultaneously gives

NC = 0.3NC

(8)

NC = 2 kip
P1 = 1.478 kip

1268

Ans.

7.2 Wedges Example 4, page 1 of 4


4. If the coefficient of static friction for all surfaces of
contact is 0.25, determine the smallest value of the forces P
that will move wedge B upward.
B
200 kg
P

C
20 kg

75

75

20 kg

1269

7.2 Wedges Example 4, page 2 of 4


1

Free-body diagram of block A


4

Weight = (20 kg)(9.81 m/s2)


= 196.2 N

Impending motion of right-side of block A


relative to block B (An observer on block B
would see block A move down.)

NAB
A
75

P
2

f AB
fA

Impending motion of
bottom of block A
relative to ground

The friction force from block B opposes


the relative motion of block A.

3 The friction force from the floor


opposes the motion of block A.

NA
Equations of equilibrium
+

Fx = 0: P

Fy = 0: NA

fA

f AB cos 75

NAB cos

196.2 N + f AB sin 75

=0

NAB sin

(1)
=0

Geometry

(2)
= 90
= 15

Slip impends so,


f A = f A-max
f AB = f AB-max

NA = 0.25NA
NAB = 0.25NAB

(3)
(4)

1270

75

75

75

7.2 Wedges Example 4, page 3 of 4


Free-body diagram of block B

8
Impending
motion of block
B relative to
block A

Impending motion of block B


relative to block C
f AB

75

f BC = f AB by symmetry (You can show this by


summing moments about the point where the
lines of action of NAB, NBC, and the weight
intersect)

75

NAB
= 15

= 15
Weight = (200 kg)(9.81 m/s2)
= 1962 N

NBC = NAB by symmetry (You can show this by


summing moments about the point where the lines
of action of f AB, f BC, and the weight intersect.)

The friction forces from


blocks A and C oppose
impending upward relative
motion of block B.

10 Equations of equilibrium
+

Fx = 0: NAB cos 15

NAB cos 15 + f AB cos 75

f AB cos 75 = 0

(5)

(Note that this equation reduces to 0 = 0. This happens because we have assumed symmetry
to conclude that f BC = f AB and NBC = NAB.)
Fy = 0: NAB sin 15 + NAB sin 15

f AB sin 75

1271

f AB sin 75

1962 N = 0

(6)

7.2 Wedges Example 4, page 4 of 4


11 Solving Eqs. 1 4 and 6 simultaneously, with

= 75, gives

f A = 294 N = 0.294 kN
NA = 1 180 N = 1.18 kN
f AB = 14 150 N = 14.15 kN
NAB = 56 600 N = 56.6 kN
P = 58 600 N = 58.6 kN

Ans.

1272

7.2 Wedges Example 5, page 1 of 4


5. The cylinder D, which is connected by a pin at A to the
triangular plate C, is being raised by the wedge B. Neglecting the
weight of the wedge and the plate, determine the minimum force
P necessary to raise the cylinder if the coefficient of static friction
is 0.3 for the surfaces of contact of the wedge.

400 lb
A

D
r = 10 in

C
B

10

1273

7.2 Wedges Example 5, page 2 of 4


Free-body diagram of triangular plate
2 Equilibrium equation for triangular plate

Fy = 0: Ay = 0

So, no vertical force is transmitted by pin A.

Ay

Resultant of
roller forces

(1)

Ax

1274

7.2 Wedges Example 5, page 3 of 4


Free-body diagram of cylinder D

Equilibrium equations for cylinder D.


The equation for the sum of forces in the x-direction has not been
included because including it would introduce an additional
unknown, Ax, which we have not been asked to determine.

r = 10 in.

400 lb

Fy = 0:

Ax

Ay = 0

f BD sin

MA = 0:

+ NBD cos

400 lb = 0

f BD(10 in.) = 0

(3)

Therefore f BD = 0, that is, no friction force acts on the cylinder.


f BD
5

10

(2)

Geometry

NBD

10 80

= 90

Solving Eq. 2 with

= 10 gives

NBD = 406.2 lb

1275

80 = 10

7.2 Wedges Example 5, page 4 of 4


7

Free-body diagram of wedge B


9

= 10

B
P

Fx = 0: f B + (406.2 lb) sin 10

NBD = 406.2 lb

Equilibrium equations for wedge B

Fy = 0: NB

(406.2 lb) cos 10 = 0

(4)
(5)

Slip impends, so
f B = f B-max

10
fB

P=0

NB = 0.3NB

(6)

Solving Eqs. 4, 5, and 6 simultaneously gives

Impending motion
of wedge B relative
to the floor.

f B = 120 lb
NB = 400 lb

NB

P = 190.5 lb

1276

Ans.

7.2 Wedges Example 6, page 1 of 3


6. To split the log shown, a 120-lb force is applied to the top of the
wedge, which causes the wedge to be about to slip farther into the log.
Determine the friction and normal forces acting on the sides of the
wedge, if the coefficient of static friction is 0.6. Also determine if the
wedge will pop out of the log if the force is removed. Neglect the
weight of the wedge.
120 lb
Wedge angle = 8

1277

7.2 Wedges Example 6, page 2 of 3


Free-body diagram of wedge

120 lb
2

Impending
motion of left side
of wedge

Equilibrium equation for the wedge

Fy = 0: 2f cos 4

Geometry

2N sin

90
4 4

N
f

3 The friction
force from the
wood opposes
the motion of
the wedge.

120 lb = 0

(1)

4 = 86

N
4
4

Because of
symmetry, the
same variables,
N and f, are used
on the right side
of the wedge.

= 90
7

86 = 4

Slip impends, so
f = f max

N = 0.6N

(2)

Solving Eqs. 1 and 2 simultaneously gives

1278

f = 53.9 lb

Ans.

N = 89.8 lb

Ans.

7.2 Wedges Example 6, page 3 of 3


Second part of the problem (Determine if the wedge
will pop out, if no force acts down on the top). To
determine if the wedge will pop out, let's first
determine what force would be needed to pull the
wedge out.

Free-body diagram of wedge based on assumption


that a force P is applied to pull the wedge out.
P
10 Impending
motion of wedge
(The wedge is
just about to pop
out.)

12 Equilibrium equation for the wedge

Fy = 0:

2f ' cos 4 + 2N' sin 4 + P = 0

(3)

Slip impends, so
f

f
4 4

N
4

f ' = f 'max

N' = 0.6N'

(4)

Substituting the expression for f ' of Eq. 4 into Eq. 3 and


solving for P gives

P = 2(0.6 cos 4
11 The friction force from the wood
opposes the motion of the wedge
(The force tries to keep the
wedge in the stump.)

sin 4)N' = 1.058 N'

(5)

Since the normal force N' always points towards the wedge,
it is always positive. Eq. 5 thus implies that the wedge will
pop out only if an upward force greater than or equal to 1.058
times the normal force is applied. For any smaller value of
P, the wedge will remain in place. Thus in particular for the
special case of P = 0 (no vertical force applied), the wedge
will remain in place.

1279

7.2 Wedges Example 7, page 1 of 6


7. The end A of the beam needs to be raised slightly to make it level. If
the coefficient of static friction of the contact surfaces of the wedge is
0.3, determine the smallest value of the horizontal force P that will raise
end A. The weight and size of the wedge are negligible. Also, if the
force P is removed, determine if the wedge will remain in place, that is, is
the wedge self-locking?
60 lb/ft

A
P

15 ft

1280

7.2 Wedges Example 7, page 2 of 6


1

Free-body diagram of wedge


y
Equilibrium equations for wedge

Impending motion of
top of wedge relative to
beam

NAB

Fx = 0: P

Fy = 0: NA + f AB sin 8

fA

f AB cos 8

NAB sin

NAB cos

=0

=0

(1)
(2)

f AB

Slip impends so,


8

Impending
motion of top
of wedge
relative to floor

f A = f A-max

fA

f AB = f AB-max

NA = 0.3NA
NAB = 0.3NAB

NA
5

Geometry
= 180
= 8

(90 +82)

8
90

1281

8 = 82

(3)
(4)

7.2 Wedges Example 7, page 3 of 6


6

Impending motion of
beam relative to wedge
(An observer on the
wedge would see the
end of the beam move
up and to the left.)

60 lb/ft

f AB A

The friction force


from the wedge
opposes the motion
of the beam.

B
Bx

8
NAB

By

15 ft

Equilibrium equation of the beam

MB = 0: (60 lb/ft)(15 ft)( 152 ft )

NAB cos 8 (15 ft) + f AB sin 8 (15 ft) = 0

Solving Eqs. 1-5 simultaneously gives


f A = 135 lb
NA = 450 lb
f AB = 142 lb
NAB = 474 lb
P = 342 lb

Ans.

1282

(5)

7.2 Wedges Example 7, page 4 of 6


9

Second part of the problem: Determine if the


wedge is self-locking. To do so, reverse the
direction of force P and calculate the value of P
necessary to cause impending motion of the
wedge to the left.
60 lb/ft

A
P

15 ft

1283

7.2 Wedges Example 7, page 5 of 6


10 Free-body diagram of wedge
11 Impending motion of
top surface of wedge
relative to beam (The
force P has been applied
to pull the wedge out.)

y
8
NAB

f AB
8

fA

12 Impending motion of
bottom surface of
wedge relative to floor

13 Equilibrium equations for wedge


NA

Fx = 0:

P + f A + f AB cos 8

Fy = 0: NA

f AB sin 8

NAB sin 8 = 0

NAB cos 8 = 0

(1)
(2)

Slip impends so,


f A = f A-max
f AB = f B-max

1284

NA = 0.3NA
NAB = 0.3NAB

(3)
(4)

7.2 Wedges Example 7, page 6 of 6


14 Free-body diagram of beam
60 lb/ft

f AB

NAB
8

B
Bx

8
By
15 ft

17 Solving Eqs. 1-5 simultaneously gives


f A = 135 lb

16 Equilibrium equation of the beam

f AB = 131 lb

15 Impending motion of beam relative to


wedge (an observer on the wedge sees the
beam move down and to the right)

MB = 0: (60 lb/ft)(15 ft)( 152 ft )


f AB sin 8 (15 ft) = 0

NA = 450 lb

NAB = 436 lb,

NAB cos 8 (15 ft)

P = 204 lb.

(5)

A force P of at least 204 lb is necessary to pull out the


wedge; any thing less than 204 lb is not enough the wedge
will remain in place. In particular, when no force is
appliced (P = 0), the wedge remains in place. Thus it is
self-locking.

1285

7.3 Square-Threaded Screws

1286

7.3 Square-Threaded Screws Procedures and Strategies, page 1 of 2


Procedures and Strategies for Solving Problems Involving
Friction: Square-Threaded Screws.

Sense of rotation produced by torque M applied to screw

1. Make a sketch of the screw, showing F, the resultant


axial force acting on the screw excluding the force from
the supporting threads.

2. Compare the direction of F with the direction of the


motion of the screw relative to the supporting threads

Direction of motion of screw


relative to supporting threads

a) If the force and motion are in opposite directions,


calculate the torque M applied to the screw from the
formula
M = Fr tan ( + )
in which
r = mean radius of the threads
= tan-1[L/(2 r)]
L = lead of the screw
= angle of friction (= tan-1 )
= coefficient of friction.

1287

7.3 Square-Threaded Screws Procedures and Strategies, page 2 of 2


b) If the force and motion are in the same direction,
then determine if the screw is self-locking by
checking the condition for a screw to be self-locking:

Direction of motion of screw

tan

i) If the screw is self-locking and the motion of the


screw is in the same direction as the axial load,
then a torque must be applied to make the screw
move in the direction of the load. This torque can
be calculated from
M = Fr tan (

Self-locking screw

)
Direction of motion of screw

ii) If the screw is not self-locking and the


impending motion of the screw is in the same
direction as the axial load, then a torque must be
applied to prevent the screw from turning on its
own. This torque can be calculated from
M = Fr tan (

Non-self-locking screw

1288

7.3 Square-Threaded Screws Problem Statement for Example 1


1. The square-threaded jack is used to raise and lower
the 100-kg block. Determine a) the torque Mu required
to begin moving the block up, and b) the torque Md
required to begin moving the block down. Also
determine if the block will remain stationary when the
torque is removed. The screw has lead L = 10 mm,
mean radius r = 8 mm, and is single-threaded. The
coefficient of static friction between the screw and the
supporting threads of the base is = 0.25.

100 kg

1289

7.3 Square-Threaded Screws Problem Statement for Example 2


2. Determine the minimum number n of threads per
inch that will prevent the square-threaded jack screw
from turning by itself under its own weight. The mean
radius r of the thread is 0.5 in., and the coefficient of
static friction between the screw and the supporting
threads of the base is = 0.15. The screw is
single-threaded.

1290

7.3 Square-Threaded Screws Problem Statement for Example 3


3. An assembly consists of a tube, washer A, and
a square-threaded screw. The end of the screw is
threaded into the support B. The tube will be
crushed if the compressive load acting on it
exceeds 100 N. Determine the maximum value
of the torque M that can be safely applied to the
tube. The screw has lead L = 8 mm and mean
radius = 10 mm. The coefficient of static friction
between the screw and the threads in the support
A is = 0.3. Neglect the friction between the
washer and the tube and between the washer and
the head of the screw.

1291

7.3 Square-Threaded Screws Problem Statement for Example 4


4. The C-clamp provides a compressive force of 900 N to
two blocks that are being glued together. Determine the
minimum force P necessary to loosen the clamp. The
clamp has a square-threaded screw with lead L = 7 mm
and mean radius r = 8 mm. The coefficient of static
friction between the screw and the supporting threads in
the frame is = 0.25.

Blocks

100 mm

1292

7.3 Square-Threaded Screws Problem Statement for Example 5


5. A square-threaded bolt and nut are used to
connect three plates as shown. Determine the
torque that must be applied to the bolt and nut
to achieve a specified bolt tension of 60 kip.
The mean diameter of the thread is d = 1 in., the
lead is L = 0.1 in., and the coefficient of static
friction between the threads of the bolt and nut
is = 0.2. Friction between the washer and nut
can be neglected.

1293

7.3 Square-Threaded Screws Problem Statement for Example 6


6. The weight W = 6 kN can be raised by rotating the
square-threaded screw of the automobile jack, causing
joints A and C to move towards each other. Determine the
force P required to raise the load when = 30. The lead
of the thread is L = 4 mm, the mean diameter is d = 8 mm,
and the coefficient of static friction is = 0.2.

W
B
150 mm

150 mm

200 mm

150 mm

150 mm

1294

7.3 Square-Threaded Screws Problem Statement for Example 7


7. Joints A and D of the truss are connected by a cable with a
turnbuckle. Determine the minimum torque Me required to
loosen the turnbuckle. The turnbuckle has a square thread
with lead L = 1/8 in. and mean radius r = 1/4 in. The
coefficient of static friction between the turnbuckle frame and
the screws is = 0.4. Both screws are prevented from
rotating as the turnbuckle is turned.
B

5 kip

Left-handed thread

Right-handed thread

Me
6 ft

Enlarged view of turnbuckle


D

6 ft

1295

7.3 Square-Threaded Screws Problem Statement for Example 8


8. Determine the minimum torque Me applied to the turnbuckle
that will cause the 500-kg block to begin to move a) up and
b) down. Also, if no torque is applied, determine if the block
will remain stationary. The turnbuckle has a square thread
with lead L = 5 mm, mean radius r = 10 mm, and coefficient
of static friction between the turnbuckle frame and the screws
of = 0.24. Both screws are prevented from rotating as the
turnbuckle is turned.
Cable

Frictionless pulley
Enlarged view of turnbuckle
Left-handed thread

Right-handed thread

500 kg
Enlarged view of turnbuckle

1296

7.3 Square-Threaded Screws Problem Statement for Example 9


9. The collar connects two shafts with threaded ends.
Both threads are single, square, and right-handed,
with lead L = 5 mm, mean radius r = 8 mm, and
coefficient of static friction = 0.4. The tension in
each shaft is 5 kN. Both screws are prevented from
turning as the collar turns. Determine the minimum
torque M required to turn the collar in either
direction.

5 kN

5 kN

1297

7.3 Square-Threaded Screws Example 1, page 1 of 5


1. The square-threaded jack is used to raise and lower
the 100-kg block. Determine a) the torque Mu required
to begin moving the block up, and b) the torque Md
required to begin moving the block down. Also
determine if the block will remain stationary when the
torque is removed. The screw has lead L = 10 mm,
mean radius r = 8 mm, and is single-threaded. The
coefficient of static friction between the screw and the
supporting threads of the base is = 0.25.

100 kg

1298

7.3 Square-Threaded Screws Example 1, page 2 of 5


1

Sense of rotation produced by torque M applied to screw

Formulas For A Screw With A Single Square Thread


Let F be the resultant axial force acting on the screw,
exclusive of the force from the supporting threads). Then
for screw motion in a direction opposite to F:
M = Fr tan ( + )

(1)
Direction of motion of screw
relative to supporting threads

in which M is the torque required to turn the screw; r is the


mean radius of the threads; = tan-1[L/(2 r)], where L is
the lead of the screw; is the angle of friction (= tan-1 ),
where is the coefficient of friction.
Condition for screw to be of self-locking:
tan

Direction of motion of screw

(3)

Non-self-locking screw with motion in same direction as


axial load:
M = Fr tan (

(2)

Self-locking screw with motion in same direction as axial


load:
M = Fr tan (

(4)

in which M is the minimum torque that must be applied to


the screw to prevent it from turning on its own.

Direction of motion of screw

1299

7.3 Square-Threaded Screws Example 1, page 3 of 5


2

Part a) Move load up.

Load on screw
(100 kg) (9.81 m/s2)
981 N

4
5

Because the motion of the screw is opposite the


direction of the load on the screw, Eq. 1 can be used
to calculate the torque:
Mu = Fr tan ( + )

(Eq.1 repeated)

Impending
motion of
screw

Here
F = 981 N
r = 0.008 m
= tan-1[L/(2 r)]
= tan-1[0.010 m/(2 (0.008 m))]
= 11.252
= tan-1 ,
= tan-1(0.25)
= 14.036

Mu

Substituting these results in Eq. 1 gives


Mu = 3.71 N m

Ans.

1300

Sense of rotation produced by


torque Mu applied to screw

7.3 Square-Threaded Screws Example 1, page 4 of 5


6 Part b) Move load down.

Load

Because the motion of the screw is in the same


direction as the load acting on the screw, the
appropriate formula to use to calculate the torque Md is
either Eq. 3 or Eq. 4. To decide which equation to use,
we have to determine if the screw is self-locking:

Impending
motion of
screw

Sense of rotation
produced by torque
Md applied to screw

Condition for screw to be self-locking:


tan
0.25

(Eq. 2 repeated)
tan 11.252= 0.199

Thus the inequality is satisfied and the screw is


self-locking. Eq. 3 then applies:
Md = Fr tan (

(Eq. 3 repeated)

Substituting F = 981 N, r = 0.008 m,


= 14.036, as before, gives
Md = 0.38 N m

Md

= 11.252 and

Ans.

Note that this moment is considerably less than the 3.71


N m moment needed to raise the block.

1301

981 N

7.3 Square-Threaded Screws Example 1, page 5 of 5


9

If the torque is removed, does the block remain


stationary? Since we have shown that the screw is
self-locking, the
block remains stationary

Ans.

Comment on "self-locking." The friction coefficient


was = 0.25. Suppose instead that it had been smaller,
say, = 0.15. Then consider the self-locking criterion:
tan
0.15

100 kg

(Eq. 2 repeated)
Sense of rotation (causes
the block to move up, or,
better stated, prevents the
block from moving down)

tan 11.252 = 0.199

Thus the inequality above is not satisfied, and the screw


is not self-locking. To keep the block from descending
rapidly under its own weight as the screw turns, we have
to apply a torque M in the same sense as if we were about
to raise the block. The smallest value of the torque that
will keep the block from coming down is given by Eq. 4:
Md = Fr tan (

Impending
motion of
screw

(Eq. 4 repeated)

Substituting F = 981 N, r = 0.008 m, and = 11.252


as before, together with = tan-10.15 = 8.531, gives
Md = 0.37 N m

1302

7.3 Square-Threaded Screws Example 2, page 1 of 3


2. Determine the minimum number n of threads per
inch that will prevent the square-threaded jack screw
from turning by itself under its own weight. The mean
radius r of the thread is 0.5 in., and the coefficient of
static friction between the screw and the supporting
threads of the base is = 0.15. The screw is
single-threaded.

1303

7.3 Square-Threaded Screws Example 2, page 2 of 3


1

Sense of rotation produced by torque M applied to screw

Formulas For A Screw With A Single Square Thread


Let F be the resultant axial force acting on the screw,
exclusive of the force from the supporting threads). Then
for screw motion in a direction opposite to F:
M = Fr tan ( + )

(1)
Direction of motion of screw
relative to supporting threads

in which M is the torque required to turn the screw; r is the


mean radius of the threads; = tan-1[L/(2 r)], where L is
the lead of the screw; is the angle of friction (= tan-1 ),
where is the coefficient of friction.
Condition for screw to be of self-locking:
tan

Direction of motion of screw

(3)

Non-self-locking screw with motion in same direction as


axial load:
M = Fr tan (

(2)

Self-locking screw with motion in same direction as axial


load:
M = Fr tan (

(4)

in which M is the minimum torque that must be applied to


the screw to prevent it from turning on its own.

Direction of motion of screw

1304

7.3 Square-Threaded Screws Example 2, page 3 of 3


2

The jack will not turn by itself if it is


self-locking. The condition for self-locking is

Solving for n gives


n

tan

1/(2 r )

(Eq. 2 repeated)
Substituting r = 0.5 in. and

= 0.15 gives

Here
n
tan

2.122 threads/in.

= L/(2 r)
Thus the minimum number is about

Using this result in Eq. 2 gives


n = 2.12 threads/in.
L/(2 r)

(5)

Because the screw is single-threaded, L is the


distance between two consecutive threads.
That is, L is the number of inches per thread. It
follows that the reciprocal of L is the number
of threads per inch:
1/L = n
Using this result in Eq. 5 gives
L/(2 r)
1/n

1305

Ans.

7.3 Square-Threaded Screws Example 3, page 1 of 4


3. An assembly consists of a tube, washer A, and
a square-threaded screw. The end of the screw is
threaded into the support B. The tube will be
crushed if the compressive load acting on it
exceeds 100 N. Determine the maximum value
of the torque M that can be safely applied to the
tube. The screw has lead L = 8 mm and mean
radius = 10 mm. The coefficient of static friction
between the screw and the threads in the support
A is = 0.3. Neglect the friction between the
washer and the tube and between the washer and
the head of the screw.

1306

7.3 Square-Threaded Screws Example 3, page 2 of 4


1

Sense of rotation produced by torque M applied to screw

Formulas For A Screw With A Single Square Thread


Let F be the resultant axial force acting on the screw,
exclusive of the force from the supporting threads). Then
for screw motion in a direction opposite to F:
M = Fr tan ( + )

(1)
Direction of motion of screw
relative to supporting threads

in which M is the torque required to turn the screw; r is the


mean radius of the threads; = tan-1[L/(2 r)], where L is
the lead of the screw; is the angle of friction (= tan-1 ),
where is the coefficient of friction.
Condition for screw to be of self-locking:
tan

Direction of motion of screw

(3)

Non-self-locking screw with motion in same direction as


axial load:
M = Fr tan (

(2)

Self-locking screw with motion in same direction as axial


load:
M = Fr tan (

(4)

in which M is the minimum torque that must be applied to


the screw to prevent it from turning on its own.

Direction of motion of screw

1307

7.3 Square-Threaded Screws Example 3, page 3 of 4


2

Free-body diagram of tube


loaded to maximum capacity

100 N

The 100-N reaction force from the tube


pushes up on the washer and thus
produces a tensile load on the screw.
F

100 N

Sense of rotation
produced by torque M

100 N

1308

Impending
motion of
screw

7.3 Square-Threaded Screws Example 3, page 4 of 4


6

The motion of the screw is opposite the direction of the


100-N axial load, so Eq.1 applies:
M = Fr tan ( + )

(Eq. 1 repeated)

Here
F = 100 N
r = 0.01 m
= tan-1[L/(2 r)]
= tan-1[0.008 m/(2 0.01 m))]
= 7.256
= tan-1
= tan-1(0.3)
16.699
Substituting these values in Eq. 1 gives
M = 0.444 N m

Ans.

1309

7.3 Square-Threaded Screws Example 4, page 1 of 4


4. The C-clamp provides a compressive force of 900 N to
two blocks that are being glued together. Determine the
minimum force P necessary to loosen the clamp. The
clamp has a square-threaded screw with lead L = 7 mm
and mean radius r = 8 mm. The coefficient of static
friction between the screw and the supporting threads in
the frame is = 0.25.

Blocks

100 mm

1310

7.3 Square-Threaded Screws Example 4, page 2 of 4


1

Sense of rotation produced by torque M applied to screw

Formulas For A Screw With A Single Square Thread


Let F be the resultant axial force acting on the screw,
exclusive of the force from the supporting threads). Then
for screw motion in a direction opposite to F:
M = Fr tan ( + )

(1)
Direction of motion of screw
relative to supporting threads

in which M is the torque required to turn the screw; r is the


mean radius of the threads; = tan-1[L/(2 r)], where L is
the lead of the screw; is the angle of friction (= tan-1 ),
where is the coefficient of friction.
Condition for screw to be of self-locking:
tan

Direction of motion of screw

(3)

Non-self-locking screw with motion in same direction as


axial load:
M = Fr tan (

(2)

Self-locking screw with motion in same direction as axial


load:
M = Fr tan (

(4)

in which M is the minimum torque that must be applied to


the screw to prevent it from turning on its own.

Direction of motion of screw

1311

7.3 Square-Threaded Screws Example 4, page 3 of 4


2

Free-body diagram of blocks

900 N

The reaction force from the block


produces an axial load on the screw,
F

900 N

900 N

Torque, M (0.1 m) P
(produced by force P)
100 mm

Impending motion
of screw

Because the motion of the screw is in the same direction as


the load acting on the screw, the appropriate formula to use
to calculate the torque M is either Eq. 3 or Eq. 4.

1312

7.3 Square-Threaded Screws Example 4, page 4 of 4


7

To decide whether to use Eq. 3 or Eq. 4, we have to


determine if the screw is self-locking.

Thus the inequality is satisfied, and the screw is


self-locking. Eq. 3 applies:
M = Fr tan (

Condition for a screw to be self-locking:

(Eq. 3 repeated)

Here

0.25
tan

M = (0.1 m)P

(Eq. 2 repeated)

F = 900 N

L/(2 r) = 0.007 m/(2 0.008 m)) = 0.139

r = 0.008 m
= tan-1[L/(2 r)]
= tan-1[0.007 m/(2 0.008 m))]
= 7.928
= tan-1
= tan-1(0.25)
14.036
Substituting these values in Eq. 3 and solving
for P gives
P = 7.70 N

1313

Ans.

7.3 Square-Threaded Screws Example 5, page 1 of 4


5. A square-threaded bolt and nut are used to
connect three plates as shown. Determine the
torque that must be applied to the bolt and nut
to achieve a specified bolt tension of 60 kip.
The mean diameter of the thread is d = 1 in., the
lead is L = 0.1 in., and the coefficient of static
friction between the threads of the bolt and nut
is = 0.2. Friction between the washer and nut
can be neglected.

1314

7.3 Square-Threaded Screws Example 5, page 2 of 4


1

Sense of rotation produced by torque M applied to screw

Formulas For A Screw With A Single Square Thread


Let F be the resultant axial force acting on the screw,
exclusive of the force from the supporting threads). Then
for screw motion in a direction opposite to F:
M = Fr tan ( + )

(1)
Direction of motion of screw
relative to supporting threads

in which M is the torque required to turn the screw; r is the


mean radius of the threads; = tan-1[L/(2 r)], where L is
the lead of the screw; is the angle of friction (= tan-1 ),
where is the coefficient of friction.
Condition for screw to be of self-locking:
tan

Direction of motion of screw

(3)

Non-self-locking screw with motion in same direction as


axial load:
M = Fr tan (

(2)

Self-locking screw with motion in same direction as axial


load:
M = Fr tan (

(4)

in which M is the minimum torque that must be applied to


the screw to prevent it from turning on its own.

Direction of motion of screw

1315

7.3 Square-Threaded Screws Example 5, page 3 of 4


M

Torques are applied to the bolt head


and to the nut to tighten the nut.

Free-body diagram of plates


F (force from washer on top)

M
3

Sense of rotation of nut


produced by torque M
applied to the nut

F (force from washer on bottom)

1316

7.3 Square-Threaded Screws Example 5, page 4 of 4


M

The reaction force from the top


plate pushes the washer and the
head of the bolt up. This causes
an axial tensile load F in the
screw.
7

The motion of the screw is opposite the


direction of the axial load F, so Eq.1 applies:.

F = 60 kip

M = Fr tan (

(Eq. 1 repeated)

Here
F = 60 kip
Sense of rotation of nut

r = d/2
= 1.0 in./2
= 0.5 in.

Impending motion
of screw

= tan-1[L/(2 r)]
= tan-1[0.1 in./(2 0.5 in.))]
= 1.823

6 The motion of the nut is up so


the motion of the screw
relative to the supporting
screws of the nut is down.

= tan-1
= tan-1(0.2)
11.310
Substituting these values in Eq. 1 gives
M = 7.00 kip in.

1317

Ans.

7.3 Square-Threaded Screws Example 6, page 1 of 6


6. The weight W = 6 kN can be raised by rotating the
square-threaded screw of the automobile jack, causing
joints A and C to move towards each other. Determine the
force P required to raise the load when = 30. The lead
of the thread is L = 4 mm, the mean diameter is d = 8 mm,
and the coefficient of static friction is = 0.2.

W
B
150 mm

150 mm

200 mm

150 mm

150 mm

1318

7.3 Square-Threaded Screws Example 6, page 2 of 6


1

Sense of rotation produced by torque M applied to screw

Formulas For A Screw With A Single Square Thread


Let F be the resultant axial force acting on the screw,
exclusive of the force from the supporting threads). Then
for screw motion in a direction opposite to F:
M = Fr tan ( + )

(1)
Direction of motion of screw
relative to supporting threads

in which M is the torque required to turn the screw; r is the


mean radius of the threads; = tan-1[L/(2 r)], where L is
the lead of the screw; is the angle of friction (= tan-1 ),
where is the coefficient of friction.
Condition for screw to be of self-locking:
tan

Direction of motion of screw

(3)

Non-self-locking screw with motion in same direction as


axial load:
M = Fr tan (

(2)

Self-locking screw with motion in same direction as axial


load:
M = Fr tan (

(4)

in which M is the minimum torque that must be applied to


the screw to prevent it from turning on its own.

Direction of motion of screw

1319

7.3 Square-Threaded Screws Example 6, page 3 of 6


2

The torque required to turn the screw depends on the


axial load acting on the screw. To find the axial
load, we have to compute the forces in the members
AB, BC, AD, and DC.

Free-body diagram of bracket D

FAD

FDC

The forces in members AB and BC can be found


from a free-body diagram of bracket B:
D

W (force from floor


under jack)
B
4
FAB

Similarly using a free body diagram of bracket D


leads to

FBC
FAD = FDC = W/(2 sin )

Equilibrium equations

Fx = 0: FAB cos

Fy = 0: FAB sin

FBC cos
+ FBC sin

=0
W=0

Solving gives
FAB = FBC = W/(2 sin )

(5)

1320

(6)

7.3 Square-Threaded Screws Example 6, page 4 of 6


Next, a free-body diagram of pin A gives the
force F of the screw acting on the pin:
FAB

W
B

F
FAD

Equilibrium equations

W/(2 sin ) by Eq. 5


+

Fx = 0: FAB cos

FAD cos

+F=0

W/(2 sin ) by Eq. 6


D

Solving gives
F = W/tan

(7)

F is the force of the screw acting on


the pin A. The equal and opposite
force of pin A acting on the screw
constitutes the load on the screw.

1321

7.3 Square-Threaded Screws Example 6, page 5 of 6


W
9 Sense of rotation
caused by torque M

B
7 Load on screw

F = W/tan

A
8

Torque M produced by
force P (M P 0.2 m)
200 mm
D

11 Because the motion of the screw is opposite the


direction of the load on the screw, Eq. 1 can be
used to calculate the torque.

1322

10 Impending motion
of screw relative
to supporting
threads at C

7.3 Square-Threaded Screws Example 6, page 6 of 6


12 Apply Eq. 1:
M = Fr tan ( + )

13 Comment: if a jack is to be safe, it must support the


weight being lifted, even when the operator of the
jack takes his hands off the handle. Is the present
jack safe? Answer: yes, if it is self-locking.

(Eq.1 repeated)

Here
Condition for screw to be self-locking:
M = 0.2P
0.2
F = W/tan
= (6000 N)/tan 30
= 10 392 N

tan
tan 9.043 by Eq. 8
= 0.159

r = (0.008 m)/2
= 0.004 m
= tan-1[L/(2 r)]
= tan-1[0.004 m/(2 (0.004 m))]
= 9.043

Because the inequality is satisfied, the jack is


self-locking and so is safe.
(8)

= tan-1 ,
= tan-1(0.2)
= 11.310
Substituting these results in Eq. 1 gives
P = 77.1 N

(Eq. 2 repeated)

Ans.

1323

7.3 Square-Threaded Screws Example 7, page 1 of 8


7. Joints A and D of the truss are connected by a cable with a
turnbuckle. Determine the minimum torque Me required to
loosen the turnbuckle. The turnbuckle has a square thread
with lead L = 1/8 in. and mean radius r = 1/4 in. The
coefficient of static friction between the turnbuckle frame and
the screws is = 0.4. Both screws are prevented from
rotating as the turnbuckle is turned.
B

5 kip

Left-handed thread

Right-handed thread

Me
6 ft

Enlarged view of turnbuckle


D

6 ft

1324

7.3 Square-Threaded Screws Example 7, page 2 of 8


1

Sense of rotation produced by torque M applied to screw

Formulas For A Screw With A Single Square Thread


Let F be the resultant axial force acting on the screw,
exclusive of the force from the supporting threads). Then
for screw motion in a direction opposite to F:
M = Fr tan ( + )

(1)
Direction of motion of screw
relative to supporting threads

in which M is the torque required to turn the screw; r is the


mean radius of the threads; = tan-1[L/(2 r)], where L is
the lead of the screw; is the angle of friction (= tan-1 ),
where is the coefficient of friction.
Condition for screw to be of self-locking:
tan

Direction of motion of screw

(3)

Non-self-locking screw with motion in same direction as


axial load:
M = Fr tan (

(2)

Self-locking screw with motion in same direction as axial


load:
M = Fr tan (

(4)

in which M is the minimum torque that must be applied to


the screw to prevent it from turning on its own.

Direction of motion of screw

1325

7.3 Square-Threaded Screws Example 7, page 3 of 8


2

The torque required to loosen the turnbuckle


depends on the value of the axial force that acts
on the turnbuckle. Thus we first have to solve
for the force in truss member AD. Using the
method of joints, we begin with a free-body
diagram of joint B:
B

FAB

5 kip
B
FBD

A
5 kip

Equilibrium equation:
+

Fx = 0:

FAB

5 kip = 0
Me

Thus
FAB = 5 kip
= 5 kip (C)

1326

7.3 Square-Threaded Screws Example 7, page 4 of 8


Free-body diagram of joint A
A
FAB 5 kip (C)

45

FAD

5 kip

FAC

Me

Equilibrium equation:
+

Fx = 0:

6 ft

5 kip + FAD sin 45 = 0

Thus
FAD = 7.071 kip (T)

(5)

This is the axial force acting on the ends of the


turnbuckle.

6 ft

1327

7.3 Square-Threaded Screws Example 7, page 5 of 8


4

To loosen the turnbuckle, we must turn it in


such a way that screw R moves to the right
relative to the turnbuckle frame.

Direction of desired motion of


screw R relative to frame

R
Axial load

Axial load

Because screw R has a right-handed


thread, it will move to the right if an
external torque Me is applied to the
frame in the sense shown.

Me
L

Impending motion of screw R


relative to frame

R
Axial load

Axial load
Sense of rotation of frame
produced by torque Me

Right-handed thread
6 Note that the motion of screw
R is in the same direction as
the axial load acting on the
screw.

1328

7.3 Square-Threaded Screws Example 7, page 6 of 8


7

Because screw L has a left-handed


thread, it will move left relative to the
frame if torque Me is applied to the
frame with the same sense as before.

Me

Impending motion of
screw relative to frame
L

R
Axial load

Axial load

Left-handed thread

Sense of rotation of frame


produced by torque Me

Just as was the case for screw R, the


impending motion of screw L is in the same
direction as the axial load. It follows that for
both screws either Eq. 3 or Eq. 4 applies. To
decide which equation to use, we have to
determine if the screws are self-locking.

1329

7.3 Square-Threaded Screws Example 7, page 7 of 8


9

Condition for screw to be self-locking:


tan

10 Apply Eq. 3.

(Eq. 2 repeated)

M = Fr tan (

Here

(Eq. 3 repeated)

Here
= 0.4

F = 7.071 kip

and

(Eq. 5 repeated)

r = 1/4 in.
tan

= L/(2 r)
= (1/8 in.)/[2 (1/4 in.)]
= 0.080

= tan-1
= tan-1(0.4)
1.801

(6)

= tan-1(0.080)
= 4.570

Since
0.4
> tan

(by Eq. 6)

Substituting these values into Eq. 3 gives

0.080

M = 0.548 kip in

the inequality, Eq. 2, is satisfied and the screws are


self-locking. Thus Eq. 3 is the appropriate equation
for calculating M, the torque acting on each screw.

(7)

Note: the torque M is not the same as the external


torque Me, as will now be shown.

1330

7.3 Square-Threaded Screws Example 7, page 8 of 8


11 The torque M can now be related to the external torque
Me by considering a free-body diagram of the frame:
F

7.07 kip

Me

12 M 0.548 kip in. by Eq. 7


(Torque from screw L
resisting rotation of frame)

14 Torque applied externally to


turnbuckle to make it rotate

7.07 kip

13 M 0.548 kip in. by Eq. 7


(Torque from screw R
resisting rotation of frame)

15 Moment equilibrium equation:


+

Maxial = 0: M

Me + M = 0

Thus
Me = M + M
= 0.548 kip in + 0.548 kip in
= 1.096 kip in

1331

Ans.

7.3 Square-Threaded Screws Example 8, page 1 of 8


8. Determine the minimum torque Me applied to the turnbuckle
that will cause the 500-kg block to begin to move a) up and
b) down. Also, if no torque is applied, determine if the block
will remain stationary. The turnbuckle has a square thread
with lead L = 5 mm, mean radius r = 10 mm, and coefficient
of static friction between the turnbuckle frame and the screws
of = 0.24. Both screws are prevented from rotating as the
turnbuckle is turned.
Cable

Frictionless pulley
Enlarged view of turnbuckle
Left-handed thread

Right-handed thread

500 kg
Enlarged view of turnbuckle

1332

7.3 Square-Threaded Screws Example 8, page 2 of 8


1

Sense of rotation produced by torque M applied to screw

Formulas For A Screw With A Single Square Thread


Let F be the resultant axial force acting on the screw,
exclusive of the force from the supporting threads). Then
for screw motion in a direction opposite to F:
M = Fr tan ( + )

(1)
Direction of motion of screw
relative to supporting threads

in which M is the torque required to turn the screw; r is the


mean radius of the threads; = tan-1[L/(2 r)], where L is
the lead of the screw; is the angle of friction (= tan-1 ),
where is the coefficient of friction.
Condition for screw to be of self-locking:
tan

Direction of motion of screw

(3)

Non-self-locking screw with motion in same direction as


axial load:
M = Fr tan (

(2)

Self-locking screw with motion in same direction as axial


load:
M = Fr tan (

(4)

in which M is the minimum torque that must be applied to


the screw to prevent it from turning on its own.

Direction of motion of screw

1333

7.3 Square-Threaded Screws Example 8, page 3 of 8


2

Part a): 500-kg block moves up.


To make the block move up, we must rotate the
turnbuckle in such a way that screw R moves left
relative to the turnbuckle frame.
3
R

L
Axial load

Direction of desired motion of


screw R relative to frame

Axial load

4
Right-handed thread

Because screw R has a right-handed thread, it


will move left if an external torque Me is
applied to the frame in the sense shown.

Me
Impending motion of screw
R relative to frame

L
Axial load

Axial load
6
5

Sense of rotation of frame


produced by torque Me

1334

Note that the motion of


screw R is in a direction
opposite to the axial load
acting on the screw.

7.3 Square-Threaded Screws Example 8, page 4 of 8


7

Because screw L has a left-handed thread, it will


move right relative to the frame if torque Me is
applied with the same sense as before.
Left-handed thread
Me

Impending motion of screw L

L
Axial load

Axial load

Sense of rotation of frame


produced by torque Me
8 Thus the motion of each screw is in a direction opposite
to the axial load acting on the screw. It follows that the
appropriate formula (for square-threaded screws) to use
for both screws is Eq.1, which applies when screw
motion and axial load are in opposite directions.

1335

7.3 Square-Threaded Screws Example 8, page 5 of 8


9

M = Fr tan ( + )

(Eq. 1 repeated)

Here M is the torque acting on either screw L or


screw R. M is not Me (recall that Me is the torque
applied to the frame of the turnbuckle, not to the
individual screws).
F = tension in cable
= weight of 500-kg mass
= (500 kg)(9.81 m/s2)
= 4905 N

(5)

r = 0.01 m

(6)

= tan-1[L/(2 r)]
= tan-1[0.005 m/(2 (0.01 m))]
= 4.550

(7)

= tan-1 ,
= tan-1(0.24)
= 13.496

(8)

Substituting these results in Eq. 1 gives


M = 15.981 N m

(9)

1336

7.3 Square-Threaded Screws Example 8, page 6 of 8


10 The torque M can now be related to the torque
Me by considering a free-body diagram.

Free-body diagram of frame of turnbuckle


F

4905 N

11 M 15.981 N m
(Torque from screw L
acting on frame)

Me

12 Torque applied externally


to turnbuckle frame to
make it rotate

13 M 15.981 N m
(Torque from screw R
acting on frame)

14 Moment equilibrium:
+

Maxial = 0:

M + Me

M=0

Thus
Me = 2M
= 2(15.981 N m)
= 32.0 N m

4905 N

(10)
Ans.

1337

7.3 Square-Threaded Screws Example 8, page 7 of 8


15 Part b): 500-kg block moves down.
To make the block move down, rotate the turnbuckle in the
opposite sense. The sense of Me and the direction of
motions of the screws reverse from what we had before.
16 Impending motion of screw L

Me

17 Impending motion of screw R


R

L
Axial load

Axial load

Sense of rotation
produced by torque Me
18 Because the motion of each screw is in the same direction as the axial
load acting on the screw, either Eq. 3 or Eq. 4 apply. To decide which,
equation to use we have to determine if the screws are self-locking:
Criterion of self-locking:
tan
0.24

(Eq. 2 repeated)
tan 4.550 by Eq. 7
0.080

1338

7.3 Square-Threaded Screws Example 8, page 8 of 8


19 Thus the inequality is satisfied, and the screws are self-locking. Eq. 3
applies:
M = Fr tan (

(Eq. 3 repeated)

Substituting the previous values from Eqs. 5-9 (F = 4905 N,


r = 0.01 m, = 4.550, and = 13.496) gives
M = 7.721 N m
Writing a moment equilibrium equation as before gives Eq. 10 again:
Mu = 2M
= 2(7.721 N m)
= 15.44 N m

(Eq. 10 repeated)
Ans.

Finally, because we have shown that the turnbuckle is self-locking


( > tan in Eq. 2), we can conclude that
the weight will remain stationary when Me = 0

Ans.

1339

7.3 Square-Threaded Screws Example 9, page 1 of 7


9. The collar connects two shafts with threaded ends.
Both threads are single, square, and right-handed,
with lead L = 5 mm, mean radius r = 8 mm, and
coefficient of static friction = 0.4. The tension in
each shaft is 5 kN. Both screws are prevented from
turning as the collar turns. Determine the minimum
torque M required to turn the collar in either
direction.

5 kN

5 kN

1340

7.3 Square-Threaded Screws Example 9, page 2 of 7


1

Sense of rotation produced by torque M applied to screw

Formulas For A Screw With A Single Square Thread


Let F be the resultant axial force acting on the screw,
exclusive of the force from the supporting threads). Then
for screw motion in a direction opposite to F:
M = Fr tan ( + )

(1)
Direction of motion of screw
relative to supporting threads

in which M is the torque required to turn the screw; r is the


mean radius of the threads; = tan-1[L/(2 r)], where L is
the lead of the screw; is the angle of friction (= tan-1 ),
where is the coefficient of friction.
Condition for screw to be of self-locking:
tan

Direction of motion of screw

(3)

Non-self-locking screw with motion in same direction as


axial load:
M = Fr tan (

(2)

Self-locking screw with motion in same direction as axial


load:
M = Fr tan (

(4)

in which M is the minimum torque that must be applied to


the screw to prevent it from turning on its own.

Direction of motion of screw

1341

7.3 Square-Threaded Screws Example 9, page 3 of 7


2

Apply a torque Me to the collar with the


sense chosen arbitrarily as shown.

3 Sense of rotation of collar


produced by torque Me

B
Me

5 kN

5 kN

Because screw B has a right-handed thread,


it will move left relative to the collar.

A
5 kN

B
Me

Impending motion of
screw B

5 kN
5

1342

Note that the motion of screw B is in a direction


opposite to the 5-kN axial load acting on the
screw. It follows that the appropriate formula to
use to calculate MB, the torque required to turn
screw B, is Eq. 1, which applies when screw
motion and axial force are in opposite direction.

7.3 Square-Threaded Screws Example 9, page 4 of 7


6

Apply Eq. 1.
MB = Fr tan (

(Eq. 1 repeated)

Here MB is the torque acting on screw B. MB is not


Me, as will be shown shortly. The other parameters in
Eq. 1 have the following values:
F = 5000 N

(5)

r = 0.008 m
= tan-1[L/(2 r)]
= tan-1[0.005 m/(2 (0.008))]
= 5.681

(6)

= tan-1
= tan-1(0.4)
21.801

(7)

Substituting these values into Eq. 1 gives


MB = 20.807 N m

(8)

1343

7.3 Square-Threaded Screws Example 9, page 5 of 7


7

Next consider screw A. Because screw A has a


right-handed thread, it will move left relative to the
collar when torque Me is applied.

Impending motion of screw A


B

A
5 kN

Me

5 kN
Sense of rotation of collar
produced by torque Me

Because the motion of screw A is in the same


direction as the axial load acting on the screw, the
appropriate formula to use to calculate MA, the torque
required to turn screw A, is either Eq. 3 or Eq. 4. To
decide which equation to use, we have to determine if
the screw is self-locking.

1344

7.3 Square-Threaded Screws Example 9, page 6 of 7


9

Condition for a screw to be self-locking:


0.4
tan

(Eq. 2 repeated)

tan 5.68 by Eq. 6


= 0.099
Thus the inequality is satisfied and the screws are
self-locking. Eq. 3 is the appropriate equation for
calculating MA, the torque acting on screw A:
MA = Fr tan (

(Eq. 3 repeated)

Here
F = 5000 N
r = 0.008 m
= 21.801

(Eq. 7 repeated)

= 5.681

(Eq. 6 repeated)

Substituting these values into Eq. 3 gives


MA = 11.561 N m

(9)

1345

7.3 Square-Threaded Screws Example 9, page 7 of 7


10 Finally, the torque Me applied to the collar can be
related to MA and MB, the torques required to turn
the screws, by considering a free-body diagram:
Free-body diagram of collar (alone, without screws A and B)
A
F

5 kN

11 MA 11.561 N m by Eq. 9
(Torque from screw A
resisting rotation of collar)

12 Torque applied externally to


collar to make it rotate

14 Moment equilibrium equation:


+

Maxial = 0:

Me

MA + Me

MB = 0

Thus
Me = MA + MB
= 11.561 N m + 20.807 N m
= 32.4 N m

Ans.

5 kN

13 MB 20.807 N m by Eq. 8
(Torque from screw B
resisting rotation of collar)

15 We initially chose the sense of the torque Me arbitrarily. Had


we chosen Me with the opposite sense, however, it would have
made no difference in the final answer, because the new value
of MB would be equal to the old value of MA, and the new value
of MA would be equal to the old value of MB. That is, MB and
MA would merely have switched values, and their sum, which is
equal to Me, would have remained the same. Thus the torque
calculated, Me, is the minimum torque required to turn the
collar in either direction.

1346

7.4 Flat Belts

1347

7.4 Flat Belts Procedures and Strategies, page 1 of 1


Procedures and Strategies for Solving Problems Involving
Friction: Flat Belts
1. Draw free-body diagrams and write equilibrium equations.
Direction of
impending motion

2. Apply the equation


T2 = T1e
in which

T2

T2 = belt tension in the direction of impending motion


T1 = belt tension in the direction opposite the motion
coefficient of friction
= angle of wrap, measured in radians

1348

T1

7.4 Flat Belts Problem Statement for Example 1


1. Determine the minimum number of turns of rope that will allow
the 5-lb force to support the 600-lb block, if the coefficient of
static friction is 0.15.
5 lb

n turns

600 lb

1349

7.4 Flat Belts Problem Statement for Example 2


2. If the coefficient of static friction between the rope and the
fixed circular drums A and B is 0.2, determine the largest value
of the force P that can be applied without moving the 150-lb
weight upwards.
P
60

Rope is horizontal.

150 lb

1350

7.4 Flat Belts Problem Statement for Example 3


3. Determine the smallest force P applied to the handle of the
band brake that will prevent the drum from rotating when the
15 lbft moment is applied. The coefficient of static friction
is 0.25, and the weight of lever arm ABC can be neglected.

15 lb-ft
6 in.
D

P
80

10 in.

15 in.

1351

7.4 Flat Belts Problem Statement for Example 4


4. The uniform beam ABC weighs 40 lb. The coefficient
of static friction between the cord and the fixed drum D is
0.3. Determine the smallest value of the weight W for
which the beam will remain horizontal.

B W

4 ft

4 ft

1352

7.4 Flat Belts Problem Statement for Example 5


5. Determine the largest value W of the weight
of block B for which neither block will move.
The coefficients of static friction are 0.2 between
the blocks and the planes, and 0.25 between the
cord and the drum.

B
A
80 lb

W
70

50

1353

7.4 Flat Belts Problem Statement for Example 6


6. A motor attached to pulley A drives the pulley clockwise with a 200 lb-in. torque.
The flat belt then overcomes the resisting torque T at pulley B and rotates the pulley B
clockwise. Determine the minimum tension that can exist in the belt without causing
the belt to slip at pulley A. Also determine the corresponding resisting torque T. The
coefficient of static friction between the belt and the pulleys is 0.3.

T
200 lbin.
A
4 in.

B
7 in.

Driving pulley
Driven pulley
22 in.

1354

7.4 Flat Belts Problem Statement for Example 7


7. If the coefficient of static friction between the fixed
drums D and E and the ropes is 0.35, determine the
largest weight W that can be supported.

A
100 lb

B
50 lb

C
W

1355

7.4 Flat Belts Problem Statement for Example 8


8. Pulley A is rotating under the action of a 6 N-m torque. This motion is transmitted
through a flat belt to drive pulley B, which is in turn acted upon by a resisting torque T
(the "load" on pulley B). The coefficient of static friction between the belt and the
pulleys is 0.45. Determine a) the maximum possible value of T, b) the maximum force
in the belt, and c) the corresponding force required in the spring C.

6 Nm

80 mm
A
B
Driven pulley

80 mm
Driving pulley

1356

7.4 Flat Belts Example 1, page 1 of 2


1. Determine the minimum number of turns of rope that will allow
the 5-lb force to support the 600-lb block, if the coefficient of
static friction is 0.15.

Tensions in the rope

5 lb
5 lb

n turns

600 lb
600 lb

1357

Impending motion
(The motion can't be up, since the 5-lb force is too
small to lift the 600-lb block.)

7.4 Flat Belts Example 1, page 2 of 2


3

Apply the equation for belt friction:


T2 = T1e

Since

= 0.15, Eq. 1 becomes


600 lb = (5 lb) e

(1)

Solving gives = 31.917 radians. If n is the


number of turns of rope, then the number of
radians is 2 n. Equating this to gives an
equation for n:

In this equation, T2 is the tension in the direction


of impending motion
T2 = 600 lb

2 n=

The other tension, T1, is in the direction opposite


the impending motion, so

= 31.917 rad

T1 = 5 lb

Solving for n gives


n = 5.08 turns
Rounding off to the next higher integer then gives
n=6

1358

Ans.

7.4 Flat Belts Example 2, page 1 of 2


2. If the coefficient of static friction between the rope and the
fixed circular drums A and B is 0.2, determine the largest value
of the force P that can be applied without moving the 150-lb
weight upwards.

Tensions in the rope on either side of drum A


TAB
Impending motion
(The rope must move to
the right if the 150-lb
weight moves up)

P
60

A
150 lb

Because slip impends between the rope and the drum, we can
apply the equation for belt friction:
T2 = T1e

(1)

where T2 is the tension in the direction of impending motion.


In our particular problem,
A

Rope is horizontal.
T2 = TAB
and the tension opposing the impending motion is

150 lb

T1 = 150 lb
Using these results and

= 0.2 and

TAB = (150)e 0.2( /2)


= 205.4 lb

1359

2 in Eq. 1 gives

7.4 Flat Belts Example 2, page 2 of 2


3 Tensions in the rope on
either side of drum B

Flat-belt friction equation:

Impending
motion

T2 = T1e
so,

P = (205.4 lb)e

(2)

60
B

Geometry

TAB = 205.4 lb

= 60 =

60
60
30
60
6

Using

= 60 =

rad in Eq. 2 gives

P = (205.4)e
= 253 lb

1360

/
Ans.

rad

7.4 Flat Belts Example 3, page 1 of 3


3. Determine the smallest force P applied to the handle of the
band brake that will prevent the drum from rotating when the
15 lbft moment is applied. The coefficient of static friction
is 0.25, and the weight of lever arm ABC can be neglected.

Free-body diagram of drum


15 lbft = 15 lbft
= 180 lbin.

6 in.
D

15 lb-ft

Dx

6 in.

TA

D
2
P
80

10 in.

Impending slip of
band relative to
drum (An observer
on the drum would
see the belt move
down.)

TB

15 in.

80

Dy

Equilibrium equation for drum


MD = 0: TA(6 in.)

1361

TB(6 in.)

180 lbin. = 0

(1)

12 in./ft

7.4 Flat Belts Example 3, page 2 of 3


4 The belt-friction equation is in general
T2 = T1e

Geometry
= 180 + 10

(2)

= 190

where T2 is the tension in the direction


of impending slip. In our particular
problem,

10
= (190/180) rad

10

= 3.316 rad

80

T2 = TA

80

and the tension opposing the impending


motion is

Eq. 3 becomes, with

T1 = TB

3.316 and

0.25

TA = TBe0.25(3.316)
Solving Eqs. 1 and 4 simultaneously gives

Eq. (2) becomes


TA = TBe

TA = 53.237 lb

(3)

TB = 23.237 lb

1362

7.4 Flat Belts Example 3, page 3 of 3


6

Free-body diagram of lever arm ABC


TA = 53.237 lb

TB sin 80 = (23.237 lb) sin 80

TB cos 80

Ax
Ay
10 in.

15 in.

Equilibrium equation for lever arm


MA = 0: [(23.237 lb) sin 80](10 in.)

P(10 in. + 15 in.) = 0

Solving gives
P = 9.15 lb

Ans.

1363

(5)

7.4 Flat Belts Example 4, page 1 of 3


4. The uniform beam ABC weighs 40 lb. The coefficient
of static friction between the cord and the fixed drum D is
0.3. Determine the smallest value of the weight W for
which the beam will remain horizontal.

Free-body diagram of block B

TB (Tension in the cord)

B W

4 ft

N=0

4 ft
3

If end A of the beam


is about to move
down, then block B
is about to lose
contact with the
beam. Thus the
normal force N is
zero.

1364

Equilibrium equation for block B


Fy = 0: TB

W=0

(1)

7.4 Flat Belts Example 4, page 2 of 3


4

Free-body diagram of beam


y

TA is the tension in the cord.

N=0
A

Cy
C

Cx

40 lb
4 ft

4 ft

Tensions in
cord on either
side of drum

6 The 40-lb weight of the beam acts


through the center of the span.
TA = 20 lb

Equilibrium equation for the beam

MC = 0: (40 lb)(4 ft)

TA(4 ft + 4 ft) = 0

(2)

TB
Impending
motion

Solving gives
9 Because the cord is on the verge of slipping, we can
apply the equation for belt friction:

TA = 20 lb

T2 = T1e

(3)

where T2 = 20 lb is the tension in the direction of


impending slip and T1 = TB is the tension opposite the
direction of impending motion

1365

7.4 Flat Belts Example 4, page 3 of 3


10 Geometry

TB

TA
11

With

and

0.3, Eq. 3 becomes

T2 = T1e
20 lb

(Eq. 3 repeated)
0.3

Solving gives
TB = 7.79 lb
Eq. 1 then gives
W = TB = 7.79 lb

Ans.

1366

7.4 Flat Belts Example 5, page 1 of 4


5. Determine the largest value W of the weight
of block B for which neither block will move.
The coefficients of static friction are 0.2 between
the blocks and the planes, and 0.25 between the
cord and the drum.

Free-body diagram of block A


x

Impending motion of
block A (Since we are
to determine the
"largest value of the
weight" of block B,
block B must be about
to move down the
plane. Thus block A
must be about to
move up the plane.)

TA
A

fA
B
A
80 lb

80 lb

NA
70

W
3

Equilibrium equations for block A:

50
Fx = 0: TA

70

fA

F = 0: NA

(80 lb) sin

(80 lb) cos

=0

(1)
(2)

Slip impends, so
f A = f A-max
= NA
= (0.2)NA

1367

(3)

7.4 Flat Belts Example 5, page 2 of 4


4

6 Tensions in cord on either side of drum

Geometry
= 70

20

TB

A
TA = 80.65 lb
20
Impending motion
(Block B moves down plane)

70
5

Solving Eqs. 1, 2, and 3, with

= 70 gives
7

TA = 80.65 lb

Because the cord is about to slip over the drum, we can


apply the equation for belt friction:
T2 = T1 e drum

f A = 5.47 lb
NA = 27.36 lb

(4)

where T2 = TB is the tension in the direction of impending


slip, and T1 = 80.65 lb is the tension opposite to the direction
of impending motion.

1368

7.4 Flat Belts Example 5, page 3 of 4


8

Geometry

50
70
40
20
50
70

= 70 + 50
70

= 120
=

50

2
rad
3

Substituting = 2 /3, T2 = TB , T1 = 80.65 lb,


and drum 0.25 in Eq. 4 gives
TB = (80.65 lb) e
Evaluating the right hand side yields
TB = 136.1 lb

1369

7.4 Flat Belts Example 5, page 4 of 4


10 Free-body diagram of block B
y

12 Equilibrium equations for block B:


+

TB = 136.1 lb
x

Fx' =0: NB

W cos

Fy' = 0: 136.1 lb + f B

=0
W sin

(5)
=0

(6)

f B = f B-max

B
fB

= NB
W
50

NB
11 Geometry

Impending
motion

= (0.2)NB
13 Solving Eqs. 5, 6, and 7, with

(7)
= 50 gives

f B = 27.4 lb
NB = 137.2 lb

W = 213 lb

= 50
40
50

1370

Ans.

7.4 Flat Belts Example 6, page 1 of 4


6. A motor attached to pulley A drives the pulley clockwise with a 200 lb-in. torque.
The flat belt then overcomes the resisting torque T at pulley B and rotates the pulley B
clockwise. Determine the minimum tension that can exist in the belt without causing
the belt to slip at pulley A. Also determine the corresponding resisting torque T. The
coefficient of static friction between the belt and the pulleys is 0.3.

T
200 lbin.
A
4 in.

B
7 in.

Driving pulley
Driven pulley
22 in.

1371

7.4 Flat Belts Example 6, page 2 of 4


Free-body diagram of pulley A
TC

Equilibrium equation for pulley A


MA = 0: TD(4 in.)

TC(4 in.)

200 lbin. = 0

(1)

200 lbin.
4 in. A
Ax

Because the belt is about to slip on the pulley, the belt friction
equation applies:

Ay
D

T2 = T1e

TD

where T2 = TD, the tension in the direction of impending motion,


and T1 = TC, so
TD = TC e

Impending motion of belt relative to pulley


(Since pulley A is driven by a clockwise
torque, the pulley would slip in a clockwise
sense relative to the belt. Thus an observer
at point D on the pulley would see the belt
move in the direction shown.)

1372

(2)

7.4 Flat Belts Example 6, page 3 of 4


4

Geometry
Radius = 7 in.
6

4 in.

Free-body diagram of pulley B

C
4 in.

7 in.
22 in.

4 in. = 3 in.

TC = 36.65 lb

From triangle ABC,

3 in.
cos ( 2 ) = 22 in.
Solving gives

= 164.33 = 2.868 rad.

So Eq. 2, with

= 0.3, becomes
0.3(2.868)

TD = TC e

By

(3)
7

Solving Eqs. 1 and 3 simultaneously gives


TC = 36.65 lb
TD = 86.65 lb

7 in.

TD = 86.65 lb
Equilibrium equation for pulley B

Bx

MB = 0: T + (36.65 lb)7 in.

(86.65 lb)7 in. = 0

Solving gives

Ans.

T = 350 lbin.

1373

Ans.

7.4 Flat Belts Example 6, page 4 of 4


8

Finally, we must check that pulley B does not slip. But this follows
from the observation that pulley B has a larger angle of wrap than
pulley A. Thus it must be able to carry a maximum possible tension
larger than the 86.5 lb maximum tension that pulley A carries.
9

More precisely, because


'>
we know that

36.65 lb tension

e
B

'

86.65 lb tension

'

>e .

Multiplying through by 36.65 lb gives


(36.65 lb)e

'

Tmax-B , the maximum


possible tension that
pulley B could support
without slipping.

>

(36.65 lb)e

86.65 lb, by Eq. 2

Thus Tmax-B > 86.65 lb and pulley B won't slip.

1374

7.4 Flat Belts Example 7, page 1 of 2


7. If the coefficient of static friction between the fixed
drums D and E and the ropes is 0.35, determine the
largest weight W that can be supported.

Free-body diagram of block C

TD

TE

A
100 lb
2

Equilibrium equation for block C

50 lb

B
C

Fy = 0: TE + TD

1375

W=0

(1)

7.4 Flat Belts Example 7, page 2 of 2


3

Rope tensions acting on drum D

Rope tensions acting on


drum E (not drum D)
=

=
E
D

TE

50 lb
TD

100 lb

Impending
motion
7

Impending motion (Since we are to determine the largest


value of weight of block C that can be supported, block C
must be about to move down, so the rope connected to C
must also be about to move down.)

Flat-belt friction equation:


T2 = T1e
or,

Because slip is about to occur, the belt-friction equation


applies:

TE = (50 lb)e0.35
= 150.1 lb

T2 = T1e
Here, T2 = TD, T1 = 100 lb,

8
0.35, and

so

TD = (100 lb)e
= 300.3 lb

Using the results of Eqs. 2 and 3 in


Eq. 1 gives
W = 450 lb

(2)

1376

(3)

Ans.

7.4 Flat Belts Example 8, page 1 of 3


8. Pulley A is rotating under the action of a 6 N-m torque. This motion is transmitted
through a flat belt to drive pulley B, which is in turn acted upon by a resisting torque T
(the "load" on pulley B). The coefficient of static friction between the belt and the
pulleys is 0.45. Determine a) the maximum possible value of T, b) the maximum force
in the belt, and c) the corresponding force required in the spring C.

6 Nm

80 mm
A
B
Driven pulley

80 mm
Driving pulley

1377

7.4 Flat Belts Example 8, page 2 of 3


1

Free-body diagram of pulley A

Equilibrium equations for pulley A

T2

Fx = 0: Ax

Fy = 0: Ay = 0

D
6 N-m

Ax

80 mm
4

T1 T2 = 0

MA = 0: T2(0.08 m)

(1)
(2)

T1(0.08 m)

6 Nm = 0

Flat-belt friction equation:


T2 = T1e

Ay

= T1e

T1

(4)

Solving Eqs. 1-4 simultaneously gives


2

(3)

Impending motion (Since the driving torque acting


on pulley A is clockwise, if slip is about to occur,
then pulley A will slip clockwise relative to the
belt. An observer located on point D on pulley A
would see the belt move in the direction shown.)

Ax = 123.21 N
Ay = 0
T1 = 24.11 N
T2 = 99.11 N

1378

Ans.

7.4 Flat Belts Example 8, page 3 of 3


5

Free-body diagram of member AC

Free-body diagram of pulley B


T2 = 99.11 N

Fspring

Ax = 123.21 N

Equilibrium equations for member AC


Fx = 0: Fspring

123.21 N = 0

80 mm
(4)

Bx

T
B

Solving gives
Fspring = 123.21 N

By
Ans.

T1 = 24.11 N
8

Equilibrium equations for pulley B


MB = 0: T + (24.11 N)(0.08 m)

99

(0.08 m) = 0

Solving gives
T = 6 Nm

Ans.

That is, the maximum resisting torque equals the driving


torque.

1379

7.5 Thrust Bearings and Disks

1380

7.5 Thrust Bearings and Disks Procedures and Strategies, page 1 of 1


Procedures and Strategies for Solving Problems Involving
Frictional Forces on Thrust Bearings and Disks
M
1. If the axial force P can be assumed to be distributed
uniformly over the bearing surface, then you can calculate the
resisting torque acting on the shaft (or disk) from the equation

2 P R23 R13
M=
3 R22 R12
2. If the force P is distributed in a non-uniform manner, then
you must calculate M by evaluating an integral:
M = r p dA
where the normal pressure p = p(r) varies with radial distance r
from the center of the disk and satisfies the equation
P = p dA

Coefficient of
friction =

Frictional force = p dA,


so torque = rp dA

R1
r
R2

1381

7.5 Thrust Bearings and Disks Problem Statement for Example 1


1. A shaft rotates at constant angular velocity and is supported
by a collar bearing. Assuming that the normal pressure between
the collar and supporting surface is uniform, determine the
torque M required to overcome friction. The coefficient of
kinetic friction is k = 0.3, and the axial force is 1.5 kN.

70 mm 120 mm
M

1382

7.5 Thrust Bearings and Disks Problem Statement for Example 2


2. A hand-held sander has a rotating disk with a circular piece of
sandpaper attached to the bottom of the disk. When the sander is
applied to the board ABCD, nails are driven in at corners A and C to
keep the board from rotating. Assuming that the downward force
exerted on the sander is P = 50 N and that this force is distributed
uniformly over the sander disk, determine the smallest pair of parallel
forces S from the nails required to keep board in place. The
coefficient of kinetic friction is k = 0.5.

M
Moment and force applied to
the sander
P

D
S
C

150 mm
350 mm

250 mm

1383

7.5 Thrust Bearings and Disks Problem Statement for Example 3


3. Assuming that the contact pressure between the conical
thrust bearing and its support surface is uniform, determine
the torque M required to rotate the shaft at a constant
velocity. The coefficient of kinetic friction is k, and the
axial load is P.

R2
R1
M

1384

7.5 Thrust Bearings and Disks Problem Statement for Example 4


4. A clutch may be modeled as two opposing circular disks loaded by an axial
force P and a torque M. Given that P = 100 lb and the coefficient of static
friction is s = 0.5, determine the maximum torque that can be applied
without slipping if a) the clutch is new, and b) the clutch has experienced
wear proportional to the distance that a point on the clutch surface has
traveled.

4 in.

1385

7.5 Thrust Bearings and Disks Problem Statement for Example 5


5. Because of wear in the collar bearing surface, the pressure
transmitted decreases linearly with radial distance from the
center of the shaft as shown. If the coefficient of kinetic
friction is k = 0.3, and the axial force transmitted by the shaft
is P = 2 kN, determine the torque M required to maintain the
rotation of the shaft at a constant angular velocity.

M
P

30 mm
15 mm

po
r (mm)

p = po

(30 mm r)
(15 mm)

1386

7.5 Thrust Bearings and Disks Example 1, page 1 of 1


1. A shaft rotates at constant angular velocity and is supported
by a collar bearing. Assuming that the normal pressure between
the collar and supporting surface is uniform, determine the
torque M required to overcome friction. The coefficient of
kinetic friction is k = 0.3, and the axial force is 1.5 kN.

70 mm 120 mm
M

1 Because the pressure between the surfaces of


contact is uniform, we can use the following
formula for disc friction:
2
M=
3

kP

R23

R13

R22

R12

(1)

Here
(120 mm/2)3 (70 mm/2)3
2
M = (0.3)(1.5 kN)
3
(120 mm/2)2 (70 mm/2)2
= 21.9 N m

1387

Ans.

7.5 Thrust Bearings and Disks Example 2, page 1 of 2


2. A hand-held sander has a rotating disk with a circular piece of
sandpaper attached to the bottom of the disk. When the sander is
applied to the board ABCD, nails are driven in at corners A and C to
keep the board from rotating. Assuming that the downward force
exerted on the sander is P = 50 N and that this force is distributed
uniformly over the sander disk, determine the smallest pair of parallel
forces S from the nails required to keep board in place. The
coefficient of kinetic friction is k = 0.5.

M
Moment and force applied to
the sander
P

D
S
C

150 mm
350 mm

250 mm

1388

7.5 Thrust Bearings and Disks Example 2, page 2 of 2


1 Because the pressure between the surface of contact is uniform, we can
use the following formula for disk friction:
R23 R13
2
P
M=
(1)
k
3
R22 R12
Here
(150 mm/2)3 (0)3
2
(0.5)(50 N)
3
(150 mm/2)2 (0)2
= 1250 N mm

M=

Free-body diagram of sander and board


ABCD (top view)
D

Forces S must be perpendicular to line AD, if S is to be as


small as possible and still produce the required moment:

Moment equilibrium

S
M = 1250 N mm

MA = S (250 mm)2 + (350 mm)2

Solving gives

350 mm

S = 2.91 N
S
A

1250 N mm = 0

B
250 mm

1389

Ans.

7.5 Thrust Bearings and Disks Example 3, page 1 of 5


3. Assuming that the contact pressure between the conical
thrust bearing and its support surface is uniform, determine
the torque M required to rotate the shaft at a constant
velocity. The coefficient of kinetic friction is k, and the
axial load is P.

R2
R1
M

1390

7.5 Thrust Bearings and Disks Example 3, page 2 of 5


1 Free-body diagram of shaft.
p (Pressure from supporting surfaces; p is not yet known.)
y

x
Mf

2
p

Sum the forces in the x direction.


+

Frictional forces from the


supporting surface produce
a resisting torque Mf .

Fx = 0: P

Rx = 0

(1)

Here Rx is the x component of the resultant


of the pressure p.
Sum the moments about the x axis.

1391

Mx = 0: M

Mf = 0

(2)

7.5 Thrust Bearings and Disks Example 3, page 3 of 5


4 The force on the differential area dA is p dA. Let
dRx equal the x-component of this force:

3 Relate Rx to the pressure p.


dRx

dRx = p dA sin

rd

p dA

(3)

Here
s
ds

dA = r d

ds

Express s in terms of r:

R1
R2

r/s = sin
Thus
ds = dr/sin
and so
5

dA = r d ds

Rx can be now found by integration,


Rx =

=p

dRx

= r d dr/sin
pr d dr, by Eq. 5

Hence by Eq.3 ,

R2 2
r
d dr
0
R1

dRx = p sin
= p sin

= p(R22

R12)

dA

(Eq. 3 repeated)

(r d dr/sin )

(6)
= pr d dr

1392

(5)

7.5 Thrust Bearings and Disks Example 3, page 4 of 5


6

Substituting the expression for Rx in Eq. 6 into Eq. 1 gives


P

Rx= 0

(Eq. 1 repeated)
p(R22 R12), by Eq. 6

Solving this equation for the (unknown) pressure p lets us express p in


terms of the axial force P:

p=

P
(R22

R12)

(7)

1393

7.5 Thrust Bearings and Disks Example 3, page 5 of 5


7

Calculate Mf

Normal force on dA = p dA

r d dr/sin

by Eq. 4
9

Friction force =

normal force

rd
=
r

k pr

d dr/sin

Torque produced by friction force:


dMf = ( kpr d dr/sin

10 Mf can now be found by integration.


Mf = dMf
M, by Eq. 2

kp

sin

2
k pr

R2 2

d dr/sin

r2 dr d

R1 0

2 kp
(R23 R13)
3 sin

(R22
=

3
k P (R2

3 sin

by Eq. 8

R13)
(R22 R12)

R12)

, by Eq. 7

Ans.

1394

(8)

7.5 Thrust Bearings and Disks Example 4, page 1 of 4


4. A clutch may be modeled as two opposing circular disks loaded by an axial
force P and a torque M. Given that P = 100 lb and the coefficient of static
friction is s = 0.5, determine the maximum torque that can be applied
without slipping if a) the clutch is new, and b) the clutch has experienced
wear proportional to the distance that a point on the clutch surface has
traveled.

4 in.

Part a): If the clutch is new, the surface of the disks are flat and it is
reasonable to assume that the pressure transmitted between the disks is
uniform. In this case, we can use the following formula for the disk friction:
M=

2
3

sP

R23
R22

R13
R12

(1)

(4 in.)3 0
2
M = (0.5)(100 lb)
3
(4 in.)2 0
= 133.3 lb.in.

Ans.

1395

7.5 Thrust Bearings and Disks Example 4, page 2 of 4


2

Part b): If the clutch is worn, the pressure between disks is


not uniform, and Eq. 1 can no longer be used.
If the wear on a disk is proportional to the distance
traveled by a point on the disk, then for a point at a
distance r from the center, the wear is proportional to the
circumference 2 r of the circle traced out by the point as
the shaft rotates. That is, wear is proportional to r. But if
the wear between the disks increases with r, it seems
reasonable to suppose that the pressure decreases with r.
The simplest way of modeling this decrease in pressure is
to assume that the pressure varies as 1/r, that is
p = k/r

(2)

where k is a constant of proportionality to be determined.

1396

7.5 Thrust Bearings and Disks Example 4, page 3 of 4


3

Determine the value of k by requiring the resultant of


the pressure forces to equal the axial force P.
d
rd

4 in.

4 Normal force acting on differential


area:

dN
r
M

dr

dN = pr d dr

k/r, by Eq. 2
= k d dr

Equate the axial force P to the resultant of the normal forces:


P = dN
k d dr, by Eq. 3
4 2
=k

d dr
0 0

6 Substituting this result for k into Eq. 2 gives the relation


between the axial force P and the pressure

= (25.133 in.)k

p = k/r

Solving for k gives

k = P/25.133 in.

1397

P
(25.133 in.)r

(Eq. 2 repeated)
(4)

(3)

7.5 Thrust Bearings and Disks Example 4, page 4 of 4


7 Now compute the resultant moment produced by the friction forces.

df = friction force

dN = pr d dr

4 in.

r
M

=
=

dN

spr

d dr

P
, by Eq. 4
(25.133 in.)r
=
8 Equate the torque M applied by the shaft to the resisting
torque produced by the friction forces:
M = r df

sP

d dr
25.133 in.

sP d dr
25.133 in. by Eq. 5

4 2

25.133 in. 0 0

r d dr

(0.5)(100 lb) (50.265 in2)


25.133 in.

= 100 lb in.

sP

Ans.

9 Note that the worn clutch transmits appreciably less


torque without slipping than the new clutch
transmits (100 lb in. compared to 133 lb in.)

1398

(5)

7.5 Thrust Bearings and Disks Example 5, page 1 of 3


5. Because of wear in the collar bearing surface, the pressure
transmitted decreases linearly with radial distance from the
center of the shaft as shown. If the coefficient of kinetic
friction is k = 0.3, and the axial force transmitted by the shaft
is P = 2 kN, determine the torque M required to maintain the
rotation of the shaft at a constant angular velocity.

M
P

30 mm
15 mm

po
r (mm)

p = po

(30 mm r)
(15 mm)

1399

7.5 Thrust Bearings and Disks Example 5, page 2 of 3


1

Determine the value of p0 by requiring the resultant of the


pressure force to equal the axial force P.
M
dN = normal force on differential area, dA
P

=p

dA

= p0

(30 mm r)
r d dr
15 mm

Equate the axial force P (= 2 kN) to the resultant of the


normal forces:
2 kN =

15 mm

dN

30 mm 2

dN
30 mm

= p0
15 mm 0

rd

(1)

(30 mm r)
r d dr
15 mm

= p0 (942.478 mm2)
r

dr

Solving for p0 gives


p0 = 2.122 N/mm2

1400

(2)

7.5 Thrust Bearings and Disks Example 5, page 3 of 3


3 Now compute the resultant moment produced by
friction forces.
M
P
5

Equate the torque M delivered by the shaft to the resisting


torque produced by friction
M = r df
k p0

15 mm

dN
30 mm

30 mm 2

k po

(30 mm

15 mm

0.3

df

df = friction force
=

dN

(3)

p0 (30 mm r) r d dr, by Eq. 1


15 mm

1401

r) r2 d dr

15 mm 0
2.122 N/mm2, by Eq. 2

= 12.38 N m
r

(30 mm r)
r d dr, by Eq.3
15 mm

Ans.

7.6 Journal Bearings

1402

7.6 Journal Bearings Procedures and Strategies, page 1 of 2


Procedures and Strategies for Solving Problems
Involving Frictional Forces on Journal Bearings

F
Pulley

For problems involving a loosely fitting pulley (or wheel


or collar or similar device) on a fixed shaft, follow these
steps.

Fixed shaft

1. Calculate the angle of friction and radius rf of the


circle of friction from the equations
tan-1

where

Belt forces
acting on
pulley are
equal.

P1
Contact point

is the coefficient of friction

F
For clarity, the difference in size between the
radius of the shaft and the inner radius of the
pulley has been exaggerated in the diagram.

rf = r sin where r is the radius of the shaft


(which, because the pulley fits "loosely", is
approximately equal to the inner radius of the
pulley)
2. Draw circles showing the positions and contact
point of the shaft and the central hole in the pulley,
when no rotation has occurred.

F
Pulley

3. Draw circles showing the positions and contact


point of the shaft and the pulley hole, after rotation
has occurred.

Belt forces
acting on
pulley are
not equal.

Fixed shaft

Unbalanced forces
cause pulley to rotate to
new contact point.

1403

P2
F+ F

7.6 Journal Bearings Procedures and Strategies, page 2 of 2


Circle of friction
(not the shaft)

4. Draw a sketch of the pulley, with the circle of friction inside. On


this sketch, show the friction force from the shaft opposing the
rotation. Furthermore, show the line of action of the resultant, say
R, of the normal and friction force passing through the contact
point and lying tangent to a circle of radius rf (the circle of friction).

rf

Frictional force
opposes rotation

5. Draw a free-body diagram of the pulley, expressing R in terms


of x and y components, Rx and Ry.
6. Write equilibrium equations: Sum forces in the x and y
directions and sum moments about the center of the pulley.
Note that the moment of R is R rf , because R is tangent to
the circle of friction, which has radius rf . Also,
R=

Pulley

Sense of
rotation of
pulley

f
P2
N

Line of action of
resultant, R, is
tangent to circle
of friction.

Rx2 + Ry2

7. Solve the equations.

Note: The above discussion applies when the pulley fits loosely on
a fixed shaft. If instead the pulley is rigidly fixed to the shaft, but
the shaft fits loosely on a supporting bearing, then the steps above
can be followed, except that a) the shaft rotates to a new contact
point relative to the fixed bearing, and b) a free-body diagram of
the shaft rather than the pulley is used.

Pulley

rf
x

Rx
P2
Ry
R = Rx2 + Ry2

1404

F+ F

7.6 Journal Bearings Problem Statement for Example 1


1. The pulley has a radius of 80 mm and has
negligible weight. If the pulley fits loosely on a
12-mm-diameter fixed shaft, and the coefficient of
static friction is s = 0.2, determine the minimum
force F required to start the pulley rotating
clockwise.

80 mm

140 N

1405

7.6 Journal Bearings Problem Statement for Example 2


2. The pulley has a radius of 80 mm and has negligible
weight. If the pulley fits loosely on a 12-mm-diameter
fixed shaft, and the pulley rotates with constant angular
velocity counterclockwise, determine the coefficient of
kinetic friction, k.

135 N

80 mm

140 N

1406

7.6 Journal Bearings Problem Statement for Example 3


3. The force from the belt causes the 130-mm-radius
pulley to rotate counterclockwise with constant angular
velocity. The pulley fits loosely on a fixed shaft of
24-mm diameter. Determine the value of the belt force F
if the coefficient of kinetic friction is k = 0.3. Assume
that no slipping occurs between the belt and pulley.
100 N

130 mm

1407

7.6 Journal Bearings Problem Statement for Example 4


4. The stepped pulley has radii of 4 in. and 8 in. and fits loosely on a
0.5-in.-diameter fixed shaft. If the given loads cause the pulley to
rotate clockwise with a constant angular velocity, determine the
normal and frictional forces from the shaft acting on the pulley. Also
find the coefficient of kinetic friction, k .

8 in.
4 in.

20 lb

40.5 lb

1408

7.6 Journal Bearings Problem Statement for Example 5


5. The couple forces P are intended to rotate the shaft and wind
the cable around the drum, thus raising the 5-kg mass. The
shaft has a diameter of 30 mm and fits loosely in the journal
bearing B. If the combined mass of the shaft handle A and
drum C is 15 kg, and the coefficient of static friction is s = 0.2,
determine the minimum value of P required to initiate upward
motion of the 5-kg mass. Assume the drum C is attached
rigidly to the shaft.

Diameter of drum = 200 mm


C
5 kg
B

P
A

150 mm

150 mm

1409

7.6 Journal Bearings Problem Statement for Example 6


6. Pulleys A and B are identical. Each has a radius of 10 in.,
weighs 10 lb and fits loosely on a 0.5-in.-diameter fixed shaft.
Determine the maximum value of the weight W that may be
supported without causing the pulleys to rotate. Also calculate
the value of the tension in the cord between the pulleys. The
coefficient of static friction is s = 0.2 .

10 in.

10 in.

100 lb

1410

7.6 Journal Bearings Problem Statement for Example 7


7. The 100-kg cart has four 200-mm-diameter wheels,
25-mm-diameter axles, and center of mass G. Each wheel has
a mass of 10 kg. The coefficient of kinetic friction is 0.06.
Determine the horizontal force P required to move the cart at
constant speed. Assume that the axles do not rotate, the wheels
fit loosely on the axles, and rolling resistance between the
wheels and the plane is negligible.

100 kg
G
P
100 mm

400 mm

400 mm

1411

7.6 Journal Bearings Problem Statement for Example 8


8. The 100-kg cart has four 300-mm-diameter wheels,
30-mm-diameter axles, and center of mass G.
Determine the angle for which the cart will roll down
the incline at constant speed. Also determine the
reactions from the incline acting on each wheel. The
coefficient of kinetic friction is 0.2. Assume that
rolling resistance and the weight of the wheels are
negligible, the wheels fit loosely on the axles, and the
axles do not rotate.

200 mm

500 mm
500 mm

1412

7.6 Journal Bearings Example 1, page 1 of 3


1. The pulley has a radius of 80 mm and has
negligible weight. If the pulley fits loosely on a
12-mm-diameter fixed shaft, and the coefficient of
static friction is s = 0.2, determine the minimum
force F required to start the pulley rotating
clockwise.

F
1 The angle of friction and the radius of the circle
of friction are

80 mm

= tan-1

= tan-10.2
= 11.3099
140 N

rf = r sin

= (12/2 mm) sin 11.3099


= 1.1767 mm

1413

(1)

7.6 Journal Bearings Example 1, page 2 of 3


Pulley rotates clockwise.

For clarity, the difference in size between the


radius of the shaft and the inner radius of the
pulley has been exaggerated in the diagram.
Suppose
F = 140 N

Fixed shaft

New contact
point

Contact
point

P2
P1

P1
Pulley

Pulley
140 N

140 N

Position of pulley if rope


forces were equal

1414

Rotated position of pulley caused by


increase in force F.

7.6 Journal Bearings Example 1, page 3 of 3


Line of action of
resultant, R, is tangent
to circle of friction.
N
P2
f

rf
O

Frictional force f
opposes clockwise
rotation of pulley

6 For convenience, express


R in terms of horizontal
and vertical components
(rather than in terms of N
and f).

Ry
P2

R = Rx2 + Ry2
Rx

80 mm

rf
O
80 mm
Pulley

Pulley

7 Equilibrium equations

Normal force N and friction force f


from shaft acting on pulley.

Fx = 0: F + 140 N
Fy = 0:

Free-body diagram of pulley

140 N
Rx = 0

Ry = 0

MO = 0: (140 N)(80 mm)

F(80 mm)

+ Rx2 + Ry2 (rf ) = 0


1.1767 mm, by Eq. 1
Solving gives
Rx = 284 N
Ry = 0
F = 144.2 N

1415

Ans.

7.6 Journal Bearings Example 2, page 1 of 4


2. The pulley has a radius of 80 mm and has negligible
weight. If the pulley fits loosely on a 12-mm-diameter
fixed shaft, and the pulley rotates with constant angular
velocity counterclockwise, determine the coefficient of
kinetic friction, k.

135 N
1 The angle of friction, k, and radius of the circle, rf ,
can be expressed in terms of k:

80 mm

= tan-1

rf = r sin

k
k

= (12/2 mm) sin ( tan-1 k)


140 N

= 6 sin ( tan-1 k)

1416

(1)

7.6 Journal Bearings Example 2, page 2 of 4


For clarity, the difference in size between the
radius of the shaft and the inner radius of the
pulley has been exaggerated in the diagram.
Fixed shaft

Pulley rotates counterclockwise.

135 N

Contact
point

135 N

New contact
point
P1

P1

P2
Pulley

Pulley

Suppose that the


140 N force is
replaced by a 135
N force.

140 N

Position of pulley if belt


forces were equal

1417

Rotated position of pulley caused by


increase of belt force to 140 N.

7.6 Journal Bearings Example 2, page 3 of 4


Line of action of resultant, R, is
tangent to circle of friction.

Frictional force opposes


counterclockwise
rotation of pulley.

rf

f
P2

R
N
4

Normal force N and friction force f


from shaft acting on pulley.

1418

7.6 Journal Bearings Example 2, page 4 of 4


5

Free-body diagram of pulley

135 N

Equilibrium equations
+ Fx = 0: 135 N + 140 N

Fy = 0:

Rx = 0

Ry = 0

80 mm
MO = 0: (140 N)(80 mm)

O
rf
Rx
R = Rx2 + Ry2

P2
Ry

Rx2 + Ry2 (rf ) = 0


Solving gives

80 mm

Rx = 275 N
140 N

For convenience, express R in


terms of horizontal and vertical
components (rather than in
terms of N and f).

(135 N)(80 mm)

Ry = 0
rf = 1.455 mm
8

Substituting the value of rf in Eq. 1 gives


rf = 6 sin (tan-1 k)

(Eq. 1 repeated)

1.455 mm
Thus
sin-1(1.455/6) = tan-1
k

= tan [sin-1(1.455/6)]
= 0.25

1419

Ans.

7.6 Journal Bearings Example 3, page 1 of 4


3. The force from the belt causes the 130-mm-radius
pulley to rotate counterclockwise with constant angular
velocity. The pulley fits loosely on a fixed shaft of
24-mm diameter. Determine the value of the belt force F
if the coefficient of kinetic friction is k = 0.3. Assume
that no slipping occurs between the belt and pulley.
1

100 N

Calculate the angle of friction and the radius of


the circle of friction.
k

130 mm

= tan-1

= tan-1 0.3
= 16.6992
rf = r sin

= (24/2 mm) sin 16.6992


F

= 3.4482 mm

1420

(1)

7.6 Journal Bearings Example 3, page 2 of 4


For clarity, the difference in size between the radius
of the shaft and the inner radius of the pulley has
been exaggerated in the diagram.

Pulley rotates counterclockwise.

Contact point

100 N

New contact point

100 N

P1
Fixed shaft

P2

P1

Pulley A
Pulley A

Suppose F = 100 N
2

Position of pulley if belt force were equal


3

1421

Rotated position of pulley caused by a


decrease in value of force F.

7.6 Journal Bearings Example 3, page 3 of 4


Line of action of resultant, R, is tangent to circle of
friction.

N
P2

Frictional force opposes


counterclockwise rotation of
pulley.

rf

Pulley A

4 Normal force N and friction force f


from shaft action on pulley

1422

7.6 Journal Bearings Example 3, page 4 of 4


For convenience, express R in
terms of horizontal and vertical
components (rather than in
terms of N and f).

100 N
Ry
P2
rf

Rx

Fx = 0: Rx

100 N = 0

Fy = 0: Ry

F=0

130 mm

6 Equilibrium equations

R = Rx2 + Ry2

MO = 0: (100 N)(130 mm)

F(130 mm)

Rx2 + Ry2 (rf ) = 0


3.4482 mm, by Eq. 1

Pulley A

Solving gives
F

Rx = 100 N

130 mm

Ry = 96.3 N

F = 96.3 N

1423

Ans.

7.6 Journal Bearings Example 4, page 1 of 5


4. The stepped pulley has radii of 4 in. and 8 in. and fits
loosely on a 0.5-in.-diameter fixed shaft. If the given loads
cause the pulley to rotate clockwise with a constant angular
velocity, determine the normal and frictional forces from
the shaft acting on the pulley. Also find the coefficient of
kinetic friction, k .

8 in.
4 in.

1 The angle of friction, k, and radius of friction circle,


rf , can be expressed in terms of k :
k

= tan-1k

rf = r sin

= (0.5/2 in) sin (tan-1k )


= 0.25 sin (tan-1k )
20 lb

40.5 lb

1424

(1)

7.6 Journal Bearings Example 4, page 2 of 5


For clarity, the difference in size between the radius
of the shaft and the inner radius of the pulley has
been exaggerated in the diagram.

Pulley rotates clockwise.

Contact point
Fixed shaft
Pulley

P1

8 in.

20 lb

P1

4 in.

40.5 lb

8 in.

20 lb

New contact
point

P2

4 in.

40.5 lb

Replace by 40 lb
2 Position of pulley if belt force were
slightly altered to produce equal moments
(20 lb 8 in. = 40 lb 4 in.) about the
center of the pulley.

3 Rotated position of pulley caused by an increase of


belt force on the right to 40.5 lb.

1425

7.6 Journal Bearings Example 4, page 3 of 5


Frictional force f opposes the
clockwise rotation of the pulley.
R
Line of action of resultant, R, is
tangent to circle of friction.

f
N
P2
rf

Normal force N and friction force f


from shaft acting on pulley.

1426

7.6 Journal Bearings Example 4, page 4 of 5


5 Free-body diagram of pulley
R = Rx 2 + R y 2

Ry

6
P2
O

Rx

rf

For convenience,
express R in terms of
horizontal and
vertical components
(rather than in terms
of N and f).

Equilibrium equations

7
+

Fx = 0: Rx = 0

8 in.

Fy = 0: Ry

4 in.

40.5 lb

(40.5 lb)(4 in.)

Rx2 + Ry2 rf = 0

Solving gives
Rx = 0
Ry = 60.5 lb
rf = 0.0331 in.

1427

40.5 lb = 0

MO = 0: (20 lb)(8 in.)


+

20 lb

20 lb

7.6 Journal Bearings Example 4, page 5 of 5


8 Substituting the value of rf just computed into Eq. 1
gives
rf = 0.25 sin (tan-1k)

(Eq.1 repeated)

R = Rx2 + Ry2 = 0 + (60.5 lb)2 = 60.5 lb


(line of action passes through P2 and is tangent to the
friction circle)

0.0331 in.

Thus,
sin-1 (0.0331/0.25) = tan-1k

N (line of action passes


through P2 and center of
circle, O)

or
-1

k = tan sin (0.0331/0.25)

= 0.134
9

Ans.

The normal force N and friction force f can be


found by resolving the resultant R into components:
= cos-1(0.0331 in./0.25 in.)
f = R cos
f
= (60.5 lb) cos 82.392
= 8.01 lb

P2

rpulley = 0.25 in.


O

Ans.

Circle of friction
drawn to scale
relative to the
0.25-in. radius
of the hole in the
pulley

N = R sin
= (60.5 lb) sin 82.392
= 60.0 lb

= 82.392

Ans.

1428

rf = 0.0331 in.

7.6 Journal Bearings Example 5, page 1 of 4


5. The couple forces P are intended to rotate the shaft and wind
the cable around the drum, thus raising the 5-kg mass. The
shaft has a diameter of 30 mm and fits loosely in the journal
bearing B. If the combined mass of the shaft handle A and
drum C is 15 kg, and the coefficient of static friction is s = 0.2,
determine the minimum value of P required to initiate upward
motion of the 5-kg mass. Assume the drum C is attached
rigidly to the shaft.

Diameter of drum = 200 mm


C

1
5 kg

The angle of friction and the radius of circle of


friction are
= tan-1s

= tan-10.2

= 11.3099
150 mm

150 mm

rf = r sin 11.3099
= (30 mm/2) sin 11.3099
= 2.9394 mm

1429

(1)

7.6 Journal Bearings Example 5, page 2 of 4


For clarity, the difference in size between the
radius of the shaft and the radius of the support
bearing has been exaggerated in the diagram.

Rotation of shaft counterclockwise

Fixed support bearing at B

Shaft (free to rotate


in support bearing)

P2

New
contact
point

P1
Contact
point

P1
3

Position of the shaft, if external


forces were balanced so that no
friction force exists at point P1. That
is, no tendency exists to either raise
or lower the weight.

1430

Rotated position of shaft caused by


increasing the forces P on the handle (The
shaft tends to "climb" the wall of the bearing
until it reaches a point where slipping
occurs).

7.6 Journal Bearings Example 5, page 3 of 4

Line of action of
resultant, R, is tangent
to circle of friction

rf
P2
N

P1

R
Frictional force opposes the
counterclockwise rotation of the shaft.
4 Normal force N and friction force f
from bearing acting on the shaft.

1431

7.6 Journal Bearings Example 5, page 4 of 4


Free body of shaft, handle A, and drum

Weight of handle and drum = (15 kg)(9.81 m/s2


= 147.15 N
Drum C

Equilibrium equations

Shaft

Fx = 0: Rx = 0
Fy = 0: Ry + P

rf

R = Rx + Ry

(49.05 N)(100 mm)

2.9394 mm, by Eq. 1

Weight of block
= (5 kg)(9.81 m/s2
= 49.05 N

Ry

200/2 mm = 100 mm
150 mm
6

MO = 0: (2 P)(150 mm)

49.05 N = 0

(Rx2 + Ry2) (rf ) = 0

Rx

P 147.15 N

150 mm

Solving gives
Rx = 0
Ry = 196.2 N
P = 18.3 N

For convenience, express R in


terms of horizontal and vertical
components (rather than in
terms of N and f).

1432

Ans.

7.6 Journal Bearings Example 6, page 1 of 7


6. Pulleys A and B are identical. Each has a radius of 10 in.,
weighs 10 lb and fits loosely on a 0.5-in.-diameter fixed shaft.
Determine the maximum value of the weight W that may be
supported without causing the pulleys to rotate. Also calculate
the value of the tension in the cord between the pulleys. The
coefficient of static friction is s = 0.2 .

1
10 in.

10 in.

The angle of friction and the radius of the circle


of friction are
s

= tan-1s

= tan-10.2
A

= 11.3099
rf = r sin

100 lb

= (0.5 in./2) sin 11.3099


= 0.0490 in.

1433

(1)

7.6 Journal Bearings Example 6, page 2 of 7


Pulley rotates clockwise (We are to find the
maximum value of the weight W, so
impending motion of the cord is to the right).

For clarity, the difference in size between the


radius of the shaft and the inner radius of the
pulley has been exaggerated in the diagram.
Fixed
shaft
Contact
point

Suppose
cord tension
is T = 100 lb

New
contact
point

P1

T > 100 lb
P2
P1

Pulley A
Pulley A
100 lb
100 lb
2 Position of pulley A if cord
forces were equal

3 Position of pulley A caused by


increase in value of force T

1434

7.6 Journal Bearings Example 6, page 3 of 7


Line of action of resultant, R, is
tangent to circle of friction.
R

Frictional force
opposes clockwise
rotation of pulley.

N
P2

f
rf

Pulley A

Normal force N and friction force f


from shaft acting on pulley A

1435

7.6 Journal Bearings Example 6, page 4 of 7


Free-body diagram of pulley A

R = Rx2 + Ry2
For convenience, express
R in terms of horizontal
and vertical components
(rather than in terms of N
and f).

T
Ry
P2

Fx = 0: T
Fy = 0: Ry

Rx = 0
100 lb 10 lb = 0

10 in.

Rx

7 Equilibrium equations

rf

MO = 0: (100 lb)(10 in.)

T(10 in.)

(Rx2 + Ry2)(rf ) = 0
0.0490 in., by Eq. 1

Pulley A

10 lb
Solving simultaneously gives

100 lb
10 in.

Rx = 100.7309 lb
(Weight of the pulley)
Ry = 110.0000 lb
T = 100.7309 lb

1436

Ans.

7.6 Journal Bearings Example 6, page 5 of 7


Pulley rotates clockwise.

T = 100.7309 lb

Contact point

T = 100.7309 lb
New contact
point
Q1

Q1

Q2
Fixed
shaft
Pulley B

Pulley B

W > 100.7309 lb

Suppose W = T = 100.7309 lb

Position of pulley B if cord forces were equal

1437

Rotated position of pulley B, caused by


increase in value of weight W

7.6 Journal Bearings Example 6, page 6 of 7


Frictional force
opposes clockwise
rotation of pulley.

Line of action of resultant, S, is


tangent to circle of friction.
S
f

N
Q2

rf

Pulley B

10 Normal force N and friction force f


from shaft acting on pulley B

1438

7.6 Journal Bearings Example 6, page 7 of 7


11 Free-body diagram of pulley B

S = Sx2 + Sy2

Fx = 0: Sx 100.7309 lb = 0

T = 100.7309 lb

12 Write Equilibrium equations

Fy = 0: Sy

10 lb = 0

Sy
10 in.
Q2
O

MO = 0: (100.7309 lb)(10 in.)

W(10 in.)

+ Sx2 + Sy2 (rf ) = 0

Sx

rf

0.0490 in., by Eq. 1


Solving Simultaneously gives

Pulley B
10 lb
(Weight of the pulley)

10 in.

Sx = 100.7 lb
W

Sy = 111.5 lb
W = 101.5 lb

1439

Ans.

7.6 Journal Bearings Example 7, page 1 of 5


7. The 100-kg cart has four 200-mm-diameter wheels,
25-mm-diameter axles, and center of mass G. Each wheel has
a mass of 10 kg. The coefficient of kinetic friction is 0.06.
Determine the horizontal force P required to move the cart at
constant speed. Assume that the axles do not rotate, the wheels
fit loosely on the axles, and rolling resistance between the
wheels and the plane is negligible.

100 kg
G
P
100 mm

400 mm

400 mm

The angle of friction and the radius of the circle of


friction are
k=

tan-1s = tan-10.06 = 3.4336

rf = r sin

= (25/2 mm) sin 3.4336


= 0.7486 mm

1440

(1)

7.6 Journal Bearings Example 7, page 2 of 5


2

Free-body diagram of cart


Rolling resistance is negligible (The
coefficient of rolling resistance is
zero), so the force from the plane
acting on the wheel is concentrated at
a single point the point where the
wheel is tangent to the plane.

Weight of 100-kg
cart = 981 N
Weight of 2
10-kg wheels
= 2(98.1 N)
G

P
100 mm
2F1

Factor of "2"
because 2 rear
wheels.

400 mm

2F2
400 mm

2G1

2G2
3 Equilibrium equations

Forces G1 and G2 normal to the


plane and F1 and F2 parallel to the
plane act on the four wheels.

Fx = 0: P

2F1 2F2 = 0

Fy = 0: 2G1 + 2G2

981 N

(1)
2(98.1 N)

2(98.1 N) = 0

MA= 0: 2(98.1 N)(400 mm) 2(98.1 N)(400 mm)


+ (2G2)(400 mm) (2G1)(400 mm) P(100 mm) = 0

1441

(2)

(3)

7.6 Journal Bearings Example 7, page 3 of 5


Clockwise rotation of wheel

Axle (does not


rotate)

P1

New contact point

Wheel P1

Wheel

Friction force from plane


acting on wheel

Contact point
4

P2

Relative position of axle and wheel when


cart is not moving (No friction force from
the plane acts on the wheel).

1442

Relative position of axle and wheel when


cart is moving (As the friction force from
the plane increases from zero, it causes
the wheel to rotate about the contact point
P1 initially. But this rotation causes the
contact point to shift up and to the left on
the axle. As the force increases further
and the wheel rotates more, the contact
point moves further left and up until
slipping occurs).

7.6 Journal Bearings Example 7, page 4 of 5

rf
Line of action of
resultant, R, is
tangent to circle of
friction.

P2
f
N

Frictional force opposes


clockwise rotation

R
6 Normal force N and friction
force f from axle acting on wheel.

1443

7.6 Journal Bearings Example 7, page 5 of 5

Weight of wheel
= 98.1 N

rf
O

P2

8 Equilibrium equations:
+

Fx = 0: Rx1 F1 = 0

Free-body diagram of a rear wheel

Fy = 0: G1 98.1 N

Rx1

200/2 mm = 100 mm

F1

Ry1
R=

(Rx1)2 + (Ry1)2

MO = 0:

Ry1 = 0

(F1)(100 mm) +

(5)

(Rx1)2 + (Ry1)2 (rf ) = 0

(6)

A free-body diagram of a front wheel of the cart would appear


similar to the free-body diagram of the rear wheel. Thus the
equilibrium equations for a front wheel are identical to those
above except the subscript "1" is replaced by "2":
Fx = 0: Rx2 F2 = 0
Fy = 0: G2
MO = 0:

G1

(4)

98.1 N

(7)
Ry2 = 0

(F2)(100 mm) +

(8)

(Rx2)2 + (Ry2)2 (rf ) = 0

(9)

Substituting the value rf = 0.7486 mm and then solving Eqs. 1-9


gives
Rx1 = F1 = 1.83 N
G2 = 343.81 N
P = 7.34 N

1444

Rx2 = F2 = 1.84 N

G1 = 342.89 N

Ry1 = 244.79 N Ry2 = 245.71 N


Ans.

7.6 Journal Bearings Example 8, page 1 of 5


8. The 100-kg cart has four 300-mm-diameter wheels,
30-mm-diameter axles, and center of mass G.
Determine the angle for which the cart will roll down
the incline at constant speed. Also determine the
reactions from the incline acting on each wheel. The
coefficient of kinetic friction is 0.2. Assume that
rolling resistance and the weight of the wheels are
negligible, the wheels fit loosely on the axles, and the
axles do not rotate.

200 mm

500 mm
500 mm

1 The angle of friction and the radius of the


circle of friction are
k

= tan-1s = tan-10.08 = 4.5739

rf = r sin

= (30/2 mm) sin 4.5739


= 1.1962 mm

1445

(1)

7.6 Journal Bearings Example 8, page 2 of 5


2

Free-body diagram of cart.


y
x

200 mm
Weight of 100 kg mass = 981 N

G
2F2

Rolling resistance is negligible (The


coefficient of rolling resistance is
zero), so the force from the plane
acting on the wheel is concentrated
at a single point the point where
the wheel is tangent to the plane.

2G2

A
2F1

500 mm
2G1

500 mm

Factor of 2 to account
for 2 front wheels

Equilibrium equations
+

Fx = 0: 2F1 + 2F2

Fy = 0: 2G1 + 2G2

(981 N) sin

MA = 0: [(981 N) sin ](200 mm) [(981 N) cos ](500 mm)


+ (2G2)(500 mm + 500 mm) = 0

(981 N) cos

1446

=0
=0

(2)
(3)

(4)

7.6 Journal Bearings Example 8, page 3 of 5


Axle (does not
rotate)

Wheel

P2
P1
Contact point
between axle
and wheel

Counterclockwise
rotation of wheel

P1

New
contact
point

Friction force from plane


acting on wheel
5 Relative position of wheel and axle when
cart is moving (As the friction force from the
plane increases from zero, it causes the
wheel to rotate about the contact point P1
initially. But this rotation causes the contact
point to shift up and to the right on the axle.
As the force increases further and the wheel
rotates more, the contact point moves further
right and up until slipping occurs).

4 Relative position of wheel and axle when cart is not


moving (We suppose that the cart is held in place by
a force that is parallel to the plane and is equal and
opposite to the component of the weight down the
plane). No friction force from the plane acts on the
wheel.

1447

7.6 Journal Bearings Example 8, page 4 of 5


8 Free-body diagram of front wheel

6 Force from axle acting on wheel.


Circle of friction
(radius rf )
rf

rf
Frictional force
opposes the
counterclockwise
rotation of the
wheel.

P2
f
P2
N
R

7 R is the resultant of the normal


force, N, and the frictional force, f,
from the axle acting on the wheel.
The line of action of R is tangent to
the circle of friction and passes
through the contact point P2.

9 We can save some work if we


use the fact that the wheel is a
two-force member. Thus the
force R from the axle must
have the same line of action as
the resultant of the forces F1
and G1 from the inclined plane.
That is, both lines of action
pass through P2 and the point
of contact, A, with the inclined
plane. It follows from
geometry that F1 and G1 must
be related by the angle :
tan

1448

F1/G1

(5)

F1

G1

Resultant of
forces F1 and
G1 from the
plane acting
on the wheel

7.6 Journal Bearings Example 8, page 5 of 5


10 From geometry,
= sin-1(rf /150)

1.1962 mm, by Eq. 1


rf

= 0.4569

Using this result in Eq. 5 gives

300/2 mm = 150 mm

tan

P2

= F1/G1

(Eq. 5 repeated)

tan 0.4569 = F1/G1

(6)

or
A

Because the geometry is the same for both wheels,


tan 0.4569 = F2/G2

(7)

Solving Eqs. 2-4, 6 and 7 gives


F1 = 1.96 N, G1 = 246 N
F2 = 1.95 N, G2 = 244 N

= 0.457

1449

Ans.

7.7 Rolling Resistance

1450

7.7 Rolling Resistance Procedures and Strategies, page 1 of 1


Procedures and Strategies for Solving Problems Involving
Rolling Resistance
1. Locate the point where the resisting force from the plane
acts on the wheel. This point lies on the circumference
of the wheel at a distance, measured parallel to the plane,
given by the coefficient of rolling resistance, "a". The
resisting force is oriented so that its component parallel
to the plane opposes the motion of the wheel relative to
the plane. The line of action of the resisting force will
pass through center of the wheel if and only if the line of
action of the resultant of the other forces acting on the
wheel passes through the center.

Direction of rotation of wheel


Direction of motion
of center of wheel
relative to plane
(Plane may be
moving)

2. For later use in calculating moments, determine the


horizontal and vertical distances between the center of
the wheel and the point of application of the resisting
force.

R (Resisting force
opposing the motion of the
wheel relative to the plane)

3. Solve the remainder of the problem in the same way you


would solve any two-dimensional rigid-body equilibrium
problem, drawing free-body diagrams and writing
equations of equilibrium.

O
r2

a2

A
Rx

a
Ry

1451

7.7 Rolling Resistance Problem Statement for Example 1


1. The fertilizer spreader and the fertilizer it contains have a combined
mass of 40 kg and a center of gravity located at point G. If the coefficient
of rolling resistance for the tires is 5 mm, determine the horizontal and
vertical components, Px and Py, of the force that must be applied to the
handle to move the spreader at a constant speed.
250 mm
Px
100 mm
Py
G

850 mm

Radius = 120 mm

1452

7.7 Rolling Resistance Problem Statement for Example 2


2. The mass of the piano is 200 kg. The coefficient of rolling
resistance of the casters is 0.4 mm. Determine the value of the
horizontal force P required to move the piano at constant speed.
Make the simplifying assumption that the resisting forces are
the same at all four casters.

Radius of the
casters = 14 mm

1453

7.7 Rolling Resistance Problem Statement for Example 3


3. If the coefficient of rolling resistance is 0.3 in., determine the
magnitude of the horizontal force F required to push the 300-lb
drum up the inclined plane at constant velocity.

8 in.
F

15

1454

7.7 Rolling Resistance Problem Statement for Example 4


4. A 100-lb steel-rim wheel of 24-in. diameter rolls
at constant velocity down the inclined plane. If the
coefficient of rolling resistance is 0.08 in.,
determine the angle .

1455

7.7 Rolling Resistance Problem Statement for Example 5


5. If the coefficient of rolling resistance at the top of the
cylinder is 0.4 mm and at bottom of the cylinder is 0.8 mm,
determine the horizontal force P required to start the block
moving to the left. The weight of the cylinder is negligible
compared to the weight of the block.

100 kg

80 mm

1456

7.7 Rolling Resistance Problem Statement for Example 6


6. The ancient Britons who constructed the prehistoric monument
Stonehenge moved massive stones over twenty miles from a quarry to
the monument site. One possible way that they might have done this is
to roll the stones over logs laid on the ground. Assuming that the
coefficient of rolling resistance for the top of a log is 0.01 ft and the
coefficient is 0.08 ft for the bottom, estimate the horizontal force
required to move the typical stone shown below. Make the simplifying
assumption that the resisting forces are the same for each log.
Rope

24 ft

Force of men
(and perhaps
oxen)
F

100,000 lb
4 ft

Diameter of
logs = 1 ft.

1457

7.7 Rolling Resistance Example 1, page 1 of 4


1. The fertilizer spreader and the fertilizer it contains have a combined
mass of 40 kg and a center of gravity located at point G. If the coefficient
of rolling resistance for the tires is 5 mm, determine the horizontal and
vertical components, Px and Py, of the force that must be applied to the
handle to move the spreader at a constant speed.
250 mm
Px
100 mm
Py
G

850 mm

Radius = 120 mm

1458

7.7 Rolling Resistance Example 1, page 2 of 4


1

Locate the point where the resisting force


from the ground acts on the wheel.

4 For later use in calculating moments, find the


vertical distance between A and B.

Rotation
of wheel

A
Direction of motion
of center of wheel

100 mm

B
B

(100 mm)2
5 mm

2 As the wheel rolls to R


the left, the ground
exerts a force, R,
opposing the motion.

5 mm ( = coefficient of rolling resistance)


3

R acts at a point B on the


circumference 5 mm
from a vertical line
through the center A of
the wheel.

1459

(5 mm)2

= 99.875 mm

7.7 Rolling Resistance Example 1, page 3 of 4


5

Free-body diagram of spreader.


250 mm
Px
100 mm

Equilibrium equations

Py
+

9.81 m/s

850 mm

Px = 0

Fy = 0: 2Ry + Py

Weight = 40 kg

Fx = 0: 2Rx

392.4 N = 0

(1)
(2)

= 392.4 N

2Ry
Components of force
from ground acting on
two wheels

99.875 mm

(3)

Three equations, four unknowns; an additional free


body is needed.

A
2Rx

MB = 0 : Px(850 mm + 99.875 mm)


+ Py(250 mm + 5 mm)
(392.4 N)(100 mm + 5 mm) = 0

5 mm

1460

7.7 Rolling Resistance Example 1, page 4 of 4


Free body diagram of a wheel and axle

Moment equation for wheel

Forces from spreader


acting on axle

MA = 0: Rx(99.875 mm)

Ry(5 mm) = 0

(4)

Solving equations 1-4 simultaneously gives


Rx = 7.1 N

Ay
Ax

100 mm
Rx

Ry = 141.9 N

99.875 mm

Px = 14.2 N

Ans.

Py = 108.7 N

Ans.

B
5 mm
Ry

1461

7.7 Rolling Resistance Example 2, page 1 of 4


2. The mass of the piano is 200 kg. The coefficient of rolling
resistance of the casters is 0.4 mm. Determine the value of the
horizontal force P required to move the piano at constant speed.
Make the simplifying assumption that the resisting forces are
the same at all four casters.

Radius of the
casters = 14 mm

1462

7.7 Rolling Resistance Example 2, page 2 of 4


1

Locate the point where the resisting force


from the floor acts on the wheel.

Rotation
of wheel

Direction of motion
of center of wheel

A
A
B

14 mm

0.2 mm ( = coefficient
of rolling resistance)

2 As the wheel rolls to the left,


the floor exerts a force, R,
opposing the motion.
3

R acts at a point B on the


circumference 0.2 mm from a
vertical line through the center
A of the wheel.

1463

B
0.2 mm

(14 mm)2

(0.2 mm)2

= 13.9986 mm

4 For later use in calculating


moments, find the vertical
distance between A and B.

7.7 Rolling Resistance Example 2, page 3 of 4


5

Free-body diagram of piano


Weight = (200 kg)(9.81 m/s2) = 1962 N

Fx = 0: 4Rx

P=0

(1)

Fy = 0: 4Ry

1962 N = 0

(2)

Rx

Rx

Ry
Ry

Rx

Rx

Equilibrium equations.

Ry
Ry

Two equations, three unknowns; an


additional free body is needed.

The resisting forces from floor


acting on caster are assumed to
be the same at each caster

1464

7.7 Rolling Resistance Example 2, page 4 of 4


7 Free-body diagram of a caster and axle
Forces from piano
acting on axle

Moment equation for caster

Ay
A
Ax

MA = 0 : Rx(13.9986 mm)

Ry(0.2 mm) = 0

Solving Eqs.1-3 simultaneously gives


B

Rx = 7.01 N

13.9986 mm

Rx

Ry = 490.5 N
0.2 mm
P = 28.0 N

Ry

1465

Ans.

(3)

7.7 Rolling Resistance Example 3, page 1 of 3


3. If the coefficient of rolling resistance is 0.3 in., determine the
magnitude of the horizontal force F required to push the 300-lb
drum up the inclined plane at constant velocity.

8 in.
1

Locate the point where the


resisting force from the inclined
plane acts on the drum.

Rotation of
drum

15

Direction of
motion of
center of drum
A

B
15

R
2

R acts at a point B on the circumference


0.3 in. from the center A of the drum.
Note that the 0.3 in. is measured parallel
to the inclined plane.

1466

0.3 in. ( = coefficient


of rolling resistance)

As the wheel rolls up


the plane, the plane
exerts a force, R,
opposing the motion.

7.7 Rolling Resistance Example 3, page 2 of 3


For use in calculating moments, find the
distance from A to B in the direction
perpendicular to the inclined plane.
D
(8 in.)2

Free-body diagram of drum


300 lb (Weight of drum)
D

(0.3 in.)2

8 in.
= 7.9944 in.

8 in.
B

7.9944 in.

C 0.3 in.

Reference line
perpendicular to
inclined plane
6

1467

B
Ry

15

0.3 in.

Rx
Components
of force from
inclined plane
acting on
wheel

Moment equilibrium equation

MB = 0: (300 cos lb)(0.3 in.)


+ (300 sin lb)(7.9944 in.)
(F cos )(7.9944 in.)
+ (F sin )(0.3 in) = 0

(1)

7.7 Rolling Resistance Example 3, page 3 of 3


7 Calculate

= 90

75 = 15
Inclined
plane

B
C
15
90

Substitute

15 = 75

= 15 into Eq. 1:

(300 cos lb)(0.3 in.)


+ (300 sin lb)(7.9944 in.)
(F cos )(7.9944 in.)
+ (F sin )(0.3 in) = 0

(Eq. 1 repeated)

Solving gives
F = 92.6 lb

Ans.

1468

7.7 Rolling Resistance Example 4, page 1 of 3


4. A 100-lb steel-rim wheel of 24-in. diameter rolls
at constant velocity down the inclined plane. If the
coefficient of rolling resistance is 0.08 in.,
determine the angle .

Locate the point where the resisting force


from the inclined plane acts as the wheel.
Rotation of wheel

A
Direction of
motion of the
center of the
wheel

0.08 in. ( = coefficient


of rolling resistance)

2 As the wheel rolls down the


plane, the plane exerts a force,
R, opposing the motion.

3 R acts at a point B on the circumference


0.08 in. from the center of the wheel,
measured parallel to the inclined plane.

1469

7.7 Rolling Resistance Example 4, page 2 of 3


4

5 Free-body diagram of wheel


y
D

For use in calculating moments, find the distance


from A to B in the direction perpendicular to the
inclined plane.
D

(12 in.)2

(0.08 in.)2

11.9997 in.

= 11.9997 in.

Components of force
from inclined plane
acting on wheel

B
C
Radius = 24/2 in.
0.08 in. Ry

= 12 in.

Equilibrium equations
+

Fx = 0: Rx

(100 lb) sin

Fy = 0: Ry

(100 lb) cos

C
B
0.08 in.

Rx

MA = 0 : Ry(0.08 in.)

=0
=0

(1)
(2)

Rx(11.9997 in.) = 0 (3)

Solving gives
Rx = 0.667 lb
Ry = 99.998 lb
= 0.382

1470

(4)

7.7 Rolling Resistance Example 4, page 3 of 3


7

Geometry

8
D

But

= 0.382, by Eq. 4 so,

= 0.382

Ans.

We could have saved same work by


noticing that the wheel is a two-force
body. Thus because the line of action
of the weight is a vertical line through
the center, the line of action of the
resisting force R must also be a vertical
line passing through the center. The
angle can be found from geometry, as
shown below.
Weight

Complement of

12 in.
B
0.08 in.
R

1471

= sin-1 0.08 in.


12 in.
= 0.382

7.7 Rolling Resistance Example 5, page 1 of 4


5. If the coefficient of rolling resistance at the top of the
cylinder is 0.4 mm and at bottom of the cylinder is 0.8 mm,
determine the horizontal force P required to start the block
moving to the left. The weight of the cylinder is negligible
compared to the weight of the block.

100 kg

80 mm

1472

7.7 Rolling Resistance Example 5, page 2 of 4


1 Locate the points where the resisting forces act on the cylinder.
0.4 mm (= coefficient of rolling resistance)
Rblock

100 kg

B
2 The resisting force from the block opposes
the rolling of the cylinder on the block.
Direction of
motion of center
of the wheel
relative to the
ground

Rotation
A
of
wheel

Direction of motion
of center of the wheel
relative to the block

C
Rground

0.8 mm (= coefficient of rolling resistance)

The resisting force from the ground opposes


the rolling of the cylinder on the ground.

1473

7.7 Rolling Resistance Example 5, page 3 of 4


4

For use in calculating moments, find the vertical distances


between the center of the wheel and the point of application of
the resisting forces.
0.4 mm
D

(80 mm)2

B
80 mm

(0.4 mm)2 = 79.9990 mm

(80 mm)2

A
80 mm
C

0.8 mm

1474

(0.8 mm)2 = 79.9960 mm

7.7 Rolling Resistance Example 5, page 4 of 4


5

Free-body diagram of block and cylinder

Free-body diagram of the cylinder


0.4 mm

Weight = (100 kg)(9.81 m/s2) = 981 N

Rblock-y
Rblock-x

100 kg

B
79.999 mm
A
79.996 mm
Rground-x

Rground-x

Rground-y

E
0.8 mm

Moment equation

Rground-y
Equilibrium equations
+

Fx = 0: Rground-x

F=0

(1)

Fy = 0: Rground-y

981 N = 0

(2)

MB = 0: Rground-x(79.9990 mm + 79.9960 mm)


Rground-y(0.8 mm + 0.4 mm) = 0

Solving Eqs. 1-3 gives


Rgroung-x = 7.358 N
Rground-y = 981 N

Two equations, three unknowns: an additional


free-body is needed.

F = 7.36 N

1475

Ans.

(3)

7.7 Rolling Resistance Example 6, page 1 of 5


6. The ancient Britons who constructed the prehistoric monument
Stonehenge moved massive stones over twenty miles from a quarry to
the monument site. One possible way that they might have done this is
to roll the stones over logs laid on the ground. Assuming that the
coefficient of rolling resistance for the top of a log is 0.01 ft and the
coefficient is 0.08 ft for the bottom, estimate the horizontal force
required to move the typical stone shown below. Make the simplifying
assumption that the resisting forces are the same for each log.
Rope

24 ft

Force of men
(and perhaps
oxen)
F

100,000 lb
4 ft

Diameter of
logs = 1 ft.

1476

7.7 Rolling Resistance Example 6, page 2 of 5


1

Direction of
motion of stone

Locate the points where the resisting


forces act on the log.
2 As the log rolls on the
underside of the stone, the
stone exerts a force opposing
the rolling motion

0.01 ft
Portion
of stone
B

Coefficient of
rolling resistance

Rotation
of log

Direction of motion of center


of log relative to ground

Direction of
motion of
center of log
relative to
stone

C
3 As the log rolls on the
ground, the stone exerts a
force opposing the rolling
motion.

0.08 ft
Rg
Coefficient of
rolling resistance

1477

7.7 Rolling Resistance Example 6, page 3 of 5


4 For use in calculating moments, find the vertical distances from
the center to the points C and B where the resisting forces act.
0.01 ft
B
(0.5 ft)2

(0.01 ft)2 = 0.4999 ft

0.5 ft
Radius = (1 ft)/2 = 0.5 ft
A
0.5 ft

(0.5 ft)2

(0.08 ft)2

C
0.08 ft

= 0.4936 ft

1478

7.7 Rolling Resistance Example 6, page 4 of 5


Free-body diagram of stone and logs
100,000 lb

Rg-x
Rg-y
The same resisting force acts on each of the ten logs.
Equilibrium equations
+

Fx = 0: F

10Rg-x = 0

Fy = 0: 10Rg-y

100,000 lb = 0

(1)
(2)

Two equations, three unknowns: an additional free-body


is needed.

1479

7.7 Rolling Resistance Example 6, page 5 of 5


6

Free-body diagram of an individual wheel

Rs-x

Rs-y Force components


from the stone
B

0.4999 ft
Moment equilibrium

MB = 0: Rg-y(0.01 ft + 0.08 ft)

Rg-x(0.4999 ft + 0.4936 ft) = 0

(3)

Solving Eqs. 1-3 gives

0.4936 ft
C
Rg-x
0.01 ft

Rg-y
0.08 ft

Rg-x = 906 lb
Rg-y = 10,000 lb
F = 9,060 lb

Ans.

8 The force F required to move the 100,000-lb stone is large, even when the
ground is level. One scholar has estimated that as many as 600 men were
needed to move such a stone up one of the slopes lying between the quarry
and the monument site.

1480

8. Internal Forces

1481

8.1 Internal Forces in Structural Members

1482

8.1 Internal Forces in Structural Members Procedures and Strategies, page 1 of 2


Procedures and Strategies for Solving Problems Involving
Internal Forces in Structural Members

1. Pass a section through the structural member at the point


where the internal forces are to be calculated.

2. Draw a free-body diagram of the portion of the member on the


side of the section that has the least number of loads acting on it,
and show the (unknown) internal forces and moments acting at
the section.

3. Write the equilibrium equations for the free body.


Dx

M
N
V

Dy

1483

8.1 Internal Forces in Structural Members Procedures and Strategies, page 2 of 2


4. Solve for the forces from the support or from other
members by using the usual techniques for analyzing a
rigid body subject to a system of forces.
5. Substitute the results found in step 5 into the
equations of step 4, and solve for the internal forces.

Dy

Find Dx and Dy by writing equilibrium


equations for these two free-body diagrams.
D
Dx

Dx
P
Bx

Dy
B

C
By

1484

Cx
Cy

8.1 Internal Forces in Structural Members Problem Statement for Example 1


1. Determine the normal force, shear force, and
moment at sections passing through a) B and b) D.
4 kN

2m

2m

4m

4m

1485

8.1 Internal Forces in Structural Members Problem Statement for Example 2


2. Determine the normal force, shear force, and moment at a
section passing through B.
300 lb/ft
A
C
B
6 ft

6 ft

1486

8.1 Internal Forces in Structural Members Problem Statement for Example 3


3. Determine the normal force, shear force, and moment
acting at a section passing through point B on the
quarter-circular rod shown.
70 lb
C
30 lb
B
4 ft
A

40
O

1487

8.1 Internal Forces in Structural Members Problem Statement for Example 4


4. Determine the internal torque at sections passing
through points a) B and b) D of the shaft.

2 kip
2 ft

2 ft

6 kipft (Torque applied at end)


C

2 kip

1488

8.1 Internal Forces in Structural Members Problem Statement for Example 5


5. Determine the axial force, shear force, and moment at
sections passing through a) C and b) immediately to the left
of roller E, and c) immediately to the right of roller E.

4 kN

Hinge (pin connection)


5 kN m

B
4m

2m 1m

5m

5m

1489

8.1 Internal Forces in Structural Members Problem Statement for Example 6


6. Two wooden blocks have been glued together and a
compressive force of 80 N is applied by the clamp to press the
blocks together as the glue dries. Determine the normal force,
shear force, and moment at a vertical section through point A.
A

35 mm

1490

8.1 Internal Forces in Structural Members Problem Statement for Example 7


7. Determine the axial force, shear force, and moment at C.
100 lb
D

5 ft

5 ft

1491

8.1 Internal Forces in Structural Members Problem Statement for Example 8


8. The spreader bar BCD is used to spread the load acting on the
2,000 lb beam EFGH while it is being lifted. Determine the normal
force, shear force, and moment at the midpoint C of the spreader bar.
Neglect the weight of the bar.
2,000 lb

A
6 ft
B

E
5 ft

5 ft

5 ft

H
5 ft

1492

8.1 Internal Forces in Structural Members Problem Statement for Example 9


9. The frame shown is pin-connected at D and rests on smooth
surfaces at A and G. Determine the normal force, shear force,
and moment acting at a section passing through point C.
D

E
C

10 m
100 kg

F
G

1m 2m

3m

2m

3m 1m

1493

8.1 Internal Forces in Structural Members Problem Statement for Example 10


10. In the floor-beam girder system shown, the four floor panels at the top
are simply supported at their ends by floor beams. The beams in turn
transmit forces to the girder ACI. Determine the axial force, shear force, and
moment in the girder at sections passing through a) point B and b) point J.

Floor panels

2 kip

10 ft

3 ft

7 ft

10 ft

6 ft
2 ft 2 ft

1494

Floor beam
(end view)

Girder
(side view)

8.1 Internal Forces in Structural Members Example 1, page 1 of 4


1. Determine the normal force, shear force, and
moment at sections passing through a) B and b) D.
4 kN

2m

2m

4m

4m

1495

8.1 Internal Forces in Structural Members Example 1, page 2 of 4


Part a): Internal forces and moment at B
4 kN

Pass a section through point B.

Free-body diagram of part of beam to left of B (This is a much


better choice of free body than the part of the beam to the right,
since we won't have to calculate the reactions at C and E.)

4 kN
VB
B

NB

MB

Solving Eqs. 1, 2, and 3 gives


NB = 0

Ans.

VB = 4 kN

Ans.

MB = 8 kNm

Ans.

Free-body diagram showing correct senses of


internal forces and moment at B:

2m
4 kN
3

Equations of equilibrium for part of beam to left of B:


VB = 4 kN

VB

(1)

Fx = 0: NB = 0
Fy = 0:

4 kN = 0

MB = 0: (4 kN)(2 m) + MB = 0

B
A

(2)

NB = 0
MB = 8 kNm

(3)

2m

1496

8.1 Internal Forces in Structural Members Example 1, page 3 of 4


Part b): Internal forces and moment at D
6

Pass a section through point D.

4 kN

Free-body diagram of part of beam to right of D


VD
MD
D
E
Ex
ND
Ey
4m
Equations of equilibrium for part of beam to right of D:

ND + Ex = 0

Fx = 0:

Fy = 0: VD

MD = 0:

Ey = 0

MD + Ey (4 m) = 0

(4)
(5)
(6)

1497

Three equations but five unkowns.


Another free body is needed.

8.1 Internal Forces in Structural Members Example 1, page 4 of 4


10 Free-body diagram of entire beam
4 kN
A

Ex

Cy

Ey

8m

4m

13 Free-body diagram showing correct senses


of internal forces and moment at D

Fx = 0: Ex = 0

(7)

11 Equilibrium equations for entire beam:

MC = 0: (4 kN)(4 m) + Ey (8 m) = 0

(8)

MD = 8 kNm
D

ND = 0

Solving Eqs. 7 and 8 gives

VD = 2 kN
Ex = 0 and Ey = 2 kN

2 kN

12 Using these results in Eqs. 4, 5, and 6 and solving gives


ND = 0

Ans.

VD = 2 kN

Ans.

MD = 8 kNm

Ans.

1498

8.1 Internal Forces in Structural Members Example 2, page 1 of 2


2. Determine the normal force, shear force, and moment at a
section passing through B.
3

300 lb/ft

Note that the resultant of the entire distributed


load, (300 lb/ft) (12 ft) = 3,600 lb, does not
act at B. The entire distributed load acts on
the entire 12-ft span; the free body shown
below has 300 lb/ft acting over only a 6-ft
span, and the resultant of this distributed load
acts halfway between B and C.

A
C
B
6 ft

6 ft

Pass a section through point B.


Equations of equilibrium for part of beam to right of B:

300 lb/ft

A
C

Fx = 0:

NB = 0

(1)

Therefore

Free-body diagram of part of


beam to right of B
300 lb/ft
V
B

NB

B
MB

6 ft

Cy

Fy = 0: VB + Cy

NB = 0

MB = 0:

Ans.
300 lb/ft)(6 ft) = 0

(2)

MB (300 lb/ft)(6 ft)( 6 ft )


2
+ Cy(6 ft) = 0

(3)

Eqs. 2 and 3 involve three unknowns. An additional free-body


diagram is needed.

1499

8.1 Internal Forces in Structural Members Example 2, page 2 of 2


5

Free-body diagram of entire beam.


300 lb/ft
A
C

Ax

B
Cy

Ay
12 ft

Equation of equilibrium for entire beam:

MA = 0: Cy(12 ft)

(300 lb/ft)(12 ft)( 12 ft) = 0


2

(4)

Solving Eq. 4 gives Cy = 1800 lb.


Using Cy = 1800 lb in Eqs. 2 and 3 and then solving gives
VB = 0

Ans.

MB = 5,400 lb ft

Ans.

1500

8.1 Internal Forces in Structural Members Example 3, page 1 of 2


3. Determine the normal force, shear force, and moment
acting at a section passing through point B on the
quarter-circular rod shown.
70 lb

Free-body diagram of portion above B


70 lb

30 lb
VB

B
MB
4 ft
40

NB

4 ft

40

Pass a section through point B

70 lb

Equilibrium equations for part of rod to right and above


section at B:

30 lb

Fx = 0:

NB sin

VB cos 40 + 30 lb = 0

(1)

C
Fy = 0:

NB cos

+ VB sin 40 + 70 lb = 0

(2)

30 lb

MO = 0:

MB + NB(4 ft)

(30 lb)(4 ft) = 0

Choosing point O for summing moments eliminates


both the 70-lb force and the shear VB from the moment
equation because their lines of action pass through O.

1501

(3)

8.1 Internal Forces in Structural Members Example 3, page 2 of 2


4

Geometry
C

= 90

50

50 = 40

40
O

Substituting
gives

= 40 into Eqs. 1, 2, and 3 and solving

NB = 72.9 lb

Ans.

VB =

.0 lb

Ans.

MB = 171.6 lb

Ans.

1502

8.1 Internal Forces in Structural Members Example 4, page 1 of 6


4. Determine the internal torque at sections passing
through points a) B and b) D of the shaft.

2 kip
2 ft

2 ft

6 kipft (Torque applied at end)


C

2 kip

1503

8.1 Internal Forces in Structural Members Example 4, page 2 of 6


Part a) point B
1

Pass a section through point B.


2 kip
2 ft

2 ft

6 kipft (Torque applied at end)


C

2 kip

1504

8.1 Internal Forces in Structural Members Example 4, page 3 of 6


2

Free-body diagram of part to right of B. This is a better choice


than using the part to the left because the reaction at the support
would have to be calculated, if we used the part to the left.
2 kip
2 ft

2 ft

MB
Torque from part
of the shaft to the
left of point B
acting on the part
to the right.

6 kipft (Torque applied at end)


x

3
2 kip

The choice of
sense of the torque
is arbitrary.

Equilibrium equation for part of shaft:


Mx = 0:

MB

(2 kip)(2 ft)

(2 kip)(2 ft) + 6 kipft = 0

Solving gives
MB = 2 kipft

1505

Ans.

8.1 Internal Forces in Structural Members Example 4, page 4 of 6


4

Free-body diagram of part to


right of B showing correct sense
of torque at B

2 kip
2 ft

2 ft

6 kipft (Torque applied at end)


x

MB = 2 kipft

2 kip

1506

8.1 Internal Forces in Structural Members Example 4, page 5 of 6


Part b) point D
5

Pass a section through point D.

2 kip
2 ft

2 ft

6 kipft
D

2 kip

1507

8.1 Internal Forces in Structural Members Example 4, page 6 of 6


6

Free-body diagram of part to right of D

6 kipft
x

MD

7
+

Equilibrium equation for part of shaft:


Mx = 0:

MD + 6 kipft = 0

Solving gives
MD = 6 kipft

Ans.

1508

8.1 Internal Forces in Structural Members Example 5, page 1 of 6


5. Determine the axial force, shear force, and moment at
sections passing through a) C and b) immediately to the left
of roller E, and c) immediately to the right of roller E.

4 kN

Hinge (pin connection)


5 kN m

B
4m

2m 1m

5m

5m

Part a): point C


1

4 kN
Pass a section through point C.
5 kN m

Hinge
D

B
4m

2m 1m

5m

5m

1509

8.1 Internal Forces in Structural Members Example 5, page 2 of 6


2

Free-body diagram of part of beam to right of section. This is a


better choice than the part to the left because only one
unknown reaction (the vertical force at E) occurs on the right
while three (two force components and a moment) occur at A.
VC
C
NC

4 kN

Hinge
D

MC

Ey
1m

5m

5m

Equilibrium equations for part of beam:

3
+

Fx = 0:

NC = 0

(1)

Therefore,

Fy = 0: VC + Ey

NC = 0

MC = 0:

Ans.
4 kN = 0

MC + Ey(1 m + 5 m)

(2)
(4 kN)(1 m + 5 m +5 m) = 0

Three equations but four unknowns so an additional equation is needed.

1510

(3)

8.1 Internal Forces in Structural Members Example 5, page 3 of 6


5

To calculate the reaction at E, pass a


section immediately to the left of hinge D.
4 kN
Hinge
5 kN m

B
4m

2m 1m

5m

The moment is zero at a hinge


(a "hinge" is just another
name for a pin connection
between two parts of the
beam; no moment is
transmitted at a pin
connection). We will make
use of this fact by summing
moments about D.

5m

Free-body diagram of part of beam to right


of section
VD
MD = 0

Hinge
D

ND
Ey
5m

1511

5m

4 kN

8.1 Internal Forces in Structural Members Example 5, page 4 of 6


8

Equilibrium equation for part of beam:


MD = 0: Ey(5 m)

4(5 m +5 m) = 0

(4)

Note that VD does not appear in this equation because


its moment arm about D is infinitesimal.
Solving Eq. 4 gives
Ey = 8 kN
Substituting Ey = 8 kN in Eqs. 2 and 3 and solving
gives
VC =

4 kN

MC = 4 kNm

Ans.

Ans.

Free-body diagram of part of beam


showing correct senses of forces and
moment at C
Hinge

VC = 4kN
NC = 0

MC = 4 kNm

Ey = 8 kN
1m

1512

4 kN

5m

5m

8.1 Internal Forces in Structural Members Example 5, page 5 of 6


Part b): point immediately to left of E.

11 Free-body diagram of part of


beam to right of section
VE

10 Pass a section through the beam


immediately to the left of E.

4 kN

Hinge
5 kN m

NE
E

ME

Ey = 8 kN

B
4m

4 kN

5m
2m 1m

5m

12 Equilibrium equations for part to right of section:

5m

Fx = 0:

NE = 0

Therefore,

13 Free-body diagram of part of beam showing correct


senses of forces and moment at section to left of E

NE = 0

Fy = 0: VE + 8kN

4 kN = 0

Ans.
(6)

Solving gives

4 kN

VE =

VE = 4 kN
E

ME = 20 kNm

(5)

NE = 0

ME = 0:

4 kN
ME

(4 kN)(5 m) = 0

Ans.
(7)

Solving gives
Ey = 8 kN
ME = 20 kNm

5m

1513

Ans.

8.1 Internal Forces in Structural Members Example 5, page 6 of 6


Part c): point immediately to right of roller E.
15
14 Pass a section through the beam
immediately to the right of E.

Free-body diagram of part of


beam to right of section

4 kN

VE'

4 kN
Hinge

5 kN m
A

NE '

ME'

5m

B
4m

2m 1m

5m

Equilibrium equations for part to right of section:

16

5m

Fx = 0:

NE' = 0

Therefore,

17 Free-body diagram of part of beam showing correct


senses of forces and moment at section to right of E
4 kN

(8)

NE' = 0

Fy = 0: VE'

4 kN = 0

Ans.
(9)

Solving gives

VE' = 4 kN

VE' = 4 kN

ME' = 20 kNm
NE' = 0

ME = 0:

ME'

Ans.
(4 kN)(5 m) = 0

(10)

Solving gives

5m

ME' = 20 kNm

1514

Ans.

8.1 Internal Forces in Structural Members Example 6, page 1 of 2


6. Two wooden blocks have been glued together and a
compressive force of 80 N is applied by the clamp to press the
blocks together as the glue dries. Determine the normal force,
shear force, and moment at a vertical section through point A.
A

35 mm

Pass a section through point A.

35 mm

1515

8.1 Internal Forces in Structural Members Example 6, page 2 of 2


2

Free-body diagram of part of clamp to right of


section. The block is not included in the free body.
VA
A
NA

MA
35 mm
80 N

Equilibrium equations for part of clamp:

The force from the block points towards


the clamp because the block is in
compression.

NA + 80 N = 0

(1)

Fx = 0:

Fy = 0: VA = 0

(2)

MA = 0:

(3)

MA + (80 N)(35 mm) = 0

Solving gives

1516

NA = 80 N

Ans.

VA = 0

Ans.

MA = 2800 Nmm = 2.8 Nm

Ans.

8.1 Internal Forces in Structural Members Example 7, page 1 of 3


7. Determine the axial force, shear force, and moment at C.
100 lb
D

100 lb
1

5 ft

Pass a section through point C.

5 ft
D

5 ft

1517

5 ft

8.1 Internal Forces in Structural Members Example 7, page 2 of 3


2

Free-body diagram of part of member


above section at C

100 lb

E
3

C
FDB

VC

DB is a two-force member so the line of


action of FDB is known to be vertical.

MC
4
5 ft

Fx = 0:

VC = 0

(1)

Therefore VC = 0

5 ft

Equations of equilibrium for part of member:

Fy = 0:

NC

MC = 0:

NC + FDB

Ans.
100 lb = 0

MC + FDB(5ft)

(100 lb)(5 ft +5 ft) = 0

Four unknowns but only three equations so an


additional equation is needed.

1518

(2)
(3)

8.1 Internal Forces in Structural Members Example 7, page 3 of 3


6

Free-body diagram of entire structure


5 ft

5 ft

100 lb
D

C
8

Equations of equilibrium for entire structure:

MA = 0: FDB(5ft)

(100 lb)(5 ft +5 ft) = 0

(4)

Ax
Solving gives
Ay

FDB
7

FDB = 200 lb

Because DB is a
two-force member, the
line of action of FDB is
known to be vertical.

Substituting FDB = 200 lb in Eqs. 2 and 3 and solving


gives

1519

NC = 100 lb

Ans.

MC = 0

Ans.

8.1 Internal Forces in Structural Members Example 8, page 1 of 3


8. The spreader bar BCD is used to spread the load acting on the
2,000 lb beam EFGH while it is being lifted. Determine the normal
force, shear force, and moment at the midpoint C of the spreader bar.
Neglect the weight of the bar.
2,000 lb

A
6 ft
B

Pass a section through point C.

2,000 lb

A
F

E
5 ft

5 ft

5 ft

H
5 ft

1520

8.1 Internal Forces in Structural Members Example 8, page 2 of 3


4

3 TBA and TBF are


the tensions in
the cables at B.

MC
B

Equilibrium equations for part of spreader bar to left of C:

NC

Fx = 0: NC + TBA cos

=0

Free-body diagram
of part of spreader
bar to left of C
TBA

Fy = 0: TBA sin

TBF

VC = 0

MC = 0: TBF(5 ft)

TBA(sin )(5 ft) + MC = 0

(1)
(2)
(3)

VC
TBF

Three equations but five unknowns, so at least one more free


body must be used.

5 ft

7
Free-body diagram of beam

TDG

TBF
F

5 ft

5 ft

MG = 0:

TBF(5 ft+5 ft) + (2,000 lb)(5 ft) = 0

Solving gives

2,000 lb
E

Equilibrium equations for beam:

5 ft

TBF = 1,000 lb
Another free body is needed, if we are to calculate the
value of TBA in Eqs. 1, 2, and 3.

5 ft

1521

(4)

8.1 Internal Forces in Structural Members Example 8, page 3 of 3


8

Free-body diagram of
connection A

Equilibrium equations for connection A:


+

Fx = 0:

2,000 lb

TBA cos

Fy = 0: 2,000 lb

+ TDA cos
TBA sin

=0
TDA sin

(5)
=0

(6)

TBA

TDA
11 Substituting

= 50.19 in Eqs. 5 and 6 and solving gives

TBA = TDA = 1,302 lb


10 Geometry

Substituting TBA = 1,302 lb and TBF = 1,000 lb into Eqs.


1, 2, and 3 and solving gives

6 ft
B

5 ft

= tan-1 ( 6 ft ) = 50.19
5 ft

1522

NC = 833 lb

Ans.

VC = 0

Ans.

MC = 0

Ans.

8.1 Internal Forces in Structural Members Example 9, page 1 of 5


9. The frame shown is pin-connected at D and rests on smooth
surfaces at A and G. Determine the normal force, shear force,
and moment acting at a section passing through point C.
D

E
C

10 m
1

100 kg
B

Pass a section through C.

F
D
G

E
1m 2m

3m

2m

3m 1m
10 m

C
100 kg
B
A

1523

F
G

8.1 Internal Forces in Structural Members Example 9, page 2 of 5

NC
MC
VC

LAC

Equilibrium equations for part ABC:


+

Fx = 0: T + NC cos

Free-body diagram of part CBA

Fy = 0: FA + NC sin

MA = 0:

LAB

A
FA

3
1m2m

=0

(1)

VC sin

=0

(2)

VC( LAC) + MC = 0

(3)

Three equations and five unknowns, so at least one


more free body is needed. But first, the angles
and and distances LAB and LAC must be determined.

B
T

T(LAB)

+ VC cos

Since the floor is smooth,


only a normal force is
present; friction is absent.

1524

8.1 Internal Forces in Structural Members Example 9, page 3 of 5


5

10 m

10 m
= tan-1 ( 1 m + 2 m + 3 m ) = 59.04
= 90

= 30.96
E

LAB = (1 m) tan

LAC
B

LAC =

LAB
A 1m2m

Free-body diagram of entire frame. This free


body will enable us to calculate FA.

Geometry

= 1.667 m
C

(1 m + 2 m)
= 5.831 m
cos
B

Weight =
(100 kg)(9.81 m/s2)
= 981 N

Equilibrium equation for entire frame:


MG = 0:

3m
A

10 m

FA(8 m + 4 m) + (981 N)(4 m) = 0

FA

(4)

FG
8m

Solving this equation gives


FA = 327.0 N
One more free body is needed, since we now have four
equations but five unknowns.

1525

4m

8.1 Internal Forces in Structural Members Example 9, page 4 of 5


Free-body diagram of member
ABCD. This free body will give
us an equation relating FA and T.

Dy
D

Forces from the pin


connection at D

Dx

10 m
B
T
A

FA = 327.0 N

LAB = 1.667 m

10 Equilibrium equations for member ABCD:


6m
MD = 0:

(327.0 N)(6 m) + T(10 m

Solving gives
T = 235.4 N

1526

1.667 m) = 0

(5)

8.1 Internal Forces in Structural Members Example 9, page 5 of 5


11 Substituting

12

Free-body diagram showing correct sense of


internal moment and forces at C

T = 235.4 N
NC = 402 N

FA = 327.0 N

MC = 40.9 Nm

= 59.04
C
= 30.96

VC = 33.6 N

LAB = 1.667 m
A

LAC = 5.831 m
into Eqs. 1, 2, and 3 and solving gives
NC = 402 N

Ans.

VC = 33.6 N

Ans

MC = 40.9 Nm

Ans

FA = 327 N

1527

8.1 Internal Forces in Structural Members Example 10, page 1 of 8


10. In the floor-beam girder system shown, the four floor panels at the top
are simply supported at their ends by floor beams. The beams in turn
transmit forces to the girder ACI. Determine the axial force, shear force, and
moment in the girder at sections passing through a) point B and b) point J.

Floor panels

2 kip

10 ft

3 ft

7 ft

10 ft

6 ft
2 ft 2 ft

1528

Floor beam
(end view)

Girder
(side view)

8.1 Internal Forces in Structural Members Example 10, page 2 of 8


Part a) point B
1

Pass a section through the girder at B.

2 kip

Floor panels

1529

Floor beam
(end view)

Girder
(side view)

8.1 Internal Forces in Structural Members Example 10, page 3 of 8


2

Free-body diagram of part of structure to left


of point B, including floor panels DE and EF
D

F
3
NB

FF

MB

Force from floor beam F acting


on floor panel EF ("simply
supported" so no moment acts
at F, only a force)

VB

FA
13 ft

7 ft

Equilibrium equations for part of structure:


+

Fx = 0: NB = 0

(1)

Fy = 0: FA

(2)

MB = 0:

VB + FF = 0

FA(13 ft) + MB + FF(7 ft) = 0

(3)

Three equations but five unknowns so at least one


more free body is needed.

1530

8.1 Internal Forces in Structural Members Example 10, page 4 of 8


6

Free-body diagram of floor panel EF:

F
7
FE

FF

Obviously, FF = 0 (Just
consider the sum of moments
about E).

Free-body diagram of entire structure (This free body will enable us to calculate FA):

2 kip

Cx
Cy

FA
10 ft

3 ft

7 ft

10 ft

6 ft
2 ft 2 ft

1531

8.1 Internal Forces in Structural Members Example 10, page 5 of 8


9

Equilibrium equation for entire structure:

MC = 0:

FA(10 ft + 3 ft + 7 ft + 10 ft)

(2 kip)(6 ft + 2ft) = 0

Solving gives
FA = 0.5333 kip
Substituting FA = 0.5333 kip in Eqs. 2 and 3 and solving gives
VB = 0.53 kip

Ans.

MB = 6.93 kipft

Ans.

10 Free-body diagram showing correct senses of


internal reactions at B
D

VB = 0.53 kip

NB = 0 FF = 0
MB = 6.93 kipft

FA
13 ft

7 ft

1532

(4)

8.1 Internal Forces in Structural Members Example 10, page 6 of 8


Part b) Internal reactions at section J

11 Pass a section through the girder at J.


2 kip

1533

8.1 Internal Forces in Structural Members Example 10, page 7 of 8


12 Free-body diagram of part of structure to
right of section at J, including floor beam H
FH 13 Force from panel GH
acting on floor beam H

MJ

H
J

NJ

14 Equilibrium equations for part of structure:

VJ
+

Fx = 0:

NJ = 0

(5)

Therefore

NJ = 0

4 ft

Fy = 0: VJ
MJ = 0:

MJ

Ans.
FH = 0

(6)

FH(4 ft) = 0

15 Three equations but four unknowns so another


free body is needed.

1534

(7)

8.1 Internal Forces in Structural Members Example 10, page 8 of 8


Free-body diagram of floor panel GH.
This free body will enable us to
calculate FH.
2 kip

17

16

Equilibrium equation for floor panel GH:


MG = 0:

(2 kip)(8 ft) + FH(8 ft + 2 ft) = 0

Solving gives
G

FH = 1.60 kip
Substituting FH = 1.60 kip in Eqs. 6 and 7 and solving gives

FG
8 ft

FH

VJ = 1.60 kip

Ans.

2 ft

MJ = 6.40 kipft

Ans.

FH = 1.6 kip

18 Free-body diagram of part of beam to right


of J showing internal forces and moment
with correct senses.
VJ = 1.60 kip
NJ = 0

MJ = 6.40 kipft
4 ft

1535

8.2 Shear and Bending-Moment Diagrams: Equation Form

1536

8.2 Shear and Bending-Moment Diagrams: Equation Form Procedures and Strategies, page 1 of 2
P

Procedures and Strategies for Solving Problems Involving


Constructing Shear and Bending-Moment Diagrams by Use
of Equations

1. Draw a free-body diagram of the beam and solve for the


reactions.
2. Introduce a coordinate x with origin at the left end of the beam.
(x will locate the position of various sections passed through the
beam, as described in the next steps).
3. Pass a section through the beam at a point between the left end
and the first "critical point" (a point at which a concentrated
force or couple moment acts or at which a distributed load
begins or ends).
4. Draw a free-body diagram of the part of the beam to the left of
the section and show the shear V acting downward and moment
M acting counterclockwise on the right end.

P
A
RA

B
RB

x
x
x

Fy = 0

A
RA

5. Write and solve equilibrium equations to find V and M as


functions of x.

M=0

V
P

6. Pass a new section through the beam at a point between the


current critical point and the next critical point.

A
7. Repeat steps 4-6 until the right end of the beam is reached.
V

RA

8. Plot V vs. x and M vs. x by using the equations you have


derived for V and M.

1537

Fy = 0

M=0

8.2 Shear and Bending-Moment Diagrams: Equation Form Procedures and Strategies, page 2 of 2
Note: By using free-body diagrams that grow
successively longer with each new section, your force
and moment equations change only by the addition of
one new term. This property helps you avoid errors in
writing the equations.

w
M

A
RA

V
Fy = 0
x

M=0

1538

8.2 Shear and Bending-Moment Diagrams: Equation Form Problem Statement for Example 1
1. Express the shear V and bending moment M as functions of x, the distance from
the left end of the beam to an arbitrary point on the beam. Plot V and M vs. x.
6 kip

9 kip

B
3 ft
x

7 ft

5 ft

1539

8.2 Shear and Bending-Moment Diagrams: Equation Form Problem Statement for Example 2
2. Express the shear V and bending moment M as functions of x, the distance from
the left end of the beam to an arbitrary point on the beam. Plot V and M vs. x.
2 N/m
20 N m

A
14 m
x

1540

8.2 Shear and Bending-Moment Diagrams: Equation Form Problem Statement for Example 3
3. Express the shear V and bending moment M as functions of x, the distance from
the left end of the beam to an arbitrary point on the beam. Plot V and M vs. x.
10 lb/ft
B

A
7 ft

10 ft

1541

3 ft

8.2 Shear and Bending-Moment Diagrams: Equation Form Problem Statement for Example 4
4. Express the shear V and bending moment M as functions of x, the distance from
the left end of the beam to an arbitrary point on the beam. Plot V and M vs. x.

B
18 kip ft

27 kip ft
3 ft
x

5 ft

7 ft

1542

8.2 Shear and Bending-Moment Diagrams: Equation Form Problem Statement for Example 5
5. Express the shear V and bending moment M in the horizontal portion ACDB of
the beam as functions of x, the distance from the left end of the beam to an
arbitrary point on the beam. Plot V and M versus x.
4 kip

4 kip

B
C
2 ft
x

2 ft

D
4 ft

2 ft

1543

2 ft

8.2 Shear and Bending-Moment Diagrams: Equation Form Problem Statement for Example 6
6. Express the shear V and bending moment M as functions of x, the distance from
the left end of the beam to an arbitrary point on the beam. Plot V and M vs. x.
4 kN

8 kN

Hinge
C

A
B
2m

2m

2m

1544

2m

8.2 Shear and Bending-Moment Diagrams: Equation Form Problem Statement for Example 7
7. Express the shear V and bending moment M as functions of x, the distance from
the left end of the beam to an arbitrary point on the beam. Plot V and M vs. x.
20 kip

4 kip/ft
Hinge
B

A
5 ft

5 ft

8 ft

1545

8.2 Shear and Bending-Moment Diagrams: Equation Form Problem Statement for Example 8
8. Express the shear V and bending moment M as functions of x, the distance from
the left end of the beam to an arbitrary point on the beam. Plot V and M vs. x.
4 kN/m
2 kN/m

A
x

6m

6m

1546

8.2 Shear and Bending-Moment Diagrams: Equation Form Example 1, page 1 of 6


1. Express the shear V and bending moment M as functions of x, the distance from
the left end of the beam to an arbitrary point on the beam. Plot V and M vs. x.
6 kip

9 kip

B
3 ft
x

Draw a free-body diagram and find the reactions.


6 kip

9 kip

3 ft

RA

5 ft

Fy = 0: RA

7 ft

5 ft

MA = 0:

9 kip

7 ft

6 kip + RB = 0

(9 kip)(3 ft)

(6 kip)(3 ft + 5 ft) + RB(3 ft + 5 ft + 7 ft) = 0

Solving gives
RA = 10 kip and RB = 5 kip

1547

RB

8.2 Shear and Bending-Moment Diagrams: Equation Form Example 1, page 2 of 6


2

Pass a section through the beam at a point between the left end and the 9-kip force.
6 kip

9 kip
x

3 ft

RA = 10 kip

5 ft

7 ft

0 < x < 3 ft
3

Draw a free-body diagram of the portion of the beam to the left


of the section and find V and M at the section.
x
A

M (sign convention: M positive counterclockwise, on right end of section)


V (sign convention: V positive down, on right end of section)

Fy = 0: 10 kip

RA = 10 kip

Mx = 0:

V=0

(10 kip)x + M = 0

Solving gives
V = 10 kip and M = 10x kip ft
valid for 0 < x < 3 ft.

1548

RB = 5 kip

8.2 Shear and Bending-Moment Diagrams: Equation Form Example 1, page 3 of 6


4

Pass a section through the beam at a point between the 9-kip force and the 6-kip force.
6 kip

9 kip
x

RA = 10 kip

3 ft

5 ft

7 ft

RB = 5 kip

3 ft < x < 8 ft
5

Draw a free-body diagram of the portion of the beam to the left


of the section and find V and M at the section.
9 kip

Fy = 0: 10 kip

Mx = 0:

9 kip

V=0

(10 kip)x + (9 kip)(x

3 ft) + M = 0

Solving gives
RA = 10 kip

(x

3 ft

3 ft)

V = 1 kip

(3)

M = (x + 27) kip ft

(4)

valid for 3 ft < x < 8 ft.

1549

8.2 Shear and Bending-Moment Diagrams: Equation Form Example 1, page 4 of 6


6

Pass a section through the beam at a point between the 6-kip force and the right end of the beam.
6 kip

9 kip

x
B

3 ft
RA = 10 kip

5 ft

7 ft

RB = 5 kip

8 ft < x < 15 ft
Draw a free-body diagram of the portion of the beam to the left
of the section and find V and M at the section.

Fy = 0: 10 kip

9 kip

6 kip

V=0

x
6 kip

9 kip

Mx = 0:

(10 kip)x + (9 kip)(x

M
A

+ (6 kip)(x

3 ft)

8 ft) + M = 0

Solving gives
3 ft
RA = 10 kip

(x

5 ft
(x

8 ft)

3 ft)

V = 5 kip

(5)

M = ( 5x + 75) kip ft

(6)

valid for 8 ft < x < 15 ft.

1550

8.2 Shear and Bending-Moment Diagrams: Equation Form Example 1, page 5 of 6


8 Collect the results from Eqs. 1-6:
0 < x < 3 ft

V = 10 kip
M = 10x kip ft

3 ft < x < 8 ft

V = 1 kip
Ans.
M = (x + 27) kip ft

8 ft < x < 15 ft

V = 5 kip
M = ( 5x + 75) kip ft

1551

8.2 Shear and Bending-Moment Diagrams: Equation Form Example 1, page 6 of 6


9

Plot V and M versus x.

6 kip

9 kip

RA = 10 kip

3 ft

5 ft

7 ft

RB = 5 kip

10
V
(kip)
1

x
5

M
(kip ft)

35
30

1552

8.2 Shear and Bending-Moment Diagrams: Equation Form Example 2, page 1 of 3


2. Express the shear V and bending moment M as functions of x, the distance from
the left end of the beam to an arbitrary point on the beam. Plot V and M vs. x.
2 N/m
20 N m

A
14 m
x
1 Draw a free-body diagram and find the reactions.
14 m
=7m
Resultant = (2 N/m)(14 m) = 28 N
2

2 N/m

MA

20 N m

A
14 m
RA

Fy = 0: RA
MA = 0: MA

2
28 N = 0

A couple-moment reaction must always be


included at a built-in end of a beam.

(28 N)(7 m) + 20 N m = 0

Solving gives
RA = 28 N and MA = 176 N m

1553

8.2 Shear and Bending-Moment Diagrams: Equation Form Example 2, page 2 of 3


3

Pass a section through the beam at an arbitrary point (located by x)


x
2 N/m

MA = 176 N m

20 N m

A
14 m
RA = 28 N
0 < x < 14 m

Fy = 0: 28 N

Draw a free-body diagram of the portion of the beam to the left


of the section and find V and M at the section.

Resultant = (2 N/m)(x)

MA = 176 N m

x
2

2x

V=0

Mx = 0: 176 N m

x+(

x
)(2 N/m)(x) + M = 0
2

Solving gives
M

V = ( 2x + 28) N
M = ( x2 +

x
RA = 28 N

1554

valid for 0 < x < 14 m.

Ans.
176) N m

Ans.

8.2 Shear and Bending-Moment Diagrams: Equation Form Example 2, page 3 of 3


5

Plot V and M versus x.


2 N/m

MA = 176 N m

20 N m

A
14 m
RA = 28 N
28

V
(N)
x
M
(N m)

20
x

176

1555

8.2 Shear and Bending-Moment Diagrams: Equation Form Example 3, page 1 of 6


3. Express the shear V and bending moment M as functions of x, the distance from
the left end of the beam to an arbitrary point on the beam. Plot V and M vs. x.
10 lb/ft
B

A
7 ft

10 ft

3 ft

x
1

Draw a free-body diagram and find the reactions.


10 ft
= 5 ft
2

Resultant = (10 lb/ft)(10 ft) = 100 lb

A
7 ft

Fy = 0: RA

RA

MA = 0:

10 ft

3 ft
RB

100 lb + RB = 0

(100 lb)(7 ft + 5 ft) + RB(7 ft + 10 ft + 3 ft) = 0

Solving gives
RA = 40 lb and RB = 60 lb

1556

8.2 Shear and Bending-Moment Diagrams: Equation Form Example 3, page 2 of 6


Pass a section through the beam at a point between the left end
of the beam and the beginning of the distributed load.

10 lb/ft
x
B

A
7 ft

10 ft

RA = 40 lb
0 < x < 7 ft
Draw a free-body diagram and find the reactions.
Fy = 0: 40 lb

Mx = 0:

V=0

(40 lb)x + M = 0

x
V

Solving gives

RA = 40 lb
V = 40 lb

(1)

M = (40x) lb ft

(2)

valid for 0 < x < 7 ft.

1557

3 ft
RB = 60 lb

8.2 Shear and Bending-Moment Diagrams: Equation Form Example 3, page 3 of 6


4

Pass a section through the beam at a point between the


beginning and end of the distributed load.
x
10 lb/ft
B

A
7 ft

10 ft

3 ft
RB = 60 lb

RA = 40 lb
7 ft < x < 17 ft

x
Resultant = (10 lb/ft)(x

7 ft)

Fy = 0: 40 lb

Draw a free-body diagram of the portion of the beam to


the left of the section and solve for V and M at the section.
7 ft
2

Mx = 0:

(10 lb/ft)(x

7 ft)

(40 lb)x + [(10 lb/ft)(x

V=0
7 ft)]

(x

7 ft ) + M = 0
2

V = ( 10x + 110) lb

(3)

M= ( 5x2 + 110x

(4)

M
Solving gives
7 ft

7 ft

x
RA = 40 lb

valid for 7 ft < x < 17 ft.

1558

245) lb ft

8.2 Shear and Bending-Moment Diagrams: Equation Form Example 3, page 4 of 6


6

Pass a section through the beam at a point between the right end
of the distributed load and the right end of the beam.
x
10 lb/ft
B

A
7 ft

10 ft

3 ft
RB = 60 lb

RA = 40 lb
17 ft < x < 20 ft
7

Draw a free-body diagram of the portion of the beam to the left


of the section and solve for V and M at the section.
Resultant = (10 lb/ft)(10 ft) = 100 lb

10 ft
= 5 ft
2

M
7 ft

10 ft
x
x

RA = 40 lb

1559

17 ft

8.2 Shear and Bending-Moment Diagrams: Equation Form Example 3, page 5 of 6


+

Fy = 0: 40 lb

MA = 0:

100 lb

V=0

(40 lb)x + (100 lb)[(x

17 ft) + 5 ft] + M = 0

Solving gives
V = 60 lb

(5)

M = ( 60x + 1200) lb ft

(6)

valid for 17 ft < x < 20 ft.

Collect the results from Eqs. 1-6:


0 < x < 7 ft

V = 40 lb
M = 40x lb ft

7 ft < x < 17 ft

V = ( 10x + 110) lb
Ans.
M = ( 5x2 + 110x

17 ft < x < 20 ft

245) lb ft

V = 60 lb
M = ( 60x + 1200) lb ft

1560

8.2 Shear and Bending-Moment Diagrams: Equation Form Example 3, page 6 of 6


10 Plot V and M versus x.

10 lb/ft
B

A
7 ft

10 ft

3 ft

RA = 40 lb

RB = 60 lb

V
(lb) 40

40
x

360
M
(lb ft)

60

60

280
180

1561

8.2 Shear and Bending-Moment Diagrams: Equation Form Example 4, page 1 of 6


4. Express the shear V and bending moment M as functions of x, the distance from
the left end of the beam to an arbitrary point on the beam. Plot V and M vs. x.

B
18 kip ft

27 kip ft
3 ft
x

7 ft

Draw a free-body diagram and find the reactions.


A
18 kip ft

27 kip ft
3 ft

RA

7 ft

5 ft

Fy = 0: RA + RB = 0

5 ft

MA = 0: RB(3 ft + 5 ft + 7 ft)

27 kip ft

18 kip ft = 0

Solving gives
RA = 3 kip = 3 kip
RB = 3 kip

1562

RB

8.2 Shear and Bending-Moment Diagrams: Equation Form Example 4, page 2 of 6


2

Pass a section through the beam at a point between the left end and the 27 kip ft
moment couple.
x
B

A
18 kip ft

27 kip ft
3 ft

RA = 3 kip

7 ft

5 ft

0 < x < 3 ft
Draw a free-body diagram of the portion of the beam to the left
of the section and find V and M at the section.
x
M

Fy = 0:

V
RA = 3 kip

3 kip

V=0

Mx = 0: (3 kip)x + M = 0

Solving gives
V = 3 kip

(1)

M = 3x kip ft

(2)

valid for 0 < x < 3 ft.

1563

RB = 3 kip

8.2 Shear and Bending-Moment Diagrams: Equation Form Example 4, page 3 of 6


4

Pass a section through the beam at a point between the


27 kip ft and 18 kip ft moment couples.
x
B

A
18 kip ft

27 kip ft
RA = 3 kip

3 ft

7 ft

5 ft

RB = 3 kip

3 ft < x < 8 ft
Draw a free-body diagram of the portion of the beam to the left
of the section and find V and M at the section.
Fy = 0:
x
M

A
27 kip ft
3 ft

3 ft

3 kip

Mx = 0: (3 kip)x

V=0
27 kip ft + M = 0

Solving gives

RA = 3 kip

V = 3 kip

(3)

M = ( 3x + 27) kip ft

(4)

valid for 3 ft < x < 8 ft.

1564

8.2 Shear and Bending-Moment Diagrams: Equation Form Example 4, page 4 of 6


6

Pass a section through the beam at a point between the 18 kip ft


moment couple and the right end of the beam.
x
B

A
18 kip ft

27 kip ft
3 ft

5 ft

7 ft

RB = 3 kip

RA = 3 kip
3 ft < x < 8 ft

x
M

3 kip

Mx = 0: (3 kip)x

V=0
27 kip ft

18 kip ft + M = 0

Solving gives

18 kip ft

27 kip ft
3 ft

Fy = 0:

Draw a free-body diagram of the portion of the beam to the left


of the section and find V and M at the section.

5 ft

RA = 3 kip

V = 3 kip

(5)

M = ( 3x + 45) kip ft

(6)

valid for 8 ft < x < 15 ft.

1565

8.2 Shear and Bending-Moment Diagrams: Equation Form Example 4, page 5 of 6


8

Collect the results from Eqs. 1-6:


0 < x < 3 ft

V = 3 kip
M = 3x kip ft

3 ft < x < 8 ft

V = 3 kip
Ans.
M = ( 3x + 27) kip ft

8 ft < x < 15 ft

V = 3 kip
M = ( 3x + 45) kip ft

1566

8.2 Shear and Bending-Moment Diagrams: Equation Form Example 4, page 6 of 6


9

Plot V and M versus x.


B

A
18 kip ft

27 kip ft
3 ft
RA = 3 kip

7 ft

5 ft

RB = 3 kip

V
(kip)

21
M
(kip ft)

18

3
x
9

1567

8.2 Shear and Bending-Moment Diagrams: Equation Form Example 5, page 1 of 6


5. Express the shear V and bending moment M in the horizontal portion ACDB of
the beam as functions of x, the distance from the left end of the beam to an
arbitrary point on the beam. Plot V and M versus x.
4 kip

4 kip

B
C
2 ft
x

4 ft

2 ft

Fy = 0: RA

Draw a free-body diagram and find the reactions.


4 kip

2 ft

4 kip

4 kip + RB = 0

4 kip

2 ft

MA = 0:

(4 kip)(2 ft)

(4 kip)(10 ft)
+ RB(12 ft) = 0

RA

B
2 ft

2 ft

4 ft

2 ft

1568

2 ft

Solving gives
RA = 4 kip

RB

RB = 4 kip

8.2 Shear and Bending-Moment Diagrams: Equation Form Example 5, page 2 of 6


Pass a section through the beam at a point between the left end and the attachment point for the first arm.
4 kip
4 kip

B
2 ft

2 ft

4 ft

2 ft

2 ft
RB = 4 kip

RA = 4 kip
0 < x < 4 ft

Fy = 0: 4 kip

Draw a free-body diagram of the


portion of the beam to the left of the
section and find V and M at the
section. Note carefully that the 4-kip
force on the left arm does not act on
this free body.

Mx = 0:

V=0

(4 kip)x + M = 0

Solving gives

A
x

V = 4 kip

(1)

M = 4x kip ft

(2)

valid for 0 < x < 4 ft.

RA = 4 kip

1569

8.2 Shear and Bending-Moment Diagrams: Equation Form Example 5, page 3 of 6


Pass a section through the beam at a point between the attachment points of the two arms.

4 kip

4 kip
x

B
2 ft

2 ft

4 ft

2 ft

2 ft
RB = 4 kip

RA = 4 kip
4 ft < x < 8 ft

4 kip
x
M

2 ft

Mx = 0:

4 kip

V=0

(4 kip)x + (4 kip)(x

4 ft + 2 ft) + M = 0

Solving gives

A
2 ft

Fy = 0: 4 kip

Draw a free-body diagram of the portion of the beam to


the left of the section and find V and M at the section.

4 ft

V=0

(3)

M = 8 kip ft

(4)

V
valid for 4 ft < x < 8 ft.

RA = 4 kip

1570

8.2 Shear and Bending-Moment Diagrams: Equation Form Example 5, page 4 of 6


6

Pass a section through the beam at a point between the point


of attachment of the right arm and the right end of the beam.
4 kip

4 kip
x

B
2 ft

2 ft

4 ft

2 ft

2 ft
RB = 4 kip

RA = 4 kip

Draw a free-body diagram of the portion of the beam to


the left of the section and find V and M at the section.
4 kip

Note that the 4-kip force on the right arm acts on the free
body.
Fy = 0: 4 kip 4 kip 4 kip V = 0

8 ft < x < 12 ft

4 kip

Mx = 0:

(4 kip)x + (4 kip)(x
+ (4 kip)(x

2 ft)
10 ft) + M = 0

Solving gives
M

A
2 ft

2 ft

4 ft

2 ft

RA = 4 kip

10 ft

V = 4 kip

(5)

M = ( 4x + 48) kip ft

(6)

valid for 8 ft < x < 12 ft.

1571

8.2 Shear and Bending-Moment Diagrams: Equation Form Example 5, page 5 of 6


9

Collect the results from Eqs. 1-6:


0 < x < 4 ft

V = 4 kip
M = 4x kip ft

4 ft < x < 8 ft

V = 0 kip
Ans.
M = 8 kip ft

8 ft < x < 12 ft

V = 4 kip
M = 4(x

12) kip ft

1572

8.2 Shear and Bending-Moment Diagrams: Equation Form Example 5, page 6 of 6


10 Plot V and M versus x.
4 kip

4 kip

B
2 ft

2 ft

4 ft

2 ft

2 ft
RB = 4 kip

RA = 4 kip
V
(kip)

11 Note that the jumps in the diagrams occur at the


attachment points of the arms, not at the points where
the 4-kip external loads act.
x
4

16

16

M
(kip ft)
8

1573

8.2 Shear and Bending-Moment Diagrams: Equation Form Example 6, page 1 of 8


6. Express the shear V and bending moment M as functions of x, the distance from
the left end of the beam to an arbitrary point on the beam. Plot V and M vs. x.
4 kN

8 kN

Hinge
C

A
B
2m

2m

2m

2m

x
Draw a free-body diagram and find the reactions.

4 kN

8 kN

hinge
C

A
B

RA

2m

2m

2m

RC

RB
Fy = 0: RA + RB
MA = 0: RB(2 m)

4 kN

2m

8 kN + RC = 0

(1)

(4 kN)(2 m + 2 m)
2 Two equations but three unknowns.
An additional equation is needed.

(8 kN)(2 m + 2 m + 2 m)
+ RC(2 m + 2 m + 2 m + 2 m) = 0

1574

(2)

8.2 Shear and Bending-Moment Diagrams: Equation Form Example 6, page 2 of 8


Pass a section through the beam at a point
immediately to the right of the hinge.

4 kN

8 kN

Hinge

2m
4

2m

Draw a free-body diagram of the portion of


the beam to the right of the section.

C
2m

(8 kN)(2 m) + Rc(2 m + 2 m) = 0

2m
RC

Write the equilibrium equation for the sum of moments about the hinge.

+
7

Because the section is next to a hinge,


the moment is known to be zero there
(that's what we mean by a "hinge").

Mhinge = 0:

2m

8 kN

M=0

2m

(3)

Note that we don't use the equation Fy = 0, because this equation


would introduce an additional unknown, the shear V at the hinge.

1575

8.2 Shear and Bending-Moment Diagrams: Equation Form Example 6, page 3 of 8


8

Solving Eqs. 1-3 gives


RA = 8 kN = 8 kN
RB = 16 kN
RC = 4 kN

Pass a section through the beam at a point


between the left end and the reaction at B.

4 kN

8 kN

Hinge

A
B
2m
RB = 16 kN

2m

2m
RC = 4 kN

2m
RA = 8 kN
0<x<2m

10 Draw a free-body diagram of the portion of the beam to


the left of the section and find V and M at the section.

Fy = 0:

kN

V=0

Mx = 0:

kN)x +

=0

Solving gives

A
V
RA = 8 kN

V=

kN

(4)

M=(

x) kNm

(5)

valid for 0 < x < 2 m.

1576

8.2 Shear and Bending-Moment Diagrams: Equation Form Example 6, page 4 of 8


11 Pass a section through the beam at a point
between the the reaction at B and the hinge.

8 kN

4 kN

Hinge

x
A

B
2m

2m
RB = 16 kN

RA = 8 kN

2m

2m
RC = 4 kN

2m<x<4m

12 Draw a free-body diagram of the portion of the beam to


the left of the section and find V and M at the section.

B
2 ft
RA = 8 kN

RB = 16 kN

Fy = 0:

8 kN + 16 kN

Mx = 0: (8 kN)x

V=0

(16 kN)(x

2 m) + M = 0

Solving gives
V = 8 kN
M = (8x

(6)
32) kNm

valid for 2 m < x < 4 m.

1577

(7)

8.2 Shear and Bending-Moment Diagrams: Equation Form Example 6, page 5 of 8


13 Pass a section through the beam at a point
between the hinge and the 8-kN force.
x

4 kN

8 kN

Hinge

B
2m
RA = 8 kN

2m
RB = 16 kN

2m

2m
RC = 4 kN

4m<x<6m
Fy = 0:

14 Draw a free-body diagram of the portion of the beam to


the left of the section and find V and M at the section.
x

V=0

Solving gives

Hinge

A
2m
RA = 8 kN

4 kN

Mx = 0: (8 kN)x 16 kN)(x 2 m)
+ (4 kN)(x 4 m) + M = 0

4 kN

8 kN + 16 kN

V = 4 kN

(8)

V
M = (4x

2m
RB = 16 kN

16) kNm

valid for 4 m < x < 6 m.

1578

(9)

8.2 Shear and Bending-Moment Diagrams: Equation Form Example 6, page 6 of 8


15 Pass a section through the beam at a point between
the 8-kN force and end C of the beam.
4 kN

8 kN

x
Hinge
A

C
B
2m
RA = 8 kN

2m
RB = 16 kN

2m

2m
RC = 4 kN

6m<x<8m

16 Draw a free-body diagram of the portion of the beam to the


left of the section and find V and M at the section.

Fy = 0:

4 kN

8 kN

Hinge
B
V
2m
RB = 16 kN

Mx = 0: (8 kN)x (16 kN)(x 2 m)


+ (4 kN)(x 4 m)
+ (8 kN)(x 6 m) + M = 0
Solving gives

A
2m
RA = 8 kN

8 kN + 16 kN 4 kN
8 kN V = 0

2m

V = 4 kN

(10)

M = ( 4x + 32) kNm

(11)

valid for 6 m < x < 8 m.

1579

8.2 Shear and Bending-Moment Diagrams: Equation Form Example 6, page 7 of 8


17 Collect the results from Eqs. 4-11:
0<x<2m

V = 8 kN
M = 8x kNm

2m<x<4m

V = 8 kN
M = (8x

32) kNm
Ans.

4m<x<6m

V = 4 kN
M = (4x

6m<x<8m

16) kNm

V = 4 kN
M = ( 4x + 32) kNm

1580

8.2 Shear and Bending-Moment Diagrams: Equation Form Example 6, page 8 of 8


18 Plot V and M versus x.
4 kN

8 kN

Hinge

x
A

B
2m
RA = 8 kN

V
(kN)

2m
RB = 16 kN

2m

2m
RC = 4 kN

8
4

4
x
4

M
(kN m)

16

1581

8.2 Shear and Bending-Moment Diagrams: Equation Form Example 7, page 1 of 7


7. Express the shear V and bending moment M as functions of x, the distance from
the left end of the beam to an arbitrary point on the beam. Plot V and M vs. x.
20 kip

4 kip/ft
Hinge
B

A
5 ft

5 ft

8 ft

x
Draw a free-body diagram and find the reactions.
20 kip
5 ft
5 ft
Hinge
A

8 ft
= 4 ft
2

Resultant = (4 kip/ft)(8 ft) = 32 kip

MA

2
RA

8 ft

A couple moment must always be


included at a built-in end of a beam.

RB

Fy = 0: RA
MA = 0: MA

20 kip

32 kip + RB = 0

(20 kip)(5 ft)

(1)

(32 kip)(5 ft + 5 ft + 4 ft)

+ RB(5 ft + 5 ft + 8 ft) = 0

1582

3 Two equations but three unknowns.


An additional equation is needed.
(2)

8.2 Shear and Bending-Moment Diagrams: Equation Form Example 7, page 2 of 7


Pass a section through the beam at a point immediately to the right of the hinge.

8 ft
= 4 ft
2

20 kip

Resultant = (4 kip/ft)(8 ft) = 32 kip

Hinge
A

MA

B
5 ft

5 ft

8 ft

RA
5

RB

Draw a free-body diagram of the portion of


the beam to the right of the section.

4 ft

Resultant = 32 kip

M=0
6

Because the section is next to a hinge,


the moment is known to be zero there
(that's what we mean by a "hinge").

B
V

7 Write the equilibrium equation for the sum of moments about the hinge.

Mhinge = 0:

(32 kip)(4 ft) + RB(8 ft) = 0

(3)

Note that we don't use the equation Fy = 0, because this equation


would introduce an additional unknown, the shear V at the hinge.

1583

8 ft
RB

8.2 Shear and Bending-Moment Diagrams: Equation Form Example 7, page 3 of 7


9

Solving Eqs. 1-3 gives


RA = 36 kip
RB = 16 kip
MA = 260 kip ft

10 Pass a section through the beam at a point between


the left end and the 20-kip force.
20 kip
x
MA = 260 kip ft

4 kip/ft
Hinge
B

A
5 ft

5 ft

8 ft

RA = 36 kip

RB = 16 kip
Fy = 0: 36 kip

11 Draw a free-body diagram of the portion of the beam to


the left of the section and find V and M at the section.

0 < x < 5 ft

Mx = 0: 260 kip ft

V = 36 kip

M
A
RA = 36 kip

(36 kip)x + M = 0

Solving gives

MA = 260 kip ft

V=0

M = (36x

(4)
260) kip ft

valid for 0 < x < 5 ft.

1584

(5)

8.2 Shear and Bending-Moment Diagrams: Equation Form Example 7, page 4 of 7


12 Pass a section through the beam at a point between
the 20-kip force and the hinge.
20 kip

4 kip/ft

x
MA = 260 kip ft

Hinge
B

A
5 ft

5 ft

8 ft

RA = 36 kip

RB = 16 kip

5 ft < x < 10 ft
13 Draw a free-body diagram of the portion of the beam to
the left of the section and find V and M at the section.
Fy = 0: 36 kip

20 kip

MA = 260 kip ft

20 kip

Mx = 0: 260 kip ft

V=0

(36 kip)x + 20 kip(x

5 ft) + M = 0

M
Solving gives

A
5 ft

V = 16 kip

x
RA = 36 kip

M = (16x

(6)
160) kip ft

valid for 5 ft < x < 10 ft.

1585

(7)

8.2 Shear and Bending-Moment Diagrams: Equation Form Example 7, page 5 of 7


14 Pass a section through the beam at a point between
the hinge and the right end of the beam.
20 kip
x

4 kip/ft

Hinge

MA = 260 kip ft

A
5 ft

5 ft

8 ft
RB = 16 kip

RA = 36 kip
10 ft < x < 18 ft

20 kip
(4 kip/ft)(x

Resultant = (4 kip/ft)(x 10 ft)


x 10 ft
2

MA = 260 kip ft

20 kip

Fy = 0: 36 kip

15 Draw a free-body diagram of the portion of the beam to


the left of the section and find V and M at the section.

Mx = 0: 260 kip ft

V=0

(36 kip)x + 20 kip(x

+ (4 kip/ft)(x

10 ft)

10 ft)( x

10 ft )
2
+M=0

Hinge

A
5 ft

5 ft

Solving gives

x
V=

RA = 36 kip

4x + 56 kip

M = ( 2x2 + 56x
valid for 10 ft < x < 18 ft.

1586

(8)
360) kip ft

5 ft)

(9)

8.2 Shear and Bending-Moment Diagrams: Equation Form Example 7, page 6 of 7


16 Collect the results from Eqs. 4-9:
0 < x < 5 ft

V = 36 kip
M = (36x

5 ft < x < 10 ft

260) kip ft

V = 16 kip
Ans.
M = (16x

160) kip ft

10 ft < x < 18 ft V = ( 4x + 56) kip


M = ( 2x2 + 56x

360) kip ft

1587

8.2 Shear and Bending-Moment Diagrams: Equation Form Example 7, page 7 of 7


17 Plot V and M versus x.
x
MA = 260 kip ft

20 kip

4 kip/ft
Hinge
B

A
5 ft

5 ft

8 ft
RB = 16 kip

RA = 36 kip
36

36

V
(kN)

16

16

x
16
32
x
M
(kN m)
80

260

1588

8.2 Shear and Bending-Moment Diagrams: Equation Form Example 8, page 1 of 8


8. Express the shear V and bending moment M as functions of x, the distance from
the left end of the beam to an arbitrary point on the beam. Plot V and M vs. x.
4 kN/m
2 kN/m

A
x
1

6m

6m

Draw a free-body diagram and find the reactions.


2 kN/m
2 kN/m
B

A
6m

6m
RA

RB

Replace the trapezoidal distributed load by the sum of a rectangular and triangular load.

1589

8.2 Shear and Bending-Moment Diagrams: Equation Form Example 8, page 2 of 8


4

Resultant of rectangular load

Resultant of triangular load


1
(12 m)(2 kN/m)
2

= (12 m)(2 kN/m)

= 24 kN

= 12 kN

12 m

5
B

1 (6 m + 6 m) = 4 m
3
(acts through centroid of triangle)

A
6m

6m
RA

Fy = 0: RA
MA = 0:

24 kN

2 kN/m

12 m

2 kN/m

RB

12 kN + RB = 0

(12 kN)(2 m) + RB(6 m) = 0

Solving gives
RA = 32 kN
RB = 4 kN

1590

6m

4m=2m

8.2 Shear and Bending-Moment Diagrams: Equation Form Example 8, page 3 of 8


7

Pass a section through the beam at a point between the left end
and the support at A.
x

4 kN/m

2 kN/m

A
6m

6m
RA = 32 kN

0<x<6m

RB = 4 kN

Draw a free-body diagram of the portion of the beam to the left


of the section and solve for V and M.

2 kN/m
w = distributed load (kN/m) at location x
M
x
V

1591

8.2 Shear and Bending-Moment Diagrams: Equation Form Example 8, page 4 of 8


9

Before we can solve for V and M, we have to express w as a function of x. This can be done
by noting that w is a linear function of x and then using the slope-intercept equation for a line.
w

(0, 2 kN/m)

(12 m, 4 kN/m)

(x, w)

Slope
x

Intercept

w = mx + b
4 kN/m 2 kN/m
x + 2 kN/m
12 m 0
x
=
+2
(1)
6
=

10 Now the distributed load on the free-body of length x can be


replaced by the resultant of a rectangular and triangular load.
11 Resultant of rectangular load
Free-body diagram

= (2 kN/m)x
w

2 kN/m

12 Resultant of triangular load

x
2

x
3

M
x

x
V

1592

x
(w
2

2)

8.2 Shear and Bending-Moment Diagrams: Equation Form Example 8, page 5 of 8


x (w 2) V = 0
2
Mx = 0: (2 kN/m)(x)( x ) + [ x (w 2)( x )] + M = 0
3
2
2

Fy = 0:

13

(2 kN/m)x

Replacing w in these equations by w = (x/6) + 2 from


Eq. 1 and solving gives
V= (
M= (

x2
12
x3
36

2x) kN

(2)

x2) kN m

(3)

valid for 0 < x < 6 m.


14 Pass a section through the beam at a point between
the support at A and the support at B.
x

4 kN/m

2 kN/m

A
6m

6m
RA = 32 kN

RB = 4 kN

6 m < x < 12 m

1593

8.2 Shear and Bending-Moment Diagrams: Equation Form Example 8, page 6 of 8


15 Free-body diagram

w(x)

2 kN/m
M
A
V
6m

x 6m
RA = 32 kN

6 m < x < 12 m

16 We can save some work if we note that this free-body diagram is identical to the
previous one except that an additional vertical force of 32 kN is present. This
increases the shear in Eq. 2 by 32 kN and the moment in Eq. 3 by (32 kN)(x 6 m) so
x2
12
x3
M= (
36
V= (

2x + 32) kN

(4)

x2 + 32x

(5)

192) kN m

valid for 6 m < x < 12 m.

1594

8.2 Shear and Bending-Moment Diagrams: Equation Form Example 8, page 7 of 8


17 Collect the results from Eqs. 4-11:
0<x<6m

6 m < x < 12 m

V=(

1 x2
12

2x) kN

M=(

1 x3
36

x2) kN m

V=(

1 x2
12

2x + 32) kN

M=(

1 x3
36

x2 + 32x

Ans.

192) kN m

1595

8.2 Shear and Bending-Moment Diagrams: Equation Form Example 8, page 8 of 8


18 Plot V and M versus x.
x
2 kN/m

4 kN/m

6m

6m
RA = 32 kN

V
(kN)

RA = 4 kN

17
4

15
2.03
x

M
(kN m)

42

1596

8.3 Shear and Bending-Moment Diagrams Constructed by Areas

1597

8.3 Shear and Bending-Moment Diagrams Constructed by Areas Procedures and Strategies, page 1 of 3
Area of rectangle = 2 kN/m

Procedures and Strategies for Solving Problems Involving


Constructing Shear and Bending-Moment Diagrams by Areas

4 kN

3 m = 6 kN

2 kN/m

1. You can construct the shear diagram by using the following


rules:
3m

a) A downward concentrated load P causes a downward jump


of magnitude P in the shear diagram.
b) The change in shear between two "shear critical points"
(points at which a concentrated load acts or a distributed
load begins or ends) equals the negative of the area under the
distributed load curve, V =
w dx.
c) The slope of the shear curve equals the negative of the
distributed load, dV/dx = w. Thus if w = constant, the
shear curve is a straight line; if w is linear, the shear curve is
parabolic; and if no distributed load is present (w = 0), the
shear curve is a horizontal line.
Application of the rules to construct a shear diagram: Draw a
free-body diagram of the beam, and solve for the reactions.
Then starting from the left end of the beam, proceed from
critical point to critical point, and apply rules a) and b) to
determine the values of the shear at the critical points. Apply
rule c) to determine the type of curve that connects the shear
values at the critical points on the shear diagram.

1598

3m

V(kN)

x(m)
4

10
Change in shear
= negative of area
= 6 kN

8.3 Shear and Bending-Moment Diagrams Constructed by Areas Procedures and Strategies, page 2 of 3
2. You can construct the moment diagram by using
the following rules:

7 kN

10 kN m

a) a clockwise couple moment M causes an


upward jump in the moment diagram.
3m
b) The change in moment between two "moment
critical points" (shear critical points plus points
at which a couple moment acts) equals the area
under the shear curve, M = V dx.

3m

V(kN)

c) The slope of the moment curve equals the


shear, dM/dx = V. Thus if V is linear, M is
parabolic; if V is constant, M is linear.

x(m)
7

Application of the rules to construct a moment


diagram: Starting from the left end of the beam,
proceed from critical point to critical point, and
apply rules a) and b) to determine the values of
the moment at the critical points. Apply rule c) to
determine the type of curve that connects the
moment values at the critical points.

10

10

7
Area = 7 kN 3 m
= 21 kN m
= change in M

M(kN m)

x(m)

11

1599

8.3 Shear and Bending-Moment Diagrams Constructed by Areas Procedures and Strategies, page 3 of 3
Notes:
1) A point on the shear diagram at which V = 0
requires special consideration because dM/dx =
V = 0 implies that the moment M is a local
maximum or minimum there. First determine
perhaps by using similar triangles the
precise location where V = 0 on the shear
diagram. Then treat this point as an additional
critical point on the moment diagram and
calculate the moment there by applying rule b).
2) Both diagrams must close. If either diagram
does not close, check for a mistake in either
your calculation of the reactions or in your
calculation of V and M at successive critical
points.

4m

4m
2 kN/m

Similar triangles:
(4 s)
s
= 6
2

V(kN)
2

(4

s)
x(m)

6
M (kN m)

x (m)

1600

8.3 Shear and Bending-Moment Diagrams Constructed by Areas Problem Statement for Example 1
1. Draw the shear and moment diagrams for the beam.
6 kN
2m
A

4m
C

1601

8.3 Shear and Bending-Moment Diagrams Constructed by Areas Problem Statement for Example 2
2. Draw the shear and moment diagrams for the beam.
2 kN
2m

4 kN

3 kN
2m

2m

2 kN

3 kN
2m

2m

2m
B

1602

8.3 Shear and Bending-Moment Diagrams Constructed by Areas Problem Statement for Example 3
3. Draw the shear and moment diagrams for the beam.
30 lb/ft

24 ft

1603

8.3 Shear and Bending-Moment Diagrams Constructed by Areas Problem Statement for Example 4
4. Draw the shear and bending moment diagrams for the beam.
2 kip/ft

B
C
8 ft

8 ft

1604

8.3 Shear and Bending-Moment Diagrams Constructed by Areas Problem Statement for Example 5
5. Draw the shear and moment diagrams for the beam.
2 kN/m
C

A
B
2m

4m

1605

8.3 Shear and Bending-Moment Diagrams Constructed by Areas Problem Statement for Example 6
6. Draw the shear and moment diagrams for the beam.
60 kN m
A

B
C
10 m

20 m

1606

8.3 Shear and Bending-Moment Diagrams Constructed by Areas Problem Statement for Example 7
7. Draw the shear and moment diagrams for the beam.

B
4 ft

12 kip ft

8 kip ft

20 kip ft

4 ft

4 ft

E
4 ft

1607

8.3 Shear and Bending-Moment Diagrams Constructed by Areas Problem Statement for Example 8
8. Draw the shear and moment diagrams for the beam.
4 kN

2 kN/m

Hinge
B

8 kN m

3m

3m

3m

3m

1608

8.3 Shear and Bending-Moment Diagrams Constructed by Areas Problem Statement for Example 9
9. Draw the shear and moment diagrams for the beam.
21 kN
C

Hinge

15 kN m

B
D
4 kN/m
3m

3m

6m

1609

8.3 Shear and Bending-Moment Diagrams Constructed by Areas Example 1, page 1 of 8


1. Draw the shear and moment diagrams for the beam.
6 kN
A

4m
C

Equilibrium equations

First determine the reactions by drawing a free-body diagram


of the whole beam and writing equilibrium equations.

Fx

0: Ax

2m

Fy

0: Ay + By

2m
A

6 kN
4m
C
B

Ax
Ay

By

1610

MA

0: ( 6 kN)

Solving gives
Ax

Ay

4 kN

By

2 kN

2 m + By

(2 m + 4 m)

8.3 Shear and Bending-Moment Diagrams Constructed by Areas Example 1, page 2 of 8


2 Free-body diagram showing reactions
6 kN
C

2 kN

4 kN

Begin plotting the shear diagram by noting that the 4-kN upward
reaction force at A causes a 4-kN upward jump at A.

V(kN)
4
4
x(m)

1611

8.3 Shear and Bending-Moment Diagrams Constructed by Areas Example 1, page 3 of 8


6 kN
w

2m
0

4m
C

4 kN

2 kN

4 No distributed load, w, acts between A and C,


so the change in shear is zero:
V

negative of area under w curve


0

V(kN)

x(m)
2

1612

8.3 Shear and Bending-Moment Diagrams Constructed by Areas Example 1, page 4 of 8


6 kN
2m

4m
C
B

4 kN

2 kN

5 The 6-kN downward force causes


a 6-kN downward jump at C.
V(kN)

4
6 kN

x(m)

2
2( 4

6)

1613

8.3 Shear and Bending-Moment Diagrams Constructed by Areas Example 1, page 5 of 8


6 kN
2m

4m
C

B
w

4 kN

0
2 kN

No distributed load, w, acts between C


and B, so the change in shear is zero:
V

V(kN)

negative of area under w curve


0

2
-2

6
-2

1614

x(m)

8.3 Shear and Bending-Moment Diagrams Constructed by Areas Example 1, page 6 of 8


6 kN
2m

4m
C

B
2 kN

4 kN
7 The 2-kN upward reaction at B causes a
2-kN upward jump in the shear at B
V(kN)

4
2 kN
2
-2

0
-2

V
2 + 2 0. The shear diagram closes at
the end of the beam, as it must because the
internal shear V
2 must be balanced by the
2-kN external reaction, as shown in a free-body
diagram of a short length of beam at end B:

x(m)
dx
B
2 kN
(negative shear)

1615

2 kN

8.3 Shear and Bending-Moment Diagrams Constructed by Areas Example 1, page 7 of 8


6 kN
2m

4m
C
B

A
4 kN
V(kN)

2 kN

10 The change in moment equals


the area under the shear curve:

cross-hatched area
4 kN

A
9 No moment
acts at the end
of the beam,
so the moment
diagram starts
M(kN m)
at 0.

2
-2

x(m)

-2

2m

8 kN m
So the moment at point C is
MC

MA + M
0 + 8 kN m

4
1
A

x(m)
C
11 The curve connecting MA and MC must have constant slope,
since dM/dx shear 4 kN for all points between A and C. But
a "curve with constant slope" is simply a straight line.

1616

8.3 Shear and Bending-Moment Diagrams Constructed by Areas Example 1, page 8 of 8


6 kN
2m

4m
C

4 kN

2 kN
12 The change in moment equals the area under the
shear curve:

V(kN)
4

( 2 kN)(6 m

2 m)

8 kN m
6

C
A

2
-2

x(m)

So the moment at point B is


MB

-2

MC + M
8 + ( 8 kN m)

M(kN m)
8

0 (The moment diagram must close.)


2

1
B

x(m)

13 MC and MB are connected by a


straight line of slope
14 Maximum moment

8 kN m
dM/dx

1617

shear

2 kN

8.3 Shear and Bending-Moment Diagrams Constructed by Areas Example 2, page 1 of 5


2. Draw the shear and moment diagrams for the beam.
2 kN
2m

4 kN

3 kN
2m

2m

2 kN

3 kN
2m

2m

2m
B

Determine the reactions by drawing a free-body diagram of


the whole beam and writing equilibrium equations.

2 kN
2m

2m

Fx

0: Ax

Fy

0: Ay + By 2 kN 3 kN
3 kN 2 kN 0

2 kN

3 kN
2m

2m

0
4 kN

2m

2m

4 kN

3 kN

Equilibrium equations

MA

0:

2 kN

Ax
Ay

By

1618

Solving gives
Ax

Ay

7 kN

By

7 kN

2 m 3 kN 4m
4 kN 6 m 3 kN 8 m
2 kN 10 m + By 12 m

8.3 Shear and Bending-Moment Diagrams Constructed by Areas Example 2, page 2 of 5


2

Free-body diagram showing reactions


2 kN
2m

3 kN
2m

3 kN

4 kN
2m

2m

2 kN
2m

2m

C
A

B
7 kN

3 The 7-kN upward


reaction at A causes a
7-kN upward jump in
the shear diagram.

7 kN

No distributed load acts between A and C


so the shear is unchanged.

V(kN)
7

x(m)

1619

8.3 Shear and Bending-Moment Diagrams Constructed by Areas Example 2, page 3 of 5


2 kN
2m

3 kN
2m

4 kN
2m

2 kN

3 kN
2m

2m

2m

C
B

7 kN
5 The remainder of the
shear diagram
consists of horizontal
portions ( V area
under load curve 0)
and jumps at
concentrated loads.

7 kN

V(kN)

7
7

5
C 5
2

3
3

6
2
4

Diagram closes

x(m)

2
2

5
5

1620

2
7

5
7

8.3 Shear and Bending-Moment Diagrams Constructed by Areas Example 2, page 4 of 5


2 kN
2m

4 kN

3 kN
2m

2m

3 kN
2m

2 kN
2m

2m

C
A

B
7 kN

7 kN

V(kN)
7

5
2
B

A
2
7

x(m)

2
5

No moment acts at
the left end of the
beam so the moment
diagram starts at 0.

8 The change in moment equals


the area under the shear curve:
M

7 kN

2m

14 kN m

Thus the moment at C is

M(kN m)
14
7

MC

MA

14 kN m

x(m)

2
9 Since the shear is constant between A and C, dM/dx
and the moment curve is a straight line.

1621

7 kN,

8.3 Shear and Bending-Moment Diagrams Constructed by Areas Example 2, page 5 of 5


3 kN

2 kN
2m

2m

3 kN

4 kN
2m

2m

2 kN
2m

2m

B
7 kN

7 kN

V(kN)
7
5

5
2

x(m)
Area
2 2
4

Area
5 2
10

10 The remainder of the


moment diagram can be
found by adding areas
under the shear curve to
moment values.

Area
( 2)
4
24

14
M(kN m)

10

14

24
5

4
2

2
28
2
1

5
Area
( 5)
10
28
5

7
A

4
1

5
7

24
24

10

14
14

C
2

6
12 Maximum moment

28 kN m
11 Diagram closes

1622

Area
( 7) 2
14

14
x(m)

8.3 Shear and Bending-Moment Diagrams Constructed by Areas Example 3, page 1 of 5


3. Draw the shear and moment diagrams for the beam.
30 lb/ft

B
+

Fx

Equilibrium equations

Fy 0: Ay + By

0: Ax

24 ft

1 Determine the reactions by drawing a


free-body diagram of the whole beam and
writing equilibrium equations:

B
By

Ay

0: By

24 ft

Solving gives
30 lb/ft

Ax

MA

24 ft

1623

(30 lb/ft)

Ax

Ay

360 lb

By

360 lb

24 ft

(30 lb/ft)(24 ft)(24/2 ft)

8.3 Shear and Bending-Moment Diagrams Constructed by Areas Example 3, page 2 of 5


2

Free-body diagram of beam showing reactions

30 lb/ft

B
24 ft

360 lb

360 lb
4 The change in shear from end A to end B equals
the negative of the area under the load curve:
V

3 The 360-lb upward


reaction at A causes a
360-lb upward jump in
the shear diagram.

(area of rectangle under w curve)


(30 lb/ft)(24 ft)

720 lb

Thus the shear at B is

V(lb)
VB

360
30
A

VA

360 lb

( 720 lb)

360 lb

1
x(ft)
B

5 The curve connecting


VA and VB must be a
straight line since it
has constant slope:
dV/dx

360 + 360

360
6 The 360-lb vertical reaction
at end B closes the diagram.

w
30

1624

8.3 Shear and Bending-Moment Diagrams Constructed by Areas Example 3, page 3 of 5


30 lb/ft
8
A

B
24 ft

360 lb

360 lb

Because of symmetry, the point where


V 0 is easily found to lie at the
middle of the beam span. This point
corresponds to a local maximum of the
moment, because there the derivative
of M is zero:

V(lb)
dM/dx

360

V
0

B
(24 ft)/2
7

No moment
acts at the left
end of the
beam so the
M(lb ft)
moment
diagram starts
at 0.

12 ft

x(ft)
9

360

Change in moment equals area under shear


curve:
M

area of cross-hatched triangle


(1/2)(360 lb)(12 ft)

2160

2160 lb ft
So

x(ft)

MC

MA + M

12
0 + 2160 lb ft

1625

8.3 Shear and Bending-Moment Diagrams Constructed by Areas Example 3, page 4 of 5


V(lb)
360
C

12

x(ft)

360
M(lb ft)

10 Because the shear is a linear function


and dM/dx V, integration would show
that M is a quadratic function. But how
can you decide which of the two
possible quadratic curves is correct?

2160

x(ft)

12
dM/dx VC 0
(horizontal tangent)

M(lb ft)

11 Answer: Draw short line segments


showing the slope of the curve at
the ends of the interval and choose
the curve that matches these slopes.

360

x(ft)

12
dM/dx

VA

360 lb

1626

8.3 Shear and Bending-Moment Diagrams Constructed by Areas Example 3, page 5 of 5


30 lb/ft

B
12 Area

24 ft
360 lb

360 lb

(1/2)( 360 lb)(24 ft

12 ft)

2160 lb ft

V(lb)
So change in moment is

360

M
C

12

24 x(ft)
B

and
MB

M(lb ft)

dM/dx VC 0
(horizontal tangent)
2160

C
12

14 Maximum moment

2160 lb ft

360

2160 lb ft

MC + M
2160 lb ft

2160 lb ft

0 (diagram closes)

B
24

x(ft)

13 Moment curve is quadratic with


zero slope at point C.

1627

8.3 Shear and Bending-Moment Diagrams Constructed by Areas Example 4, page 1 of 6


4. Draw the shear and bending moment diagrams for the beam.
2 kip/ft

B
C
8 ft

8 ft

Determine the reactions by drawing a


free-body diagram of the whole beam
and writing equilibrium equations:

Ax

Fx

0: Ax

Fy

0: Ay

By

2 kip/ft

Equilibrium equations

MA

0: (8 ft + 8 ft)

Solving gives

C
Ay

By
8 ft

Ax

Ay

12 kip

By

4 kip

8 ft

1628

(2 kip/ft)

8 ft

By (2 kip/ft)
(8 ft)[(8 ft)/2] 0

8.3 Shear and Bending-Moment Diagrams Constructed by Areas Example 4, page 2 of 6


2

Free-body diagram showing reactions


2 kip/ft

B
C
12 kip

4 kip
8 ft

8 ft
4

3 The 12-kip
reaction at A
causes a jump in
the shear diagram.

V(kip)

The change in shear equals the negative


of the area under the load curve, w:
V

12

(area of rectangle under w)


(2 kip/ft)

2
1

16 kip

C
8

x(ft)
Thus the shear at C is
4

5 The curve connecting VA and


VC must be a straight line
since it has constant slope:
dV/dx

VC

VA
12
4

w
2

1629

V
16

8 ft

8.3 Shear and Bending-Moment Diagrams Constructed by Areas Example 4, page 3 of 6


2 kip/ft

A
C
12 kip

4 kip

Since the distributed load, w, between


C and B is zero, the change in shear is
zero:

V(kip)
V

7 The reaction at B causes a


jump in the shear curve
and closes the diagram.

area under w curve

12
0 (no change in V)

x(ft)

1630

8.3 Shear and Bending-Moment Diagrams Constructed by Areas Example 4, page 4 of 6


8 ft

8 ft
2 kip/ft
B

A
C
12 kip
V(kip)

10 One way to locate the V 0


point is to use similar triangles:

4 kip
12

12

s C
4

s
8

A
Because no
moment reaction is
present at A, the
moment diagram
starts at 0.

M(kip ft)

C
8

4
9

x(ft)

V
0 (the condition for a maximum or minimum M)

16

1631

(8

s)/4

Solving gives s

We have to locate the point where V 0 (that is, find s) because


the moment diagram has a local maximum or minimum there:
dM/dx

s/12

x(ft)

8.3 Shear and Bending-Moment Diagrams Constructed by Areas Example 4, page 5 of 6


8 ft

8 ft
2 kip/ft

A
C
12 kip

4 kip

V(kip)
12
8

11 Change in moment equals area


A
under shear curve:

C
8

s=6
M

B
4

area of triangle
12 M

MA

x(ft)

(1/2)(12 kip)(6 ft)


0

36 kip ft

36 kip ft
M(kip ft)

36
dM/dx

dM/dx 12
A

12
6

1632

13 Because V is linear, M is
quadratic. Because dM/dx V,
M has a slope of 12 at the left
end of the curve and a slope of
zero at the right end.
x(ft)

8.3 Shear and Bending-Moment Diagrams Constructed by Areas Example 4, page 6 of 6


8 ft

8 ft
2 kip/ft
B

A
C

4 kip

12 kip
V(kip)
12

14 Change in moment:
M

16 Change in moment:
M

area of triangle
6) A

(1/2)( 4)(8

( 4)(16
16 B

8C
4

36 + M
36

36

So
MB

32

4
1
A
6

8 C

MC

32

( 32)

4
32 kip ft

x(ft)

17 The curve is a line with constant slope:


18 Maximum moment

dM/dx

1633

8)

32 kip ft

15 Quadratic curve has zero


slope at the left end.

M(kip ft)
MC

x(ft)

4 kip ft
So

area of rectangle

V= 4

8.3 Shear and Bending-Moment Diagrams Constructed by Areas Example 5, page 1 of 6


5. Draw the shear and moment diagrams for the beam.
2 kN/m
C

A
B

4m

Determine the reactions by drawing a


free-body diagram of the whole beam
and writing equilibrium equations:

Equilibrium equations

A
B

0: Cx

Fy

0: Cy + By

MC

0:

By

Cx
Solving gives

Cy

By
2m

Fx

2 kN/m

2m

4m

1634

By

9 kN

Cx

Cy

3 kN

(2 kN/m)(2 m + 4 m)

4 m + (2 kN/m)(2 m + 4 m)[(2 m + 4 m)/2]

8.3 Shear and Bending-Moment Diagrams Constructed by Areas Example 5, page 2 of 6


2 Free-body diagram showing reactions
2 kN/m

C
B
9 kN

3 kN

2m

4m
4

Change in shear from A to B equals the negative


of the area under the load curve w:
V

3 No concentrated force
reaction at end A, so the
shear curve starts at 0.

V(kN)

(area of cross-hatched rectangle)


(2kN/m)(2 m)
4 kN

x(m)

2
1
5

4 ( VA + V

0 + [ 4 kN])

Slope of shear curve equals negative


of load, w:
dV/dx

1635

2kN

8.3 Shear and Bending-Moment Diagrams Constructed by Areas Example 5, page 3 of 6


2 kN/m
A

C
B
9 kN

3 kN

2m

4m

6 The 9-kN upward


reaction causes a
9-kN upward jump in
the shear curve.

7 Change in shear from B to C


equals negative of area of
cross-hatched rectangle:
V
V(kN)

5(

(2kN/m)(4 m)
8

4 + 9)

9 The end reaction


causes a 3-kN jump.

2
1

9 kN

0(

3 + 3)
C

x(m)

2
1

3( 5

4
8

8)

Slope of shear curve equals negative of distributed load, w:


dV/dx

1636

8.3 Shear and Bending-Moment Diagrams Constructed by Areas Example 5, page 4 of 6


2 kN/m
A

B
2m

4m
9 kN

V(kN)

3 kN

2 B

A
10 Because no moment
reaction acts at end A,
the moment diagram
starts at zero.

3
11 Change in moment equals area under shear
curve:

M(kN m)
dM/dx

area of triangle
(1/2)(2 m)( 4 kN) = 4 kN m

x(m)

MB = MA + M
4

x(m)

= 0 + ( 4 kN m)
= 4 kN m

1637

12 Because V is linear, M is quadratic.


Because dM/dx V, the M curve has a slope
of zero at the left end of the curve and a
slope of 4 at the right end.

8.3 Shear and Bending-Moment Diagrams Constructed by Areas Example 5, page 5 of 6


2 kN/m

B
2m

4m

9 kN

V(kN)

5
4

13 We have to locate the point


where V 0, that is, find s.

3 kN

14 By similar triangles,

s/5

x(m)

(4 - s)/3

Solving gives

15 Change in moment:
M

area of triangle

2.5

(1/2)(2.5 m)(5 kN) = 6.25 kN m

M(kN m)
s

2.5
M

MB + M

4 kN m + 6.25 kN m

2.25 kN m

x(m)
5
1

16 Because V is linear, M is quadratic.


Because dM/dx = V, the M curve has a
slope of 5 at left end and zero at the right.

1638

8.3 Shear and Bending-Moment Diagrams Constructed by Areas Example 5, page 6 of 6


2 kN/m
A

B
2m

4m
3 kN

9 kN
V(kN)

2.5

17 Change in moment:
M

1.5

area of triangle
(1/2)(1.5 m)( 3 kN)

2
s = 2.5

4
M(kN m)

dM/dx

2.25 kN m

x(m)

MC

2.25 + M
2.25 + ( 2.25)

2.25
A

B
2

x(m)

4.5
3

1639

18 Because dM/dx = V, the M


curve has a horizontal slope at
the left end and a slope of 3 at
the right.

8.3 Shear and Bending-Moment Diagrams Constructed by Areas Example 6, page 1 of 5


6. Draw the shear and moment diagrams for the beam.
60 kN m
A

B
C

Equilibrium equations

Determine the reactions by drawing a


free-body diagram of the whole beam and
writing equilibrium equations.
60 kN m

Ax

B
C
By

Ay
10 m

20 m

Fx

0: Ax

20 m

Fy

0: Ay + By

10 m

MA

0: 60 kN m + By(10 m + 20 m)

Solving gives
Ax

Ay

2 kN

By

1640

2 kN

8.3 Shear and Bending-Moment Diagrams Constructed by Areas Example 6, page 2 of 5


2

Free-body diagram showing reactions


60 kN m
A

B
C

2 kN

2 kN
10 m

3 The 2-kN upward


reaction causes a 2-kN
upward jump in the
shear diagram.

20 m

4 No distributed load, w, acts on the beam so the


change in shear is zero:
V

negative of area under w curve


0

V(kN)

The 2-kN downward


reaction causes a 2-kN
downward jump in the
shear diagram.

2
A

30

10

x(m)

6 The couple moment at C does not cause a jump in the shear diagram.

1641

8.3 Shear and Bending-Moment Diagrams Constructed by Areas Example 6, page 3 of 5


60 kN m
A

B
C
2 kN

2 kN
10 m

20 m

V(kN)
2

2
7

No moment at A so the
moment diagram starts at 0.

C
10

B
30

x(m)

8 The change in moment equals the


area under the shear curve:
80?
M(kN m)

20 kN m

20
9 The moment curve is a
straight line between A and
C because
dM/dx

1 2

(2 kN)(10 m)

B
30

C
10
40?

x(m)

10 The couple moment at C causes a jump of 60 kN m


in the moment diagram. But how can you decide if
the jump is up or down?

V
2 kN

1642

8.3 Shear and Bending-Moment Diagrams Constructed by Areas Example 6, page 4 of 5


x

11 Consider the moment equilibrium equation for a small


length of the beam containing the couple moment.
x
B

A
2 kN

60 kN m
60 kN m
M

2 kN

M+ M
V C

MLeft end

M + (M + M) + 60 kN m

Taking the limit as x


M

V+ V
(V + V)( x)

0 gives

60 kN m

General conclusion:
A counterclockwise couple moment applied to the beam produces a
downward jump in the moment diagram.

1643

8.3 Shear and Bending-Moment Diagrams Constructed by Areas Example 6, page 5 of 5


60 kN m
A

B
C

2 kN

12 The change in moment is

2 kN
10 m

20 m

V(kN)

area under shear curve


(2 kN)(30 m
40 kN m

So
C
10

B
30

x(m)

MB

MC + M
40 + 40

M(kN m)
0

20
B

A
60

10

x(m)

30
1
20

2
13 The moment curve is a straight line:
60

40

dM/dx

V
2

1644

10 m)

8.3 Shear and Bending-Moment Diagrams Constructed by Areas Example 7, page 1 of 3


7. Draw the shear and moment diagrams for the beam.

B
4 ft

12 kip ft

8 kip ft

20 kip ft

4 ft

D
4 ft

E
4 ft

Determine the reactions by drawing a free-body


diagram of the whole beam and writing
equilibrium equations:

Fx

0: Ax

Fy

0: Ay

Equilibrium equations

MA
Ax

20 kip ft
A

12 kip ft

8 kip ft
C

MA

0: MA

20 kip ft

Solving gives

Ay

Ax

Ay

MA

1645

16 kip ft

8 kip ft + 12 kip ft

8.3 Shear and Bending-Moment Diagrams Constructed by Areas Example 7, page 2 of 3


2

Free-body diagram showing reactions


16 kip ft

8 kip ft

20 kip ft

4
V(kip)
V(x)

12 kip ft
D

Because the distributed load w is zero everywhere


and the change in shear equals the negative of the
area under the w curve, the shear never changes
from its starting value: 0.
x(ft)

Because the reaction force at


the end of the beam is zero,
the shear there is zero.

1646

8.3 Shear and Bending-Moment Diagrams Constructed by Areas Example 7, page 3 of 3


Free-body diagram showing reactions
16 kip ft

20 kip ft

12 kip ft

8 kip ft

V(kip)
V(x)

0
x(ft)

6 Because the change in moment equals the area under the shear curve,
and the shear curve is V(x) 0, the moment does not change
between points where the couple moments are applied.
12 ( 4 + 8)

M(kip ft)
5 The counterclockwise
16 kip ft couple
moment causes a 16
kip ft downward
jump in the moment
diagram.

4(
A

20

16

16 + 20)

16

4
C

1647

12
12
0 ( 12
D

12)

x(ft)
E

Clockwise couple moments produce upward


jumps; counterclockwise, downward jumps.

8.3 Shear and Bending-Moment Diagrams Constructed by Areas Example 8, page 1 of 11


8. Draw the shear and moment diagrams for the beam.
4 kN

2 kN/m

Hinge
B

8 kN m

3m

3m

3m

3m
3

Free-body diagram of whole beam


4 kN

2 kN/m

Hinge

MA
Ax A

B
8 kN m

E
By

Ay
3m

3m

3m

3m

1648

Writing the three equilibrium equations (below)


gives three equations but four unknowns: Ax, Ay,
By, and MA, so an additional free body is needed.
+

Fx

0: Ax

Determine the reactions by drawing free-body


diagrams and writing equilibrium equations.

Fy

0: Ay + (2 kN/m)(3 m + 3 m)
4 kN + By 0

(2)

0: MA 8 kN m
+ (2 kN/m)(3 m + 3 m)
(3 m) (4 kN)
(3 m + 3 m + 3 m)
+ By(3 m + 3 m
+ 3 m + 3 m) 0

(3)

MA

(1)

8.3 Shear and Bending-Moment Diagrams Constructed by Areas Example 8, page 2 of 11


Free-body diagram of the whole beam (repeated)

4 kN

2 kN/m

Hinge

MA
Ax A

B
C

8 kN m

By

Ay
3m

3m

3m

3m

5 To obtain an additional free-body diagram, pass a section


through the beam close to the hinge. Then consider the
portion of the beam to the right of the section.
Free-body diagram of the portion of the
beam to the right of the section.

VD (unknown)
shear in beam Hinge
D

4 kN

7
B

MD 0
(moment is zero
at a hinge)

MHinge

0: By(3 m + 3 m)

(4 kN)(3 m)

(4)

Using a free body extending from the hinge to end B gives an


equation involving only one unknown, By.

By
3m

Moment equilibrium for portion DEB of the beam:

3m
8

Solving Eqs.1-4 gives


Ax

1649

0, Ay

10 kN, By

2 kN, and MA

16 kN m

8.3 Shear and Bending-Moment Diagrams Constructed by Areas Example 8, page 3 of 11


9 Free-body diagram showing reactions

4 kN

2 kN/m

Hinge

16 kN m

B
A

10 kN

8 kN m

E
2 kN

3m

13 The couple moment


has no effect on the
shear curve.

3m

3m

3m
12 An upward distributed load
is considered negative.

11 Change in shear:
V

area under load curve


( 2 kN/m)(3 m + 3 m)

V(kN)

14 VD

12 kN

10 kN + 12 kN
2 kN

10 The downward
reaction causes a
downward jump.

VA + V

x(m)

D
2

15 Slope of shear curve:

dV/dx

w
( 2)

10
2

1650

8.3 Shear and Bending-Moment Diagrams Constructed by Areas Example 8, page 4 of 11


4 kN

2 kN/m

Hinge
w 0

16 kN m
A
10 kN

0
B

8 kN m

w
E

2 kN

3m

3m

3m

3m

17 Jump in shear caused by


concentrated forces.

16 No change in shear because distributed load is zero:


V(kN)

3
A

2
6

2
9

10

1651

12
B
2

x(m)

8.3 Shear and Bending-Moment Diagrams Constructed by Areas Example 8, page 5 of 11


V(kN)
2

2
6
D

12
B
2

E
2
6

10

x(m)

s
18 To draw the moment diagram, we will need to calculate s,
the distance to the point where the shear curve crosses the
x-axis. By similar triangles, we have

s
6

2
D

s/10

(6-s)/2

Solving gives
10
s

1652

8.3 Shear and Bending-Moment Diagrams Constructed by Areas Example 8, page 6 of 11


20 Change in moment from A to
C equals cross-hatched area.
To calculate this area, first use
similar triangles to calculate
distance h.
h/10

4 kN

2 kN/m

Hinge

16 kN m

B
A

10 kN

8 kN m

E
2 kN

3m

2/5

C
3m

3m

3m

Solving gives
V(kN)
h

4
s

3C

3
A

2
6

F D

9
2

12
B
2

x(m)

10

10
5

M(kN m)
16
19 The clockwise couple moment at the end
causes an upward jump in the moment
diagram.

x(m)

1653

8.3 Shear and Bending-Moment Diagrams Constructed by Areas Example 8, page 7 of 11


4 kN

2 kN/m

Hinge

16 kN m
A
10 kN

8 kN m

21 Now that the value of h is known, the


cross-hatched area can be calculated:

E
2 kN

3m

3m

3m

3m

C
4

V(kN)
h
3 C

2
6

F D

10
12
B
2

9
2

( 4)

x(m)
Area

rectangle + triangle
(3 m)( 4 kN) + (1/2)(3 m)( 6 kN)
21 kN m

10
s

Thus the change in moment is


M

M(kN m)
16
A

3 C

x(m)
22 MC

MA + M

6 kN m + ( 21 kN m)

1654

5 kN m

21 kN m

8.3 Shear and Bending-Moment Diagrams Constructed by Areas Example 8, page 8 of 11


4 kN

2 kN/m

Hinge

16 kN m
A
10 kN

8 kN m

E
2 kN

3m

3m

3m

3m

V(kN)
3m

1m

2m

B
2

x(m)
24 Clockwise couple moment
causes an upward jump.

4
10

23 dM/dx V so curve is quadratic with


slopes of 10 and 4 at the ends

M(kN m)
16

10

1
3
A

C
4
1 5

x(m)

1655

8.3 Shear and Bending-Moment Diagrams Constructed by Areas Example 8, page 9 of 11


4 kN
2 kN/m

Hinge

16 kN m
A

8 kN m

10 kN

E
2 kN

3m

3m

3m

3m

V(kN)
3m

1m

2m

area of triangle
(1/2)( 4 kN)(2 m)

x(m)

26 MF

M(kN m)
16

27 dM/dx V, so
moment curve is
quadratic with slopes
of 4 and 0 at the ends

10

4 kN m

25 Change in moment:

Slope

MC + M

4
3 kN m + ( 4 kN m)
1 kN m

3
C

F
1

5
Slope

1656

x(m)

8.3 Shear and Bending-Moment Diagrams Constructed by Areas Example 8, page 10 of 11


4 kN

2 kN/m

Hinge

16 kN m
A

8 kN m

10 kN

E
2 kN

3m

3m

3m

3m

V(kN)
3m

F D

4
29 MD

area of triangle
(1/2)(2 kN)(1 m)

x(m)

B
2

28 Change in moment:
M

1m

2m

MF + M

10

1 kN m + 1 kN m
30 dM/dx V, so
moment curve is
quadratic with slopes
of 0 and 2.

1 kN m
M(kN m)
16
3
A

Slope

1657

(Actually, we knew in
advance that MD 0,
because a hinge
exists at point D.)
x(m)

1
5

8.3 Shear and Bending-Moment Diagrams Constructed by Areas Example 8, page 11 of 11


4 kN

2 kN/m

Hinge

16 kN m
A

8 kN m

10 kN

E
2 kN

3m

3m

3m

3m

V(kN)
3m

2m

1m

x(m)

B
2

31 ME

MD + M
MD + area

M(kN m)

0 + (2 kN)(3 m)

16

2 so
34 dM/dx
moment curve is a
straight line

ME + M
ME + area
6 kN m + ( 2 kN)(3 m)
0

6 kN m

3
A

33 MB

32 dM/dx V 2, so
moment curve is a
straight line

10

6
2 2
E

1
5

1658

1
B

x(m)

8.3 Shear and Bending-Moment Diagrams Constructed by Areas Example 9, page 1 of 11


9. Draw the shear and moment diagrams for the beam.
21 kN
C

Hinge

15 kN m

B
D
4 kN/m
3m

6m

Determine the reactions by drawing free-body diagrams


and writing equilibrium equations.

Free-body diagram of whole beam

3 Writing the three equilibrium equations (below) gives


three equations but four unknowns: Ax, Ay, By, and
MB, so an additional free body is needed.
+

Fx

0: Bx

3m

Fy

0:

Hinge

15 kN m

MB

By

4 kN/m
3m

3m

Bx

6m

1659

21 kN

Ay

MA

(1)

Ay + (4 kN/m)(3 m + 3 m)
21 kN + By 0

(2)

0: (4 kN/m)(3 m + 3 m)(3 m)
(21 kN)(3m) 15 kN m
+ By(3 m + 3 m + 6 m)
+ MB 0

(3)

8.3 Shear and Bending-Moment Diagrams Constructed by Areas Example 9, page 2 of 11


Free-body diagram of whole beam (repeated)
Hinge
21 kN
Ay
C
15 kN m
A
D

MB
Bx
B

4 kN/m
3m
6

6m

Free-body diagram of the portion of the beam


to the left of the section.
Hinge
21 kN
Ay
C
M 0 at hinge
15 kN m
Dx
A
D
V(unknown)
4 kN/m
shear in beam
3m

MHinge

To obtain an additional free-body diagram,


pass a section through the beam close to
the hinge. Then consider the portion of the
beam to the left of the section.

3m
8

Moment equilibrium

3m

By

0: (Ay)(3 m + 3 m) (4 kN/m)(3 m + 3 m)(3m)


15 kN m + (21 kN)(3 m) 0

Using a free body extending from the hinge to end A gives an


equation involving only one unknown, Ay.

(4)

Solving Eqs. 1-4 gives


Ay

4 kN

Bx

By

1 kN

MB

1660

6 kN m

8.3 Shear and Bending-Moment Diagrams Constructed by Areas Example 9, page 3 of 11


9

Free-body diagram showing reactions


4 kN

21 kN

Hinge

15 kN m

6 kN m

A
D

4 kN/m
3m
10 The 4-kN downward
reaction causes a
downward jump

3m

6m
11 VC

V(kN)

4 kN + [ ( 4 kN/m)(3 m)]
8 kN

A
4

13 ( 8
12 Slope of shear curve equals
negative of distributed load, w:
dV/dx

( 4)

VA + V
VA + [ cross-hatched area]

4 1

An upward
distributed load is
considered negative.

1 kN

x(m)

21

21)

13 The 21-kN
concentrated load
causes a 21-kN jump
in the shear diagram

1661

8.3 Shear and Bending-Moment Diagrams Constructed by Areas Example 9, page 4 of 11


4 kN

Hinge

21 kN
C

15 kN m

6 kN m

A
D

4 kN/m
3m

1 kN

3m

6m

14 VD

VC + V
VC + [ cross-hatched area]

V(kN)

13 kN + [ ( 4 kN/m)(3 m)]
8
1 kN

x(m)

4
4

15 Slope of shear curve:

1
13

dV/dx

w
( 4)
4

1662

8.3 Shear and Bending-Moment Diagrams Constructed by Areas Example 9, page 5 of 11


21 kN

4 kN

Hinge

15 kN m

6 kN m

A
D

4 kN/m
3m

1 kN

3m

6m
17 The 1-kN upward reaction
at B closes the diagram.

V(kN)
8

16 Because the distributed load is zero,


the change in shear is zero.

D
1

13

1663

x(m)

8.3 Shear and Bending-Moment Diagrams Constructed by Areas Example 9, page 6 of 11


4 kN

Hinge

21 kN

15 kN m

6 kN m

A
D

4 kN/m
3m

1 kN

3m

6m

V(kN)
8

4
s

18 To draw the moment diagram, we will need to


calculate s, the distance to the point where the
shear crosses the x-axis. By similar triangles,

13

s/4
8
A
4

s
E 3

x(m)

(3

s)/8

Solving gives

C
s

1664

8.3 Shear and Bending-Moment Diagrams Constructed by Areas Example 9, page 7 of 11


4 kN

Hinge

21 kN

15 kN m

6 kN m
D

4 kN/m
3m

1 kN

3m

6m

V(kN)
8
19 No moment acts at the
left end so the moment
curve starts at zero.

4
1m

20 ME

2m

x(m)

MA + M
MA + area of triangle

13
M(kN m)

0 + (1/2)( 4 kN)(1 m)
4

2 kN m
A

E
1

x(m)
2

slope

21 dM/dx V, so the moment curve is


quadratic with slopes of 4 and 0.

1665

8.3 Shear and Bending-Moment Diagrams Constructed by Areas Example 9, page 8 of 11


4 kN

Hinge

21 kN
C

15 kN m

6 kN m
D

4 kN/m
3m

1 kN

3m

6m

V(kN)
8

E
1m

22 MC

2m
13

M(kN m)
6
E

8
1 C

x(m)

D
ME + M

ME + area of triangle
2 kN m + (1/2)(8 kN)(2 m)
6 kN m
x(m)

2
Slope

23 dM/dx V, so the moment curve is quadratic


with slopes of 0 and 8.

1666

8.3 Shear and Bending-Moment Diagrams Constructed by Areas Example 9, page 9 of 11


Hinge

21 kN

4 kN
3m

3m
15 kN m

6m
6 kN m
D

4 kN/m

V(kN)

B
1 kN

D
1

x(m)

2m

1m

13

24 The clockwise 15 kN m couple moment causes an


upward jump in the moment curve.

M(kN m)
21 ( 6 + 15)

15
6
A

x(m)

1667

8.3 Shear and Bending-Moment Diagrams Constructed by Areas Example 9, page 10 of 11


4 kN

21 kN
3m

6m

3m
15 kN m

Hinge
6 kN m
D
4 kN/m

V(kN)

B
1 kN

4
2m

1m

25 MD
13

M(kN m)

x(m)

MC + M
MC + cross-hatched area

Slope

13
MC + area of rectangle + area of triangle

21

21 + ( 1 kN)(3 m) + (1/2)( 13

Slope
6
A

E
2

( 1))(3 m)

0
(A value of zero was expected because a
hinge is present at D.)

x(m)
26 dM/dx V, so moment curve is quadratic with
slopes of 13 and 1.

1668

8.3 Shear and Bending-Moment Diagrams Constructed by Areas Example 9, page 11 of 11


4 kN

Hinge

21 kN
C

15 kN m

6 kN m

A
D

4 kN/m

V(kN)

1 kN

8
3m

6m
B

27 MB

2m

1m

x(m)

MD + M
MD + cross-hatched area

13
M(kN m) 30 Maximum moment

21 kN m

0 + ( 1 kN)(6 m)

21
6 kN m

29 The clockwise 6 kN m couple moment


at the end closes the diagram.

6
A

B
1

2
28 dM/dx

x(m)

1
6

1669

9. Centroids and Mass Centers

1670

9.1 Centroids by Integration

1671

9.1 Centroids by Integration Procedures and Strategies, page 1 of 2


y

Procedures and Strategies for Solving Problems


Involving Calculating Centroids by Integration

y = f (x)
(x, y)
(xel, yel)

1. Determine the coordinates of the centroid by


evaluating integrals such as
xel dA
dA
For a planar area, the differential area dA is usually a
rectangular strip of finite length and differential width dx
(for a vertical strip) or dy (for a horizontal strip). Use a
vertical strip if the curve bounding the planar region is
given as a function of x, y = f(x). Use a horizontal strip
if the bounding curve is given as a function of y, x = g(y).
The integrand xel is the x coordinate of the centroid of the
strip. It must be expressed as a function of x for a
vertical strip and as a function of y for a horizontal strip.

xel = x

xc =

x
dx
y
(xel, yel)
xel = x/2
= g(y)/2
(x, y)

x = g(y)

dy

1672

9.1 Centroids by Integration Procedures and Strategies, page 2 of 2


2. For a line (a wire), the area element dA is replaced by
dL =
=

(dx)2 + (dy)2)
dL

1+ (dy/dx)2 dx

dy

dx
if the line is given as a function of x: y = f(x). Use
dL =

(dx/dy)2 + 1 dy

if the line is given as function of y: x = g(y).


z
3. For volumes with some degree of symmetry (for example, a solid
of revolution), dA can be replaced by a circular disk of finite radius
and differential thickness.

Radius = x
dy

4. Using the integral function on a scientific graphing calculator


simplifies the work and helps avoid errors.
z = f(x)

1673

9.1 Centroids by Integration Problem Statement for Example 1


1. Locate the centroid of the plane area shown. Use a
differential element of thickness dx.

y = 3x2

12 ft

x
2 ft

1674

9.1 Centroids by Integration Problem Statement for Example 2


2. Locate the centroid of the plane area shown, if a = 3 m
and b = 1 m. Use a differential element of thickness dy.
y

a
y = a sin(

x)
2b

x
b

1675

9.1 Centroids by Integration Problem Statement for Example 3


3. Locate the centroid of the plane area shown.
y
1 in

y = 4x5

3x2 + 12x + 1

13 in.

1 in.

1676

9.1 Centroids by Integration Problem Statement for Example 4


4. Locate the centroid of the plane area shown.
y
0.5 m
xy = 1

2m

0.5 m
x
2m

1677

9.1 Centroids by Integration Problem Statement for Example 5


5. Locate the centroid of the plane area shown.
y

y=

x(13
6

x)

6m
y = x2 + 14 3 11x
2m
x
1m
4m

1678

9.1 Centroids by Integration Problem Statement for Example 6


6. Locate the centroid of the plane area shown.
y

x=4

y2

x = 3y
1m
x
3m

1m

1679

9.1 Centroids by Integration Problem Statement for Example 7


7. Locate the centroid of the plane area shown. Use a
differential element of thickness dx.
y
y = hb x

x
b

1680

9.1 Centroids by Integration Problem Statement for Example 8


8. Locate the centroid of the plane area shown. Use a
differential element of thickness dy.
y

b
x = a[1 (

y 2
)]
b

x
a

1681

9.1 Centroids by Integration Problem Statement for Example 9


9. A sign is made of 0.5 in. thick steel plate in the shape shown.
Determine the reactions at supports B and C.
y
B
x = 50 + (10) sin y
24

72 in.

Specific weight = 490 lb3


ft

C
50 in.

1682

9.1 Centroids by Integration Problem Statement for Example 10


10. Locate the centroid of the wire shown.
y
3m

y = 2x2

18 m

1683

9.1 Centroids by Integration Problem Statement for Example 11


11. Locate the centroid of the wire shown.
y

x = 300[1

y 4
)]
200

200 mm

x
300 mm

1684

9.1 Centroids by Integration Problem Statement for Example 12


12. The rod is bent into the shape of a circular arc.
Determine the reactions at the support A.

0.2 lb/ft
3 ft
A

20

1685

9.1 Centroids by Integration Problem Statement for Example 13


13. a) Locate the centroid of the Gateway Arch in St.
Louis, Missouri, USA. b) During the pre-dawn hours of
September 14, 1992, John C. Vincent of New Orleans,
Louisiana, USA, climbed up the outside of the Arch to the
top by using suction cups and then parachuted to the
ground. Estimate the length of his climb.
Approximate equation of centerline:
y = 639.9 ft

(68.78 ft) cosh[(0.01003 ft-1)x]


y

625 ft

x
299 ft

299 ft

1686

9.1 Centroids by Integration Problem Statement for Example 14


14. Locate the centroid of the cone shown.
y
Radius = 2 m

3m

1687

9.1 Centroids by Integration Problem Statement for Example 15


15. Locate the centroid of the volume shown.
y
One-eighth of a
sphere of radius "a"

1688

9.1 Centroids by Integration Problem Statement for Example 16


16. Determine the x coordinate of the centroid of the solid
shown. The solid consists of the portion of the solid of
revolution bounded by the xz and yz planes.
y
a

x
b
z

x = a[1

( bz )2]

(This curve is rotated about the


x-axis to generate the solid.)

1689

9.1 Centroids by Integration Problem Statement for Example 17


17. Locate the centroid of the pyramid shown.
y

h
b
x
b
a
z

1690

9.1 Centroids by Integration Example 1, page 1 of 4


1. Locate the centroid of the plane area shown. Use a
differential element of thickness dx.
y

Definition of centroid coordinates

xc =
y = 3x2
yc =

xel dA

(1)

dA
yel dA

(2)

dA

where (xel, yel) are the coordinates of the


centroid of the differential area
element dA.

12 ft

x
2 ft

1691

9.1 Centroids by Integration Example 1, page 2 of 4


2

Locate the differential element so that it extends


from an arbitrary point (x, y) on the curve to an
opposite boundary of the crosshatched region.
y

y = 3x2

(x, y)

Express the element area in terms of the coordinates of a


point (x, y) on the curve:
dA = height width (of rectangle)

y = height

or,
dA = y dx

x
dx = width

(3)
x

By choosing an element of width dx, we have also


implicitly chosen x to be the variable of integration.

1692

9.1 Centroids by Integration Example 1, page 3 of 4


4

y = 3x2

Express the coordinates of the element


centroid in terms of the coordinates of the
point (x, y) on the curve. The x
coordinate of the element centroid is the
same as the x coordinate of the point on
the curve:
xel = x

Since the variable of integration is x, we


now have to express dA and yel in terms of
x (As can be seen from Eq. 4, xel already is
a function of x). The point (x, y) on the
curve satisfies
y = 3x2

(4)

Substituting the expression for y in Eq. 6


into the equations for dA (Eq. 3) and for yel
(Eq. 5) gives

Since the centroid of the differential


element is located in the center of the
element, the y coordinate of the element
centroid is

(x, y)

y
yel =
2

(6)

dA = y dx

(5)

(Eq. 3 repeated)

= 3x2 dx

(xel, yel)

y
yel = 2
3x2
= 2

y
x

dx
x

(7)

(8)

Evaluate the integral in the denominator


of the equation for xc over the range (from
0 to 2):
2

dA = 3x2 dx = 8 ft2

2 ft

1693

(9)

9.1 Centroids by Integration Example 1, page 4 of 4


7

Evaluate the integral in the numerator of the equation for xc


over the range from 0 to 2:
2

xel dA =

x(3x2) dx = 12 ft3

(10)

Evaluate the integral in the numerator of the equation for yc over


the range from 0 to 2:
2

yel dA =

[ 3x ](3x2) dx = 28.8 ft3


2

(11)

Substitute the results given in Eqs. 9, 10, and 11 into the


definitions for xc and yc:

xc =

yc =

xel dA
dA
yel dA
dA

= 12
8 = 1.5 ft

Ans.

= 28.8
8 = 3.6 ft

Ans.

1694

9.1 Centroids by Integration Example 2, page 1 of 3


2. Locate the centroid of the plane area shown, if a = 3 m
and b = 1 m. Use a differential element of thickness dy.

y
1

a
y = a sin(

x)
2b
3

Definition of centroid coordinates


xc =

xel dA

yc =

yel dA

Locate the differential element so that it


extends from an arbitrary point (x, y)
on the curve to an opposite boundary
of the crosshatched region.
y

(1)

dA
x
(2)

dA

dy

where (xel, yel) are the coordinates of the


centroid of the differential area
element dA.

Express the element area in terms of the


coordinates of a point (x, y) on the curve:
dA = width height (of rectangle)

(x, y)

y = a sin(

x)
2b

x
b

or,
dA = x dy

(3)

By choosing an element of width dy, we have


also implicitly chosen y to be the variable of
integration.

1695

9.1 Centroids by Integration Example 2, page 2 of 3


4
y
x
(xel, yel)

dy

(x, y)

x
xel = 2

y = a sin( x )
2b

yel = y

Since the variable of integration is y, we now


have to express dA and xel in terms of y (As
can be seen from Eq. 5, yel already is expressed
as a function of y). The point (x, y) on the
curve satisfies
y = a sin (

x)
2b

Solving this equation for x gives


(4)

y
x = ( 2b ) sin-1( a )

The y coordinate of the element


centroid is the same as the y
coordinate of the point on the
curve:

a
y

Express the coordinates of the


element centroid in terms of the
coordinates of the point (x, y) on
the curve. Since the centroid of
the differential element is
located in the center of the
element, the x coordinate of the
element centroid is:

(6)

Substituting the expression for x in Eq. 6 into


the equations for dA (Eq. 3) and for xel (Eq. 4)
gives
dA = x dy

(5)

(Eq. 3 repeated)

y
= [( 2b ) sin-1( a )] dy

(7)

x
x
xel = 2
6

Substitute a = 3 m and b = 1 m, and evaluate the integral in


the denominator of the equation for xc over the range from 0
to 3 (Use the integral function on your calculator):
3

dA =

y
) sin-1( ) dy = 1.090 m2
3

(9)

1696

(Eq. 4 repeated)

y
2b
= 12 ( ) sin-1( a )
y
= ( b ) sin-1( a )

(8)

9.1 Centroids by Integration Example 2, page 3 of 3


7

Similarly, evaluate the integral in the numerator of the equation for xc over the range
from 0 to 3 (Use the integral function on your calculator):
3

xel dA =

sin-1 (y/3)

] [(

sin-1(

y
)] dy = 0.2841 m3
3

(10)

And similarly evaluate the integral in the numerator of the equation for yc over the
range from 0 to 3 (Use the integral function on your calculator):
3

yel dA =

y [(

sin-1(

y
)] dy = 2.2500 m3
3

(11)

Substitute the results given in Eqs. 9, 10, and 11 into the definitions
for xc and yc:

xc =

xel dA

yc =

yel dA

dA

dA

= 0.2841
1.090 = 0.261 m

Ans.

= 2.2500
1.090 = 2.06 m

Ans.

1697

9.1 Centroids by Integration Example 3, page 1 of 5


3. Locate the centroid of the plane area shown.
y

y = 4x5

Definition of centroid coordinates

1 in

3x2 + 12x + 1

dx

y = 4x

3x + 12x + 1

xc =

xel dA

yc =

yel dA

dA

(1)

(2)

dA

14

where (xel, yel) are the coordinates of the


centroid of the differential area
element dA.
2

13 in.

Locate the differential element so that it extends from


an arbitrary point (x, y) on the curve to an opposite
boundary of the crosshatched region.

14 in.
(x, y)

Express the element area in terms of the coordinates


of a point (x, y) on the curve:
y

dA = height width (of rectangle)


or,
1 in.

dA = (14

y) dx

(3)
x

x
By choosing an element of width dx, we have also
implicitly chosen x to be the variable of integration.

1698

9.1 Centroids by Integration Example 3, page 2 of 5


4

Express the coordinates of the


element centroid in terms of the
coordinates of the point (x, y) on
the curve. The x coordinate of the
element centroid is the same as the
x coordinate of the point on the
curve:

y
dx

y = 4x5
(xel, yel)

xel = x

(4)

14

Since the centroid of the differential


element is located in the center of
the element, the y coordinate of the
element centroid is:
yel = y +

14

14

dA = (14

y
2

= [14
14 in

(x, y)

y = 4x

y) dx
(4x5

= ( 4x5 + 3x2
y
yel = 7 + 2
4x5
=7+

(5)

3x2 + 12x + 1

5
= 4x

3x + 12x + 1

x
x

1699

(6)

Substituting the expression for y from Eq. 6 into the


equations for dA (Eq. 3) and for yel (Eq. 5) gives

y
=7+ 2

Since the variable of integration is x, we now have


to express dA and yel in terms of x (As can be seen
from Eq. 4, xel already is expressed as a function of
x). The point (x, y) on the curve satisfies

(Eq. 3 repeated)
3x2 + 12x + 1)] dx
12x + 13) dx

(7)

(Eq. 5 repeated)

3x2 + 12x + 1
2
2
3x + 12x + 15
2

(8)

9.1 Centroids by Integration Example 3, page 3 of 5


6

What if we had chosen a differential element dy wide instead of


dx? The figure shows that now the element area is
dA = x dy
Since dy is the variable of integration, we must express x as a
function of y. But x and y are related by
y = 4x5

3x2 + 12x + 1

and this equation is difficult to invert, that is, to solve for x as a


function of y. So it is much easier to use a differential element
dx wide, because then we don't have to solve for x as a function
of y.
7

dy

y = 4x5

The general conclusion to draw is that whether we should choose


a differential element dx or dy wide depends on whether the
equation defining the boundary of the region can be more easily
written as a function of x or as a function of y.

x
x

1700

3x2 + 12x +1

9.1 Centroids by Integration Example 3, page 4 of 5


y
8

dx

Return now to a differential element dx wide.

9 Evaluate the integral in the denominator of the equation for xc over


the range from 0 to 1 (Use the integral function on your calculator):
1
5

y = 4x

dA =

3x + 12x + 1

( 4x5 + 3x2

12x + 13) dx = 7.333 in2

(9)

10 Similarly, evaluate the integral in the numerator of the equation for xc


over the range from 0 to 1 (Use the integral function on your calculator):
1

xel dA =

x ( 4x5 + 3x2

12x + 13) dx = 2.679 in3

(x, y)

11 And similarly evaluate the integral in the numerator of the equation for yc
over the range from 0 to 1 (Use the integral function on your calculator):
1

yel dA =
x
=

5
[ 4x

69.849 in3

1 in.

1701

3x2 + 12x + 15 ]( 4x5 + 3x2


2

12x + 13) dx

(11)

(10)

9.1 Centroids by Integration Example 3, page 5 of 5


12 Substitute the results given in Eqs. 9, 10, and 11 into the
definitions for xc and yc:

xc =

xel dA

yc =

yel dA

dA

dA

= 2.679
7.333 = 0.37 in.

Ans.

= 69.849
7.333 = 9.52 in.

Ans.

1702

9.1 Centroids by Integration Example 4, page 1 of 4


4. Locate the centroid of the plane area shown.

Definition of centroid coordinates

xc =

xel dA

yc =

yel dA

(1)

dA

0.5 m
xy = 1

(2)

dA
where (xel, yel) are the coordinates of the centroid of the
differential area element dA.
y

2m

xy = 1

0.5 m
x
dy

2m

(x, y)

2m
2

Locate the differential element so that it


extends from an arbitrary point (x, y)
on the curve to an opposite boundary.

y
0.5 m
x

1703

9.1 Centroids by Integration Example 4, page 2 of 4


y

Express the element area in terms of the coordinates of a point


(x, y) on the curve:

xy = 1

dA = height width (of rectangle)


or,

(xel, yel)
dA = x dy

(3)

(x, y)
By choosing an element of width dy, we have also implicitly
chosen y to be the variable of integration.

dy
y
4
x
x

Express the coordinates of the element centroid in terms of the


coordinates of the point (x, y) on the curve. Since the centroid
of the differential element is located in the center of the element,
the x coordinate of the element centroid is
xel = x2

(4)

The y coordinate of the element is the same as the y coordinate


of the point on the curve:
yel = y

1704

(5)

9.1 Centroids by Integration Example 4, page 3 of 4


5

Since the variable of integration is y, we now have to


express dA and xel in terms of y (As can be seen from
Eq. 5, yel already is a function of y). The point (x, y) on
the curve satisfies

xy = 1

xy =1
Solving for x gives
x = y1

dy

(6)
2m

Substituting the expression for x in Eq. 6 into the


equations for dA (Eq. 3) and for xel (Eq. 4) gives
dA = x dy
= 1y dy

(Eq. 3 repeated)
0.5 m
(7)

xel = x2
1/y
= 2

(Eq. 4 repeated)

= 1
2y

(8)

Evaluate the integral in the denominator of the equation for


xc over the range from 0.5 to 2:
2

dA =
0.5

1705

1 dy = 1.3863 m2
y

(9)

9.1 Centroids by Integration Example 4, page 4 of 4


7

Evaluate the integral in the numerator of the equation for xc


over the range from 0.5 to 2:
2

xel dA =

( 1 )( 1y ) dy = 0.7500 m3
2y

0.5

(10)

8 Evaluate the integral in the numerator of the equation for yc


over the range from 0.5 to 2:
2

yel dA =

0.5

y ( 1y ) dy = 1.5 m3

10

What if we had chosen a vertical element rather


than a horizontal element? The figure below
shows that we would have to define the element
areas in two equations, one for the region where y
is constant (y = 2 m), and one for the region where
y varies. We would also have to evaluate two
integrals, one over each region. Thus using a
vertical element rather than a horizontal element
would almost double the amount of work required.
Nevertheless, it would give us the correct answer.
y

(11)

(x, 2)
xy = 1

Substitute the results given in Eqs. 9, 10, and 11 into the


definitions for xc and yc:

xc =

xel dA

yc =

yel dA

dA

dA

(x, y)
2m

0.7500 = 0.541 m
= 1.3863

Ans.

1.5 = 1.082 m
= 1.3863

Ans.
x

1706

9.1 Centroids by Integration Example 5, page 1 of 3


5. Locate the centroid of the plane area shown.
y

x(13
y=
6

Definition of centroid coordinates

xc =

x)

yc =

xel dA
dA
yel dA

(1)

Locate the differential element so that it


extends from an arbitrary point on the
lower curve to a point directly above on
the upper curve.
y

(2)

dA
6m

dx
y = x2 +
2m

14

11x where (xel, yel) are the coordinates of the


centroid of the differential area element dA.
3

(x, y2)
y2 y1

x
3

1m

Express the element area in terms of the


coordinates of a point (x, y) on the curve:

(x, y1)

4m

y1
dA = height width (of rectangle)

x
x

or,
dA = (y2 y1) dx

(3)

By choosing an element of width dx, we have


also implicitly chosen x to be the variable of
integration.

1707

9.1 Centroids by Integration Example 5, page 2 of 3


4

Express the coordinates of the element centroid in terms


of the coordinates of the point (x, y) on the curve. The
x coordinate of the element is the same as the x
coordinate of the point on the curve:
xel = x

(4)

x(13 x)
6
14 11x
y1 = x2 +
3
x(13 x)
14 11x
y2 y1 = [
] [x2 +
]
3
6
2
= 7x + 35x 28
(6)
6

Since the centroid of the differential element is located


in the center of the element, the y coordinate of the
element centroid is average of the y coordinates of the
points on the top and bottom curves.
yel =

y2 + y1
2

Since the variable of integration is x, we now have to express


dA and yel in terms of x (As can be seen from Eq. 4, xel already
is a function of x). The y coordinates of the top and bottom of
the element satisfy
y2 =

(5)

Substituting the expression for y in Eq. 6 into the equations for


dA (Eq. 3)

dx

(x, y2)
dA = (y2 y1) dx
=

(xel, yel)
y2 y1
2

y2 y1

28

dx

(7)

and for yel (Eq. 5) gives


y2 + y1
2
[x(13 x)/6] + [x2 + (14
=
2
(5x2 9x + 28)
=
12

yel =
(x, y1)

7x2 + 35x
6

(Eq. 3 repeated)

y1
x

x = xel

1708

(Eq. 5 repeated)
11x)/3]
(8)

9.1 Centroids by Integration Example 5, page 3 of 3

dA =

Evaluate the integral in the denominator of the equation for xc over


the range from 1 to 4:

7x2 + 35x
6

28

y=

dx = 5.2500 m2

(9)

x(13
6

x)

Evaluate the integral in the numerator of the equation for xc over the
range from 1 to 4:
4

xel dA =

x[

7x2 + 35x
6

28

] dx = 13.1250 m3

y = x2 + 14 3 11x
dx

(10)

1m
4m
8

Evaluate the integral in the numerator of the equation for yc


over the range from 1 to 4:
4

yel dA =

(5x2

= 17.0625 m3

2
9x + 28)
][ 7x + 35x
12
6

Substitute the results given in Eqs. 9, 10, and 11 into the


definitions for xc and yc:

28 ] dx
xc =

xel dA

yc =

yel dA

= 13.125
5.25 = 2.50 m
dA

Ans.

(11)

1709

dA

= 17.0625
5.25 = 3.25 m

Ans.

9.1 Centroids by Integration Example 6, page 1 of 3


6. Locate the centroid of the plane area shown.
1

Definition of centroid coordinates

y
xc =
x=4

y2

x = 3y
yc =

1m

xel dA

(1)

dA
yel dA

(2)

dA
x
3m

where (xel, yel) are the coordinates of


the centroid of the differential area
element dA.

1m

Locate the differential element so that it


extends from an arbitrary point (x, y)
on the curve to an opposite boundary.

Express the element area in terms of the


coordinates of a point (x, y) on the curve:
dA = width

x=4
(x1, y)

y2

height (of rectangle)

or,

(x2, y)

dA = (x2

x1) dy

(3)

x = 3y
y

dy
x
x1

x2

x1

1710

By choosing an element of height dy, we


have also implicitly chosen y to be the
variable of integration.

9.1 Centroids by Integration Example 6, page 2 of 3


4

Express the coordinates of the element centroid in terms


of the coordinates of the point (x, y) on the curve. Since
the centroid of the differential element is located in the
center of the element, the x coordinate of the element
centroid is
xel =
=

x2
x2

x1
2

Since the variable of integration is y, we now have to


express dA and xel in terms of y (As can be seen from
Eq. 5, yel already is a function of y). The point (x, y)
on the curve satisfies
x1 = 3y

+ x1

x1

x2 = 4

(4)

x2

The y coordinate of the element is the same as the y


coordinate of the point on the curve:
yel = y

y2

= y2

(5)

x=4

(xel, yel)
(x1, y)

y2

xel =
dy

x
x1

=
=

x2

x1

1711

3y + 4) dy

(6)
x1 in Eq. 6 into the
(Eq. 3 repeated)
(7)

and for xel (Eq. 4) gives

x = 3y

x1

3y + 4

x1) dy

= ( y2

(x2, y)

(3y)

Substituting the expression for x2


equations for dA (Eq. 3) gives
dA = (x2

x2

y2)

x1 = (4

x2

x1
2
y2) + (3y)
2
y2 + 3y + 4
2

(Eq. 4 repeated)

(4

(8)

9.1 Centroids by Integration Example 6, page 3 of 3


6

Evaluate the integral in the denominator of the equation for


xc over the range from 0 to 1 (Use the integral function on
your calculator):
1

dA =

( y2

3y + 4) dy = 2.1667 m2

x=4

y2

x = 3y

(9)
1m

Evaluate the integral in the numerator of the equation for xc over the range
from 0 to 1 (Use the integral function on your calculator):
9
xel dA =

1
0

y2 + 3y + 4
] [ y2
2

3y + 4] dy = 5.2667 m3

Substitute the results given in Eqs. 9, 10, and 11


into the definitions for xc and yc:

(10)
xc =

dA

Evaluate the integral in the numerator of the equation for yc over


the range from 0 to 1 (Use the integral function on your calculator):
yc =
1

yel dA =

xel dA

yel dA
dA

y( y2

3y + 4) dy = 0.75 m3

(11)

1712

5.2667 = 2.43 m
= 2.1667

Ans.

0.75 = 0.346
= 2.1667

Ans.

9.1 Centroids by Integration Example 7, page 1 of 3


7. Locate the centroid of the plane area shown. Use a
differential element of thickness dx.
y
1 Definition of centroid coordinates
y = hb x

xc =

yc =

x
b
3

xel dA
dA
yel dA
dA

(1)

(2)

Locate the differential element so that it


extends from an arbitrary point (x, y)
on the curve to an opposite boundary.

where (xel, yel) are the coordinates of


the centroid of the differential area
element dA
y

Express the element area in terms of the


coordinates of a point (x, y) on the curve:

(x, y)
y = hb x

dA = height width (of rectangle)


or,
dA = y dx

(3)

By choosing an element of width dx, we


have also implicitly chosen x to be the
variable of integration.

dx
x

1713

9.1 Centroids by Integration Example 7, page 2 of 3


4

Express the coordinates of the element centroid in terms of the


coordinates of the point (x, y) on the curve. The x coordinate
of the element is the same as the x coordinate of the point on
the curve:
xel = x

(4)

y
y=

Since the centroid of the differential element is located in the


center of the element, the y coordinate of the element centroid
is
y
yel = 2

h
x
b

(x, y)
(5)
(xel, yel)
y

Since the variable of integration is x, we now have to express dA and


yel in terms of x (As can be seen from Eq. 4, xel already is a function
of x). The point (x, y) on the curve satisfies
h
y=
x
b

(6)

Substituting the expression for y in Eq. 6 into the equations for dA


(Eq. 3) and for yel (Eq. 5) gives
h
x) dx
b
h
yel =
x
2b

dA = (

x
dx

(7)
(8)

1714

y
2

9.1 Centroids by Integration Example 7, page 3 of 3


6

dA =

h x dx = bh
2
b

dx

(9)

Evaluate the integral in the numerator of the equation for


xc over the range from 0 to b:
b

xel dA =

Evaluate the integral in the denominator of the equation for


xc over the range from 0 to b:

2
x( hb x) dx = b 3h

x
b

(10)

Evaluate the integral in the numerator of the equation for


yc over the range from 0 to b:
b

yel dA =

2
h x)( h x) dx = bh
( 2b
6
b

(11)

1715

Substitute the results given in Eqs. 9, 10, and 11


into the definitions for xc and yc:

xc =

xel dA

yc =

yel dA

dA

dA

b2h/3
= 2b
3
bh/2

Ans.

bh2/6 h
= 3
bh/2

Ans.

9.1 Centroids by Integration Example 8, page 1 of 3


8. Locate the centroid of the plane area shown. Use a
differential element of thickness dy.
y
1

b
x = a[1 (

Definition of centroid coordinates

y 2
)]
b

xc =

xel dA

yc =

yel dA

dA

(1)

(2)

dA

Locate the differential element so that it


extends from an arbitrary point (x, y) on
the curve to an opposite boundary of the
crosshatched region.

y
(a, y)

where (xel, yel) are the coordinates of


the centroid of the differential area
element dA.

a
(x, y)
3

Express the element area in terms of the


coordinates of a point (x, y) on the curve:
dA = width

x = a[1 (

dy

y 2
)]
b

height (of rectangle)

or,
x
dA = (a

x) dy

(3)

x
a

By choosing an element of width dy, we have


also implicitly chosen y to be the variable of
integration.

1716

9.1 Centroids by Integration Example 8, page 2 of 3


4

Express the coordinates of the element centroid in terms of the


coordinates of the point (x, y) on the curve. Since the centroid
of the differential element is located in the center of the
element, the x coordinate of the element centroid is
xel =

(a

(xel, yel)
(a, y)

x)

2 +x
a+x
2

(4)
y

The y coordinate of the element is the same as the y coordinate


of the point on the curve:
yel = y

dy

(x, y)

x = a[1 (

y 2
)]
b

(5)

Since the variable of integration is y, we now have to express dA and


xel in terms of y (As can be seen from Eq. 5, yel already is a function
of y). The point (x, y) is on the curve so it satisfies
x = a[1 (

y 2
)]
b

(6)

Substituting the expression for x in Eq. 6 into the equation for dA


(Eq. 3) gives
dA = (a

x dy

= (a

a[1 (

y
= a( )2 dy
b

(Eq. 3 repeated)
y 2
) ] dy
b
(7)

1717

(a x)
2
a

x
x

a
5

Substituting the expression for x in Eq. 6 into


the equation for xel (Eq. 4) gives
a+x
2
a + a[1 (y/b)2]
=
2
a[2 (y/b)2]
=
2

xel =

(Eq. 4 repeated)

(8)

9.1 Centroids by Integration Example 8, page 3 of 3


6 Evaluate the integral in the denominator of the equation for x
c over the range from 0 to b:

dA =

b
0

a(

y 2
ab
) dy = 3
b

(9)

dy

Evaluate the integral in the numerator of the equation for xc


over the range from 0 to b:

xel dA =

b
0

[2a

y
(a/b2) y2]
][a( )2] dy
b
2
x

= 7 a2b
30
8

(10)

Evaluate the integral in the numerator of the equation for yc over


the range from 0 to b:

yel dA =

y
ab2
y[a b )2] dy = 4

Substitute the results given in Eqs. 9, 10, and 11


into the definitions for xc and yc:
xc =
yc =

(11)

1718

xel dA
dA
yel dA
dA

7a
(7/30)a2b
=
10
ab/3

Ans.

ab2/4
3b
=
ab/3
4

Ans.

9.1 Centroids by Integration Example 9, page 1 of 3


9. A sign is made of 0.5 in. thick steel plate in the shape shown.
Determine the reactions at supports B and C.

Free-body diagram of plate

y
Specific weight of steel = 490 lb/ft3

By

B
x = 50 + (10) sin y
24

Bx

The weight acts through the


centroid of the plate.

x = 50 + (10) sin

y
24

72 in.
72 in.

C
50 in.
3

Cx

Equilibrium equations for the plate

Fx = 0: Bx + Cx = 0
Fy = 0: By

Weight, W

xc

MB = 0: (72 in.)Cx

4
(1)

W=0

(2)
xcW = 0

Calculate the weight of the plate


by calculating the area. Also
calculate the distance xc to the
center of gravity.

(3)

Definition of centroid coordinate

xc =

xel dA
dA

(1)

where xel is the coordinate of the


differential element dA.

1719

9.1 Centroids by Integration Example 9, page 2 of 3


6

Locate the differential element so that it


extends from an arbitrary point (x, y)
on the curve to an opposite boundary.
y
y
(xel, yel)
x = 50 + (10) sin 24
dy

Express the element area in terms of the coordinates of a


point (x, y) on the curve:
dA = width height (of rectangle)
or,

(x, y)

dA = x dy

(4)

By choosing an element of width dy, we have also implicitly


chosen y to be the variable of integration.
y

9
x
2

x
x

dA = x dy
= [50 + (10) sin

Express the coordinate of the x-element centroid


in terms of the coordinates of the point (x, y) on
the curve. Since the centroid of the differential
element is located in the center of the element, the
x coordinate of the element centroid is
xel = x2

Since the variable of integration is y, we now have to express


dA and xel in terms of y (As can be seen from the equation of
the curve, x already is a function of y). Therefore

y
dy
24

xel = x2
50 + (10) sin( y/24)
=
2
y
= 25 + 5 sin
24

(5)

1720

(7)

(8)

9.1 Centroids by Integration Example 9, page 3 of 3


10 Evaluate the integral in the denominator of the equation
for xc over the range from 0 to 72 (Use the integral
function on your calculator):
dA =
(9)
y

72
0

[50 + (10) sin

y
dy = 3753 in2
24
12 Substitute the results given in Eqs. 9 and 10 into the
definition for xc:
xc =

72 in.

xel dA
dA

99,439
3753 = 26.5 in.

13 Thus the weight is

x = 50 + (10) sin y
24

W = area

thickness

specific weight

1 ft )3
= (3753 in2) (0.5 in.) (490 lb/ft3) ( 12
in.
x

= 532.1 lb

11 Evaluate the integral in the numerator of the equation for


xc over the range from 0 to 72 (Use the integral function
on your calculator):

Substituting xc = 26.5 in. and W = 532.1 lb in the


equilibrium equations, Eqs. 1, 2, and 3, and solving gives

72

xel dA =

[25 + 5 sin y ][50 + (10) sin y ]dy


24
24

= 99,439 in3

(10)

1721

Bx = 196 lb

Ans.

By = 532 lb

Ans.

Cx = 196 lb

Ans.

9.1 Centroids by Integration Example 10, page 1 of 3


10. Locate the centroid of the wire shown.

y
1

Definition of centroid coordinates

3m
xc =
y = 2x2

yc =

xel dL
dL
yel dL

y = 2x2

(xel, yel)

(1)

(2)

dL
18 m

dL
y

where (xel, yel) are the coordinates of


the centroid of the differential length
element dL.

x
x

3
x

Since dy/dx is slightly easier to compute than


dx/dy, express dL in terms of dy/dx and dx:
dL =

[(dx)2 + (dy)2]
2

dL
dx

2
2
[1 + ( dy
dx ) ] (dx)

2
[1 + ( dy
dx ) ] dx

dy

(5)

By expressing the length dL in terms of dx, we


have also implicitly chosen x to be the variable
of integration.

1722

Locate the differential element at an


arbitrary point (x, y) on the curve and
express the centroidal coordinates of
the element in terms of x and y.
xel = x

(3)

yel = y

(4)

9.1 Centroids by Integration Example 10, page 2 of 3


4

Since the variable of integration is x, we now have to express


dL and yel in terms of x (As shown in Eq. 3, xel already is
expressed as a function of x, since xel = x). The point (x, y) on
the curve satisfies:
y = 2x2

(6)

y = 2x2

(x, y)

Thus
dy = 4x
dx

(7)

dL

Substituting these expressions for y and dy/dx into the


equations for yel (Eq. 4) and dL (Eq. 5) gives
yel = y

= 2x2
dL =
=

3m

(8)

2
[1 + ( dy
dx ) ] dx

[1 + (4x)2] dx

(9)

Evaluate the integral in the denominator of the equation for


xc over the range from 0 to 3:
3

dL =

1723

[1 + (4x)2] dx = 18.46 m

(10)

9.1 Centroids by Integration Example 10, page 3 of 3


6

Evaluate the integral in the numerator of the equation for xc over the range
from 0 to 3:

xel dL =

3
0

(11)

Evaluate the integral in the numerator of the equation for yc over


the range from 0 to 3:
3

yel dL =

x [1 + (4x)2] dx = 36.36 m2

(2x2) [1 + (4x)2] dx = 163.11 m2

(12)

Substitute the results given in Eqs. 10, 11, and 12 into the definitions for xc
and yc:

xc =

yc =

xel dL
dL
yel dL
dL

36.36 = 1.97 m
= 18.46

Ans.

= 163.11
18.46 = 8.84 m

Ans.

1724

9.1 Centroids by Integration Example 11, page 1 of 3


11. Locate the centroid of the wire shown.
1 Definition of centroid coordinates

y
x = 300[1

y 4
(
)]
200

xc =

yc =

xel dL

(1)

dL
yel dL

(2)

dL
200 mm

where (xel, yel) are the coordinates of the


centroid of the differential length element dL.

300 mm
dL
x = 300[1

(3)

yel = y

(4)

y 4
)]
200

(xel, yel)

Locate the differential element at an arbitrary


point (x, y) on the curve and express the
centroidal coordinates of the element in terms
of x and y.
xel = x

x
x

1725

9.1 Centroids by Integration Example 11, page 2 of 3


y

4
dL

x = 300[1

y 4
)]
200

Since the variable of integration is y, we now have to


express dL and xel in terms of y (As shown in Eq. 4, yel
already is expressed as a function of y, since yel = y). The
coordinates of the point (x, y) on the curve satisfy
x = 300[1

(xel, yel)

y 4
)]
200

(6)

Thus
y

dx
= 300[
dy
= 7.5

x
x

4y3
2004

10-7 y3

(7)

Substituting these expressions for x and dx/dy into the


equations for xel (Eq. 3) and dL (Eq. 5) gives

Since dx/dy is slightly easier to compute than dy/dx, express


dL in terms of dx/dy and dy:
dL
dy
dL = [(dx)2 + (dy)2]
dx
dx 2
2
= [(
) + 1] (dy)
dy
= [( dx )2 + 1] dy
(5)
dy
By expressing the length dL in terms of dy, we have also
implicitly chosen y to be the variable of integration.

1726

xel = x
= 300[1
dL =
=

y 4
)]
200

(8)

[( dx )2 + 1] dy
dy
[(

.5

10-7 y3)2 + 1] dy

(9)

9.1 Centroids by Integration Example 11, page 3 of 3


5

Evaluate the integral in the denominator of the equation for xc over the
range from 0 to 200 (Use the integral function on your calculator):
200

dL =

[(

.5

y
dL
x = 300[1

10-7 y3)2 + 1] dy = 407.4 mm

(10)

y 4
)]
200

200 mm
6

200

xel dL =

y
300[1 ( 200 )4] [(

= 75 209.6 m2

.5

10-7 y3)2 + 1] dy

(11)
8

(x, y)

Evaluate the integral in the numerator of the equation for xc over the
range from 0 to 200 (Use the integral function on your calculator):

Evaluate the integral in the numerator of the equation for yc over


the range from 0 to 200 (Use the integral function on your
calculator):
200

yel dL =

y [(

.5

10-7 y3)2 + 1] dy = 54 861.7 m2 (12)

1727

Substitute the results given in Eqs. 10, 11, and 12 into


the definitions for xc and yc:

xc =

yc =

xel dL
dL
yel dL
dL

209.6
= 75
407.4 = 184.6 mm

Ans.

861.7 = 134.7 mm
= 54407.4

Ans.

9.1 Centroids by Integration Example 12, page 1 of 3


12. The rod is bent into the shape of a circular arc.
Determine the reactions at the support A.
4

0.2 lb/ft

Equations of equilibrium for the rod.


+

Fx = 0: Ax = 0

(1)

3 ft

Free-body diagram of rod

y
xc

If L represents the length


of the rod, the weight W
is (0.2 lb/ft)L.

3 ft

20

(0.2)L = 0

x
6

(radius)

20 )( /180)

3 ft
(4)

Substituting L = 8.378 ft into Eq. 2 and solving


gives
Ay = 1.68 lb

1728

(3)

The length L of the rod can be calculated without


using an integral:

= 8.378 ft

3 The weight of the rod acts


through the centroid (center of
gravity), located by xc.

(2)

(0.2 lb/ft)(L)(xc) = 0

= (180

MA
Ay

MA = 0: MA

Ans.

L = (angle in radians)

0.2 lb/ft
Ax

+
1

Fy = 0: Ay

Therefore Ax = 0

20

Ans.

9.1 Centroids by Integration Example 12, page 2 of 3


7

To solve for the moment MA in Eq. 3 we have to


calculate the horizontal coordinate of the centroid:
xel dL

xc =

(5)

dL

Express the element length in terms of the polar


coordinate angle
dL = (3 ft) d

where xel is the horizontal coordinate of the centroid


of the differential length element dL.

(6)

10 Express the horizontal coordinate of the element in


terms of .
8

y
dL
(xel, yel)

Locate the differential


element at an arbitrary point
on the curve.

xel = 3 ft + (3 ft) cos

(7)

11 We do not need to evaluate the integral in the


denominator of Eq. 5 for xc, since we already know
that

d
3 ft
20

dL = L = 8.378 ft
x
by Eq. 4.

(3 ft) cos
3 ft
xe1

1729

(8)

9.1 Centroids by Integration Example 12, page 3 of 3


12 To evaluate the integral in the numerator in the
expression for xc, we note the limits of
integration.

y
radians

13

(3 + 3 cos )(3)d = 22.055 ft2

(9)

20
180

Substitute the results given in Eq. 8 and 9 into the


definition of xC:

x
20

xel dL =

180
xc =

xel dL
dL

= 22.055
8.378 = 2.632 ft

(10)

Substituting this result in Eq. 3 and solving for MA gives


MA = 4.41 lbft

1730

Ans.

9.1 Centroids by Integration Example 13, page 1 of 3


13. a) Locate the centroid of the Gateway Arch in St.
Louis, Missouri, USA. b) During the pre-dawn hours of
September 14, 1992, John C. Vincent of New Orleans,
Louisiana, USA, climbed up the outside of the Arch to the
top by using suction cups and then parachuted to the
ground. Estimate the length of his climb.

Definition of centroid coordinates

(1)

dL
yel dL

yc =

Approximate equation of centerline:


y = 639.9 ft

xel dL

xc =

(2)

dL

(68.78 ft) cosh[(0.01003 ft-1)x]

where (xel, yel) are the coordinates of the centroid of the


differential length element dL.

y
Because of symmetry about the y-axis, the centroid must lie
on the y-axis, that is,
xc = 0
2

To avoid cluttering the equations with so many digits, define

625 ft

x
299 ft

Ans.

a = 639.9 ft

(3)

b = 68.78 ft

(4)

c = 0.01003 ft-1

(5)

Then the equation of the arch becomes

299 ft

y = 639.9 ft
=a

1731

(68.78 ft) cosh[(0.01003 ft-1)x]

b cosh(cx)

(6)

9.1 Centroids by Integration Example 13, page 2 of 3


y

3
dL

Locate the differential element at a general point (x, y)


on the curve and express the y-centroidal coordinate of
the element in terms of the point on the curve.
yel = y

y=a

b cosh (cx)

(7)

Since the variable of integration is x, we now have to


express dL and yel in terms of x. The point (x, y) on the
curve satisfies
y=a

b cosh (cx)

(9)

x
Thus
4 Since dy/dx is easier to compute than dx/dy, express dL in
terms of dy/dx and dx:
dL
dy
dL = [(dx)2 + (dy)2]
dx
= [1+(

dy 2
) ] (dx)2
dx

dy
= bc sinh (cx)
dx

(10)

dy
Substituting these expressions for y and
into Eq. 7 for
dx
yel and Eq. 8 for dL gives
yel = y

dy 2
) ] dx
(8)
dx
By expressing the length dL in terms of dx, we have also
implicitly chosen x to be the variable of integration.

=a

= [1 + (

dL =

1732

b cosh (cx)
[1+(

(11)

dy 2
) ] dx
dx

[1 + ( bc sinh (cx))2] dx

(12)

9.1 Centroids by Integration Example 13, page 3 of 3


y=a

b cosh (cx)

Evaluate the integral in the denominator of the equation for yc over the
range from 299 ft to +299 ft (Use the integral function on a calculator):
299

dL =
dL

-299

[1 + ( bc sinh (cx))2] dx = 1,476 ft

(13)

Similarly evaluating the numerator of the equation for yc gives


299

yel dL =

299 ft

b cosh (cx)) [1 + ( bc sinh (cx))2] dx

-299

= 439,685 ft2

x
299 ft

(a

(14)

Substitute the results given in Eqs. 13 and 14 into the definitions for yc.
yc =

yel dL
dL

439,685
= 298 ft
1,476

Ans.

Since Mr. Vincent only climbed half of the total arc length, we must
divide the length given in Eq. 13 by 2:
Mr. Vincent's climb =

1,476
= 738 ft
2

Ans.

(Actually he climbed a little more than 738 ft, since he climbed on the
outside surface, not on the centerline of the cross section.)

1733

9.1 Centroids by Integration Example 14, page 1 of 4


14. Locate the centroid of the cone shown.
y
Radius = 2 m
1

Definition of centroid coordinates

xc =
3m
yc =

xel dV
dV
yel dV

(1)

(2)

dV
O
z

x
zc =

zel dV
dV

(3)

where (xel, yel) are the coordinates of the centroid of


the differential element dV.
By symmetry, the centroid must lie on the y-axis, so
xc = 0

Ans.

zc = 0

Ans.

The centroidal coordinate yc remains to be calculated.

1734

9.1 Centroids by Integration Example 14, page 2 of 4


y

2 Use a differential element in the form of a


disk of thickness dy. The volume of the
disk is

3
B

dV = (area of circular base)


thickness of disk
= r2 dy

P(x, y, 0)

dy

(4)

By choosing an element of width dy, we


have also implicitly chosen y to be the
variable of integration.

The boundary of the disk


intersects the xy plane at an
arbitrary point P(x, y, 0).

r
y

(0, yel, 0)
O

4 The y coordinate of the


element centroid equals the
y coordinate of the point P
in the xy plane.
yel = y

1735

(5)

9.1 Centroids by Integration Example 14, page 3 of 4


5 The distance from the y axis to
the boundary of the disk is x, so
r=x

6
2m

(6)

Since the variable of integration is y, we now have to


express dV and x in terms of y.
By similar triangles OCP and OBA, we have

y
x
C
B

Thus
y

P (x, y, 0)

3m

2m

2y
x= 3

dy

Thus the volume of the differential element in Eq. 4


becomes

(7)

2y
Substituting x = 3 into Eq. 6 gives
2y
r= 3
(8)

3m

y
x
2 = 3

dV = r2 dy
z

=
7

Evaluate the integral in the denominator of the


equation for yc over the range from 0 to 3:
3

dV =

2y 2
) dy = 4 m3
3

(10)

1736

2y 2
) dy
3

(9)

9.1 Centroids by Integration Example 14, page 4 of 4


8

Evaluate the integral in the numerator of the equation for yc:


3

yel dV =

y[

2y 2
) ] dy = 9 m4
3

(11)

Using the results given by Eqs. 10 and 11 in the definition of yc yields


yc =

yel dV
dV

9
= 2.25 m
4

Ans.

1737

9.1 Centroids by Integration Example 15, page 1 of 4


15. Locate the centroid of the volume shown.
y
One-eighth of a
sphere of radius "a"

Definition of centroidal coordinates

xc =
a

yc =

xel dV
dV
yel dV

(1)

(2)

dV
x

zc =

yel dV
dV

(3)

where (xel, yel, zel) are the coordinates of the centroid


of the differential volume-element dV.

Because of symmetry, xc = yc = zc, so we only have


to compute one centroidal distance. Let's arbitrarily
choose to compute zc.

1738

9.1 Centroids by Integration Example 15, page 2 of 4


2

The differential element is one-quarter of a disk of thickness dz.


The volume of the disk is
dV = (one-fourth of the area of a circle)

thickness of disk

2
= r dz
4

(4)

Note that by choosing an element of width dz, we have also


implicitly chosen z to be the variable of integration.
4
y

The z coordinate of the element


centroid equals the z-coordinate of
the point P in the yz-plane.
zel = z

z
P(0, y, z)

The boundary of the quarter disk


intersects the yz plane at an
arbitrary point P(0, y, z).

(xel, yel, zel)

dz
z

1739

(5)

9.1 Centroids by Integration Example 15, page 3 of 4


5

The distance from the z axis to the


boundary of the disk is y so
r=y

(6)

Since the variable of integration is z, we now have to express y


and dV in terms of z. Because point P lies on a circle of radius
"a" in the yz plane, y and z must satisfy the equation of a
circle
y2 + z2 = a2

Solving this for y gives


z

a2

y=
P (0, y, z)

z2

Substituting y =

a2

(7)
z2 into Eq. 6 gives

r=y
a
y

a2

=
x

z2

(8)

Thus the volume of the differential element in Eq. 4 becomes


dV =

dz
z

1740

r2
dz
4

[ a2 z2 ]2
dz
4

a2 z2)
dz
4

(9)

9.1 Centroids by Integration Example 15, page 4 of 4


7 Evaluate the integral in the denominator of the
equation for yc over the range from 0 to a:
a2 z2)
a3
dz =
4
6

dV =

(10)

8 Evaluate the integral in the numerator of the equation


for zc over the range from 0 to a:
a

zel dV =

z[

4
a2 z2)
] dz = 16a
4

(11)
a
dz

9 Using the results given by Eqs. 10 and 11 in the


definition of zc yields

zc =

zel dV
dV

a4/16
= 3a
8
a3/6

Ans.

Thus from symmetry,


xc = yc = 3a
8

Ans.

1741

9.1 Centroids by Integration Example 16, page 1 of 3


16. Determine the x coordinate of the centroid of the solid
shown. The solid consists of the portion of the solid of
revolution bounded by the xz and yz planes.

Definition of centroidal coordinates

xc =

y
a

xel dV

(1)

dV

where xel is the coordinate of the centroid


of the differential volume-element dV.

x
b
z

x = a[1

( bz )2]

dx
r

(This curve is rotated about the


x-axis to generate the solid.)

The differential element is one-half of a disk of thickness dx.


The volume of the disk is
dV = (one-half of the area of a circle)
=

r2
dx
2

x = a[1

P(x, 0, z)

( bz )2]

thickness of disk
(2)
3

By choosing an element of width dx, we have also implicitly


chosen x to be the variable of integration.

1742

The boundary of the half disk intersects the xz


plane at an arbitrary point P(x, 0, z) on the
generating curve.

9.1 Centroids by Integration Example 16, page 2 of 3


4

The x coordinate of the element centroid


equals the x coordinate of the point P in
the xy plane.
xel = x

(3)

Since the variable of integration is x, we now have to


express z and dV in terms of x. Because point P lies
on the generating curve, x and z must satisfy the
equation of that curve
( bz )2]

x = a[1

y
(xel, yel, 0)

Solving this for z gives


z=b 1

( xa )

r
x
z
z

Substituting z = b [1

(5)
( ax )] into Eq. 4 gives

r=z

=b 1
P (x, 0, z)

( xa )

(6)

Thus the volume of the differential element in Eq. 2


becomes
5

r2
dV = 2 dx
b 1 (x/a) ]2
=
dx
2

The distance from the x-axis to the


boundary of the half-disk is z so
r=z

(4)

1743

b2

x/a)
2

dx

(7)

9.1 Centroids by Integration Example 16, page 3 of 3


y
7

Evaluate the integral in the denominator of the equation for xc


over the range from 0 to a:
a

P(x, 0, z)

dV =

b2

x/a)
2

ab2
4

dx =

(8)

x
dx
a

Evaluate the integral in the numerator of the equation for xc over


the range from 0 to a:
a
xel dV =

x[

b2

x/a)
2

] dx =

a2b2
12

(9)

Using the results given by Eqs. 8 and 9 in the definition of xc


yields
xc =

1744

xel dV
dV

a2b2/12
=
= a3
2
ab /4

Ans.

9.1 Centroids by Integration Example 17, page 1 of 4


17. Locate the centroid of the pyramid shown.
1

Definition of centroid coordinates

xc =

yc =

xel dV

(1)

dV
yel dV

(2)

dV

zc =
x

zel dV

(3)

dV

b
a
z

where ( xel, yel) are the coordinates of the


centroid of the differential volume-element
dV.

By symmetry, the centroid must lie on the


y-axis, so
xc = 0

Ans.

zc = 0

Ans.

yc remains to be calculated.

1745

9.1 Centroids by Integration Example 17, page 2 of 4


Because of symmetry, we need consider
only one-fourth of the pyramid.

The differential element is a rectangular box of


thickness dy. The volume of the box is

dVo = (area of base)

= (xz) dy

(4)

By choosing an element of width dy, we have also


implicitly chosen y to be the variable of integration.

(xel, yel, zel)


h

P(x, y, 0)
Q(0, y, z)

dy

C
x

b
B
z

thickness

a
5

The boundary of the differential element


intersects the xy and xz planes at arbitrary points
P and Q on the sloping lines in those planes.

The y coordinate of the element centroid equals


the y coordinate of the point P in the xy plane.
yel = y

(5)

1746

9.1 Centroids by Integration Example 17, page 3 of 4


6

Because the variable of integration is y, we now have


to dexpress dV in terms of y. Because point P lies on
the line AC in the xy plane, similar triangles can be
used to derive an equation relating x and y:
x= h y
a
h

Because point Q lies on the line AB in the yz plane,


similar triangles can be used to derive an equation
relating z and y:
z =h y
h
b
Solving for x gives

Solving for x gives


z = b(1

y
)
h

x = a(1

(6)

y
)
h

(7)
y

y
A

h
h

y
h

P(x, y, 0)

Q(0, y, z)

y
C
O

B
z

1747

9.1 Centroids by Integration Example 17, page 4 of 4


Substituting Eqs. 6 and 7 in Eq. 4 and multiplying by
4 to get the volume of the whole pyramid (not just one
fourth) gives the volume element in terms of y.

dV = 4(dVo)

Evaluate the integral in the denominator of the equation for yc


over the range from 0 to h:

= 4(xz)dy
y
)] [b(1
h
y 2
) dy
h

= 4[a(1
= 4ab(1

y
] dy
h

dV = 4

h
ab(1
0

y 2
4abh
)
dy
=
3
h

(9)

(8)
Evaluate the integral in the numerator of the equation for yc:

y
yel dV = 4

h
y[ab(1
0

y 2
) ]dy =
h

abh2
3

(10)

Using the results given by Eqs. 9 and 10 in the definition of yc


yields

yc =
x

1748

yel dV
dV

abh2/3
= 4abh/3 = h4

Ans.

9.2 Centroids: Method of Composite Parts

1749

9.2 Centroids: Method of Composite Parts Procedures and Strategies, page 1 of 1


Procedures and Strategies for Solving Problems Involving
Centroids: Method of Composite Parts
Geometric Problems
1. Divide the given region into a collection of simple shapes that
are listed in a table of geometric properties of lines, areas, or
volumes. Sometimes it is convenient to subtract one region from
another, when dividing the given region. For example, if a hole is
present in a rectangle, then a circular region should be subtracted
from a rectangular region.
2. Set up a coordinate system in which all centroidal coordinates are
expressed. Note that some coordinates may be negative.
3. Set up a table, and evaluate the formula (for area A)
Xc =

xcA
A

Area A is replaced by L or V for lengths or volumes.

Weighted Averages
Decide what are the weights and what is to be averaged. The
weights alone go in the denominator of the weighted average
formula.
Xc =

xcW
W

1750

9.2 Centroids: Method of Composite Parts Problem Statement for Example 1


1. Locate the center of mass of the system of particles.
y

A 5 kg

D 10 kg

C 4 kg

B 2 kg

x
5m

3m

2m

4m

1751

9.2 Centroids: Method of Composite Parts Problem Statement for Example 2


2. Determine where a 9-lb weight should be placed on
the y axis so that the center of gravity of the system
lies at the origin O of the coordinate system.
y
E
9 lb

A
2 lb
2 ft

B
5 lb

3 ft
O

C
7 lb

1 ft
4 ft

D
3 lb

1752

9.2 Centroids: Method of Composite Parts Problem Statement for Example 3


3. Determine the mass mo of the particle located at point A
so that the center of mass of the system lies at x = 2 m.
y

D 6 kg

C 4 kg

4m

2m

1m

12 kg

A mo

3m

1753

9.2 Centroids: Method of Composite Parts Problem Statement for Example 4


4. Locate the center of mass of the system of particles.
y
60 m
A
65 m

5 kg
3 kg
80 m

B
70 m
x

40 m

4 kg
C
100 m

1754

9.2 Centroids: Method of Composite Parts Problem Statement for Example 5


5. Locate the centroid of the wire.
y

140 mm
B

80 mm

x
30
100 mm

1755

9.2 Centroids: Method of Composite Parts Problem Statement for Example 6


6. The wire ABCD can rotate freely about the pin support at B. Determine
the length b for which the wire will remain horizontal.
y
Radius of arc = 4 in.
b
A

6 in
B

45
C

45

1756

9.2 Centroids: Method of Composite Parts Problem Statement for Example 7


7. Locate the centroid of the wire.
y
BC is a circular arc
lying in the xy plane.

4m
C

3m
B

D
x

A
z

2m

1757

9.2 Centroids: Method of Composite Parts Problem Statement for Example 8


8. Locate the centroid of the beam cross section shown.
y

70 mm

70 mm
25 mm

80 mm
x

20 mm
40 mm

1758

9.2 Centroids: Method of Composite Parts Problem Statement for Example 9


9. Locate the centroid of the crosshatched region.
y

2m

4m

1759

9.2 Centroids: Method of Composite Parts Problem Statement for Example 10


10. Locate the centroid of the plate with hole.
y
400 mm

250 mm

60 mm
160 mm
x
500 mm

1760

9.2 Centroids: Method of Composite Parts Problem Statement for Example 11


11. Determine distance h so that the centroid of the crosshatched
region lies at y = h.
y
2 in. 2 in.
4 in.

h
x
3 in.

3 in.

1761

9.2 Centroids: Method of Composite Parts Problem Statement for Example 12


12. Locate the centroid of the crosshatched region.
y
21 mm

60 mm

x
78 mm

1762

9.2 Centroids: Method of Composite Parts Problem Statement for Example 13


13. Determine the x coordinate of the
centroid of the crosshatched region.
y
D

15 in.

E
O

9 in.

12 in.

1763

9.2 Centroids: Method of Composite Parts Problem Statement for Example 14


14. Four rolled-steel angles are welded to a plate to form a
girder with the cross section shown. Locate the centroid.
y
15 mm

19.0 mm
Area = 932 mm2
C

700 mm

25.1 mm
C

Area = 772 mm2

1764

9.2 Centroids: Method of Composite Parts Problem Statement for Example 15


15. Two rolled-steel angles and a plate are welded
together to form a beam with the cross section shown.
Locate the centroid.
Area = 2430 mm2

30.2 mm
C
15 mm
x
120 mm

120 mm

1765

9.2 Centroids: Method of Composite Parts Problem Statement for Example 16


16. The door of an autoclave unit is composed of a curved surface
ABCD bounded by two angular sectors on the ends. The door is made
of sheet metal of uniform thickness. Locate the centroid of the door.
Neglect the irregularities in geometry near the holes.
D

y
28 in.

B
x

C
60
34 in

A
z

1766

9.2 Centroids: Method of Composite Parts Problem Statement for Example 17


17. A bird bath has been made by cutting a hemispherical
cavity in the top of a wooden block. The designer is
concerned that the block may tip over. As part of a study of
the stability of the structure, you are asked to locate the center
of gravity.
y
9 in.
9 in.
r = 8 in.
10 in.
10 in.
24 in.

1767

9.2 Centroids: Method of Composite Parts Problem Statement for Example 18


18. Ice cream was loosely packed into a cone, and
then a solid ball of ice cream of density 920 kg/m3
was placed on top. Estimate the density of the
loosely packed ice cream, if the center of mass of the
entire ice-cream cone is 120 mm above the tip of the
cone.
y
Radius of ball = 30 mm

130 mm

x
z

1768

9.2 Centroids: Method of Composite Parts Problem Statement for Example 19


19. Locate the center of mass of the buoy shown. The
buoy is of uniform density.
y

Hemisphere

150 mm
350 mm
250 mm

500 mm
Hemisphere

x
z

1769

9.2 Centroids: Method of Composite Parts Problem Statement for Example 20


20. Locate the centroid of the bracket shown.
y

60 mm
70 mm
50 mm

Circular arc
60

50 mm

x
40 mm

8 mm

10 mm
10 mm

1770

9.2 Centroids: Method of Composite Parts Problem Statement for Example 21


21. Determine the greatest depth, h, of the circular hole of
diameter 180 mm for which the cube will remain in the
position shown. The cube is made of a material of uniform
density and is supported by a hinge A along the edge
perpendicular to the plane of the figure.
10 mm

180 mm
h
10 mm
B
40
A

1771

9.2 Method of Composite Parts Example 1, page 1 of 1


1. Locate the center of mass of the system of particles.
y

A 5 kg

D 10 kg

C 4 kg

B 2 kg

x
5m

3m

2m

4m
3

xc is a coordinate, not a distance,


so it can have a negative value.

1 Definition of center of mass


Xc =

xcm
m

Set up a table.

where xc is the x coordinate of mass m.

4 The summation sign means add


all the numbers in the column.

Particle
A
B
C
D

1772

Xc =

m (kg)
5
2
4
10
m = 21

xc (m)
8
3
2
6

xcm 22
= 21 = 1.048 m
m

xcm (kg m)
40
6
8
60
xcm = 22

Ans.

9.2 Method of Composite Parts Example 2, page 1 of 2


2. Determine where a 9-lb weight should be placed on
the y axis so that the center of gravity of the system
lies at the origin O of the coordinate system.
y
1

Place the 9-lb weight at an unknown distance y


from the origin.

9 lb
2
A
2 lb
2 ft

where yc is the coordinate of weight W.


B

3 Set up a table.

5 lb

Particle
A
B
C
D
E

3 ft
O

x
x

C
7 lb

1 ft

Definition of center of gravity


ycW
Yc =
(1)
W

W (lb)
2
5
7
3
9
W = 26

yc (ft)
5
3
1
y

ycW (lb ft)


10
15
7
15
9y
ycW = 3 + 9y

4 ft
4
D
3 lb

Substitute the results from the table into the definition of Yc.
Yc =

ycW 3 + 9y
= 26
W

1773

(2)

9.2 Method of Composite Parts Example 2, page 2 of 2


5 If the center of gravity is to be at the origin, then
Yc = 0
6

Eq. 2 becomes
Yc =

3 + 9y
26

(Eq. 2 repeated)

0
and solving for y gives
y = 0.333 ft

Ans.

1774

9.2 Method of Composite Parts Example 3, page 1 of 1


3. Determine the mass mo of the particle located at point A so
that the center of mass of the system lies at x = 2 m.
y

D 6 kg

C 4 kg

4m
1

2m

12 kg

1m

Definition of center of mass


xcm
Xc =
m

A mo

3m

3
(1)

xcm
=
m

32 + 4mo
22 + mo

(2)

Since the center of mass is to lie at x = 2 m, substitute Xc


= 2 m into Eq. 2 to get

Set up a table.
Particle
A
B
C
D

Substitute the results from the table into Eq. 1:


Xc =

where xc is the coordinate of mass m.


2

2=

m (kg)
xc (m) xcm (kg m)
4
mo
4mo
12
1
12
4
2
8
36
6
6
xcm = 32 + 4mo
m = 22 + mo

32 + 4mo
22 + mo

Solving gives
mo = 2 kg
Since mass cannot be negative, this result shows that there
is no particle which can be placed at point A to make the
center of mass lie at 2 m.
Ans.

1775

9.2 Method of Composite Parts Example 4, page 1 of 1


4. Locate the center of mass of the system of particles.
y
60 m
A
65 m

5 kg
3 kg

80 m

70 m
x

40 m

4 kg
C
100 m

Definition of center of mass

Set up a table.
Particle
A
B
C

xcm
m
ycm
Yc =
m
Xc =

where (xc, yc) are the coordinates of mass m

m (kg)
5
3
4
m = 12

xc (m)
80
60
100

Xc =

xcm
=
m

Yc =

ycm 375
= 12 = 31.2 m
m

1776

yc (m)
65
70
40

620 = 51.7 m
12

xcm (kg m)
400
180
400
xc m = 620

Ans.

Ans.

ycm (kg m)
325
210
160
ycm = 375

9.2 Method of Composite Parts Example 5, page 1 of 3


5. Locate the centroid of the wire.
y

140 mm

80 mm

x
30
100 mm

Definition of centroidal coordinates


Xc =
Yc =

xcL
L
ycL
L

(1)
(2)

where (xc, yc) are the coordinates of the


centroid of line segment L.

1777

9.2 Method of Composite Parts Example 5, page 2 of 3


2

Locate the centroids of the individual segments.


y
140 mm = 70 mm
2
A

80 mm = 40 mm
2

centroid of AB: ( 70, 80)

80 mm = 40 mm
2

centroid of BC: (0, 40)

50 cos 30 = 43.30 mm
C

x
50 sin 30 = 25 mm

30
100 mm = 50 mm
2

D
centroid of CD: (43.30, 25)

Set up a table.
Segment
AB
BC
CD

L ( mm ) xc ( mm ) yc ( mm ) xcL ( mm2 )
140
9800
70
80
0
80
40
0
100
43.30 25
4330
L = 320
xcL = 5470

ycL ( mm2 )
11200
3200
2500
ycL = 11900

1778

9.2 Method of Composite Parts Example 5, page 3 of 3


4

Xc =

xcL
5470
= 320 = 17.1 mm
L

Ans.

Yc =

ycL 11 900
=
= 37.2 mm
320
L

Ans.

1779

9.2 Method of Composite Parts Example 6, page 1 of 3


6. The wire ABCD can rotate freely about the pin support at B. Determine
the length b for which the wire will remain horizontal.
y
Radius of arc = 4 in.
b
6 in
45
A

45

Free body diagram

B
A

Bx

Equilibrium equation
MB = WXc = 0

(1)

Therefore
Xc = 0

(2)

By
Xc

So the length L must be chosen such that the centroidal


coordinate of the entire wire satisfies Eq. 2.

W, weight of entire wire acts through the center of gravity


(a distance Xc from the origin of coordinates at B)
3

Definition of centroid
Xc =

xcL
L

(3)

where xc is the centroidal coordinate of line segment L.

1780

9.2 Method of Composite Parts Example 6, page 2 of 3


4

Locate the centroids of the individual segments.

Arc length = angle

radius
Angle = 45 = 4 radians

y
= 2(

Centroid of arc
Centroid of AB

6 in.
B

A
b
2

5 in

(4 in)

= 6.283 in

C
Lc

b
2

5 in

A table of geometric properties of planar curves gives


the information shown below.

Centroid of BD
Centroid Location

Length = 2 r

r
C
7

In our example, r = 4 in. and


Lc =

r sin

(4 in.) sin
=

= 45 = 4 rad, so

= 3.601 in.

r sin

Thus the coordinate of the centroid of the arc is

Circular arc segment

xc = 6 in. + 3.601 in. = 9.601 in.

1781

9.2 Method of Composite Parts Example 6, page 3 of 3


8

Set up a table.
xc (in.)

xcL ( in2 )

b/2
b
(6 + 4)
5
6.283 9.601
L = 16.283 + b

b2/2
50
60.323

L ( in.)

Segment
AB
BD
Arc

xc L = 110.323

b2/2

Substitute the results from the table into the definition of the
centroidal distance Xc and set Xc = 0:
Xc =

xcL 110.323 b2/2


=
L
16.283 + b

or
0=

110.323 b2/2
16.283 + b

Solving for b gives


b = 14.85 in

Ans.

1782

9.2 Method of Composite Parts Example 7, page 1 of 3


7. Locate the centroid of the wire.
y
BC is a circular arc
lying in the xy plane.

4m
C

3m
B

D
x

A
z

2m

Centroid of AB as viewed in xz plane


B

1 Definition of centroidal coordinates


Xc =
Yc =
Zc =

2m=1m
2

xcL
L
ycL
L
zcL
L

where (xc, yc, zc) are the centroidal coordinates of


line segment L.

3m

Centroid

2m

xz plane

A
3
2 m = 1.5 m
z

Use the Pythagorean Theorem to calculate the length:


LAB =

2 Locate the centroids of the individual segments.

1783

(3 m)2 + (2 m)2 = 3.606 m

9.2 Method of Composite Parts Example 7, page 2 of 3


5 A table of geometric properties of planar
curves gives the information shown below.

Centroid of CD

y
C

Centroid Location

Centroid

Length =

3
2 m = 1.5 m

3m

yz plane

2r

4m=2m
2
4m

x
Quarter circular segment

9
y

Centroid of BC

r=3m

2(3 m)

LBC =

(3 m)

= 4.712 m

= 1.910 m
B

x
1.910 m

1784

LCD =

(3 m)2 + (4 m)2 = 5 m

9.2 Method of Composite Parts Example 7, page 3 of 3


10 Set up a table.
Segment
AB
BC
CD

xc ( m ) yc ( m ) zc ( m )
L
3.606 1.500 0.000 1.000
4.712 1.910 1.910 0.000
5.000 0.000 1.500 2.000
L = 13.318

11 Xc =

xcL
14.409
= 13.318 =
L

12 Yc =

ycL 16.500
= 13.318 = 1.239 m
L

Ans.

13 Zc =

zcL
.394
= 13.318 = 0.480 m
L

Ans.

.082 m

xcL ( m2 )
ycL ( m2 )
zcL ( m2 )
5.409
0.000
3.606
0.000
9.000
9.000
10.000
0.000
7.500
xcL = 14.409
ycL = 16.500
zcL = 6.394

Ans.

1785

9.2 Method of Composite Parts Example 8, page 1 of 3


8. Locate the centroid of the beam cross section shown.
y

70 mm

70 mm
25 mm

80 mm
x

20 mm
40 mm
1

By symmetry, Xc = 0

Ans.

Definition of y coordinate of centroid.


ycA
Yc =
A
where yc is the y coordinate of composite part with area A.

Consider the beam cross-section to be composed of two rectangles.


y

=
x

+
x

1786

9.2 Method of Composite Parts Example 8, page 2 of 3


4

Area and centroid of upper rectangle

Area A = (140 mm)(25 mm) = 3500 mm2

140 mm

140 mm

25 mm = 12.5 mm
2
C

25 mm
yc

80 mm

25 mm

80 mm
x

x
6 yc = 80 mm + 12.5 mm = 92.5 mm
7

Area and centroid of lower rectangle


8

Area A = (80 mm)(40 mm) = 3200 mm2


y

80 mm

C
yc
x

40 mm

yc = 40 mm

1787

80 mm = 40 mm
2
80 mm = 40 mm
2
x

9.2 Method of Composite Parts Example 8, page 3 of 3


10 Set up a table.
Region

A ( mm2 )

yc ( mm )

upper rectangle
lower rectangle

3500
3200
A = 6700

92.5
40

11 Yc =

ycA
451 750
= 6700 = 67.4 mm
A

ycA ( mm3 )
323 750
128 000
ycA = 451 750

Ans.

1788

9.2 Method of Composite Parts Example 9, page 1 of 3


9. Locate the centroid of the crosshatched region.
y

2m

4m

x
1

By symmetry, Xc = 0

Ans.

Definition of y coordinate of centroid.


ycA
Yc =
A

Consider the shaded area to be composed of a rectangle and a semicircle.


y
y
y

where yc is the y centroidal coordinate of the


composite part with area A.
=

1789

9.2 Method of Composite Parts Example 9, page 2 of 3


4

Area and centroid of rectangle

Area A = 4 m
y

A table of properties of planar regions


gives the information shown below.

4 m = 16 m2

Centroid Location
y

6 yc = 2 m

4m

Area =
yc

r2
2

x
4m
7

4r
3

Area and centroid of semicircle


y

r
x

Semicircular region

2m

In our particular problem, r = 2 m, so the distance to the centroid of


the semicircle is
4(2 m)
yc = 4 m + 4r
=
4
m
+
= 4 m + 0.849 m = 4.849 m
3
3

0.849 m
4m
and

2
(2 m)2
r
Area = 2 =
= 6.283 m2
2

1790

9.2 Method of Composite Parts Example 9, page 3 of 3


10 Set up a table.
Region
Rectangle
Semicircle

11 Yc =

A ( mm2 ) yc ( mm )
2
16
6.283
4.849
A = 22.283

ycA 62.466
= 22.283 = 2.80 m
A

ycA ( mm3 )
32.000
30.466
ycA = 62.466

Ans.

1791

9.2 Method of Composite Parts Example 10, page 1 of 2


10. Locate the centroid of the plate with hole.
y
400 mm

250 mm

60 mm
160 mm
x
500 mm

Definition of y coordinate of centroid.


xcA
A
ycA
Yc =
A
Xc =

where (xc, yc) are the coordinates of the centroid of the


composite part with area A.
2

Consider the shaded area to be composed of a rectangle minus a circle.


y
y

=
x

1792

9.2 Method of Composite Parts Example 10, page 2 of 2


3

Area and centroid of rectangle


y
500 mm = 250 mm
2
4 xc = 0.250 m

Area and centroid of circular hole


y
8 xc = 0.400 m
400 mm
C

250 mm

500 mm
6

60 mm
250 mm = 125 mm
2
x
5

yc = 0.160 m

160 mm
x
10 Area A = r2

yc = 0.125 m

= (0.060 m)2

Area A = (0.250 m)(0.500 m)

= 0.011310 m2

= 0.125 m

11 Set up a table.
Area
rectangle
circular hole

A ( m2 )
0.125000
0.011310
A = 0.113690

xc ( m )
0.250
0.400

yc ( m )
0.125
0.160

12 Xc =

xcA 0.026726
= 0.113690 = 0.235 m
A

Ans.

Yc =

ycA 0.013815
= 0.113690 = 0.122 m
A

Ans.

1793

xcA ( m2 )
0.031250
0.004524
xcA = 0.026726

ycA ( m2 )
0.015625
0.001810
ycA = 0.013815

9.2 Method of Composite Parts Example 11, page 1 of 3


11. Determine distance h so that the centroid of the crosshatched
region lies at y = h.
y
2 in. 2 in.
4 in.

h
x
3 in.

3 in.

By symmetry, Xc = 0.

Definition of y coordinate of the centroid


Yc =

yc A
A

where yc is the y centroidal coordinate of the


composite part with area A.

1794

9.2 Method of Composite Parts Example 11, page 2 of 3


3

Consider the shaded area to be composed of two rectangles and two triangles.
y

h
x

2 in.

Area A = 6h
6

C
3 in.

h
2

Triangle 1

yc = h2

4 in. = 2 in.
2

2 in.

9
yc = h + 2 in.
h

x
x

1795

Triangle 2

8 Area A = (4 in.)(4 in.) = 16 in2

4 in. = 2 in.
2

3 in.

Area and centroid of rectangle 2


y

Area and centroid of rectangle 1


5

h
x

Rectangle 2
7

h
x

x
Rectangle 1

9.2 Method of Composite Parts Example 11, page 3 of 3


10 Area and centroid of triangle 1

16 Set up table.

A ( in2 )

Region
Rectangle 1
4 in.

C
4 in. = 1.333 in.
3

12
yc = h + 1.333 in.

Rectangle 2
Triangle 1
Triangle 2

1 in. h
11 Apply the formula for
the centroid of a
triangle.

ycA ( in3 )

yc ( in )
h
2

3h2
16h + 32
h+2
16
h + 1.333
2h + 2.666
2
h + 1.333
2h + 2.666
2
2
A = 20 + 6h
yc A = 3h + 20h + 37.332
6h

13 Area A = 12 (4 in.)(1 in.) = 2 in2


17 Yc =
14 Area and centroid of triangle 2

h=

ycA
3h2 + 20h + 37.332
=
A
20 + 6h

3h2 + 20h + 37.332


20 + 6h

Simplifying gives
3h2

37.333 = 0

15 Same as triangle 1:
yc

which has the root

Area A = 2 in2
yc = h + 1.333 in.

h = 3.53 in.

1796

Ans.

9.2 Method of Composite Parts Example 12, page 1 of 3


12. Locate the centroid of the crosshatched region.
y
21 mm

60 mm

x
78 mm

We know that the centroid of a triangle lies one-third


the distance from the base to the opposite vertex.
Thus Yc can be computed directly:
Yc =

60 mm
= 20 mm
3

Ans.

60 mm
C
Yc
x

1797

9.2 Method of Composite Parts Example 12, page 2 of 3


2

Unfortunately the "one-third of the distance"


rule does not give us Xc.
y

3
h
3

Thus to calculate Xc, we have to either integrate or consider


the triangle to be made up of simpler shapes and then apply
the definition of the coordinate of the centroid:

It gives this.
Xc =

xcA
A

where xc is the x centroidal coordinate of the composite part


with area A.

C
x
Xc
4 But we want this.

Let's consider the given triangle to be composed of two right triangles, P and Q.
y

y
P

Q
+

1798

9.2 Method of Composite Parts Example 12, page 3 of 3


7

Area and centroid of region P


y
P

11 Area and centroid of region Q


y

8 Area A = 21 (0.021 m)(0.060 m)

xc
Q

= 0.000630 m
60 mm

xc

h
3

60 mm

9 Apply the formula for the


centroid of a triangle

10 xc = 0.021 m

21 mm

P
Q

18 Xc =

57 mm

12
Base of triangle =
78 mm 21 mm =
57 mm

78 mm

0.007 m = 0.014 m
13 Apply the formula for the centroid
of a triangle

16 Set up a table.
Region

h
3

h = 0.021 m = 0.007 m
3
3

h = 21 mm

A ( m2 ) xc ( m )

xcA ( m3 )

0.014
0.040

0.882 10-5
6.840 10-5
xcA = 7.722 10-5

0.000630
0.001710
A = 0.002340

xcA 7.722 10-5


= 0.002340 = 0.033 m
A

h = 0.057 m = 0.019 m
3
3
14 xc = 0.021 m + 0.019 m = 0.040 m
15 Area A = 1 (0.060 m)(0.057 m)
2
= 0.001710 m2

Ans.

1799

9.2 Method of Composite Parts Example 13, page 1 of 3


13. Determine the x coordinate of the
centroid of the crosshatched region.
y
D
1

15 in.

We know the rule that the centroid of a triangle


is located one-third the distance (the altitude)
from a base to the opposite vertex. Thus we can
compute the three centroidal distances shown
below. But we can't compute Xc directly.
y

E
O

9 in.

Altitude for vertex E

12 in.

hE
3

hE

hD

C
hD
3

F
O

Altitude for vertex D


Xc
This is the distance we want.

1800

hF
3
hF

Altitude for vertex F


Extension of side DE

9.2 Method of Composite Parts Example 13, page 2 of 3


2 Definition of x coordinate of the centroid
Xc =

xcA
A

where xc is the x centroidal coordinate of the


composite part with area A.

Consider the shaded region to be composed of


two right triangles, one subtracted from the
other.
y

y
P

1801

9.2 Method of Composite Parts Example 13, page 3 of 3


4

Area and centroid of region P


y
5 Area A = 21 (15 in.)(21 in.) = 157.5 in2
P

6 Apply the formula for the


centroid of a triangle

15 in.
C

21
xc = 3 in. = 7 in.

Area and centroid of region Q


y
8 Area A = 21 (15 in.)(9 in.) = 67.5 in2
Q
9

15 in.
C

xc = 93 in. = 3 in.

x
xc

xc

9 in.

9 in. + 12 in. = 21 in.

10 Set up a table. Note the minus sign for the area


of region Q (We are subtracting this region from
region P).
Region A ( in2 ) xc ( in. ) xcA ( in3 )
P
7
157.5
1102.5
Q
3
67.5
202.5
A = 90
xcA = 900
11 Xc =

xcA 900
A = 90 = 10 in.

Apply the formula for the


centroid of a triangle

Ans.

1802

9.2 Method of Composite Parts Example 14, page 1 of 2


14. Four rolled-steel angles are welded to a plate to form a
girder with the cross section shown. Locate the centroid.
y
15 mm

19.0 mm
Area = 932 mm2
C

700 mm

25.1 mm
C

Area = 772 mm2

3
y
1

By symmetry, Xc = 0

Definition of y coordinate of centroid


Yc =

Consider the cross section to be composed of


a plate and angles at the top and bottom.
y
y
y

Angles (top)

ycA
A

where yc is the y centroidal coordinate of


the composite section with area A.

+
Plate

Angles (bottom)
x

1803

9.2 Method of Composite Parts Example 14, page 2 of 2


4

Area and centroid of plate


y
5

Area and centroid of top angles


y

A = (15 mm)(700 mm) = 10500 mm2

A = 932 mm2
C

yc =

700
mm = 350 mm
2

19.0 mm
700 mm

700 mm

yc = 700 mm

19.0 mm = 681 mm

x
13 Set up a table.

15 mm

10 Area and centroid of bottom angles


y

Member
Plate
Angles (top)
Angles (bottom)

A = 772 mm2
C
x
25.1 mm

932 mm2 = 1864 mm2

8 Total A = 2
yc

11 Total A = 2

A ( mm2 ) yc ( mm )
10500
350.00
1864
681.00
25.10
1544
A = 13908

14 Yc =
772 mm2 = 1544 mm2

ycA
A
4983138.4
13908

= 358 mm

12 yc = 25.1 mm

1804

ycA ( mm3 )
3675000.00
1269384.00
38754.40
ycA = 4983138.40

Ans.

9.2 Method of Composite Parts Example 15, page 1 of 2


15. Two rolled-steel angles and a plate are welded
together to form a beam with the cross section shown.
Locate the centroid.
Area = 2430 mm2

30.2 mm
C
15 mm
x
120 mm

120 mm
2

By symmetry, Xc = 0.

Definition of y coordinate of centroid


ycA
Yc =
A
where yc is the y centroidal coordinate of the
composite part with area A.

Consider the section shown to be composed of two angles and a plate.


y

Angle 1

Angle 2

1805

x
Plate

9.2 Method of Composite Parts Example 15, page 2 of 2


4

Area and centroid of angles


y
y

6
A = 2430 mm2

30.2 mm

30.2 mm
C

C
yc

15 mm
Angle 1

x
15 mm

x
120 mm

Angle 2

A = 2430 mm2

Yc =

ycA
A

246672
8460

= 29.2 mm

A = ( 15 mm )( 240 mm ) = 3600 mm2


yc = 7.5 mm

Set up a table.
Member A ( mm2 ) yc ( mm )
Angle 1
45.2
2430
Angle 2
45.2
2430
Plate
7.5
3600
A = 8460

120 mm
Plate

yc = 15 mm + 30.2 mm = 45.2 mm

15 mm = 7.5 mm
2

15 mm

yc
x

Area and centroid of plate


y

ycA ( mm3 )
109836
109836
27000
ycA = 246672
Ans.

1806

9.2 Method of Composite Parts Example 16, page 1 of 5


16. The door of an autoclave unit is composed of a curved surface
ABCD bounded by two angular sectors on the ends. The door is made
of sheet metal of uniform thickness. Locate the centroid of the door.
Neglect the irregularities in geometry near the holes.
D

y
28 in.

B
x

C
60
34 in

A
z

By symmetry,
Zc =

34 in.
= 17 in.
2

Ans.

Definition of centroidal coordinates


Xc =
Yc =

xcA
A
ycA
A

where (xc, yc) are the centroidal coordinates of


the composite part with area A.

1807

9.2 Method of Composite Parts Example 16, page 2 of 5


3

Consider the door to be composed of two circular sector


areas and a part of a circular cylinder.
y

P
=
x

+
Q

1808

9.2 Method of Composite Parts Example 16, page 3 of 5


y

4 A table of properties of planar regions gives


the information shown below using the
entry for a circular sector area (Note that in
the formula equals half the angle of the arc).

Lc

60
yc
x

28 in.

Centroid Location
y

30
30

xc

Region P (same as Q)
Area = r2
5

r
C

In our particular problem for regions P and Q,


and r = 28 in., so
A = r2 = ( /6)(28 in.)2 = 410.501 in2
Lc =

2r sin

xc = Lc cos

2r sin
Circular sector region

= 60/2 = /6 radians

yc = Lc sin

1809

2(28 in.) sin ( /6)


= 17.825 in .
( /6)

= (17.825 in.) cos ( /6) = 15.437 in.


= (17.825 in.) sin ( /6) = 8.912 in.

9.2 Method of Composite Parts Example 16, page 4 of 5


6

A table of properties of planar regions


gives the information shown below
using the entry for a circular arc
segment (Note that in the formula
equals half the angle of the arc).

C
60

Lc
x
L

Centroid Location
y

30

34 in.
r

Length = 2 r
C

28 in.
z

xc

yc

34 in.

28 in.
z
Region R

7 In our particular problem for region R,


r = 28 in., so

= 60/2 = /6 radians and

r sin
Circular arc segment

Area = arc length


=2 r

Lc =

r sin

xc = Lc cos
yc = Lc sin

1810

length of straight edge

34 in. = 2( /6)(28 in.)

34 in. = 996.932 in2

(28 in.) sin ( /6)


= 26.738 in.
/6
= (26.738 in.) cos ( /6) = 23.156 in.

= (26.738 in.) sin ( /6) = 13.369 in.

9.2 Method of Composite Parts Example 16, page 5 of 5


8

Set up a table.
Region
P
Q
R

Xc =

10 Yc =

A ( in2 ) xc ( in. ) yc ( in. )


8.912
410.501 15.437
8.912
410.501 15.437
996.932 23.156 13.369
A = 1817.934

xcA =

xcA ( in3 )
6336.904
6336.904
23084.957
35758.765

35785.765
xcA
=
= 19.7 in.
1817.934
A

Ans.

ycA 20644.754
=
= 11.4 in.
1817.934
A

Ans.

1811

ycA ( in3 )
3658.385
3658.385
13327.984
ycA = 20644.754

9.2 Method of Composite Parts Example 17, page 1 of 5


17. A bird bath has been made by cutting a hemispherical
cavity in the top of a wooden block. The designer is
concerned that the block may tip over. As part of a study of
the stability of the structure, you are asked to locate the center
of gravity.
y
9 in.
9 in.
r = 8 in.
10 in.
10 in.

Because the block is made of a single material of


approximately uniform density, the center of gravity
coincides with the centroid.

Because of symmetry, the centroid must lie on the


y-axis (which passes through the center of the block), so

24 in.

1812

Xc = 0

Ans.

Zc = 0

Ans.

9.2 Method of Composite Parts Example 17, page 2 of 5


3

Definition of y coordinate of centroid


Yc =

ycV
V

where yc is the y coordinate of the centroid of


the composite part with volume V.
Consider the bird bath to be composed of two
simpler solid shapes, a block and a hemisphere.

5 Subtraction

1813

9.2 Method of Composite Parts Example 17, page 3 of 5


6

Volume Vand centroidal coordinate yc of block


V = (24 in.)(18 in.)(20 in.) = 8640 in3
y

18 in.

20 in.
24 in.

yc =
z

24
in. = 12 in.
2

1814

9.2 Method of Composite Parts Example 17, page 4 of 5


7

A table of properties of solid volumes


gives the information shown below.

y
r = 8 in.

Centroid Location
2
Volume = 3 r3

z
3
r = 3 (8 in.) = 3 in.
8
8
yc

24 in.

C
3r
8

r
y

Hemisphere
8

For our particular problem,


yc = 24 in.

in.

= 21 in.
2 3
r
3
2
=
(8 in.)3
3
= 1072.33 in3

V=

1815

9.2 Method of Composite Parts Example 17, page 5 of 5


9

Set up a table.
Solid
Block
Hemisphere

10 Yc =

V ( in3 )
8640.00
1072.33
V = 7567.67

yc ( in.)
12.00
21.00

ycV
81161.07
=
= 10.72 in.
V
7567.67

ycV ( in4 )
103680.00
22518.93
ycV = 81161.07
Ans.

1816

9.2 Method of Composite Parts Example 18, page 1 of 6


18. Ice cream was loosely packed into a cone, and
then a solid ball of ice cream of density 920 kg/m3
was placed on top. Estimate the density of the
loosely packed ice cream, if the center of mass of the
entire ice-cream cone is 120 mm above the tip of the
cone.
y
Radius of ball = 30 mm

130 mm

x
z

Because of symmetry,
Xc = 0

Ans.

Zc = 0

Ans.

1817

Definition of y-coordinate of center of mass


Yc =

ycm
m

(1)

where yc is the y-coordinate of the center of


mass of the composite part with mass m.

9.2 Method of Composite Parts Example 18, page 2 of 6


3

Consider the ice-cream cone to be composed of


the sum of three solids of simpler shape.
y

y
Sphere

y
Spherical cap

Cone

(Solid ice
cream)
=

(Loosely
packed
ice
cream)
x

x
z

(Loosely
packed
ice
cream)

1818

x
z

9.2 Method of Composite Parts Example 18, page 3 of 6


4 Volume of sphere
4 3
r
3
4
=
(0.03 m)3
3
= 1.13097 10-4 m3

Volume V =

5 Calculate the centroidal coordinate


of the center of the sphere.
y
CD = radius = 0.03 m

7 From triangle BCD,


BD = (0.03 m) cos

From triangle OBD,

D
B
6

By similar
triangles,
angles are
equal.

BD = (0.13 m) tan
Thus
0.13 m

yc

must satisfy
(0.03 m) cos

= (0.13 m) tan

Solving numerically gives


= 12.688

x
Geometric relations between sphere and cone

(2)

and so
yc = 0.13 m + (0.03 m) sin 12.688
= 0.13659 m

1819

(3)

9.2 Method of Composite Parts Example 18, page 4 of 6


8

A table of properties of solid volumes gives the


information shown below.

y
0.03 m

Centroid Location
z
2
rh
Volume =
3

= 12.688
C
B
G

h
4

C
r
x

9 For our problem,


yc = h

h
4

yc

0.13 m

= 3h/4

Cone

= 3(0.13 m)/4

= 0.09750 m
10 Volume of cone V =
=

r2h
3
(0.03 m) cos 12.688 2(0.13 m)
3

= 1.16611 10-4 m3

1820

9.2 Method of Composite Parts Example 18, page 5 of 6


11 A table of properties of solid volumes gives the
information shown below.

V=

1 R3(2
3

+ 3 sin3

3 sin
z

3R(1+ sin )2
4(2+ sin )

C
R
D
G

3R(1+ sin )2
4(2+ sin )

0.13 m

yc

y
x
x

Spherical cap
z
0.03 m
12 For our problem,

yc = 0.13 m

R = 0.03 m
3R(1+ sin )2
4(2+ sin )

V=

= 12.688, by Eq. 2

1 R3(2
3

= 0.38817

R sin

0.12151 m

1821

12.688

3 sin
10-4 m3

+ 3 sin3

9.2 Method of Composite Parts Example 18, page 6 of 6


13 Set up a table.

14 Mass = density

V ( m3 )

Solid

Density,

Sphere

1.13097 10-4

Cone

1.16611 10-4

Cap

-4

0.38817 10

( kg/m3 )

920

m = V ( kg )

yc ( m )

0.10405
1.16611 10-4

-4

0.38817 10

m = 0.10405 + 7.7794
15 Yc =
0.12 =

ycm
m

Center of mass is to lie 0.12 m


above the tip of the cone.

0.01421 + 0.66529

10-5

0.10405 + 7.7794 10-5

Solving gives
c

= 643 kg/m3

Ans.

1822

c
c

10-5

volume
ycm ( kg m )

0.13659

0.01421

0.09750

1.13696 10-5

0.12151

-5

0.47167 10

c
c

ycm = 0.01421 + 0.66529 10-5

9.2 Method of Composite Parts Example 19, page 1 of 9


19. Locate the center of mass of the buoy shown.
The buoy is of uniform density.
y
1
Hemisphere

Because of symmetry,

150 mm
Xc = 0

Ans.

Zc = 0

Ans.

350 mm
Definition of y coordinate of center of mass

250 mm

Yc =
500 mm
Hemisphere

ycV
V

where yc is the y coordinate of the center of


mass of the composite part with mass m.

x
z

1823

9.2 Method of Composite Parts Example 19, page 2 of 9


2

Consider the buoy to be composed of a


sum of solids that have simple shapes.
y
y

Top
hemisphere

Cylinder

Large cone
=

x
z

x
z

x
z

x
z
y

y
Bottom
hemisphere

Small cone
3

The frustum of a cone can be considered to be


a small cone subtracted from a large cone.
+

x
z

1824

x
z

9.2 Method of Composite Parts Example 19, page 3 of 9


y

4 A table of properties of solid volumes


gives the information shown below.

r = 150 mm

3r
8

150 mm
Centroid Location
2
Volume = 3 r3

500 mm + 250 mm + 350 mm


= 1100 mm

yc

1100 mm

C
3r
8

r
y

Hemisphere

x
z
Top hemisphere

For our particular problem,


3
r
8
3
= 1100 mm + (150 mm)
8

yc = 1100 mm +

= 1156.25 mm
2
Volume V = 3 r3
2
= 3 (150 mm)3
= 7.069 106 mm3

1825

9.2 Method of Composite Parts Example 19, page 4 of 9


y

y
r = 150 mm

175 mm
350 mm

175 mm

yc

500 mm + 250 mm
= 750 mm

750 mm

x
Cylinder

z
6

yc = 750 mm + 175 mm
= 925 mm
V = area of base
= r2

height of cylinder

height

= (150 mm)2(350 mm)


= 24.740

1826

106 mm3

9.2 Method of Composite Parts Example 19, page 5 of 9


7

Before we can calculate the volume of the


large cone, we must first find its altitude.
y

y
A

150 mm
250 mm

s
C

150 mm
D

500 mm

250 mm
E

500 mm

x
z

8 Triangles ABC and ADE are similar, so


s
s + 250 mm
=
150 mm
500 mm
Solving gives
s = 107.143 mm

1827

9.2 Method of Composite Parts Example 19, page 6 of 9


y

9 A table of properties of solid volumes


gives the information shown below.
Centroid Location

s = 107.143 mm

z
2

rh
Volume =
3

107.143 mm + 250 mm
= 357.143 mm

250 mm

500 mm

h
4

C
r
x

x
z

y
Large cone
y

Cone

h
10 yc = 500 mm + 4
357.143 mm
= 500 mm +
4

h = 357.143 mm

= 589.286 mm

h
4

V = ( r2h)/3

500 mm r = 500 mm

= [ (500 mm)2(357.143 mm)]/3

yc

= 93.500
x
z

1828

106 mm3

9.2 Method of Composite Parts Example 19, page 7 of 9


11 Using the same table of properties of solid volumes as was
used for the large cone gives the information shown below.

Centroid Location

107.143 mm

z
Volume =

r2h
3
500 mm + 250 mm
= 750 mm

h
4

C
r

Cone
Small cone
y
h = 107.143 mm

r = 150 mm

h
12 yc = 750 mm + 4
107.143 mm
= 750 mm +
4
= 776.786 mm

h
4
yc

V = ( r2h)/3
750 mm

= [ (150 mm)2(107.143 mm)]/3


= 2.524

1829

106 mm3

9.2 Method of Composite Parts Example 19, page 8 of 9


y

13 A table of properties of solid volumes


gives the information shown below.

Centroid Location
2
Volume = 3 r3

r = 500 mm

500 mm

C
3r
8

r
y

3r
8
yc
x

x
z

Bottom hemisphere

Hemisphere

14 For our particular problem,


yc = 500 mm
= 500 mm

3r
8
3
(500 mm)
8

= 312.5 mm
2
V = 3 r3
=

2
(500 mm)3
3

= 261.799 106 mm3

1830

9.2 Method of Composite Parts Example 19, page 9 of 9


15 Set up a table.
Volume, V ( mm3 )

Solid

7.069 106
24.740 106
93.500 106
2.524 106
261.799 106
V = 384.584 106

Top hemisphere
Cylinder
Large cone
Small cone
Bottom hemisphere

16 Yc =

ycV 166.007 109


=
= 432 mm
V
384.584 106

ycV ( mm4 )

yc ( mm )
1156.250
925.000
589.286
776.786
312.500

8.173
22.885
55.098
1.961
81.812
ycV = 166.007

Ans.

1831

109
109
109
109
109
109

9.2 Method of Composite Parts Example 20, page 1 of 9


20. Locate the centroid of the bracket shown.
y

60 mm
70 mm
50 mm

Circular arc

50 mm

60

x
40 mm

8 mm

10 mm
10 mm

Definition of centroidal coordinates


Xc = xcV
V
Yc =

ycV
V

Zc = zcV
V
where (xc, yc, zc) are the coordinates of the
centroid of the composite part with volume V.

1832

9.2 Method of Composite Parts Example 20, page 2 of 9


2

Consider the bracket to be composed of a sum of solids that have simple shapes.
y
y

P
Q
+

+
R

x
z

z
3

y
+

The bottom plate extends to the


far left side of the bracket.
y

x
S
T

z
4

1833

The plate and hole can be considered to be a


solid cylinder subtracted from a uniform plate.

9.2 Method of Composite Parts Example 20, page 3 of 9


5

Calculate dimensions of solids P, Q, and R.

(70 mm) cos 60 = 35.0 mm

60 mm

60 mm

(70 mm) sin 60 = 60.622 mm

Q
70 mm
R

y
35.0 mm = 25.0 mm

60
9

70 mm + 25.0 mm = 95.0 mm

70 mm
11 View of solid P as seen from the positive x axis
y

10 Volume and centroidal coordinate of solid P.


y
10 mm
xc =

10 mm
2

25 mm

25 mm
2

= 12.5 mm

= 5 mm

zc =

60.622 mm

60.622 mm
C

x
25.0 mm

yc =
z

60.622 mm
2

= 30.311 mm

Volume V = (10 mm)(60.622 mm)(25 mm) = 1.516


z

1834

104 mm3

9.2 Method of Composite Parts Example 20, page 4 of 9


12 Volume and centroidal coordinate of solid Q.
y
10 mm
2

xc =

25 mm
35 mm

= 5 mm
C

60.622 mm
x

15 Volume V = area of triangle


=

1
(60.622 mm)(35.0 mm)(10 mm)
2

= 1.061
z

thickness

104 mm3

13 View of solid Q as seen


from the positive x axis
y
zc = 25.0 mm + 11.667 mm = 36.667 mm

10 mm

zc

60.622 mm
= 20.207 mm
3
C

60.622 mm
yc

yc = 60.622 mm

x
35.0 mm 25.0 mm

35.0 mm
= 11.667 mm
3

1835

20.207 mm = 40.415 mm
14 Apply formula to locate centroid of triangle at h
3
.

9.2 Method of Composite Parts Example 20, page 5 of 9


16 Volume and centroidal coordinate of solid R.

17 View of solid R as seen from the positive x axis

y
Radius = 70 mm

10 mm

Lc

70 mm
x
C

60

yc
25 mm

25.0 mm
zc
xc =

C
60
2 = 30

10 mm
2

= 5 mm

1836

9.2 Method of Composite Parts Example 20, page 6 of 9


18 A table of properties of planar regions gives
the information below.

19 For our particular problem,


= 30 =

Centroid Location

radians

r = 70 mm

y
Area = r2

Thus

r
C

Area =

(70 mm)2 = 2566 mm2

2(70 mm) sin


Lc =
2r sin

yc = Lc sin

6 = 44.563 mm

6
= (44.563 mm) sin 30 = 22.282 mm

Circular sector region


zc = 25 mm + Lc cos
y

= 25 mm + (44.563 mm) cos 30

Radius = 70 mm
Lc

= 63.593 mm
Volume = area

thickness

C
z

60
2 = 30

yc
25 mm
zc

1837

= (2566 mm2)(10 mm)


= 2.566

104 mm3

9.2 Method of Composite Parts Example 20, page 7 of 9


20 Volume and centroidal coordinate of solid S (Recall that the
bottom plate extends to the far left side of the bracket.)
y

100 mm

x
xc = 55 mm

10 mm = 45 mm
100 mm + 10 mm
= 55 mm
2

C
95.0 mm
z

10 mm

10 mm

10 mm
10 mm

yc = 10 mm = 5 mm
2

zc =

21 Volume = (10 mm)(110 mm)(95.0 mm)


= 10.450

104 mm3

1838

95.0 mm = 47.5 mm
2

9.2 Method of Composite Parts Example 20, page 8 of 9


22 Volume and centroidal coordinate of solid T.
y

50 mm

50 mm

x
40 mm

8 mm
y
50 mm

x
40 mm

8 mm
10 mm

10 mm

z
10 mm
= 5 mm
2
23 (xc, yc, z c) = (50 mm, 5 mm, 40 mm)
Volume = area of circle

height of cylinder

= (8 mm)2(10 mm)
= 2.011

1839

103 mm3

9.2 Method of Composite Parts Example 20, page 9 of 9


24 Set up a table.
Solid Volume, V ( mm3 ) xc ( mm ) yc ( mm ) zc ( mm )
xcV ( mm4 )
P
30.311
12.500
1.516 104
5
0.758 105
4
Q
40.415
36.667
1.061 10
5
0.531 105
R
22.282
63.593
2.566 104
5
1.283 105
S
10.450 104
45
5.000
47.025 105
47.500
T
201 104
50
5.000
1.005 105
40.000
V = 15.392 104
xcV = 43.448 105

25 Xc =

xcV 43.448
=
V
15.392

105
= 28.2 mm
104

ycV
9.477 105
Yc =
=
= 6.2 mm
V
15.392 104
Zc =

zcV
70.902 105
=
= 46.1 mm
V
15.392 104

Ans.

Ans.

Ans.

1840

ycV ( mm4 )

zcV ( mm4 )

4.595
4.288
5.718
5.225
0.101

105
105
105
105
105

1.895
3.891
16.318
49.638
0.840

105
105
105
105
105

ycV = 9.477

105

zcV = 70.902

105

9.2 Method of Composite Parts Example 21, page 1 of 4


21. Determine the greatest depth, h, of the circular hole of
diameter 180 mm for which the cube will remain in the
position shown. The cube is made of a material of uniform
density and is supported by a hinge A along the edge
perpendicular to the plane of the figure.
10 mm

180 mm
h
10 mm
B
40
A

1841

9.2 Method of Composite Parts Example 21, page 2 of 4


Free-body diagram of cube with hole

y The line of action of the


weight, W, of the cube
passes through the
centroid, C.

RB = 0
(The reaction at
the roller is zero,
if the cube is just
about to tip.)

The center of gravity (same


as the centroid, C) lies
C
directly above point A.

Yc

90

40 = 50

Ax

Ay

40
40

2 Moment equilibrium:
MA = 0:

x
200 mm
= 100 mm
2
(one half of length of side of cube)

3 Geometry

Wb = 0

Therefore b = 0. That is, the line of


action of the weight, W, must pass
through the corner of the cube, A.

Now the centroidal distance, Yc, can be


calculated independent of distance h:
Yc = (100 mm) tan 50
= 119.175 mm

1842

(1)

9.2 Method of Composite Parts Example 21, page 3 of 4


5

Next, we have to derive an expression for Yc in terms


of the depth h of the hole.

Consider the cube with a hole to consist of a solid


cylinder subtracted from a solid cube.
y

Radius = 90 mm

=
200 mm

x
Volume V = (200 mm)3
=8

106 mm3

Centroidal height yc = (200 mm)/2


= 100 mm

1843

z
Volume V = (90 mm)2h
= (8100 h) mm3
Centroidal height yc = h/2

9.2 Method of Composite Parts Example 21, page 4 of 4


7 Set up a table.
Volume, V ( mm3 )

Solid
Cube

Cylinder

106

8100 h
V=8

106

yc ( mm )

ycV ( mm4 )

100
h
2

8100 h

108

4050 h2
ycV = 8

ycV
8 108 4050 h2
Yc = V =
8 106 8100 h

108

4050 h2

(2)

Substituting Yc = 119.175 mm from Eq. 1 into the left-hand side of Eq. 2 gives
119.175 =

8 108 4050 h2
8 106 8100 h

Solving gives
h = 72.8 mm

Ans.

1844

9.3 Theorems of Pappus and Guldinus

1845

9.3 Theorems of Pappus and Guldinus Procedures and Strategies, page 1 of 2


a

Procedures and Strategies for Solving Problems Involving


the Theorems of Pappus and Guldinus
To calculate the area of a solid of revolution,

Curve of length L
1. identify the axis of revolution,
2. sketch the generating curve,
3. find the distance rC from the axis of revolution to the
centroid of the curve, and
4. Apply the formula for the area:

a
a

A = 2 rCL

rC

Centroid of curve

where L is the length of the curve.


To calculate the volume of a solid of revolution,
1. identify the axis of revolution,
2. sketch the generating area,
3. find the distance rC from the axis of revolution to the
centroid of the area, and
4. apply the formula for the volume:

b
b

V = 2 rCA

rC

Planar region of area A

where A is the generating area.


Centroid of region
b

1846

9.3 Theorems of Pappus and Guldinus Procedures and Strategies, page 2 of 2


Notes:
1) If the curve or area is rotated an amount less than a complete
revolution, then the factor of 2 in the equations for the volume
and area must be replaced by (expressed in radians).
2) If the centroidal distance cannot be found from a table, then you
will have to calculate the distance by evaluating an integral. But you
can save work by noting that you do not have to calculate rC and L
independently you need calculate only the product rCL and that
can be found by noting that

xC =

xel dL
dL

implies
xCL = xel dL
So you only have to evaluate one integral, namely
xel dL

1847

9.3 Theorems of Pappus and Guldinus Problem Statement for Example 1


1. Determine the amount of paint required to paint
the inside and outside surfaces of the cone, if one
gallon of paint covers 300 ft2.
y

3 ft

10 ft

1848

9.3 Theorems of Pappus and Guldinus Problem Statement for Example 2


2. Determine the volume of the cone.
y

3 ft

10 ft

1849

9.3 Theorems of Pappus and Guldinus Problem Statement for Example 3


3. Determine the area of the
half-torus (half of a doughnut).
y
1m
C
4m
x

1850

9.3 Theorems of Pappus and Guldinus Problem Statement for Example 4


4. Determine the volume of the
half-torus (half of a doughnut).
y
1m
C
4m
x

1851

9.3 Theorems of Pappus and Guldinus Problem Statement for Example 5


5. Determine the area of the frustum of the cone.
y
3m

4m

2m
O

1852

9.3 Theorems of Pappus and Guldinus Problem Statement for Example 6


6. Determine the volume of the frustum of the cone.
y
3m

4m

2m
O

1853

9.3 Theorems of Pappus and Guldinus Problem Statement for Example 7


7. Determine the centroidal coordinate rc of a
semicircular arc of radius R, given that the area of a
sphere of radius R is known to be 4 R2.
y

rC
C

1854

9.3 Theorems of Pappus and Guldinus Problem Statement for Example 8


8. Determine the centroidal coordinate rc of a
semicircular area of radius R, given that the volume of a
sphere is known to be (4/3) R3.
y
Radius = R
rC
C

1855

9.3 Theorems of Pappus and Guldinus Problem Statement for Example 9


9. A concrete dam is to be constructed in the shape
shown. Determine the volume of concrete that
would be required.

20 m

3.5 m

40

3m

1m 2m

1856

9.3 Theorems of Pappus and Guldinus Problem Statement for Example 10


10. The concrete steps shown are in the shape of a quarter
circle. Determine the amount of paint required to paint
the steps, if one liter of paint covers 1.5 m2.
y
260 mm

190 mm

260 mm

190 mm
x
z

1857

9.3 Theorems of Pappus and Guldinus Problem Statement for Example 11


11. The concrete steps shown are in the shape of a quarter circle.
Determine the total number of cubic meters of concrete required to
construct the steps.
y
260 mm

190 mm

260 mm

190 mm
x
z

1858

9.3 Theorems of Pappus and Guldinus Problem Statement for Example 12


12. Determine the mass of the steel V-belt pulley
shown. The density of the steel is 7840 kg/m3.

12 mm 10 mm 12 mm
4 mm

4 mm

15 mm

100 mm 70 mm

20 mm
15 mm

Front view

Side view

1859

9.3 Theorems of Pappus and Guldinus Problem Statement for Example 13


13. Determine the area of the surface of
revolution generated by rotating the curve
y = z4, 0 z 1 m, about the z axis.
y

1m

1860

9.3 Theorems of Pappus and Guldinus Problem Statement for Example 14


14. Determine the volume of the solid of
revolution generated by rotating the curve
y = z4, 0 z 1 m, about the z axis.
y

1m

1861

9.3 Theorems of Pappus and Guldinus Problem Statement for Example 15


15. A pharmaceutical company plans to put a coating
0.01 mm thick on the outside of the pill shown.
Determine the amount of coating material required.
Radius = 20 mm

7 mm

1.5 mm

1862

9.3 Theorems of Pappus and Guldinus Problem Statement for Example 16


16. Determine the volume of the funnel.

10 mm

60 mm

5 mm
70 mm

5 mm

1863

9.3 Theorems of Pappus and Guldinus Problem Statement for Example 17


17. A satellite dish is shaped in the form of a paraboloid of
revolution to take advantage of the geometrical fact that all
signals traveling parallel to the axis of the paraboloid are
reflected through the focus. Determine the amount, in m2,
of reflecting material required to cover the inside surface of
the dish.

0.3 m
Signals parallel
to axis of dish
0.3 m

0.2 m

1864

9.3 Theorems of Pappus and Guldinus Problem Statement for Example 18


18. Determine the amount of coffee that
the coffee mug holds when full to the
brim. The radius of the rounded corners
and the rim is 15 mm.
80 mm

90 mm

1865

9.3 Theorems of Pappus and Guldinus Problem Statement for Example 19


19. Determine the capacity of the small bottle of lotion if the
bottle is filled half way up the neck.
5 mm

17.5 mm
Radius = 20 mm
15 mm

1866

9.3 Theorems of Pappus and Guldinus Example 1, page 1 of 2


1. Determine the amount of paint required to paint
the inside and outside surfaces of the cone, if one
gallon of paint covers 300 ft2.
y

1 The y axis is the axis of revolution.


y
3 ft
3 ft
3 ft
= 1.5 ft
2

2 The generating curve is a


straight line through the
origin.

10 ft

10 ft

x
x

3 The distance to the centroid of the line is


rC = 1.5 ft

1867

9.3 Theorems of Pappus and Guldinus Example 1, page 2 of 2


3 ft

4 The length of the curve can be found from the Pythagorean theorem:
L=

(10 ft)2 + (3 ft)2

= 10.4403 ft

10 ft

Applying the first theorem of Pappus-Guldinus gives the area:


A = 2 rcL
= 2 (1.5 ft)(10.4403 ft)
= 98.3975 ft2
Calculate the volume of paint required:
Volume of paint = 2(98.3975 ft2)(

= 0.656 gal

1 gal
300 ft2

Ans.

Because both the inside and outside surfaces must be painted, the value of
the computed area must be doubled.

1868

9.3 Theorems of Pappus and Guldinus Example 2, page 1 of 1


2. Determine the volume of the cone.
y
1 The y axis is the axis of rotation.
y

3 ft
rc =

3 ft
= 1 ft
3

3 ft
2 The generating area is
a triangle

10 ft

3 The centroid of a
triangle is located
one-third of the
distance from the base
to the opposite vertes.

C
10 ft
4 The area of the triangle is
x

A = (1/2)(10 ft)(3 ft)


= 15 ft2

Applying the second theorem of


Pappus-Guldinus gives the volume:
V = 2 rcA
=2

ft)( 15 ft2)

= 94.2 ft3

1869

Ans.

9.3 Theorems of Pappus and Guldinus Example 3, page 1 of 1


3. Determine the area of the
half-torus (half of a doughnut).

1 The axis of revolution is the x axis.

y
1m

2 The generating curve


is a circle.

y
C

4 The length of
the curve is the
circumference
of the circle:

1m

4m

C
x

4m

L = 2 (1m)
x

= 6.2832 m

z
3

The distance to the centroid


is rc = 4 m.
5 Applying the first theorem of
Pappus-Guldinus gives the area:
A = rcL
= (4 m)(6.2832 m)
= 79.0 m2

Ans.

The angle of revolution is , not 2 , because


the figure is a half -torus.

1870

9.3 Theorems of Pappus and Guldinus Example 4, page 1 of 1


4. Determine the volume of the
half-torus (half of a doughnut).

1 The axis of revolution is the x axis

y
1m

2 The generating area is the


region bounded by a circle

y
C

4 The area bounded by


the circle is

1m

4m

C
x

A = (1 m)2

4m

= m2
x

z
3 The distance to the centroid is rc = 4m
5

Applying the second theorem of


Pappus-Guldinus gives the volume:
V = rcA
= (4 m)( m2)
= 39.5 m3

Ans.

The angle of revolution is , not 2 , because


the figure is a half -torus.

1871

9.3 Theorems of Pappus and Guldinus Example 5, page 1 of 2


5. Determine the area of the frustum of the cone.

1 The y axis is the axis of rotation


y

2 The generating curve is the


straight line BD. The
horizontal coordinate d of
the lower end of the line
can be found by similar
triangles:
3m
D

3m

3m

D
4m

4m
d
B

2m
O

2m
x

4m

C
d

2m

O
3m
d
=
2m
2m+4m
Solving gives
d=1m

1872

9.3 Theorems of Pappus and Guldinus Example 5, page 2 of 2


y
3m

The distance to
the centroid is
rc = (1 m + 3 m)/2

C
B

1m

=2m

3m

O
D
4 The length of the generating curve BD is
4m
B
O

3m

L = (2 m)2 + (4 m)2

2m

= 4.4721 m

4m

1m=2m
D

x
B

5 Applying the first theorem of Pappus-Guldinus gives the area:


A = 2 rcL
= 2 (2 m)(4.4721 m)
= 56.2 m2

Ans.

1873

9.3 Theorems of Pappus and Guldinus Example 6, page 1 of 3


6. Determine the volume of the frustum of the cone.
1 The y axis is the axis of rotation
y
rC
3m
2 The generating area is the
trapezoid BDEF. The
horizontal coordinate d of
D
E
the lower end of the line
C
BD can be found by similar
4m
triangles:
3m
B
F
D
d
2m
4m
x
O
d
B
2m

y
3m

4m

2m
O

O
3 The distance rc to the centroid of the
area can be calculated by dividing the
crosshatched trapezoid into composite
parts and using the formula

rc =

xelAel
Ael

Solving gives
d=1m

(1)

where xel is the centroidal coordinate


of the part with area Ael.

1874

3m
d
=
2m
2m+4m

9.3 Theorems of Pappus and Guldinus Example 6, page 2 of 3


4

However, we can save some work by noting that the


second theorem of Pappus-Guldinus involves the
product of rc and the generating area A:
V = 2 rcA

(2)

where V is the volume of the solid of revolution.


Solving Eq. 1 for the product of rc and Ael gives
rc Ael = xelAel
A = total area = sum of individual elements of area
and substituting this result in Eq. 2 gives
V=2

xelAel

(3)

Thus we do not need to calculate rc and A


independently we need only evaluate the first
moment of the area, xelAel.

1875

9.3 Theorems of Pappus and Guldinus Example 6, page 3 of 3


5

To calculate xelAel, divide the trapezoidal region into


the sum of a rectangle and triangle, and set up a table.
y

y
3m

2m

1m

1m
Ael = (4 m)(1 m)

4m

4m

= 4 m2

1m
O

Ael = (1/2)(4 m)(2 m)

C
+

4m

xel = 0.5 m
x

4m

= 4 m2

xel = 1 m + (1/3)(2 m)
Region

Ael (m2)

xel (m)

Rectangle
Triangle

4
4

0.5
1.6667

xelAel (m3)

= 1.6667 m

2
6.6668
xelAel = 8.6668

Substituting the value of xelAel into Eq. 3 gives the volume of the solid:
V=2

xelAel

(Eq. 3 repeated)
8.6668

= 54.5 m3

Ans.

1876

9.3 Theorems of Pappus and Guldinus Example 7, page 1 of 1


7. Determine the centroidal coordinate rc of a
semicircular arc of radius R, given that the area of a
sphere of radius R is known to be 4 R2.
y

1 If the semicircle is revolved around the y axis,


a sphere of radius R is generated. The first
theorem of Pappus-Guldinus says that the
area of the sphere is given by
A = 2 rcL

rC
C

Because we already know A (= 4 R2), we can


solve this equation for rc in terms of R and L:

4 R2
rc =

A
2 L

(1)

2 The length L in Eq. 1 is the circumference of


the semicircle:
L= R
Substituting this result in Eq. 1 gives
2
rc = 4 R
2 L

(Eq. 1. repeated)
R

1877

2R

Ans.

9.3 Theorems of Pappus and Guldinus Example 8, page 1 of 1


8. Determine the centroidal coordinate rc of a
semicircular area of radius R, given that the volume of a
sphere is known to be (4/3) R3.

1 If the semicircular area is revolved around the y axis,


a sphere of radius R is generated. The second
theorem of Pappus-Guldinus says that the volume of
the sphere is given by

y
Radius = R

V = 2 rcA
rC
C

(1)

Because we already know V (= (4/3) R3), we can


solve Eq. 1 for rc in terms of A and R:

(4/3) R3
rc =

V
2 A

(2)

2 The area A in Eq. 2 is the area bounded by the


semicircle:
R2
2
Substituting this result in Eq. 2 gives
A=

rc =

(4/3) R3
2 A

(Eq. 2 repeated)
R2
2

1878

4R
3

Ans.

9.3 Theorems of Pappus and Guldinus Example 9, page 1 of 4


9. A concrete dam is to be constructed in the shape
shown. Determine the volume of concrete that
would be required.
1 The y axis is the axis of revolution
y

3.5 m
20 m

3.5 m

x
40

20 m
3m

3m

1m 2m

1m 2m

2 The crosshatched region is the generating area.

1879

9.3 Theorems of Pappus and Guldinus Example 9, page 2 of 4


y

rc

V = 2 40/360)rcA

(1)

where rc is the distance to the centroid of the generating


area, A is the magnitude of the area, and the
factor 40/360 accounts for the fact that the dam
corresponds to 40 rather than to a complete circle.
Thus we must calculate the product rcA. This product
may be found by dividing the crosshatched area into
composite parts and then using the formula

3.5 m
x
20 m
3m

The second theorem of Pappus-Guildinus gives the


volume as

1m 2m

Generating area divided into a


rectangle and two triangles

rc =

xelAel
Ael

(2)
A = sum of element areas

where xel is the centroidal coordinate of the part with


area Ael. Solving for the product rcA gives
rcA = xelAel
Thus Eq. 1 can be written as
V = 2 40/360)(rcA)
= (2/9)

xelAel

(Eq. 1 repeated)
(3)

and, to calculate V, we have to evaluate the sum xelAel.

1880

9.3 Theorems of Pappus and Guldinus Example 9, page 3 of 4


y

4 Calculate the areas and centroidal


distances, and set up a table.

xel = 23 m + (3 m)/3 = 24 m

3.5 m
Ael = (1/2)(3.0 m)

x
23 m

3m

rc

Triangle 1

3.5 m

xel = 20 m + 2 m + 0.5 m = 22.5 m

Ael = (3.5 m)

3.5 m

20 m
3m

3.5 m) = 5.25 m2

1m 2m

m) = 3.5 m2
O

x
Rectangle
1m 2m

20 m
O
+

xel = 20 m + (2 m)(2/3) = 21.333 m

Ael = (1/2)(3.5 m)
= 3.5 m2

3.5 m
x

O
Triangle 2

1881

m)

2m

20 m

9.3 Theorems of Pappus and Guldinus Example 9, page 4 of 4


5 Table
Region

Ael (m2)

xel (m)

Triangle 1
Rectangle
Triangle 2

5.25
3.5
3.5

24
22.500
21.333

xelAel (m3)
126
78.750
74.665
xelAel = 279.415

6 Substituting the value of xelAel into Eq. 3 gives the


volume of the solid:
2
V= 9

xelAel

(Eq. 3 repeated)

279.415 m3
= 195.1 m3

Ans

1882

9.3 Theorems of Pappus and Guldinus Example 10, page 1 of 4


10. The concrete steps shown are in the shape of a quarter
circle. Determine the amount of paint required to paint
the steps, if one liter of paint covers 1.5 m2.
y
260 mm

190 mm

260 mm

190 mm
1 The y axis is the axis of revolution.

x
z

y
2 The generating
curve is a series of
four straight
line-segments.

190 mm
190 mm

x
260 mm

1883

260 mm

9.3 Theorems of Pappus and Guldinus Example 10, page 2 of 4


3 The first theorem of Pappus-Guildinus gives the area as
A = 2 (90/360)rcL
y

= ( /2)rcL

190 mm
190 mm
x
260 mm

(1)

where rc is the distance to the centroid of the generating


curve, L is the length of the area, and the factor (90/360)
accounts for the fact that the steps are in the shape of a
quarter circle. Thus we must calculate the product rcL.
This product may be found from the formula for the centroid
of a composite curve made of a of collection of line
segments:

260 mm
rc =

xelLel
Lel

(2)
L = sum of segment lengths

where xel is the centroidal coordinate of the line segment


with length Lel. Solving for the product rcL gives
rcL = xelLel
Thus Eq. 1 can be written as
A = ( /2)(rcL)
= ( /2) xelLel

(Eq. 1 repeated)
(3)

and, to calculate A, we have to evaluate the sum xelLel.

1884

9.3 Theorems of Pappus and Guldinus Example 10, page 3 of 4


4

Calculate the centroidal coordinates


and lengths of the line segments, and
set up a table.

Lel = 190 mm
Lel = 260 mm

y
y

xel = 260 mm

260 mm
=

190 mm

190 mm

xel = 260 mm/2


= 130 mm

380 mm

+
190 mm
x

190 mm
x
260 mm

Line 2

Line 1

260 mm
y

xel = 260 mm + 260 mm


= 520 mm

Lel = 260 mm
+

260 mm

Lel = 190 mm

260 mm
+
190 mm
x

190 mm
x
260 mm

260 mm

xel = 260 mm + (260 mm)/2 = 390 mm


Line 3

1885

Line 4

9.3 Theorems of Pappus and Guldinus Example 10, page 4 of 4


5 Table
Region

Lel (mm)

xel (mm)

Line 1
Line 2
Line 3
Line 4

260
190
260
190

130
260
390
520

xelLel (mm2)
33 800
49 400
104 100
98 800
xelLel = 283 400

6 Substituting the value of xelAelinto Eq. 3 gives the area of the steps
A = 2 xelLel

(Eq. 3 repeated)
2

283 400 mm
= 445 164 mm2
7

Amount of paint required = 445 164 mm2


= 0.297 liter

[1 m/(1000 mm)]2

[1 liter of paint/1.5 m2 of covered area]


Ans.

1886

9.3 Theorems of Pappus and Guldinus Example 11, page 1 of 3


11. The concrete steps shown are in the shape of a quarter circle.
Determine the total number of cubic meters of concrete required to
construct the steps.
y

The y axis is the axis of revolution

260 mm

190 mm

190 mm
260 mm
190 mm
x

190 mm

260 mm

260 mm

x
2 The generating area is the crosshatched region
shown.

1887

9.3 Theorems of Pappus and Guldinus Example 11, page 2 of 3


3 The second theorem of Pappus-Guildinus gives the
volume as
V = 2(90/360 rcA
= ( /2)rcA

where rc is the distance to the centroid of the generating


area, A is the magnitude of the area, and the factor
(90/360) accounts for the fact that the steps form a
quarter circle. Thus we must calculate the product rcA.
This product may be found by dividing the crosshatched
area into composite parts and then using the formula

190 mm
190 mm
O

260 mm

(1)

x
260 mm

rc =

Generating area divided into two


rectangles

xelAel
Ael

(2)
A = sum of element areas

where xel is the centroidal coordinate of the part with


area Ael. Solving for the product rcA gives
rcA = xelAel
Thus Eq. 1 can be written as
V = ( /2)(rcA)
= ( /2) xelAel

(Eq. 1 repeated)
(3)

and, to calculate V, we have to evaluate the sum xelAel.

1888

9.3 Theorems of Pappus and Guldinus Example 11, page 3 of 3


4

Calculate the centroidal distances and areas


and set up a table.
y

190 mm

260 mm

xel = 260 mm + 260 mm/2 = 390 mm

380 mm

190 mm
O

xel = 260 mm/2 = 130 mm y

O 260 mm

260 mm
Ael = (380 mm)(260 mm)
= 98 800 mm2

Region

Ael (mm2)

xel (mm)

Rectangle 1
Rectangle 2

98 800
49 400

130
390

xelAel (mm3)

12 844 000
19 266 000
xelAel = 32 110 000

Substituting the value of xelAelinto Eq. 3 gives the volume of the solid
V = 2 xelAel

32 110 000 mm3

= 50 438 270 mm3


= 0.0504 m3

O
260 mm

Rectangle 1

190 mm
x

Ans.

1889

260 mm

Rectangle 2
Ael = (190 mm)(260 mm)
= 49 400 mm2

9.3 Theorems of Pappus and Guldinus Example 12, page 1 of 4


12. Determine the mass of the steel V-belt pulley
shown. The density of the steel is 7840 kg/m3.

12 mm 10 mm 12 mm
4 mm

4 mm

The x axis is the axis


of revolution:

15 mm

(20 mm)/2 = 10 mm

100 mm 70 mm
2
20 mm
15 mm

Front view

Side view

1890

The crosshatched area is


the generating area for
the right half of the
pulley.

9.3 Theorems of Pappus and Guldinus Example 12, page 2 of 4


3 The second theorem of Pappus-Guildinus gives the
volume as
y

5 mm 12 mm

V = 2 rcA

(1)

4 mm
15 mm
(70 mm
10 mm

20 mm)/2 = 25 mm

where rc is the distance to the centroid of the generating


area, and A is the magnitude of the area. Thus we must
calculate the product rcA. This product may be found
by dividing the crosshatched area into composite parts
and then using the formula

Generating area divided into two


rectangles, and a triangle

rc =

yelAel
Ael

(2)
A = sum of element areas

where xel is the centroidal coordinate of the part with


area Ael. Solving for the product rcA gives
rcA = yelAel
Thus Eq. 1 can be written as
V = 2 (rcA)
=2

yelAel

(Eq. 1 repeated)
(3)

and, to calculate V, we have to evaluate the sum yelAel.

1891

9.3 Theorems of Pappus and Guldinus Example 12, page 3 of 4


4 Calculate the centroidal distances
and areas for the rectangles and
triangle, and set up a table.
y

5 mm 12 mm

Ael = (21 mm)(25 mm)


= 525 mm2
y

4 mm

Ael = (1/2)(12 mm)(15 mm)


= 90 mm2
y 12 mm

5 mm + 12 mm + 4 mm
= 21 mm

15 mm

15 mm
+
25 mm
10 mm

=
x

Generating area divided into


two rectangles, and a triangle

25 mm
10 mm

10 mm + 5 mm = 35 mm
x

Rectangle 1
yel = 10 mm + 25/2 mm
= 22.5 mm
yel = 35 mm + 15/3 mm
= 40 mm
+
yel = 35 mm + 15/2 mm
= 42.5 mm
Ael = (4 mm)(15 mm)
= 60 mm2

1892

Triangle
y
4 mm
15 mm

35 mm
x
Rectangle 2

9.3 Theorems of Pappus and Guldinus Example 12, page 4 of 4


5

Table

Region

Ael (mm2)

yel (mm)

Rectangle 1
Triangle
Rectangle 2

525
90
60

22.5
40.0
42.5

yelAel (mm3)

11 812.5
3 600.0
2 550.0
yelAel = 17 962.5

Substituting the value of yelAelinto Eq. 3 gives half the volume of the
V-belt pulley
V=

yelAel
17 962.5 mm3

= 112 861.7 mm3 = 0.000 112 861 7 m3


7

Total mass of the V-belt = total volume


= (2

density

0.000 112 861 7 m3)

7840 kg/m3

= 1.770 kg

Ans.

Double the half-volume.

1893

9.3 Theorems of Pappus and Guldinus Example 13, page 1 of 3


13. Determine the area of the surface of
revolution generated by rotating the curve
y = z4, 0 z 1 m, about the z axis.
y
1 The z axis is the axis of revolution.
2 The generating curve is

y = z4, 0 z

y
1m
1m
z
z
1m

1894

9.3 Theorems of Pappus and Guldinus Example 13, page 2 of 3


3

The first theorem of Pappus-Guildinus gives the area of


the surface of revolution as
A = 2 rcL

(1)

where rc is the distance to the centroid of the generating


curve, and L is the length of the curve. Thus we must
calculate the product rcL. This product may be found
by considering the equation for the centroidal
coordinate:

rc =

1m

rc
z
1m

yel dL
dL

L
y

where dL is an increment of curve length, and yel is the


coordinate of the increment. Solving for the product rcL
gives

dL
1m

rcL = yel dL

yel

Thus Eq. 1 can be written as

z
1m

A = 2 (rcL)
=2

yel dL

(2)

and, to calculate A, we have to evaluate a single integral,


yeldL.

1895

9.3 Theorems of Pappus and Guldinus Example 13, page 3 of 3


4

To evaluate the integral yel dL in Eq. 2, we use the


equation of the curve,
y = z4

(3)

to express dL as a function of z. Thus


dL =
=

(dy)2 + (dz)2
( dy )2 + 1 dz
dz

(4)

y
dL

and differentiating Eq. 3 gives

dz

dy = 4z3
dz
so Eq. 4 can be written as
dL =

yel = y = z4

(4z3)2 + 1 dz

(5)
1m

Substitute this expression for dL into Eq. 2


A=2

dy dL

yel dL

(Eq. 2 repeated)

5 Evaluating the integral by use of the integral


function key on a calculator gives

1
= 2 z4 (4z3)2 + 1 dz
0

A = 3.44 m2

1896

Ans.

9.3 Theorems of Pappus and Guldinus Example 14, page 1 of 3


14. Determine the volume of the solid of
revolution generated by rotating the curve
y = z4, 0 z 1 m, about the z axis.
y

The z axis is the axis of rotation.

2 The generating area is the


area under the y = z4 curve.
y
y = z4
x
1m

1m

z
1m

1897

9.3 Theorems of Pappus and Guldinus Example 14, page 2 of 3


y

The second theorem of Pappus-Guildinus gives the


volume as
V = 2 rcA

y = z4
C

rC

where rc is the distance to the centroid of the generating


area, and A is the magnitude of the area. Thus we must
calculate the product rcA. This product may be found
by considering the equation for the centroidal distance:

rc =

y = z4

(1)

yel dA
dA

where dA is an increment of area, and yel is the


coordinate of the centroid of the incremental region.
Solving for the product rca gives

(x, y)
yel

rcA = yel dA
Thus Eq. 1 can be written as
V = 2 (rcA)
=2

yel dA

(2)

and, to calculate V, we have to evaluate a single integral,


yel dA.

1898

9.3 Theorems of Pappus and Guldinus Example 14, page 3 of 3


4 To evaluate the integral yel dL in Eq. 2, we use the
equation of the curve,
y = z4
to express dA as a function of z. Thus
y

dA = y dz
= z4 dz
y = z4

(y, z)

yel = y/2 = z4/2

Substitute this expression for dA into Eq. 2


A=2

yel dz

(Eq. 2 repeated)

z
dz
1m

=2

z4/2)(z4) dz
(
0

Evaluating the integral by use of the integral function key


on a calculator gives
V = 0.349 m2

1899

Ans.

9.3 Theorems of Pappus and Guldinus Example 15, page 1 of 5


15. A pharmaceutical company plans to put a coating
0.01 mm thick on the outside of the pill shown.
Determine the amount of coating material required.
Radius = 20 mm

1 The x axis is the axis of revolution.


y
(1.5 mm)/2 = 0.75 mm

3.5 mm
x
7 mm

1.5 mm
2 The generating curve for half of the
pill surface is a composite curve
consisting of one straight line and a
circular arc. By symmetry, the total
surface area of the pill will be two
times the area generated by the curve
above.

1900

9.3 Theorems of Pappus and Guldinus Example 15, page 2 of 5


3 The first theorem of Pappus-Guildinus gives the area as
A = 2 rcL
y
0.75 mm

where rc is the distance to the centroid of the generating


curve and L is the length of the curve. Thus we must
calculate the product rcL. This product may be found
by dividing the curve into composite parts and then
using the formula

3.5 mm
rc =
x
Generating curve divided into a
straight line segment and an arc

(1)

yelLel
Lel

(2)
L

where xel is the centroidal coordinate of the part with


length Lel. Solving for the product rcL gives
rcL = yelLel
Thus Eq. 1 can be written as
A = 2 (rcL)
=2

(Eq. 1 repeated)

yelLel

(3)

and, to calculate A, we have to evaluate the sum yelLel.

1901

9.3 Theorems of Pappus and Guldinus Example 15, page 3 of 5


4 Calculate the centroidal distances and
the lengths, and set up a table.
y

y
0.75 mm

0.75 mm

yel = 3.5 mm

3.5 mm

=
x
Radius = 20 mm

3.5 mm

x
20 mm

Straight line

Arc
y
5 For the straight line, the length and
coordinate of the centroid are easily
calculated..
Lel = 0.75 mm

0.75 mm

yel = 3.5 mm
x
Straight line

1902

9.3 Theorems of Pappus and Guldinus Example 15, page 4 of 5


6 To calculate xelLel for the arc,
use the information shown below,
which has been taken from a table
of properties of common
geometric shapes.

A
20 mm

3.5 mm

7 For our particular arc, r = 20 mm and


= (1/2) sin-1(3.5/20)

2
B

= 0.08795 rad
Thus

Centroid Location
Length = 2 r

y
r

Length = 2 r
C

= 2(0.08795)

20 mm

= 3.5180 mm
rarc = (r sin
= (20 mm)(sin 0.08795)/(0.08795)

r sin

= 19.9742 mm

Circular arc segment


y

yel = rarc sin


A

19.9742 mm (sin 0.08795)

rarc

= 1.7545 mm

3.5 mm
yel
B

20 mm
Arc

1903

9.3 Theorems of Pappus and Guldinus Example 15, page 5 of 5


8 Table
yelLel (m2)

Region

Lel (m)

yel (m)

Line
Arc

0.75
3.5180

3.5
2.6250
1.7545
6.1723
yelLel = 8.7973

Substituting the value of yelAelinto Eq. 3 gives the area of the solid
A=2

yelLel
8.7973 mm2

= 110.5501 mm2
where a factor of 2 has been inserted to account for the fact that we took advantage
of symmetry to calculate the area of only half of the body.
Amount of coating material required = 110.5501 mm2

0.01 mm

= 1.106 mm3

Ans.

1904

9.3 Theorems of Pappus and Guldinus Example 16, page 1 of 4


16. Determine the volume of the funnel.
1

The y axis is the axis of revolution

10 mm

y
10 mm
5 mm

60 mm

60 mm
5 mm
70 mm

70 mm

5 mm

2.5 mm

2 The generating area is the


crosshatched area shown.

1905

9.3 Theorems of Pappus and Guldinus Example 16, page 2 of 4


3 The second theorem of Pappus-Guildinus gives the
volume as

y
10 mm

V = 2 rcA

5 mm

where rc is the distance to the centroid of the generating


area and A is the magnitude of the area. Thus we must
calculate the product rcA. This product may be found
by dividing the cross-hatched area into composite parts
and then using the formula

60 mm

rc =

70 mm

2.5 mm

(1)

Generating area divided into two


rectangles and two triangles

xelAel
Ael

(2)
A

where xel is the centroidal coordinate of the part with


area Ael. Solving for the product rcA gives
rcA = xelAel
Thus Eq. 1 can be written as
V = 2 (rcA)
=2

xelAel

(Eq. 1 repeated)
(3)

and, to calculate V, we have to evaluate the sum xelAel.

1906

9.3 Theorems of Pappus and Guldinus Example 16, page 3 of 4


4

To calculate xelAel, divide the area


into the sum of two rectangles and
two triangles, and set up a table.
y

10 mm

5 mm
xel = 2.5 mm

y 10 mm
xel = 5 mm + (5 mm)/3
= 6.6667 mm
5 mm
60 mm

60 mm

5 mm

Ael = (5 mm)(60 mm)


= 300 mm2

60 mm
Ael = (1/2)(5 mm)(60 mm)
= 150 mm2
O

=
70 mm

2.5 mm

Rectangle 1

x
Triangle 1

y
y

2.5 mm

2.5 mm

xel =1.25 mm
xel = 2.5 mm + (2.5 mm)/3
= 3.3333 mm
+

+
70 mm
Ael = (2.5 mm)(70 mm)
= 175 mm2
x
Rectangle 1

1907

70 mm
Ael = (1/2)(2.5 mm)(70 mm)
= 87.5 mm2
x
2.5 mm
Triangle 2

9.3 Theorems of Pappus and Guldinus Example 16, page 4 of 4


5 Table
Region

Ael (mm2)

xel (mm)

Rectangle 1
Rectangle 1
Triangle 1
Triangle 2

300
175
150
87.5

2.5
750
1.25
218.75
6.6667
1000.0050
3.3333
291.6637
xelAel = 2260.4187

xelAel (mm3)

Substituting the value of xelAelinto Eq. 3 gives the volume of the funnel
V=

xelAel

(Eq. 3 repeated)

2260.4187 mm3
= 14 200 mm3

Ans.

1908

9.3 Theorems of Pappus and Guldinus Example 17, page 1 of 3


17. A satellite dish is shaped in the form of a paraboloid of
revolution to take advantage of the geometrical fact that all
signals traveling parallel to the axis of the paraboloid are
reflected through the focus. Determine the amount, in m2,
of reflecting material required to cover the inside surface of
the dish.

The x axis is the axis of revolution


y
(0, 0.3)

0.3 m

( 0.2, 0)
Signals parallel
to axis of dish

2 The generating
curve is a parabola.

3 The general form for a parabola with vertex on the x


axis is
x = ay2 + b

0.3 m

Evaluating this equation at the points ( 0.2, 0) and


(0, 0.3) gives the equations
0.2 = a(0)2 + b
and

0.2 m

(1)

0 = a(0.3)2 + b

Solving for a and b and substituting back in Eq. 1


gives
x = 0.2222y2

1909

0.2

(2)

9.3 Theorems of Pappus and Guldinus Example 17, page 2 of 3


y

4 Apply the first theorem of Pappus-Guldinus


to calculate the surface area of the dish:
A = 2 rcL

dL

yel
rc
x

( 0.2, 0)
5

yel dL
dL

(0, 0.3)

dx

(3)

Thus we must calculate the product of the


length of the parabolic curve and its
centroidal coordinate. The product may be
found by considering the equation for the
centroidal coordinate:

rc =

dL

dy

To evaluate the integral yel dL in Eq. 4, we have to use


the equation of the parabola,
x = 0.2222y2

0.2

(Eq. 2 repeated)

Solving for the product rcL gives


to express dL as a function of y. Thus
rcL = yel dL

dL =

Thus Eq. 3 can be written as


=
A = 2 (rcL)

(dx)2 + (dy)2
dx
dy

+ 1 dy

(5)

Differentiating Eq. 2 gives


=2

yel dL

(4)

dx = 0.4444y,
dy
so Eq. 5 can be written as
dL =

1910

(0.4444y)2 + 1 dy

(6)

9.3 Theorems of Pappus and Guldinus Example 17, page 3 of 3


6

Noting that y = yel and also using Eq. 6 to replace dL in


Eq. 4 gives

A=2

=2

y
yel dL

y
(0, 0.3)

(Eq. 4 repeated)

(x, y)

0.3
y (0.4444y)2 + 1 dy
0

Using the integral function on a calculator gives


A = 0.284 m2

yel
rc

( 0.2, 0)

Ans

1911

9.3 Theorems of Pappus and Guldinus Example 18, page 1 of 6


18. Determine the amount of coffee that
the coffee mug holds when full to the
brim. The radius of the rounded corners
and the rim is 15 mm.

1 The y axis is the axis of revolution, and the


generating area is the cross-hatched area shown.
y

80 mm
Radius = 15 mm

90 mm

x
Radius = 15 mm

2 The distance rc to the centroid of the area can


be calculated by dividing the crosshatched
area into composite parts and using the
formula

rc =

xelAel
Ael

(1)

where xel is the centroidal coordinate of the


part with area Ael.

1912

9.3 Theorems of Pappus and Guldinus Example 18, page 2 of 6


3

However, we can save some work by noting that the second


theorem of Pappus-Guldinus involves the product of rc and the
generating area A:
V = 2 rcA

(2)

where V is the volume of solid of revolution.


Solving Eq. 1 for the product gives
rc Ael = xelAel
A
and substituting this result in Eq. 2 gives
V=2

xelAel

(3)

Thus we do not need to calculate rc and A independently


need only evaluate the first moment of the area, xelAel.

we

1913

9.3 Theorems of Pappus and Guldinus Example 18, page 3 of 6


4

To calculate xelAel, divide the area into the algebraic


sum of a rectangle, two squares, and two quarter circles,
and set up a table.
y
y
40 mm

For clarity, the areas near the rim and


rounded corner have been drawn
disproportionately large.
y

Radius = 15 mm

90 mm

Radius = 15 mm
5 Subtract quartercircular area
from square to
form area near
rim.

Square 1
6 Subtract square
from quartercircular area to
form rounded
corner.

x
Quarter circle 1

Rectangle

Square 2

1914

x
Quarter circle 2

9.3 Theorems of Pappus and Guldinus Example 18, page 4 of 6


7

Calculate the areas and centroidal coordinates of the


rectangle and the squares.
y
y
40 mm
40 mm
Radius = 15 mm

90 mm

40 mm

15 mm

Ael = (40 mm)(90 mm)


= 3600 mm2

15 mm

90 mm

xel = 40 mm + (15 mm)/2


= 47.5 mm
x

x
xel = 20 mm

Radius = 15 mm
Original area

Square 1

Rectangle
y

40 mm

Ael = (15 mm)2 = 225 mm2

15 mm

xel = 40 mm (15 mm)/2


= 32.5 mm

15 mm
x
Square 2

1915

9.3 Theorems of Pappus and Guldinus Example 18, page 5 of 6


8

Calculate the area and centroidal coordinates


of the quarter-circular regions..

9 A table of properties of planar regions


gives the information shown below.

y
Centroid location
y

40 mm

15 mm

xel = 40 mm + 15 mm
= 48.6338 mm
x
Quarter circle 1

A=

r2
4

6.3662 mm
C
4r
3

r
x

Quarter circular region

y
40 mm
15 mm
xel = 40 mm 15 mm
+ 6.3662 mm
= 31.3662 mm
x
Quarter circle 2

10 In our particular problem, r = 15 mm, so the


distance to the centroid is
4r = 4(15 mm) = 6.3662 mm4
3
3
Also, the area is
(15 mm)2
Ael =
= 176.7146 mm2
4

1916

9.3 Theorems of Pappus and Guldinus Example 18, page 6 of 6


11 Table
Region

Ael (mm2)

xel (mm)

xelAel (mm3)

Rectangle
Square 1
Quarter circle 1
Square 2
Quarter circle 2

3600
225
176.7146
225
176.7146

20
72 000
47.5
10 687.5
48.6338
8 594.3025
32.5
7 312.5
31.3662
5 542.8655
xelAel = 72 323.5630

12 Substituting the value of xelAelinto Eq. 3 gives the volume of the solid
V=

xelAel
72 323.563 0 mm3

= 454 000 mm3

Ans.

1917

9.3 Theorems of Pappus and Guldinus Example 19, page 1 of 6


19. Determine the capacity of the small bottle of lotion if the
bottle is filled half way up the neck.
5 mm

1 The y axis is the axis of revolution.

Half of the
neck is filled:
(5 mm)/2

2.5 mm
17.5 mm
17.5 mm
Radius = 20 mm
Radius = 20 mm

15 mm
15 mm

2 The generating area is the


crosshatched area shown.

1918

9.3 Theorems of Pappus and Guldinus Example 19, page 2 of 6


y

3 The second theorem of Pappus-Guildinus gives the


volume as

2.5 mm

V = 2 rcA

17.5 mm

Radius = 20 mm

(1)

where rc is the distance to the centroid of the generating


area, and A is the magnitude of the area. Thus we must
calculate the product rcA. This product may be found
by dividing the cross-hatched area into composite parts
and then using the formula

15 mm
rc =

x
Generating area divided into a rectangle,
two triangles and a circular sector

xelAel
Ael

(2)
A

where xel is the centroidal coordinate of the part with


area Ael. Solving for the product rcA gives
rcA = xelAel
Thus Eq. 1 can be written as
V = 2 (rcA)
=2

xelAel

(Eq. 1 repeated)
(3)

and, to calculate V, we have to evaluate the sum xelAel.

1919

9.3 Theorems of Pappus and Guldinus Example 19, page 3 of 6


4

Before the areas and centroidal coordinates can be


found, we first must find the distances and angles
shown below.
C

D
17.5 mm

2.5 mm

C
D

20 mm

17.5 mm
O

Radius = 20 mm
2

DC = (20 mm)

(17.5 mm)

15 mm

= 9.6825 mm
DOC = cos-1

17.5
20

= 28.9550

O
(4)

15 mm

20 mm

EB = (20 mm)2
= 13.2288 mm

1920

(15 mm)2

EOB = cos-1

15
20

= 41.4096

(5)

9.3 Theorems of Pappus and Guldinus Example 19, page 4 of 6


5 The areas and centroidal coordinates of
the rectangle and triangles can now be
calculated.

9.6825 mm
C

D
17.5 mm

2.5 mm

17.5 mm

Ael = (1/2)(17.5 mm)(9.6825 mm)


= 84.7219 mm2

C
D

Ael = (1/2)(15 mm)(13.2288 mm)


= 99.2160 mm2

xel = 9.6825 mm/3


= 3.2275 mm

xel = (13.2288 mm)/3


= 4.4096 mm

15 mm

B
13.2288 mm

x
15 mm
E

Triangle 2

Triangle 1

Radius = 20 mm

9.6825 mm
2.5 mm

C
D

C
xel = 9.6825 mm/2
= 4.8412 mm

Ael = (2.5 mm)(9.6825 mm)


= 24.2062 mm2

x
Rectangle

1921

+
O

B
Circular sector

9.3 Theorems of Pappus and Guldinus Example 19, page 5 of 6


6 To calculate the area and centroidal
coordinate of the circular sector, we can use
the information shown below, which has been
taken from a table of properties of planar
regions. Note that in the formula equals
half the angle of the arc.
y

= (180

DOC

EOB)/2

by Eq. 4

by Eq. 5

DOC = 28.9550
= (180

rc

y
D C

Centroid Location

28.9550

41.4096)/2

= 54.8177
r = 20 mm

A= r
r

EOB

90

= 54.8177 + 41.4096
C

= 6.2273
rc =

2r sin

xel

Circular sector region

90

2r sin
3
2(20 mm) sin 54.8177
3(54.8177
/180)

EOB = 41.4096
= 11.3903 mm
xel = rc cos

8 Ael = r2
=(

54.8177/180)

20 mm)2

= (11.3903 mm) cos 6.2273


= 11.3231 mm

= 382.6997 mm2

1922

9.3 Theorems of Pappus and Guldinus Example 19, page 6 of 6


9

Table
Region

Ael (mm2)

xel (mm)

xelAel (mm3)

Triangle 1
Triangle 2
Rectangle
Circular sector

84.7219
99.2160
24.2062
382.6997

3.2275
273.4399
4.4096
437.5029
4.8412
117.1871
11.3231
4333.3470
xelAel = 5161.4768

10 Substituting the value of xelAel into Eq. 3 gives the capacity of the bottle:
V=

xelAel
5161.4768 mm3

= 32 400 mm3

Ans.

1923

9.4 Hydrostatic Pressure on Submerged Surfaces

1924

9.4 Hydrostatic Pressure on Submerged Surfaces Procedures and Strategies, page 1 of 1


Procedures and Strategies for Solving Problems
Involving Hydrostatics

(Symbol for a free surface)

1. On a sketch of the surface in contact with the liquid,


show the pressure acting perpendicular to the surface. If the
surface is horizontal, then the pressure is uniform;
otherwise the pressure should be shown increasing linearly
with depth.
2. Calculate numerical values of the pressure by using the
equation p = h, where h is the depth below the free surface,
and is the specific weight of the liquid.
3. For a flat surface of constant width, calculate the
resultant and line of action of the pressure force by applying
the same techniques used for finding these quantities for
distributed forces acting on a beam.
4. For a curved surface of constant width, calculate the
resultant by considering a free-body diagram consisting of
the liquid bounded by the curved surface and by horizontal
and vertical planes. If the width is not constant, then you
must integrate to obtain the resultant.
5. Remember that pressure exerts an upward force on a
submerged surface if the liquid lies beneath the surface
(That is why ships float).

1925

9.4 Hydrostatic Pressure on Submerged Surfaces Problem Statement for Example 1


1. To prevent water pressure from pushing gate AB open, a small
extension, or lip, is provided at A. If the gate is 4-m wide
(measured perpendicular to the plane of the figure), determine the
force acting on the lip. The density of water is
103 kg/m3.

Hinge

5m

A
3m

1926

9.4 Hydrostatic Pressure on Submerged Surfaces Problem Statement for Example 2


2. Hydraulically-operated equipment is designed to transform a
relatively small input force into a much larger output force. For the
system shown, determine the weight W that can be supported by the
piston at B when a 200-N force is applied to the piston at A.
200 N

300-mm dia
200 mm
W
B

25-mm dia

Density of hydraulic fluid

1927

900 kg/m3

9.4 Hydrostatic Pressure on Submerged Surfaces Problem Statement for Example 3


3. During construction, gate AB is temporarily held
in place by the horizontal strut CD. Determine the
force in the strut, if the gate is 4-m wide.
A

2m
C

D
3m

Density of water

103 kg/m3

1928

9.4 Hydrostatic Pressure on Submerged Surfaces Problem Statement for Example 4


4. Determine the magnitude and line of action
of the resultant hydrostatic force acting on a 1-m
wide section of the seawall. Assume that the
density of sea water is = 1.02 103 kg/m3.
x
Vertex
A

Parabola
12 m

B
10 m

1929

9.4 Hydrostatic Pressure on Submerged Surfaces Problem Statement for Example 5


5. Determine the distance h for which the gate is just about to open.
Neglect the weight of the gate. The specific weight of the fluid is .
A
2 ft
B
C
h
D

6 ft

1930

9.4 Hydrostatic Pressure on Submerged Surfaces Problem Statement for Example 6


6. Concrete is poured into the open top of a form to produce a
ramp. If the ramp is 0.8-m wide, determine the minimum mass m
needed to keep the form from lifting off the ground.
Open top
0.7 m

0.5 m
B

0.4 m
A

D
Open bottom

Mass of form 60 kg
Density of concrete

2.4

103 kg/m3

1931

9.4 Hydrostatic Pressure on Submerged Surfaces Problem Statement for Example 7


7. Determine the minimum weight W of gate BC required to keep the gate closed. The
gate is 2-ft wide and of uniform density. The specific weight of water is
62.4 lb/ft3.
A

2.5 ft
B

3 ft
C

4 ft

1932

9.4 Hydrostatic Pressure on Submerged Surfaces Problem Statement for Example 8


8. Determine the magnitude and line of action of the resultant hydrostatic force
acting on the semicircular end of the tank. The tank is filled to the top with water.
The specific weight of water is
62.4 lb/ft3.
B

Cables

Radius

4 ft

1933

9.4 Hydrostatic Pressure on Submerged Surfaces Problem Statement for Example 9


9. Determine the magnitude and line of action of the resultant
hydrostatic force acting on the end of the tank, which is filled to
the top with water. The density of water is
103 kg/m3.
C

1.1 m
0.55 m

D
1m
E

1.5 m

1934

9.4 Resultants of Hydrostatic Pressure Forces Example 1, page 1 of 3


1. To prevent water pressure from pushing gate AB open, a small
extension, or lip, is provided at A. If the gate is 4-m wide
(measured perpendicular to the plane of the figure), determine the
force acting on the lip. The density of water is
103 kg/m3.

Hinge

5m

A
3m

1935

9.4 Resultants of Hydrostatic Pressure Forces Example 1, page 2 of 3

5m
A

pAB

2 The pressure must be pAB at all other


points at the same elevation. Since
these points lie 5 m below the free
surface of the water,

Since AB is horizontal, the pressure


pAB acting on it is uniform.

pAB

5m

(103 kg/m3)(9.81 m/s2)(5 m)


49.05

103 (kg m/s2)/m2

49.05

103 N/m2

49.05 kN/m2
49.05 kPa

1936

9.4 Resultants of Hydrostatic Pressure Forces Example 1, page 3 of 3


Free-body diagram of gate AB

Force acting on lip, FA

By

Forces from hinge

Bx

Convert the pressure to a force per length by


multiplying by the width:
w

pAB

width of gate

w
(49.05 kPa)

4m

3m
196.20 kN/m
Equilibrium equation for gate AB

MB

FA(3 m)

(196.20 kN/m)(3 m)(3 m/2)

resultant of w
Moment arm of
pressure resultant

Solving gives
FA

294 kN

1937

Ans.

9.4 Resultants of Hydrostatic Pressure Forces Example 2, page 1 of 3


2. Hydraulically-operated equipment is designed to transform a
relatively small input force into a much larger output force. For the
system shown, determine the weight W that can be supported by the
piston at B when a 200-N force is applied to the piston at A.
200 N

300-mm dia
200 mm
W
B

25-mm dia

Density of hydraulic fluid

1938

900 kg/m3

9.4 Resultants of Hydrostatic Pressure Forces Example 2, page 2 of 3


200 N
1 The pressure on the underside of the piston at A is
Force
Area of piston

200 N
(0.025 m)2

407 437 N/m2


407.437 kN/m2
407.437 kPa

200 mm
W
B
Pressure p
2

The pressure 200 mm below A equals the pressure at A


plus an additional amount caused by the weight of the fluid
p = 407.437 kPa + (900 kg/m3)(9.81 m/s2)(200 mm)
407.437 kPa + 1.766 kPa

0.2 m

409.203 kPa

25-mm dia

The pressure at two points at


the same elevation in a fluid
at rest must be equal.

1939

3 This is a typical result for


hydraulic equipment. The
effect of the weight of the fluid
is small compared to the
pressure caused by the pistons.

9.4 Resultants of Hydrostatic Pressure Forces Example 2, page 3 of 3


5

Free-body diagram of piston B


W

409.203 kPa

300-mm dia

Equilibrium equation
Fy

W + (409.203 kPa)[

7
4

(0.3 m)2]

28.9 kN
200 N

Solving gives
W

28.9 kN

Thus the 200-N input force produces a 28.9 kN


output force, a multiplication factor of

Ans.

1940

28.9 103 N
200 N

144

9.4 Resultants of Hydrostatic Pressure Forces Example 3, page 1 of 4


3. During construction, gate AB is temporarily held
in place by the horizontal strut CD. Determine the
force in the strut, if the gate is 4-m wide.
A

2m
C

D
3m

Density of water

103 kg/m3

1941

9.4 Resultants of Hydrostatic Pressure Forces Example 3, page 2 of 4


A
1
2m
C

The water pressure varies linearly from 0 to pB, where


pB

(2 m + 3 m)

(103 kg/m3)(9.81 m/s2)(5 m)

D
3m

pB

49.05 kPa

B
A

2m
C

wB
3m

wB

Convert the pressure to a force per length by


multiplying by the width, 4 m:
pB

4m

(49.05 kPa)
196.20 kN/m

1942

4m
(1)

9.4 Resultants of Hydrostatic Pressure Forces Example 3, page 3 of 4


3

Resultant of distributed load


area under load curve
A

area of triangle
(1/2)(196.2 kN/m)(2 m + 3 m)

2m
490.5 kN
D

3m
wB

196.20 kN/m

B
4

The resultant passes through the centroid of the


triangle, or one-third of the distance from the
base to the opposite vertex:
(1/3)(2 m + 3 m)

1943

1.667 m above point B

9.4 Resultants of Hydrostatic Pressure Forces Example 3, page 4 of 4


5

Free-body diagram of gate AB

MB

Equilibrium equation
(FCD)(3 m)

FCD
3m

1.667 m
Bx

solving gives

FCD

490.5 kN

(490.5 kN)(1.667 m)

273 kN

Ans.

This is a large force. Most likely more than one strut would
be used.

By

1944

9.4 Resultants of Hydrostatic Pressure Forces Example 4, page 1 of 10


4. Determine the magnitude and line of action
of the resultant hydrostatic force acting on a 1-m
wide section of the seawall. Assume that the
density of sea water is = 1.02 103 kg/m3.
x
Vertex
A

Parabola
12 m

B
10 m

1945

9.4 Resultants of Hydrostatic Pressure Forces Example 4, page 2 of 10


A

Pressure

C
1

Integration can be used to calculate the


resultant of the pressure forces acting on
the curved wall, but it is easier to consider
a free body consisting of a portion of the
water behind the sea wall.
B

1946

9.4 Resultants of Hydrostatic Pressure Forces Example 4, page 3 of 10


2

Free-body diagram of region of ABC


4

weight of water

10 m
C

A
5

Rx

12 m

wB

Force per length, wB

Ry
B

(pressure at B)
( g
(1.02

(1-m distance normal to plane of figure)

12 m)(1 m)
103 kg/m3)(9.81 m/s2)(12 m)(1 m)

120.1 kN/m

1947

Components of force from


seawall acting on water

9.4 Resultants of Hydrostatic Pressure Forces Example 4, page 4 of 10


6

The magnitude of the weight, W, can be calculated by using information


from a table of properties of planar regions as shown below:
Vertex
Centroid Location
y
a

10 m

2h/5
C

2ah
Area = 3
Vertex

12 m

Semiparabolic region

In our particular example, the length of the side


opposite the vertex 12 m a, and h 10 m, so
Area

2ah/3
2(12 m)(10 m)/3
80 m2

(1)

1948

9.4 Resultants of Hydrostatic Pressure Forces Example 4, page 5 of 10


10 W

Free-body diagram of region ABC (repeated)

weight
g

10 m

area

103 kg/m3)(9.81 m/s2)(80 m2)(1 m)

(1.02

width

800.5 kN

Rx

12 m

Ry

Fx

0:

Fy

0: Ry

Rx + 720.6 kN
800.5 kN

0
0

(2)
(3)

Solving gives

B
wB

11 Equilibrium equations

Rx

720.6 kN

Ry

800.5 kN

120.1 kN/m

resultant of pressure forces

12 Resultant
R

area of w-diagram

800.5 kN

(1/2)(12 m)(120.1 kN/m)


720.6 kN
720.6 kN

(720.6 kN)2 + (800.5 kN)2


1.077 MN

1949

9.4 Resultants of Hydrostatic Pressure Forces Example 4, page 6 of 10


13 R is the force of the seawall on
the water. The force of the
water on the seawall is equal
and opposite.

R = 1.077 MN

Ans.
720.6 kN

800.5 kN
B

1950

9.4 Resultants of Hydrostatic Pressure Forces Example 4, page 7 of 10


14 To determine the line of action of the resultant force,
consider a free-body diagram of region ABC again.
10 m
Line of action
of resultant, R

xC
C

A
Vertex

16 The 800.5 kN weight of the water acts


through the centroid of the semiparabolic
area. The centroidal distance xc can be
calculated using a table giving geometric
properties of common areas.

800.5 kN
8m
12 m
Resultant, R
800.5 kN
B

720.6 kN

15 Resultant of triangular
distribution of pressure
equals 720.6 kN and acts
8 m [= (2/3)(12 m)] from
point C.

1951

9.4 Resultants of Hydrostatic Pressure Forces Example 4, page 8 of 10


17 To locate the centroid, we again make use
of a table of properties of planar regions as
shown below:

18 In our particular example, the length of the straight side


adjacent to the vertex 10 m h, and so the centroid is
located 4 m [ = (2/5)(10)] to the right of of C.

Centroid Location
y
a

10 m

xc = 4 m

2h/5
C

2ah
Area = 3
Vertex

Semiparabolic region

x
12 m

1952

Vertex

9.4 Resultants of Hydrostatic Pressure Forces Example 4, page 9 of 10


19 The free-body diagram of region ABC can be re-drawn
with the known distances specified.

Line of action
of resultant, R

xC = 4 m
C
D

A
Vertex

800.5 kN

8m
Resultant, R
720.6 kN
800.5 kN
B

720.6 kN

20 A force can be considered to act


anywhere along its line of action
(Principle of transmissibility).
We can simplify our calculations
by considering the resultant R to
act at point D where the line of
action intersects the left edge of
the free body.

1953

9.4 Resultants of Hydrostatic Pressure Forces Example 4, page 10 of 10


xC = 4 m

800.5 kN
A
Vertex

C
d

R
D

720.6 kN

Line of action
of resultant, R

21

8m

MC = (720.6 kN)(8 m)

(720.6 kN)(d)

(800.5 kN)(4 m) = 0

Solving gives the vertical distance below point C to the line of action of
the resultant:
d = 3.56 m

1954

Ans.

9.4 Resultants of Hydrostatic Pressure Forces: Example 5, page 1 of 6


5. Determine the distance h for which the gate is just about to open.
Neglect the weight of the gate. The specific weight of the fluid is .
A
2 ft
B
C
h
D

6 ft

The value of h will not depend on the gate width (distance


measured perpendicular to the plane of the figure) because the
width would cancel out of the equation for the sum of moments
about the support C. Accordingly we will base our calculations
on a 1-ft width of gate.

1955

9.4 Resultants of Hydrostatic Pressure Forces: Example 5, page 2 of 6


2

Free-body diagram of gate (distributed forces)

6 ft
A
Cy
3

2 ft

Pressure:

C
B

pB

(2 ft)

pD

(2 ft + h)

Cx

h
wB

wD

D
FD (force from ledge in ground)

1-ft width
4

Force per length:


wB

pB

(1 ft)

(2 )

wD

pD

(1 ft)

(2 + h)

2
1

(2 + h)

1956

9.4 Resultants of Hydrostatic Pressure Forces: Example 5, page 3 of 6


5

Resultant force on AB

Area

(1/2)(2 ft)(2 )
2

A
2 ft

2
wB

(2/3) ft
6

Resultant acts through centroid of


triangle (at a distance equal to one-third
of the height of the triangle)
D

1957

9.4 Resultants of Hydrostatic Pressure Forces: Example 5, page 4 of 6


7

Resultant force on BC
3 ft

6 ft

3 ft

C
wB = 2

12
D

D
Area

12

1958

9.4 Resultants of Hydrostatic Pressure Forces: Example 5, page 5 of 6


8

Resultant force on CD

wB

C
B
h

+
wD
Trapezoid

D
(2+h)

h
triangle + rectangle

Area

(1/2)(h )(h)
Area

h2 /2
A

h/2
2 h
(h2

1959

2h/3

9.4 Resultants of Hydrostatic Pressure Forces: Example 5, page 6 of 6


Free-body diagram (resultant forces)

3 ft

Cy
2

C
B

Cx
2 h

2/3 ft
12

h/2

2h/3

(h2

10 FD

D
0, since gate is just about to open

11 Equilibrium equation

MC

0: ( 2 )(2/3 ft)

(12 )(3 ft) + (2 h)(h/2) + (h2 /2)(2h/3)

Solving gives
h

4.0 ft

Ans.

1960

9.4 Resultants of Hydrostatic Pressure Forces: Example 6, page 1 of 3


6. Concrete is poured into the open top of a form to produce a
ramp. If the ramp is 0.8-m wide, determine the minimum mass m
needed to keep the form from lifting off the ground.
Open top
0.7 m

0.5 m
B

0.4 m
A

D
Open bottom

Mass of form 60 kg
Density of concrete

2.4

103 kg/m3

1961

9.4 Resultants of Hydrostatic Pressure Forces: Example 6, page 2 of 3


B

We can solve the problem by finding the


resultant of the pressure of the concrete acting
on part AB of the form. But we would have to
compute the length AB and also compute the
angle the resultant makes with the vertical.
Although the problem can be done in this
manner, it is easier to consider a free body
consisting of the form together with the
concrete. Thus let's first find the resultant
force of the pressure of the floor pushing up
on the concrete.

Pressure at D

0.7 m

0.5 m
B

2.4 103 kg/m3)(9.81 m/s2)


(0.4 m)
= 9.418

0.4 m
A

103 N/m2

Distributed load (force per length) =


pressure

width of gate

(9.418

103N)(0.8 m)

7.534

103 N/m

Resultant force = (7.534 103 N/m)


(0.7 m + 0.5 m)
= 9.041

1962

103 N

9.4 Resultants of Hydrostatic Pressure Forces: Example 6, page 3 of 3


3

Free-body diagram of form together with concrete

0.7 m
4

0.5 m

Weight of form

(60 kg)(9.81 m/s )

mg

= 588.6 N

0.4 m
D

A
Reaction from floor = 9.041

Equilibrium equation
Fy

0: 9.041

10 N

5
mg

588.6 N

.404

10 N

Setting g equal to 9.81 m/s2 and solving gives


m = 209 kg

103 N

Weight of concrete

1963

volume

2.4 103 kg/m3)(9.81 m/s2)[(1/2)(0.7 m)


(0.4 m)(0.8 m) + (0.5 m)(0.4 m)(0.8 m)]
6.404

Ans.

cg

103 N

9.4 Resultants of Hydrostatic Pressure Forces: Example 7, page 1 of 5


7. Determine the minimum weight W of gate BC required to keep the gate closed. The
gate is 2-ft wide and of uniform density. The specific weight of water is
62.4 lb/ft3.
A

2.5 ft
B

3 ft
C

4 ft

1964

9.4 Resultants of Hydrostatic Pressure Forces: Example 7, page 2 of 5


1

If we attempt to solve the problem by considering the pressure acting directly


on BC, then we have to calculate the length and angle of inclination of BC.

1965

9.4 Resultants of Hydrostatic Pressure Forces: Example 7, page 3 of 5


A
2 Although the problem can be solved in this
manner, it is easier to consider a free body
consisting of gate BC together with the water
under BC.

2.5 ft
B

wB

3 ft
wC

C
wC

Pressures:
at B,
pB

2.5 ft

2.5

pC

(2.5 ft + 3 ft)

at C,
5.5

Distributed loads (force per length):


wB

pB

(width of gate)

(2.5 )(2 ft)

wC

pC

(width of gate)

(5.5 )(2 ft)

11

1966

(1)

9.4 Resultants of Hydrostatic Pressure Forces: Example 7, page 4 of 5


4

Free-body diagram of gate BC


together with the water under BC
4 ft
6

4/3 ft

Weight of water

2 ft
5

Volume
(1/2)(4 ft)(3 ft)(2 ft)

Weight of gate
By

12
(Acts through centroid of triangle)

B
Bx
W
3 ft

+
C
wB
wC

The force from the ground at C acting


on the gate is zero since the gate is
about to lose contact with ground.

wC

wB

The trapezoidal distributed load from the fluid


acting on the right side of the free body can be
considered to be the sum of a rectangular and
triangular distributed load.

1967

9.4 Resultants of Hydrostatic Pressure Forces: Example 7, page 5 of 5

Equation of equilibrium
MB

0: W(2 ft) + 12 (4/3 ft)

4wC(2 ft)

Resultant force from


uniform load on bottom

3wB(3/2 ft)

(1/2)(wC

Resultant force from


uniform load on right side

10 Substituting wB 5 (Eq. 1), wC 11 (Eq. 2), and


62.4 lb/ft3 into Eq. 3 and solving gives
W

3,510 lb

Ans.

1968

wB)(3 ft)(2/3

3 ft) = 0

(3)

Resultant force from


triangular load on right side

9.4 Resultants of Hydrostatic Pressure Forces: Example 8, page 1 of 4


8. Determine the magnitude and line of action of the resultant hydrostatic force
acting on the semicircular end of the tank. The tank is filled to the top with water.
The specific weight of water is
62.4 lb/ft3.
B

Cables

Radius

4 ft

1969

Because the end of the tank is not of uniform


width, we have to use integration to compute the
magnitude of the resultant.

9.4 Resultants of Hydrostatic Pressure Forces: Example 8, page 2 of 4


2

Pressure force on elemental area dA


y

(x, y)
5

Distance
y (Insert a minus sign
to get a positive distance y is
negative in the region shown)

dy

x2 + y2

42

x
3
6

dA

Differential force:
dR

pressure

area

[ ( y)]

[2 42

2x dy
2 42

y2 dy]

dR
0
4

[ ( y)] [2 42

2,662 lb
Use = 62.4 lb/ft3and evaluate
with the integral function on a
calculator.

1970

y2 dy

Resultant force:
R

Solve for x
and substitute

y2 dy]
Ans.

9.4 Resultants of Hydrostatic Pressure Forces: Example 8, page 3 of 4


9

To determine the line of action of the resultant, equate the moment of the
resultant to the integral of the moment of the differential force, dR.
y

x
Distance

x
y

10 By symmetry, the line of action


passes through the y axis.

2,662 lb

11 Moment of R about x axis = Integral of moment of dR about x axis


R

( y)

2,662 lb

dR

(1)

or
0
4

( y) [ 2 y 42

y2 ] dy

62.4 lb
Evaluating the integral by using the integral function on
a calculator and then solving for d gives
d

2.36 ft

Ans.

1971

dR

9.4 Resultants of Hydrostatic Pressure Forces: Example 8, page 4 of 4


12 We could have saved some work by using a table of moments of
inertia as follows: Eq. 1 is
R

( y)

dR
pressure

(Eq.1 repeated)
dA

[ ( y)] dA
y2dA

Ix, the moment of inertia of the


area about the x axis.

1972

9.4 Resultants of Hydrostatic Pressure Forces: Example 9, page 1 of 8


9. Determine the magnitude and line of action of the resultant
hydrostatic force acting on the end of the tank, which is filled to
the top with water. The density of water is
103 kg/m3.
C

1.1 m
0.55 m

D
1m
E

1.5 m

1973

Because the end of the tank is not of


uniform width, we have to use integration
to compute the magnitude of the resultant.

9.4 Resultants of Hydrostatic Pressure Forces: Example 9, page 2 of 8


2

Let's divide the end of the tank into three regions.

0.55 m

0.55 m
1.5 m

1.1 m
+

1m
E

E
Region 1

Region 2

Because of symmetry, Regions 1 and 3 have the same resultant


force. Thus we need to consider only one of the regions. Let's
choose Region 3.

1974

Region 3

9.4 Resultants of Hydrostatic Pressure Forces: Example 9, page 3 of 8


4

Calculation of resultant of pressure on Region 3


y

Depth below free surface


1.1 m

0.55 m

C
dy

x
(x, y)

1.1 m

Pressure force acting on dA: dR3

y
x
Equation of line:
y

Elemental area:
dA

2x

dy

(y/2) dy

1.1 x
0.55

Solve for x and


substitute in the
expression for dA.

Differential force
dR3

pressure
[ g(1.1

1975

area
y)]

[(y/2) dy]

(1)

9.4 Resultants of Hydrostatic Pressure Forces: Example 9, page 4 of 8


7

Resultant force for Region 3


R3

Eq. 1

dR3
g

1.1
0

(1.1

y) (y/2) dy

Use the integral function


on a calculator to evaluate

0.1109 g

(2)

We also need to determine the line of action of


R3. We can do this by equating the moment
produced by R3 acting along its line of action
to the integral of the moment produced by dR3.

1976

9.4 Resultants of Hydrostatic Pressure Forces: Example 9, page 5 of 8


9

11 Moment of differential force about FC

Moment of R3 about FC

(1.1 m
C

y)

d3
(x , y)
R3

10 Vertical distance to
line of action of R3

1.1 m

dR3

x
12 Equating moments about the top edge FC gives

R3

d3 = (1.1

y) dR3
Eq. 1

0.1109 g by Eq. 2
1.1
0

(1.1

y) [ g(1.1

y) (y/2) dy]

Evaluating the integral and solving for d3 gives


d3

1977

0.5501 m

(3)

9.4 Resultants of Hydrostatic Pressure Forces: Example 9, page 6 of 8


13 Resultant force acting on Region 2
1.5 m
Distributed load acting on Region 2

1.1 m + 1.0 m = 2.1 m

E
Pressure at point E

wE

(2.1 m) g

Distributed force (force per length), wE

pressure
2.1 g

width

1.5 m

3.15 g
14 Resultant force
R2

(1/2)(2.1 m)(3.15 g)
3.3075 g

(4)

1978

9.4 Resultants of Hydrostatic Pressure Forces: Example 9, page 7 of 8


15 Line of action of R2 passes through centroid of triangle.
d2

(2/3)(2.1)

1.4 m

(5)

2.1 m

16 Total force acting on end of tank

wE

R = R1 + R2 + R3
R3, by symmetry
R2 + 2R3
by Eq. 4

by Eq. 2

3.3075 g + 2(0.1109 g)
3.5293 g
103 kg/m3 and g

Using
R

1979

34.6 kN

(6)
9.81 m/s2 gives
Ans.

9.4 Resultants of Hydrostatic Pressure Forces: Example 9, page 8 of 8


y

d1 = d3 = 0.5501 m

d2 = 1.4 m

R2 = 3.3075 g

17 Determine the line of action of the resultant R by


equating the moment of R to the sum of moments of R1,
R2, and R3:
MBC: Rd

R 1d1 + R2d2 + R3d3

Substituting the R, R1, d1, R2, d2, R3, and d3 values


from the figures above into this equation and solving
gives
d = 1.347 m

Ans.

1980

+
R3 = 0.1109 g

R1 = R3 = 0.1109 g
R

d3 = 0.5501 m

10. Inertia Properties of Plane Areas

1981

10.1 Moments of Inertia by Integration

1982

10.1 Moments of Inertia by Integration Procedures and Strategies, page 1 of 1


Procedures and Strategies for Solving Problems
Involving Moments of Inertia by Integration.

y = f(x)
(x, y)

To calculate the moment of inertia Ix about the x axis, use


one of the following two approaches:
1. If the curve bounding the planar region is given as a
function of x, y = f(x), use the formula
Ix = dIx
where, dIx = (1/3)y3dx = (1/3) f(x) 3dx is the moment of
inertia (about the x axis) of a vertical rectangular strip of
length y (= f(x)) and width dx.

x
dx
y

2. If the curve bounding the planar region is given as a


function of y, x = g(y), use the formula
Ix = y2 dA
where the differential dA is the area of a horizontal strip
of length x and width dy.

dy

(x, y)

x = g(y)

To calculate Iy, follow the same procedure described


above but with all x's replaced by y's and all original y's
replaced by x's.
x

1983

10.1 Moments of Inertia by Integration Problem Statement for Example 1


1. Determine the moment of inertia of the rectangle
about the x axis, which passes through the centroid C.
y
b

h/2
C

h/2

1984

10.1 Moments of Inertia by Integration Problem Statement for Example 2


2. Determine the moment of inertia of the rectangle
about its base, which coincides with the x axis.
y

1985

10.1 Moments of Inertia by Integration Problem Statement for Example 3


3. Determine the moment of inertia of the
right triangle about the x axis.
y

x
b

1986

10.1 Moments of Inertia by Integration Problem Statement for Example 4


4. Determine the moments of inertia of the area
bounded by an ellipse about the x and y axes.

x2 y2
+ =1
a2 b2

1987

10.1 Moments of Inertia by Integration Problem Statement for Example 5


5. Determine the moments of inertia of the
crosshatched area about the x and y axes.
y

4 ft

y=4

x
2 ft

1988

x2

10.1 Moments of Inertia by Integration Problem Statement for Example 6


6. Determine the moment of inertia of the
crosshatched area about the y axis.
y

1.156 m

x = 2y6 50y5

Scales on
the x and y
axes are
not the
same.

x
100 m

1989

y3 + 100

10.1 Moments of Inertia by Integration Problem Statement for Example 7


7. Determine the moment of inertia of the
crosshatched area about the x axis.
y

y=9

x2

9m

y = 3x

x2
x

3m

1990

10.1 Moments of Inertia by Integration Problem Statement for Example 8


8. Determine the moment of inertia of the
crosshatched area about the y axis.
y
1/4 m

4m

xy = 1

y = 2x

1/2 m

4m

1m

1m
x

1991

10.1 Moments of Inertia by Integration Problem Statement for Example 9


9. Determine the moment of inertia of the
crosshatched area about the x axis.
y
2
y = 10e x

2
y = 10 e x

1m
1.5 m

1992

10.1 Moments of Inertia by Integration Problem Statement for Example 10


10. Determine the moment of inertia of the
crosshatched area about the y axis.
y
2 in.

1 in.

4 in.

y = 3x
3 in.

y = x2

1993

10.1 Moments of Inertia by Integration Problem Statement for Example 11


11. Given that the centroid C of the area bounded by a
quarter-circle lies a distance 4a/(3 above the base of the
quarter-circle, determine the moment of inertia of the
crosshatched area about an axis xc through the centroid.
y

yc

x2 + y2 = a2

xc
4a
3
x
a

1994

10.1 Moments of Inertia by Integration Example 1, page 1 of 2


1. Determine the moment of inertia of the rectangle
about the x axis, which passes through the centroid C.
y
b

h/2
C

h/2

1995

10.1 Moments of Inertia by Integration Example 1, page 2 of 2


y

1 We want to evaluate
Ix = y2 dA
b
where the differential element dA is located a
distance y from the x axis (y must have the same
value throughout dA).

dy
h/2
y
C

2 dA = area of rectangle
=b

dy

h/2
h 2

3 Ix =

-h 2

y2dA

h 2

y2(b

-h 2

dy)

4 Limits of integration
=

by3
3

h 2
-h 2

1996

bh
12

Ans.

10.1 Moments of Inertia by Integration Example 2, page 1 of 2


2. Determine the moment of inertia of the rectangle
about its base, which coincides with the x axis.
y

1997

10.1 Moments of Inertia by Integration Example 2, page 2 of 2


y

1 We want to evaluate
Ix = y2 dA
b

where the differential element of area dA is located a distance y from the


x-axis (y must have the same value throughout the element dA).

dy

2 dA = area of rectangle

h
y

=b

3 Ix = y2dA
h

4 Limits of integration

1998

y2(b

by3
3
bh3
3

dy)

h
0

Ans.

dy

10.1 Moments of Inertia by Integration Example 3, page 1 of 3


3. Determine the moment of inertia of the
right triangle about the x axis.
y

x
b

1999

10.1 Moments of Inertia by Integration Example 3, page 2 of 3


y

1 We want to evaluate
x

Ix = y2 dA
(x,y)

where y has the same value throughout


differential element dA.

dy
4 Equation of line

y = slope

x + intercept

h
= ( )x+h
b
x
5

Solve for x to get

x= ( b )y+b
h
so,

2 dA = area of rectangle
= x dy

dA = x

dy

= ( b y + b) dy
h

3 Since we will integrate with respect to y,


we must replace x by a function of y.

2000

(1)

10.1 Moments of Inertia by Integration Example 3, page 3 of 3

6 Ix = y2 dA

7 Limits of integration are from


bottom of triangle to top.

h
0

=b

= b[

y2 ( b y + b) dy
h
h
0

y3 2
+ y ) dy
h

y4 y3 h
+ ]
4h 3 0

1
= bh3[ 1 + ]
4 3
bh3
= 12

Ans.

2001

10.1 Moments of Inertia by Integration Example 4, page 1 of 3


4. Determine the moments of inertia of the area
bounded by an ellipse about the x and y axes.

x2 y2
+ =1
a2 b2

2002

10.1 Moments of Inertia by Integration Example 4, page 2 of 3


1 We want to evaluate
Ix = y2 dA
where y has the same value throughout the
differential element dA.

2 x = half the length of the


differential element,
so 2x = entire length

y
x

3 dA = area of rectangle
= 2x dy

(x, y)

dy
y
x

4 Since we will integrate with


respect to y, we must replace x
by a function of y

5
x2 y2
+ =1
a2 b2

6 Since x locates a point to the right of the y axis, choose


the plus sign:

Solve for x to get


x=

a 1

(y/b)2

x = +a 1

2003

(y/b)2

(1)

10.1 Moments of Inertia by Integration Example 4, page 3 of 3


7 Using Eq. 1 in the expression for dA gives:
dA = 2x

dy

= 2a 1

(y/b)2

dy
8 Ix = y2 dA

9 Limits of integration from


bottom to top of ellipse

b
-b

y2(2a 1

(y/b)2 ) dy

ab
4

Ans.

11 To calculate Iy, use symmetry in the following way: in all the


above equations, replace x's by y's, original y's by x's, a's by b's
and original b's by a's. Then the result would be
Iy =

a3b
4

Ans.

2004

10 Evaluate the integral either with a


calculator that does symbolic
integration or use a table of integrals.

10.1 Moments of Inertia by Integration Example 5, page 1 of 4


5. Determine the moments of inertia of the
crosshatched area about the x and y axes.
y

4 ft

y=4

x
2 ft

2005

x2

10.1 Moments of Inertia by Integration Example 5, page 2 of 4


y

We want to evaluate
I x = y2 dA
where y has the same value throughout the differential element dA.

(x,y)
dy
2 dA = area of rectangle
= x dy
4 ft
3 Since we will integrate with respect to y,
we must replace x by a function of y.

y
y=4

x2

4 Solving for x to get


x=

2 ft

5 Since x locates a point to the right of the y axis, choose the plus sign:
x=+ 4

2006

(1)

10.1 Moments of Inertia by Integration Example 5, page 3 of 4


6 Using Eq. 1 in the expression for dA gives
dA = x
=

dy
4

y dy

7 Ix = y2 dA
8 y = 4 at the top of
the crosshatched
area
9 Use the integral function
on your calculator

y2 4

= 19.50 ft4

y dy

Ans.

2007

10.1 Moments of Inertia by Integration Example 5, page 4 of 4


10 Next we want to evaluate
y

Iy = x2 dA
where x has the same value throughout the differential element dA. Thus, we
choose a vertical rectangular strip.

11 dA = area of rectangle
=y
= (4

4 ft

y=4

x2

x
dx
2 ft

12

Iy = x2 dA

2
0

x2(4

x2) dx = 4.27 ft4

2008

Ans.

dx
x2) dx

10.1 Moments of Inertia by Integration Example 6, page 1 of 3


6. Determine the moment of inertia of the
crosshatched area about the y axis.
y

1.156 m

x = 2y6 50y5

Scales on
the x and y
axes are
not the
same.

x
100 m

2009

y3 + 100

10.1 Moments of Inertia by Integration Example 6, page 2 of 3


1

We want to evaluate
Iy = x2 dA

where x has the same value throughout the differential


element dA. Thus, we choose a vertical rectangular strip.
2

dA = area of rectangle
=y

3
y

dx

But this approach won't work because we


can't express y as a function of x.

x = 2y6
x
100 m

2010

50y5

y3 + 100

10.1 Moments of Inertia by Integration Example 6, page 3 of 3


4

An alternative approach is to use a horizontal


rectangular strip and employ the equation for the
moment of inertia of a rectangle about its base (BB)
:
3
IBB = bh
3
(1)
h
B
b

Applying Eq.1 to the differential element


gives the differential moment of inertia.

3
dIy = bh
3

dy

(dy)x3
3

x = 2y6

50y5

y3 + 100

B
1.156 m
6

Replacing x by the function of y gives


dIy = 1 x3dy
3
= 1 (2y6
3

50y5

y3 + 100)3 dy
x

1.156

Iy = dIy =

= 2.72

1
6
3 (2y

50y5

105 m4

100 m

y3 + 100)3 dy
Ans.
8

2011

Use the integral function on a calculator to evaluate


this integral.

10.1 Moments of Inertia by Integration Example 7, page 1 of 3


7. Determine the moment of inertia of the
crosshatched area about the x axis.
y

y=9

x2

9m

y = 3x

x2
x

3m

2012

10.1 Moments of Inertia by Integration Example 7, page 2 of 3


1

We want to evaluate
3

A better approach is to use a vertical strip and then


apply the parallel-axis theorem to the strips.

Ix = y dA

where y has the same value throughout the differential


element dA. Thus it appears that we should use a
horizontal rectangular strip.
y

Centroid

Parallel-axis theorem
for a general region

But using a horizontal strip is awkward we would


have to use three different expressions for dA,
depending on the position of the strip.

Area
A

Ix = Ic + Ad2

d
x
y

For a rectangle in particular,

Ix =

1 3
bh + (b
12

h)d2

d
=(

1 3
h + hd2 )b
12

x
y

The moment of inertia of a strip of width


b dx is then
dIx = ( 1 h3 + hd2 ) dx
12
x

(1)

dx

C
d
x

2013

10.1 Moments of Inertia by Integration Example 7, page 3 of 3


y

The y-coordinate of the element


centroid is the average of y1 and y2

(x,y2)

d = yel = 1 ( y1 + y2 )
2

6 Eq. 1 becomes

y=9

x2

y2 = 9

4
h = y2

y1

y1 = 3x

y = 3x

x2

Ix = dIx
3

3x )3 + (9

3x) ( 9 + 3 x
2 2

= 328 m

Ans.

x2

x2 )2 } dx
8

x2, we

dIx = { 1 ( y2 y1 )3 + ( y2 y1 ) [ 1 ( y2 + y1 )]2} dx
12
2
1
= { [( 9 x2 ) ( 3x x2 )]3 + [( 9 x2 )
12
1
( 3x x2 )][ 1 ( 9 x2 ) + ( 3x x2 )]2} dx
2
2
9
3
= { 1 ( 9 3x )3 + ( 9 3x )[ + x x2 ]2} dx
2 2
12

= {1(9
0 12

x2

in Eq. 2. Thus Eq. 2 becomes

(x,y1)

(2)

x2, we

in Eq. 2. Similarly, since (x, y1) satisfies y = 3x


can substitute

yel

x ranges
from 0 to 3.

y1 ) [ 1 ( y1 + y2 )]2} dx
2

Since the point (x, y2) lies on the curve y = 9


can substitute

centroid

(xel,yel)

dIx = ( 1 h3 + hd2 ) dx
12
= { 1 ( y2 y1 )3 + ( y2
12

Enter this expression directly into the


integral function of a calculator.

2014

10.1 Moments of Inertia by Integration Example 8, page 1 of 4


8. Determine the moment of inertia of the
crosshatched area about the y axis.
y
1/4 m

4m

xy = 1

y = 2x

1/2 m

4m

1m

1m
x

2015

10.1 Moments of Inertia by Integration Example 8, page 2 of 4


1

We want to evaluate
Iy = x2 dA
where x has the same value throughout the differential
element dA. Thus it appears that we should use
vertical differential strips.
y

But using a vertical strip is awkward


we would have to use three different
expressions for dA, depending on the
location of the strip.

2016

10.1 Moments of Inertia by Integration Example 8, page 3 of 4


3

A better approach is to use a horizontal strip and then


apply the parallel-axis theorem to the strip.

centroid

area A
d
Parallel-axis theorem for a general region
Iy = Ic + Ad2

x
For a rectangle in particular,
Iy = 1 bh3 + (b
12

y
Area = b

h)d

1
= ( h3 + hd2 )b
12

h
x
y

The moment of inertia of a strip of width b


dIy = ( 1 h3 + hd2 ) dy
12

dy is then
d

(1)
C

h
x

2017

dy

10.1 Moments of Inertia by Integration Example 8, page 4 of 4


y
h = x2

From the figure, we see that Eq. 1 can be written as

1
dIy = { h3 + hd2} dy
12
= { 1 ( x2 x1 )3 + ( x2
12

x1
Centroid of
strip

(Eq. 1 repeated)
x1 )[

1
( x + x )]2 } dy
2 2 1

(2)

(xel,yel)
(x1,y)

(x2,y)

Since the point (x1,y) lies on the line y = 2x,


we can solve for x and substitute

xy = 1
x1 =

y
2

in Eq. 2. Similarly, since (x2,y) lies on the


curve xy = 1, we can substitute

y = 2x
1
d = | xel | = | average of x1 and x2 | = | 2 ( x1 + x2 )|

x2 = 1y
in Eq. 2. Thus Eq. 2 becomes

x
6

=
7
Top of region
at y = 4
Bottom of
region at y = 1

1
dIy = { ( x2 x1 )3 + ( x2 x1 )[ 1 ( x2 + x1 )]2 } dy
12
2
y
y 1 1
y 2
1
1
={ (1 (
))3 + ( y (
))[ ( y + (
) )] } dy
y
2
2
2
12
2
y
y
y 2
1 1
= { ( y + )3 + ( 1y + )[ 1
] } dy
2
2 2y
12
4

Iy = dIxy
4
1

y
1 y 1
[ 1 ( 1y + )3 + ( y + ) (
2
2 2y
12
8

y 2
) ] dy
4

Enter this expression directly into the


integral function of a calculator
= 2.81 m4

Ans.

2018

10.1 Moments of Inertia by Integration Example 9, page 1 of 3


9. Determine the moment of inertia of the
crosshatched area about the x axis.
y
2
y = 10e x

2
y = 10 e x

1m
1.5 m

2019

10.1 Moments of Inertia by Integration Example 9, page 2 of 3


1

We want to evaluate
Ix =

Ix for region D

y2dA
3.6788 m

Because of the shape of the region, the integral has to


be evaluated over two sub-regions, D and E.
y
1m

3.6788 m
Region D

2
y(1) = 10 e (1) = 3.6788 m

4
x

1m
No integration needed.
Use the formula for the moment of inertia of a
rectangle about a centroidal axis.
Ic =

Region E

bh3
12

(1)

h/2

So
IxD =

(1 m)(2 3.6788 m)3


12

4
= 33.1915 m

2020

C
h/2
(2)

10.1 Moments of Inertia by Integration Example 9, page 3 of 3


5

Ix for region E. Because the region is


symmetrical about the x axis, we can save
work by using vertical rectangular strips and
by applying Eq. 1 to these strips:
1
Ic = bh3
12
or,
y
1
dx
dIxE =
(dx)(2y)3
12
=
2
y = 10 e x

Thus, with the aid of the integral function on a


calculator, we have
IxE = dIx
1.5

2
1
(dx)[2(10e x )]3
12

2
= 2000 e 3x dx
3

2000 e 3x2 dx
3

= 4.7985 m4

(3)

Adding the results for regions D and E gives

Ix = IxD + IxE
by Eq. 2

by Eq. 3

= 33.1915 + 4.7985
1m
= 38.0 m4
x
1.5 m

2021

Ans.

10.1 Moments of Inertia by Integration Example 10, page 1 of 3


10. Determine the moment of inertia of the
crosshatched area about the y axis.
y
2 in.

1 in.

4 in.

y = 3x
3 in.

y = x2

2022

10.1 Moments of Inertia by Integration Example 10, page 2 of 3


1

We want to evaluate

Iy = x2dA

Integrate over region B.

(1)
4

where x has the same value throughout the differential


element dA.

dA = area of rectangle

Given the shape of the area, we have to evaluate the


integral in Eq.1 over two sub-regions, B and C.

= ( y2

y1 )

= ( 3x

x2 ) dx

x
y = 3x

(x,y2)
y
Region C
y2

y1
y = x2
(x,y1)

Region B

dx
1 in.

IyB = x2 dA
1

x2(3x

= 0.5500 in4

2023

x2) dx
(2)

dx

10.1 Moments of Inertia by Integration Example 10, page 3 of 3


6

Integrate over region C.

y
x

y = x2

Write the equation of the line:


y 3
x 1
=
4 3
2 1
Solving gives
y=x+2

(x,y2)
8
y2

4 in.
3 in.

y1

= ( y2

(x,y1)
1 in.

dA = Area of rectangle
y1 )

= [(x + 2)

dx
x2 ] dx

dx
x

2 in.

IyC = x2 dA
=

2
1

x2( x + 2

x2 ) dx

= 2.2167 in4

(3)

10 Adding the results for regions B and C gives


Iy = IyB + IyC
by Eq. 3
by Eq. 2
= 0.5500 + 2.2167 = 2.77 in4

2024

Ans.

10.1 Moments of Inertia by Integration Example 11, page 1 of 2


11. Given that the centroid C of the area bounded by a
quarter-circle lies a distance 4a/(3 above the base of the
quarter-circle, determine the moment of inertia of the cross
hatched area about an axis xc through the centroid.
y

yc
1

We want to evaluate
Ixc = yc2 dA
but because the equation of the circle is given in
terms of x and y instead of xc and yc, it is easier to
evaluate

x2 + y2 = a2

y2 dA

Ix

and then use the parallel-axis theorem:


C

Ix = Ixc + Ad2

xc
4a
3

= Ixc +

Area of quarter circle

a2 ( 4a )2
4 3

x
Solving gives
a
Ixc = Ix

4a4
9

Thus now we need to calculate Ix.

2025

10.1 Moments of Inertia by Integration Example 11, page 2 of 2


y
2

dA = Area of rectangle

x
= x

x2 + y2 = a2
3
(x,y)
a

dy

Solve x2 + y2 = a2
to get
a2

x=
4

dy

y2

Substitute into the equation for dA


dA = x

dy

y
=
x

Evaluate the integral with a


calculator that does symbolic
integration or use a table.

Use the result given by Eq. 2 in Eq. 1:


4a4
9
4
a
4a4
16
9

IxC = Ix

(Eq. 1 repeated)
(

16

4 ) a4
9

Ans.

2026

a2

y2 dy

Integrate
Ix = y2 dA
a

y2 a2

0
a4
=
16

y2 dy
(2)

10.2 Method of Composite Areas

2027

10.2 Method of Composite Areas Procedures and Strategies, page 1 of 1


Procedures and Strategies for Solving Problems Involving
Moments of Inertia of Composite Bodies

1. Divide the given region into a collection of simple


sub-regions that are listed in a table of geometric properties of
planar regions. Sometimes it is convenient to subtract one
region from another, when dividing the given region.

2. If the moment of inertia is to be calculated with respect to


an axis through the centroid of the entire region, then follow
the procedure that you have already learned for locating the
centroid.

d'
x'
Centroidal axis of semicircular region

3. Calculate the moment of inertia of each sub-region with


respect to an axis through the centroid of the sub-region.

The table gives Ix', not what


you want, Ic.

The parallel-axis theorem says

4. If the centroidal axis of the sub-region does not coincide


with the moment of inertia axis for the entire given region,
then use the parallel-axis theorem, I = Ic + Ad2, where Ic is the
moment of inertia with respect to the sub-region centroidal
axis, A is the area, and d is the distance between the
centroidal axis of the sub-region and the x axis (the axis about
which the moment of inertia is to be found).

Ix' = Ic + A(d')2
Solve for Ic:
Ic = Ix'

A(d')2

Now that Ic is known, you can


get the moment of inertia with
respect to x by using the
parallel-axis theorem again:

5. The parallel-axis theorem requires that you know Ic but


sometimes tables of geometric properties do not give you that
information directly and you must use the the parallel-axis
theorem to compute Ic.

Ix = Ic

2028

Ad2

10.2 Method of Composite Areas Problem Statement for Example 1


1. Determine the moment of inertia of the crosshatched region
about the x axis.
y

50 mm

x
20 mm

2029

10.2 Method of Composite Areas Problem Statement for Example 2


2. The figure shows the cross section of a
beam made by gluing four planks together.
Determine the moment of inertia of the
cross section about the x axis.
y
150 mm 150 mm

60 mm

200 mm
x

60 mm

60 mm

200 mm

60 mm

2030

10.2 Method of Composite Areas Problem Statement for Example 3


3. Determine the moment of inertia of the
beam cross section about the x centroidal
axis.
y
80 mm 80 mm
20 mm

120 mm
x
120 mm

20 mm
20 mm

2031

10.2 Method of Composite Areas Problem Statement for Example 4


4. A composite beam is constructed
from three plates and four standard
rolled-steel angles. Determine the
moment of inertia of the cross
section about the x centroidal axis.
y
110 mm 110 mm

10 mm

xc
140 mm

A = 877 mm2
Ixc = 0.202

16.2 mm

140 mm
10 m
10 mm

2032

106 mm4

10.2 Method of Composite Areas Problem Statement for Example 5


5. Determine the moment of inertia of the
trapezoidal region about the x and y axes.
y
3 in.

3 in.

6 in.

x
4 in.

4 in.

2033

10.2 Method of Composite Areas Problem Statement for Example 6


6. Determine the moment of inertia of the
crosshatched region about the y axis.
y
2 in.

4 in.
0.8 in.

4 in.

2 in.
0.8 in.
x

2034

10.2 Method of Composite Areas Problem Statement for Example 7


7. A precast concrete floor beam has the cross section shown.
Locate the centroid of the section and determine the moment
of inertia about a horizontal axis through the centroid.
y
250 mm

300 mm

300 mm

250 mm
75 mm

425 mm

50 mm

50 mm

2035

10.2 Method of Composite Areas Problem Statement for Example 8


8. A beam is built up from two standard rolled-steel channels and a
cover plate. Locate the centroid of the section and determine the
moments of inertia with respect to horizontal and vertical axes through
the centroid.
y
100 mm

100 mm
yc
Centroid of channel

40 mm

127 mm

A = 3780 mm2

254 mm

xc

127 mm
x
15.3 mm

2036

Ixc = 32.6

106 mm4

Iyc = 1.14

106 mm4

10.2 Method of Composite Areas Example 1, page 1 of 2


1. Determine the moment of inertia of the crosshatched region
about the x axis.
y

50 mm

x
20 mm

1 Consider the crosshatched region to


be composed of the difference of
two circular regions.

=
x

2037

10.2 Method of Composite Areas Example 1, page 2 of 2


2

A table of properties of planar regions gives the


information shown below.

For our particular problem,


1
(50 mm)4
4
= 4.9087 106 mm4
1
Small circle Ix =
(20 mm)4
4
= 0.1257 106 mm4
Large circle Ix =

Ix = 1 r4
4
Iy = 1 r4
4

Moment of Inertia
y

For the composite region, subtracting gives


r
C

Ix = Large circle Ix

= 4.9087
= 4.78

Circle

2038

Small circle Ix

106 mm4
106 mm4

0.1257

106 mm4
Ans.

10.2 Method of Composite Areas Example 2, page 1 of 2


2. The figure shows the cross section of a
beam made by gluing four planks together.
Determine the moment of inertia of the
cross section about the x axis.

y
150 mm 150 mm

60 mm

200 mm
x
200 mm

60 mm

60 mm
1

60 mm

Consider the crosshatched region to be consist of a


small rectangle subtracted from a large rectangle.
y

60 mm + 200 mm + 200 mm + 60 mm = 520 mm y

200 mm + 200 mm = 400 mm

=
x

520 mm

300 mm

420 mm
60 mm + 150 mm + 150 mm + 60 mm = 420 mm
Large rectangle

2039

400 mm

150 mm + 150 mm = 300 mm


Small rectangle

10.2 Method of Composite Areas Example 2, page 2 of 2


y

2 A table of properties of planar regions


gives the information shown below.
3
I x = bh
12

For our particular problem,


3
Large rectangle Ix = bh
12

Moment of Inertia

(420 mm)(520 mm)3


=
12

3
Iy = hb
12

b
2

b
2

= 4.9213

520 mm

109 mm4

h
2

420 mm
x

h
2

For our particular problem,


3
Small rectangle Ix = bh
12

Rectangle

400 mm
3

(300 mm)(400 mm)


12

= 1.6000

109 mm4
300 mm

For the composite region, subtracting gives


Ix = Large rectangle Ix
= 4.9213
= 3.32

109 mm4
109 mm4

Small rectangle Ix
1.6000

109 mm4
Ans.

2040

10.2 Method of Composite Areas Example 3, page 1 of 3


3. Determine the moment of inertia of the
beam cross section about the x centroidal
axis.
y
80 mm 80 mm
20 mm

120 mm
x
120 mm

20 mm
20 mm

2041

10.2 Method of Composite Areas Example 3, page 2 of 3


1

Consider the cross section to be composed of a large rectangle minus two small rectangles.
y
y
y
120 mm + 120 mm = 240 mm
80 mm 80 mm
20 mm

120 mm

=
x

280 mm

240 mm

240 mm

120 mm
20 mm
20 mm

80 mm

180 mm

80 mm

80 mm + 20 mm + 80 mm = 180 mm
20 mm + 120 mm + 120 mm + 20 mm = 280 mm
y
2

2042

Two regions of the same size and same position


relative to the x axis can be combined into 2
times a single region.

10.2 Method of Composite Areas Example 3, page 3 of 3


3

3
Large rectangle Ix = bh
12

(180 mm)(280 mm)3


=
12
= 3.2928

108 mm4
x

280 mm

180 mm

3
4 Small rectangle Ix = bh
12

240 mm

(80 mm)(240 mm)3


=
12
= 0.9216

108 mm4
5
80 mm

For the composite region, subtracting gives


Ix = Large rectangle Ix
= 3.2928
= 1.450

2043

108 mm4
108 mm4

Small rectangle Ix

2(0.9216

108 mm4)
Ans.

10.2 Method of Composite Areas Example 4, page 1 of 3


4. A composite beam is constructed
from three plates and four standard
rolled-steel angles. Determine the
moment of inertia of the cross
section about the x centroidal axis.

y
110 mm 110 mm

10 mm

140 mm
C

xc

A = 877 mm2
Ixc = 0.202

16.2 mm

140 mm
10 mm
10 mm

Consider the cross section to be composed of four angles and three rectangles.
y
y
y

=
x

+2

+4

x
2

The top and bottom


plates are identical.

2044

x
3 The four angles
are identical.

106 mm4

10.2 Method of Composite Areas Example 4, page 2 of 3


4

Middle rectangle
3
Middle rectangle Ix = bh
12

140 mm
3

(10 mm)(140 mm + 140 mm)


12

= 1.8293

107 mm4

C'
(1)

140 mm
10 mm

Upper rectangle
Use parallel axis theorem:
Ix = Ixc' + d2A

(2)
Area
Distance between xc' and x

Moment of inertia
about axis xc' through
centroid of rectangle

Here,

y
10 mm
= 5 mm
2
xc'

110 mm 110 mm
C'
140 mm

Ixc' =

(110 mm + 110 mm)(10 mm)3


bh3
=
12
12

= 18333 mm4
d = 140 mm + 5 mm = 145 mm

10 mm

A = (110 mm + 110 mm)(10 mm)


x

= 2200 mm2

2045

10.2 Method of Composite Areas Example 4, page 3 of 3


7 The parallel axis theorem, Eq. 2, now gives

Upper rectangle Ix = Ixc' + d2A

The parallel axis theorem gives


Angle Ix = Ixc + d2A

= 18333 mm4 + (145 mm)2(2200 mm2)


107 mm4

= 4.6273
8

= 0.202
= 1.3643

107 mm4

(3)

16.2 mm

Angle

106 mm4 + (123.8 mm)2(877 mm2)

xc

140 mm

A = 877 mm2

16.2 mm
Ixc = 0.202

C'
d = 140 mm
x

106 mm4

10 For the composite region, adding gives


Ix = Middle rectangle Ix + 2
= 1.8293
= 165.4

Upper rectangle Ix + 4

107 mm4 + 2(4.6273


106 mm4

Angle Ix

107 mm4) + 4(1.3643


Ans.

2046

107 mm4)

16.2 mm = 123.8 mm

10.2 Method of Composite Areas Example 5, page 1 of 4


5. Determine the moment of
inertia of the trapezoidal region
about the x and y axes.

y
3 in.

3 in.

6 in.

x
4 in.
1

4 in.

Consider the trapezoid to be the sum of a rectangle and two triangles.


y
y
3 in.

3 in.

3 in.

3 in.

=
6 in.

6 in.

x
4 in.

+2

6 in.

x
3 in.

4 in.

2047

1 in.

10.2 Method of Composite Areas Example 5, page 2 of 4


2

Ix and Iy for the rectangle

Since the centroid C' does not lie on the x axis, we have to use
the parallel axis theorem to calculate Ix.

Rectangle Ix = Ixc' + d2A


(6 in.)(3 in. + 3 in.)3
=
12

3 in.

+ (3 in.)2[(3 in. + 3 in.)(6 in.)]

xc'

C'

= 432 in4

3 in.

6 in.

(1)

Since the centroid C' lies on the y axis, we do not have to


use the parallel axis theorem for Iy.
Rectangle Iy = Iyc'
(3 in. + 3 in.)(6 in.)3
=
12
= 108 in4

2048

(2)

10.2 Method of Composite Areas Example 5, page 3 of 4


5

Ix and Iy for the triangle

bh3
Ixc' =
36

For our particular triangle


y
yc'

A table of properties of planar regions gives the


information below.

=
3
Ixx = bh
36
3
IBB = hb
12

Moment of Inertia

= 6 in4

6 in.

Area =

xc'

C'

bh
2

2h
3
x

x
C

h
3

1 in.

b
Triangle

(1 in.)(6 in.)3
36

b'(h')3
Iyc' =
36
=

(6 in.)(1 in.)3
36

= 0.1667 in4

7 Parallel axis theorem applied to triangle


y
yc'
Triangle Ix = Ixc' + dx2A
= 6 in4 + (2 in.)2(3 in2)

Area, A =

6 in.

1
(1 in.)(6 in.)
2

= 3 in2

1
dy = 3 in. +
in. = 3.3333 in.
3

= 18 in4
xc'

1
in.
3

C'
dx =

3 in.

1 in.

2049

6 in.
3 = 2 in.
x

(3)

Triangle Iy = Iyc' + d2A


= 0.1667 in4
+ (3.3333 in.)2(3 in2)
= 33.4994 in4

(4)

10.2 Method of Composite Areas Example 5, page 4 of 4


8 For the composite region, using Eqs. 1, 2, 3, and 4 gives
Ix = Rectangle Ix + 2

Triangle Ix

= 432 in4 + 2(18 in4)


= 468 in4
`

Iy = Rectangle Iy + 2

Ans.
Triangle Iy

= 108 in4 + 2(33.4994 in4)


= 175 in4

Ans.

2050

10.2 Method of Composite Areas Example 6, page 1 of 5


6. Determine the moment of inertia of the
crosshatched region about the y axis.
2 in.

4 in.

4 in.

2 in.

0.8 in.

0.8 in.
x

Consider the crosshatched region to be composed of a rectangle


minus two circular regions plus two semicircular regions.
y

=
x

y
+2

2
x

2051

10.2 Method of Composite Areas Example 6, page 2 of 5


2

Iy for rectangle
y
4 in.

4 in.
2 in.
x

C'
2 in.

Since the y axis passes through the centroid of the


rectangle, the parallel axis theorem is not needed.
Rectangle Iy = Iyc'
(2 in. + 2 in.)(4 in. + 4 in.)3
=
12
= 170.6667 in4

(1)

2052

10.2 Method of Composite Areas Example 6, page 3 of 5


4

For the circular region, a table of properties of planar


regions gives the information shown below.

yc'

y
d = 4 in.

Ix = 1 r4
4
Iy = 1 r4
4

C'

Moment of Inertia

r = 0.8 in.
x

y
5

For our particular problem,


Iyc' =

r
C

1
4

(0.8 in.)4

= 0.3217 in4

Area, A = r2
= (0.8 in.)2

Circle

= 2.0106 in2
6

Applying the parallel axis theorem gives


Circle Iy = Iyc' + d2A
= (0.3217 in4) + (4 in.)2(2.0106 in2)
= 32.4913 in4

2053

(2)

10.2 Method of Composite Areas Example 6, page 4 of 5


7

For the semicircle, a table of properties of


planar regions gives the information shown
below.
Ix =

r4
8

r4
8
4r
yc =
3

4 in.
4r
3

IBB =
r
x

r4
8
9

Semicircle

(3)

to compute Iy for the semicircle.


Unfortunately, the table gives us the
moment of inertia with respect to the
base, BB, of the semicircle, not with
respect to the axis through the
centroid yc'.

Iy = Iyc' + d2A

C'

yc

We would like to apply the


parallel-axis theorem:

r = 2 in.

Moment of Inertia

Iy =

yc'

But we can still make use of the result IBB from the table by
applying the parallel axis theorem between the BB axis and
the yc ( yc' ) axis :
IBB = Iyc' + d2A
or,
r4
4r 2 r2
=
I
+
(
)(
)
yc'
8
3
2
Solving gives
Iyc' = (

2054

8 4
)r
9

(4)

10.2 Method of Composite Areas Example 6, page 5 of 5


yc'

y
4r
3

r = 2 in.
x

C'

4 in.
10 Eqs. 3 and 4 can now be applied to the semicircular region
Semicircle Iy = Iyc' + d2A
=(

8 )r4 + (4 in. + 4r )2( r2)


3
9
2

Substituting r = 2 in. and evaluating the resulting expression gives


Semicircle Iy = 149.4808 in4

(5)

11 For the composite region, using Eqs. 1, 2, and 5 gives


Iy = Rectangle Iy
= 170.6667 in4
= 405 in4

Circle Iy + 2

Semicircle Iy

2(32.4913 in4) + 2(149.4808 in4)


Ans.

2055

10.2 Method of Composite Areas Example 7, page 1 of 5


7. A precast concrete floor beam has the cross section shown.
Locate the centroid of the section and determine the moment of
inertia about a horizontal axis through the centroid.
y
250 mm

300 mm

300 mm

250 mm
75 mm

425 mm

50 mm

50 mm

x
1

Definition of centroid
Xc = 0, by symmetry
Yc =

ycA
A

(1)

where yc is the centroidal coordinate of the


region with area A.

2056

10.2 Method of Composite Areas Example 7, page 2 of 5


2

Consider the section to be composed of a horizontal


rectangle and and two identical vertical rectangles.
y

y
=

+2
x

2057

10.2 Method of Composite Areas Example 7, page 3 of 5


y

Upper rectangle

C' (centroid)

75 mm

A = (1200 mm)(75 mm)


=9

75 mm
= 37.5 mm
2

104 mm2

yc' = 425 mm + 37.5 mm

yc'

425 mm

= 462.5 mm
x
2
4

yc'

Lower rectangle

(250 mm + 50 mm + 300 mm) = 1200 mm

A = (425 mm)(50 mm)


C' (centroid)
4

= 2.125 10 mm
425 mm
yc' =
2

425 mm
yc'

= 212.5 mm

x
50 mm

Set up table
Region
upper rectangle
lower rectangles

2 lower rectangles
A ( mm2 ) yc' ( mm )
9.000
2(2.125

104
104)

A = 13.250

104

462.5
212.5

yc'A ( mm3 )
41.625 106
9.031 106
yc'A = 50.656 106

2058

10.2 Method of Composite Areas Example 7, page 4 of 5


6

Eq. 1 gives the distance to the centroid of the entire cross section.
ycA
A
50.656
=
13.250

Yc =

106
104

= 382.31 mm

75 mm = 37.5 mm
2

Ans.
y

Ix of upper rectangle

C' (centroid of rectangle)

75 mm

bh
12
(1200 mm)(75 mm)3
=
12

Ixc' =

xc'
d

107 mm4

= 4.2188

xc

C (centroid of entire section)


425 mm
Yc = 382.31 mm

d = 75 mm + 425 mm
(37.5 mm + 382.31 mm)

= 80.19 mm
Parallel axis theorem:
Upper rectangle Ixc = Ixc' + d2A

Area A was calculated previously (See the table).

= 4.2188

107 mm4 + (80.19 mm)2(9

= 6.2093

108 mm4

104 mm2)
(2)

2059

10.2 Method of Composite Areas Example 7, page 5 of 5


y

Ix of lower rectangle

yc'

bh3
12
(50 mm)(425 mm)3
=
12

C (centroid of entire section)

Ixc' =

= 3.1986

xc
d

10 mm

d = 382.31 mm

425 mm

xc'
425 mm
= 212.5 mm
2

212.5 mm

x
50 mm C' (centroid of rectangle)

= 169.81 mm
Yc = 382.31 mm

Parallel axis theorem:


Lower rectangle Ixc = Ixc' + d2A

Area A was calculated previously.

= 3.1986

108 mm4 + (169.81 mm)2(2.125

= 9.3261

108 mm4

104 mm2)

(3)

For the composite region, using Eqs. 1and 2 gives


Ixc = Upper rectangle Ixc + 2
= 6.2093
= 249

Lower rectangle Ixc

108 mm4 + 2(9.3261

107 mm4

108 mm4)
Ans.

2060

10.2 Method of Composite Areas Example 8, page 1 of 6


8. A beam is built up from two standard rolled-steel channels and a
cover plate. Locate the centroid of the section and determine the
moments of inertia with respect to horizontal and vertical axes through
the centroid.
y
100 mm

100 mm
yc
Centroid of channel

40 mm

127 mm

A = 3780 mm2

254 mm

xc

127 mm
x
15.3 mm
1

Definition of centroid
Xc = 0, by symmetry
Yc =

ycA
A

(1)

where yc is the centroidal coordinate of the region with area A.

2061

Ixc = 32.6

106 mm4

Iyc = 1.14

106 mm4

10.2 Method of Composite Areas Example 8, page 2 of 6


2

Consider the cross section to be composed of a rectangle and two channels.


y

+2

x
y
Centroid of channel

127 mm
C'
3
127 mm

yc' = 127 mm
x

2062

Note that yc' is known.

10.2 Method of Composite Areas Example 8, page 3 of 6


4

Rectangle

C'

Area, A = (40 mm)(100 mm + 100 mm)


= 8000 mm2

100 mm

40 mm
100 mm
40 mm
= 20 mm
2

yc'

5 Locate centroid of rectangle:


yc' = 254 mm + 20 mm

127 mm + 127 mm = 254 mm

= 274 mm

6 Set up table

A ( mm2 ) yc' ( mm )

Region
Channel
Rectangle

2 channels (area was given)

2(3780)
8000
A = 15560

127
274

yc'A ( mm3 )
0.9601 106
2.1920 106
yc'A = 3.1521 106

Eq. 1 gives the distance to the centroid of the entire cross section:
ycA
A
3.1521 106
=
15560

Yc =

= 202.58 mm

(Eq. 1 repeated)

Ans.

2063

10.2 Method of Composite Areas Example 8, page 4 of 6


8

Ixc and Iyc of channels (Ixc', Iyc', and A are given.)

y yc'
Centroid of entire beam section

Use the parallel axis theorem.


Channel Ixc = Ixc' + (dx)2A
= 32.6
= 54.193

106 mm4 + (202.58 mm

(2)

Yc = 202.58 mm

106 mm4 + (15.3 mm)2(3780 mm2)

xc'

x
dy = 15.3 mm

= 2.025

C'

127 mm

Channel Iyc = Iyc' + (dy)2A


= 1.140

xc

dx

127 mm)2(3780 mm2)

106 mm4

Centroid of
channel

106 mm4

(3)

2064

10.2 Method of Composite Areas Example 8, page 5 of 6


9

Ixc and Iyc of upper rectangle


Ixc' =

bh3
12

100 mm
40 mm

(100 mm + 100 mm)(40 mm)


=
12
= 1.067

100 mm

40 mm
= 20 mm
2

C'

xc'
d

106 mm4

d = (40 mm + 254 mm)

(20 mm + 202.58 mm)

254 mm

xc
Yc = 202.58 mm

= 71.42 mm
Use the parallel axis theorem.
Area A was calculated previously.

Rectangle Ixc = Ixc' + d A


= 1.067
= 41.874
Rectangle Iyc =
=

Centroid of entire beam section

106 mm4 + (71.42 mm)2(8000 mm2)


106 mm4

(4)

bh3
12

(40 mm)(100 mm + 100 mm)3


12

= 26.667

106 mm4

(5)

2065

10.2 Method of Composite Areas Example 8, page 6 of 6


10 For the composite region,
Ixc = Rectangle Ixc + 2
by Eq. 4
= 41.874
= 150.3

106 mm4)

106 mm4

by Eq. 5

= 30.7

by Eq. 2

106 mm4 + 2(54.193

Iyc = Rectangle Iyc + 2

= 26.667

Channel Ixc

Ans.
Channel Iyc
by Eq. 3

106 mm4 + 2(2.025


106 mm4

106 mm4)
Ans.

2066

10.3 Products of Inertia

2067

10.3 Products of Inertia Procedures and Strategies, page 1 of 1


Procedures and Strategies for Solving Problems
Involving Products of Inertia
1. If the given area can be expressed as a sum of
simple shapes for which tabulated products of
inertia are available, then you can calculate the
product of inertia by applying the parallel-axis
theorem to each shape:

=
Ixy = Ic-xy + xcycA
x

In applying this equation, remember that


coordinates of the centroid, xc and yc, may be
negative. Also remember that you can subtract
as well as add areas.

x
y

2. If the area cannot be expressed as a sum of simple


shapes, then you must evaluate the integral
defining the product of inertia:
x

Ixy = xy dA
Note: You can often save work by using the fact
that the product of inertia about an axis of symmetry
is zero.

Axis of symmetry
Ixy = 0

2068

10.3 Products of Inertia Problem Statement for Example 1


1. Determine the product of inertia of the narrow
strip with respect to the x and y axes.
y

900 mm

30
3 mm
900 mm

2069

10.3 Products of Inertia Problem Statement for Example 2


2. Determine the product of inertia of the crosshatched
area with respect to the x and y axes.
y

x2 + y2 = 1002

x
100 mm

2070

10.3 Products of Inertia Problem Statement for Example 3


3. Determine the product of inertia of the crosshatched
area with respect to the x and y axes.
y
x = 2y6 50y5

1.156 m

Scales on the x
and y axes are
not the same.

x
100 m

2071

y3 + 100

10.3 Products of Inertia Problem Statement for Example 4


4. Determine the product of inertia of the crosshatched area
with respect to the x and y axes, if a = 3 m and b = 1 m.
y

a
y = a sin(

x)
2b

x
b

2072

10.3 Products of Inertia Problem Statement for Example 5


5. Determine the product of intertia of the crosshatched
area with respect to the x and y axes.
y

x=4

y2

x = 3y
1m
x
3m

1m

2073

10.3 Products of Inertia Problem Statement for Example 6


6. Determine the product of inertia of the crosshatched
area with respect to the x and y axes.
y
y = 10e-x

3.68 m

x
1m
1.5 m

2074

10.3 Products of Inertia Problem Statement for Example 7


7. Determine the product of inertia of the beam's
cross-sectional area with respect to axes passing through the
centroid C.
105 mm
y

15 m

52.5 mm

80 mm

15 mm

7.5 mm
80 mm

15 mm

2075

10.3 Products of Inertia Problem Statement for Example 8


8. Determine the product of inertia of the beam's
cross-sectional area with respect to x and y axes passing
through the centroid C.
y

1 in.

11 in.

6 in.

5 in.
x

8 in.
1 in.

1 in.

6 in.
C

5 in.

11 in.

2076

10.3 Products of Inertia Problem Statement for Example 9


9. The cross section of a standard rolled-steel angle is shown.
Determine the product of inertia of the section with respect to x
and y axes passing through the centroid C. Make the
simplifying assumption that all corners are square.
y
0.987 in.
0.5 in.

6 in.
x
C
1.99 in.
0.5 in.
4 in.

2077

10.3 Products of Inertia Problem Statement for Example 10


10. Determine the product of inertia of the crosshatched
area with respect to the x and y axes.
y

150 mm

15 mm
30 mm

2078

10.3 Products of Inertia Problem Statement for Example 11


11. Determine the product of inertia of the crosshatched
area with respect to the x and y axes.
y

280 mm 210 mm

40 mm
50 mm

x
210 mm
330 mm

2079

10.3 Products of Inertia Example 1, page 1 of 2


1. Determine the product of inertia of the narrow
strip with respect to the x and y axes.
y

900 mm

30
3 mm

900 mm

1 The product of inertia is defined as


Ixy = xy dA

(1)
ds

Here dA can be taken as an infinitesimal rectangle


with sides ds and 3 mm.

2080

3 mm

10.3 Products of Inertia Example 1, page 2 of 2


dx
2

From geometry

30

dy

ds = dx/cos 30
ds

dA = (3 mm) ds
= (3/cos 30) dx
900 mm
y = x tan 30 (equation of the line)

x
30

Thus Eq 1 becomes
Ixy =

30

xy dA

(Eq. 1 repeated)

(900 mm) cos 30


ds

900 cos 30

x( x tan 30)[(3/cos 30) dx]


900 cos 30

= 6.31

108 mm4

(900 mm) cos 30

Ans.

2081

10.3 Products of Inertia Example 2, page 1 of 2


2. Determine the product of inertia of the crosshatched
area with respect to the x and y axes.
y

x2 + y2 = 1002

x
100 mm

The product of inertia ,


Ixy = xy dA
can be evaluated using double integration. Instead, we will use an
alternative approach based on the equation
Ixy = dIxy

(1)

where dIxy is the product of inertia of an infinitesimal strip.

2082

10.3 Products of Inertia Example 2, page 2 of 2


2 Arbitrarily choosing vertical, rather than horizontal,
strips and applying the parallel-axis theorem to the
strip gives
dIxy = dIx'y' + xelyel dA

(2)

x'
dA = y

yel = y/2

Because the x axis is an axis of symmetry for the


strip, dIx'y' = 0 and Eq. 2 reduces to

xel = x

= x(y/2)(y dx)

100 mm

(3)
3 Thus,

Because the variable of integration is x, we have to


use the equation of the bounding curve, x2 + y2 =
1002, to obtain y as a function of x :

Ixy = dIxy

x2
=

Using this expression in Eq. 3 gives, after


simplifying,

100
x(1002 x2)/2 dx
0

= 1.25
2

dIxy = x(100

dx

dIxy = xelyel dA

1002

Centroid of the
infinitesimal
strip
x2 + y2 = 1002

where dIx'y' is the product of inertia of the


differential strip about the x y axes (centroidal axes
for the strip).

y=

y'

x )/2 dx

2083

107 mm4

Ans.

10.3 Products of Inertia Example 3, page 1 of 2


3. Determine the product of inertia of the crosshatched
area with respect to the x and y axes.
y
x = 2y6 50y5

y3 + 100

1.156 m

Scales on the x
and y axes are
not the same.

x
100 m
1

The product of inertia,


Ixy = xy dA
can be evaluated using double integration. An
alternative approach, that will be used here, is to
evaluate
Ixy = dIxy
where dIxy is the product of inertia of an
infinitesimal strip.

2084

(1)

10.3 Products of Inertia Example 3, page 2 of 2


2

Choosing horizontal strips and applying the


parallel-axis theorem to the strip gives
dIxy = dIx'y' + xelyel dA

Centroid of the
infinitesimal
strip

(2)

where dIx'y' is the product of inertia for the


differential strip about the x y axes.

dA = x

dy

Because the x axis is an axis of symmetry for the


strip, dIx'y' = 0 and Eq.2 reduces to

dy
x

1.156 m

yel = y
dIxy = xelyel dA
x
= (x/2)(y)(x dy)
= x2y dy/2
3

xel = x/2
x

(3)

Replacing x in Eq. 3 by the expression for the


bounding curve gives
dIxy = x2y dy/2

Ixy = dIxy

(Eq. 3 repeated)

(2y6 50y5

Thus the product of inertia is

y3 + 100)

1.156
= (2y6 50y5
0
= 1973 m4

2085

y3 + 100)2y dy/2

Ans.

10.3 Products of Inertia Example 4, page 1 of 3


4. Determine the product of inertia of the crosshatched area
with respect to the x and y axes, if a = 3 m and b = 1 m.
y

a
y = a sin(

x)
2b

x
b

2086

10.3 Products of Inertia Example 4, page 2 of 3


y

Evaluate the integral

x
Ixy = dIxy

(1)

y'
xel = x/2

dy

dA = x

where dIxy is the product of inertia of a horizontal


infinitesimal strip.

dy

2 Applying the parallel axis theorem to the strip gives


dIxy = dIx'y' + xelyel dA

x'

0, because y is an axis of symmetry of the strip


Centroid of the
infinitesimal
strip

x/2
yel = y

x dy

= xelyel dA
y
= x2y dy/2
x

(2)

The x2 in this expression can be replaced by a function of y by


solving for x from the equation of the bounding curve:
y = a sin(

2087

x)
2b

10.3 Products of Inertia Example 4, page 3 of 3


3 Thus
y

x = (2b/ ) sin-1(y/a)
Eq. 2 becomes
dIxy = x2y dy/2

(Eq. 2 repeated)

= (2b/ ) sin-1(y/a) 2 y/2 dy


Eq. 1 then gives, after substituting a = 3 m and b = 1 m,
Ixy = dIxy

a
x = 2b/ sin-1 (y/a)

3
=

(Eq. 1 repeated)

(2/ ) sin-1(y/3) 2 y/2 dy

0
= 0.669 m4

Ans.

2088

10.3 Products of Inertia Example 5, page 1 of 3


5. Determine the product of inertia of the crosshatched
area with respect to the x and y axes.
y

x=4

y2

x = 3y
1m
x
3m

1m

2089

10.3 Products of Inertia Example 5, page 2 of 3


1 Evaluate the integral
Ixy = dIxy
where dIxy is the product of inertia of a
horizontal infinitesimal strip.
Centroid
of the
strip

dA = (x2

x1)

dy

Applying the parallel-axis theorem to the strip


gives
dIxy = dIxy' ' + xel yel dA
0, because y' is an axis of
symmetry of the strip

(xel, yel)

(x1, y)

(x2, y)

(x1 + x2)/2
x

(x2

xel = x1 + (x2

x1)/2

(x2

x1)/2

= (x2 + x1)/2

y
= y(x22

x1)/2

'

2090

x1) dy

= xel yel dA

dy yel = y
x
x1

(x2

x12) dy

(2)

10.3 Products of Inertia Example 5, page 3 of 3


y
y2

x2 = 4
1m

x1 = 3y

x
3

Replacing x1 and x2 in Eq. 2 by functions of y gives


dIxy = y(x22 x12) dy
= y[(4

y2)2

(Eq. 2 repeated)

(3y)2]dy

Thus Eq. 1 becomes


Ixy = dIxy

1
y[(4
0

(3)

y2)2

= 3.917 m4

(3y)2] dy

Ans.

2091

10.3 Products of Inertia Example 6, page 1of 2


6. Determine the product of inertia of the crosshatched
area with respect to the x and y axes.
y
y = 10e-x

3.68 m
1
x

Ixy = dIxy

1m
1.5 m
y

(1)

where dIxy is the product of inertia of a


vertical infinitesimal strip.
y'

x = xel
y = 10e-x
3.68 m

Evaluate the integral

x'

Applying the parallel-axis theorem to the strip gives


dIxy' ' = dIxy + xelyel dA

Centroid
of the
strip

0, because y' is an axis of


symmetry of the strip

x
= xelyel dA

y dx
yel = y/2

y/2

dx

= xy2 dx/2

dA = y dx

2092

(2)

10.3 Products of Inertia Example 6, page 2 of 2


y

y = 10e-x

3.68 m

x
1m
1.5 m
3

Replacing y in Eq. 2 by y = 10e-x gives


dIxy = xy2 dx/2

(Eq. 2 repeated)

-x 2

10e

Thus the equation for Ixy becomes,


Ixy = dIxy
1.5
2
= x(10e-x )2 dx/2
1
= 1.553 m4

(Eq. 1 repeated)

(Evaluate numerically with a calculator)

Ans.

2093

10.3 Products of Inertia Example 7, page 1 of 4


7. Determine the product of inertia of the beam's
cross-sectional area with respect to axes passing through the
centroid C.
105 mm
y

15 m

52.5 mm

80 mm

15 mm

7.5 mm
80 mm

15 mm

2094

10.3 Products of Inertia Example 7, page 2 of 4


1

Express the area as the sum of three rectangles.


All lengths are in millimeters.

105
15

y, y

y
52.5

52.5

15/2 = 45

80
=

15

C
7.5

15

80/2 + 7.5 = 47.5

105

80

Rectangle 2

80

y
105/2 - 15/2 = 45

y
Centroids of
rectangles

15
15

x, x

Rectangle 1
x

80

80/2 + 7.5 = 47.5


x

C
15
Rectangle 3

2095

10.3 Products of Inertia Example 7, page 3 of 4


2

Apply the parallel-axis theorem to rectangle 1:

45 mm
C

Ixy1 = Ix'y' + dxdy A


0, because the x' and y' axes are axes
of symmetry of the rectangle

47.5 mm
x

80 mm
= 0 + ( 45 mm)( 47.5 mm)[(80 mm)(15 mm)]
= 2.5650

106 mm4

y, y

(1)
15 mm

Similarly for rectangle 2:

15 mm

Ixy2 = Ix'y' + dxdy A

105 mm

= 0 + (0)(0)A

y
=0
4

x, x

45 mm

(2)

Rectangle 3:
80 mm

Ixy3 = Ix'y' + dxdy A

x
47.5 mm

= 0 + (45 mm)(47.5 mm)[(80 mm)(15 mm)]


= 2.5650

106 mm4

(3)

15 mm

2096

10.3 Products of Inertia Example 7, page 4 of 4


5

Add the Ixy terms from Eqs. 1, 2, and 3:


Ixy = Ixy1 + Ixy2 + Ixy3
= 2.565
= 5.13

106 mm4 + 0 + 2.5650


106 mm4

106 mm4
Ans.

2097

10.3 Products of Inertia Example 8, page 1 of 4


8. Determine the product of inertia of the beam's
cross-sectional area with respect to axes passing through
the centroid C.
y

1 in.

11 in.

6 in.

5 in.
x

8 in.
1 in.

1 in.

6 in.
C

5 in.

11 in.

2098

10.3 Products of Inertia Example 8, page 2 of 4


1

Express the area as the sum of three rectangles.


y
All lengths are in inches.

1 in.

11 in.

y, y
y y

6 in.

6 in.

5 in.

6 11/2 = 0.5
x

8 in.
1 in.

+
5 1/2 = 4.5

5 in.

x
1 in.

11 in.

x, x
C

11

Rectangle 2

Rectangle 1
11 in.
y

1 in.

x
y

5 1/2 = 4.5

6 11/2 = 0.5
C
Rectangle 3

2099

10.3 Products of Inertia Example 8, page 3 of 4


2

Apply the parallel-axis theorem to rectangle 1:

y y
0.5 in.

Ixy1 = Ix'y' + dxdyA

x
C

0, because the x' and y' axes are axes


of symmetry of the rectangle

4.5 in.
x

= 0 + ( 0.5 in.)( 4.5 in)[(1 in.)(11 in.)]


4

= 24.75 in.

1 in.

(1)

11 in.
Rectangle 1

Similarly for rectangle 2:


y, y
Ixy2 = Ix'y' + dxdyA
= 0 + (0)(0)A
=0

(2)

8 in.

x, x
C

1 in.
Rectangle 2

2100

10.3 Products of Inertia Example 8, page 4 of 4


4

Rectangle 3:

11 in.
y

1 in.

Ixy3 = Ix'y' + dxdyA


y

= 0 + (0.5 in.)(4.5 in.)[(11 in.)(1 in.)]


= 24.75 in.4
5

(3)

Add the Ixy terms from Eqs. 1, 2, and 3:

Rectangle 3

Ixy = Ixy1 + Ixy2 + Ixy3


= 24.75 in.4 + 0 + 24.75 in.4
= 49.50 in.4

4.5 in.

0.5 in.

Ans.

2101

10.3 Products of Inertia Example 9, page 1 of 3


9. The cross section of a standard rolled-steel angle is shown.
Determine the product of inertia of the section with respect to x
and y axes passing through the centroid C. Make the
simplifying assumption that all corners are square.
y
0.987 in.
0.5 in.

6 in.
x
C
1.99 in.
0.5 in.
4 in.

2102

10.3 Products of Inertia Example 9, page 2 of 3


Express the area as the sum of two rectangles.

All lengths are in inches.

y
0.987 in.

y
0.5 in.

0.987

0.5/2 = 0.737

y
x

6 in.

6 in.
6/2
x

1.99 = 1.01

3.5/2 + 0.5 0.987


= 1.263
y

1.99
x

0.5/2 = 1.74

1.99 in.
x
0.5 in.

0.5

4 in.

Rectangle 1

Centroids
of rectangles

3.5

Rectangle 2

2103

10.3 Products of Inertia Example 9, page 3 of 3


2

Apply the parallel-axis theorem to rectangle 1 .


Rectangle 1:
Ixy1 = Ix'y' + dxdy A

0.737 in.

0, because the x' and y' axes are axes


of symmetry of the rectangle
x

= 0 + ( 0.737 in.)(1.01 in.)[(0.5 in.)(6 in.)]

6 in.
1.01 in.

= 2.233 in.4
3

(1)

x
C

Rectangle 2 :
Ixy2 = Ix'y' + dxdy A
= 0 + (1.263 in.)( 1.74 in.)[(3.5 in.)(0.5 in.)]

0.5 in.

(2)

Rectangle 1
= 3.846 in.4
y
4

Add the Ixy terms from Eqs. 1 and 2:

1.263 in.
x

Ixy = Ixy1 + Ixy2


=

C
4

1.74 in.

2.233 in. + ( 3.846 in. )

= 6.08 in.4

Rectangle 2

Ans.
3.5 in.

2104

10.3 Products of Inertia Example 10, page 1 of 4


10. Determine the product of inertia of the crosshatched
area with respect to the x and y axes.
y

150 mm

15 mm
30 mm

2105

10.3 Products of Inertia Example 10, page 2 of 4


1

Express the area as a rectangle minus a circle


plus a semicircle.

75 mm
150 mm

150 mm
y

15 mm
x

60 mm
30 mm

30 mm
x
y

Radius
= 15 mm

x
150 mm
2 A table of properties of planar regions
gives the information shown below.

x
Centroid

Centroid location
A=

r
2

30 mm

150 mm
4r
3

4(30 mm)
= 12.7324 mm
3
30 mm

x
Semicircular area

x
30 mm
x

2106

10.3 Products of Inertia Example 10, page 3 of 4


y

3 Apply the parallel-axis theorem to the rectangle.

150 mm

Ixy-rectangle = Ix'y' + dxdy A

0, because the x' and


y' axes are axes of
symmetry

75 mm

= 0 + (75 mm)(30 mm) (150 mm)(60 mm)


= 2.0250

107 mm4

60 mm

30 mm

(1)

x
4

Similarly for the circle,


Ixy-circle = Ix'y' + dxdy A
= 0 + (150 mm)(30 mm)
0.3181

107 mm4

15 mm)2

(2)

y
150 mm
Radius = 15 mm

x
30 mm

2107

10.3 Products of Inertia Example 10, page 4 of 4


5 Similarly for the semicircle,

y
150 mm

Ixy-semicircle = Ix'y' + dxdy A


= 0 + (150 mm + 12.7324 mm)(30 mm)
30 mm)2/2

12.7324 mm
= 0.6902

10 mm

(3)

30 mm

x
30 mm
x

Add the Ixy values for the rectangle and semicircle and subtract the circle.
Ixy = Ixy-rectangle + Ixy-semicircle
= 2.0250
= 2.40

Ixy-circle

107 mm4 + 0.6902


107 mm4

107 mm4

0.3181

107 mm4
Ans.

2108

10.3 Products of Inertia Example 11, page 1 of 6


11. Determine the product of inertia of the crosshatched
region with respect to the x and y axes.
y

280 mm 210 mm

40 mm
50 mm

x
210 mm
330 mm

2109

10.3 Products of Inertia Example 11, page 2 of 6


y

1 Express the area as a rectangle minus a triangle.

330 mm/2 = 165 mm


y

280 mm

=
280 mm 210 mm

x
330 mm
280 mm/2 = 140 mm
y

40 mm
50 mm

y
50 mm + 210 mm/3
= 120 mm

x
210 mm

40 mm + 210 mm/3
= 110 mm

330 mm
210 mm

x
210 mm

2110

10.3 Products of Inertia Example 11, page 3 of 6


2

Apply the parallel-axis theorem to the rectangle.

Ixy-rectangle= Ix'y' + dxdy A

165 mm

0, because the x' and y' axes are


axes of symmetry
= 0 + (165 mm)(140 mm) (280 mm)(330 mm)
= 21.3444

108 mm4

280 mm

(1)
140 mm
x
330 mm

2111

10.3 Products of Inertia Example 11, page 4 of 6


3

Apply the parallel-axis theorem to the triangle. Note that Ix'y'

0.

y
120 mm

Ixy-triangle = Ix'y' + dxdy A

A = (1/2)(210 mm)(210 mm)


= 22050 mm2

= Ix'y' + (120 mm)(110 mm)(22050 mm2)


= Ix'y' + 2.9106

108 mm4

(2)

210 mm
x
110 mm

Compute Ix'y' by integration. The integration will be


simpler if we first find Ix''y'' and then use the
parallel-axis theorem. That is, by the parallel axis
theorem,

x
210 mm

Ix''y'' = Ix'y' + dx'dy'A

y
210 mm/3 = 70 mm

Thus
Ix'y' = Ix''y'' dx'dy'A
= Ix''y'' (70mm)(70mm)(22050 mm2)
= Ix''y'' 1.0804

108 mm4

210 mm
x

(3)

210 mm/3 = 70 mm
x
210 mm

2112

10.3 Products of Inertia Example 11, page 5 of 6


5

Integrate by using vertical strips. Applying the parallel-axis


theorem to the vertical strip gives
y

dIx''y'' = dIxcyc + xel'' yel" dA


= 0 + x''(y''/2)(y'' dx'')

yc xel = x

Equation of line:
y'' = x'' + 210

Centroid of
infinitesimal strip

= x''( x'' + 210)2 dx''/2


210 mm
Thus

xc y
Ix''y'' =

dIx''y''

x
dx
210 mm

210
=
x'' ( x'' + 210)2 dx''/2
0
= 0.8103

108 mm4

(4)

2113

yel = y /2

10.3 Products of Inertia Example 11, page 6 of 6


6

Substitute in Eq. 3:
Ix'y' = Ix''y''

108 mm4

1.0804
0.8103

(Eq. 3 repeated)

108, by Eq. 4

108 mm4

= 0.2701

(5)

Substitute this result in Eq. 2:


Ixy-triangle = Ix'y' + 2.9106
0.2701
= 2.6405

108 mm4

(Eq. 2 repeated)

108 mm4, by Eq. 5

108 mm4

(6)

The product of inertia for the entire area is the difference between
Ixy-rectangle and Ixy-triangle:
Ixy = Ixy-rectangle
21.3444

Ixy-triangle

108 mm4, by Eq.1

= 1.870

109 mm4

2.6405

108, by Eq. 6
Ans.

2114

10.4 Moments of Inertia About Inclined Axes; Principal Moments

2115

10.4 Moments of Inertia About Inclined Axes; Principal Moments Procedures and Strategies, page 1 of 2
Procedures and Strategies for Solving Problems Involving
Moments of Inertia About Inclined Axes; Principal Moments

Begin by calculating Ix, Iy, and Ixy. Then to determine moments of inertia about
axes rotated with respect to the x and y axes, use the formulas
Ix + Iy
Ix Iy
Iu = 2
+
cos 2
Ixy sin 2
2
Iv =

Ix + Iy
2

Iuv =

Ix

Ix
2

Iy
2

Iy

sin 2

cos 2

Determine Ix, Iy, and Ixy first.


Ixy cos 2

1. Formulas:
Ix + Iy
2

Ixy sin 2

Determine the principal (maximum and minimum) moments of inertia either by


using formulas or by using Mohr's circle.

I max, min =

( Ix

Iy
2

)2 + I2xy

The axes corresponding to Imax and Imin are given by the roots
the equation
2Ixy
tan 2 =
Ix Iy

and

of

To determine which root, 1 or , goes with Imax and which goes with Imin,
substitute 1 into Eq. 1 and see whether the formula gives Imax or Imin.

2116

10.4 Moments of Inertia About Inclined Axes; Principal Moments Procedures and Strategies, page 2 of 2
2. Mohr's circle:
a) Construct a coordinate system in which
the horizontal axis is labeled I (for values of
the moment of inertia, Ix and Iy), and the
vertical axis is labeled Ixy (for values of the
product of inertia).

Ixy
X

Ixy

X
R

Ixy

Iave = (Ix + Iy)/2

b) Plot the point (Ix, Ixy) and label it "X."

Ix
c) Plot the center of the circle, (Iave, 0),
where Iave = (Ix + Iy)/2 = average moment of
inertia.

Radius R =
Ixy

d) Draw a straight line connecting X and the


center, use geometrical relations to calculate
the value, R, of the radius, and then draw the
circle.

( Ix

Iy
2

Iy)/2

)2 + I2xy

Iave
X
Imax = Iave + R

Imin
R

e) Imax and Imin are the far left and far right
points on the circle.

Imax

Imin = Iave
y

f) The principal axes (axes corresponding to


Imax and Imin) can be found by rotating the
x and y axes half the amount of the rotation
in Mohr's circle.

Iave = (Ix

y'

Ixy
X
p

Imax

x
p/2

x'

2117

10.4 Moments of Inertia About Inclined Axes; Principal Moments Problem Statement for Example 1
1. Determine the moments of inertia of the standard rolled-steel
angle section with respect to the u and v axes.
y
0.987 in.
0.5 in.

Ix = 17.40 in4
v

Iy = 6.27 in4
Ixy = 6.08 in.4

6 in.
x
C

45
u

1.99 in.
0.5 in.

4 in.

2118

10.4 Moments of Inertia About Inclined Axes; Principal Moments Problem Statement for Example 2
2. Determine the moments of inertia of the crosshatched area with
respect to the u and v axes for a) = 25 and b) = 90

40 mm
v

220 mm
u

x
100 mm

100 mm
20 mm

2119

10.4 Moments of Inertia About Inclined Axes; Principal Moments Problem Statement for Example 3
3. Determine the value of for which the product of inertia of the
crosshatched area with respect to the u and v axes is zero. Calculate
Iu and Iv for this value of and compare Iu and Iv to Imax and Imin
v

y
x2 + y2 = 1002
Ix = 1.9635 x 107 mm4
u

Iy = 1.9635 x 107 mm4


x

Ixy = 1.2500 x 107 mm4

2120

10.4 Moments of Inertia About Inclined Axes; Principal Moments Problem Statement for Example 4
4. Determine the principal moments of inertia with respect to all
possible rectanglular coordinate systems with their origin at the
centroid C.
y

20 mm
50 mm
C

Ix = 2.2013 x 107 mm4


Iy = 0.9213 x 107 mm4

x
50 mm
20 mm

60 mm

60 mm
10 mm

2121

10.4 Moments of Inertia About Inclined Axes; Principal Moments Problem Statement for Example 5
5. Determine the principal moments of inertia and
principal axes having their origin at point O.
y

150 mm

15 mm
Ix = 1.1714

107 mm4

Iy = 8.9083

107 mm4

30 mm
Ixy = 2.3971
O

2122

107 mm4

10.4 Moments of Inertia About Inclined Axes; Principal Moments Problem Statement for Example 6
6. Determine the principal moments of inertia
and principal axes having their origin at point O.
y

6 in.

O
4 in.

2123

10.4 Moments of Inertia About Inclined Axes; Principal Moments Problem Statement for Example 7
7. Use Mohr's circle to determine the principal moments of
inertia and principal axes having their origin at the centroid C
of the standard rolled-steel channel section.
y
15.3 mm
Ix = 32.6

106 mm4

Iy = 1.14

106 mm4

127 mm

127 mm

2124

10.4 Moments of Inertia About Inclined Axes; Principal Moments Problem Statement for Example 8
8. Use Mohr's circle to determine the principal moments of
inertia and principal axes having their origin at the centroid C
of the standard rolled-steel angle section.
y
0.987 in.
0.5 in.

Ix = 17.40 in4
Iy = 6.27 in4
Ixy = 6.08 in.4

6 in.
x
C
1.99 in.
0.5 in.
4 in.

2125

10.4 Moments of Inertia About Inclined Axes; Principal Moments Problem Statement for Example 9
9. Use Mohr's circle to determine the principal moments of inertia
and principal axes having their origin at the centroid C.
105 mm
y

15 m
Ix = 6.7245

106 mm4

Iy = 6.3520

106 mm4

52.5 mm

80 mm

Ixy = 5.1300

15 mm

7.5 mm
80 mm

15 mm

2126

106 mm4

10.4 Moments of Inertia About Inclined Axes; Principal Moments Problem Statement for Example 10
10. Use Mohr's circle to determine the principal moments of
inertia and principal axes having their origin at point O
y

2 in.

2 in.

2 in.

2 in.

2 in.

2 in.
O

2127

10.4 Moments of Inertia About Inclined Axes; Principal Moments Example 1, page 1 of 3
1. Determine the moments of inertia of the standard rolled-steel
angle section with respect to the u and v axes.
y
0.987 in.
0.5 in.

Ix = 17.40 in4
v

Iy = 6.27 in4
Ixy = 6.08 in.4

6 in.
x
C

45
u

1.99 in.
0.5 in.

4 in.

2128

10.4 Moments of Inertia About Inclined Axes; Principal Moments Example 1, page 2 of 3
1 The formula for Iu is
Iu =

Ix + Iy
I I
+ x y cos 2
2
2

Ixy sin 2

(1)

We can save some work later, if we calculate and save the expressions
Ix + Iy
=
2

17.40 in4 + 6.27 in4


= 11.835 in4
2

Ix

17.40 in4
2

(2)

and
Iy
2

6.27 in4

(3)

= 5.565 in4

Eq. 1 becomes,
11.835 in4, by Eq. 2
Ix + Iy
I I
Iu =
+ x y cos 2
2
2

6.08 in.4
Ixy sin 2

45 ( is negative because the x axis


must be rotated clockwise to make it
coincide with the u axis)

5.565 in4, by Eq. 3


= 5.76 in4

Ans.

2129

10.4 Moments of Inertia About Inclined Axes; Principal Moments Example 1, page 3 of 3
2

Similarly, for Iv and Iuv,


11.835 in4, by Eq. 2
Iv =

Ix + Iy
2

Ix

Iy
2

cos 2

6.08 in.4

45

Ixy sin 2

5.565 in4, by Eq. 3

= 17.92 in4

Ans.
5.565 in4, by Eq. 3
45

Iuv =

Ix

Iy
2

sin 2

Ixy cos 2
6.08 in.4

= 5.57 in4

Ans.

2130

10.4 Moments of Inertia About Inclined Axes; Principal Moments Example 2, page 1 of 5
2. Determine the moments of inertia of the crosshatched area with
respect to the u and v axes for a) = 25 and b) = 90

40 mm
v

220 mm
u

x
100 mm

100 mm
20 mm

Before we can use the equations for Iu, Iv, and Iuv, we must
determine Ix, Iy, and Ixy. Determining Ixy is easy: the y axis is
an axis of symmetry, so
Ixy = 0

(1)

2131

10.4 Moments of Inertia About Inclined Axes; Principal Moments Example 2, page 2 of 5
2 To find Ix and Iy, consider the crosshatched area to be
the sum of two rectangles and calculate Ix and Iy for
each rectangle
y

y
y

y
y

Centroid

20 mm

20 mm
40 mm
Centroid

220 mm

220 mm

120 mm

120 mm

220 mm

220mm/2 = 110 mm

100 mm

100 mm

Ix-rectangle-1 = Ix' + d2A


=

(40 mm)(220 mm)3

= 1.4197

12
108 mm4

Rectangle 2

Rectangle 1
I = bh3/12 for
rectangle about
centroidal axis

20 mm

+ (110 mm)2[(40 mm)(220 mm)


(2)

2132

Iy-rectangle-1

(220 mm)(40 mm)3


=
= 0.0117

12
108 mm4

(3)

10.4 Moments of Inertia About Inclined Axes; Principal Moments Example 2, page 3 of 5
4

Ix-rectangle-2 = Ix' + d2A


=

(240 mm)(40 mm)3


108 mm4

= 5.5424
Iy-rectangle-2 =
=

+ (220 mm + 20 mm)2 (240 mm)(40 mm)

12

(4)

bh3
12
(40 mm)(240 mm)3

12
= 0.4608 108 mm4

(5)

Adding the results for rectangles 1 and 2 gives

0.4608
Iy = Iy-rectangle-1 + Iy-rectangle-2

Ix = Ix-rectangle-1 + Ix-rectangle-2
1.4197

108 mm4, by Eq. 2


= 6.9621

5.5424

108 mm4, by Eq. 3

108 mm4, by Eq. 4

0.0117

(6)

= 0.4725

108 mm4

108 mm4

Substitute for Ix, Iy, and Ixy in the equation for Iu, Iv, and Iuv.
We can save work if we calculate and save the expressions
Ix + Iy
=
2
and
Ix

Iy
2

6.9621

108 mm4 + 0.4725

108 mm4

= 3.7173

(8)

108 mm4

2
6.9621

108 mm4

108 mm4, by Eq. 5

0.4725

108 mm4
= 3.2448

2133

108

mm4

(9)

(7)

10.4 Moments of Inertia About Inclined Axes; Principal Moments Example 2, page 4 of 5
7

Part a): Calculate Iu, Iv, and Iuv for

= 25
25
0, by Eq. 1
Ixy sin 2

Ix + Iy
I I
+ x y cos 2
2
2

Iu =
3.7173

108 mm4, by Eq. 8

= 5.80

Iv =

108 mm4

Ix + Iy
2

Ix
2

Ix

Iy
2

3.2448
= 2.49

sin 2

3.2448

108 mm4, by Eq. 9

Ans.
25
Ixy cos 2

108 mm4, by Eq. 9


108 mm4

0, by Eq. 1

25
Iy
Ixy sin 2
cos 2

108 mm4

= 1.632

108 mm4, by Eq. 9


Ans.

108 mm4, by Eq. 8

3.7173

Iuv =

3.2448

0, by Eq. 1
Ans.

2134

10.4 Moments of Inertia About Inclined Axes; Principal Moments Example 2, page 5 of 5
8

Part b) :

When
the u and y axes coincide, so no transformation
equations are needed. Instead, we have
y, u
Iu = Iy
0.4725
= 0.47

108 mm4

Ans.

Similarly,

90
v

108 mm4, by Eq. 7

Iv = Ix
6.9621
= 6.96

108 mm4, by Eq. 6

108 mm4

Ans.

and
Iuv = Ixy
= 0, by Eq. 1

2135

Ans.

10.4 Moments of Inertia About Inclined Axes; Principal Moments Example 3, page 1 of 4
3. Determine the value of for which the product of inertia of the
crosshatched area with respect to the u and v axes is zero. Calculate
Iu and Iv for this value of and compare Iu and Iv to Imax and Imin
v

y
x2 + y2 = 1002
Ix = 1.9635 x 107 mm4
u

Iy = 1.9635 x 107 mm4


x

Ixy = 1.2500 x 107 mm4

2136

10.4 Moments of Inertia About Inclined Axes; Principal Moments Example 3, page 2 of 4
1 The formula for Iuv is
Iuv =

Ix Iy
sin 2
2

(1)

Ixy cos 2

We can save some work later, if we calculate and save


Ix Iy
=
2

1.9635 x 107
2

1.9635 x 107
= 0

(2)

Setting Iuv equal to zero in Eq. 1 gives,


0, by Eq. 2
0 =

Ix+ Iy
sin 2
2

Solving this equation for


= 45, 45

Ixy cos 2
1.2500 x 107 mm4

leads to two roots:


Ans.

2137

10.4 Moments of Inertia About Inclined Axes; Principal Moments Example 3, page 3 of 4
2

The formula for Iu is


Ix Iy
Ix + Iy
cos 2
+
2
2

Iu =

(3)

Ixy sin 2

We can save some work by calculating and saving


Ix + Iy
1.9635
=
2

107 1.9635
2

107

107

= 1.9635

Substituting numerical values in Eq. 3 gives


1.9635
Iu =

107 mm4

1.2500

Ix + Iy
I I
Ixy sin 2
+ x y cos 2
2
2
0, by Eq. 2

= 0.7135

107 mm4

45

107 mm4

Ans.

Similarly for Iv,


1.9635
Iv =

Ix + Iy
2

= 3.2135

107 mm4

Ix Iy
Ixy sin 2
cos 2
2
0, by Eq. 2
107 mm4

107 mm4

1.2500

45
Ans.

2138

(4)

10.4 Moments of Inertia About Inclined Axes; Principal Moments Example 3, page 4 of 4
Calculate Imax and Imin

107 mm4

1.9635
Imax,min = Ix + Iy
2

Ix
2

Iy )2

1.2500

107 mm4

Ixy2

0, by Eq. 2
= 3.2135

107 mm4, 0.7135

same as Iv

107 mm4

Same as Iu

Thus Iv = Imax and Iu = Imin. In general if the product of inertia, Iuv, is zero for a
given orientation, then the moments of inertia, Iu and Iv, are the maximum and
minimum moments of inertia possible for any orientation.
4

Show the u and v axes' orientation.


y

v
(Axis of maximum
moment of inertia)

x2 + y2 = 1002
u (Axis of minimum moment of inertia)

45
x

2139

10.4 Moments of Inertia About Inclined Axes; Principal Moments Example 4, page 1 of 2
4. Determine the principal moments of inertia with respect to all
possible rectangular coordinate systems with their origin at the
centroid C.
y

20 mm
50 mm
C

Ix = 2.2013 x 107 mm4


Iy = 0.9213 x 107 mm4

x
50 mm
20 mm

60 mm

60 mm
10 mm

Because the x and y axes are axes of symmetry for the crosshatched area
Ixy = 0

(1)

2140

10.4 Moments of Inertia About Inclined Axes; Principal Moments Example 4, page 2 of 2
2 Apply the equation for the principal moments of inertia.
2.2013
Imax,min =

Ix + Iy
2

(Ix
0.9213

= 2.2013

107 mm4

0, by Eq. 1

Iy)/2 2 + Ixy2
107 mm4

107 mm4, 0.9213

107 mm4

Ans.

That is, Ix and Iy are principal moments of inertia. In general, you can
immediately recognize if Ix and Iy are principal moments of inertia by noting if
the product of inertia is zero: Ixy = 0 implies that Ix and Iy are principal moments
of inertia.

2141

10.4 Moments of Inertia About Inclined Axes; Principal Moments Example 5, page 1 of 3
5. Determine the principal moments of inertia and
principal axes having their origin at point O.
y

150 mm

15 mm
Ix = 1.1714

107 mm4

Iy = 8.9083

107 mm4

30 mm
Ixy = 2.3971
O

2142

107 mm4

10.4 Moments of Inertia About Inclined Axes; Principal Moments Example 5, page 2 of 3
1

Apply the formula for the principal moments of inertia,


1.1714
Imax,min =

Ix + Iy
2

Ix
8.9083

= 9.5908
2

107 mm4
Iy )2

2
107 mm4

2.3971

107 mm4

Ixy2

107 mm4, 4.8892

107 mm4

(1)

Apply the formula for the principal directions


2.3971 107 mm4
2Ixy
= (1/2) tan-1
p
Ix Iy
1.1714

107 mm4

8.9083

107 mm4

= 15.8923 and 74.1077


To determine which p value corresponds to Imax and which to Imin,
substitute
15.8923 into the transformation equation for Ix':

Iu =

1.1714 107 mm4


Ix + Iy
I I
+ x y cos 2
2
2
8.9083 107 mm4

= 4.8892

Ixy sin 2
2.3971

107 mm4

107 mm4

= Imin, by Eq. 1

2143

Ans.

10.4 Moments of Inertia About Inclined Axes; Principal Moments Example 5, page 3 of 3
3

Thus a 15.9 counterclockwise rotation of the


x axis would give the axis for which the
quantity
Iu = v2 dA
is smaller than the same integral evaluated for
any other orientation of the axis.
y
Axis of minimum
moment of inertia

15.9
x

74.1

Axis of maximum
moment of inertia

2144

10.4 Moments of Inertia About Inclined Axes; Principal Moments Example 6, page 1 of 4
6. Determine the principal moments of inertia
and principal axes having their origin at point O.
y

6 in.

O
4 in.

2145

10.4 Moments of Inertia About Inclined Axes; Principal Moments Example 6, page 2 of 4
y

Calculate the moment of inertia.


Ix = Ix' + d2A

2 in.

(parallel axis theorem)

(4 in.)(6 in.)3
=
+ (3 in.)2(24 in.2)
12
= 288 in.4
Iy =

6 in.

(1)

(6 in.)(4 in.)3
+ (2 in.)2 (24 in.2)
12

= 128 in.4

(2)

3 in.
O

x
4 in.

Ixy = Ix'y' + dxdy A


2
= 0 + (2 in.)(3 in.)(24 in. )

= 144 in.4
Area: A = (4 in.)(6 in.) = 24 in2

2146

(3)

10.4 Moments of Inertia About Inclined Axes; Principal Moments Example 6, page 3 of 4
2

Apply the formulas for the principal moments of inertia.


288 in.4, by Eq. 1
Imax, min=

Ix + Iy
(Ix Iy)/2
2
128 in.4, by Eq. 2

144 in.4, By Eq. 3

Ixy2

372.73 in.4, 43.27 in.4


3

(4)

Ans.

Apply the formulas for the principal directions


144 in.4, By Eq. 3
p

= 12 tan-1 2Ixy
Ix Iy

288 in.4, by Eq. 1

128 in.4, by Eq. 2

= 30.47 and 59.53


To determine which p value corresponds to max and which to
substitute
30.47 into the transformation equation for Iu:
288 in.4, by Eq. 1
Iu =

Ix + Iy
I I
+ x y cos 2
2
2
4
128 in. , by Eq. 2

Ixy sin 2
144 in.4, By Eq. 3

= 372.73 in.4
= Imax, by Eq. 4

2147

min,

10.4 Moments of Inertia About Inclined Axes; Principal Moments Example 6, page 4 of 4
4 Thus a 30.47 clockwise rotation of the x axis
would give the axis for which the quantity
Iu = v2 dA
is larger than the same integral evaluated for any
other orientation of the axis.
y
Axis of minimum
moment of inertia

59.53

O
30.47

Axis of maximum
moment of inertia

2148

10.4 Moments of Inertia About Inclined Axes; Principal Moments Example 7, page 1 of 3
7. Use Mohr's circle to determine the principal moments of
inertia and principal axes having their origin at the centroid C
of the standard rolled-steel channel section.
y
15.3 mm
Ix = 32.6

106 mm4

Iy = 1.14

106 mm4

127 mm

x
1
127 mm

2149

Because the x axis is an


axis of symmetry, Ixy = 0

10.4 Moments of Inertia About Inclined Axes; Principal Moments Example 7, page 2 of 3
2 Draw the I and Ixy axes.
Ixy
3 Plot the point corresponding to the x axis:
(Ix, Ixy) = (32.6

106, 0).

R
X

C
4

Plot the point C at the center of Mohr's circle:


(Iaverage, 0) = ((Ix + Iy)/2, 0)

Calculate the radius:

= ((32.6

106 + 1.14 x 106)/2, 0)

R = 32.6 x 106

= (16.87

106, 0)

= 15.73

16.87 x 106

106

2150

10.4 Moments of Inertia About Inclined Axes; Principal Moments Example 7, page 3 of 3
6 Draw Mohr's circle.
R = 15.73

Ixy

106
7

Calculate Imax.
Imax = 16.87

Imin

Imax

= 32.6
I

106

106 mm4

Ans.

Calculate Imin.
Imin = 16.87
= 1.14

16.87

106 + 15.73

106

15.73

106

106 mm4

106
9

Principal axes: Because Ix is the same


as Imax and Iy is the same as Imin, the x
and y axes are the principal axes.

2151

10.4 Moments of Inertia About Inclined Axes; Principal Moments Example 8, page 1 of 4
8. Use Mohr's circle to determine the principal moments of
inertia and principal axes having their origin at the centroid C
of the standard rolled-steel angle section.
y
0.987 in.
0.5 in.

Ix = 17.40 in4
Iy = 6.27 in4
Ixy = 6.08 in.4

6 in.
x
C
1.99 in.
0.5 in.
4 in.

2152

10.4 Moments of Inertia About Inclined Axes; Principal Moments Example 8, page 2 of 4
1 Draw the I and Ixy axes.
Ixy
4

Calculate the radius.

17.40

R=

= 8.242

C
R

( 6.08)2 + (5.565)2

6.08
2

Plot the point corresponding to the x axis:

X
(Ix, Ixy) = (17.40, 6.08).

11.835
17.40

11.835 = 5.565

3 Plot the point C at the center of Mohr's circle:


(Iaverage, 0) = ((17.40 + 6.27)/2 , 0)
= (11.835, 0)

2153

10.4 Moments of Inertia About Inclined Axes; Principal Moments Example 8, page 3 of 4
5

Draw Mohr's circle.


6 Calculate Imax.

Ixy

Imax = 11.835 + 8.242

R = 8.242

= 20.1 in.4
C
R

R
6.08
X
11.835

Ans.

8 Determine the orientation of the Imax axis.


= (1/2) tan-1( 6.08 )
5.665

5.565

= 23.8 (Counterclockwise rotation of the x


axis gives the axis of Imax.)
7

Calculate Imin.
Imin = 11.835
= 3.59 in.4

8.242
Ans.

2154

10.4 Moments of Inertia About Inclined Axes; Principal Moments Example 8, page 4 of 4
9

Sketch the principal axes.

y
Axis of minimum
moment of inertia

0.987 in.

Axis of maximum
moment of inertia
23.8
x
C
1.99 in.

2155

10.4 Moments of Inertia About Inclined Axes; Principal Moments Example 9, page 1 of 4
9. Use Mohr's circle to determine the principal moments of inertia
and principal axes having their origin at the centroid C.
105 mm
y

15 m
Ix = 6.7245

106 mm4

Iy = 6.3520

106 mm4

52.5 mm

80 mm

Ixy = 5.1300

15 mm

7.5 mm
80 mm

15 mm

2156

106 mm4

10.4 Moments of Inertia About Inclined Axes; Principal Moments Example 9, page 2 of 4
1 Draw the I and Ixy axes.
2
Ixy

(Ix, Ixy) = (6.7245

106

6.7245

Plot the point corresponding to the x axis:


106, 5.1300

106).

X
5.1300

106

6.5382

I
4

106

Calculate the radius.


R=

6.7245

106

6.5382

(5.1300

106
= 5.1334

= 0.1863

106

3 Plot the point C at the center of Mohr's circle:


(Iaverage, 0) = ((Ix + Iy)/2, 0)
=

(6.7245

= (6.5382

106 + 6.3520
2

106 , 0)

106, 0)

2157

106)2 + (0.1863
106

106)2

10.4 Moments of Inertia About Inclined Axes; Principal Moments Example 9, page 3 of 4
5

Draw Mohr's circle.


6 Calculate Imax.

Ixy

Imax = 6.5382

X
5.1300

= 11.67

106

106 + 5.1334
106 mm4

106
Ans.

R
R

I
R = 5.1334

6.5382

106

8 Determine the orientation of the Imax axis.

106

= (1/2) tan-1 5.1300


0.1863
0.1863

106

106

5.1334

= 44.0 (Clockwise rotation of the


x axis gives the axis of Imax.)

Calculate Imin.
Imin = 6.5382
= 1.40

106 mm4

106
106

106
Ans.

2158

10.4 Moments of Inertia About Inclined Axes; Principal Moments Example 9, page 4 of 4
9

Sketch the principal axes.


y
Axis of minimum moment of inertia

C
44.0

Axis of maximum moment of inertia

2159

10.4 Moments of Inertia About Inclined Axes; Principal Moments Example 10, page 1 of 5
10. Use Mohr's circle to determine the principal moments of
inertia and principal axes having their origin at point O
y

2 in.

2 in.

2 in.

2 in.

2 in.

2 in.
O

2160

10.4 Moments of Inertia About Inclined Axes; Principal Moments Example 10, page 2 of 5
1 Consider the crosshatched area to be the difference
between a large square and a small square.
y

y
2 in.

2 in.

2 in.

3 in.
y
2 in.

3 in.
=

2 in.

6 in.
3 in.

2 in.

2 in.
x

Square 1

2 Calculate the moment of inertia of square 1 about the x


axis (the moment about the y axis will be the same, by
symmetry):

(6 in.)(6 in.)
+ (3 in.)2[(6 in.)(6 in.)]
12
(1)

2161

3 in.
x

Square 2

= 0 + (3 in.)(3 in.) (6 in.)(6 in.)


= 324 in4

= 432 in4

3 Ixy-square-1 = Ix'y' + dxdy A

Ix-square-1 = Ix'-square-1 + d2A


=

2 in.

(2)

10.4 Moments of Inertia About Inclined Axes; Principal Moments Example 10, page 3 of 5
y

4 Calculate the moment of inertia of square 2.

3 in.

Ix-square-2 = Ix'-square-2 + d2A


=

(2 in.)(2 in.)3
+ (3 in.)2 (2 in.)(2 in.)
12

2 in.

= 37.333 in.4

(3)

Ixy-square-2 = Ix'y' + dxdy A

2 in.
O

= 0 + (3 in.)(3 in.)[(2 in.)(2 in.)]


= 36 in4

(4)

5 Calculate the moment of inertia for the composite square.


432 in.4, by Eq. 1
Ix = Ix-square-1 Ix-square-2

x
Square 2

Ixy = Ixy-square-1 Ixy-square-2


324 in.4, by Eq. 2

37.333 in.4, by Eq. 3

= 288 in.4

= 394.667 in.4

(5)

Iy = Ix, by symmetry
= 394.667 in.4

(6)

2162

3 in.

36 in.4, by Eq. 4
(7)

10.4 Moments of Inertia About Inclined Axes; Principal Moments Example 10, page 4 of 5
7

Draw the I and Ixy axes


Ixy

8
394.667

(Ix, Ixy) = (394.667, 288)

288
C

9
10 Calculate the radius
R = 288

Plot the point corresponding to the x axis:

Plot the point C at the center of Mohr's circle:


(Iaverage, 0) = ((Ix + Iy)/2, 0)
= ((394.667 + 394.667)/2, 0)
= (394.667, 0)

2163

10.4 Moments of Inertia About Inclined Axes; Principal Moments Example 10, page 5 of 5
R = 288

11 Calculate Imax and Imin

Ixy

Imax = 394.667 + 288


X
= 683 in.4
Imin = 394.667
C
Imin

Imax

Ans.
288

= 106.7 in.4

Ans.

12 Clockwise rotation of the x axis 90/2 = 45 gives the


axis of Imax.
y

394.667
Ixy

Axis of
minimum
moment of
inertia

90
Imin

Imax

I
O

2164

x
45
Axis of maximum moment of inertia

11. Energy Methods

2165

11.1 Virtual Work

2166

11.1 Virtual Work Procedures and Strategies, page 1 of 2


Procedures and Strategies for Solving Problems Involving Virtual
Work
1. Identify a single coordinate, q, that will completely define the
configuration (position of all parts) of the machine being analyzed
(The method of virtual work is often the best method to use when
you are analyzing a machine or mechanism, rather than a rigid
structure).

A
q=
k

B
Ay

2. Draw a free-body diagram of the machine.


3. On the free-body diagram, superimpose a dashed-line sketch of the
position of the machine when the coordinate q is increased a small
amount, q.

Ax

4. Identify the active forces, that is, the forces that do work when q is
increased by an amount, q.

mg

Ax and Ay are not


active forces
because they do not
move and thus do
no work.

Fspring

5. Introduce coordinates, xi, measured from a fixed point to the point


of application of each active force.

7. Relate each xi to q by using relations derived from the geometry


of the mechanism. Often these relations between differentials can be
derived by differentiating equations relating xi to q.

2167

x1

x1

6. Write the virtual work equation, W = 0. W is the total work done


by each active force when its coordinate xi is increased a small
amount. Assign a positive sign to the work if the force and
incremental displacement are in the same direction, a minus sign if
in opposite directions.

x1
B
x2

x2

L/2
L/2
B
x2 = L sin
x2 = L cos

11.1 Virtual Work Procedures and Strategies, page 2 of 2


8. Substitute, in the virtual work equation, W = 0, the expression for each
xi in terms of q, and then factor out q from each term in the equation.
The coefficient of q must equal zero if W is to be zero for arbitrary
values of q. This condition gives an equation from which you can find
the unknown force (or unknown value of q, if that is what is requested in
the problem statement).
Notes:
a) A good way to account for the effect of the force F in a spring is to
remove the spring and replace it by a pair of forces of magnitude F.
The virtual work of each of these forces is then F xi, where xi is the
virtual displacement of the points where the ends of the spring were
attached. After the virtual work equation has been derived, then you
can replace F by ks, where s is the extension of the spring. Note that
you do not apply a virtual displacement to s.
b) If you find that it is very difficult to derive the relations between the
virtual displacements xi and q, then probably you should analyze
the machine by drawing free-body diagrams and writing equilibrium
equations for the separate parts of the machine.
c) The above procedure applies to one degree-of-freedom machines,
that is, to machines the position of which can be described by a single
coordinate, q. For systems with more than a single degree of freedom,
multiple coordinates qi must be defined. The steps described above
can then be followed.

2168

q1 =

q2 =

11.1 Virtual Work Problem Statement for Example 1


1. Determine the force P required to keep the two rods in
equilibrium when the angle = 30 and weight W is 50 lb.
The rods are each of length L and of negligible weight.
They are prevented from moving out of the plane of the
figure by supports not shown.
B
L

L
W

Smooth surface

2169

11.1 Virtual Work Problem Statement for Example 2


2. Determine the value of moment M required to
maintain the mechanism in the position shown,
if = 35 and W = 200 lb.
A

C
M

2 ft
2 ft
W
B

2170

11.1 Virtual Work Problem Statement for Example 3


3. Determine the value of the weight W
required to maintain the mechanism in the
position shown, if P = 50 N.
3m

2m

3m

3m

D
E

P
A
B

2171

11.1 Virtual Work Problem Statement for Example 4


4. Determine the force Q necessary to maintain
equilibrium when force P = 400 N.
A
P
300 mm
250 mm C
B

150 mm
250 mm
E

300 mm
400 mm

2172

11.1 Virtual Work Problem Statement for Example 5


5. Link AB is connected to collar A, which can slide
with negligible friction on horizontal rod EF. Determine
the value of force Q necessary to maintain equilibrium
when = 50, L = 300 mm, and P = 100 N.

A
E

Q
P
L

L/2
B

L/2
D

2173

11.1 Virtual Work Problem Statement for Example 6


6. Rotating the threaded rod AC of the automobile jack causes
joints A and C to move closer together, thus raising the weight W.
Determine the axial force in the rod, if = 30 and W = 2 kN.
W
B
150 mm

150 mm
C

150 mm

150 mm
D

2174

11.1 Virtual Work Problem Statement for Example 7


7. The original length of the spring is L. Determine the
angle for equilibrium if L = 3 m and P = 300 N.
A

C
L

L
Spring constant,
k = 200 N/m
E

D
k
L

F
P

2175

11.1 Virtual Work Problem Statement for Example 8


8. Collars A and B can slide freely on rods CD and CE. Determine the
values of x and y, given that forces P = 900 N and Q = 800 N. The
unstretched length of the spring is 0.2 m, and the weight of the collars is
negligible.
x
P
A

9 kN/m

Q
E

2176

11.1 Virtual Work Problem Statement for Example 9


9. Determine the moment M applied to the crankshaft that
will keep the piston motionless when a pressure p = 400 psi
acts on the top of the piston and = 25. The diameter of
the piston is 3 in., and the piston slides with negligible
friction in the cylinder.
p
C

9 in.

B
4 in.

M
A

2177

11.1 Virtual Work Problem Statement for Example 10


10. Pin B is rigidly attached to member AC and moves in the smooth
quarter-circle slot EF. Determine the value of force Q necessary to
keep the system in equilibrium, if = 30, L = 400 mm, a = 120 mm,
and P = 200 N.

L/2
E

L/2
a
Q

2178

11.1 Virtual Work Problem Statement for Example 11


11. A scissors lift is used to raise a weight W = 800 lb.
Determine the force exerted on pin F by the hydraulic
cylinder AF when = 35. Each linkage member is 2-ft
long and pin connected at its midpoint and endpoints. The
lift consists of two identical linkages and cylinders the one
shown and one directly behind it.

W
K

G
F

D
C

2179

11.1 Virtual Work Problem Statement for Example 12


12. The unstretched length of the spring is 1 m. Determine
the value of for equilibrium when force P = 2 kN.
3m

A
1.5 m
k

1.5 kN/m

2m

4m
D

2180

11.1 Virtual Work Problem Statement for Example 13


13. a) Determine the moment reaction at the wall F.
b) Determine the force reaction at the roller D.
In both cases P = 60 lb.
Hinges

D
A

5 ft

5 ft

5 ft

5 ft

10 ft

2181

11.1 Virtual Work Problem Statement for Example 14


14. Determine the vertical reaction at
support C, if P = 2 kN.

C
P
3m
B

4m

A
3m

2182

11.1 Virtual Work Problem Statement for Example 15


15. Determine the vertical reaction at
support I of the truss, if P = 10 kip = Q.

P
A

Q
B
5 ft

C
5 ft

D
5 ft

E
5 ft

F
5 ft

G
5 ft

2183

H
5 ft

5 ft

11.1 Virtual Work Problem Statement for Example 16


16. Determine the tension in the cord. The pulleys are
frictionless and m = 90 kg.

D
m

2184

11.1 Virtual Work Problem Statement for Example 17


17. Determine the equilibrium values of and for the
two-bar linkage. The couple moment M = 5 N m; each bar is
uniform and has a mass m of 5 kg; the length L = 400 mm; and
the unstretched length of the spring is 250 mm.
500 mm
A

L
2

k = 0.2 kN/m
B
D
L

M
C

2185

11.1 Virtual Work Example 1, page 1 of 5


1. Determine the force P required to keep the two rods in
equilibrium when the angle = 30 and weight W is 50 lb.
The rods are each of length L and of negligible weight.
They are prevented from moving out of the plane of the
figure by supports not shown.
B
L

L
W

Smooth surface

2186

11.1 Virtual Work Example 1, page 2 of 5


B
1
L

L
W

The system has one degree of freedom,


because specifying the value of a single
coordinate, , completely determines the
configuration (shape) of the system.
Consider a free-body diagram and identify
the active forces those forces that would
do work if were increased slightly.

Free-body diagram (The dashed line shows the position


of the system after has been increased a small amount.)

4 The force P does work as


point A moves to the right,
so P is an active force.
P
A

2 The force W does work because point B


moves up, so W is an active force.

Cx

Cy

N
5 The normal force N does no work
because it is perpendicular to the
displacement of point A. Thus N
is not an active force.

2187

3 The reactions Cx and Cy do no


work because point C does not
move. Thus Cx and Cy are not
active forces.

11.1 Virtual Work Example 1, page 3 of 5


6

Introduce coordinates measured from a fixed point,


point C in the figure, to the point of application of the
active forces.

yB

yB
P

C
Cx

Compute the work done when the coordinates are


increased positive infinitesimal amounts, xA and xB
(The custom followed by textbook writers is to use the
Greek letter rather than simply writing dxA and dyB
because the infinitesimals represent hypothetical
motions motions that are possible but are not
necessarily motions that actually occur). The principle
of virtual work says that the total work must add to zero
for all possible motions, real or hypothetical that is,
"virtual."
U = 0:

N
xA

Cy

P xA

W yB = 0

(1)

A negative sign is present because the force P and


displacement xA are in opposite directions. That is, P
does negative work (absorbs work from the system
rather than adding work to the system). Similarly, the
force W does negative work because W points down
and yB is directed up.

xA

2188

11.1 Virtual Work Example 1, page 4 of 5


8 Relate the differentials xA and yB through the change
in the angle, : From the figure, it follows that
yB = L sin

B
L

(2)

To relate yB to , use the ordinary formula from


calculus for calculating a differential: if y = f( ), then the
differential is

L
C

A
xA

dy = ddf d
Applying this formula to Eq. 2 and using
gives
yB = L cos

rather than d

(3)
9 or,

Similarly

(2P sin

xA = 2L cos
xA = 2L sin

Because L

(4)

2P sin

Substitute Eqs. 3 and 4 for yB and xA into the


virtual-work equation:

W yB = 0

W cos )(L

W cos

P = 43.3 lb

(Eq. 1 repeated)

L cos

2189

)=0

0, it follows that
=0

Substituting the given values


and solving gives

2L sin
P xA

yB

= 30 and W = 50 lb

Ans.

11.1 Virtual Work Example 1, page 5 of 5


10 Observation: the forces acting between the rods at pin B never
occurred in the virtual-work equation because the work done
by the equal-and-opposite force pairs acting between the parts
of the body cancel out for example the work done by Bx
acting on rod AB has the opposite sign of the work done by Bx
acting on rod BC. Forces such as Bx and By would have had to
be considered if equilibrium equations rather than virtual work
had been used.

Free body of AB
By

11 Often virtual work is easier to use than equilibrium


equations for problems involving connected rigid
bodies (typically machines and mechanisms), but
this advantage exists only if the relation between
displacements can be found easily. If the geometry
is difficult, then using equilibrium equations is
probably the better approach.

Free body of BC plus pin at B


By

Bx

Bx

50 lb

Cy

2190

Cx

11.1 Virtual Work Example 2, page 1 of 3


2. Determine the value of moment M required to
maintain the mechanism in the position shown,
if = 35 and W = 200 lb.
A

C
M

2 ft
2 ft
W
B

2191

11.1 Virtual Work Example 2, page 2 of 3


A

C
M
2 ft

2 ft

1 The system has one degree of freedom: specifying the


value of the single coordinate, , completely
determines the configuration of the system. Consider a
free-body diagram and identify the active forces,

W
B

Free-body diagram (The dashed line shows the position of


the system after has been increased a small amount.)

2 The reactions at A and


C do no work because
points A and C do not
move. Thus the
reactions are not active
forces.

Ax

Cx

C
M

W
Ay
B

Cy

D
4 The weight W of the
block does work because
the center of gravity of
the block moves
vertically; thus the
weight is an active force.

2192

Couple-moment M does
work because member CD
rotates, so M is an active
"force" (better said, "an
active moment" or "active
generalized force")

11.1 Virtual Work Example 2, page 3 of 3


5

Introduce a coordinate y measured from the fixed


point A to the point of application of the force W.
Compute the work done when y and
positive infinitesimal amount.

Ax

Cx

are increased a

y
Ay

U=

+W y=0

Cy

(1)
B

Note that the work done by a moment equals moment


times angle of rotation. Here the work is negative
because M and
have opposite senses.

y
C

Next use geometry to relate the y and :


y = (2 ft) sin

y
2 ft

Differentiating gives
y = 2 cos

(2)

Substitute Eq. 2 for y into Eq. 1.


M

+W y=0

6
(Eq. 1 repeated)

Substituting the given values W = 200 lb and


= 35 into Eq. 3 and noting
0 gives
M

2 cos

2(200 lb) cos 35 = 0

Solving gives

Thus
( M + 2W cos )

=0

M = 328 lb ft

(3)

2193

Ans.

11.1 Virtual Work Example 3, page 1 of 4


3. Determine the value of the weight W
required to maintain the mechanism in the
position shown, if P = 50 N.
3m

2m

3m

3m

D
E

P
A
B

2194

11.1 Virtual Work Example 3, page 2 of 4


1

The system has one degree of freedom because if the


displacement of one end of a bar is known, the
displacement of the other bars can be found by similar
triangles (as will be shown below). Consider a free-body
diagram and identify the active forces corresponding to a
small change in configuration of the system.

D
E

P
A
B

Free-body diagram (The dashed line shows the


position of the system after the bars have been
displaced a small amount.)
Ex

E
F
3
Ey

Bx

The force P does work as point F moves


vertically, so P is an active force.

By

2 The force W does work


if A moves vertically, so
W is an active force.

4 The reaction forces at B and E


do no work because B and E do
not move; thus the reactions are
not active forces.

2195

11.1 Virtual Work Example 3, page 3 of 4


5 Introduce coordinates measured from a fixed point to
yF
the point of application of the active forces.
D

yF
F

E
yA

yA
A

7
yF

D 2m

yD

3m

Now relate the differentials yA and yF. By


similar triangles

yD/2 = yF/3

(2)

yA/3 = yC/3

(3)

and
yA
A
6

B
3m

3m

C
yC

Member DC does not change length so ends C


and D move down the same amount, that is,

Compute the work done when the coordinates


are increased a positive infinitesimal amount.
U = W yA + P yF = 0

yC = yD

(1)

(4)

Eqs. 2, 3, and 4 imply

The force W does negative work because it is


directed down, while the displacement is up.

yA= (2/3) yF

2196

(5)

11.1 Virtual Work Example 3, page 4 of 4


8

Substitute Eq. 5 into the virtual-work equation, Eq. 1:


W yA + P yF = 0

(Eq. 1 repeated)

(2/3) yF
Thus
W(2/3) + P
or, since yF

yF = 0

0 and P is given as 50 N,

W(2/3) + 50 N = 0
Solving gives
W = 75 N

Ans.

2197

11.1 Virtual Work Example 4, page 1 of 7


4. Determine the force Q necessary to maintain
equilibrium when force P = 400 N.
A
P
300 mm
250 mm C
B

150 mm
250 mm
E

300 mm
400 mm

2198

11.1 Virtual Work Example 4, page 2 of 7


A

300 mm
B

The system has one degree of freedom because if member ABC is


rotated about point B a small amount, then the position of CD and
DEFG can be determined. Consider a free-body diagram and identify
the active forces corresponding to the displacements shown.

250 mm C
Free-body diagram (The dashed line shows the position of the
system after the bars have been displaced a small amount.)
A
P

150 mm
250 mm
E

Q
300 mm
400 mm
F

P does work so
is an active
force.
Bx

Ey

By

2199

Points B and E do not


move, so the reactions at
these points do no work.

Ex

F
4

Q does
work so is an
active force.

11.1 Virtual Work Example 4, page 3 of 7


5

Introduce coordinates measured from a fixed point


to the point of application of the forces.
xA
xA

Bx

Ey

B
D

U = 0: P xA + Q yG = 0

E
Ex

By

Q
yG

Compute the work done when the coordinates are


increased a positive infinitesimal amount.

yG

2200

(1)

11.1 Virtual Work Example 4, page 4 of 7


7

Relate the differentials xA and yG. Begin by


noting that because A is a small angle, the tangent
of A can be replaced by the angle itself:
A=

xA
300 mm

Member ABC is a rigid body, and all parts


must rotate the same amount. Thus
C

(2)

C
A

300

Substituting for

A xA

250

from Eq. 2 then gives

xA
= 300 mm

Again using the small angle approximation for


the tangent gives
yC = (250 mm)

C
yC

xA
= (250 mm) 300 mm

150
D
yD

250

= (5/6) xA
10 Member CD is a rigid body and thus doesn't
shorten or lengthen. It follows that

All dimensions are in mm.

300

yD = yC

yG
F

400

Thus

yD = (5/6) xA

2201

(3)

11.1 Virtual Work Example 4, page 5 of 7


A xA

11 Using the small angle approximation for the


tangent gives

300

yD
= 250 mm
[(5/6) xA], by Eq. 3

250

xA
= 300 mm

C
yC
150
D
yD

Member DEFG is a rigid body and so all parts


must rotate the same amount. Thus

250
D

All dimensions are in mm.

300

xA
= 300 mm

12 Thus

yG
F

400

2202

(4)

11.1 Virtual Work Example 4, page 6 of 7


13 The remaining step is to relate yG to F. We can do
this in two ways, by geometry or by calculus. Let's begin
with the geometric approach. First consider a rotation of
line EG by an amount F.
14 Length =

(300 mm)2 + (400 mm)2 = 500 mm

15 (500 mm)

300 mm

400 mm G

16

yG is the vertical distance that point G moves up.


So, considering the small triangle gives
400 mm
4
=
5
500 mm
| yG| = (500

F)

sin

xA
, by Eq. 4
300
or,
yG =

4 xA
3

(5)

(Insert a minus sign, because yG was originally


defined as positive down)

2203

11.1 Virtual Work Example 4, page 7 of 7


17 Consider an alternative solution for yG,
based on calculus:
E

19 Note that we can't write


yG = 300 mm

yG

500 mm

300 mm
F

400 mm

and then differentiate to get yG (which would give yG = 0).


The equation for yG must define a continuous and differentiable
function, not a relationship that is only valid at a single value
of .

18 yG = (500 mm) cos


yG = 500 sin
= 500(

= F because both angles


measure the rotation of line EG)

400 mm
500 mm )

400 xA
300
4 xA
3

20 Substitute for yG from Eq. 5 into the virtual work equation:


P xA + Q yG

=0

(Eq. 1 repeated)

xA , by Eq. 4
300

4 xA , by Eq. 5
3

or,

[P + ( 4/3)Q] xA = 0
(Same as Eq. 5)

Dividing through by xA and using the given value P = 400 N


yields
Q = 300 N

2204

Ans.

11.1 Virtual Work Example 5, page 1 of 4


5. Link AB is connected to collar A, which can slide
with negligible friction on horizontal rod EF. Determine
the value of force Q necessary to maintain equilibrium
when = 50, L = 300 mm, and P = 100 N.

A
E

Q
P
L

L/2
B

L/2
D

2205

11.1 Virtual Work Example 5, page 2 of 4


A
E

Q
P
L

L/2
B

1
Ay
Q

L/2
D

Free-body diagram showing active forces


corresponding to a small increase in

The force from the rod acting on


the collar is not an active force.

Active force
Active force
P
B

Dy
Forces from pin D are
not active forces.
D

2206

Dx

11.1 Virtual Work Example 5, page 3 of 4


2

Introduce coordinates measured from the fixed point D


to the point of application of the active forces.

4 Relate the differential xA to


the change in angle,
A

Ay
Q

A
L
P

yC

xA

Dy

B
Dx

xA

L/2

xA
5

yC

L/2

L
L
+ 2 + 2
L
L
xA = L sin
+ 2 + 2

xA = L cos

(2)

0
0
(Length L does not change)
3

Compute the work done when the coordinates are


increased a positive infinitesimal amount.
U = 0: Q xA + P yC = 0

(1)

2207

11.1 Virtual Work Example 5, page 4 of 4


A

Substitute Eqs. 2 and 3 for xA and yC into the virtual


work equation, Eq. 1:
L sin

yB

Q xA + P yC = 0
B

L/2

L/2

(Eq. 1 repeated)

L cos
2

yC

yB

, by Eq. 3

Thus
( Q sin

, by Eq. 2

Relate the differential yC to the change in


angle
From the figure, we have

Because L

yB = L sin

Q sin

yB = L cos

Substituting
gives

+ P cos
2

)(L

)=0

0, it follow that
+ P cos
2

=0

= 50 and P = 100 N and solving for Q

By similar triangles,
Q = 42.0 N
L cos
yB
L/2 + L/2

yC
L/2

Thus
yC =

L cos
2

(3)

2208

Ans.

11.1 Virtual Work Example 6, page 1 of 5


6. Rotating the threaded rod AC of the automobile jack causes
joints A and C to move closer together, thus raising the weight W.
Determine the axial force in the rod, if = 30 and W = 2 kN.
W
B
150 mm

150 mm
C

150 mm

150 mm
D

2209

11.1 Virtual Work Example 6, page 2 of 5


1

The system has one degree of freedom because once


is specified, the location of all parts of the jack can be
determined. Consider a free-body diagram of the jack
and identify the active forces corresponding to a small
change in .
Free-body diagram

2 To get a virtual-work equation that contains the


axial force in the rod, it is necessary to exclude the
rod from the free-body diagram. The effect of the
rod is then represented by the two forces Fr. W and
the two Fr forces are the active forces.

W
B

Fr

Fr

2210

11.1 Virtual Work Example 6, page 3 of 5


3

Introduce coordinates measured from fixed points to


the points of application of the active forces.
W
B

yB

Calculate the work done.


U = 0:

yB

Fr

Fr

D
xA

xA

xC

xC

2211

W yB

Fr xA

Fr xC = 0

(1)

11.1 Virtual Work Example 6, page 4 of 5


Relate yB, xA, and xC through the angle
change, d .

B
a

E
150 mm

Distance "a," from the intersection of the two sloping


members, point E, to point B, does not change as
changes. Thus when "a" is differentiated, the result is
zero: a = 0.

150 mm
W

yB

C
150 mm

150 mm
C

A
D
xA
7

xC

xA = (150 mm) cos


xA = 150 sin

(2)

xC = (150 mm) cos


xC = 150 sin

(3)

yB = 2(150 mm) sin


yB = 300 cos

+a

+ a

(4)
0

2212

11.1 Virtual Work Example 6, page 5 of 5


8

Substituting the expressions for yB, xA, and xC and into the
virtual work equation, Eq. 1, gives
300 cos

by Eq. 4

W yB

Fr xA

150 sin

150 sin
Fr xC = 0

by Eq. 3
(Eq. 1 repeated)

by Eq. 2

or,
[ 300W cos

+ 2(150)Fr sin ]

=0

Dividing through by , substituting the given values W = 2 kN


and = 30, and solving gives
Fr = 3.46 kN

Ans.

2213

11.1 Virtual Work Example 7, page 1 of 5


7. The original length of the spring is L. Determine the
angle for equilibrium if L = 3 m and P = 300 N.
A

C
L

L
Spring constant,
k = 200 N/m
E

D
k
L

F
P

2214

11.1 Virtual Work Example 7, page 2 of 5


A

The system can be described by a single coordinate,


Consider a free-body diagram and identify the active
forces corresponding to a small change in

L
A

Free-body diagram
C
Ay
L

By

Reactions Ay and By do
no work because they
are perpendicular to the
displacement of points
A and B.

L
C
E

D
k
L

D
4

F
P

Forces P and Fs are


active forces because
their points of
application move in
the direction of the
forces.

Fs Fs

F
P

2215

3 The spring is not part of the


free-body; the effect of the
spring is represented by the
E
forces Fs.

11.1 Virtual Work Example 7, page 3 of 5


5

Introduce coordinates measured from the point O directly


above pin C to the point of application of the active
forces.

B
O

Compute the work done when the coordinates are


increased a positive infinitesimal amount.
U = P yF + Fs xD + Fs xE = 0

Ay

By

(1)

yF

Fs Fs

yF

P
xD

2216

xD

xE

xE

11.1 Virtual Work Example 7, page 4 of 5


7

Relate the differentials yF, xD, and xE to the angle


change
From the figure, we see that
yF = 3L sin

xE = L cos
xD = L cos

Differentiating gives
yF = 3L cos

(2)

xE = L sin

(3)

xD = L sin

(4)

yF

We can use the same figure to calculate the length of the


spring, L , say:

L = distance DE
= 2L cos

(5)

F
xD

2217

xE

11.1 Virtual Work Example 7, page 5 of 5


8 The force in the spring is, then,
Fs = k

compression of spring

=k

(original length
L (given)

= kL(1

final length)

L = 2L cos , by Eq. 5

2 cos )

(6)

Substitute from Eqs. 2, 3, 4, and 6 into the virtual


work equation, Eq. 1:
kL(1 2 cos ), by Eq. 6
P yF + Fs xD + Fs xE = 0
3L cos

by Eq. 2

L sin

(Eq. 1 repeated)

by Eq. 3

L sin

by Eq. 4

Thus
3P cos
or, since L
3P cos

2kL(1

2 cos ) sin

(L

)=0
9

0,
2kL(1 2 cos ) sin

=0

(7)

Substituting the given values P = 300 N,


k = 200 N/m, and L = 3 m into Eq. 7 and
solving numerically gives
= 69.1

2218

Ans.

11.1 Virtual Work Example 8, page 1 of 5


8. Collars A and B can slide freely on rods CD and CE. Determine the
values of x and y, given that forces P = 900 N and Q = 800 N. The
unstretched length of the spring is 0.2 m, and the weight of the collars is
negligible.
x
P
A

9 kN/m

Q
E

2219

11.1 Virtual Work Example 8, page 2 of 5


1

x
P
A

The system has two degree of freedom since both x and


y coordinates must be known if the configuration of the
system is to be determined. Consider a free-body
diagram and identify the active forces corresponding to
a small change in x, while y is held fixed.

Free-body diagram
y

9 kN/m

The forces acting on


collar A, Fs (the spring
force) and the 900-N
force, are the only active
forces.

900 N
A
Fs
NA (Force from rod CD)

Fs
3
Q
E

Because y is fixed,
collar B does not
move, and so none
of the forces acting
on B is an active
force.

2220

NB (Force from rod CE)

800 N

11.1 Virtual Work Example 8, page 3 of 5


4 The coordinate x locates the position of the
700-N force. Introduce an additional
coordinate, L, that locates the point of
application of the spring force, Fs.

Compute the work done:


U = 0: (900 N) x

Fs L = 0

Relate x and L:

x
x

L2 = x2 + y2
A

(1)

900 N

(2)

Differentiating gives
2L L = 2x x + 2y y

L
Fs

0 (Because y is fixed)
Thus
NA
L = x/L x

Introduce the latter equation into Eq. 1:

Fs
L
B

(900 N) x

Fs L = 0

(Eq. 1 repeated)

NB
x/L x

y 0
(y held fixed)

Thus
(900

Fsx/L) x = 0

Dividing through by x and re-arranging gives


800 N

Fsx = 900L

2221

(3)

11.1 Virtual Work Example 8, page 4 of 5


x(fixed)

Next, hold x fixed and compute the work done when


collar B moves an amount y. Following the same
steps as were used for x leads to

Fsy = 800L

900 N

(4)

The spring force, Fs, is related to L:

Fs

Fs = k

NA
y

extension of the spring

= (9000 N/m)

original length)
0.2 m

Fs

Thus

L
B

(L

NB

Fs = 9000L

1800

(5)

y
We now have four simultaneous nonlinear equations
to solve:
L
800 N

L2 = x2 + y2

(2)

Fsx = 900L

(3)

Fsy = 800L

(4)

Fs = 9000L

2222

1800

(5)

11.1 Virtual Work Example 8, page 5 of 5


7 These equations can be solved directly if a calculator that
is able to handle such systems is available.

Distance x can now be found from Eq. 3:


Fsx = 900L

Alternatively, proceed as follows: square both sides of


Eqs. 3 and 4 and add the results to get

1204 N

(Fsx)2 + (Fsy)2 = (900L)2 + (800L)2.

(Eq. 3 repeated)
0.3338 m

Solving gives
x = 0.250 m

or
Fs2(x2+ y2) = L2(9002 + 8002)

Distance y can be found from Eq. 4:

L2, by Eq. 2

Fsy = 800L
1204 N

Solving gives

(Eq. 4 repeated)
0.3338 m

Solving gives

Fs = 1204 N

y = 0.222 m

Using this result in Eq. 5 gives


Fs = 9000L

Ans.

1800

(Eq. 5 repeated)

1204 N
Solving gives
L = 0.3338 m
.

2223

Ans.

11.1 Virtual Work Example 9, page 1 of 4


9. Determine the moment M applied to the crankshaft that
will keep the piston motionless when a pressure p = 400 psi
acts on the top of the piston and = 25. The diameter of
the piston is 3 in., and the piston slides with negligible
friction in the cylinder.

1 The system can be described by a single coordinate, .


Consider a free-body diagram and identify the active
forces corresponding a small change in .
2 The resultant of the pressure is an active force:

(400 psi)( )(3 in./2)2 = 2827 lb


C
Free-body diagram
C
N

9 in.
3

B
4 in.

Since friction is negligible, only


the normal force N acts on the
side of the piston. The normal
force does no work since it acts
perpendicular to the motion of
the piston.

B
A

M
Ax
A
Ay

2224

The reaction forces Ax and Ay


do no work, because point A
does not move. The moment M
does work as link AB rotates.

11.1 Virtual Work Example 9, page 2 of 4


5

Introduce coordinates measured from


a fixed reference at point A.
6

2827 lb

Compute the work done when the coordinates are increased


a positive infinitesimal amount.
U = (2827 lb) yC

yC
N

yC
d
B
M
Ax

Ay

2225

=0

(1)

11.1 Virtual Work Example 9, page 3 of 4


7

Distance "a" does not


change as is
changed so a = 0.

Relate yC to
yC = (4 in.) cos
yC = 4 sin

+ (9 in.) cos
9 sin

+a
+ a

(2)

Relate

to

2827 lb

by the law of sines,

sin
4 in.

sin
(3)

9 in.

yC

9 in.

Differentiating gives
B
cos

cos
4 in.

Thus

Ax
=

4 cos

A
(4)

9 cos
Ay

2226

11.1 Virtual Work Example 9, page 4 of 4


9

Using Eq. 4 in Eq. 2 gives


yC = 4 sin

10 Using
9 sin

(Eq. 2 repeated)
4 cos
9 cos

= ( 4 sin

4 tan

M=

cos )

=0

( 4 sin

cos )

by Eq. 5

or
[4(2827)(sin

+ tan

Dividing through by

cos )

=0

and solving gives

M = 4(2827)(sin
Substituting the given value

tan

cos )

which can be solved to give

(6)

= 25 into Eq. 3 yields

sin
sin
=
9 in.
4 in.

0 lb in

Ans.

11 Observation: this problem may be more easily


solved by using equations of equilibrium.
Virtual work is superior to using equations of
equilibrium provided that the relation between
displacements can be easily obtained. In the
present problem, deriving the relation between
and yC, Eq. 5, was more complicated than
simply writing equations of equilibrium.

(5)

(Eq. 1 repeated)

4 tan

= 10.83 in Eq. 6 produces

, by Eq. 4

Substituting Eq. 5 in the virtual work equation gives


(2827) yC

= 25 and

(Eq. 3 repeated)

= 10.83.

2227

11.1 Virtual Work Example 10, page 1 of 5


10. Pin B is rigidly attached to member AC and moves in the smooth
quarter-circle slot EF. Determine the value of force Q necessary to
keep the system in equilibrium, if = 30, L = 400 mm, a = 120 mm,
and P = 200 N.

L/2
E

L/2
a
Q

2228

11.1 Virtual Work Example 10, page 2 of 5


C
L/2
E

1 The system configuration can be


defined by the single coordinate, .
Consider a free-body diagram
showing the active forces
corresponding to a small increase in

L/2
a
A

F
C

D
Free-body diagram

Pin B must move in the slot, that is, in a direction tangent to


the quarter circle and thus perpendicular to radius DB .
Thus the force R from the slot does no work because R is
perpendicular to the motion of the pin.

Point A must move


horizontally. It is difficult
to tell if A moves to the
right or left, but fortunately Q (active) A
it makes no difference. The
important thing is to note
that force Ay from the
rollers does no work so is
Ay
not an active force.

2229

P (active)
B

4 Point C moves both


horizontallly and vertically.
It is difficult to tell if C
Motion of pin B
R
moves vertically up or
is perpendicular
vertically down, but it
to BD.
makes no difference. All
we need to note is that
force P is an active force.

11.1 Virtual Work Example 10, page 3 of 5


C
5

yC

Introduce coordinates measured from


the fixed point D to the point of
application of the active forces P and Q.
P
B
yC
R

A
D
6 Compute the work done when the coordinate are
increased a positive infinitesimal amount.

Ay
xA
xA

U = 0:

2230

Q xA + P yC = 0

(1)

11.1 Virtual Work Example 10, page 4 of 5


C

7 Relate the differentials xA and yC to the angles


Begin with yC.

and

yC = L sin

L/2

yC = L cos

yC

(2)

Note that this equation shows yC is positive if


is
positive, that is, point C moves up as increases.

L/2
a
9

Relate

xA = (L/2) cos
xA = (L/2) sin

+ a sin

xA = (L/2) sin

(3)

a sin

sin
a
) = sin , the last equation can

= (L/2) sin

(4)

Differentiating gives
+ a sin

(Eq. 3 repeated)
L cos
2a cos

+ cos

Because sin (180


be written as

a cos

10 Using Eq. 5 in Eq. 3 gives

= ( sin

through the law of sines:

sin (180
L/2

xA
8

and

tan )(L/2)

, by Eq. 5

a cos
so
=

(6)

2231

= (L/2) cos

L cos
2a cos

(5)

11.1 Virtual Work Example 10, page 5 of 5


11 The angle in Eq. 6 can be calculated by substituting the given
values = 30, L = 400 mm, and a = 120 mm into Eq. 4:
a sin

= L/2 sin

and solving to get

Substituting

(0.8054)(L/2)

(Eq. 4 repeated)

= 56.44.

+ cos

= 30 and

tan )(L/2)

, by Eq. 7

Q xA + P yC = 0

Although it is not necessary for solving the problem, we can


now determine whether point A moves to the left or to the
right. From Eq. 6 we have
xA = ( sin

12 Substituting Eqs. 2 and 7 for yC and xA into the


virtual-work equation, Eq. 1, gives

Eq. 6 repeated)

L cos

Q(0.8054)/2 + P cos

Substituting

That is, xA is positive when


is positive, so xA increases,
that is, point A moves to the left for the particular values of
a, and L of our problem.

)=0

=0

= 30 and P = 200 N and solving gives

Q = 430 N

2232

(L

0, it follows that

Q(0.8054)/2 + P cos

xA = (0.8054)(L/2)

, by Eq. 2

so

Because L

= 56.44 into this equation gives

(Eq. 1 repeated)

Ans.

11.1 Virtual Work Example 11, page 1 of 5


11. A scissors lift is used to raise a weight W = 800 lb.
Determine the force exerted on pin F by the hydraulic
cylinder AF when = 35. Each linkage member is 2-ft
long and pin connected at its midpoint and endpoints. The
lift consists of two identical linkages and cylinders the one
shown and one directly behind it.

W
K

G
F

D
C

2233

11.1 Virtual Work Example 11, page 2 of 5


1

Consider a free-body diagram and identify the active


forces associated with a small change in
2

Each side of the lift carries half of


the load. The W/2 load does work,
so it is an active force.

W/2
Free-body diagram

FFA

B
5 The reaction force at B does no
work because it is vertical while
the motion of point B is
By
horizontal.

3 The force FFA of the hydraulic


cylinder acting on pin F does
work as pin F moves.

Ax

Ay

2234

4 The force FFA of the hydraulic


cylinder acting on pin A does no
work because pin A does not
move. For the same reason, the
reaction forces Ax and Ay from the
support do no work.

11.1 Virtual Work Example 11, page 3 of 5


W/2

yJ
J
sF
6
F
sF
FFA

yJ

Introduce coordinates yJ and sF measured from the fixed point A


to the point of application of the active forces.
Compute the work done when the coordinates are increased a
positive infinitesimal amount:
U = (W/2) yJ + FFA sF = 0

2235

(1)

11.1 Virtual Work Example 11, page 4 of 5


J

(1 ft) sin

1 ft

1 ft

(1 ft) sin

H
1 ft
F

(1 ft) sin

yJ = (6 ft) sin

yJ
1 ft

(1 ft) sin

Relate the coordinate yJ to the angle

yJ = 6 cos

(2)

(1 ft) sin

1 ft
C

(1 ft) sin

O
1 ft
(1 ft) cos

8 To relate sF to ,
consider triangle AFCO.

Use the Pythagorean Theorem and then differentiate to get sF.


sF = (3 sin )2 + (1 cos )2

sF
(3 ft) sin

sF =

A
=

+ 2(cos )( sin )
1 2(3 sin )(3 cos )
2
(3 sin )2 + (1 cos )2
8 sin

cos
2

9 sin

(1 ft) cos

2236

(3)
2

+ cos

11.1 Virtual Work Example 11, page 5 of 5


10 Substituting for sF and yJ from Eqs. 2 and 3 in the
virtual-work equation, Eq. 1, gives
6 cos

, by Eq. 2

(W/2) yJ + FFA sF = 0

(Eq. 1 repeated)

8 sin

cos
2

9 sin

, by Eq. 3
2

+ cos

Thus
3W +

8FFA sin

] cos

=0

9 sin2 + cos2
This implies, since cos

3W +

0, that

8FFA sin
2

9 sin

=0

+ cos

Substituting the given values


solving gives
FFA = 997 lb

= 35 and W = 800 lb and

Ans.

2237

11 Observation: Solving this problem by using


the equations of equilibrium would have
required drawing several free-body diagrams
and writing equations for each diagram.
Solving the problem by virtual work is much
easier because we don't have to calculate the
forces acting between the various links. In
general, problems involving connected rigid
bodies can be solved more easily by virtual
work than by equilibrium equations provided
that the relations between displacements can
be easily obtained.

11.1 Virtual Work Example 12, page 1 of 5


12. The unstretched length of the spring is 1 m. Determine
the value of for equilibrium when force P = 2 kN.

1 The system can be described by a single


coordinate, . Consider a free-body
diagram and identify the active forces
corresponding to a small change in .

3m

Free-body diagram
1.5 m

By
B

1.5 kN/m

2m

2 The spring is not part of


the free-body; the effect of
the spring is represented
by the force Fs, which is an
active force.

Fs

Bx

4m
D
D
P

3 Force P is an active force.


P

2238

11.1 Virtual Work Example 12, page 2 of 5


Procedures and Strategies for Solving Problems Involving
Potential Energy
1. Make a sketch that shows the system in an arbitrary position.
2. Express the gravitational potential energy of each weight Wi in the
system as Wiyi, where yi is the vertical distance measured from a
fixed reference point to the weight.

3. Express the elastic potential energy of each spring in the system


with spring constant ki as (1/2)kisi2, where si is the change in length
of the spring.
A
4. Add the gravitational and elastic potential energies to obtain the
total system potential energy, V.

s
L/2

4. Express V as a function of a single variable, q, by using geometry to


express all distances si and yi in terms of q.

2239

L/2

5. Obtain the equilibrium equation of the system by setting the


derivative of V with respect to q to zero. Solve this equation for
the value of q or, if q is given, for the value of one of the other
parameters of the problem that you were asked to determine, such
as a weight or spring constant.
6. Determine the stability of the equilibrium position by calculating
the second derivative of V. If the second derivative is positive,
then the equilibrium is stable; if the second derivative is negative,
the equilibrium is unstable; and if derivatives of all orders equal
zero, the equilibrium is neutral.

s = (L/2) sin
A
y = L sin
L/2

L/2

11.1 Virtual Work Example 12, page 3 of 5


6 Relate the differentials sC and yD to the angle
change

sC2 = (2 m)2 + (sB)2

3m

Law of cosines applied to triangle ABC:


2(2 m)(sB) cos ( + )

(3)

Here
1.5 m

sB

sB = (3 m)2 + (1.5 m)2 = 3.354 m

sC

and

(5)

To avoid having to write equations containing several four and


five-digit numbers, introduce intermediate variables a and b:

2m
C
4m

= tan-1 1.5 m = 26.565


3m

(4)

sC2 = (22 + sB2)

yD

2(2)(sB) cos ( + ) (Eq. 3 repeated)

Thus
sC2 = a
D

b cos ( + )

(6)

where
3.354 m by Eq. 4
7

a = 22 + (sB)2

yD = (2 m + 4 m) sin
yD = 6 cos

= 4 + 3.3542

(2)

= 15.249 m

2240

(7)

11.1 Virtual Work Example 12, page 4 of 5


9 The parameter b can also be evaluated, for later
use:

10 The spring force Fs can be expressed in terms of sC:


Fs = k

(extension of the spring)

3.354 m, by Eq. 4
b = 2(2)(sB)

=k

(stretched length

= 4(3.354)

=k

(sC

= 13.416 m
sC can be related to
sC2

=a

1 m)

(11)

Substituting for yD, sC, and Fs from Eqs 2, 9, and 11 into


the virtual-work equation, Eq. 1, gives

(8)
by differentiating Eq. 6:

b cos ( + )

k(sC

1), by Eq. 11

(Eq. 6 repeated)
P yD

2sC sC = b sin ( + )

6 cos

Fs sC = 0

(Eq. 1 repeated)
b sin ( + )
2sC

, by Eq. 2

so
b sin ( + )
sC =
2sC

b cos ( + )

, by Eq. 9

or
(9)
[(6P) cos

Taking the square root of both sides of Eq. 6


gives an equation for sC.
sC = a

unstretched length)

Since
zero.

(10)

1)

b sin ( + )
2sC

=0

0, the expression in brackets must equal to

(6P) cos

2241

k(sC

k(sC

1)

b sin ( + )
=0
2sC

(12)

11.1 Virtual Work Example 12, page 5 of 5


11 Eq. 12 contains the distance sC, which can be calculated by using Eq. 10:
(6P) cos

k(sC
a

1)

b sin ( + )
=0
2sC

(Eq. 12 repeated)

b cos ( + ) , by Eq. 10

or
(6P) cos

k( a

b cos ( + )

1)

b sin ( + )
2 a

=0

b cos ( + )

Substituting in the latter equation the values


P = 2 kN

(Given)

k = 1.5 kN/m

(Given)

a = 15.249 m

(Eq. 7 repeated)

b = 13.416 m

(Eq. 8 repeated)

= 26.565

(Eq. 5 repeated)

and solving numerically gives


= 53.4

Ans.

2242

11.1 Virtual Work Example 13, page 1 of 6


13. a) Determine the moment reaction at the wall F.
b) Determine the force reaction at the roller D.
In both cases P = 60 lb.
Hinges

D
A

5 ft

5 ft

5 ft

5 ft

10 ft

1
D

Part a. Replace the wall at F by a moment


couple MF and a pin support.
F

2243

MF

11.1 Virtual Work Example 13, page 2 of 6


2

Consider a free-body diagram and identify the active forces


associated with a small rotation of the segments of the beam
Free-body diagram
P

Dy
F

MF

Fx
C
By

Introduce coordinates yA and

D
The active "forces" are the force
P and the couple moment MF.

Fy

U = 0: P yA MF

for calculating the work.

=0

(1)

Dy

F
A

MF

Fx
yA

D
yA
Fy

By

2244

11.1 Virtual Work Example 13, page 3 of 6


5

Relate the differentials yA and


yC
A

5 ft

5 ft
B

5 ft

5 ft

10 ft

D
yE

yA
6

By similar triangles,
yE = yC and yC = yA

7 For small angles, yE is given by


yE = (10 ft)
But this becomes, after using the relation yE = yA,

so
yE = yA

yA = 10

(2)

Substitute this result in the virtual-work equation to get


P yA

MF

=0

(Eq. 1 repeated)

10
Dividing through by , substituting the known value P = 60 lb,
and then solving for MF gives,
MF = 600 lb ft

2245

Ans.

11.1 Virtual Work Example 13, page 4 of 6


8

Part b. Replace the roller at D by a vertical force, Dy.


Dy

60 lb
A

B
C

9 Draw a free-body diagram and show a


small rotation of segments AC and CE.
60 lb
A

10 Segment EF of the beam does


not move because the wall
support prevents both rotation
and vertical displacement.
Thus MF and Fy do no work.

Dy
B
C
By

11 The active forces are


the 60-lb force and Dy.

12 Introduce coordinates yA and yD for calculating the work.


60 lb
A
yA y
A

Fy

U = 0: (60 lb) yA Dy yD = 0

Dy
yD

yD

B
C

MF

2246

(3)

11.1 Virtual Work Example 13, page 5 of 6


13 Relate the differentials yA and yD.
yC
A

yD

5 ft
B

5 ft

5 ft

5 ft

yA
14 By similar triangles,
yA = yC
and
yC
yD
=
5
5+5
Eliminating yC gives
yD =

yA
2

Use this equation to replace yD in the virtual-work equation:


(60) yA Dy yD = 0

(Eq. 3 repeated)

yA
2
Dividing through by yA and solving gives
Dy = 120 lb

Ans.

2247

11.1 Virtual Work Example 13, page 6 of 6


15 Comment: Let's extend the discussion. If we were asked to
calculate the vertical reaction force at the wall, we would
replace the wall by a support that prevents rotation but permits
vertical displacement.
Fy
60 lb
D
A

16 Corresponding displacements
Fy
60 lb
D
A

17 Segment EF translates but does not rotate. Thus the reaction


moment at the support does no work. The reaction force, Fy,
however, does work and is thus an active force.

2248

18 Observation: The method applied in this


beam example can be generalized.
Virtual work can be used to calculate a
force of constraint (a reaction) by
considering displacements which violate
the constraints and then accounting for
the work done by the force of constraint.
This procedure is equivalent to
converting a rigid structure into a
mechanism, as was done at the
beginning of the present example.

11.1 Virtual Work Example 14, page 1 of 5


14. Determine the vertical reaction at
support C, if P = 2 kN.

C
P

Convert the structure


into a mechanism with
one degree of freedom
by replacing the pin
support at C by a roller
support and a vertical
force, Cy.

Cy

3m
P

B
B
4m

A
3m
A

2249

11.1 Virtual Work Example 14, page 2 of 5


Cy

Cy

Cx

Cx

yC

P
2

Cy and P are active


forces for the
displacements
shown.

yB
yC

B
Ay
yB

Ay

Ax

Ax

A
A
3

Define coordinates yB and yC locating the point of


application of the active forces, and compute the work.
U = 0:

2250

P yB + Cy yC = 0

(1)

11.1 Virtual Work Example 14, page 3 of 5


4 Relate the differentials yB and yC through the angles and
Begin by computing the lengths of bars BC and BA (Note that
these lengths do not change, as the angles and change).

C
3 2 m

C
5

BC =

(3 m)2 + (3 m)2

B
3 2 m

yC

3m
B

AB =

(3 m)2 + (4 m)2

yB

3m

5m

5m

4m
7
A

From the above figure,


yB = (5 m) cos
yB = 5 sin
yC = (5 m) cos
yC = 5 sin

(2)
+ (3 2 m) cos
(3 2 ) sin

Use the law of sines to relate


sin
sin
=
5m
3 2 m

2251

(3)

and
(4)

11.1 Virtual Work Example 14, page 4 of 5


8

Differentiating Eq. 4 gives


cos
5

9 Thus

cos
3 2

P sin
Dividing by 5

Thus
=

5 cos

P sin

(5)

3 2 cos

The equation relating


and , Eq. 5, can be used in Eq. 3
to express yC in terms of
alone:
yC = 5 sin

3 2 sin

(Eq. 3 repeated)

5 cos
3 2 cos
= ( 5 sin

5 tan

cos )

(6)

Substituting Eqs. 2 and 6 for yB and yC into the virtual


work equation gives
5 sin

, by Eq. 2

P yB + Cy yC = 0
( 5 sin

(Eq. 1 repeated)
5 tan

cos )

, by Eq. 6

2252

Cy (sin

+ tan

cos ) (5

)=0

gives
Cy (sin

+ tan

cos ) = 0

(7)

11.1 Virtual Work Example 14, page 5 of 5


C
3m

3m

5m

4m

tan

3
=1
3

sin

3
5

cos

4
5

10 Evaluating the functions of and in Eq. 7, substituting the given


value P = 2 kN, and then solving gives
3
5
P sin

Cy (sin

3
5
+ tan

4
5

cos ) = 0

(Eq. 7 repeated)

Cy = 0.857 kN

11 Observation: This example demonstrates that virtual


work can be used to calculate the reaction forces
from the supports acting on a structure. The example
also demonstrates that just because virtual work can
be used doesn't necessarily mean that it should be
used the reaction at support C could have been
found much more easily by employing equilibrium
equations. The usefulness of virtual work depends
on how easy it is to express relations between
coordinates.

2253

Ans.

11.1 Virtual Work Example 15, page 1 of 3


15. Determine the vertical reaction at
support I of the truss, if P = 10 kip = Q.

P
A

Q
B
5 ft

C
5 ft

D
5 ft

5 ft

F
5 ft

G
5 ft

H
5 ft

5 ft

Convert the structure to a mechanism with one degree of


freedom by replacing the pin support at I by a vertical force Iy
and a roller.

P
A

Iy
F

2254

11.1 Virtual Work Example 15, page 2 of 3


2

Identify the active forces corresponding to a set of


displacements compatible with the constraints.
P
A

By

Iy

E
Dy

Fy

H
Hy

Iy

A
yA

yA

By

yE
Dy

yE

Fy

Introduce coordinates measured from fixed points to the


points of application of the applied forces.
Calculate the work done.
U = 0: P yA + Q yE

Iy yI = 0

Ix

(1)

2255

yI
Hy

I
yI

Ix

11.1 Virtual Work Example 15, page 3 of 3


4 Relate yA, yE, and yI
by geometry (similar
triangles).

yC

A 5 ft

yA = yC

yA

5 ft

5 ft

C 5 ft

(2)

Similarly,
yC = yE, yE = yG, and yG = yI
These equations imply
yA = yI and yE = yI
Substituting the latter pair of equations into the
virtual-work equation, Eq. 1, gives
P yA + Q yE I yI = 0
yI
yI

(Eq. 1 repeated)

or
(P + Q

Iy) yI = 0

Dividing through by yI, substituting the given values P


= 10 kip = Q, and solving gives
Iy = 20 kip

Ans.

2256

E
yE

yG

5 ft
F

5 ft G

5 ft
5 ft

I
yI

11.1 Virtual Work Example 16, page 1 of 4


16. Determine the tension in the cord. The pulleys are
frictionless and m = 90 kg.

C
A
1

Convert the pulley-cord


system into a mechanism
with one degree of
freedom by replacing the
support B by a tensile
force T acting on the end
of the cord.

D
m
D
Weight = mg

2257

11.1 Virtual Work Example 16, page 2 of 4


2

The tension T and the weight do work (are active


forces) if end B of the cord moves up a small
amount.

Introduce the coordinates yB and yD.


T
A

yB
yB

4
yD

yD

Calculate the work done:


U = 0:

mg

2258

T yB + mg yD = 0

(1)

11.1 Virtual Work Example 16, page 3 of 4


5 Relate yB to yD by first expressing the length, say L, of the cord
in terms of yB and yD:

Diameter dC
A

yB

L = (yD

t)

yB

+ (yD

t)

+ (yD

t)

+ dD/2

yD

Half circumference of pulleys


+ dC/2

u
Thus
Diameter dD

t
D

L = 3yD

3t

yB

2s + dD/2 + dC/2

Now differentiate, taking into account that because L, t, u, s, dD,


and dC do not vary as yD and yB vary, we have L = 0 = t = u = s
= dD = dC; the result of the differentiation is, then,
0 = 3 yD

yB

Thus
yB = 3 yD

2259

(2)

11.1 Virtual Work Example 16, page 4 of 4


6

Substituting this result in the virtual work equation, Eq. 1, gives


T yB + mg yD = 0

(Eq. 1 repeated)

3 yD, by Eq. 2
Thus
( 3T + mg) yD = 0
Dividing through by yD, substituting m = 90 kg, g = 9.81 m/s2
and solving gives
T = 294 N

Ans.

2260

11.1 Virtual Work Example 17, page 1 of 9


17. Determine the equilibrium values of and for the
two-bar linkage. The couple moment M = 5 N m; each bar is
uniform and has a mass m of 5 kg; the length L = 400 mm; and
the unstretched length of the spring is 250 mm.
500 mm
A

L
2

k = 0.2 kN/m
B
D
L

M
C

2261

11.1 Virtual Work Example 17, page 2 of 9


1 The system has two degrees of freedom because two
coordinates and must be specified to define the position
of the linkage. Consider a free-body diagram, and identify the
active forces corresponding to a small change in
while
is held fixed .
Ay
A
Ax
2
2

Fs
(spring force)

Because point A does not move and is


fixed, the reactions Ax and Ay, the weight
mg, and the spring force Fs do no work
when 1 is varied a small amount. Thus
they are not active forces.

mg
B

3
M
C

mg

The couple moment M and the weight


mg of the lower bar BC do work when 1
is varied, so M and mg are active forces.

2262

11.1 Virtual Work Example 17, page 3 of 9


4

In addition to the coordinate 1, introduce a vertical


coordinate yE measured downward from point A.
Ay
A
A
Ax
L/2
L

yE

yE

L/2
B

mg

Fs

L/2

B
yE

E
L/2

E
L/2

Relate the differential yE to the angle change,


writing

1,

by

mg
yE = L cos

Compute the work done when the coordinates


are increased a positive infinitesimal amount.
dU = 0: M

1+

mg yE = 0

+ (L/2) cos

and then differentiating with respect to 1, while holding


fixed. That is, take the partial derivative with respect to
to obtain

(1)
yE = (L/2) sin

2263

(2)

11.1 Virtual Work Example 17, page 4 of 9


7

Substitute Eq. 2 for yE into the virtual-work equation:


M

1+

mg yE = 0

(Eq. 1 repeated)

(L/2) sin

1,

by Eq. 2

Thus

Since

[M

(mgL/2) sin 1]

0, it follows that

(mgL/2) sin

1=

1=

(4)

Substituting the following values into Eq. 4


M = 5 N m = 5 000 N mm
L = 400 mm
m = 5 kg
g = 9.81 kg m/s2
and solving gives
1=

30.6

Ans.

2264

11.1 Virtual Work Example 17, page 5 of 9


8

Next identify the active forces corresponding to


a small change in 2 while 1 is held fixed .

Ay
A

9 Because point A does not


move, Ax and Ay do no work
and thus are not active forces.
Ax

10 Because 1 is fixed, link BC


does not rotate. That is, the
dashed line representing the new
position of link BC is parallel
to BC. Hence couple moment
M does no work, and M is not
an active force.
M
C

mg

Fs
B

11 Active forces

mg

2265

11.1 Virtual Work Example 17, page 6 of 9


12 Introduce coordinates measured from a fixed point to
the point of application of the active forces.
xB
xB
13 Compute the work done.

Ay

U = 0: mg yE + mg yF + Fs xB = 0
A

Ax
yF

L/2

yF
yE

mg

Fs
B

L/2
1

E
C

yE

mg

2266

(5)

11.1 Virtual Work Example 17, page 7 of 9


14 Relate the differentials yE, yF, and yF to the change in angle
xB

xB = L sin

yF = (L/2) cos
L/2
F
L/2

yF

yE = L cos

(L/2) cos

Differentiating each equation with respect to


gives
2

yE

with

held fixed,

xB = L cos

(6)

yF = (L/2) sin

(7)

yE = L sin

(8)

L/2
1

L/2

2267

11.1 Virtual Work Example 17, page 8 of 9


15 Substituting Eqs. 6-8 for the differentials into the virtual-work
equation, Eq. 5, gives
L sin

500 mm

by Eq. 8
A

mg yF + mg yE + Fs xB = 0
L/2) sin

by Eq. 7

(Eq. 5 repeated)

L cos

by Eq. 6
500 mm

Thus

L sin

L
2
2

[ 3mg/2) sin
Since L

+ Fs cos

]L

=0
B

0, it follows that
D
3mg/2) sin

+ Fs cos

=0

(9)

C
16 The force Fs in the spring can be related to
Fs = k

extension of spring

=k

(current length

=k

[(500 mm

= k(250

2268

2:

L sin

unstretched length)

L sin
2)

2)

250 mm]
(10)

11.1 Virtual Work Example 17, page 9 of 9


17 Substituting for Fs from Eq. 10 into the virtual-work equation, Eq. 9, gives
3mg/2) sin

+ Fs cos
k(250

=0
L sin

(Eq. 9 repeated)
2),

by Eq. 10

or,
(3mg/2) sin

+ k(250

L sin

) cos

=0

(11)

Substituting the following values into Eq. 11


L = 400 mm
m = 5 kg
k = 0.2 kN/m = 0.2 N/mm
g = 9.81 kg m/s2
and solving numerically gives
= 18.50

Ans.

2269

11.2 Potential Energy

2270

11.2 Potential Energy Procedures and Strategies, page 1 of 2


Procedures and Strategies for Solving Problems Involving
Potential Energy
1. Make a sketch that shows the system in an arbitrary position.
2. Express the gravitational potential energy of each weight Wi in the
system as Wiyi, where yi is the vertical distance measured from a
fixed reference point to the weight.

3. Express the elastic potential energy of each spring in the system


with spring constant ki as (1/2)kisi2, where si is the change in length
of the spring.
A
4. Add the gravitational and elastic potential energies to obtain the
total system potential energy, V.

s
L/2

4. Express V as a function of a single variable, q, by using geometry to


express all distances si and yi in terms of q.

2271

L/2

5. Obtain the equilibrium equation of the system by setting the


derivative of V with respect to q to zero. Solve this equation for
the value of q or, if q is given, for the value of one of the other
parameters of the problem that you were asked to determine, such
as a weight or spring constant.
6. Determine the stability of the equilibrium position by calculating
the second derivative of V. If the second derivative is positive,
then the equilibrium is stable; if the second derivative is negative,
the equilibrium is unstable; and if derivatives of all orders equal
zero, the equilibrium is neutral.

s = (L/2) sin
A
y = L sin
L/2

L/2

11.2 Potential Energy Procedures and Strategies, page 2 of 2


Notes:
a) The method of potential energy cannot be applied to solve
problems involving friction because friction forces are
non-conservative, that is, the work done by a friction force
depends on the path that the force follows when the system
configuration is changed.
b) The above procedure applies to one degree-of-freedom
systems, that is, to systems the position of which can be
described by a single coordinate, q. For systems with more
than a single degree of freedom, multiple coordinates qi must
be defined. Equilibrium equations can then be derived by
setting to zero the first derivative of V with respect to each qi.
Determining the stability of systems with multiple degrees of
freedom is more complicated than in the single
degree-of-freedom case. For example, an equilibrium
configuration for a system with two degrees of freedom, q1
and q2, will be stable provided that
2

V
q q

V
q2

V <0
q2

and
2

V > 0 or
q2

V >0
q2

2272

q1 =

q2 =

11.2 Potential Energy Problem Statement for Example 1


1. The rigid rod ABC may rotate
freely about the support at B.
Weights of magnitude W and 2W
are attached at ends A and C.
Determine the possible equilibrium
positions of the rod and investigate
their stability.
W
A

L
B

L
C
2W

2273

11.2 Potential Energy Problem Statement for Example 2


2. Determine if the composite solid is
in a stable equilibrium position. The
solid is of uniform density .

Vertex
Parabola

L
O
R

Semicircle

2274

11.2 Potential Energy Problem Statement for Example 3


3. Determine the mass m of the disk required for
equilibrium when = 50, and investigate the stability of
this equilibrium. The spring is unstretched when = 0,
and the masses of the rod AB and the roller C are
negligible.
L = 1.5 m
k = 200 N/m
m
C

L
A

2275

11.2 Potential Energy Problem Statement for Example 4


4. The disk of mass m is supported by the two rods AC and
BC. Determine the angle for equilibrium and determine if
the equilibrium is stable. The weights of the rods are
negligible, and the unstretched length of the spring is L.
L = 0.7 m
k = 200 N/m
m = 10 kg
m
L
A

C
k

L
B

2276

11.2 Potential Energy Problem Statement for Example 5


5. Determine the spring constant k required for
equilibrium when = 75, and investigate the
stability of this equilibrium. The rod is uniform and
has mass m. The spring wraps around the cylinder
and is unstretched when = 0.
m = 10 kg
L = 0.5 m
R = 0.3 m

Mass, m

2277

11.2 Potential Energy Problem Statement for Example 6


6. Determine the value of for equilibrium, and investigate the
stability of this equilibrium. The block of mass m is connected
by a cord to a small collar at B, which slides freely on the
circular ring. The unstretched length of the spring is R.
R = 0.8 m
k = 60 N/m
m = 5 kg
B
k

O
A

C
m

2278

11.2 Potential Energy Problem Statement for Example 7


7. The uniform bar of mass m is connected to two small
rollers at ends B and C. Spring AB, which is in tension,
has an unstretched length of L/4, and spring AC, which is
in compression, has an uncompressed length of L/4.
Determine the value of for equilibrium and investigate
the stability of the equilibrium.
L = 0.4 m
k = 800 N/m
m = 40 kg
A

Mass m

2279

11.2 Potential Energy Problem Statement for Example 8


8. Light rods AB and BC support a block of weight W at
C. End D of the spring is attached to a light collar that
moves freely on the vertical rod. Assume that the spring is
always horizontal. The spring is unstretched when = 0.
Determine the angle for equilibrium and also determine
if the equilibrium is stable.
L = 9 in.
k = 100 lb/in.
W = 200 lb

C
L

D
k

B
L

2280

11.2 Potential Energy Problem Statement for Example 9


9. Determine the spring constant k required to keep the
uniform rod AB in equilibrium at an angle of = 20. Also
determine the stability of the equilibrium. The spring is
unstretched when = 0. Neglect the size of the pulley at C.
L = 30 in.
W = 200 lb

k
C
L
Weight, W
A

2281

11.2 Potential Energy Problem Statement for Example 10


10. The waste bin has a center of gravity at C and weight W.
Determine the steepest slope upon which the bin can be placed
without tipping. Also determine the stability of this
equilibrium position.
a = 25 in.
b = 10 in.
W = 60 lb

THA
YO NK
U

C
a

A
B

b
b

2282

11.2 Potential Energy Problem Statement for Example 11


11. The outer rim of stepped pulley A is connected by a
belt to pulley B. A spring is wrapped around the inner
rim and the cord supporting mass m is attached at point
C on the large pulley. Determine the equilibrium
position of the system and investigate its stability. The
spring is unstretched when the pulleys are in the position
shown.
R = 0.2 m
m = 2 kg
k = 900 N/m
k

A
2R

D
m

2283

4R

11.2 Potential Energy Problem Statement for Example 12


12. Determine if rods AB and BC are in stable equilibrium in the
position shown. The rods are uniform and have mass m. Rod BD is
pinned to a light slider at C that can move freely up and down between
the vertical walls. The spring is unstretched when rod AB is vertical.
One end of the spring is attached to a light collar at E that moves freely
on the rod. Assume that the spring remains horizontal for all positions
of the rods.
D
L/2
C
L/2

k
B

L
A

2284

11.2 Potential Energy Problem Statement for Example 13


13. Determine the range of values of the force P for which the
vertical equilibrium position = 0 = 2 is stable. The springs
are connected to collars D and E, which can slide freely on the
vertical rod. The springs are unstretched when = 0 = 2.
Assume that the springs remain horizontal for all positions of
the rods and the weights of the rods and the frictionless collars
are negligible.
P
k

D
L
k

E
1

L
A

2285

11.2 Potential Energy Example 1, page 1 of 3


1. The rigid rod ABC may rotate
freely about the support at B.
Weights of magnitude W and 2W
are attached at ends A and C.
Determine the possible equilibrium
positions of the rod and investigate
their stability.

Let point B be the datum for gravitational


potential energy, V. Thus
V = WL cos

2WL cos
L cos

= WL cos

(1)

The equilibrium position can be found from


the equation

L
dV = 0
d

L cos

L
2W

Substituting for V from Eq. 1 and


differentiating gives
L
C
2W

dV = WL sin
d

=0

(2)

The equilibrium positions of the rod are the


roots of Eq. 2:
1=

0 and

2286

= 180

Ans.

11.2 Potential Energy Example 1, page 2 of 3


2 To investigate stability, evaluate d2V/d
expression for dV/d in Eq. 2, we have
d2V d
=
d
d

for each root. Using the

dV
d

= d (WL sin
d
WL cos
Substituting

1=

(3)
0 in Eq. 3 gives

d2V
= WL cos 0 = WL > 0
d
Since d2V/d

Substituting

is positive, the equilibrium position


=

1=

0 is stable.

= 180 in Eq. 3 gives

d2V
= WL cos 180 = WL < 0
d
Since d2V/d
unstable.

is negative, the equilibrium position

= 180 is

2287

11.2 Potential Energy Example 1, page 3 of 3


3

A plot of of the potential energy shows the


stable and unstable equilibrium points.
4

wL cos

dV
= 0 (equilibrium)
d

wL
d2V
< 0 (unstable)
d

Comment: The equations and the plot of V


confirm what you would expect; the system is
stable when the light weight is on top and
unstable when the heavy weight is on top.

Stable

Unstable
W

-wL

90

2W

180

d2V
> 0 (stable)
d
dV
= 0 (equilibrium)
d

180

2W

2288

11.2 Potential Energy Example 2, page 1 of 6


2. Determine if the composite solid is
in a stable equilibrium position. The
solid is of uniform density .

Vertex
Parabola

L
O

Center of gravity of
parabolic cylinder

Semicircle

Let the point O be the datum


for gravitational potential
energy, and rotate the body
by a small angle from the
(vertical) equilibrium
position.

O
Center of gravity
of semicircular
cylinder

2289

11.2 Potential Energy Example 2, page 2 of 6


2

A table of properties of planar regions gives the


information shown below.
C is the centroid of
the region.

5 L

3L
5
3L
5

2L
5

Parabolic region
Area

4ab
3

C
3a
5

4R
3

y
Semicircular region
a2
Area = 2

C
a

4a
3
x

6 The potential energy can be expressed in terms of :


V = mpg(
Mass of parabolic
part of solid

2L
cos )
5

msg(

4R
cos )
3

Mass of
semicircular
part of solid
Negative because mass
center is below datum
(point O)

2290

(1)

11.2 Potential Energy Example 2, page 3 of 6


7

Mass of parabolic part of solid:


mp = ( 4LR )d
3

(2)

C is the centroid of
the region.

d
b

Parabolic region
Area

4ab
3

3a
5

x
R

y
9 The potential energy can now be written as
Semicircular region
a2
Area = 2

V = mpg( 2L cos )
5
msg( 4R cos )
3

4a
3
x

C
a

( 4LR )d, by Eq. 2


3
8 Mass of semicircular part of solid:
2
ms = ( R )d
2

(Eq. 1 repeated)

2
( R )d, by Eq. 3
2

Thus V becomes, after collecting terms and


simplifying,
(3)
V=

2291

2 gRd
(4L2
15

5R2) cos

(4)

11.2 Potential Energy Example 2, page 4 of 6


10 The equilibrium position can be found from
dV
=0
d
or,
dV d 2 gRd
=
[
(4L2
d
d
15

2 gRd
2
15 (4L

5R2) cos ] = 0

5R2)( sin

=0

(5)

The root of this equation is = 0. To investigate stability, evaluate


d2V/d 2 for = 0. Using the expression for dV/d in Eq. 5 gives
d2V d
=
d
d
If

2 gRd
dV
= 15 (4L2
d

5R2)( cos

(6)

= 0, Eq. 6 becomes
d2V 2 gRd
=
(4L2
15
d

5R2)( 1

(7)

2292

11.2 Potential Energy Example 2, page 5 of 6


11 Eq. 7 shows that d2V/d 2 will be positive (and thus the = 0
equilibrium position will be stable) if 4L2 5R2 < 0, which implies
L < 5R/2
What if 4L2

V=

5R2 = 0? Then

2 gRd
(4L2
15

5R2)( cos
0

=0
for all values of . Furthermore, all derivatives of V equal zero.
Thus if 4L2 5R2 = 0, the composite solid is in neutral equilibrium
for all values of .

2293

11.2 Potential Energy Example 2, page 6 of 6


12 These results can be summarized as follows:

1. If L < 5R/2, then the only


equilibrium position in the range
90 < < 90 is = 0 and this
position is stable.
Ans.

L
R

13 2. If L > 5R/2, then the only


equilibrium position in the range
90 < < 90 is = 0 and this
Ans.
position is unstable.

14 3. If L = 5R/2, then equilibrium is


possible for all values of in the
range 90 < < 90 and all of
these equilbrium positions are
neutral.
Ans.

L
R

2294

11.2 Potential Energy Example 3, page 1 of 3


3. Determine the mass m of the disk required for
equilibrium when = 50, and investigate the stability of
this equilibrium. The spring is unstretched when = 0,
and the masses of the rod AB and the roller C are
negligible.
L = 1.5 m
k = 200 N/m

Let point A be the datum for the gravitational


potential energy, Vg. Thus

Vg = mg(L cos )

(1)

m
B

L
L cos

L
A

2295

11.2 Potential Energy Example 3, page 2 of 3


2 The change in length of the spring is L sin , so the elastic potential energy is
Ve = (1/2)k(L sin )2

(2)

The potential energy of the system is, then,


L sin
mgL cos

by Eq. 1

m
B

V = Ve + Vg
(1/2)k(L sin )2 by Eq. 2
The equilibrium position can be found from

L
dV
=0
d

or,
dV d
=
[(1/2)k(L sin )2 + mgL cos ] = 0
d
d
or,
dV
= kL2 sin
d
Substituting

cos

mgL sin

=0

(3)

= 50, g = 9.81 m/s2, k = 200 N/m, and L = 1.5 m and solving for m gives

m = 19.66 kg

Ans.

2296

11.2 Potential Energy Example 3, page 3 of 3


3

To investigate stability of the equilibrium, evaluate d2V/d


= 50. Using the expression for dV/d in Eq. 3 gives
d2V d
=
d
d
=

for

dV
d

d
[kL2 sin
d

= kL2(cos2

cos
sin2 )

Substituting m = 19.66 kg,


and L = 1.5 m gives

mgL sin

mgL cos

= 50, g = 9.81 m/s2, k = 200 N/m,

d2V
= 264 N m
d
Since d2V/d 2 is negative, the equilibrium position is unstable:
the bar could not remain inclined at an angle of = 50 in a real
(that is, not mathematically idealized) system.

2297

11.2 Potential Energy Example 4, page 1 of 3


4. The disk of mass m is supported by the two rods AC and
BC. Determine the angle for equilibrium and determine if
the equilibrium is stable. The weights of the rods are
negligible, and the unstretched length of the spring is L.
L = 0.7 m
k = 200 N/m
m = 10 kg
m
L

Angle equals
because triangle
ABC is isosceles.

C
k

B
Mass, m

Let point A be the datum for the gravitational potential


energy, Vg. Thus
Vg = mgL sin

L sin

C
k

(1)
A

The elastic potential energy is


2L cos
Ve = (1/2)k(current length
= (1/2)k(2L cos

L)2

= (1/2)kL2(2 cos

1)2

unstretched length)

(2)

2298

L
B

11.2 Potential Energy Example 4, page 2 of 3


2 The potential energy of the system is, then,
mgL sin

by Eq. 1

V = Ve + Vg

(3)

(1/2)kL2(2 cos

)2, by Eq. 2

The equilibrium position can be found from


dV
=0
d
or,
dV d
=
[(1/2)kL2(2 cos
d
d

)2 + mgL sin ] = 0

dV
= kL2(2 cos
d

)+ mgL cos

or,

)( 2 sin

=0

(4)

Substituting m = 10 kg, g = 9.81 m/s2, k = 200 N/m and L = 0.7 m, and then solving
Eq. 4 numerically gives two roots:
= 22.43 and

= 49.48

Ans.

2299

11.2 Potential Energy Example 4, page 3 of 3


3 To investigate stability, evaluate d2V/d 2. Using the
expression for dV/d in Eq. 4 gives
d2V d
= d
d

dV
d

= d [kL2(2 cos
d

A plot of of the potential energy shows the stable and


unstable equilibrium points. From Eq. 3, the potential
energy is
V( ) = (1/2)kL2(2 cos

)( 2 sin

)+ mgL cos ]

= kL2[( 2 sin )( 2 sin ) + (2 cos


( 2 cos )] mgL sin

)2 + mgL sin

V( )
120 m N

Substituting = 22.43, m = 10 kg, g = 9.81 m/s2, k = 200


N/m and L = 0.7 m into the above equation gives

dV
= 0 (equilibrium)
d

80 m N

d2V
= 123 N m
d
Because d2V/d
Substituting

40 m N

is negative, the equilibrium is unstable.

d2V
= 136 N m
d
2

22.43
d2V
< 0 (unstable)
d

= 49.48into Eq. 5 gives

Because d2V/d

is positive, the equilibrium is stable.

2300

49.48

90
d2V
> 0 (stable)
d

Observation: If you have difficulty finding


multiple roots of the equilibrium equation
with your calculator, plotting V( ) will give
you the approximate location of the roots,
which you can then use as initial estimates
in your calculator's solver.

11.2 Potential Energy Example 5, page 1 of 3


5. Determine the spring constant k required for
equilibrium when = 75, and investigate the
stability of this equilibrium. The rod is uniform and
has mass m. The spring wraps around the cylinder
and is unstretched when = 0.
m = 10 kg
L = 0.5 m
R = 0.3 m

Mass, m

Center of mass

L
2

Let point A be the datum for the gravitational


potential energy Ve. Thus
Ve = mg(L/2) cos

2301

L cos
2

(1)

11.2 Potential Energy Example 5, page 2 of 3


2

The change in length of the spring equals the


arc length, R
expressed in radians), so
the elastic potential energy Ve is

The potential energy of the system is


mg(L/2) cos

by Eq. 1

V = Ve + Vg
Ve = (1/2)k(R )2

(2)

(1/2)k(R )2 by Eq. 2

Arc length = R
The equilibrium position can be found from
k

dV
=0
d
R

or,
dV d
= d [(1/2)k(R )2 + mg(L/2) cos ] = 0
d
or,
dV
= kR2
d

mg(L/2) sin

=0

(3)

Substituting = (75/180) rad, R = 0.3 m, g = 9.81 m/s2,


m = 10 kg, and L = 0.5 m, and then solving gives
k = 201.1 N/m

2302

Ans.

11.2 Potential Energy Example 5, page 3 of 3


4

To investigate stability, evaluate dV2/d 2. Using the


expression for dV/d in Eq. 3 gives
d2V d
=
d
d
=

dV
d

d
[kR2
d

= kR2

mg(L/2) sin ]

mg(L/2) cos

Substituting k = 201.1 N/m, = (75/180) rad, m = 10 kg, g


= 9.81 m/s2, and L = 0.5 m, R = 0.3 m gives
d2V
= 11.8 N m
d
Because d2V/d
is stable.

is positive, the equilibrium position

= 75

2303

11.2 Potential Energy Example 6, page 1 of 5


6. Determine the value of for equilibrium, and investigate the
stability of this equilibrium. The block of mass m is connected
by a cord to a small collar at B, which slides freely on the
circular ring. The unstretched length of the spring is R.
R = 0.8 m
k = 60 N/m
m = 5 kg
B
1

O
A

Vg = mg(L

Let point O be the datum for the


gravitational potential energy Ve.
Thus
y)

(1)
O

C
m

where y is the vertical distance


from O to the collar at B, and L is
the length of the cord BC plus half
the height of the block. The value
of L is unknown, but this won't
matter because we will
differentiate Vg later, and the
derivative of L will be zero.
Another way of saying the same
thing is to say that the magnitude
of Vg doesn't matter; only the
change in Vg matters.

2304

A
L
C
m

11.2 Potential Energy Example 6, page 2 of 5


2

The coordinates and y are not independent


but can be related through geometry. First
note that the angles at A and B are equal
because triangle AOB is isosceles.
B
3
y

The vertical distance


between O and B is
y = R sin

O
4

The sum of the angles of triangle AOB must


equal 180, that is, + 2 = 180, so =
180 2 and thus
y = R sin (180

= R sin [180

(180

= R sin 2

The angle can expressed as


180
so
y = R sin (180

6 The gravitational potential energy becomes, from Eq. 1,


Vg = mg(L

2 )]

y)

(Eq. 1 repeated)

R sin 2 by Eq. 2

(2)
or,
Vg = mgR sin 2

2305

mgL

(3)

11.2 Potential Energy Example 6, page 3 of 5


7

The elastic potential energy of the spring is


Ve = (1/2)k(current length
= (1/2)k(s

unstretched length)

R)2

(4)

where s is the length of the spring (distance AB) and R is


known to be equal to the unstreteched length.
B

From geometry,
s = 2R cos

Thus Eq. 4 becomes


Ve = (1/2)k(s

A
2

R)

2R cos
or,
Ve = (1/2)kR2(2 cos

1)2

(5)

The potential energy of the system is, then,


mgR sin 2

mgL, by Eq. 3

V = Ve + Vg
(1/2)kR2(2 cos

1)2, by Eq. 5

2306

11.2 Potential Energy Example 6, page 4 of 5


8

The equilibrium position can be found from


dV
=0
d
or,
dV
d
=
[(1/2)kR2(2 cos
d
d

1)2 + mgR sin 2

mgL] = 0

or,
dV
= kR2(2 cos
d

1)( 2 sin ) + 2mgR cos 2 = 0

(6)

Substituting k = 60 N/m, R = 0.8 m, g = 9.81 m/s2, and m = 5 kg and solving numerically gives
= 34.47

Ans.

To investigate stability, evaluate dV2/d 2. Using the expression for dV/d in Eq. 6 gives
d2V d
=
d
d
=

dV
d

d
[kR2(2 cos
d

1)( 2 sin ) + 2mgR cos 2

= kR2[( 2 sin )( 2 sin ) + (2 cos

1)( 2 cos )]

2307

4mgR sin 2

(7)

11.2 Potential Energy Example 6, page 5 of 5


9

Substituting k = 60 N/m, R = 0.8 m, =


34.47, m = 5 kg, and g = 9.81 m/s2 in Eq. 7
gives
d2V
= 138 N m
d
Because d2V/d 2 is negative, the equilibrium
position = 34.47 is unstable.

2308

11.2 Potential Energy Example 7, page 1 of 3


7. The uniform bar of mass m is connected to two small
rollers at ends B and C. Spring AB, which is in tension,
has an unstretched length of L/4, and spring AC, which is
in compression, has an uncompressed length of L/4.
Determine the value of for equilibrium and investigate
the stability of the equilibrium.
L = 0.4 m
k = 800 N/m
m = 40 kg
A

Mass m
(L/2) cos
L/2
k

L
Mass center
B
B

Let point A be the datum for the gravitational


potential energy Vg. Thus
Vg = mg(L/2) cos

2309

(1)

11.2 Potential Energy Example 7, page 2 of 3


2 The elastic potential energy is
Ve = (1/2)k(L cos

L sin
2

L/4) + (1/2)k(L/4

L sin )

A
2

= (1/2)kL [(cos

1/4) + (1/4

sin ) ]

(2)

The potential energy of the system is


(1/2)mgL cos

L cos

by Eq. 1

V = Ve + Vg
(1/2)kL2[(cos

L
1/4)2 + (1/4

sin )2] by Eq. 2

The equilibrium position can be found from

or,

dV
=0
d
dV
d
= d {(1/2)kL2[(cos
d

B
1/4)2 + (1/4

sin )2]

(1/2)mgL cos

=0

or, after differentiating and simplifying,


dV
= (L/4)[(kL + 2mg) sin
d

kL cos ] = 0

(3)

Substituting k = 800 N/m, L = 0.4 m, g = 9.81 m/s2, and m = 40 kg and solving gives
= 16.15

Ans.

2310

11.2 Potential Energy Example 7, page 3 of 3


3

To investigate stability, evaluate dV2/d 2. Using the


expression for dV/d in Eq. 3 gives
d2V d
=
d
d
=

dV
d

d
(L/4)[(kL + 2mg) sin
d

= (L/4)[(kL + 2mg) cos

kL cos ]

+ kL sin ]

Because all quantities on the right-hand side are positive


when = 16.15, d2V/d 2 is positive, and the equilibrium is
stable.

2311

11.2 Potential Energy Example 8, page 1 of 4


8. Light rods AB and BC support a block of weight W at
C. End D of the spring is attached to a light collar that
moves freely on the vertical rod. Assume that the spring is
always horizontal. The spring is unstretched when = 0.
Determine the angle for equilibrium and also determine
if the equilibrium is stable.
L = 9 in.
k = 100 lb/in.
W = 200 lb

C
L

D
k

B
L

2312

11.2 Potential Energy Example 8, page 2 of 4


Weight, W
Let point A be the datum for the gravitational
potential energy Vg. Thus

3
L

Angle equals
because triangle
ABC is isosceles.

Vg = W(2L cos )

(2)

D
1

L cos

L cos
L

The potential energy of the system is


2WL cos

L sin

V = Ve + Vg

(3)

(1/2)k(L sin )2, by Eq. 1

2 The spring is unstretched when = 0.


Thus the extension of the spring is L sin
and the elastic potential energy is
Ve = (1/2)k(L sin )2

by Eq. 2

The equilibrium position can be found from


dV
=0
d

(1)
or,

dV d
=
[(1/2)k(L sin )2 + 2WL cos ] = 0
d
d
or,
dV
= (kL2 cos
d

2313

2WL) sin

=0

(4)

11.2 Potential Energy Example 8, page 3 of 4


5

The equilibrium equation, Eq. 4, has two solutions. One


solution, 1, can be found by requiring sin = 0, which
implies
1=

d2V = d
d
d

Ans.

The other solution, , follows from requiring the term in


parentheses in Eq. 4 to vanish.
(kL2 cos

2WL) sin

=0

(Eq. 4 repeated)

Thus
(kL2 cos

2WL) = 0

dV
d

= d [(kL2 cos
d

2WL) sin ]

= ( kL2 sin ) sin


2WL) cos

+ (kL2 cos
(6)

7 The stability of the equilibrium position 1 = 0 can


be determined by substituting 1 = 0 (and L = 9 in.,
k = 100 lb/in., and W = 200 lb) into Eq. 6. The
result is

Substituting L = 9 in., k = 100 lb/in., and W = 200 lb and


solving gives
= 63.62

To investigate stability, evaluate dV2/d 2. Using the


expression for dV/d in Eq. 4 gives

Ans.

d2V
= 4500 lb in
d
Because d2V/d
stable.

2314

is positive, the position

1=

0 is

11.2 Potential Energy Example 8, page 4 of 4


2
8 Similarly evaluating d V/d

at

= 63.62 gives

d2V
= 6501 lb in.
d
Because d2V/d
unstable.

is negative, the position

Stable equilibrium

= 63.62 is

Unstable equilibrium

5000

A plot of V = V( ) shows the equilibrium points. From Eq. 3,


we have
V( ) = (1/2)kL2 sin2 + 2WL cos

(Eq. 3 repeated)

= (1/2)(100 lb/in.)(9 in.)2 sin2


+ 2(200 lb)(9 in.) cos
2

= 4050 sin

V( )
(in. lb)

4050

3600

0
0

+ 3600 cos

2315

63.62

90

11.2 Potential Energy Example 9, page 1 of 5


9. Determine the spring constant k required to keep the
uniform rod AB in equilibrium at an angle of = 20. Also
determine the stability of the equilibrium. The spring is
unstretched when = 0. Neglect the size of the pulley at C.
L = 30 in.
W = 200 lb

k
C
1
L
Weight, W

Let point A be the datum for the


gravitational potential energy, Vg. Thus
Vg = (WL/2) sin

(1)

Weight, W
L

(L/2) sin
L/2

Center of gravity

2316

11.2 Potential Energy Example 9, page 2 of 5


2

Spring stretched

To determine the elastic potential energy, we have to determine the


change in length of the spring. But the change in length of the spring is
just the change in distance between points B and C. First let's calculate
the length of BC, say, BCstretched , when rod AB is rotated by an angle .
By the law of cosines,

k
C

BCstretched =

L2 + L2

= L 2[1

2L2 cos(90 + )

cos(90 + )]

(2)

L
The formula for the cosine of a sum gives
90
cos (90 + ) = cos 90 cos

sin 90 sin

A
=
L

sin

B
Thus Eq. 2 becomes
BCstretched = L 2(1 + sin )

2317

(3)

11.2 Potential Energy Example 9, page 3 of 5


3

Next calculate the length BC when the spring is unstretched.


Spring unstretched
k
C
BCunstretched =

L2 + L2

L
A

90
B
L

Now the change in length of the spring can be computed:


BCstretched

BCunstretched = L 2(1 + sin )


= L 2 ( 1 + sin

L 2
1)

(4)

The elastic potential energy is then


Ve= (1/2)k[L 2 ( 1 + sin
= kL2 ( 1 + sin

1)]2

1)2

(5)

2318

11.2 Potential Energy Example 9, page 4 of 5


5 The potential energy of the system is, then,
W(L/2) sin

by Eq. 1

V = Ve + Vg
kL2( 1 + sin

1)2 by Eq. 5

The equilibrium position can be found from


dV = 0
d
or,
dV
= d [kL2( 1 + sin
d
d

1)2

W(L/2) sin ] = 0

or, after differentiating and simplifying,


dV
= (L cos )(kL
d
Substituting

W
2

kL

)=0

(6)

1 + sin

= 20, W = 200 lb, and L = 30 in. and solving for k gives

k = 24.37 lb/in.

Ans.

2319

11.2 Potential Energy Example 9, page 5 of 5


6

To investigate stability, evaluate dV2/d 2. Using the expression for dV/d


in Eq. 6 gives
d2V d
=
d
d
=

dV
d

d
[(L cos
d

kL W
2

kL

)]

1 + sin

Differentiating gives
d2V
= ( L sin )(kL
d

W
2

Substituting k = 24.37 lb/in. (and


gives

kL

) + (L cos )[

1 + sin

kL cos
2(1 + sin )3/2

= 20, L = 30. in., and W = 200 lb)

d2V
= 6229 in. lb
d
Because d2V/d

is positive, the equilibrium position

= 20 is stable.

2320

11.2 Potential Energy Example 10, page 1 of 4


10. The waste bin has a center of gravity at C and weight W.
Determine the steepest slope upon which the bin can be placed
without tipping. Also determine the stability of this
equilibrium position.
a = 25 in.
b = 10 in.
W = 60 lb

THA
YO NK
U

C
a

A
B

b
b

2321

11.2 Potential Energy Example 10, page 2 of 4


1

Because we are looking for the steepest


slope upon which the bin can be placed
without tipping, we know that the force F
from the sloping surface acting on the
bottom of the bin is concentrated at the
corner B the bin is just about to tip.

2 The potential energy V of the system is entirely


gravitational, since there is no elastic potential energy.
We want to write an expression for the gravitational
potential energy of the forces shown, as a function of .
Because point B does not move as changes, B can be
used as the datum. Thus

C
C

a
a

A
b
B

A
b

2322

(vertical distance between B and C)

W(a cos
a cos

b sin

+ b sin )

(1)

11.2 Potential Energy Example 10, page 3 of 4


3

4 To investigate stability of the equilibrium, evaluate d2V/d 2


for = 21.8. Using the expression for dV/d in Eq. 2 gives

The equilibrium position can be found from


dV
d =0

d2V d
=
d
d

or,
dV d
d = d [W(b sin

=
+ a cos ] = 0

or,

dV
d

d
[W(b cos
d

= W( b sin
dV
d = W(b cos

a sin ) = 0

a cos )]

Because both terms are negative when = 21.8, d2V/d


negative, and the equilibrium position is unstable.

(2)

Substituting b = 10 in., and a = 25 in. into this


equation and solving gives
= 21.8

a sin )

is

Comment No. 1: We have implicitly assumed that the corner


of the bin at B can develop enough friction to prevent the bin
from sliding down the slope. If sliding rather than tipping
does occur, then the friction force does work, the
potential-energy function does not exist, and the
potential-energy criterion cannot be used to find the
equilibrium state.

Ans.

2323

11.2 Potential Energy Example 10, page 4 of 4


Comment No. 2: Consider a plot of V/W = b sin

+ a cos

= 10 sin

+ 25 cos

Equilibrium state (unstable)


25

V( )
W
(in.)
21.8
0

6
0 < < 21.8
The bin is not tipped far
enough to be balanced. The
bin is not in equilibrium.

100

= 21.8
The bin is tipped to a position where the line of action
of the weight passes through the bottom right corner of
the bin. The bin is in (unstable) equilibrium.

90
7 21.8
The bin is tipped too far to
be balanced. The bin is not
in equilibrium.
C

C
= 21.8

W
W

2324

11.2 Potential Energy Example 11, page 1 of 4


11. The outer rim of stepped pulley A is connected by a
belt to pulley B. A spring is wrapped around the inner
rim and the cord supporting mass m is attached at point
C on the large pulley. Determine the equilibrium
position of the system and investigate its stability. The
spring is unstretched when the pulleys are in the position
shown.
R = 0.2 m
m = 2 kg
k = 900 N/m
k

A
2R

D
m

2325

4R

11.2 Potential Energy Example 11, page 2 of 4


1

Let point B be the datum for gravitational potential


energy, Vg. Thus
k
Vg = mg(4R sin

+ L)

(1)

The length L is unknown but will drop out of the


equations later when the expression for Vg is
differentiated.

B
4R sin
A

4R

L
D
m

2 The elastic potential energy is


Ve = (1/2)k(extension of spring)2

(2)

Arc length = R
( measured in radians)

k
B

R
3 The potential energy of the system is
mg(4R sin
V = Ve + Vg
(1/2)kR2

+ L) by Eq. 1
(3)

by Eq. 2

D
m

2326

4R

11.2 Potential Energy Example 11, page 3 of 4


4

Angles and are not independent but can be related by noting that the
belt is inextensible, so that the arc lengths must be equal:
4R = 2R
Thus
2R

= /2

4R
B

Using this result in Eq. 3 gives


V = (1/2)kR2

A
mg(4R sin

+ L)

(Eq. 3 repeated)

/2
The equilibrium position can be found from
dV
=0
d
or,
dV d
=
[(1/2)kR2
d
d

mg(4R sin ( /2) + L)] = 0

or,
dV
= kR2
d

mgR cos ( /2) = 0

(4)

(We have used the fact that dL/d = 0 because L does not vary with

2327

11.2 Potential Energy Example 11, page 4 of 4


5

Substituting R = 0.2 m, m = 2 kg, k = 900 N/m and g = 9.81 m/s2 into Eq. 4
and solving numerically gives
= 0.2167 rad = 12.416

Ans.

To investigate stability, evaluate dV2/d 2. Using the expression for dV/d


in Eq. 4 gives
d2V d
=
d
d

dV
d

or,
d2V d
=
(kR2
d
d

2mgR cos ( /2)

or,
d2V
= kR2 + mgR sin ( /2)
d
Because both terms in this equation are positive for = 12.4, d2V/d
positive, and the equilibrium position = 12.4 is stable.

2328

is

11.2 Potential Energy Example 12, page 1 of 4


12. Determine if rods AB and BC are in stable equilibrium in the
position shown. The rods are uniform and have mass m. Rod BD is
pinned to a light slider at C that can move freely up and down between
the vertical walls. The spring is unstretched when rod AB is vertical.
One end of the spring is attached to a light collar at E that moves freely
on the rod. Assume that the spring remains horizontal for all positions
of the rods.
D
L/2
C
L/2

k
B

L
A

2329

11.2 Potential Energy Example 12, page 2 of 4


1

Let point A be the datum for gravitational potential energy, Vg. Thus
Vg = mg(L/2) cos

+ mg[L cos

+ (L/2) cos ]
D
C
(L/2) cos

The elastic potential energy is


(L/2) cos

Ve = (1/2)k(extension of spring)2
= (1/2)k(L sin )2

(2)

B
Center of
mass

(L/2) cos
A
L sin

3 The potential energy of the system is


mg(L/2) cos

+ mg[L cos

V = Vg + Ve

+ (L/2) cos ] by Eq. 1


(3)

(1/2)k(L sin )2 by Eq. 2

2330

11.2 Potential Energy Example 12, page 3 of 4


4 The angles

and

L sin

are not independent but can be related by noting that

= (L/2) sin

(L/2) sin
4
D

If we now consider only the case where rods AB and BD are nearly vertical, then
both and are small and sin
and sin
Substituting these
approximations into Eq. 4 gives, after cancelling L and multiplying by 2,

=2

k
E

Using this result in Eq. 3 gives


2
2

V = (1/2)kL sin

+ (1/2)mgL(3 cos

+ cos

(Eq. 3 repeated)

The equilibrium position can be found from


A
dV
d =0

L sin

or,
dV d
=
[(1/2)kL2 sin2
d
d

+ (1/2)mgL(3 cos

+ cos 2 ] = 0

or,
dV
= kL2 sin
d
Note that

cos

(1/2)mgL(3 sin

+ 2 sin 2

=0

= 0 (both rods are vertical) satisfies the equilibrium equation, Eq. 5.

2331

(5)

11.2 Potential Energy Example 12, page 4 of 4


5

To investigate stability of the equilibrium, evaluate d2V/d


expression for dV/d in Eq. 5 gives
d2V d
=
d
d
=

d
[ kL2 sin
d

= 0. Using the

cos
sin2 )

(1/2)mgL(3 sin

+ 2 sin 2

(1/2)mgL(3 cos + 4 cos 2

= 0 gives, after simplifying,

d2V
= (L/2)(2kL
d

7mg)

The equilibrium will be stable if d2V/d


2kL

for

dV
d

= kL2(cos2
Substituting

is positive, that is, if

7mg > 0

or,
kL 7
mg > 2

(6)

Comment: The result expressed in Eq. 6 makes intuitive sense: The rods will be
stable in a vertical position if the spring is stiff (large k in the numerator of Eq.6)
and the mass is small (small m in the denominator of Eq. 6).

2332

11.2 Potential Energy Example 13, page 1 of 5


13. Determine the range of values of the force P for which the
vertical equilibrium position = 0 = 2 is stable. The springs
are connected to collars D and E, which can slide freely on the
vertical rod. The springs are unstretched when = 0 = 2.
Assume that the springs remain horizontal for all positions of
the rods and the weights of the rods and the frictionless collars
are negligible.
P
k

D
L
k

E
1

L
A

2333

11.2 Potential Energy Example 13, page 2 of 5


1

Let point A be the datum for the potential


energy of the force P, VP. Thus
VP = P(L cos

+ L cos

2)

2 The elastic potential energy is


Ve = (1/2)k(L sin 1)2
+ (1/2)k(L sin

(1)

1+

2
2)

L sin

(2)

P
L sin

L sin

C
k
L

L cos

1+

L cos

A
A
3 The potential energy of the system is
P(L cos

1+

L cos

2)

by Eq. 1

V = Vg + VP
(1/2)k(L sin 1)2 + (1/2)k(L sin

(3)
1+

L sin

2
2)

2334

by Eq. 2

11.2 Potential Energy Example 13, page 3 of 5


4 The equilibrium equations are
V

= 0

or,
V

[(1/2)k(L sin 1)2 + (1/2)k(L sin

1+

L sin

2
2)

+ P(L cos

1+

L cos

2)]

=0

or,
V = kL2[sin

1 cos 1 +

(sin

1+

sin

2)(

cos 1)]

PL sin

1=

(4)

and,
V

= 0

or,
V

[(1/2)k(L sin 1)2 + (1/2)k(L sin

1+

L sin

2
2)

+ P(L cos

1+

L cos

2)]

=0

or,
V

= kL2(sin

1+

sin

2)

cos

PL sin

Note that the vertical position of the rods,

1=

=0

0=

2,

(5)

is a solution of Eqs. 4 and 5.

2335

11.2 Potential Energy Example 13, page 4 of 5


5 The equilibrium configuration
stable provided that
2

1=

0=

will be

V <0
2

Differentiating the expression for V/ 2 (Eq. 5) with respect


to 2 and substituting 1 = 0 = 2 in the result gives

(6)

and

= L(kL

P)

(10)

The stability condition Eq. 6 becomes


2

> 0 or

V
2

>0

kL2 by Eq. 9

(7)
2

Differentiating the expression for V/ 1 (Eq. 4)


with respect to 1 and substituting 1 = 0 = 2 in
the result gives

= L(2kL

P)

L(2kL

Differentiating the expression for V/ 1 (Eq. 4)


with respect to 2 and substituting 1 = 0 = 2 in
the result gives
V

= kL2

V
2

P) by Eq. 8

<0

L(kL

(Eq. 6 repeated)

P) by Eq. 10

That is,

(8)

(kL2)2

L(2kL

P)L(kL

P) < 0

or, after dividing by L2 and re-arranging terms,


P2

3kLP + k2L2 > 0

Using the quadratic formula to factor this expression gives

(9)

(P

2336

3+ 5
2 kL)(P

5
2

kL) > 0

(11)

11.2 Potential Energy Example 13, page 5 of 5


7 The inequality given in Eq. 11 will be satisfied if both factors are positive:
3 + 5 kL) > 0, which implies P > 3 + 5 kL
2
2
5 kL
3
5 kL) > 0, which implies P > 3
2
2

(P
(P

Alternatively, the inequality given in Eq. 11 will be satisfied if both factors are negative:
3 + 5 kL) < 0, which implies P < 3 + 5 kL
2
2
5 kL
3
5 kL) < 0, which implies P < 3
2
2

(P
(P

(12)
(13)

The inequalities in Eq. 7 become


L(2kL

P), by Eq. 8

P), by Eq. 10

>0

or

P < 2kL

or

L(kL
V
2

>0

(Eq. 7 repeated)

That is,
P < kL

(14)

Noting that (3
5 )/2 = 0.382 < 1, we see that we can satisfy the inequalities of Eq. 14 and
Eqs. 12 and 13 simultaneously, and thus have a stable equilibrium, provided that we restrict
P/(kL) to the range
P
5
<3
2
kL

Ans.

2337

Appendix: Geometric Properties of Lines, Areas, and Solid Shapes

2338

Geometric Properties of Lines, Areas, and Solid Shapes page 1 of 4


y

Centroid Location

Centroid Location

y
r

Length =

Length = 2 r
C

2r
x

r sin
Circular arc segment

Quarter circular segment

Centroid location

Centroid location

Area =

bh
2

r2
Area =
4

2h
3
h
3

C
b

Triangular region

4r
3

Quarter circular region

2339

Geometric Properties of Lines, Areas, and Solid Shapes page 2 of 4


Centroid Location

Centroid Location
y
b

y
2

Area = r

r
C

3a
5

4ab
Area = 3
2r sin
Parabolic region

Circular sector region

Centroid Location

Centroid Location
y
b

1
Volume = 3 r3(2

3 sin

+ 3 sin3

z
2a/5
C

2ab
Area = 3
Vertex

x
Spherical cap

Semiparabolic region

2340

3r(1+ sin )2
4(2+ sin )

Geometric Properties of Lines, Areas, and Solid Shapes page 3 of 4


Centroid Location

Centroid Location

2
Volume = 3 r3

rh
Volume =
3

h
4

Hemisphere

Cone
3
I x = bh
12

Moment of Inertia

Moment of Inertia
y

Area =

Iy = hb
12

bh
2

C
B

x
b

b
2

b
2

h
2

2h
3
h
3

3r
8

3
I xx = bh
36
3
IBB = hb
12

Area = bh
h
2

Triangular

Rectangle

2341

Geometric Properties of Lines, Areas, and Solid Shapes page 4 of 4


Moment of Inertia

Ix = 1 r4
4
Iy = 1 r4
4

Ix =

r4
8

Moment of Inertia

r4
8
4r
yc =
3

Iy =

r
C
2

Area = r

yc
2

r
Area = 2

x
Semicircle

Circle

2342

by composite parts, 1748 1844


theorems of Pappus and Guldinus, 1846 1923
volumes, 1673, 1687 1690, 1734 1748, 1767 1770,
1810 1840
wires, 1673, 1683 1685, 1722 1730, 1755 1757,
1777 1785

Index
A
Adding forces
parallelogram law, 24 59
rectangular components in plane, 61, 70 75, 97 112
rectangular components in space, 114, 119 122, 133 145,
148, 156 158, 182 199
Arch, Gateway, 1686, 1731 1733

Clamp
internal forces, 1490, 1515 1516
square-threaded screw, 1292, 1310 1313
wedges, 1253, 1263 1265
Clothesline, 1088, 1103 1104
Components
from parallelogram law, 32 34
rectangular components in plane, 61 112
rectangular components in space, 114 145,
148, 152 158, 167 199
Constraints, 607 614, 626 670
Couple, 402 443
Cross product
multiplication of base vectors, 354, 358, 367, 387, 394, 399
determinant, 363, 373, 376, 383, 388

B
Beams
distributed loads, 557 604
shear and bending-moment diagrams, 1536 1596,
1598 1669
Bridge, suspension, 1086, 1095 1098
Buoy, center of mass of, 1769, 1823 1831

C
D

Cables: concentrated loads, 1034 1081


Cables: uniform loads, 1083 1132
formula, 1085
Catenary, 1134 1190
formulas, 1134
Centroids
areas, 1672, 1674 1682, 1691 1721, 1750, 1758 1765,
1786 1811
concentrated masses, 1751 1754, 1772 1776
by integration, 1672 1748

Direction cosines, angles, 114, 119 121, 129 145, 163 171
Distributed loads on beams, 557 603
Dot products, use
angles, 201 206, 209 228
force projections, 201, 207 208, 229 238

2343

flat belts, 1348 1379


journal bearings, 1403 1449
rolling resistance, 1451 1480
square-threaded screws, 1287 1346
thrust bearings and disks, 1381 1401
wedges, 1251 1285

E
Equilibrium
particle in plane, 241 260
particle in space, 262 305
rigid bodies in plane, 689 730
rigid bodies in space, 732 789
Equivalent force-couple systems, 473 555

Geometric properties
centroids, 2339 2341
moments of inertia, 2341 2342

Flat belts, friction, 1348 1379


formula, 1348
Formulas
catenary cables, 1134
cables with uniform loads, 1087
flat belts, 1348
law of cosines, 24
law of sines, 24
magnitude of two-dimensional force vector, 61
magnitude of three-dimensional force vector, 114
Mohrs circle, 2117
moments of inertia about inclined axes, 2116
principal moments, 2116
square-threaded screws, 1287 1288
stability conditions for a two-degree-of-freedom
system, 2272
theorems of Pappus and Guldinus, 1846
thrust bearings and disks, 1381
Frames and machines 791 902
Friction
basic applications, 1193 1249

H
Hydrostatic pressure, 1925 1980

I
Ice cream cone, density of, 1768, 1817 1822
Improperly constrained, 607
Inclined axes, moments of inertia, 2116, 2118 2120, 2128 2139
formulas, 2116
Internal forces in structural members, 1482 1535

J
Jack, automobile,
square-threaded screw, 1294, 1318 1323
virtual work, 2174, 2209 2213
Journal bearings, 1403 1449
2344

N
L
Nutcracker, 796, 823 825
Law of cosines, 24
Law of sines, 24

Parallelogram law, 24 59
Partially constrained, 607
Piano moving, 1453, 1462 1465
Position vectors
defining, 147 199
use in finding force, 148, 152 158, 167 199
Potential energy, 2271 2337
two degrees of freedom, 2285, 2332 2346
Principal moments of inertia, 2116, 2120 2127, 2136 2164
formulas, 2116
Pulleys
belt friction, 1354, 1356, 1371 1374, 1377 1379
free-body analysis, 798 802, 826 839
virtual work, 2184, 2256 2259
Products of inertia, 2068 2114
Properly constrained, 607

Magnitude of a force vector,


two-dimensional, 61
three-dimensional, 114
Mohrs circle, 2116, 2124 2127, 2149 2164
formulas, 2117
Moment diagram
equation form, 1540 1599
constructed by areas, 1601 1672
Moments
two-dimensional, 308 339
three-dimensional, 341 400
couple, 402 443
force about a line, 445 471
Moments of inertia of areas
about inclined axes, 2115, 2117 2120, 2128 2139
by composite parts, 2028 2066
by integration, 1983 2026
Mohrs circle, 2116, 2124 2127, 2149 2164
principal, 2116, 2120 2127, 2136 2164

R
Rolling resistance, 1451 1480
Rounding off, 2, 4, 7 11, 16 19

2345

Scalar triple product, 445, 452 471


Self-locking
square-threaded screws, 1288
wedges, 1258, 1280 1285
Shear diagram
equation form, 1540 1599
constructed by areas, 1601 1672
Square-threaded screws, 1287 1346
formulas, 1287 1288
Stability, 2271 2337
conditions for a two-degree-of-freedom system, 2272
Static determinacy, 607, 615 617, 671 687
Stonehenge monument, 1457, 1476 1480

Units, 3 6, 12 21
Unit vectors, 148, 151, 163 166, 170, 174 175, 178, 181, 186,
191, 197 198

V
Virtual work, 2167 2269
active forces, 2167
used to calculate reactions, 2181 2183, 2243 2256
two degrees of freedom, 2176, 2185, 2219 2223,
2261 2279

W
Wedges, friction, 1251 1285
Wrench (equivalent force-couple system), 489 490, 536 555

T
Theorems of Pappus and Guldinus, 1846 1923
formulas, 1846
Thrust bearings and disks, friction, 1381 1401
formula, 1382
Tripod cooking-stand, 156, 182 186
Trusses
method of joints, 904 959, 1002 1005, 1006 1029
zero-force members, 905, 911 912, 944 959
method of sections, 961 999, 1006, 1030 1034
space, 1002 1034
Turnbuckles, 1295 1296, 1324 1339

Z
Zero-force truss members, 905, 911 912, 944 959

2346

Potrebbero piacerti anche